You are on page 1of 506

I.

PROVISIONAL REMEDIES (Rules 57-61)

A. PRELIMINARY ATTACHMENT (RULE 57) CASE DIGESTS

A.1. ALFREDO C. LIM, JR. VS. SPOUSES TITO S. LAZARO AND CARMEN T. LAZARO

G.R. No. 185734; July 3, 2013; Perlas-Bernabe, J.

FACTS: Petitioner filed a complaintfor sum of money with prayer for the issuance of a writ of
preliminary attachment before the RTC. The RTC granted the writ of preliminary attachment application
and upon the posting of the required bond, issued the corresponding writ. In this accord, three (3) parcels
of land situated in Bulacan, were levied upon.

Nonetheless, the parties entered into a Compromise Agreement whereby Sps. Lazaro agreed to pay Lim,
Jr. on an installment basis. The aforesaid compromise agreement was approved by the RTC.
Subsequently, Sps. Lazaro filed an Omnibus Motion, seeking to lift the writ of preliminary attachment,
which the RTC granted.

Lim, Jr. filed a motion for reconsideration which was, however, denied, prompting him to file a petition
for certiorari before the CA. The CA rendered the assailed decision, finding no grave abuse of discretion
on the RTC's part. The said Court stated that since the principal cause of action had already been declared
closed and terminated by the RTC, the provisional or ancillary remedy of preliminary attachment would
have no leg to stand on, necessitating its discharge.

Aggrieved, Lim, Jr. moved for reconsideration which was likewise denied by the CA.

ISSUE: The sole issue in this case is whether or not the writ of preliminary attachment was properly
lifted.

RULING: The petition is meritorious.

Records indicate that while the parties have entered into a compromise agreement which had already been
approved by the RTC, the obligations thereunder have yet to be fully complied with particularly, the
payment of the total compromise amount. Hence, given that the foregoing debt remains unpaid, the
attachment of Sps. Lazaro's properties should have continued to subsist.

In Chemphil Export & Import Corporation v. CA, the Court pronounced that a writ of attachment is not
extinguished by the execution of a compromise agreement between the parties. The parties to the
compromise agreement should not be deprived of the protection provided by an attachment lien especially
in an instance where one reneges on his obligations under the agreement.If we were to rule otherwise, we
would in effect create a back door by which a debtor can easily escape his creditors. Consequently, we
would be faced with an anomalous situation where a debtor, in order to buy time to dispose of his
properties, would enter into a compromise agreement he has no intention of honoring in the first place.
The purpose of the provisional remedy of attachment would thus be lost. It would become, in analogy, a
declawed and toothless tiger.

In fine, the Court holds that the writ of preliminary attachment subject of this case should be restored and
its annotation revived in the subject TCTs, re-vesting unto Lim, Jr. his preferential lien over the properties
covered by the same as it were before the cancellation of the said writ. Lest it be misunderstood, the lien
or security obtained by an attachment even before judgment, is in the nature of a vested interest which
affords specific security for the satisfaction of the debt put in suit. Verily, the lifting of the attachment lien
would be tantamount to an abdication of Lim, Jr.'s rights over Sps. Lazaro's properties which the Court,
absent any justifiable ground therefor, cannot allow.

WHEREFORE, the petition is GRANTED. The Decision and the Resolution of the Court of Appeals are
REVERSED and SET ASIDE, and the Order of the Regional Trial Court of Quezon Cityis NULLIFIED.
Accordingly, the trial court is directed to RESTORE the attachment lien in favor of petitioner Alfredo C.
Lim, Jr.

DOCTRINE: By its nature, preliminary attachment, under Rule 57 of the Rules of Court (Rule 57), is an
ancillary remedy applied for not for its own sake but to enable the attaching party to realize upon the
relief sought and expected to be granted in the main or principal action; it is a measure auxiliary or

Remedial Law Review II (CY 2018-2019) – Saturday 1:00pm – 5:00pm: Case Digest Compilation [Atty. Brondial] Page 1
incidental to the main action. As such, it is available during its pendency which may be resorted to by a
litigant to preserve and protect certain rights and interests during the interim, awaiting the ultimate
effects of a final judgment in the case. In addition, attachment is also availed of in order to acquire
jurisdiction over the action by actual or constructive seizure of the property in those instances where
personal or substituted service of summons on the defendant cannot be effected.

In this relation, while the provisions of Rule 57 are silent on the length of time within which an
attachment lien shall continue to subsist after the rendition of a final judgment, jurisprudence dictates
that the said lien continues until the debt is paid, or the sale is had under execution issued on the
judgment or until the judgment is satisfied, or the attachment discharged or vacated in the same manner
provided by law.

Remedial Law Review II (CY 2018-2019) – Saturday 1:00pm – 5:00pm: Case Digest Compilation [Atty. Brondial] Page 2
A.2. LIGON VS. RTC OF MAKATI

G.R. No. 190028, February 26, 2014

FACTS: Petitioner Ligon alleged that Rosario Baladjay enticed her to extend a short-term loan payable in
a month’s time and secured by an Allied Bank post-dated check for the same amount. Ligon likewise
claimed that Rosario, as further enticement for the loan extension, represented that she and her husband
Saturnino were in the process of selling their property in Ayala Alabang Village and that the proceeds of
the said sale could easily pay-off the loan. Unfortunately, the Allied Bank check was dishonored and
despite assurances Rosario failed to replace the checks. Ligon discovered that the subject property was
already transferred to Polished Arrow, the alleged dummy corporation of Sps. Baladjay and other
defendants. As a result, TCT No. 8502 was cancelled and replaced on October 11, 2002 by TCT No.
9273[11] in the name of Polished Arrow. Thus, Ligon prayed that all defendants be held solidarily liable
to pay her the amount of P3, 000,000.00, with interest plus attorney’s fees and for moral and exemplary
damages.

Asserting that the transfer of the subject property to Polished Arrow was made in fraud of Sps. Baladjay’s
creditors, Ligon also prayed that the said transfer be nullified, and that a writ of preliminary attachment be
issued in the interim against defendants’ assets, including the subject property. Subsequently, an
Amended Writ of Preliminary Attachment was issued and annotated on the title.

A similar complaint for collection of sum of money, damages, and cancellation of title with prayer for
issuance of a writ of preliminary attachment was lodged before the RTC Makati by Spouses Vicente
against Sps. Baladjay, Polished Arrow. It was established that Sps. Baladjay solicited millions of pesos in
investments from Sps. Vicente using conduit companies that were controlled by Rosario. During the
proceedings therein, a writ of preliminary attachment also against the subject property was issued and
annotated on the title.

Thereafter, but before the Quezon City Case was concluded, the Makati City RTC rendered a Decision
rescinding the transfer of the subject property from Sps. Baladjay to Polished Arrow upon a finding that
the same was made in fraud of creditors. Consequently, the Makati City RTC directed the Register of
Deeds of Muntinlupa City to cancel the title that was under the dame of Polished Arrow and to restore the
title to its previous condition". Meanwhile, after due proceedings, the Quezon City RTC rendered a
Decision directing Sps. Baladjay to pay Ligon with interest, as well as attorney’s fees and costs of suit,
the decision later on became final and executory.

However, when Ligon sought its execution, she discovered that the attachment annotation had been
deleted from TCT No. 9273 when the subject property was sold by way of public auction to the highest
bidder, respondent Ting, for the amount of P9,000,000.00 during the execution proceedings in the Makati
City Case, as evidenced by the Officer’s Final Deed of Sale issued by Sheriff Alejo.

Ligon learned that the Makati City RTC had issued its first assailed Order dated February 9, 2007,
directing Atty. Garing, as the Register of Deeds of Muntinlupa City, to: (a) register the Officer’s Final
Deed of Sale on the official Record Book of the Register of Deeds of Muntinlupa City; and (b) cancel
TCT No. 8502 in the name of Sps. Baladjay and issue a new title in the name of Ting, free from any liens
and encumbrances.

Atty. Garing manifested before the Makati City RTC that it submitted the matter en consulta to the Land
Registration Authority as he was uncertain whether the annotations on TCT No. 9273 should be carried
over to TCT No. 8502. In response to the manifestation, theMakati City RTC issued its second assailed
Order dated March 20, 2007, directing Atty. Garing to comply with the First Assailed Order under pain of
contempt. It explained that it could not allow the LRA to carry over all annotations previously annotated
on TCT No. 9273 in the name of Polished Arrow as said course of action would run counter to its
December 9, 2004 Decision which specifically ordered the cancellation of said TCT and the restoration of
TCT No. 8502 in its previous condition. Based on the foregoing, it pronounced that it was Atty. Garing’s
ministerial duty “to promptly cancel TCT No. 8502/T-44 in the name of defendant-spouses Baladjay and
to issue a new Transfer Certificate of Title in the name of the highest bidder, Leonardo J. Ting.”

In its third assailed Order, the Makati City RTC directed Ting to pay the aforesaid taxes and ordered Atty.
Garing to immediately cancel TCT No. 8502 and issue a new title in the former’s name.

Remedial Law Review II (CY 2018-2019) – Saturday 1:00pm – 5:00pm: Case Digest Compilation [Atty. Brondial] Page 3
On June 7, 2007, Atty. Garing issued TCT No. 19756 in the name of Ting, free from any liens and
encumbrances. Later, Ting sold the subject property to respondent Benito G. Techico, resulting in the
cancellation of TCT No. 19756 and the issuance of TCT No. 31001 in Techico’s name.

Ligon filed a certiorari petition against respondent Presiding Judge Reynaldo Laigo, Sheriff Alejo, Atty.
Garing, Ting, and Techico alleging, among others, that the Makati City RTC committed grave abuse of
discretion in issuing the Assailed Orders. In this relation, she prayed that the said orders be declared null
and void for having been issued in violation of her right to due process, and resulting in (a) the deletion of
the December 3, 2002 attachment annotation on TCT No. 9273 which evidences her prior attachment lien
over the subject property, and (b) the issuance of new titles in the names of Ting and Techico.

Consolidated with Ligon’s certiorari petition is a complaint for indirect contempt whereby it was alleged
that the latter unlawfully interfered with the court processes, particularly by deleting the December 3,
2002 attachment annotation on TCT No. 9273 which thereby prevented the execution of the Quezon City
RTC’s March 26, 2008 Decision.

The CA dismissed Ligon’s certiorari petition, finding that the Makati City RTC did not gravely abuse its
discretion in issuing the Assailed Orders, adding further that the same was tantamount to a collateral
attack against the titles of both Ting and Techico. Likewise, the indirect contempt charge was dismissed
for lack of sufficient basis. Aggrieved, Ligon filed the present petition.

ISSUE: Whether or not the CA erred in ruling that the Makati City RTC did not gravely abuse its
discretion in issuing the Assailed Orders

RULING: CA erred in holding that the RTC did not gravely abuse its discretion in issuing the Assailed
Orders as these issuances essentially disregarded Ligon’s prior attachment lien over the subject property.
In this case, Ligon, in order to secure the satisfaction of a favorable judgment in the Quezon City Case,
applied for and was eventually able to secure a writ of preliminary attachment over the subject property
which was later annotated on the TCT No. 9273 in the name of Polished Arrow. Notwithstanding the
subsequent cancellation of TCT No. 9273 due to the Makati City RTC’s December 9, 2004 Decision
rescinding the transfer of the subject property from Sps. Baladjay to Polished Arrow upon a finding that
the same was made in fraud of creditors, Ligon’s attachment lien over the subject property continued to
subsist since the attachment she had earlier secured binds the property itself, and, hence, continues until
the judgment debt of Sps. Baladjay to Ligon as adjudged in the Quezon City Case is satisfied, or the
attachment discharged or vacated in some manner provided by law. The grave abuse of discretion of the
Makati City RTC lies with its directive to issue a new certificate of title in the name of Ting free from any
liens and encumbrances. This course of action clearly negates the efficacy of Ligon’s attachment lien and,
also, defies the legal characterization of attachment proceedings. Sps. Ligon’s attachment lien against
Sps. Baladjay as well as their successors-in-interest should have been preserved, and the annotation
thereof carried over to any subsequent certificate of title.

WHEREFORE, the petition is PARTLY GRANTED. The Decision dated October 30, 2009 of the Court
of Appeals in CA-G.R. SP No. 106175 is REVERSED and SET ASIDE. Accordingly, the Assailed
Orders subject of this case are hereby declared NULL and VOID only insofar as they relate to the
issuance of Transfer Certificate of Title No. 19756 in the name of respondent Leonardo J. Ting free from
any liens and encumbrances. The Register of Deeds of Muntinlupa City is DIRECTED to carry over and
annotate on TCT No. 31001 in the name of respondent Benito G. Techico the original attachment lien of
petitioner Leticia P. Ligon as described in this Decision. The indirect contempt charges are, however,
DISMISSED.

Remedial Law Review II (CY 2018-2019) – Saturday 1:00pm – 5:00pm: Case Digest Compilation [Atty. Brondial] Page 4
A.3. TORRES VS. SATSATIN

G.R. No. 166759; November 25, 2009; Peralta, J.

FACTS: NicanorSatsatin (Nicanor) asked petitioners’ mother, AgripinaAledia, if she wanted to sell their
lands. Since Agripina agreed, petitioners, thus, authorized Nicanor, through a Special Power of Attorney,
to negotiate for the sale of the properties. Nicanor offered to sell the properties to Solar Resources, Inc.
(Solar). Solar allegedly agreed to purchase the three parcels of land, together with the 10,000-square-
meter property owned by a certain RusticaAledia, for ₱35M. Petitioners alleged that Nicanor was
supposed to remit to them the total amount of ₱28M or ₱9.3M each to Sofia, Fructosa, and the heirs of
Mario (the children of Agripina) as he was already paid by Solar. However, despite the payment for the
subject property, Nicanor only remitted the total amount of ₱9M.

Consequently, on October 25, 2002, petitioners filed before the regional trial court (RTC) a Complaint for
sum of money and damages, against Nicanor, et all. On October 30, 2002, petitioners filed an Ex-Parte
Motion for the Issuance of a Writ of Attachment. On the same date, the trial court issued an Order
directing the petitioners to post a bond in the amount of ₱7M. Thereafter, the RTC issued a Writ of
Attachment dated November 15, 2002, directing the sheriff to attach the estate, real or personal, of the
respondents. On November 19, 2002, a copy of the writ of attachment was served upon the respondents.
On the same date, the sheriff levied the real and personal properties of the respondent, including
household appliances, cars, and a parcel of land located at Las Piñas, Manila. On November 21, 2002,
summons, together with a copy of the complaint, was served upon the respondents. On November 29,
2002, respondents filed their answer, they also filed a Motion to Discharge Writ ofAttachment. In the
alternative, respondents offered to post a counter-bond for the lifting of the writ of attachment. On March
11, 2003, after the parties filed their respective pleadings, the RTC issued an Order denying the motion of
the respondents, but at the same time, directing the respondents to file a counter-bond.

Aggrieved, respondents filed before the CA a Petition for Certiorari, Mandamus and Prohibition with
Preliminary Injunction and Temporary Restraining Order. On November 23, 2003, the CA rendered the
assailed Decision in favor of the respondents.

ISSUES:
(1)Whether or not the issuance of the writ of preliminary attachment was irregular based on the following
grounds:
(a)That the attaching bond was improper
(b)That the court did not acquire jurisdiction over the person of the defendant

(2)Whether or not respondents are barred by estoppel, laches, and prescription from questioning the
orders of the RTC issuing the writ of attachment and that the issue has become moot and academic by the
renewal of the attachment bond covering after its expiration.

RULING:
(1)Whether or not the issuance of the writ of preliminary attachment was irregular

Yes, the issuance of the preliminary attachment was not proper.

(a)That the attaching bond was improper


In the case at bar, the CA correctly found that there was grave abuse of discretion amounting to lack of or
in excess of jurisdiction on the part of the trial court in approving the bond posted by petitioners despite
the fact that not all the requisites for its approval were complied with. In accepting a surety bond, it is
necessary that all the requisites for its approval are met; otherwise, the bond should be rejected.

Every bond should be accompanied by a clearance from the Supreme Court showing that the company
concerned is qualified to transact business which is valid only for thirty (30) days from the date of its
issuance. However, it is apparent that the Certification issued by the Office of the Court Administrator
(OCA) at the time the bond was issued would clearly show that the bonds offered by Western Guaranty
Corporation may be accepted only in the RTCs of the cities of Makati, Pasay, and Pasig. Therefore, the
surety bond issued by the bonding company should not have been accepted by the RTC of Dasmariñas,
Branch 90, since the certification secured by the bonding company from the OCA at the time of the
issuance of the bond certified that it may only be accepted in the above-mentioned cities. Thus, the trial
court acted with grave abuse of discretion amounting to lack of or in excess of jurisdiction when it issued
the writ of attachment founded on the said bond.

Remedial Law Review II (CY 2018-2019) – Saturday 1:00pm – 5:00pm: Case Digest Compilation [Atty. Brondial] Page 5
(b)That the court did not acquire jurisdiction over the person of the defendant
This Court has long put to rest the issue of when jurisdiction over the person of the defendant should be is
devoid of meritacquired in cases where a party resorts to provisional remedies. A party to a suit may, at
any time after filing the complaint, avail of the provisional remedies under the Rules of Court.
Specifically, Rule 57 on preliminary attachment speaks of the grant of the remedy "at the commencement
of the action or at anytime before entry of judgment." This phrase refers to the date of the filing of the
complaint, which is the moment that marks "the commencement of the action." The reference plainly is to
a time before summons is served on the defendant, or even before summons issues.

In the instant case, assuming arguendo that the trial court validly issued the writ of attachment on
November 15, 2002, which was implemented on November 19, 2002, it is to be noted that the summons,
together with a copy of the complaint, was served only on November 21, 2002.

At the time the trial court issued the writ of attachment on November 15, 2002, it can validly to do so
since the motion for its issuance can be filed "at the commencement of the action or at any time before
entry of judgment." However, at the time the writ was implemented, the trial court has not acquired
jurisdiction over the persons of the respondent since no summons was yet served upon them. The proper
officer should have previously or simultaneously with the implementation of the writ of
attachment,served a copy of the summons upon the respondents in order for the trial court to have
acquired jurisdiction upon them and for the writ to have binding effect. Consequently, even if the writ of
attachment was validly issued, it was improperly or irregularly enforced and, therefore, cannot bind and
affect the respondents.

(2)Whether or not respondents are barred by estoppel, laches, and prescription from questioning the
orders of the RTC issuing the writ of attachment and that the issue has become moot and academic by the
renewal of the attachment bond covering after its expiration.
No, respondents are not barred. As correctly held by the CA:

There are two ways of discharging the attachment. First, to file a counter-bond in accordance with
Section12 of Rule 57.Second[,] [t]o quash the attachment on the ground that it was irregularly or
improvidently issued, as provided for in Section 13 of the same rule.

Whether the attachment was discharged by either of the two ways indicated in the law, the attachment
debtor cannot be deemed to have waived any defect in the issuance of the attachment writ by simply
availing himself of one way of discharging the attachment writ, instead of the other. The filing of a
counter-bond is merely a speedier way of discharging the attachment writ instead of the other way.

Moreover, again assuming arguendo that the writ of attachment was validly issued, although the trial
court later acquired jurisdiction over the respondents by service of the summons upon them, such
belatedservice of summons on respondents cannot be deemed to have cured the fatal defect in the
enforcementof the writ. The trial court cannot enforce such a coercive process on respondents without
first obtaining jurisdiction over their person. The preliminary writ of attachment must be served after or
simultaneous with the service of summons on the defendant whether by personal service, substituted
service or by publication as warranted by the circumstances of the case. The subsequent service of
summons does not confer a retroactive acquisition of jurisdiction over her person because the law does
not allow for
retroactivity of a belated service.

Remedial Law Review II (CY 2018-2019) – Saturday 1:00pm – 5:00pm: Case Digest Compilation [Atty. Brondial] Page 6
A.4. ANITA MANGILA, petitioner,VS. COURT OF APPEALS and LORETA GUINA,
respondents.

G.R. No. 125027; August 12, 2002; CARPIO, J.

FACTS: Petitioner contracted the freight forwarding services of Private Respondent for shipment of
Petitioner’s products. Petitioner agreed to pay Private Respondent cash on delivery. However, Petitioner
failed to pay Private Respondent shipping charges amounting to P109,376.95. Despite several demands,
Petitioner never paid Private Respondent. Private Respondent filed a civil case before the Regional Trial
Court (Trial Court) of Pasay City for collection of sum of money.

The sheriff filed his Sheriffs Return showing that summons was not served on petitioner. The sheriff
found out further that petitioner had left the Philippines for Guam. Private Respondent filed a Motion for
Preliminary Attachment construing Petitioner’s departure from the Philippines as done with intent to
defraud his creditors.The Trial Court issued a Writ of Preliminary Attachment. Petitioner filed an Urgent
Motion to Discharge Attachment because of failure to serve summons on her before or simultaneously
with the writs implementation. Petitioner claims that the trial court had not acquired jurisdiction over her
person and thus the service of the writ is void. The Trial Court granted the Motion to Discharge
Attachment upon filing of Petitioner’s counter-bond. Private Respondent applied for an alias summons,
which the trial court issued on January 19, 1989

The Trial Court issued an Order terminating the pre-trial and allowing the private respondent to present
evidence ex-parte because the Petitioner’s counsel is nowhere to be found. The Trial Court ruled in favor
of Private Respondent. Petitioner appealed the case to the Court of Appeals. The Court of Appeals upheld
the validity of the issuance of the writ of attachment and sustained the filing of the action in the RTC of
Pasay.

ISSUE: Whether or not Respondent Court erred in not holding that the Writ of Attachment was
improperly issued and served?

RULING: Yes. Writ of Attachment was improperly issued and served.

This Court has long settled the issue of when jurisdiction over the person of the defendant should be
acquired in cases where a party resorts to provisional remedies. We have held that the grant of the
provisional remedy of attachment involves three stages: first, the court issues the order granting the
application; second, the writ of attachment issues pursuant to the order granting the writ; and third, the
writ is implemented. For the initial two stages, it is not necessary that jurisdiction over the person of the
defendant be first obtained. However, once the implementation of the writ commences, the court must
have acquired jurisdiction over the defendant for without such jurisdiction, the court has no power and
authority to act in any manner against the defendant. Any order issuing from the Court will not bind the
defendant.

In the instant case, the Writ of Preliminary Attachment was issued on September 27, 1988 and
implemented on October 28, 1988. However, the alias summons was served only on January 26, 1989 or
almost three months after the implementation of the writ of attachment.

We hold that the alias summons belatedly served on petitioner cannot be deemed to have cured the fatal
defect in the enforcement of the writ. The trial court cannot enforce such a coercive process on petitioner
without first obtaining jurisdiction over her person. The preliminary writ of attachment must be served
after or simultaneous with the service of summons on the defendant whether by personal service,
substituted service or by publication as warranted by the circumstances of the case. The subsequent
service of summons does not confer a retroactive acquisition of jurisdiction over her person because the
law does not allow for retroactivity of a belated service.

DOCTRINE:The court must have acquired jurisdiction over the defendantonce the implementation of
the Writ of Attachment commences. For without such jurisdiction, the court has no power and authority
to act in any manner against the defendant. The subsequent service of summons cannot be deemed to
have cured the fatal defect in the enforcement of the writ.

Remedial Law Review II (CY 2018-2019) – Saturday 1:00pm – 5:00pm: Case Digest Compilation [Atty. Brondial] Page 7
A.5. VICENTE B. CHUIDIAN vs. SANDIGANBAYAN

G.R. No. 139941, January 19, 2001

FACTS: Petitioner Chuidian, allegedly, was a dummy of the Marcoses in several companies, one of
which iss the Asian Reliability Company, Incorporated (ARCI) which is 98% owned by him.

PHILGUARANTEE granted ARCI a loan guarantee of US$25 million. In June 1985, Philguarantee sued
Chuidian before a US Court for violating of the terms of the loan as Chuidian not only defaulted in
payment, but also misused the funds by investing them in US corporations for his personal benefit.

During the pendency of the suit, Philguarantee entered into a compromise agreement with Chuidian
whereby the latter shall assign and surrender title to all his companies in favor of the Philippine
government and the former shall absolve Chuidian from all civil and criminal liability. It was further
stipulated that the Philippine government shall pay Chuidian the amount of US$5.3 million. Initial
payment of US$0.7million was actually received by Chuidian. While for the remaining balance of
US$4.6million was to be paid through an irrevocable Letter of Credit (L/C).

On July 30, 1987, the government filed before the Sandiganbayan civil suit against Chuidian seeking the
reconveyance, reversion, accounting and restitution of all forms of wealth allegedly procured illegally and
stashed away by the defendants. While the case was pending, the Republic of the Philippines filed a
motion for issuance of a writ of attachment over the L/C, on the ground that “Chuidian embezzled or
fraudulently misapplied the funds of ARCI acting in a fiduciary capacity, justifying issuance of the writ
under Section 1(b), Rule 57 of the Rules ofCourt”. Sandiganbayan finding that there was a prima facie
case of fraud committedissued a writ of attachment.

Four (4) years after the issuance of the order of attachment, Chuidian filed a motionto lift the attachment
based on the following grounds: (1) First, he had returned tothe Philippines; hence, the Sandiganbayan's
"most potent ground" for the issuanceof the writ of preliminary attachment no longer existed. Since his
absence in thepast was the very foundation of the Sandiganbayan's writ of preliminary attachment,his
presence in the country warrants the immediate lifting thereof; and (2) Second,there was no evidence at
all of initial fraud or subsequent concealment except forthe affidavit submitted by the PCGG Chairman
citing mere "belief and information"and "not on knowledge of the facts."The Sandiganbayan, however,
denied Chuidian's motion for reconsideration of thedenial of his motion to lift attachment. Hence, the
instant petition for certioraricontending that the respondent Sandiganbayan committed grave abuse of
discretionamounting to lack or excess of jurisdiction.

ISSUES:
(1) Whether the reappearance of a previously absent person against whomwrit of attachment had been
issued is a ground for lifting the writ of preliminaryattachment.
(2) Whether a writ of attachment may be discharged when the fraud which isthe ground for its issuance is
not present.

RULING:
(1)No. To quash a writ of preliminary attachment, the petitioner has two (2) options,namely: (1) to file a
counterbond in accordance with Rule 57, Section 12; or (2) toquash the attachment on the ground that it
was irregularly or improvidently issued,as provided for in Section 13 of the same Rule.

It would appear that petitioner chose the latter because the grounds he raised assailthe propriety of the
issuance of the writ of attachment. By his own admission,however, he repeatedly acknowledged that his
justifications to warrant the lifting ofthe attachment are facts or events that came to light or took place
after the writ ofattachment had already been implemented.
The rule contemplates that the defect must be in the very issuance of the attachmentwrit. For instance, the
attachment may be discharged under Section 13 of Rule 57when it is proven that the allegations of the
complaint were deceptively framed, orwhen the complaint fails to state a cause of action. Supervening
events which mayor may not justify the discharge of the writ are not within the purview of thisparticular
rule.

(2)The merits of the action in which a writ of preliminary attachment has been issuedare not triable on a
motion for dissolution of the attachment, otherwise an applicantfor the lifting of the writ could force a
trial of the merits of the case on a meremotion.When the preliminary attachment is issued upon a ground

Remedial Law Review II (CY 2018-2019) – Saturday 1:00pm – 5:00pm: Case Digest Compilation [Atty. Brondial] Page 8
which is at the same timethe applicant's cause of action; e.g., "an action for money or property embezzled
orfraudulently misapplied or converted to his own use by a public officer, or an officerof a corporation, or
an attorney, factor, broker, agent, or clerk, in the course of hisemployment as such, or by any other person
in a fiduciary capacity, or for a wilful violation of duty," or "an action against a party who has been guilty
of fraud incontracting the debt or incurring the obligation upon which the action is brought,"the defendant
is not allowed to file a motion to dissolve the attachment underSection 13 of Rule 57 by offering to show
the falsity of the factual averments in theplaintiff's application and affidavits on which the writ was based
– and consequentlythat the writ based thereon had been improperly or irregularly issued – the reasonbeing
that the hearing on such a motion for dissolution of the writ would betantamount to a trial of the merits of
the action. In other words, the merits of theaction would be ventilated at a mere hearing of a motion,
instead of at the regulartrial.

Remedial Law Review II (CY 2018-2019) – Saturday 1:00pm – 5:00pm: Case Digest Compilation [Atty. Brondial] Page 9
A.6. LUZON DEVELOPMENT BANK v. ERLINDA KRISHNAN

GR No. 203530; Apr 13, 2015; PERALTA, J.:

FACTS: This is a Petition for Review on Certiorari under Rule 45 of the Rules of Civil Procedure
praying for the annulment of the Decision and Resolution Court of Appeals (CA) which affirmed the
Orders of Regional Trial Court (RTC)-Manila.

ErlindaKhrishnan filed a complaint for Collection of Sum of Money and Damages against petitioners
Luzon Development Bank, Tomas Clemente, and Oscar Ramirez. Erlinda claimed that she is a client of
the petitioner bank wherein she maintained several accounts including time deposits. On several
occasions, when Erlinda presented her Time Deposits Certificates amounting to P28,597,472.70 for
payment because they have become due, petitioners refused to honor them for the reason that they were
fraudulent. Respondent Erlinda likewise applied for a Preliminary Writ of Attachment.

In a Decision granting a Petition for Certiorari filed by Erlinda, the RTC ordered the petitioner bank file a
counter bond in accordance with Sec. 12, Rule 57 within 10 days from the finality of this decision;
otherwise the court in Manila shall immediately reinstate the writ of attachment issued and implemented
in Civil Case No. 01-100046. Respondent Erlinda filed her attachment bond on June 25, 2009 in the
amount of P35,000,000.00 through Visayan Suretyand Insurance Corporation. Meanwhile, petitioners
filed an Omnibus Motion praying that a hearing be held to determine the sufficiency of the attachment
bond and they be allowed to deposit Certificates of Title of real property, and the issuance of the writ of
attachment be held in abeyance. The CA affirmed the Orders of the RTC reinstating the Writ of
Attachment for failure of petitioners to file the required counter-bond. In their petition, petitioners
contend that it has the option to deposit real property, in lieu of cash or a counter-bond, to secure any
contingent lien on its property in the event respondent wins the case. They argue that Section 2 of Rule 57
only mentions the term "deposit," thus, it cannot only be confined or construed to refer to cash.

ISSUE: Whether or not the real property of the bank be deposited in lieu of cash or a counter bond?

RULING: NO, Section 2, Rule 57 of the Rules of Court explicitly states that "[a]n order of attachment
may be issued either ex parte or upon motion with notice and hearing by the court in which the action is
pending, or by the Court of Appeals or the Supreme Court, and must require the sheriff of the court to
attach so much of the property in the Philippines of the party against whom it is issued, not exempt from
execution, as may be sufficient to satisfy the applicant's demand, unless such party makes deposit or gives
a bond as hereinafter provided in an amount equal to that fixed in the order, which may be the amount
sufficient to satisfy the applicant's demand or the value of the property tobe attached as stated by the
applicant, exclusive of costs."

Section 5 of the same Rule likewise states that "[t]he sheriff enforcing the writ shall without delay and
with all reasonable diligence attach, to await judgment and execution in the action, only so much of the
property in the Philippines of the party against whom the writ is issued, not exempt from execution, as
may be sufficient to satisfy the applicant's demand, unless the former makes a deposit with the court from
which the writ is issued, or gives a counter-bond executed to the applicant, in an amount equal to the bond
fixed by the court in the order of attachment or to the value of the property to be attached, exclusive of
costs."

DOCTRINE: Once the writ of attachment has been issued, the only remedy of the petitioners in lifting
the same is through a cash deposit or the filing of the counter-bond. The proximate relation of the word
"deposit" and "amount" is unmistakable in Section 5 of Rule 57. Plainly, in construing said words, it can
be safely concluded that Section 5 requires the deposit of money as the word "amount" commonly refers
to or is regularly associated with a sum of money

Remedial Law Review II (CY 2018-2019) – Saturday 1:00pm – 5:00pm: Case Digest Compilation [Atty. Brondial] Page 10
A.7. NORTHERN ISLANDS, CO., INC.vs SPOUSES DENNIS AND CHERYLIN* 
GARCIA, DOING BUSINESS UNDER THE NAME AND STYLE “ECOLAMP MULTI
RESOURCES

G.R. No. 203240; March 18, 2015; PERLAS-BERNABE, J.

FACTS: Petitioner Northern Islands Co., Inc. (petitioner) filed a Complaint with application for a writ of
preliminary attachment, before the RTC against respondents.  Petitioner caused the delivery to
respondents of various appliances in the aggregate amount of P8,040,825.17; however, the value of the
goods were not paid by respondents despite repeated demands;and respondents fraudulently asserted that
petitioner had no proof that they  had indeed received the quantity of the subject goods.

In connection with the application for a writ of preliminary attachment, petitioner posted a bond, in the
amount of P8,040,825.17.  The RTC issued the writ sought for. Respondents filed a Motion to Discharge
Excess Attachment,alleging that the attachment previously ordered by the RTC exceeded by
P9,232,564.56 given that the estimated value of the attached properties, including the garnished bank
accounts, as assessed by their appraiser, (Lapaz), amounted to P17,273,409.73, while the attachment bond
is only in the amount of P8,040,825.17.

RTC rendered a Decision in the Main Case. Essentially, it dismissed petitioner’s Complaint due to the
absence of any evidence to prove that respondents had agreed to the pricing of the subject goods.

ISSUE: Whether the RTC had lost jurisdiction over the matter of the preliminary attachment after
petitioner appealed the decision in the Main Case

RULING: Yes. Section 9, Rule 41 of the Rules of Court provides that in appeals by notice of appeal,
the court loses jurisdiction over the case upon the perfection of the appeals filed in due time and the
expiration of the time to appeal of the other parties. In this case,  petitioner had duly perfected its
appeal of the RTC’s Decision resolving the Main Case through the timely filing of its Notice of Appeal.

With the RTC’s loss of jurisdiction over the Main Case necessarily comes its loss of jurisdiction over all
matters merely ancillary thereto. Thus, the propriety of conducting a trial by commissioners in order to
determine the excessiveness of the subject preliminary attachment, being a mere ancillary matter to the
Main Case, is now mooted by its supervening appeal. That being said, it is now unnecessary to discuss the
other issues raised herein.

DOCTRINE: Attachment is defined as a provisional remedy by which the property of an adverse party is
taken into legal custody, either at the commencement of an action or at any time thereafter, as a security
for the satisfaction of any judgment that may be recovered by the plaintiff or any proper party.

It is an auxiliary remedy and cannot have an independent existence apart from the main suit or claim
instituted by the plaintiff against the defendant. Being merely ancillary to a principal proceeding, the
attachment must fail if the suit itself cannot be maintained as the purpose of the writ can no longer be
justified.

The consequence is that where the main action is appealed, the attachment which may have been issued
as an incident of that action, is also considered appealed and so also removed from the jurisdiction of the
court a quo.  The attachment itself cannot be the subject of a separate action independent of the
principal action because the attachment was only an incident of such action.

Remedial Law Review II (CY 2018-2019) – Saturday 1:00pm – 5:00pm: Case Digest Compilation [Atty. Brondial] Page 11
A.8. EXCELLENT QUALITY APPAREL, INC., Petitioners, vs. VISAYAN SURETY &
INSURANCE CORPORATION, and FAR EASTERN SURETY & INSURANCE CO., INC.,
Respondent.

G.R. No. 212025;July 1, 2015; MENDOZA, J.:

FACTS: Win Multi-Rich filed a complaint for sum of money and damages against petitioner and Ying
before the RTC. It also prayed for the issuance of a writ of attachment, claiming that Ying was about to
abscond and that petitioner had an impending closure. Win Multi-Rich then secured the necessary bond in
the amount of P8,634,448.20 from respondent Visayan Surety and Insurance Corporation (Visayan
Surety). The RTC issued a writ of preliminary attachment in favor of Win Multi-Rich. To prevent the
enforcement of the writ of preliminary attachment on its equipment and machinery, petitioner issued a
Check in the amount of P8,634,448.20 payable to the Clerk of Court of the RTC. Petitioner, then, filed its
Omnibus Motion, seeking to discharge the attachment. The motion, however, was denied by the RTC.

Win Multi-Rich then filed a motion, to release petitioner’s cash deposit to it. The motion was granted by
the RTC. Subsequently, Win Multi-Rich posted a Surety Bond issued by respondent FESICO for the
amount of P9,000,000.00, to secure the withdrawal of the cash deposited by petitioner. Thus, Win Multi-
Rich was able to receive the funds of petitioner even before the trial began. Petitioner filed a petition for
certiorari before the CA. The petition sought to annul and set aside the Orders of the RTC. The CA
rendered a decision, annulling the Orders of the RTC. Petitioner filed a motion for reconsideration
arguing, among others, that the CA decision failed to state an order to return the garnished amount of
P8,634,448.20, which was taken from its bank account and given to Win Multi-Rich. The CA denied the
motion.

Aggrieved, petitioner elevated the matter to the Court by way of a petition for review on certiorari under
Rule 45. The Court promulgated a Decision in favor of petitioner and RULING: first, that Win Multi-
Rich was not a real party in interest; second, that the RTC should not have taken cognizance of the
collection suit because the presence of the arbitration clause vested jurisdiction on the CIAC over all
construction disputes between petitioner and Multi-Rich; and lastly, that Win Multi-Rich could not retain
the garnished amount, as the RTC did not have jurisdiction to issue the questioned writ of attachment and
to order the release of the funds. Petitioner moved for execution thereof, praying for the return of its cash
deposit and, in the event of refusal of Win Multi-Rich to comply, to hold Visayan Surety and FESICO
liable under their respective bonds. Win Multi-Rich opposed the motion for Execution because the cash
deposit awarded to it by the RTC had been paid to suppliers and the said amount was long overdue and
demandable. The RTC granted the motion for execution and issued a writ of execution. The CA found
petitioner’s appeal without merit. Citing Section 20, Rule 57 of the 1997 Rules of Civil Procedure
(Section 20, Rule 57), the CA held that petitioner failed to timely claim damages against the surety before
the decision of the Court became final and executory. It further stated that a court judgment could not
bind persons who were not parties to the action as the records showed that Visayan Surety and FESICO
were neither impleaded nor informed of the proceedings before the Court. It was the view of the CA that
"having failed to observe very elementary rules of procedure which are mandatory, [petitioner] caused its
own predicament."

ISSUE: Whether or not petitioner can claim damages against the bond issued by both surety companies
in the execution of the judgment with preliminary attachment against them.

RULING: No. Visayan Surety could not be held liable under Section 20, Rule 57. In a catena of cases,
the Court has cited the requisites under Section 20, Rule 57 in order to claim damages against the bond, as
follows:
1. The application for damages must be filed in the same case where the bond was issued;
2. Such application for damages must be filed before the entry of judgment; and
3. After hearing with notice to the surety.

The first and second requisites, as stated above, relate to the application for damages against the bond. An
application for damages must be filed in the same case where the bond was issued, either (a) before the
trial or (b) before the appeal is perfected or (c) before the judgment becomes executory. The usual
procedure is to file an application for damages with due notice to the other party and his sureties. The
other method would be to incorporate the application in the answer with compulsory counterclaim.

In the present petition, the requisite that must be satisfied by petitioner to hold Visayan Surety liable
would be that the judgment against the wrongful attachment was promulgated after the hearing with

Remedial Law Review II (CY 2018-2019) – Saturday 1:00pm – 5:00pm: Case Digest Compilation [Atty. Brondial] Page 12
notice to the surety. Certainly, the surety must be given prior notice and an opportunity to be heard with
respect to the application for damages before the finality of the judgment. The Court rules that petitioner
did not satisfy this crucial element.

While Visayan Surety could not be held liable under Section 20, Rule 57, the same cannot be said of
FESICO. To forestall the enforcement of the writ of preliminary attachment, petitioner issued a Check in
the amount of P8,634,448.20 payable to the Clerk of Court of the RTC. The procedure to discharge the
writ of preliminary attachment is stated in Section 12, Rule 57, to wit:

Sec. 12. Discharge of attachment upon giving counterbond.


After a writ of attachment has been enforced, the party whose property has been attached, or the
person appearing on his behalf, may move for the discharge of the attachment wholly or in part
on the security given. The court shall, after due notice and hearing, order the discharge of
the attachment if the movant makes a cash deposit, or files a counter-bond executed to the
attaching party with the clerk of the court where the application is made, in an amount
equal to that fixed by the court in the order of attachment, exclusive of costs.

Strictly speaking, the surety bond of FESICO is not covered by any of the provisions in Rule 57 of the
Rules of Court because, in the first place, Win Multi-Rich should not have filed its motion to release the
cash deposit of petitioner and the RTC should not have granted the same. The release of the cash deposit
to the attaching party is anathema to the basic tenets of a preliminary attachment.

The chief purpose of the remedy of attachment is to secure a contingent lien on defendant’s property until
plaintiff can, by appropriate proceedings, obtain a judgment and have such property applied to its
satisfaction, or to make some provision for unsecured debts in cases where the means of satisfaction
thereof are liable to be removed beyond the jurisdiction, or improperly disposed of or concealed, or
otherwise placed beyond the reach of creditors. The garnished funds or attached properties could only be
released to the attaching party after a judgment in his favor is obtained. Under no circumstance,
whatsoever, can the garnished funds or attached properties, under the custody of the sheriff or the
clerk of court, be released to the attaching party before the promulgation of judgment.

The surety bond of FESICO substituted the cash deposit of petitioner as a security for the judgment. Thus,
to claim damages from the surety bond of FESICO, Section 17, Rule 57 could be applied.

A surety on a counter-bond given to secure the payment of a judgment becomes liable for the payment of
the amount due upon: (1) demand made upon the surety; and (2) notice and summary hearing on the same
action. Noticeably, unlike Section 20, Rule 57, which requires notice and hearing before the finality of the
judgment in an application for damages, Section 17, Rule 57 allows a party to claim damages on the
surety bond after the judgment has become executory.

Under Section 20, Rule 57, in relation to Section 4 therein, the surety bond shall answer for all the costs
which may be adjudged to the adverse party and all damages which he may sustain by reason of the
attachment. In other words, the damages sought to be enforced against the surety bond are unliquidated.
Necessarily, a notice and hearing before the finality of judgment must be undertaken to properly
determine the amount of damages that was suffered by the defendant due to the improper attachment.
These damages to be imposed against the attaching party and his sureties are different from the principal
case, and must be included in the judgment. On the other hand, under Section 17, Rule 57, in relation to
Section 12 therein, the cash deposit or the counter-bond shall secure the payment of any judgment that the
attaching party may recover in the action. Stated differently, the damages sought to be charged against the
surety bond are liquidated. The final judgment had already determined the amount to be awarded to the
winning litigant on the main action. Thus, there is nothing left to do but to execute the judgment against
the losing party, or in case of insufficiency, against its sureties.

FESICO cannot escape liability on its surety bond issued in favor of petitioner. The purpose of FESICO's
bond was to secure the withdrawal of the cash deposit and to answer any damages that would be inflicted
against petitioner in the course of the proceedings. Also, the undertaking signed by FESICO stated that
the duration of the effectivity of the bond shall be from its approval by the court until the action is fully
decided, resolved or terminated.

DOCTRINE: Attachment; By its nature, preliminary attachment, under Rule 57 of the Rules of Court,
“is an ancillary remedy applied for not for its own sake but to enable the attaching party to realize upon

Remedial Law Review II (CY 2018-2019) – Saturday 1:00pm – 5:00pm: Case Digest Compilation [Atty. Brondial] Page 13
relief sought and expected to be granted in the main or principal action; it is a measure auxiliary or
incidental to the main action.”
Attachment Bond; The party applying for the order of attachment must thereafter give a bond executed to
the adverse party in the amount fixed by the court in its order granting the issuance of the writ.
Damages; The usual procedure is to file an application for damages with due notice to the other party
and his sureties. The other method would be to incorporate the application in the answer with
compulsory counterclaim.

Remedial Law Review II (CY 2018-2019) – Saturday 1:00pm – 5:00pm: Case Digest Compilation [Atty. Brondial] Page 14
A.9. WATERCRAFT VENTURE CORPORATION, REPRESENTED BY ITS VICE-
PRESIDENT, ROSARIO E. RAÑOA, PetitionerVS. ALFRED RAYMOND WOLFE, Respondent

G. R. No. 181721; September 9, 2015; Peralta, J.

FACTS: Sometime in June 1997, Watercraft hired respondent Wolfe, a British national and resident of
Subic Bay Freeport Zone,Zambales, as it’s Shipyard Manager.Duringhis employment, Wolfe stored the
sailboat, Knotty Gull, within Watercraft's boat storage facilities, but never paid for the storage fees. On
March 2002, Watercraft terminated the employment of Wolfe.Sometime in June 2002, Wolfe pulled out
his sailboat fromWatercraft's storage facilities after signing a Boat Pull-Out Clearance where he allegedly
acknowledged the outstanding obligationof Sixteen Thousand Three Hundred and Twenty-Four and
82/100 USDollars (US$16,324.82) representing unpaid boat storage fees for the periodof June 1997 to
June 2002. Despite repeated demands, he failed to pay the said amount.

On July 2005, Watercraft filed against Wolfe a Complaint forCollection of Sum of Money with Damages
with an Application for theIssuance of a Writ of Preliminary Attachment, which the RTC granted.
Pursuant to the Order dated July 2005, the Writ of Attachmentand the Notice of Attachment were issued,
and Wolfe's two vehicles, a gray Mercedes Benz and a maroon Toyota Corolla,were levied upon. On
August 2005, Wolfe's accounts at the Bank of the PhilippineIslands were also garnished.

On November 8, 2005, Wolfe filed a Motion to Discharge the Writ ofAttachment, arguing that Watercraft
failed to show the existence of fraudand that the mere failure to pay or perform an obligation does not
amount tofraud. He also claimed that he is not a flight risk citing different reasons. The RTC denied
Wolfe's Motion toDischarge Writ of Attachment and Motion for Preliminary Hearing for
lackofmerit.Wolfe filed a motion for reconsideration, but the RTC also denied itfor lack of merit.
Aggrieved, Wolfefiled a petition for certiorari before the CA.The CA granted Wolfe's petition stating that
the act of issuing the writ of preliminary attachmentex-parte constitutes grave abuse of discretion on the
part of the RTC. Dissatisfied with the CA Decision and Resolution, Watercraft filedthis petition for
review on certiorari

ISSUE: WON Watercraft's affidavit of merit sufficiently established facts which constituteas grounds
upon which attachment may be issued under Section 1 (a) and (d), Rule 57.

RULING: NO. It [Writ of Preliminary Attachment] must be granted only on concrete and specific
grounds and not merely ongeneral averments quoting the words of the rules. Since attachment isharsh,
extraordinary, and summary in nature, the rules on the application ofa writ of attachment must be strictly
construed in favor of the defendant.

After a careful perusal of the […] allegations, the Court agreeswith the CA that Watercraft failed to state
with particularity thecircumstances constituting fraud, as required by Section 5, Rule 8 of theRules of
Court, and that Wolfe's mere failure to pay the boat storage feesdoes not necessarily amount to fraud,
absent any showing that such failurewas due to insidious machinations and intent on his part to
defraudWatercraft of the amount due it.

The Courtexplained that to constitute a ground for attachment in Section 1(d), Rule 57of the Rules of
Court, it must be shown that the debtor in contracting thedebt or incurring the obligation intended to
defraud the creditor. A debt isfraudulently contracted if at the time of contracting it, the debtor has
apreconceived plan or intention not to pay. “The fraud must relate to theexecution of the agreement and
must have been the reason which inducedthe other party into giving consent which he would not have
otherwisegiven.”1

[…] the applicant for a writ of preliminaryattachment must sufficiently show the factual circumstances of
the allegedfraud because fraudulent intent cannot be inferred from the debtor's merenon-payment of the
debt or failure to comply with his obligation. Theparticulars of such circumstances necessarily include the
time, persons,places and specific acts of fraud committed. An affidavit which does notcontain concrete
and specific grounds is inadequate to sustain the issuance ofsuch writ. In fact, mere general averments
render the writ defective and thecourt that ordered its issuance acted with grave abuse of discretion
amounting to excess of jurisdiction.

1
Liberty Insurance Corporation vs. Court of Appeals

Remedial Law Review II (CY 2018-2019) – Saturday 1:00pm – 5:00pm: Case Digest Compilation [Atty. Brondial] Page 15
With respect to Section 1 (a), Rule 57, the other ground invoked byWatercraft for the issuance of the writ
of preliminary attachment, the Courtfinds no compelling reason to depart from the CA's exhaustive ruling
to theeffect that such writ is unnecessary because Wolfe is not a flight risk.

Remedial Law Review II (CY 2018-2019) – Saturday 1:00pm – 5:00pm: Case Digest Compilation [Atty. Brondial] Page 16
A.10. PHIL-AIR CONDITIONING CENTER VS. RCJ LINES AND ROLANDO ABADILLA, JR.,

G.R. No. 193821; November 23, 2015; BRION, J.

FACTS: On various dates betweenMarch 5, 1990, andAugust 29, 1990, petitioner Phil-Air sold to
respondent RCJ Lines four Carrier Paris 240 airconditioning units for buses (units). The unitsincluded
compressors, condensers,evaporators, switches, wiring, circuit boards,brackets, and fittings.Phil-Air
allegedly performed regular maintenance check on the units pursuant tothe one-year warranty on parts
and labor. RCJLines issued three post-dated checks in favor ofPhil-Air to partly cover the unpaid balance.
All the post-dated checks were dishonoured when Phil-Air subsequently presented them forpayment.
Check No. 479759 was returnedbecause it was drawn against insufficient funds,while Check Nos. 479760
and 479761 werereturned because payments were stopped.Before presenting the third check for payment,
Phil-Air sent a demand letter to RolandoAbadilla, Sr. asking him to fund the post-datedchecks. In view of
the failure of RCJ Lines to paythe balance despite demand, Phil-Air filed onApril 1, 1998 the complaint
for sum of moneywith prayer for the issuance of a writ ofpreliminary attachment.In its answer with
compulsory counterclaim, RCJLines admitted that it purchased the units in thetotal amount of
P1,240,000.00 and that it hadonly paid P400,000.00. It refused to pay thebalance because Phil-Air
allegedly breached itswarranty.RCJ Lines averred that the units did notsufficiently cool the buses despite
repeatedrepairs. Phil-Air purportedly represented thatthe units were in accord with RCJ Lines’
coolingrequirements as shown in Phil-Air’s pricequotation. The price quotation provided thatfull payment
should be made upon the units’complete installation. Complete installation,according to RCJ Lines, is
equivalent to being inoperational condition.RCJ Lines claimed that it was also entitled to bereimbursed
for costs and damages occasionedby the enforcement of the writ of attachment.

ISSUE: WON Phil-Air should reimburse RCJ Linesfor the counter-bond premium and its
allegedunrealized profits

RULING: A writ of preliminary attachment is a provisionalremedy issued by the court where an action
ispending to be levied upon the property orproperties of the defendant. The property isheld by the sheriff
as security for thesatisfaction of whatever judgment that mightbe secured by the attaching party against
thedefendant.The grant of the writ is conditioned not only onthe finding of the court that there exists a
validground for its issuance. The Rules also requirethe applicant to post a bond.Section 4 of Rule 57 of
the Rules of CivilProcedure (Rules) provides that “the partyapplying for the order must…give a
bondexecuted to the adverse party in the amountfixed by the court in its order granting theissuance of the
writ, conditioned that the latterwill pay all the costs that may be adjudged tothe adverse party and all
damages that he maysustain by reason of the attachment, if thecourt shall finally adjudge that the
applicantwas not entitled thereto.”The enforcement of the writ notwithstanding,the party whose property
is attached isafforded relief to have the attachment lifted.There are various modes of
discharginganattachment under Rule 57, viz.:

(1) by depositing cash or posting a counterbond under Section 12;


(2) by proving that the attachment bond was improperly or irregularly issued or enforced, orthat the bond
is insufficient under Section 13;
(3) by showing that the attachment is excessiveunder Section 13; and
(4) by claiming that theproperty is exempt from execution underSection 2.RCJ Lines availed of the first
mode by posting acounter-bond.Under the first mode, the court will order thedischarge of the attachment
after (1) themovant makes a cash deposit or posts acounterbond and (2) the court hears the motionto
discharge the attachment with due notice tothe adverse party.The amount of the cash deposit or
counterbond must be equal to that fixed by the court inthe order of attachment, exclusive of costs.
Thecash deposit or counter-bond shall secure thepayment of any judgment that the attachingparty may
recover in the action.The discharge under Section 12 takes effectupon posting of a counter-bond or
depositingcash, and after hearing to determine thesufficiency of the cash deposit or counter-bond.On the
other hand, the discharge under Section13 takes effect only upon showing that theplaintiff’s attachment
bond was improperly orirregularly issued, or that the bond isinsufficient. The discharge of the
attachmentunder Section 13 must be made only afterhearingAs discussed above, it is patent that under
theRules, the attachment bond answers for alldamages incurred by the party against whomthe attachment
was issued. Thus, Phil-Aircannot be held directly liable for the costsadjudged to and the damages
sustained by RCJLines because of the attachment. Section 4 ofRule 57 positively lays down the rule that
theattachment bond will pay “all the costs whichmay be adjudged to the adverse party and alldamages
which he may sustain by reason ofthe attachment, if the court shall finallyadjudge that the applicant was
not entitledthereto.”The RTC, instead of declaring Phil-Air liable forthe alleged unrealized profits and
counter-bondpremium, should have ordered the execution ofthe judgment award on the attachment

Remedial Law Review II (CY 2018-2019) – Saturday 1:00pm – 5:00pm: Case Digest Compilation [Atty. Brondial] Page 17
bond.To impose direct liability to Phil-Air woulddefeat the purpose of the attachment bond,which was not
dissolved despite the lifting ofthe writof preliminary attachment.The order to refund the counter-bond
premiumis likewise erroneous. The premium paymentmay be deemed a cost incurred by RCJ Lines tolift
the attachment. Such cost may be chargedagainst the attachment bond.

Remedial Law Review II (CY 2018-2019) – Saturday 1:00pm – 5:00pm: Case Digest Compilation [Atty. Brondial] Page 18
B. PRELIMINARY INJUNCTION (RULE 58)

B.1. TERESITA V. IDOLOR, Petitioner, VS. HONORABLE COURT OF APPEALS, SPS


GUMERSIDO DE GUZMAN AND ILUMINADA DE GUZMAN AND HON. PRUDENCIO
CASTILLO, JR., PRESIDING JUDGE REGIONAL TRIAL COURT, NATIONAL CAPITAL
JUDICIAL REGION, BRANCH 220, QUEZON CITY, Respondents

GR no. 141853; February 7, 2001; Gonzaga – Reyes, J

FACTS: Idolor the petitioner executed Real Estate Mortgage with extrajudicial foreclosure in favor of De
Guzman to secure the contracted loan, Idolor failed to settle the loan on the agreed date in which De
Guzman filed a complaint before the office of Barangay Captain, the complaint resulted a compromise
agreement in which Idolor was given 90 days extension including a condition that upon failure to settle
the amount due on the agreed date, Idolor will execute a deed of sale with agreement to repurchase
without interest within one year.

On the agreed date Idolor failed to settle the loan as extended in the compromise agreement, De Guzman
filed an extra-judicial foreclosure of the Real Estate Mortgage pursuant to compromise agreement and
thereafter sold through Public Auction, Idolor file a complaint before the RTC for annulment of sheriff’s
certificate of sale with prayer for the issuance of Temporary Restraining Order and Writ of Preliminary
Injunction against the private respondent De Guzman, Deputy Sheriff and the Registry of Deeds of QC
for the issuance of the Deed of Sale and Consolidation of ownership, the Trial Court issued Temporary
Restraining Order and thereafter writ of Preliminary Injunction.

Private respondent De Guzman filed petition for certiorari before the Court of Appeals for the annulment
of Writ of Preliminary Injunction in which CA thereafter granted, Idolor filed for petition for review
before the Supreme Court.

ISSUE: Whether the petitioner has proprietary right to the issuance of Writ of Injunction?

RULING: The Petitioner has no more proprietary right to speak of over the foreclose property to entitle
the petitioner to the issuance of Writ of Preliminary Injunction, as the petitioner failed to exercise the
redeemed the property within one year from the registration of sheriff’s sale. The petitioner failed to show
sufficient interest or title in the property.

As the “KasunduangPagaayos” be executed before the “LuponTagapamayapa” did not novate the original
contract, as there exists no express agreement to abrogate their original undertaking and the compromise
agreement acknowledges the existence of the petitioner indebtedness as the agreed amount will be
liquidated within the 90 days extension, a failure of which the petitioner agreed to execute deed of sale of
the mortgage property.

The compromise agreement has the force and effect of a final judgement and the precisely the reason why
the respondent foreclose the mortgage after the failure of the petitioner to comply with her obligation and
right to redeem the property has already expired.

The failure to settle the obligation effectively delayed the private respondent’s right to extrajudicially
foreclose the Real Estate Mortgage hence the petitioner is not entitled to the equitable relief of injunction.

DOCTRINE: The existence of a right violated is a pre-requisite to the granting of injunction.

Injunction is NOT designed to protect contingent and future rights, failure to established either the
existence of clear and positive right or the defendant has committed or has attempted to commit any act
which endangered or tends to endanger the existence of such right is sufficient ground for denying the
injunction.

The controlling reason to issue the writ of injunction is that the court may prevent a threatened or
continuous irremediable injury before their claims is adjudicated, it is only resorted to when there is
pressing necessity to avoid injurious consequences which cannot be remedied under any standard
compensation.

Remedial Law Review II (CY 2018-2019) – Saturday 1:00pm – 5:00pm: Case Digest Compilation [Atty. Brondial] Page 19
B.2. RIMEO S. GUSTILO vs. HON. RICARDO S. REAL, SR.,

A.M. No.MTJ-00-1250; February 28, 2001;QUISUMBING, J.

FACTS: Rimeo S. Gustilo charged respondent Judge Ricardo S. Real, Sr., of the Municipal Circuit Trial
Court of Victorias-Manapla, Negros Occidental with gross misconduct, gross incompetence, gross
ignorance of the law, and violation of the Anti-Graft and Corrupt Practices Act relative to Civil Case No.
703-M entitled Weddy C. Libo-on v. Rimeo S. Gustilo, et al. for recounting of two precincts.

Complainant was a candidate for punong barangay of Barangay Punta Mesa, Manapla, Negros Occidental
in the May 12, 1997 elections. His lone opponent was Weddy C. Libo-on, then the incumbent punong
barangay and the representative of the Association of Barangay Captains. Both complainant and Libo-on
garnered eight hundred nineteen (819) votes during the elections, resulting in a tie. The breaking of the tie
by the Board of Canvassers was in complainants favor and he was proclaimed duly elected punong
barangay of Punta Mesa, Manapla.

His opponent filed an election protest case, docketed as Civil Case No. 703-M, before the MCTC of
Victorias-Manapla, Negros Occidental. The respondent judge ordered the issuance of summons to the
parties and set the hearing on June 6, 1997. However, Libo-on filed a motion to advance the hearing to
May 29 and 30, 1997. The next day, respondent granted Libo-ons motion. The hearing was advanced to
May 29 and 30, 1997 cancelling the hearing for June 6, 1997. Complainant avers that he was not
furnished a copy of this Order dated May 28, 1997. The respondent judge issued a temporary restraining
order (TRO) and annulled the proclamation of complainant as the duly elected punong barangay of Punta
Mesa, Manapla.

On May 30, 1997, complainant took his oath of office as punong barangay. That same day, he also filed a
petition for certiorari before the Regional Trial Court of Silay City.

ISSUE: Whether or not Judge Ricardo Real, Sr. erroneously issued a TRO for the annulment of the
proclamation.

RULING: Yes. The Office of the Court Administrator (OCA) found that respondent errors were not
honest mistakes in the performance of his duties. Rather, his actions showed a bias in favor of Libo-on
and evinced a pattern to prevent the complainant from assuming office as the duly elected punong
barangay despite his having been proclaimed as such by the Board of Canvassers. The OCA recommends
that respondent be fined P20,000.00 and warned that a repetition of similar acts in the future will be dealt
with more severely.

Supreme Court Administrative Circular No. 20-95 provides:


2. The application for a TRO shall be acted upon only after all parties are heard in a summary
hearing conducted within twenty-four (24) hours after the records are transmitted to the branch selected
by raffle. The records shall be transmitted immediately after raffle
x xx
4. With the exception of the provisions which necessarily involve multiple-sala stations, these
rules shall apply to single-sala stations especially with regard to immediate notice to all parties of all
applications for TRO.

Before an injunctive writ can be issued, it is essential that the following requisites be present: (1) there
must be a right in esse or the existence of a right to be protected; and (2) the act against which injunction
to be directed is a violation of such right. The onus probandi is on movant to show that there exists a right
to be protected, which is directly threatened by the act sought to be enjoined. Further, there must be a
showing that the invasion of the right is material and substantial and that there is an urgent and paramount
necessity for the writ to prevent a serious damage. In this case, complainant had been duly proclaimed as
the winning candidate for punong barangay. He had taken his oath of office. Unless his election was
annulled, he was entitled to all the rights of said office.

DOCTRINE: Whenever an application for a TRO is filed, the court may act on the application only after
all parties have been notified and heard in a summary hearing. In other words, a summary hearing may
not be dispensed with.

Remedial Law Review II (CY 2018-2019) – Saturday 1:00pm – 5:00pm: Case Digest Compilation [Atty. Brondial] Page 20
B.3. MICHAEL J. LAGROSAS, petitioner, vs. BRISTOLMYERS SQUIBB (PHIL.), INC./ MEAD
JOHNSON PHIL., RICHARD SMYTH as General Manager and FERDIESARFATI, as Medical
Sales Director, respondents.

G.R. No. 168637; September 12, 2008; Second DivisionPonente: Justice Quisumbing

FACTS: Michael J. Lagrosas was employed by Bristol-Myers Squibb (Phil.), Inc./Mead Johnson Phil.
from January 6, 1997 until March 23, 2000 as Territory Manager in its Medical Sales Force Division. On
February 4, 2000, Ma.Dulcinea S. Lim, also a Territory Manager and Lagrosas former girlfriend, attended
a district meeting of territory managers at McDonalds Alabang Town Center. After the meeting, she dined
out with her friends. She left her car at McDonalds and rode with Cesar R. Menquito, Jr. When they
returned to McDonalds, Lim saw Lagrosas car parked beside her car. Lim told Menquito not to stop his
car but Lagrosas followed them and slammed Menquitos car thrice. Menquito and Lim alighted from the
car. Lagrosas approached them and hit Menquito with a metal steering wheel lock. When Lim tried to
intervene, Lagrosas accidentally hit her head.

Upon learning of the incident, Bristol-Myers required Lagrosas to explain in writing why he should not be
dismissed for assaulting a co-employee outside of business hours. While the offense is not covered by the
Code of Discipline for Territory Managers, the Code states that other infractions not provided for herein
shall be penalized in the most appropriate manner at the discretion of management. On March 23, 2000,
Bristol-Myers dismissed Lagrosas effective immediately. Lagrosas then filed a complaint for illegal
dismissal, non-payment of vacation and sick leave benefits, 13th month pay, attorney’s fees, damages and
fair market value of his Team Share Stock Option Grant.

On February 28, 2002, Labor Arbiter Renaldo O. Hernandez rendered a Decision declaring the dismissal
illegal. He noted that while Lagrosas committed a misconduct, it was not connected with his work. On
appeal, the National Labor Relations Commission (NLRC) set aside the Decision of Labor Arbiter
Hernandez in its Decision dated September 24, 2002. It held that Lagrosas was validly dismissed for
serious misconduct in hitting his co-employee and another person with a metal steering wheel lock. The
gravity and seriousness of his misconduct is clear from the fact that he deliberately waited for Lim and
Menquito to return to McDonalds.

Bristol-Myers moved to quash the writ of execution contending that it timely filed a petition for certiorari
with the Court of Appeals. The appellate court gave due course to Bristol-Myers petition and issued a
temporary restraining order (TRO) enjoining the enforcement of the writ of execution and notices of
garnishment. Upon the expiration of the TRO, the appellate court issued a writ of preliminary injunction
dated September 17, 2004. Bristol-Myers moved to release the TRO cash bond and injunction cash bond
in view of the Decision dated January 28, 2005. On August 12, 2005, the appellate court denied the
motion as premature since the decision is not yet final and executory due to Lagrosas appeal to this Court.

ISSUE: Whether or not the Court of Appeals erred in disallowing the discharge and release of the
injunction cash bond

RULING: A preliminary injunction may be granted only when, among other things, the applicant, not
explicitly exempted, files with the court where the action or proceeding is pending, a bond executed to the
party or person enjoined, in an amount to be fixed by the court, to the effect that the applicant will pay
such party or person all damages which he may sustain by reason of the injunction or temporary
restraining order if the court should finally decide that the applicant was not entitled thereto. Upon
approval of the requisite bond, a writ of preliminary injunction shall be issued.

The injunction bond is intended as a security for damages in case it is finally decided that the injunction
ought not to have been granted. Its principal purpose is to protect the enjoined party against loss or
damage by reason of the injunction, and the bond is usually conditioned accordingly.

By its Decision dated January 28, 2005, the appellate court disposed of the case by granting Bristol-
Myers petition and reinstating the Decision dated September 24, 2002of the NLRC which dismissed the
complaint for dismissal. It also ordered the discharge of the TRO cash bond and injunction cash bond.
Thus, both conditions of the writ of preliminary injunction were satisfied.

Doctrine:The bond which was posted has no more legal basis considering that the principal action was
already dismissed.

Remedial Law Review II (CY 2018-2019) – Saturday 1:00pm – 5:00pm: Case Digest Compilation [Atty. Brondial] Page 21
B.4. NELSON JENOSA et al. vs. REV. FR. JOSE RENE C. DELARIARTE, O.S.A. et al.

G.R. No. 172138; September 8, 2010

FACTS: Petitioners are students (and their parents) of the University who were caught engaging in
hazing outside the school premises. Dialogues and consultations were conducted among the school
authorities, the apprehended students and their parents. During the November 2002 meeting, the parties
agreed that, instead of the possibility of being charged and found guilty ofhazing, the students who
participated in the hazing incident as initiators, includingpetitioner students, would just transfer to another
school, while those whoparticipated as neophytes would be suspended for one month. The parents of the
apprehended students, including petitioners, affixed their signatures to the minutesof the meeting to
signify their conformity. In view of the agreement, the Universitydid not anymore convene the
Committee on Student Discipline (COSD) toinvestigate the hazing incident.

On 3 January 2003, petitioners filed a complaint for injunction and damages.Petitioners assailed the
Principal’s decision to order the immediate transfer ofpetitioner students as a violation of their right to
due process because the COSD was not convened.

ISSUE: Whether injunction may be issued to enjoin an act previously agreed upon between the applicant
and the party against whom the injunction is sought to be issued.

RULING: The answer is no. Since injunction is the strong arm of equity, he who must applyfor it must
come with equity or with clean hands. This is so because among themaxims of equity are (1) he who
seeks equity must do equity, and (2) he who comesinto equity must come with clean hands. The latter is a
frequently stated maximwhich is also expressed in the principle that he who has done inequity shall not
haveequity. It signifies that a litigant may be denied relief by a court of equity on theground that his
conduct has been inequitable, unfair and dishonest, or fraudulent, or deceitful as to the controversy in
issue.

Here, petitioners, having reneged on their agreement without any justifiable reason, come to court with
unclean hands. This Court may deny a litigant relief if his conduct has been inequitable, unfair and
dishonest as to the controversy in issue.

Remedial Law Review II (CY 2018-2019) – Saturday 1:00pm – 5:00pm: Case Digest Compilation [Atty. Brondial] Page 22
B.5. SOLID BUILDERS, INC. AND MEDINA FOODS INDUSTRIES, INC. VS. CHINA
BANKING CORPORATION

G.R. No. 179665; April 3, 2013; J. Leonardo-De Castro

FACTS: China Banking Corporation (CBC) granted several loans to Solid Builders, Inc. (SBI)
amounting to PhP139M exclusive of interests and other charges.  To secure the said loans, Medina Foods
Industries, Inc. (MFII) [accommodation mortgagor] executed in CBC’s favor several surety agreements
and contracts of real estate mortgage over parcels of land. For failure of SBIto pay its loan (secured with
mortgage), CBC demanded SBI to settle its obligation. Claiming that the interests, penalties and charges
imposed by CBC were iniquitous and unconscionable, and to enjoin CBC from initiating foreclosure
proceedings on the mortgaged properties, SBI and MFII filed a Complaint “To Compel Execution of
Contract and for Performance and Damages, With Prayer for Writ of Preliminary Injunction and Ex-
Parte Temporary Restraining Order” in the Regional Trial Court (RTC) of Pasig City.

The trial court GRANTED the application for issuance of writ of preliminary injunction.Then, CBC filed
a Petition for Certiorari with the Court of Appealswhichgranted the same and ordered the dissolution of
the writ of preliminary injunction. Thus, this petition for review on certiorariby SBI and MFII assailing
the Decision of the Court of Appeals.

ISSUE: Whether or not a writ of preliminary injunction may issue to prevent thecreditor-mortgagee from
foreclosing the mortgaged properties on the ground that the interest rate and penalty charges imposed on
the loan and secured by the mortgage are iniquitous and unconscionable.

RULING: No,in a usurious loan with mortgage, the right to foreclose the mortgage subsists, and this
right can be exercised by the creditor-mortgagee upon failure by the debtor-mortgagor to pay the debt
due.As debtor-mortgagors, however, SBI and MFII do not have a right to prevent the creditor-mortgagee
CBC from foreclosing on the mortgaged properties simply on the basis of alleged “usurious, exorbitant
and confiscatory rate of interest.”  Assuming that the interest rate agreed upon by the parties is usurious,
the nullity of the stipulation of usurious interest does not affect the lender’s right to recover the principal
loan, nor affect the other terms thereof.

Hence, the Supreme Court denied the instant Petition for Review on Certiorari.

DOCTRINE: A writ of preliminary injunction is an extraordinary event which must be granted only in
the face of actual and existing substantial rights.  The duty of the court taking cognizance of a prayer for
a writ of preliminary injunction is to determine whether the requisites necessary for the grant of an
injunction are present in the case before it. In this connection, a writ of preliminary injunction is issued
to preserve the status quo ante, upon the applicant’s showing of two important requisite conditions,
namely: (1) the right to be protected exists prima facie, and (2) the acts sought to be enjoined are
violative of that right.  It must be proven that the violation sought to be prevented would cause an
irreparable injury.

Remedial Law Review II (CY 2018-2019) – Saturday 1:00pm – 5:00pm: Case Digest Compilation [Atty. Brondial] Page 23
B.6. SPOUSES SILVESTRE O. PLAZA AND ELENA Y. PLAZA vs. GUILLERMO LUSTIVA,
ELEODORA VDA. DE MARTINEZ AND VICKY SAYSON GOLOSENO

G.R. No. 172909; March 5, 2014; BRION, J.:

FACTS: Petitioners, SPS PLAZA, filed a complaint for injunction with a prayer for the issuance of a writ
of Preliminary Injunction against the respondents seeking to enjoin the latter from unlawfully and
illegally threatening to take possession of a property they acquired by way of redemption from Virginia
Tuazon a bidder and winner in a tax delinquency sale for the subject property.

Respondents controverted in their answer that they were never delinquent in payment of land taxes and
were not aware that their property has been offered for public auction. Plus, they found out that the said
bidder is a government employee and for being such, she is disqualified from participating into a public
auction; thus, logically, she could not have transferred the property to the petitioners.

The RTC denied the prayer for a Writ of Preliminary Injunction, and ordered that the possession and
occupation of the land be returned to the respondents for the auction sale was tainted with irregularity.
They were also found to have falsified the tax declaration they redeemed the property with. On petition
for review on certiorari, the CA affirmed the RTC decision. In SC, the petitioner alleged that it is the City
Government which is considered the purchaser of the property in auction; therefore, they redeemed it
directly from the said LGU and not from Tuazon. The respondent controverted that the grant of
preliminary injunction lies in the sound discretion of the court and the petitioners failed to show proof that
they are entitled to it. Meanwhile, the main case for injunction was dismissed by the RTC

ISSUE: W/N the petitioner is not entitled to a writ of preliminary injunction on the grounds that they
failed to show proof thereto.

RULING: The petitioner is not entitled to a writ of preliminary injunction because they failed to show
clear and unmistakable rights to be protected by the writ. The lower courts correctly found that Tuazon
had no ownership to confer to the petitioners. Because they were never owners of the property, the
petitioners failed to establish entitlement to the writ of preliminary injunction. To be entitled to an
injunctive writ, the right to be protected and the violation against that right must be shown.

Likewise, upon the dismissal of the main case by the RTC, the question of issuance of the writ of
preliminary injunction has become moot and academic. Upon the dismissal of the main action, the
question of the non-issuance of a writ of preliminary injunction automatically died with it.

DOCTRINE: A writ of preliminary injunction may be issued only upon clear showing of an actual
existing right to be protected during the pendency of the principal action. When the complainant’s right
or title is doubtful or disputed, he does not have a clear legal right and, therefore, the issuance of
injunctive relief is not proper.

A writ of preliminary injunction is a provisional remedy; it is auxiliary, an adjunct of, and subject to the
determination of the main action. It is deemed lifted upon the dismissal of the main case, any appeal
therefrom notwithstanding.

Remedial Law Review II (CY 2018-2019) – Saturday 1:00pm – 5:00pm: Case Digest Compilation [Atty. Brondial] Page 24
B.7. OFFICE OF THE OMBUDSMAN vs. ERNESTO M. DE CHAVEZ, ROLANDO L.
LONTOC, SR., DR. PORFIRIO C. LIGAYA, ROLANDO L. LONTOC, JR. AND GLORIA M.
MENDOZA

G.R. No. 172206; July 3, 2013; PERALTA, J.:

FACTS: Respondents filed a petition for injunction with prayer for the issuance of a temporary
restraining order (preliminary injunction) with the RTC against the Batangas State University Board of
Regents wherein the former group was hired as public officers (maybe teachers?) because the latter
received an order from the Deputy Ombudsman directing it to enforce its decision imposing the penalty
of dismissal from service against the respondents for being found guilty of dishonesty and grave
misconduct.

Respondents alleged that the board should be enjoined from enforcing the Ombudsman decision because
it is still pending on appeal and the same has not yet attained finality. The RTC dismissed the petition for
lack of cause of action. On notice of appeal, the CA issued a resolution granting the prayer for TRO. The
Ombudsman filed with the CA a motion to intervene and to recall the issued TRO; however, the CA
ratiocinated that they will only take cognizance of the pleadings filed by the Ombudsman to give the latter
an opportunity to comment and oppose the application for injunctive relief without allowing them to
formally intervene. The CA favored the respondents stating that the disciplinary action has not yet
attained finality and therefore can be stayed by a TRO.

Petitioner-Ombudsman then filed a petition for review on certiorari with the SC alleging that an appeal
cannot stay the execution of Ombudsman decision. On the other hand, the respondents controverted that
the Ombudsman has no legal personality to intervene; and assuming it has, petition for review on
certiorari is not the proper remedy available.

ISSUE: W/N the respondents are not entitled with the injunctive relief prayed for on the grounds that
decisions of the Ombudsman cannot be stayed by an appeal.

SUB ISSUE: W/N petition for review on certiorari is not the proper remedy available to question an order
or resolution granting an application for injunctive relief.

RULING: Petition for review on certiorari is not the proper remedy available to question an order or
resolution granting an application for injunctive relief.

However, since the Ombudsman was denied the opportunity to be heard, the situation resulted in an issue
of primordial importance which warrants for the relaxation of the application of the rules of court by the
Supreme Court. The Ombudsman had sufficiently alleged its legal interest, as a disciplinary authority
over public officials, in the resolution of the assailed order granting the Writ of Preliminary Injunction;
therefore, they can formally intervene with the pending appeal.

To answer the primary issue, the Supreme Court resolved that the respondents are not entitled with the
injunctive relief prayed for on the grounds that decisions of the Ombudsman cannot be stayed by an
appeal. The rationale is as follows:

For a writ of preliminary injunction to issue, the following essential requisites must concur: 1) that the
applicant has a right which is clear and unmistakable; 2) that there has been a material and substantial
invasion of said right; and, 3) that there is an urgent and paramount necessity to prevent serious damage.

In the present case, the right of respondents is not clear and unmistakable. The reason is that the Rules of
Procedure of the Office of the Ombudsman is merely procedural that it does not touch on vested rights
because the concerned public officer meted by a disciplinary action shall only considered preventively
suspended while his case is on appeal and shall be paid the salary and other emoluments that he did not
receive by reason of suspension when he wins on appeal. Besides, there is no such thing as a vested
interest in an office, or even an absolute right to hold office except in constitutional offices granting
immunity on salary and tenure (stating the other way around, a public officer has no protected rights
against an unfavorable disciplinary action by the Ombudsman).

Moreover, decisions of the Ombudsman on administrative disciplinary cases are immediately


executorywhich may not be stayed by the filing of an appeal or the issuance of an injunctive writ or TRO.
An issuance of a preliminary injunction staying the penalty imposed by the Ombudsman is an

Remedial Law Review II (CY 2018-2019) – Saturday 1:00pm – 5:00pm: Case Digest Compilation [Atty. Brondial] Page 25
encroachment on the rule-making power of its office under the Constitution of the law (R.A. 6770;
stating the other way around, an issuance of a writ of preliminary injunction or TRO on an Ombudsman
decision is unconstitutional).

DOCTRINE: A petition for review on certiorari (under Rule 45) is not the proper remedy to question an
order or resolution granting (or even denying) a Writ of Preliminary Injunction because it is merely an
interlocutory order which do not completely dispose of the case on the merits (Rule 41 Sec. 1 of the Rules
of Court). However, an interlocutory order may be questioned by petition for certiorari (under Rule 65)
when there is no more adequate and expeditious relief. To simplify, interlocutory orders may be question
under Rule 65 in the absence of other adequate reliefs.

The only time wherein a petition for review on certiorari (under Rule 45) may be adequate as a remedy in
assailing interlocutory orders is when the Supreme Court resolves to relax the application of procedural
rules when “erroneously assailed” order is patently erroneous – and that is when there is an outright
deprivation of liberty or property, or where the interest of justice so requires, rendering it of primordial
importance. This determination by the SC is grounded upon the limits in the exercise of court discretion
that it must not be exercised in contravention of a provision of a law or of the Constitution.

For a writ of preliminary injunction to issue, primarily, the applicant must have a right which is clear
and unmistakable.

Decisions and Resolutions of the Ombudsman are immediately executory which may not be stayed by the
filing of an appeal or the issuance of an injunctive writ or TRO.

Remedial Law Review II (CY 2018-2019) – Saturday 1:00pm – 5:00pm: Case Digest Compilation [Atty. Brondial] Page 26
B.7.a. OFFICE OF THE OMBUDSMAN vs. MAXIMO D. SISON

G.R. No. 185954; February 16, 2010; VELASCO, JR, J.:

FACTS: The members of the Isog Han Samar Movement filed a letter-complaint with the Office of the
Ombudsman informing the latter of a well-founded joint audit investigation conducted by the Special
Audit Team composed of the members of the Legal and Adjudication Office and of the Commission on
Audit which yielded anomalies on the expenditure of the calamity fund; t hus, the Ombudsman proceeded
with an administrative case against the impleaded provincial officials of Samar.

One of the said officials is Maximo D. Sison, and in his counter-affidavit, and later on in his Position
Paper, he denied the accusations claiming innocence on the charges. The Ombudsman found that Sison
several other local officials are guilty of grave misconduct, dishonesty, and conduct prejudicial to the best
interest of the service and dismissing him from service.

On a petition for review (under Rule 43), the CA reversed and set aside the Ombudsman decision for
insufficiency of evidence absolving Sison from the administrative liability as charged. On the course of
the appeal, the Ombudsman filed an Omnibus Motion for Intervention and to Admit Attached Motion for
Reconsideration. The motion was denied.

ISSUE: W/N the Ombudsman may intervene in the appeal on the grounds that it has sufficient legal
interest warranting a right to intervene.

RULING: The Ombudsman cannot intervene in the appeal because it has no sufficient legal interest.
Being the one who had previously heard and decided the administrative case, the Ombudsman is not the
proper party to intervene on the pending appeal from the decision which itself decided. It must remain
partial and detached. More importantly, it must be mindful of its role as an adjudicator, not an advocate.

Rule 43, as well as Administrative Circular No. 1-95, provides that a petition for review stemming from
decisions of quasi-judicial agencies shall not implead the deciding court or agency either as petitioners or
respondents. Thus, the only parties in such an appeal are the appellant as petitioner and appellee as
respondent. The court or, in this case, the administrative agency that rendered the judgment appealed
from, is not a party in the said appeal.

The Ombudsman has no legal interest to intervene because it has no legal interest which could be affected
by the outcome of the appeal. In fact, being the one who passed on the judgment, the Ombudsman has no
interest in the case because of its impartial character.

DOCTRINE: The (dis)allowance of a Motion to Intervene depends on the discretion of the court (Rule
19 of the Rules of Court).

To warrant an intervention: 1) the intervenor must have a legal interest in the matter in litigation; and, 2)
the intervention must not unduly delay or prejudice the adjudication of the rights of the persons already
parties to the case.

The legal interest contemplated must be one which will be directly and immediately affected by the
judgment of the case; on the other hand, an intervention shall be deemed an undue delay of the
proceedings if the claim of the intervenor is one which is capable of being properly decided in a separate
proceeding.

If a government party shall intervene in an appeal, it must be the one which prosecutes the administrative
case and not the one which has decided the case.

Other Matters on the Syllabus

Comparing with the De Chavez case, here the Ombudsman intervened in an appeal from the decision
which it decided while in the former, the Ombudsman intervened in an appeal from the decision of the
RTC which decided the case.

Comparing with the De Chavez case, here the appeal stemmed from a decision or order which completely
disposes of the case on the merits while in the former, the appeal stemmed from a mere interlocutory
order (issuance of a writ of preliminary injunction).

Remedial Law Review II (CY 2018-2019) – Saturday 1:00pm – 5:00pm: Case Digest Compilation [Atty. Brondial] Page 27
B.8. SATURNINO NOVECIO, et al. & VERGELIO ROSALES, et al. vs. HON. RODRIGO F.
LIM, JR., as Chairman, HON. LEONCIA R. DIMAGIBA as ponente and as member and HON.
ANGELITA A. GACUTAN as member, former Twenty-Third Division, COURT OF APPEALS,
Mindanao Station, HON. JUDGE BENJAMIN ESTRADA, in his capacity as Presiding Judge of
Branch 9, RTC, Malaybalay, Bukidnon, MARIA CARMEN J. TUAZON and MANUEL V.
NIETO, rep. by their Attorney-in-fact, LOPE DUROTAN

G.R. No. 193809; March 23, 2015; BRION, J.:

FACTS: The respondents filed with the MTC a complaint for forcible entry against the petitioners
alleging that on February 15, 2004, the latter, by force, strategy, threat, intimidation, and stealth (FISTS),
unlawfully squatted and took possession of several portions of land belonging to the respondents. The
petitioners, on the other hand, contended that they have already been in possession of the land for more
than two years when the complaints were filed; further, the landholding of the respondents in the subject
area of land was covered by CARP and was sub-divided in favor of their tenants.

The MTC denied the complaint because they have no evidence of actual and physical possession of the
land; that what the respondents have is only a certification authorizing land survey, a piece of evidence
proving the fact that they have applied title for the subject land. On the other hand, the MTC favored the
petitioners for having prior actual and physical possession of the land for more than two years as
evidenced by a certification issued by the barangay captain proving the fact of their residency. On
ordinary appeal, the RTC reversed the MTC decision ruling that the MTC should have given credence to
the certification issued by the DENR-CENRO which is a proof that respondents were the actual occupants
of the property in litigation long before the petitioners had taken possession of the same property. The
RTC ordered the petitioners' ejectment.

The petitioners file a petition for review with the CA. Meanwhile, the respondents sought the execution of
the RTC judgment, thus the petitioners filed an Extremely Urgent Application. The CA issued a TRO
effective for 60 days. Later on, the CA denied the petitioner’s application for preliminary injunction
without resolving on the merits of the case on appeal. The Motion for Reconsideration thereon was
denied. Now, the petitioners filed a petition for certiorari with the SC alleging that CA committed grave
abuse of discretion in denying the prayer for a writ of preliminary injunction despite the pressing need for
it to prevent grave and irreparable injury to them. On the other hand, the respondents controverted that the
abuse of discretion the CA committed is so grave because it does not amount to a capricious and
whimsical exercise of judgment.

ISSUE: W/N the petitioner is entitled to a writ of preliminary injunction on the grounds that the CA
committed grave abused of discretion on its denial.

RULING: The petitioner is entitled to a writ of preliminary injunction because the CA committed grave
abused of discretion on its denial.

The CA committed grave abuse of discretion when it denied the injunctive relief prayed for by the
petitioners. There is grave abuse of discretion when an act is (1) done contrary to the Constitution, the law
or jurisprudence or (2) executed whimsically, capriciously or arbitrarily, out of malice, ill will or personal
bias. A review of the records shows that the CA ignored relevant facts that would have justified the
issuance of a preliminary injunction. The CA also denied the prayer for preliminary injunction without
giving the factual and legal bases for such denial.

A preliminary injunction is proper when the plaintiff appears to be clearly entitled to the relief sought and
has substantial interest in the right sought to be defended. The existence of a right need not be
conclusively established – it must only appear to be clear based on initial evidence that could readily
justify for the issuance of the writ as it is inconclusive of the principal action which has yet to be decided.
Only an apparent or ostensible right is needed to be established.

In this case, the petitioners have adequately shown their entitlement to a preliminary injunction all in
accordance with the import of Sec. 3 of Rule 58. First, the relief demanded consists in restraining the
execution of the RTC decision ordering their ejectment from the disputed land. Second, their ejectment
from the land from which they derive their source of livelihood would work injustice to the petitioners.
Finally, the execution of the RTC decision is probably in violation of the rights of the petitioners, tending
to render the MTC judgment dismissing the forcible cases entry ineffectual.

Remedial Law Review II (CY 2018-2019) – Saturday 1:00pm – 5:00pm: Case Digest Compilation [Atty. Brondial] Page 28
In determining the issuance of an application for a writ of preliminary injunction, the court must state in
its order the findings and conclusions based on the evidence and the law. In this case, the CA failed to cite
any legal or factual basis for its denial. Thus, it is necessary to go beyond the function of a certiorari
under Rule 65 and examine the factual findings of the MTC and the RTC (again. relaxation of rules).

The MTC found that the petitioners have been in actual and physical possession of the land for more than
two (2) years prior to the institution of the complaints for forcible entry. The MTC also found that the
respondents were not even sure how the petitioners entered the land. In their complaints, they alleged that
petitioners entered the land by means of "force, intimidation, threat, stealth and strategy," a shotgun
allegation which shows that respondents' lack knowledge of how the petitioners entered the disputed
property despite the prescriptive period for filing an action for forcible entry within one year from
dispossession.

The RTC, on the other hand, relied on a mere request for authority to conduct a land survey, allegedly
showing that respondent Manuel V. Nieto was the occupant and tiller of the land. However, this
document does not prove prior possession of the subject land. It only points to the fact that there was an
application for a land title in the name of one of the respondents, which application was not even shown
to have been granted. The declaration regarding possession was just incidental to the application for land
survey.

The clearer findings of the MTC shall be given more weight than the findings of the RTC which is not
based on specific contrary evidence. Under this factual backdrop, the CA committed grave abuse of
discretion when it denied the prayer for preliminary injunction without explanation and justification.

Again, the grant of preliminary injunction would have only been provisional and would not be
conclusively determinative of the principal action. The issuance of the writ would have served its
purpose, i.e., to preserve the status quo or to prevent future wrongs in order to preserve and protect the
interests of the petitioners during the pendency of the action.

DOCTRINE: A preliminary injunction is proper when the plaintiff appears to be clearly entitled to the
relief sought and has substantial interest in the right sought to be defended. The existence of a right need
not be conclusively established – it must only appear to be clear based on initial evidence that could
readily justify for the issuance of the writ as it is inconclusive of the principal action which has yet to be
decided. Only an apparent or ostensible right is needed to be established.

The remedy of petition for certiorari (under Rule 65) is appropriate in assailing interlocutory orders or
resolutions because the same are not appealable (by ordinary appeal) and because there is no more
adequate remedy available for its challenge. An order granting the issuance of a writ of preliminary
injunction is interlocutory in nature.

The grant of preliminary injunction is only provisional and not determinative of the principal action. Its
purpose is to preserve the status quo or to prevent future wrongs in order to preserve and protect the
interests of the applicant during the pendency of the action.

Remedial Law Review II (CY 2018-2019) – Saturday 1:00pm – 5:00pm: Case Digest Compilation [Atty. Brondial] Page 29
B. 9. Liberty Broadcasting Network vs. Atlocom

G.R. No. 205875; June 30, 2015;VILLARAMA, JR., J.:

FACTS: Atlocom Wireless System, Inc. (Atlocom) is a grantee of a legislative franchise under R.A. No.
8605. On October 8, 2003, the National Telecommunications Commission (NTC) issued an Order
granting Atlocom Provisional Authority (PA) to install, operate, and maintain a Multiple-Channel
Distribution Center (MCDC) in Metro Manila. The PA was valid for a period of 18 months, or until April
8, 2005. In a letter dated April 5, 2004, Atlocom requested for an extension of time of the allocation of
frequencies and for the period for the construction and installation of the radio stations.

On August 23, 2005, NTC issued Memorandum Circular No. (MC) 06-08-2005 re-allocating, among
others, band 2500 - 2700 MHz for broadband wireless access for fixed, nomadic and mobile networks.
On December 23, 2008, NTC denied Atlocom's motion for extension of PA, citing the re-allocation of
MMDS frequencies for Broadband Wireless Access in accordance with MC 06-08-2005 and the
unavailability of other alternative frequencies.

In 2009, Atlocom filed in the RTC of Quezon City a Petition to enjoin the implementation of MC 06-08-
2005 and reinstate the frequencies of Atlocom. It was further prayed that after hearing the court render
judgment declaring the said issuance as null and void because NTC unlawfully deprived Atlocom of the
right to its assigned frequencies without notice and hearing.

Liberty Broadcasting Network, Inc. (LBNI), also a grantee of a legislative franchise for radio and
television broadcasting, as well as radio stations for international and domestic communications of all
types and services, and holder of a Certificate of Public Convenience and Necessity (CPCN) to operate a
radio communications network, was allowed to intervene in the case, joining the defendant NTC in
opposing Atlocom's claims. Pursuant to MC 06-08-2005, frequency bands 2535-2545 MHz and 2565-
2595 MHz were re-allocated and assigned to LBNI, which covered the 2572-2596 MHz being claimed by
Atlocom as allegedly assigned to it.

The RTC, after due hearing, issued an Order denying Atlocom's application for a writ of preliminary
prohibitory or mandatory injunction. Atlocom filed a motion for reconsideration but it was likewise
denied by the RTC. In a petition for certiorari filed before the CA, Atlocom questioned the validity of the
aforesaid orders of the RTC. The CA ruled in favor of Atlocom and reversed the RTC's denial of
application for preliminary injunction.

ISSUE: Whether Atlocom had a right to be protected which would justify the issuance of a writ of
preliminary injunction.

RULING: No. Considering that Atlocom has not even launched its MMDS network nor constructed
radio stations, it is doubtful whether Atlocom can exercise the foregoing rights of an affected frequency
user. Neither can Atlocom attribute its non-operational state to the delayed action on its motion for
extension of PA. Among the conditions of its PA is the commencement of the construction and
installation of its station within six months from issuance of the order granting it the provisional authority
and its complete three months thereafter. Perusal of the motion for extension reveals that Atlocom at the
time, was still in the process of identifying and finalizing arrangements with its potential investors for the
establishment of a nationwide MMDS network coverage.

From the evidence on record, no clear, actual and existing right to the subject frequencies or to the
extension of PA had been shown by Atlocom. Accordingly, no grave abuse of discretion was committed
by the RTC in denying Atlocom's application for a writ of preliminary injunction to restrain the
implementation of MC 06-08-2005 insofar as the use of the re-allocated frequencies claimed by Atlocom.
The CA thus seriously erred in reversing the RTC and holding that Atlocom was entitled to injunctive
relief due to alleged violation of its right by the NTC.

DOCTRINE: A right to be protected by injunction, means a right clearly founded on or granted by law
or is enforceable as a matter of law. An injunction is not a remedy to protect or enforce contingent,
abstract, or future rights; it will not issue to protect a right not in esse, and which may never arise, or to
restrain an act which does not give rise to a cause of action.

A writ of preliminary injunction being an extraordinary event, one deemed as a strong arm of equity or a
transcendent remedy, it must be granted only in the face of actual and existing substantial rights. In the

Remedial Law Review II (CY 2018-2019) – Saturday 1:00pm – 5:00pm: Case Digest Compilation [Atty. Brondial] Page 30
absence of the same, and where facts are shown to be wanting in bringing the matter within theconditions
for its issuance, the ancillary writ must be struck down for having been rendered in grave abuse of
discretion.

Remedial Law Review II (CY 2018-2019) – Saturday 1:00pm – 5:00pm: Case Digest Compilation [Atty. Brondial] Page 31
B.10. Republic of the Philippines represented by Commander Raymond Alpuerto of the Naval Base
Camillo Oasis, Port San Vicente, Sta. Ana Cagayan vs. Rev. Claudio R. Cortez, Sr.

G.R. No. 197472; September 07, 2015; J. Del Castillo

FACTS: This is petition for review on certiorari assailing the decision of CA which affirmed the decision
of RTC Cagayan.

Rev. Cortez is a missionary; he established an orphanage in Cagayan. He claimed that since 1962 he had
been in peaceful possession of 50 hec. of land in Palaui isl., with the help of Aetas they cleared the and
developed the land for agricultural purpose to support his charitable activities. The proclamation of Pres.
Marcos in 1967 reserved 2,000 hec. of land in Palaui isl for military purposes but subject to private rights
if there any. During the Ramos administration, the President proclaimed that the whole island of Palaui
and the surrounding waters as marine protected area but again subject to any private rights.

In 2000 Rev. Cortes filed a petition for injunction with prayer of issuance of writ of preliminary
mandatory injunction against Commanding Officer Biñas of Phil. Navy; to restore him possession and to
not disturb the same. Rev. Cortez alleged that through force and intimidation, he and his men disturbed
the lawful possession of the land ordered to vacate the area.

RTC granted the petition, it made reference to the Indigenous People’s Right Act (IPRA). The affected
community belongs to the group of indigenous people which are protected by the State their rights to
continue in their possession of the lands they have been tilling since time immemorial.

On appeal, the OSG contend that Rev. Cortez is not a real party-in-interest; the injunction was filed on
behalf of indigenous people and not in his own capacity as pastor. However CA, dismissed the appeal and
affirmed the decision of RTC.

ISSUE: Whether or not Rev. Cortez is entitled to a final writ of mandatory injunction?

RULING: NO, the RTC confused itself, what it cited in its decision were jurisprudence relating to
preliminary injunction and/or mandatory injunction as an ancillary writ and not as a final injunction. The
duty of RTC was to determine if final injunction is proper, based on evidence presented during trial if
Rev. Cortez conclusively established his claimed right over the subject area, as opposed to preliminary
injunction where an applicant only needs to at least tentatively show that he has the right.

Injunction is a judicial writ, process or proceeding whereby a party is directed either to do a particular act
in which case called mandatory injunction; or to refrain from doing a particular act in which case a
prohibitory injunction. It may be the main action or merely a provisional remedy for and as an incident in
the main action. The main action for injunction is distinct from the provisional or ancillary remedy of
preliminary injunction. The preliminary injunction does not determine the merits of a case or decide
controverted facts, since it is a mere preventive remedy, it only seeks to prevent threatened wrong, further
injury and irreparable harm or injustice until the rights of the parties are settled. On the other hand the
main action for injunction seeks a judgment that embodies a final injunction which perpetually restrain
the party or person enjoined from the commission or continuance of an act, or in case or mandatory
injunctive writ one which confirm the preliminary mandatory injunction. It is issued after the trial on the
merits of the case, that the applicant is entitled to have the acts complained of permanently enjoined.

The case filed by Rev. Cortez in RTC failed to comply with the two requisite for final injunction to be
issued: (1) there must be a right protected and (2) the acts against which the injunction is to be directed
are violative of said right. It the case, Rev. Cortez failed to show right of possession over the area.
Pursuant to Regalian Doctrine all lands of public domain belong to the State since there was no showing
that the subject portion of land has been declared alienable and disposable when Rev. Cortez occupied it,
the land is still considered part of public domain. Thus Rev. Cortez failed to establish his claim right over
said land would not entitle him to the issuance of final injunction.

DOCTRINE:an alienable public land cannot be appropriated and thus may not be proper object of
possession. Hence injunction cannot be issued in order to protect one’s alleged right of possession over
the same.

Remedial Law Review II (CY 2018-2019) – Saturday 1:00pm – 5:00pm: Case Digest Compilation [Atty. Brondial] Page 32
C. Receivership (Rule 59)

C.1. SPS. CESAR A. LARROBIS, JR. and VIRGINIA S. LARROBIS, petitioners, vs.


PHILIPPINE VETERANS BANK, respondent.

G.R. No. 135706; October 1, 2004;AUSTRIA-MARTINEZ, J.:

FACTS: In 1980, spouses Larobis contracted a monetary loan with Philippine Veterans Bank, evidenced
by a promissory note and and secured by a Real Estate Mortgage executed on their lot.
In 1985, Philippine Veterans Bank was placed under receivership/liquidation.

On August 23, 1985, the bank sent the spouses a demand letter for "accounts receivable in the total
amount of ₱6,345.00” which pertains to the insurance premiums advanced by respondent bank over the
mortgaged property of petitioners.

On August 23, 1995, more than fourteen years from the time the loan became due and demandable,
respondent bank filed a petition for extrajudicial foreclosure of mortgage of petitioners’ property.

ISSUE: Whether or not the period within which the bank was placed under receivership and liquidation
was a fortuitous event, which disallowed the bank to do business, thereby suspending the running of the
ten-year prescriptive period.

RULING: No.

One characteristic of a fortuitous event, in a legal sense and consequently in relations to contract, is that
its occurrence must be such as to render it impossible for a party to fulfill his obligation in a normal
manner.

While it is true that foreclosure falls within the broad definition of "doing business," that is:
…a continuity of commercial dealings and arrangements and contemplates to that extent, the
performance of acts or words or the exercise of some of the functions normally incident to and in
progressive prosecution of the purpose and object of its organization.
it should not be considered included, however, in the acts prohibited whenever banks are "prohibited from
doing business" during receivership and liquidation proceedings.

When a bank is prohibited from continuing to do business by the Central Bank and a receiver is appointed
for such bank, that bank would not be able to do new business, i.e., to grant new loans or to
accept new deposits. However, the receiver of the bank is in fact obliged to collect debts owing to the
bank, which debts form part of the assets of the bank. The receiver must assemble the assets and
pay the obligation of the bank under receivership, and take steps to prevent dissipation of such
assets. Accordingly, the receiver of the bank is obliged to collect pre-existing debts due to the bank,
and in connection therewith, to foreclose mortgages securing such debts.

Settled is the principle that a bank is bound by the acts, or failure to act of its receiver.

Remedial Law Review II (CY 2018-2019) – Saturday 1:00pm – 5:00pm: Case Digest Compilation [Atty. Brondial] Page 33
C.2. Mila Caboverde Tantano and Roseller Caboverde, petitioners, v. Dominalda Espina-
Caboverde, Eve Caboverde-Yu, Fe Caboverde-Labrador, and Josephine E.
Caboverde, respondents.

G.R.No. 203585; July 29, 2013; Justice Presbitero Velasco, Jr.

FACTS: Petitioners file a complaint of annulment of the Deed of Sale purportedly transferring lots from
their parents Maximo and Dominalda. During the pendency of the case the parties executed a Partial
Settlement Agreement (PSA) where they fixed the sharing of the uncontroverted properties among
themselves, in particular, the adverted eight (8) parcels of land including their respective products and
improvements. Under the PSA, Dominalda’s daughter, Josephine, shall be appointed as Administrator.
The PSA provided that Dominalda shall be entitled to receive a share of one-half (1/2) of the net income
derived from the uncontroverted properties. The PSA also provided that Josephine shall have special
authority, among others, to provide for the medicine of her mother.

Both Annabelle Saldia and Jesus Tan then took their respective oaths of office and filed a motion to fix
and approve bond which was approved by the trial court over petitioners’ opposition.

Petitioners anchored that the court a quo failed to require Dominalda to post a bond prior to the issuance
of the order appointing a receiver, in violation of Section 2, Rule 59 of the Rules of court

Respondents insist that where there is sufficient cause to appoint a receiver, there is no need for an
applicant’s bond because under Sec. 2 of Rule 59, the very purpose of the bond is to answer for all
damages that may be sustained by a party by reason of the appointment of a receiver in case the applicant
shall have procured such appointment without sufficient cause.

ISSUE: Whether or not posting bond is required in receivership?

RULING: Sec. 2 of Rule 59 is very clear in that before issuing the order appointing a receiver the court
shall require the applicant to file a bond executed to the party against whom the application is presented.
The use of the word "shall" denotes its mandatory nature; thus, the consent of the other party, or as in this
case, the consent of petitioners, is of no moment. Hence, the filing of an applicant’s bond is required at all
times. On the other hand, the requirement of a receiver’s bond rests upon the discretion of the court. Sec.
2 of Rule 59 clearly states that the court may, in its discretion, at any time after the appointment, require
an additional bond as further security for such damages.

Doctrine:
1. Even in cases falling under Sec 1 (d), it is essential that there is a clear showing that there is imminent
danger that the properties sought to be placed under receivership will be lost, wasted or injured.
2. Sec 2 uses the word “shall” thus, it is mandatory. The consent of the parties to place the properties in
receivership is of no moment.

Remedial Law Review II (CY 2018-2019) – Saturday 1:00pm – 5:00pm: Case Digest Compilation [Atty. Brondial] Page 34
C.3. ANA MARIA A. KORUGA vs. TEODORO O. ARCENAS, JR., ALBERT C. AGUIRRE,
CESAR S. PAGUIO, FRANCISCO A. RIVERA, and THE HONORABLE COURT OF APPEALS,
THIRD DIVISION,
G.R. No. 168332; June 19, 2009; NACHURA, J.:

FACTS: Koruga is a minority stockholder of Banco Filipino and on August 20, 2003, she filed a
complaint before the Makati RTC against respondents, alleging the following violations:
10.1 Violation of Sections 31 to 34 of the Corporation Code ("Code") which
prohibit self-dealing and conflicts of interest of directors and officers
10.2 Right of a stockholder to inspect the records of a corporation (including financial
statements) under Sections 74 and 75 of the Code
10.3 Receivership and Creation of a Management Committee

On September 12, 2003, Respondent Arcenas, et al. filed their Answer raising, among others, the trial
court's lack of jurisdiction to take cognizance of the case. They also filed a Manifestation and Motion
seeking the dismissal of the case. In an Order dated October 18, 2004, the trial court denied the
Manifestation and Motion. On February 9, 2005, the CA issued a 60-day TRO enjoining Judge Marella
from conducting further proceedings in the case.

On February 22, 2005, the RTC issued a Notice of Pre-trial setting the case for pre-trial on June 2 and 9,
2005. Arcenas, et al. filed a Manifestation and Motion[10] before the CA, reiterating their application for
a writ of... preliminary injunction. Thus, on April 18, 2005, the CA issued the assailed Resolution, which
reads in part:

Considering that the Temporary Restraining Order issued by this Court on February 9, 2005 expired on
April 10, 2005, it is necessary that a writ of preliminary injunction be issued in order not to render
ineffectual whatever final resolution this Court may render... in this case, after the petitioners shall have
posted a bond

Dissatisfied, Koruga filed this Petition for Certiorari under Rule 65 of the Rules of Court. Koruga alleged
that the CA effectively gave due course to Arcenas, et al.'s petition when it issued a writ of preliminary
injunction without factual or legal basis

Meanwhile, on March 13, 2006, this Court issued a Resolution granting the prayer for a TRO and
enjoining the Presiding Judge of Makati RTC, Branch 138, from proceeding with the hearing of the case
upon the filing by Arcenas, et al. of a P50,000.00 bond.
G.R. No. 169053

In their Petition, Arcenas, et al. asked the Court to set aside the Decision[14] dated July 20, 2005 of the
CA in CA-G.R. SP No. 88422, which denied their petition, having found no grave abuse of discretion on
the part of the Makati RTC. The CA said that... the RTC Orders were interlocutory in nature and, thus,
may be assailed by certiorari or prohibition only when it is shown that the court acted without or in excess
of jurisdiction or with grave abuse of discretion.

ISSUE: Whether the RTC or BSP has jurisdiction over Koruga’s complaint.

RULING: BSP.
We hold that it is the BSP that has jurisdiction over the case. The acts complained of pertain to the
conduct of Banco Filipino's banking business. The law vests in the BSP the supervision over operations
and activities of banks. Specifically, the BSP's supervisory and regulatory powers include: conduct of
examination to determine compliance with laws and regulations if the circumstances so warrant as
determined by the Monetary Board; overseeing to ascertain that laws and Regulations are complied with;
regular investigation which shall not be oftener than once a year from the last date of examination to
determine whether an institution is conducting its business on a safe or sound basis; inquiring into the
solvency and liquidity of the institution.

Correlatively, the General Banking Law of 2000 specifically deals with loans contracted by bank
directors or officers, thus:

SECTION 36. Restriction on Bank Exposure to Directors, Officers, Stockholders and Their Related
Interests.

Remedial Law Review II (CY 2018-2019) – Saturday 1:00pm – 5:00pm: Case Digest Compilation [Atty. Brondial] Page 35
The Monetary Board may regulate the amount of loans, credit accommodations and guarantees that may
be extended, directly or indirectly, by a bank to its directors, officers, stockholders and their related
interests, as well as investments of such bank in enterprises owned or... controlled by said directors,
officers, stockholders and their related interests.
Furthermore, the authority to determine whether a bank is conducting business in an unsafe or unsound
manner is also vested in the Monetary Board.

Finally, the New Central Bank Act grants the Monetary Board the power to impose administrative
sanctions on the erring bank:

Section 37.
the Monetary Board may, at its discretion, impose upon... any bank or quasi-bank, their directors and/or
officers... or any commission of irregularities, and/or conducting business in an unsafe or unsound
manner as may be determined by the Monetary Board
Koruga's invocation of the provisions of the Corporation Code is misplaced. In an earlier case with
similar antecedents, we ruled that:
The Corporation Code, however, is a general law applying to all types of corporations, while the New
Central Bank Act regulates specifically banks and other financial institutions, including the dissolution
and liquidation thereof. As between a general and special... law, the latter shall prevail - generalia
specialibus non derogant.
Consequently, it is not the Interim Rules of Procedure on Intra-Corporate Controversies,[32] or Rule 59
of the Rules of Civil Procedure on Receivership, that would apply to this case. Instead, Sections 29 and
30 of the New Central Bank Act should be... followed
, viz.:

Section 30.
The Monetary Board may summarily and without need for prior... hearing forbid the institution from
doing business in the Philippines and designate the Philippine Deposit Insurance Corporation as receiver
of the banking institution.

Actions of the Monetary Board taken under this section or under Section 29 of this Act shall be final and
executory, and may not be restrained or set aside by the court except on petition for certiorari on the
ground that the action taken was in excess of... jurisdiction or with such grave abuse of discretion as to
amount to lack or excess of jurisdiction.

The appointment of a receiver under this section shall be vested exclusively with the Monetary Board.
On the strength of these provisions, it is the Monetary Board that exercises exclusive jurisdiction over
proceedings for receivership of banks. From the foregoing disquisition, there is no doubt that the RTC has
no jurisdiction to hear and decide a suit that seeks to place Banco Filipino under receivership.
The court's jurisdiction could only have been invoked after the Monetary Board had taken action on the
matter and only on the ground that the action taken was in excess of jurisdiction or with such grave abuse
of discretion as to amount to lack or excess of... jurisdiction.

Remedial Law Review II (CY 2018-2019) – Saturday 1:00pm – 5:00pm: Case Digest Compilation [Atty. Brondial] Page 36
C.4. EVELINA G. CHAVEZ and AIDA CHAVEZ-DELES vs. COURT OFAPPEALS and ATTY.
FIDELA Y. VARGAS

G.R. No. 174356, January 20, 2010

FACTS: Fidela Y. Vargas, Respondent, owned a five-hectare land. Petitioner Evelina had been staying in
a remote portion of the land. Fidela and Evelina agreed to divide the gross sales of all products from the
land between themselves. Since Fidela was busy with her law practice, Evelina undertook to hold in trust
for Fidela her half of the profits. Fidela claimed that Evelina had failed to remit her share of the profits
and, despite demand to turn over the administration of the property to Fidela, had refused to do so.

Consequently, Fidela filed a complaint against Evelina for recovery of possession, rent, and damages with
prayer for the immediate appointment of a Receiver Fidela also filed three estafa cases with the RTC of
Olongapo City and a complaint for dispossession with the Department of Agrarian Reform Adjudication
Board (DARAB) against Evelina and Aida. In all these cases, Fidela asked for the immediate appointment
of a receiver for the property.

ISSUE: Whether the filing of an administrative, civil and criminal cases, all of which with prayer for
appointment of receiver constitutes forum shopping.

RULING: The answer is in the negative. By forum shopping, a party initiates two or more actions in
separate tribunals, grounded on the same cause, trusting that one or the other tribunal would favorably
dispose of the matter. The elements of forum shopping are: (1) identity of parties, or at least such parties
as would represent the same interest in both actions; (2) identity of rights asserted and relief prayed for,
the relief being founded on the same facts; and (3) identity of the two preceding particulars such that any
judgment rendered in the other action will, regardless of which party is successful, amount to res judicata
in the action under consideration.

The above cases are similar only in that they involved the same parties and Fidela sought the placing of
the properties under receivership in all of them. But receivership is not an action. It is but an auxiliary
remedy, a mere incident of the suit to help achieve its purpose. Consequently, it cannot be said that the
grant of receivership in one case will amount to res judicata on the merits of the other cases. The grant or
denial of this provisional remedy will still depend on the need for it in the particular action.

A petition for receivership under Section 1(b), Rule 59 of the Rules of Civil Procedure requires that the
property or fund subject of the action is in danger of being lost, removed, or materially injured,
necessitating its protection or preservation. Its object is the prevention of imminent danger to the
property. If the action does not require such protection or preservation, the remedy is not receivership.
Here Fidela’s main gripe is that Evelina and Aida deprived her of her share of the land’s produce. Fidela
does not claim that the land has been materially injured, necessitating its protection and preservation.
Hence, receiver may not be appointed.

Remedial Law Review II (CY 2018-2019) – Saturday 1:00pm – 5:00pm: Case Digest Compilation [Atty. Brondial] Page 37
D. Replevin (Rule 60)

D.1.JOSE S. OROSA and MARTHA P. OROSA, vs. HON. COURT OF APPEALS, FCP CREDIT
CORPORATION

G.R. No. 111080.; April 5, 2000; YNARES_SANTIAGO, J.:

FACTS: On December 6, 1984, private respondent FCP Credit Corporation filed a complaint for replevin
and damages in the Regional Trial Court of Manila against petitioner Jose S. Orosa and one John Doe to
recover possession of a 1983 Ford Laser 1.5 Sedan.

The petitioner purchased the subject motor vehicle on installment from Fiesta Motor Sales Corporation.
To secure payment, petitioner executed a chattel mortgage over the subject motor vehicle in favor of
Fiesta Motor Sales Corp. On September 28, 1983, Fiesta Motor Sales assigned the promissory note and
chattel mortgage to private respondent FCP Credit Corporation. The petitioner failed to pay 4 installments
which led to FCP Credit Corporation to demand from petitioner payment of the entire outstanding
balance of the obligation with accrued interest and to surrender the vehicle which petitioner was allegedly
detaining. 

Ruling on petitioner's counterclaim, the trial court stated that there was no legal or factual basis for the
writ of replevin and that its enforcement by the sheriff was "highly irregular, and unlawful, done, as it
was, under shades of extortion, threats and force." 

The trial court ordered private respondent to pay moral damages; exemplary damages and  attorney's fees
and to return to petitioner the 1983 Ford Laser 1.5 Sedan, or its equivalent, in kind or value in cash, as of
date of judgment and to pay the costs of the suit.

On June 7, 1988, a "Supplemental Decision" was rendered by the trial court ordering private respondent's
surety, Stronghold Insurance Co., Inc. to jointly and severally [with private respondent] return to
petitioner the 1983 Ford Laser 1.5 Sedan or its equivalent in kind or in cash and to pay the damages
specified in the main decision to the extent of the value of the replevin bond.

ISSUE: Whether or not it was proper for the respondent to return the subject car to petitioner.

RULING: We also agree with the Court of Appeals that the trial court erred when it ordered private
respondent to return the subject car or its equivalent considering that petitioner had not yet fully paid the
purchase price. Verily, to sustain the trial court's decision would amount to unjust enrichment. The Court
of Appeals was correct when it instead ordered private respondent to return, not the car itself, but only the
amount equivalent to the fourteen installments actually paid with interest.

Remedial Law Review II (CY 2018-2019) – Saturday 1:00pm – 5:00pm: Case Digest Compilation [Atty. Brondial] Page 38
D.2. Smart Communications, Inc. vs. Regina M. Astorga,

GR No. 148132; January 28, 2008; Nachura, J.

Facts: Regina Astorga was employed by Smart Communications District Sales Manager of the Corporate
Sales Marketing Group/ Fixed Services Division (CSMG/FSD). She enjoyed benefits including a car
plan. SMART then launched an organizational realignment to achieve more efficient operation which
includes the outsourcing of the marketing and sales force. Thus, SMART entered into a joint venture
agreement with NTT of Japan, and formed SNMI. Since SNMI was formed to do the sales and marketing
work, SMART abolished the CSMG/FSD division.
 
To soften the blow of the realignment, SNMI agreed to absorb the CSMG personnel who would be
recommended by SMART. SMART then conducted a performance evaluation of CSMG personnel and
those who garnered the highest ratings were favorably recommended to SNMI. Astorga landed last in the
performance evaluation, thus, she was not recommended by SMART. SMART, nonetheless, offered her a
supervisory position in the Customer Care Department, but she refused the offer because the position
carried lower salary rank and rate. Despite the abolition of the CSMG/FSD, Astorga continued reporting
for work. But on March 3, 1998, SMART issued a memorandum advising Astorga of the termination of
her employment on ground of redundancy, effective April 3, 1998. Astorga received it on March 16,
1998.
 
The termination of her employment prompted Astorga to file a Complaint for illegal dismissal, non-
payment of salaries and other benefits with prayer for moral and exemplary damages against SMART.
Later, SMART sent a letter to Astorga demanding that she pay the current market value of the Honda
Civic Sedan which was given to her under the company’s car plan program, or to surrender the same to
the company for proper disposition. Astorga failed and refused to do either, thus prompting SMART to
file a suit for replevin with the Regional Trial Court of Makati. Astorga moved to dismiss the complaint
on the ground the regular courts have no jurisdiction over the complaint because the subject thereof
pertains to a benefit arising from an employment contract; hence, jurisdiction over the same is vested in
the labor tribunal and not in regular courts.

The Arbiter ruled that the termination was illegal. Subsequently, the RTC denied Astorga’s MTD. The
NLRC and the CA ruled that the termination was valid. The CA, in the replevin case, reversed the
decision of the RTC and granted the MTD.

Issue: Whether the RTC has jurisdiction over the replevin case for the recovery of the car which Astorga
acquired as part of her employment case.

RULING: Yes. SMARTs demand for payment of the market value of the car or, in the alternative, the
surrender of the car, is not a labor, but a civil, dispute. It involves the relationship of debtor and creditor
rather than employee-employer relations. As such, the dispute falls within the jurisdiction of the regular
courts.
 
Replevin is a possessory action, the gist of which is the right of possession in the plaintiff. The primary
relief sought therein is the return of the property in specie wrongfully detained by another person. It is an
ordinary statutory proceeding to adjudicate rights to the title or possession of personal property. The
question of whether or not a party has the right of possession over the property involved and if so,
whether or not the adverse party has wrongfully taken and detained said property as to require its return
to plaintiff, is outside the pale of competence of a labor tribunal and beyond the field of specialization of
Labor Arbiters.
  
The labor dispute involved is not intertwined with the issue in the Replevin Case. The respective issues
raised in each forum can be resolved independently on the other. In thus ruling, this Court is not
sanctioning split jurisdiction but defining avenues of jurisdiction as laid down by pertinent laws.

Remedial Law Review II (CY 2018-2019) – Saturday 1:00pm – 5:00pm: Case Digest Compilation [Atty. Brondial] Page 39
D.3.KENNETH HAO vs. ABE C. ANDRES, Sheriff IV, RTC, Branch 16, Davao City

A.M. No. P-07-2384; June 18, 2008; QUISUMBING, J.:

FACTS: Hao is one of the defendants in a civil case for replevin pending before the RTC of Davao City,
Branch 16. On October 17, 2005, Judge Renato A. Fuentes issued an Order of Seizure against 22 motor
vehicles allegedly owned by the complainant. On the strength of the said order, Andres was able to seize
nine motor vehicles.

Hao alleged that Andres gave undue advantage to Zenaida Silver in the implementation of the order and
that Andres seized the nine motor vehicles in an oppressive manner. Hao also averred that Andres was
accompanied by unidentified armed personnel on board a military vehicle which was excessive since
there were no resistance from them. Hao also discovered that the compound where the seized motor
vehicles were placed is actually owned by Silver.
In view of the approval of the complainant’s counter-replevin bond, Judge Carpio ordered Andres to
immediately cease and desist from further implementing the order of seizure, and to return the seized
motor vehicles including its accessories to their lawful owners.

However, on October 24, 2005, eight of the nine seized motor vehicles were reported missing. In his
report, Andres stated that he was shocked to find that the motor vehicles were already missing when he
inspected it on October 22, 2005. He narrated that on October 21, 2005, PO3 Rodrigo Despe, one of the
policemen guarding the subject motor vehicles, reported to him that a certain "Nonoy" entered the
compound and caused the duplication of the vehicles’ keys. 11 But Andres claimed the motor vehicles were
still intact when he inspected it on October 21, 2005.

After the OCA recommended that the matter be investigated, we the case was referred to Executive Judge
Renato A. Fuentes for investigation, report and recommendation.
In his Investigation Report17 dated September 21, 2006, Judge Fuentes found Andres guilty of serious
negligence in the custody of the nine motor vehicles. He recommended that Andres be suspended from
office.

The OCA disagreed with the observations of Judge Fuentes. It recommended that Andres be held liable
only for simple neglect of duty and be suspended for one (1) month and one (1) day.

ISSUE: Whether Sheriff Andres complied with his duties under the writ of replevin?

RULING: No. First, the rules provide that property seized under a writ of replevin is not to be delivered
immediately to the plaintiff. In accordance with the said rules, Andres should have waited no less than
five days in order to give the complainant an opportunity to object to the sufficiency of the bond or of the
surety or sureties thereon, or require the return of the seized motor vehicles by filing a counter-bond.
This, he failed to do.

Records show that Andres took possession of two of the subject motor vehicles on October 17, 2005, four
on October 18, 2005, and another three on October 19, 2005. Simultaneously, as evidenced by the
depository receipts, on October 18, 2005, Silver received from Andres six of the seized motor vehicles,
and three more motor vehicles on October 19, 2005. Consequently, there is no question that Silver was
already in possession of the nine seized vehicles immediately after seizure, or no more than three days
after the taking of the vehicles. Thus, Andres committed a clear violation of Section 6, Rule 60 of the
Rules of Court with regard to the proper disposal of the property.

It matters not that Silver was in possession of the seized vehicles merely for safekeeping as stated in the
depository receipts. The rule is clear that the property seized should not be immediately delivered to the
plaintiff, and the sheriff must retain custody of the seized property for at least five days. 23 Hence, the act
of Andres in delivering the seized vehicles immediately after seizure to Silver for whatever purpose,
without observing the five-day requirement finds no legal justification.

When a writ is placed in the hands of a sheriff, it is his duty, in the absence of any instructions to the
contrary, to proceed with reasonable celerity and promptness to execute it according to its mandate.
However, the prompt implementation of an order of seizure is called for only in instances where there is
no question regarding the right of the plaintiff to the property. 38 Where there is such a question, the
prudent recourse for Andres is to desist from executing the order and convey the information to his judge
and to the plaintiff.

Remedial Law Review II (CY 2018-2019) – Saturday 1:00pm – 5:00pm: Case Digest Compilation [Atty. Brondial] Page 40
WHEREFORE, the Court finds Abe C. Andres, Sheriff IV, RTC of Davao City, Branch 16, GUILTY of
gross neglect of duty and grave abuse of authority (oppression) and is SUSPENDED for one (1) year and
six (6) months without pay. He is also hereby WARNED that a repetition of the same or similar offenses
in the future shall be dealt with more severely.

Remedial Law Review II (CY 2018-2019) – Saturday 1:00pm – 5:00pm: Case Digest Compilation [Atty. Brondial] Page 41
D.4. Navarro vs. Escobido,

GR 153788, November 27, 2009


(Provisional Remedies: Replevin: Prior demand is not a condition precedent)

Facts: Private respondent (Karen Go) files a complaint with a prayer for the issuance of a writ of replevin
against petitioner (Navarro) for the seizure of 2 motor vehicles under lease agreement. Petitioner
maintains among others in the case at bar that the complaints were premature because no prior demand
was made on him to comply with the provisions of the lease agreements before the complaints for
replevin were filed.

ISSUE: WON prior demand is a condition precedent to an action for a writ of replevin.

RULING: No. Petitioner erred in arguing that prior demand is required before an action for a writ of
replevin is filed since we cannot liken a replevin action to an unlawful detainer.

For a writ of replevin to issue, all that the applicant must do is to file an affidavit and bond, pursuant to
Section 2, Rule 60 of the Rules, which states:

Sec. 2. Affidavit and bond.


The applicant must show by his own affidavit or that of some other person who personally knows the
facts:

(a) That the applicant is the owner of the property claimed, particularly describing it, or is entitled to
the possession thereof;
(b) That the property is wrongfully detained by the adverse party, alleging the cause of detention
thereof according to the best of his knowledge, information, and belief;
(c) That the property has not been distrained or taken for a tax assessment or a fine pursuant to law, or
seized under a writ of execution or preliminary attachment, or otherwise placed under custodialegis, or if
so seized, that it is exempt from such seizure or custody; and
(d) The actual market value of the property.

The applicant must also give a bond, executed to the adverse party in double the value of the property as
stated in the affidavit aforementioned, for the return of the property to the adverse party if such return be
adjudged, and for the payment to the adverse party of such sum as he may recover from the applicant in
the action.

The SC held that there is nothing in the afore-quoted provision which requires the applicant to make a
prior demand on the possessor of the property before he can file an action for a writ of replevin. Thus,
prior demand is not a condition precedent to an action for a writ of replevin.

Remedial Law Review II (CY 2018-2019) – Saturday 1:00pm – 5:00pm: Case Digest Compilation [Atty. Brondial] Page 42
D.5. SPOUSES DEO AGNER and MARICON AGNER,Petitioners, vs.BPI FAMILY SAVINGS
BANK, INC.,Respondent.

G.R. No. 182963; June 3, 2013; PERALTA, J.:

FACTS: On February 15, 2001, Deo Agner and Maricon Agner executed a Promissory Note with Chattel
Mortgage in favor of Citimotors, Inc. The contract provides, among others, that: for receiving the amount
of Php834, 768.00, petitioners shall pay Php 17,391.00 every 15th day of each succeeding month until
fully paid; the loan is secured by a 2001 Mitsubishi Adventure Super Sport; and an interest of 6% per
month shall be imposed for failure to pay each installment on or before the stated due date.

On the same day, Citimotors, Inc. assigned all its rights, title and interests in the Promissory Note with
Chattel Mortgage to ABN AMRO Savings Bank, Inc. (ABN AMRO), which, on May 31, 2002, likewise
assigned the same to respondent BPI Family Savings Bank, Inc.

For failure to pay four successive installments from May 15, 2002 to August 15, 2002, respondent,
through counsel, sent to petitioners a demand letter dated August 29, 2002, declaring the entire obligation
as due and demandable and requiring to pay Php576,664.04, or surrender the mortgaged vehicle
immediately upon receiving the letter. 6 As the demand was left unheeded, respondent filed on October 4,
2002 an action for Replevin and Damages before the Manila Regional Trial Court (RTC).
A writ of replevin was issued.7 Despite this, the subject vehicle was not seized. 8 Trial on the merits
ensued. On August 11, 2005, the Manila RTC Br. 33 ruled for the respondent and ordered petitioners to
jointly and severally pay the amount of Php576,664.04 plus interest at the rate of 72% per annum from
August 20, 2002 until fully paid, and the costs of suit.

Petitioners appealed the decision to the Court of Appeals (CA), but the CA affirmed the lower court’s
decision and, subsequently, denied the motion for reconsideration; hence, this petition.

ISSUE: Whether or not respondent’s remedy of resorting to both actions of replevin and collection of
sum of money is contrary to the provision of Article 1484 of the Civil Code and the Elisco Tool
Manufacturing Corporation v. Court of Appeals ruling

RULING: Compared with Elisco, the vehicle subject matter of this case was never recovered and
delivered to respondent despite the issuance of a writ of replevin. As there was no seizure that transpired,
it cannot be said that petitioners were deprived of the use and enjoyment of the mortgaged vehicle or that
respondent pursued, commenced or concluded its actual foreclosure. The trial court, therefore, rightfully
granted the alternative prayer for sum of money, which is equivalent to the remedy of "exacting
fulfillment of the obligation." Certainly, there is no double recovery or unjust enrichment to speak
of.

The remedies provided for in Art. 1484 are alternative, not cumulative. The exercise of one bars the
exercise of the others.

Art. 1484. In a contract of sale of personal property the price of which is payable in installments,
the vendor may exercise any of the following remedies:
(1) Exact fulfillment of the obligation, should the vendee fail to pay;
(2) Cancel the sale, should the vendee's failure to pay cover two or more installments;
(3) Foreclose the chattel mortgage on the thing sold, if one has been constituted, should the vendee's
failure to pay cover two or more installments. In this case, he shall have no further action against
the purchaser to recover any unpaid balance of the price. Any agreement to the contrary shall be
void

Remedial Law Review II (CY 2018-2019) – Saturday 1:00pm – 5:00pm: Case Digest Compilation [Atty. Brondial] Page 43
E. Support (Rule 61)

E.1. De Asis vs. Court of Appeals

303 SCRA 176; February 15, 1999; PURISIMA, J.

FACTS: Vircel D. Andres, (herein private respondent), in her capacity as the legal guardian of the
Minor, Glen Camil Andres de Asis, brought an action for maintenance and support against against
Manuel de Asis (petitioner) before the Regional Trial Court of Quezon City, alleging that petitioner is the
father of subject minor, and the former refused and/or failed to provide for the maintenance of the latter,
despite repeated demands.

In his answer, petitioner denied his paternity of the said minor alleged and that he cannot be required to
provide support for him. Subsequently, private respondent sent in a manifestation stating that because of
petitioner's judicial declarations, it was futile and a useless exercise to claim support from him. Hence,
she was withdrawing her complaint against petitioner subject to the condition that the latter should not
pursue his counterclaim. By virtue of the said manifestation, the parties mutually agreed to move for the
dismissal of the complaint.

The motion was granted by the trial court, which then dismissed the case with prejudice. Subsequently,
another Complaint for maintenance and support was brought against petitioner, this time in the name of
Glen Camil Andres de Asis, represented by her legal guardian, herein private respondent. Petitioner
moved to dismiss the complaint on the ground of res judicata. The trial court denied the motion, ruling
that res judicata is inapplicable in an action for support for the reason that renunciation or waiver of
future support is prohibited by law. The trial court likewise denied petitioner's motion for reconsideration.
Petitioner filed with the Court of Appeals a petition for certiorari. The Court of Appeals dismissed the
same. Hence, this petition.

Petitioner contends that the aforecited manifestation, in effect, admitted the lack of filiation between him
and the minor child, which admission binds the complainant, and since the obligation to give support is
based on the existence of paternity and filiation between the child and the putative parent, the lack thereof
negates the right to claim for support. Thus, petitioner maintains that the dismissal of the Complaint by
the lower court on the basis of the said manifestation bars the present action for support, especially so
because the order of the trial court explicitly stated that the dismissal of the case was with prejudice.

ISSUE: Whether or not the CA erred in denying petitioner's Motion to Dismiss on the ground of res
judicata.

RULING: No. Petition dismissed and the decision of the Court of Appeals was affirmed.
The right to receive support can neither be renounced nor transmitted to a third person. Furthermore,
future support cannot be the subject of a compromise. The manifestation sent by private respondent
amounted to renunciation as it severed the vinculum that gives the subject minor, the right to claim
support from his putative parent, the petitioner. Furthermore, the agreement entered into between the
petitioner and private respondent for the dismissal of the counterclaim was in the nature of a compromise,
which cannot be countenanced. It violated the prohibition against any compromise of the right to support.
Moreover, it is true that that in order to claim support, filiation and/or paternity must first be shown
between the claimant and the parent. However, paternity and filiation or the lack of the same is a
relationship that must be judicially established and it is for the court to declare its existence or absence. It
cannot be left to the will or agreement of the parties. Hence, the Supreme Court ruled that the dismissal
with prejudice of the first case cannot bar the subsequent case for support. The second action for support
may still prosper.

Remedial Law Review II (CY 2018-2019) – Saturday 1:00pm – 5:00pm: Case Digest Compilation [Atty. Brondial] Page 44
E.2. PEOPLE OF THE PHILIPPINES, plaintiff-appellee, vs. MANUEL MANAHAN, alias Maning,
defendant-appellant.

G.R. No. 128157; September 29, 1999; BELLOSILLO, J.:

FACTS: Teresita Tibigar, 16 years old, is a stay-in waitress at Espiritu Canteen in Dagupan City while
Manuel Manahan, alias “Maning”, is the brother-in-law of the owner of the canteen. Manuel and his
pregnant wife, Primadonna, temporarily reside at the canteen together with the family of Josefina. On 5
January 1995, at about two o’clock in the morning, Teresita who was asleep was suddenly awakened
when she felt someone beside her. Upon opening her eyes she saw accused Manuel Manahan as he
immediately placed himself on top of her and successfully had forceful carnal knowledge of her despite
her resistance. Within the month Teresita left the canteen and returned home in Pangasinan. The sexual
encounter resulted in her pregnancy. When her parents discovered it and learned the story they filed a
case of rape against “Maning”. On 2 October 1995, she gave birth to a healthy baby girl and christened
her Melanie Tibigar.

On trial, Maning interposed the “sweetheart doctrine” but the trial court finds that there was no substantial
evidence, e.g., love notes, mementos or pictures, presented to support it. Hence, Maning was found guilty
of rape and sentenced to death by court a quo. Hewas also ordered to indemnify the victim P50,000.00 as
moral damages, pay the costs, and acknowledge and support the offspring of his indiscretion. This case is
now before us on automatic review.

ISSUE: Whether or not the order of court a quo to acknowledge and support the offspring resulting from
his crime is valid?

RULING: NO. Article 345 of The Revised Penal Code provides that persons guilty of rape shall also be
sentenced to "acknowledge the offspring, unless the law should prevent him from doing so," and "in every
case to support the offspring." In the case before us, compulsory acknowledgment of the child Melanie
Tibigar is not proper there being a legal impediment in doing so as it appears that the accused is a married
man. As pronounced by this Court in People v. Guerrero, "the rule is that if the rapist is a married man,
he cannot be compelled to recognize the offspring of the crime, should there be any, as his child, whether
legitimate or illegitimate."

Consequently, that portion of the judgment under review is accordingly deleted. In any case, we sustain
that part ordering the accused to support the child as it is in accordance with law.

DOCTRINE: Although support may be imposed against a married man, compulsory acknowledgment of
child is not proper for being contrary to law and jurisprudence.
In People v. Guerrero, "the rule is that if the rapist is a married man, he cannot be compelled to
recognize the offspring of the crime, should there be any, as his child, whether legitimate or illegitimate."

Remedial Law Review II (CY 2018-2019) – Saturday 1:00pm – 5:00pm: Case Digest Compilation [Atty. Brondial] Page 45
E.3. SPOUSES PRUDENCIO and FILOMENA LIM vs. MA. CHERYL S. LIM

G.R. No. 163209; October 30, 2009

FACTS: Respondent, Cheryl S. Lim married to Edward Lim. They have three children. Thefamily
resided at the house of petitioners, Edward’s parents in Forbes Park, MakatiCity. On 14 October 1990,
Cheryl abandoned the Forbes Park residence, bringingthe children with her (then all minors), after a
violent confrontation with Edward.Cheryl, for herself and her children, sued petitioners, Edward, Chua
Giak andMariano (petitioners) in the Regional Trial Court of Makati City, Branch 140 (trialcourt) for
support.

Relying on provisions found in Title IX of the Civil Code, as amended, on ParentalAuthority, petitioners
theorize that their liability is activated only upon default ofparental authority, conceivably either by its
termination or suspension during thechildren’s minority. Because at the time respondents sued for
support, Cheryl andEdward exercised parental authority over their children, petitioners submit that
theobligation to support the latter’s offspring ends with them.

ISSUE: Whether the grandparents may be sued for support of their grandchildren and daughter-in-law.

RULING: Petitioners Liable to Provide Support but only to their Grandchildren. By statutoryand
jurisprudential mandate, the liability of ascendants to provide legal support totheir descendants is beyond
cavil.Although the obligation to provide support arising from parental authority endsupon the
emancipation of the child, the same obligation arising from spousal andgeneral familial ties ideally lasts
during the obligee's lifetime. Also, while parentalauthority under Title IX (and the correlative parental
rights) pertains to parents,passing to ascendants only upon its termination or suspension, the obligation to
provide legal support passes on to ascendants not only upon default of the parents but also for the latter’s
inability to provide sufficient support.

Remedial Law Review II (CY 2018-2019) – Saturday 1:00pm – 5:00pm: Case Digest Compilation [Atty. Brondial] Page 46
E.4. CHARLES GOTARDO vs. DIVINA BULING

G.R. No. 165166; August 15, 2012

FACTS: Divina Buling filed a complaint for compulsory recognition and support pendente lite for her
son, claiming that the petitioner is the father of Gliffze. During thependency of the case, the RTC, on the
respondent’s motion, granted a P2,000.00 monthly child support, retroactive from March 1995.RTC
dismissed the complaint for insufficiency of evidence proving Gliffze’sfiliation. CA consequently set
aside the RTC decision and ordered the petitioner torecognize his minor son Gliffze. It also reinstated the
RTC order granting a P2,000.00 monthly child support.

ISSUE: Whether the amount in the order granting support is fixed and may notbe changed.

RULING: The answer is in the negative. Support consists of everything indispensable forsustenance,
dwelling, clothing, medical attendance, education and transportation, inkeeping with the financial
capacity of the family. Thus, the amount of support isvariable and, for this reason, no final judgment
on the amount of support ismade as the amount shall be in proportion to the resources or means of
thegiver and the necessities of the recipient. It may be reduced or increasedproportionately according to
the reduction or increase of the necessities of therecipient and the resources or means of the person
obliged to support.

Remedial Law Review II (CY 2018-2019) – Saturday 1:00pm – 5:00pm: Case Digest Compilation [Atty. Brondial] Page 47
E.5 SUSAN LIM-LUA VS. DANILO LUA

G.R. No. 175279-80; June 5, 2013; Villarama Jr., J:

FACTS: On September 3, 2003, petitioner Susan Lua, filed an action for the declaration of nullity of
marriage with respondent Danilo Lua. In her prayer for support pendente lite for herself and her two
children, petitioner sought the amount ₱500,000.00 as a monthly support, citing respondent’s huge
earnings from salaries and dividends in several companies and businesses here and abroad. After due
hearing, Judge Yrastorza issued an order dated March 31, 2004 granting support pendent lite as follows
that the amount of Two Hundred Fifty Thousand Pesos would be sufficient to take care of the needs of the
plaintiff this excludes the One hundred Thirty-Five Thousand Pesos for medical attendance expenses
needed by the plaintiff for the operation of both her eyes. Respondent filed a motion for reconsideration
asserting that petitioner is not entitled to spousal support and the ₱250,000.00 monthly support is
unconscionable. However, the March 13 decision had become final and executory.

Respondent then filed a Petition for Certiorari in the Court of Appeals and on its April 12, 2005, the CA
rendered its decision finding merit in the respondent’s contention that the trial court gravely abused its
discretion in granting ₱250,000.00 monthly support to petitioner without evidence to prove his actual
income. The parties did not appeal the decision of the court. In compliance, respondent attached a copy of
a check he issued in the amount of ₱162,651.90 payable to petitioner. Petitioner explained that, as
decreed in the CA decision, he deducted from the amount of support in arrears totaling to 2,645,000.00,
the advances given by him to his children and petitioner in the sum of 2,482,348.16. In her comment,
petitioner asserted that none of the deductions by the respondent may be chargeable as part of the monthly
support contemplated by the CA.

The CA granted the petition of respondent ordering the deductions prayed for by the petitioner. Thus, the
petitioner raised the question in this court.

ISSUE: Whether or not certain expenses already incurred by the respondent may be deducted from the
total support in arrears owing to petitioner and her children pursuant to the Decision of the CA.

RULING: No. the general rule is to the effect that when a father is required by a divorce decree to pay to
the mother money for the support of their dependent children and the unpaid and accrued installments
become judgment in her favor, he cannot, as a matter of law, claim credit on account of payments
voluntarily made directly to the children. However, special considerations of an equitable nature may
justify a court in crediting such payment on his indebtedness to the mother, when that can be done
without injustice to her.

Here, the CA should not have allowed all expenses incurred by respondent to be credited against the
accrued support pendente lite. As earlier mentioned, the monthly support pendente lite granted by the trial
court was intended primarily for food, household expenses.

DOCTRINE: In the absence of some finding of consent by the mother, most courts refuse to allow a
husband to dictate how he will meet the requirements for support payments when the mode of payment is
fixed by a decree of Court.

Remedial Law Review II (CY 2018-2019) – Saturday 1:00pm – 5:00pm: Case Digest Compilation [Atty. Brondial] Page 48
E.6. REPUBLIC OF THE PHILIPPINES, represented by the Armed Forces of the Philippines
Finance Center (AFPFC), Petitioner, vs. DAISY R. YAHON, Respondent.

G.R. No. 201043; June 16, 2014;Villarama, JR., J.

FACTS: Sgt. Charlse A. Yahon, an enlisted personnel of the Philippine Army, was married to Daisy R.
Yahon, respondent. Daisy R. Yahon filed a petition for the issuance of protection order under the
provisions of Republic Act (R.A.) No. 9262, otherwise known as the "Anti-Violence Against Women and
Their Children Act of 2004," against her husband, Sgt. Yahon.

A TPO has been issued against Sgt Yahon to protect the respondent from further abuses. In the TPO, Sgt
Yahon was ordered to provide reasonable financial spousal support to the respondent in the amount of
₱4,000.00 per month and fifty percent (50%) of his retirement benefits which shall be automatically
deducted and given directly to her. In his failure to appear before the court with a counsel and with an
answer to the charges against him, the RTC allowed the ex-parte presentation of evidence to determine
the necessity of issuance of a Permanent Protection Order (PPO). The court has granted PPO.

A copy of the decision was sent to the Commanding General/Officer of Finance Center of the Armed
Forces of the Philippines, Camp Emilio Aguinaldo, Quezon City; the Management of RSBS, Camp
Emilio Aguinaldo, Quezon City and the Regional Manager of PAG-IBIG, Mortola St., Cagayan de Oro
City for their guidance and strict compliance.

Petitioner filed before the RTC a Manifestation and Motion To Lift Temporary Protection Order Against
the AFP dated November 10, 2008.  Stating, the RTC had not acquired jurisdiction over the military
institution due to lack of summons, and hence they cannot be bound by the said court order. The court
denied the motion as it was filed out of time. Petitioner’s motion for reconsideration was likewise denied
by the RTC.

Petitioner filed a petition for certiorari before the CA praying for the nullification of the aforesaid orders
and decision insofar as it directs the AFPFC to automatically deduct from S/Sgt. Yahon’s retirement and
pension benefits and directly give the same to respondent as spousal support. Respondent, in her
Comment with Prayer for Issuance of Preliminary Injunction, manifested that there is no information as to
whether S/Sgt. Yahon already received his retirement benefit and that the latter has repeatedly violated
the TPO, particularly on the provision of spousal support.

CA issued a writ of preliminary injunction, enjoining the Armed Forces of the Philippines Finance
Center, its employees, agents, representatives, and any all persons acting on its behalf, from releasing the
remaining pension that may be due to S/Sgt. Charles A. Yahon.

The CA denied the petition for certiorari and affirmed the assailed orders and decision of the RTC. The
CA likewise denied petitioner’s motion for reconsideration. Hence this present petition for review on
certiorari.

Petitioner argues that it cannot comply with the RTC’s directive for the automatic deduction of 50% from
S/Sgt. Yahon’s retirement benefits and pension to be given directly to respondent, as it contravenes an
explicit mandate under the law governing the retirement and separation of military personnel.
(Presidential Decree (P.D.) No. 1638,Section 31 & R.A. No. 8291, Sec 39)

Petitioner further contends that the directive under the TPO to segregate a portion of S/Sgt. Yahon’s
retirement benefits was illegal because said moneys remain as public funds and that money due to
government employees is not liable to the creditors of the said employees in the process of garnishment,
citing the case of Pacific Products v. Ong

ISSUE: Whether petitioner military institution may be ordered to automatically deduct a percentage from
the retirement benefits of its enlisted personnel, and to give the same directly to the latter’s lawful wife as
spousal support in compliance with a protection order issued by the RTC pursuant to R.A. No. 9262.

RULING: Yes, Section 8(g) of R.A. No. 9262 used the general term "employer," which includes in its
coverage the military institution, S/Sgt. Yahon’s employer. Where the law does not distinguish, courts
should not distinguish. Thus, Section 8(g) applies to all employers, whether private or government.

Remedial Law Review II (CY 2018-2019) – Saturday 1:00pm – 5:00pm: Case Digest Compilation [Atty. Brondial] Page 49
It bears stressing that Section 8(g) providing for spousal and child support, is a support enforcement
legislation. In the United States, provisions of the Child Support Enforcement Act24 allow garnishment
of certain federal funds where the intended recipient has failed to satisfy a legal obligation of child
support. As these provisions were designed "to avoid sovereign immunity problems" and provide that
"moneys payable by the Government to any individual are subject to child support enforcement
proceedings," the law is clearly intended to "create a limited waiver of sovereign immunity so that state
courts could issue valid orders directed against Government agencies attaching funds in their possession."

The relief provided in Section 8(g) thus fulfills the objective of restoring the dignity of women who are
victims of domestic violence and provide them continued protection against threats to their personal
safety and security.

DOCTRINE: Support; Section 8(g) of R.A. No. 9262 Section 8(g) applies to all employers, whether
private or government.It bears stressing that Section 8(g) providing for spousal and child support, is a
support enforcement legislation.

Remedial Law Review II (CY 2018-2019) – Saturday 1:00pm – 5:00pm: Case Digest Compilation [Atty. Brondial] Page 50
E.7.Narciso Salas v. Annabelle Matusalem

G.R. No.180284; September 11, 2013; Villarama, Jr., J.:

FACTS: On May 26, 1995, Annabelle Matusalem (respondent) filed a complaint for Support/Damages
against Narciso Salas (petitioner) in the Regional Trial Court (RTC) of Cabanatuan City (Civil Case No.
2124-AF).

Respondent claimed that petitioner is the father of her son Christian Paulo Salas who was born on
December 28, 1994. Petitioner, already 56 years old at the time, enticed her as she was then only 24 years
old, making her believe that he is a widower. Petitioner rented an apartment where respondent stayed and
shouldered all expenses in the delivery of their child, including the cost of caesarian operation and
hospital confinement. However, when respondent refused the offer of petitioner’s family to take the child
from her, petitioner abandoned respondent and her child and left them to the mercy of relatives and
friends. Respondent further alleged that she attempted suicide due to depression but still petitioner refused
to support her and their child. The testimonial evidence of the owner of the apartment where petitioner
allegedly housed respondent when she was pregnant was one of the basis for respondent’s claim for
support. The RTC ruled in favour of respondent to which the CA affirmed on appeal.

ISSUE: Whether the trial and appellate courts erred in ruling that respondent’s evidence (the testimonial
evidence of the owner of apartment, the Baptismal Certificate, the Birth Certificate which however does
not contain the necessary signature of petitioner) sufficiently proved that her son Christian Paulo is the
illegitimate child of petitioner.

RULING: Respondent presented the Certificate of Live Birth of Christian Paulo Salas in which the name
of petitioner appears as his father but which is not signed by him. Admittedly, it was only respondent who
filled up the entries and signed the said document though she claims it was petitioner who supplied the
information she wrote therein.

We have held that a certificate of live birth purportedly identifying the putative father is not competent
evidence of paternity when there is no showing that the putative father had a hand in the preparation of
the certificate.Thus, if the father did not sign in the birth certificate, the placing of his name by the
mother, doctor, registrar, or other person is incompetent evidence of paternity.Neither can such birth
certificate be taken as a recognition in a public instrument and it has no probative value to establish
filiation to the alleged father.

As to the Baptismal Certificate of Christian Paulo Salas also indicating petitioner as the father, we have
ruled that while baptismal certificates may be considered public documents, they can only serve as
evidence of the administration of the sacraments on the dates so specified. They are not necessarily
competent evidence of the veracity of entries therein with respect to the child’s paternity.

The rest of respondent’s documentary evidence consists of handwritten notes and letters, hospital bill and
photographs taken of petitioner and respondent inside their rented apartment unit. Pictures taken of the
mother and her child together with the alleged father are inconclusive evidence to prove
paternity. Exhibits “E” and “F” showing petitioner and respondent inside the rented apartment unit thus
have scant evidentiary value. The Statement of Account from the Good Samaritan General Hospital
where respondent herself was indicated as the payee is likewise incompetent to prove that petitioner is the
father of her child notwithstanding petitioner’s admission in his answer that he shouldered the expenses in
the delivery of respondent’s child as an act of charity.

As to the handwritten notes of petitioner and respondent showing their exchange of affectionate words
and romantic trysts, these, too, are not sufficient to establish Christian Paulo’s filiation to petitioner as
they were not signed by petitioner and contained no statement of admission by petitioner that he is the
father of said child. Thus, even if these notes were authentic, they do not qualify under Article 172 (2)
vis-à- vis Article 175 of the Family Code which admits as competent evidence of illegitimate filiation an
admission of filiation in a private handwritten instrument signed by the parent concerned.

DOCTRINE: High standard of proof is required to establish paternity and filiation. An order for
recognition and support may create an unwholesome situation or maybe an irritant to the family or the
lives of the parties so that it must be issued only if paternity or filiation is established by clear and
convincing evidence. Moreover, the death of the putative father is not a bar to the action commenced
during his lifetime by one claiming to be his illegitimate child.

Remedial Law Review II (CY 2018-2019) – Saturday 1:00pm – 5:00pm: Case Digest Compilation [Atty. Brondial] Page 51
E.8. NORMA A. DEL SOCORRO, FOR AND IN BEHALF OF HER MINOR CHILD
RODERIGO NORJO VAN WILSEM v. ERNST JOHAN BRINKMAN VAN WILSEM

G.R. No. 193707; December 10, 2014; THIRD DIVISION, (Peralta, J.)

Foreign law should not be applied when its application would work undeniable injustice to the citizens or
residents of the forum.

FACTS: Norma A. Del Socorro and Ernst Van Wilsem contracted marriage in Holland. They were
blessed with a son named Roderigo Norjo Van Wilsem. Unfortunately, their marriage bond ended by
virtue of a Divorce Decree issued by the appropriate Court of Holland. Thereafter, Norma and her son
came home to the Philippines. According to Norma, Ernst made a promise to provide monthly support to
their son. However, since the arrival of petitioner and her son in the Philippines, Ernst never gave support
to Roderigo. Norma filed a complaint against Ernst for violation of R.A. No. 9262 for the latter’s unjust
refusal to support his minor child with petitioner.

The trial court dismissed the complaint since the facts charged in the information do not constitute an
offense with respect to the accused, he being an alien.

ISSUE: Does a foreign national have an obligation to support his minor child under Philippine law?

RULING: Yes, since Ernst is a citizen of Holland or the Netherlands, we agree with the RTC that he is
subject to the laws of his country, not to Philippine law, as to whether he is obliged to give support to his
child, as well as the consequences of his failure to do so. This does not, however, mean that Ernst is not
obliged to support Norma’s son altogether. In international law, the party who wants to have a foreign law
applied to a dispute or case has the burden of proving the foreign law. In the present case, Ernst hastily
concludes that being a national of the Netherlands, he is governed by such laws on the matter of provision
of and capacity to support. While Ernst pleaded the laws of the Netherlands in advancing his position that
he is not obliged to support his son, he never proved the same. It is incumbent upon Ernst to plead and
prove that the national law of the Netherlands does not impose upon the parents the obligation to support
their child. Foreign laws do not prove themselves in our jurisdiction and our courts are not authorized to
take judicial notice of them. Like any other fact, they must be alleged and proved. Moreover, foreign law
should not be applied when its application would work undeniable injustice to the citizens or residents of
the forum. To give justice is the most important function of law; hence, a law, or judgment or contract
that is obviously unjust negates the fundamental principles of Conflict of Laws. Applying the foregoing,
even if the laws of the Netherlands neither enforce a parent’s obligation to support his child nor penalize
the non-compliance therewith, such obligation is still duly enforceable in the Philippines because it would
be of great injustice to the child to be denied of financial support when the latter is entitled

Remedial Law Review II (CY 2018-2019) – Saturday 1:00pm – 5:00pm: Case Digest Compilation [Atty. Brondial] Page 52
A. Interpleader (Rule 62)

A.1. WACK WACK GOLF & COUNTRY CLUB, INC., plaintiff-appellant, vs. LEE E. WON alias
RAMON LEE and BIENVENIDO A. TAN, defendants-appellees.

G.R. No. L-23851; March 26, 1976

FACTS: The WackWack Golf & Country Club, Inc., a non-stock, civic and athletic corporation duly
organized under the laws of the Philippines, with principal office in Mandaluyong, Rizal, alleged for its
first cause of action, that the defendant Lee E. Won claims ownership of its membership fee certificate
201, by virtue of the decision rendered in civil case 26044 of the CFI of Manila, entitled "Lee E. Won
alias Ramon Lee vs. WackWack Golf & Country Club, Inc." and also by virtue of membership fee
certificate 201-serial no. 1478 issued on October 17, 1963 by Ponciano B. Jacinto, deputy clerk of court
of the said CFI of Manila, for and in behalf of the president and the secretary of the Corporation and of
the People's Bank & Trust Company as transfer agent of the said Corporation, pursuant to the order of
September 23, 1963 in the said case; that the defendant Bienvenido A. Tan, on the other hand, claims to
be lawful owner of its aforesaid membership fee certificate 201 by virtue of membership fee certificate
201-serial no. 1199 issued to him on July 24, 1950 pursuant to an assignment made in his favor by "Swan,
Culbertson and Fritz," the original owner and holder of membership fee certificate 201.

For its second cause of action. it alleged that the membership fee certificate 201-serial no. 1478 issued by
the deputy clerk of court of court of the CFI of Manila in behalf of the Corporation is null and void
because issued in violation of its by-laws, which require the surrender and cancellation of the outstanding
membership fee certificate 201 before issuance may be made to the transferee of a new certificate duly
signed by its president and secretary, aside from the fact that the decision of the CFI of Manila in civil
case 26044 is not binding upon the defendant Tan, holder of membership fee certificate 201-serial no.
1199; that Tan is made a party because of his refusal to join it in this action or bring a separate action to
protect his rights despite the fact that he has a legal and beneficial interest in the subject matter of this
litigation; and that he is made a part so that complete relief may be accorded herein.

The Corporation prayed that (a) an order be issued requiring Lee and Tan to interplead and litigate their
conflicting claims; and (b) judgment. be rendered, after hearing, declaring who of the two is the lawful
owner of membership fee certificate 201, and ordering the surrender and cancellation of membership fee
certificate 201-serial no. 1478 issued in the name of Lee.

ISSUE: Whether or not the instant interpleader will prosper?

RULING: NO.The action of interpleader, under section 120 of the Code of Civil Procedure, is a remedy
whereby a person who has personal property in his possession, or an obligation to render wholly or
partially, without claiming any right to either, comes to court and asks that the persons who claim the said
personal property or who consider themselves entitled to demand compliance with the obligation, be
required to litigate among themselves in order to determine finally who is entitled to tone or the one thing.

There is no question that the subject matter of the present controversy, i.e., the membership fee certificate
201, is proper for an interpleader suit. What is here disputed is the propriety and timeliness of the remedy
in the light of the facts and circumstances obtaining.

A stakeholder should use reasonable diligence to hale the contending claimants to court. He need not
await actual institution of independent suits against him before filing a bill of interpleader. He should file
an action of interpleader within a reasonable time after a dispute has arisen without waiting to be sued by
either of the contending claimants. Otherwise, he may be barred by laches or undue delay. But where he
acts with reasonable diligence in view of the environmental circumstances, the remedy is not barred.

Has the Corporation in this case acted with diligence, in view of all the circumstances, such that it may
properly invoke the remedy of interpleader? We do not think so. It was aware of the conflicting claims of
the appellees with respect to the membership fee certificate 201 long before it filed the present
interpleader suit. It had been recognizing Tan as the lawful owner thereof. It was sued by Lee who also
claimed the same membership fee certificate. Yet it did not interplead Tan. It preferred to proceed with
the litigation (civil case 26044) and to defend itself therein. As a matter of fact, final judgment was
rendered against it and said judgment has already been executed. It is not therefore too late for it to
invoke the remedy of interpleader.

Remedial Law Review II (CY 2018-2019) – Saturday 1:00pm – 5:00pm: Case Digest Compilation [Atty. Brondial] Page 53
It has been held that a stakeholder's action of interpleader is too late when filed after judgment has been
rendered against him in favor of one of the contending claimants, 13 especially where he had notice of the
conflicting claims prior to the rendition of the judgment and neglected the opportunity to implead the
adverse claimants in the suit where judgment was entered. This must be so, because once judgment is
obtained against him by one claimant he becomes liable to the latter.

Remedial Law Review II (CY 2018-2019) – Saturday 1:00pm – 5:00pm: Case Digest Compilation [Atty. Brondial] Page 54
A.2. ETERNAL GARDENS MEMORIAL PARKS CORPORATION, petitioner, vs. FIRST
SPECIAL CASES DIVISION INTERMEDIATE APPELLATE COURT and NORTH
PHILIPPINE UNION MISSION OF THE SEVENTH-DAY ADVENTISTS, respondents.

G.R. No. 73794; September 19, 1988

FACTS: Eternal Garden and Philippine Union Mission Corporation of the Seventh Day Adventists
(MISSION), executed a Land Development Agreement, whereby Eternal Garden would construct at its
own expense a memorial park subdivided into and sold as memorial plot lots, on the property owned by
MISSION. 40% of the proceeds be remitted monthly by Eternal Garden to MISSION through a
designated depositary trustee bank.

They also executed a Deed of Absolute Sale with Mortgage on said lots. All went well until Maysilo
Estate asserted its claim of ownership over the land in question.Confronted with such conflicting claims,
petitioner filed a complaint for interpleader against MISSION and Maysilo Estate. Alleging among others
that petitioner was not yet the owner but a purchaser thereof, and its willingness to pay to whoever will be
declared as owner. Trial granted the interpleader.

MISSION filed a motion for the placing on judicial deposit the amounts due and unpaid from
petitioner.The motion was denied. An amended order was issued, still in favor of Eternal Garden. Trial
court passed a resolution reversing the judgment and ruled in favour of MISSION ordering the judicial
deposit and dismissal of the interpleader.

The Supreme Court in another case involving the same parties, ordered that Eternal Gardens to deposit
the amounts due and unpaid.

ISSUE: Whether Eternal Gardens should deposit the amounts due and unpaid.

RULING: YES as correctly observed by the Court of Appeals, the essence of an interpleader, aside from
the disavowal of interest in the property in litigation on the part of the petitioner, is the deposit of the
property or funds in controversy with the court. it is a rule founded on justice and equity: "that the
plaintiff may not continue to benefit from the property or funds in litigation during the pendency of the
suit at the expense of whoever will ultimately be decided as entitled thereto."

The case at bar was elevated to the Court of Appeals on certiorari with prohibitory and mandatory
injunction. Said appellate court found that more than twenty million pesos are involved; so that on interest
alone for savings or time deposit would be considerable, now accruing in favor of the Eternal Gardens.
Finding that such is violative of the very essence of the complaint for interpleader as it clearly runs
against the interest of justice in this case, the Court of Appeals cannot be faulted for finding that the lower
court committed a grave abuse of discretion which requires correction by the requirement that a deposit of
said amounts should be made to a bank approved by the Court. 

DOCTRINE: Deposit is proper in interpleader since the petitioner may not continue to benefit from the
property/funds in litigation during the pendency of the suit at the expense of whoever will ultimately be
decided as entitled thereto.

Remedial Law Review II (CY 2018-2019) – Saturday 1:00pm – 5:00pm: Case Digest Compilation [Atty. Brondial] Page 55
A.3. SUBHASH C. PASRICHA and JOSEPHINE A. PASRICHA vs. DON LUISDISON REALTY,
INC.

G.R. No. 136409, March 14, 2008

FACTS: Respondent, Don Luis Dison Realty, Inc. and petitioners executed two Contracts of Lease
whereby the former, as lessor, agreed to lease to the latter several units of theSan Luis Building.
Petitioners, in turn, agreed to pay monthly rentals.While the contracts were in effect, petitioners dealt with
Francis Pacheco (Pacheco), then General Manager of private respondent. Thereafter, Pacheco was
replaced byRoswinda Bautista. Petitioners religiously paid the monthly rentals until May 1992.
After that, however, despite repeated demands, petitioners continuously refused to pay the stipulated rent.
Consequently, respondent filed a complaint for ejectment was filed by private respondent through its
representative, Ms. Bautista, before the Metropolitan Trial Court. Petitioners admitted their failure to pay
the stipulated rent for the leased premises starting July until November 1992, but claimed that such
refusal was justified because of the internal squabble in respondent company as to the person authorized
to receive payment.

ISSUE: Whether the obligor may refuse payment of an obligation on the ground that he does not know to
whom payment must be made.

RULING: The answer is in the negative. Non-payment of rentals because ostensibly they did
not know to whom payment should be made did not justify their failure to pay, because if such were the
case, they were not without any remedy. They should have availed of the provisions of the Civil Code of
the Philippines on the consignation of payment and of the Rules of Court on interpleader. Article 1256 of
the Civil Code provides: If the creditor to whom tender of payment has been made refuses without just
cause to accept it, the debtor shall be released from responsibility by the consignation of the thing or sum
due. Consignation shall be made by depositing the things due at the disposal of a judicial authority, before
whom the tender of payment shall be proved in a proper case, and the announcement of the consignation
in other cases.

Moreover, Section 1, Rule 62 of the Rules of Court provides: Whenever conflicting claims upon the same
subject matter are or may be made against a person who claims no interest whatever in the subject matter,
or an interest which in whole or in part is not disputed by the claimants, he may bring an action against
the conflicting claimants to compel them to interplead and litigate their several claims
among themselves.

Otherwise stated, an action for interpleader is proper when the lessee does not know to whom payment of
rentals should be made due to conflicting claims on the property (or on the right to collect).60 The
remedy is afforded not to protect a person against double liability but to protect him against double
vexation in respect of one liability.Notably, instead of availing of the above remedies, petitioners opted to
refrain from making payments.

Remedial Law Review II (CY 2018-2019) – Saturday 1:00pm – 5:00pm: Case Digest Compilation [Atty. Brondial] Page 56
A.4. BANK OF COMMERCE,Petitioner, vs. PLANTERS DEVELOPMENT BANK and BANGKO
SENTRAL NG PILIPINAS,Respondent.

G.R. Nos. 154470-71; September 24, 2012


x-----------------------x

BANGKO SENTRAL NG PILIPINAS,Petitioner, vs.PLANTERS DEVELOPMENT


BANK,Respondent.

G.R. Nos. 154589-90; BRION, J.:

FACTS: Before the Court are two consolidated petitions for review on certiorari under Rule 45on pure
questions of law, filed by the petitioners Bank of Commerce (BOC) and the BangkoSentralngPilipinas
(BSP), assailing the January 10, 2002 and July 23, 2002 RTC of Makati City, dismissing (i) the petition
filed by the Planters Development Bank (PDB), (ii) the "counterclaim" filed by the BOC, and (iii) the
counter-complaint/cross-claim for interpleader filed bythe BSP; and denied the BOC’s and the BSP’s
motions for reconsideration.

Rizal Commercial Banking Corporation (RCBC) was the registered owner of 2 sets of Central Bank (CB)
bills, with total face value of ₱ 70 million and ₱ 20 million, which were eventually negotiated to Planters
Development Bank (PDB). After different transfers/ negotiations involving several banks/institutions, the
CBs were ultimately acquired bythe Bank of Commerce (BOC). Upon learning of the transfers involving
the CB bills, the PDB informed the Officer-in-Charge of the BSP’s Government Securities Department,
Nuqui, of the PDB’s claim over these CB bills, based on the Detached Assignments in its possession. It
requested the BSP to record its claim in the BSP’s books, explaining that its non-possession of the CB
bills is "on account of imperfect negotiations thereof and/or subsequent setoff or transfer” and that,
subsequent transferees thereof were not holders in due course. Nuqui denied the request, invoking Section
8 of CB Circular No. 28, the existing BSP Regulation, which requires the presentation of the bond before
a registered bond may be transferred on the books of the BSP. This prompted PDB to file with the RTC
petitions for mandamus, prohibition, and injunction against BSP and BOC reiterating that there was no
intent on its part to transfer title of the CB bills, as shown by its non-issuance of a detached assignment,
particularly alleging that it merely "warehoused" the first set of CB bills with the BOC, as security
collateral.

BOC filed its answer, praying for the dismissal of the case, alleging that PDB has no cause of action
considering that PDB is no longer the owner of the CBs. Alternatively, the BSP in its counter-
complaint/cross-claim, asked that an interpleader suit be allowed between and among the claimants to the
subject CB bills on the position that while it is able and willing to pay the subject CBs, it is duty bound to
ensure that payment is made to the rightful owner.

ISSUE: Whether or not an interpleader suit may be initiated through the petitioner’s answer

RULING: YES. The remedy of interpleader, as a special civil action, is primarily governed by the
specific provisions in Rule 62 of the Rules of Court and secondarily by the provisions applicable to
ordinary civil actions. Indeed, Rule 62 does not expressly authorize the filing of a complaint-in-
interpleader as part of, although separate and independent from, the answer. Similarly, Section 5, Rule 6,
in relation to Section 1, Rule 9 of the Rules of Court does not include a complaint-in-interpleader as a
claim, a form of defense, or as an objection that a defendant may be allowed to put up in his answer or in
a motion to dismiss. This does not mean, however, that the BSP’s "counter-complaint/cross-claim for
interpleader" runs counter to general procedures.

Apart from a pleading, the rules allow a party to seek an affirmative relief from the court through the
procedural device of a motion. While captioned "Answer with counter complaint/cross-claim for
interpleader," the RTC understood this as in the nature of a motion, seeking relief which essentially
consists in an order for the conflicting claimants to litigate with each other so that "payment is made to
the rightful or legitimate owner" of the subject CB bills.

The rules define a "civil action" as "one by which a party sues another for the enforcement or protection
of a right, or the prevention or redress of a wrong." Interpleader may be considered as a stakeholder’s
remedy to prevent a wrong, that is, from making payment to one not entitled to it, thereby rendering itself
vulnerable to lawsuit/s from those legally entitled to payment.

Remedial Law Review II (CY 2018-2019) – Saturday 1:00pm – 5:00pm: Case Digest Compilation [Atty. Brondial] Page 57
Interpleader is a civil action made special by the existence of particular rules to govern the uniqueness of
its application and operation. Under Section 2, Rule 6 of the Rules of Court, governing ordinary civil
actions, a party’s claim is asserted "in a complaint, counterclaim, cross-claim, third (fourth, etc.)-party
complaint, or complaint-in-intervention." In an interpleader suit, however, a claim is not required to be
contained in any of these pleadings but in the answer-(of the conflicting claimants)-in-interpleader. This
claim is different from the counter-claim (or cross-claim, third party-complaint) which is separately
allowed under Section 5, par. 2 of Rule 62.

WHEREFORE, premises considered the consolidated PETITIONS are GRANTED. The Planters
Development Bank is hereby REQUIRED to file with the Regional Trial Court its comment or answer-in-
interpleader to Bank of Commerce’s Amended Consolidated Answer with Compulsory Counterclaim, as
previously ordered by the Regional Trial Court. The Regional Trial Court of Makati City, Branch 143, is
hereby ORDERED to assess the docket fees due from Planters Development Bank and Bank of
Commerce and order their payment, and to resolve with DELIBERATE DISPATCH the parties’
conflicting claims of ownership over the proceeds of the Central Bank bills.

The Clerk of Court of the Regional Trial Court of Makati City, Branch 143, or his duly authorized
representative is hereby ORDERED to assess and collect the appropriate amount of docket fees separately
due the Bank of Commerce and Planters Development Bank as conflicting claimants in
BangkoSentralngPilipinas’ interpleader suit, in accordance with this decision. SO ORDERED.

DOCTRINE: Interpleader is a civil action made special by the existence of particular rules to govern
the uniqueness of its application and operation. Under Section 2, Rule 6 of the Rules of Court, governing
ordinary civil actions, a party’s claim is asserted "in a complaint, counterclaim, cross-claim, third
(fourth, etc.)-party complaint, or complaint-in-intervention." In an interpleader suit, however, a claim is
not required to be contained in any of these pleadings but in the answer-(of the conflicting claimants)-in-
interpleader. This claim is different from the counter-claim (or cross-claim, third party-complaint) which
is separately allowed under Section 5, par. 2 of Rule 62.

Remedial Law Review II (CY 2018-2019) – Saturday 1:00pm – 5:00pm: Case Digest Compilation [Atty. Brondial] Page 58
B. Declaratory Relief and Similar Remedies

B.1. EufemiaAlmeda and RomelAlmeda vs. Bathala Marketing Industries, Inc.

G.R. No. 150806; January 28, 2008; Nachura, J.:

FACTS: Sometime in May 1997, respondent Bathala Marketing Industries, Inc., as lessee, represented by
its president Ramon H. Garcia, renewed its Contract of Lease with Ponciano L. Almeda (Ponciano), as
lessor, husband of petitioner Eufemia and father of petitioner RomelAlmeda. The contract of lease
contained the following pertinent provisions which gave rise to the instant case:

SIXTH It is expressly understood by the parties hereto that the rental rate stipulated is based on the
present rate of assessment on the property, and that in case the assessment should hereafter be increased
or any new tax, charge or burden be imposed by authorities on the lot and building where the leased
premises are located, LESSEE shall pay, when the rental herein provided becomes due, the additional
rental or charge corresponding to the portion hereby leased; provided, however, that in the event that the
present assessment or tax on said property should be reduced, LESSEE shall be entitled to reduction in
the stipulated rental, likewise in proportion to the portion leased by him;

SEVENTH In case an extraordinary inflation or devaluation of Philippine Currency should supervene, the
value of Philippine peso at the time of the establishment of the obligation shall be the basis of payment;

During the effectivity of the contract, Ponciano died. Thereafter, respondent dealt with petitioners. In a
letter, petitioners advised respondent that the former shall assess and collect Value Added Tax (VAT) on
its monthly rentals.

Again, respondent received another letter from petitioners informing the former that its monthly rental
should be increased by 73% pursuant to condition No. 7 of the contract and Article 1250 of the Civil
Code.

Respondent instituted an action for declaratory relief for purposes of determining the correct
interpretation of condition Nos. 6 and 7 of the lease contract to prevent damage and prejudice.

Petitioners in turn filed an action for ejectment, rescission and damages against respondent for failure of
the latter to vacate the premises after the demand made by the former.

The RTC ruled in favor of respondent and against petitioners.

Petitioners elevated the aforesaid case to the Court of Appeals which affirmed with modification the RTC
decision.

ISSUE: Whether the action for declaratory relief is proper.

RULING: Yes,the action for declaratory relief is proper.

It is beyond cavil that the requisites for availing a declaratory relief are present in the instant case.

WHEREFORE, premises considered, the petition is DENIED. The Decision of the Court of Appeals in
CA-G.R. CV No. 67784, dated September 3, 2001, and its Resolution dated November 19, 2001, are
AFFIRMED.

DOCTRINE: Declaratory relief is defined as an action by any person interested in a deed, will, contract
or other written instrument, executive order or resolution, to determine any question of construction or
validity arising from the instrument, executive order or regulation, or statute, and for a declaration of his
rights and duties thereunder. The only issue that may be raised in such a petition is the question of
construction or validity of provisions in an instrument or statute. Corollary is the general rule that such
an action must be justified, as no other adequate relief or remedy is available under the circumstances.

Decisional law enumerates the requisites of an action for declaratory relief, as follows: 1) the subject
matter of the controversy must be a deed, will, contract or other written instrument, statute, executive
order or regulation, or ordinance; 2) the terms of said documents and the validity thereof are doubtful
and require judicial construction; 3) there must have been no breach of the documents in question; 4)

Remedial Law Review II (CY 2018-2019) – Saturday 1:00pm – 5:00pm: Case Digest Compilation [Atty. Brondial] Page 59
there must be an actual justiciable controversy or the ripening seeds of one between persons whose
interests are adverse; 5) the issue must be ripe for judicial determination; and 6) adequate relief is not
available through other means or other forms of action or proceeding.

Remedial Law Review II (CY 2018-2019) – Saturday 1:00pm – 5:00pm: Case Digest Compilation [Atty. Brondial] Page 60
B.2. Republic vs. Orbecido III, 472 SCR 114,

G.R. No. 154380; October 5, 2005

FACTS: Cipriano Orbecido III married Lady Villanueva in the Philippines. They had a son and a
daughter, Kristoffer and Kimberly. But, Cipriano’s wife left for USA bringing along Kristoffer. A few
years later, Cipriano discovered that his wife had been naturalized as an American citizen. Cipriano
learned from his son that his wife had obtained a divorce decree and then married a certain Innocent
Stanley. She, Stanley and her child by him currently live in California. Cipriano thereafter filed with the
RTC a petition for authority to remarry invoking Paragraph 2 of Article 26 of the Family Code. No
opposition was filed. The RTC granted the same.

In this petition for review, the Solicitor General assails the decision and resolution of the RTC that herein
respondent Cipriano Orbecido III is capacitated to remarry. The OSG contends that Paragraph 2 of Article
26 of the Family Code is not applicable to the instant case because it only applies to a valid mixed
marriage; that is, a marriage celebrated between a Filipino citizen and an alien. The proper remedy,
according to the OSG, is to file a petition for annulment or for legal separation. Furthermore, the OSG
argues there is no law that governs respondents situation. The OSG posits that this is a matter of
legislation and not of judicial determination.

On the other hand, respondent admits that Article 26 is not directly applicable to his case but insists that
when his naturalized alien wife obtained a divorce decree which capacitated her to remarry, he is likewise
capacitated by operation of law pursuant to Section 12, Article II of the Constitution.

ISSUE:Given a valid marriage between two Filipino citizens, where one party is later naturalized as a
foreign citizen and obtains a valid divorce decree capacitating him or her to remarry, can the Filipino
spouse likewise remarry under Article 26 of the Family Code?

RULING: Remanded to lower court for further submission of evidence. Paragraph 2 of Article 26 of the
Family Code, should be interpreted to allow a Filipino citizen, who has been divorced by a spouse who
had acquired foreign citizenship and remarried, also to remarry. However, considering that in the present
petition there is no sufficient evidence submitted, this Court is unable to declare, based on respondents
bare allegations that his wife, who was naturalized as an American citizen, had obtained a divorce decree
and had remarried an American, that respondent is now capacitated to remarry. Such declaration could
only be made properly upon respondent’s submission of the evidence in his favor.

DOCTRINE: Civil Procedure; Declaratory Relief; Requisites of a Petition for Declaratory Relief.- The
requisites of a petition for declaratory relief are: (1) there must be a justiciable controversy; (2) the
controversy must be between persons whose interests are adverse; (3) that the party seeking the relief has
a legal interest in the controversy; and (4) that the issue is ripe for judicial determination.

Remedial Law Review II (CY 2018-2019) – Saturday 1:00pm – 5:00pm: Case Digest Compilation [Atty. Brondial] Page 61
B.3. Malana vs. Tappa

G.R. No. 181303; September 17, 2009; Chico-Nazario, J.

FACTS: Petitioners alleged in their Complaint that they are the owners of a parcel of situated in
Tuguegarao City, Cagayan (subject property). Petitioners inherited the subject property from Anastacio
Danao (Anastacio), who died intestate. During the lifetime of Anastacio, he had allowed Consuelo Pauig
(Consuelo), who was married to Joaquin Boncad, to build on and occupy the southern portion of the
subject property. Anastacio and Consuelo agreed that the latter would vacate the said land at any time that
Anastacio and his heirs might need it. Petitioners claimed that respondents, Consuelos family members,
continued to occupy the subject property even after her death, already building their residences thereon
using permanent materials.

Petitioners referred their land dispute with respondents to the Lupong Tagapamayapa of Barangay
Annafunan West for conciliation. During the conciliation proceedings, respondents asserted that they
owned the subject property and presented documents ostensibly supporting their claim of ownership.
According to petitioners, respondent’s documents were highly dubious, falsified, and incapable of
proving the latters claim of ownership over the subject property; nevertheless, they created a cloud upon
petitioner’s title to the property. Thus, petitioners were compelled to file before the RTC a Complaint to
remove such cloud from their title. Before respondents could file their answer, the RTC issued an Order
dated 4 May 2007 dismissing petitioners Complaint on the ground of lack of jurisdiction.

Petitioners filed a Motion for Reconsideration of the aforementioned RTC Order dismissing their
Complaint. They argued that their principal cause of action was for quieting of title; the accion
reivindicacion was included merely to enable them to seek complete relief from respondents.
Petitioners Complaint should not have been dismissed, since Section 1, Rule 63 of the Rules of Court
states that an action to quiet title falls under the jurisdiction of the RTC. In an Order dated 30 May 2007,
the RTC denied petitioners Motion for Reconsideration. It reasoned that an action to quiet title is a real
action. Pursuant to Republic Act No. 7691, it is the Municipal Trial Court (MTC) that exercises exclusive
jurisdiction over real actions where the assessed value of real property does not exceed P20,000.00. Since
the assessed value of subject property per Tax Declaration No, 02-48386 was P410.00, the real action
involving the same was outside the jurisdiction of the RTC.

ISSUE: Whether the RTC has jurisdiction in the instant case.

RULING: Yes, the dismissal of the instant case is valid. Moreover, the remedy of declaratory relief is
not proper.

Issue on jurisdiction
An action for declaratory relief should be filed by a person interested under a deed, a will, a contract or
other written instrument, and whose rights are affected by a statute, an executive order, a regulation or an
ordinance. The relief sought under this remedy includes the interpretation and determination of the
validity of the written instrument and the judicial declaration of the parties rights or duties there under.

Petitions for declaratory relief are governed by Rule 63 of the Rules of Court. The RTC correctly made a
distinction between the first and the second paragraphs of Section 1, Rule 63 of the Rules of Court.

As the afore-quoted provision states, a petition for declaratory relief under the first paragraph of Section
1, Rule 63 may be brought before the appropriate RTC.

The second paragraph of Section 1, Rule 63 of the Rules of Court specifically refers to (1) an action for
the reformation of an instrument, recognized under Articles 1359 to 1369 of the Civil Code; (2) an action
to quiet title, authorized by Articles 476 to 481 of the Civil Code; and (3) an action to consolidate
ownership required by Article 1607 of the Civil Code in a sale with a right to repurchase.

These three remedies are considered similar to declaratory relief because they also result in the
adjudication of the legal rights of the litigants, often without the need of execution to carry the judgment
into effect.

To determine which court has jurisdiction over the actions identified in the second paragraph of
Section 1, Rule 63 of the Rules of Court, said provision must be read together with those of the
Judiciary Reorganization Act of 1980, as amended.

Remedial Law Review II (CY 2018-2019) – Saturday 1:00pm – 5:00pm: Case Digest Compilation [Atty. Brondial] Page 62
In contrast, the mandatory provision of the Judiciary Reorganization Act of 1980, as amended, uses the
word shall and explicitly requires the MTC to exercise exclusive original jurisdiction over all civil actions
which involve title to or possession of real property where the assessed value does not exceed P20,000.00,

As found by the RTC, the assessed value of the subject property as stated in Tax Declaration No.
02-48386 is only P410.00; therefore, petitioners Complaint involving title to and possession of the said
property is within the exclusive original jurisdiction of the MTC, not the RTC.

Proper remedy on the instant case


Furthermore, an action for declaratory relief presupposes that there has been no actual breach of the
instruments involved or of rights arising thereunder. Since the purpose of an action for declaratory relief
is to secure an authoritative statement of the rights and obligations of the parties under a statute, deed, or
contract for their guidance in the enforcement thereof, or compliance therewith, and not to settle issues
arising from an alleged breach thereof, it may be entertained only before the breach or violation of the
statute, deed, or contract to which it refers. A petition for declaratory relief gives a practical remedy for
ending controversies that have not reached the state where another relief is immediately available; and
supplies the need for a form of action that will set controversies at rest before they lead to a repudiation of
obligations, an invasion of rights, and a commission of wrongs.

Where the law or contract has already been contravened prior to the filing of an action for declaratory
relief, the courts can no longer assume jurisdiction over the action. In other words, a court has no more
jurisdiction over an action for declaratory relief if its subject has already been infringed or transgressed
before the institution of the action.

In the present case, petitioners Complaint for quieting of title was filed after petitioners already demanded
and respondents refused to vacate the subject property. In fact, said Complaint was filed only subsequent
to the latters express claim of ownership over the subject property before the Lupong Tagapamayapa, in
direct challenge to petitioners title.

Since petitioners averred in the Complaint that they had already been deprived of the possession of their
property, the proper remedy for them is the filing of an accion publiciana or an accion reivindicatoria, not
a case for declaratory relief

Remedial Law Review II (CY 2018-2019) – Saturday 1:00pm – 5:00pm: Case Digest Compilation [Atty. Brondial] Page 63
B.4. FRANCISCO I. CHAVEZ, Petitioner, vs. JUDICIAL AND BAR COUNCIL, SEN. FRANCIS
JOSEPH G. ESCUDERO and REP. NIEL C. TUPAS, JR., Respondents.

G.R. No. 202242; April 16, 2013; MENDOZA, J.:

FACTS: The issue at hand has been in hibernation until the unexpected departure of Chief Justice Renato
C. Corona on May 29, 2012, and the nomination of former Solicitor General Francisco I. Chavez
(petitioner), as his potential successor, triggered the filing of this case. The issue has constantly been
nagging legal minds, yet remained dormant for lack of constitutional challenge.

Prompted by the clamor to rid the process of appointments to the Judiciary from political pressure and
partisan activities, the members of the Constitutional Commission saw the need to create a separate,
competent and independent body to recommend nominees to the President. Thus, it conceived of a body
representative of all the stakeholders in the judicial appointment process and called it the Judicial and Bar
Council (JBC). Its composition, term and functions are provided under Section 8, Article VIII of the
Constitution, viz: Section 8, Article VIII of the Constitution, viz:

“Section 8. (1) A Judicial and Bar Council is hereby created under the supervision of the
Supreme Court composed of the Chief Justice as ex officio Chairman, the Secretary of
Justice, and a representative of the Congress as ex officio Members, a representative of the
Integrated Bar, a professor of law, a retired Member of the Supreme Court, and a
representative of the private sector.

(2) The regular members of the Council shall be appointed by the President for a term of four
years with the consent of the Commission on Appointments. Of the Members first appointed,
the representative of the Integrated Bar shall serve for four years, the professor of law for
three years, the retired Justice for two years, and the representative of the private sector for
one year.

(3) The Clerk of the Supreme Court shall be the Secretary ex officio of the Council and shall
keep a record of its proceedings.

(4) The regular Members of the Council shall receive such emoluments as may be determined
by the Supreme Court. The Supreme Court shall provide in its annual budget the
appropriations for the Council.

(5) The Council shall have the principal function of recommending appointees to the
Judiciary. It may exercise such other functions and duties as the Supreme Court may assign
to it.”

In compliance therewith, Congress, from the moment of the creation of the JBC, designated one
representative to sit in the JBC to act as one of the ex officio members. Perhaps in order to give equal
opportunity to both houses to sit in the exclusive body, the House of Representatives and the Senate
would send alternate representatives to the JBC. In other words, Congress had only one (1) representative.

In 1994, the composition of the JBC was substantially altered. Instead of having only seven (7) members,
an eighth (8th) member was added to the JBC as two (2) representatives from Congress began sitting in
the JBC - one from the House of Representatives and one from the Senate, with each having one-half
(1/2) of a vote. Then, curiously, the JBC En Banc, in separate meetings held in 2000 and 2001, decided to
allow the representatives from the Senate and the House of Representatives one full vote each. At present,
Senator Francis Joseph G. Escudero and Congressman Niel C. Tupas, Jr. (respondents) simultaneously sit
in the JBC as representatives of the legislature. It is this practice that petitioner has questioned in this
petition.

ISSUE: Whether or not the petition is in the nature of an action under declaratory relief under Rule 63?

RULING: YES, the Court views the petition as essentially an action for declaratory relief under Rule 63
of the 1997 Rules of Civil Procedure.

Before addressing the above issues in seriatim, the Court deems it proper to first ascertain the nature of
the petition. Pursuant to the rule that the nature of an action is determined by the allegations therein and

Remedial Law Review II (CY 2018-2019) – Saturday 1:00pm – 5:00pm: Case Digest Compilation [Atty. Brondial] Page 64
the character of the relief sought, the Court views the petition as essentially an action for declaratory
relief under Rule 63 of the 1997 Rules of Civil Procedure.

The Constitution as the subject matter, and the validity and construction of Section 8 (1), Article VIII as
the issue raised, the petition should properly be considered as that which would result in the adjudication
of rights sans the execution process because the only relief to be granted is the very declaration of the
rights under the document sought to be construed. It being so, the original jurisdiction over the petition
lies with the appropriate Regional Trial Court (RTC). Notwithstanding the fact that only questions of law
are raised in the petition, an action for declaratory relief is not among those within the original
jurisdiction of this Court as provided in Section 5, Article VIII of the Constitution.

At any rate, due to its serious implications, not only to government processes involved but also to the
sanctity of the Constitution, the Court deems it more prudent to take cognizance of it. After all, the
petition is also for prohibition under Rule 65 seeking to enjoin Congress from sending two (2)
representatives with one (1) full vote each to the JBC.

DOCTRINE: An action for declaratory relief is not among those within the original jurisdiction of this
Court as provided in Section 5, Article VIII of the Constitution.

Remedial Law Review II (CY 2018-2019) – Saturday 1:00pm – 5:00pm: Case Digest Compilation [Atty. Brondial] Page 65
B.5. SPOUSES SABITSANA VS MUERTEGUI

G.R. No. 181359; 5 August 2013; J. DEL CASTILLO

FACTS: Clemencio Sabitsana (P), lawyer, was the counsel of Respondents. The dispute involved a
parcel of land bought by Juanito (R) by virtue of an unnotarized deed of sale from Garcia. Juanito’s
father and his brother Domingo, also R herein, took actual possession of the land. Later, Garcia sold the
same land to P, this time, through a notarized deed of sale.

When Rs’ father passed away, the heirs applied for the registration and coverage of the lot under Public
Land Act or CA No. 141. P opposed the application, claiming as a true owner of the lot. Respondents
filed for quieting of title and preliminary injunction against Ps, Clemencio and his wife, Rosario, claiming
that they bought the land in bad faithandareexercisingpossessionandownershipofthesame,whichactthus
constitute cloud over the title. RTC and CA ruled in favor of Rs.

ISSUE: Whether RTC has jurisdiction over the declaratory relief.


(P’s contention: should be based on assessed value of the property; hence, jurisdiction should only be first
level court-No!)

RULING: YES, petition dismissed. The RTC has jurisdiction over the suit for quieting of title.

On the question of jurisdiction, it is clear under the Rules that an action for quieting of title may be
instituted in the RTCs, regardless of the assessed value of the real property in dispute. Under Rule 63 of
the Rules of Court, an action to quiet title to real property or remove clouds therefrom may be brought in
the appropriate RTC.

Additional:
Both the trial court and the CA are, however, wrong in applying Article 1544 of the Civil Code. That
provision does not apply to sales involving unregistered land. Act No. 3344 applies to sale of unregistered
lands. What applies in this case is Act No. 3344, as amended, which provides for the system of recording
of transactions over unregistered real estate. Act No. 3344 expressly declares that any registration made
shall be without prejudice to a third party with a better right. The question to be resolved therefore is: who
between petitioners and respondent has a better right to the disputed lot? Respondent has a better right to
the lot since P purchaser in bad faith.

DOCTRINE: Declaratory relief is an action incapable of pecuniary estimation; hence, RTC has
jurisdiction.

Remedial Law Review II (CY 2018-2019) – Saturday 1:00pm – 5:00pm: Case Digest Compilation [Atty. Brondial] Page 66
B.6. Republic vs. Roque (In relation to southern Hemisphere case)REPUBLIC v. HERMINIO
HARRY ROQUE

GR No. 204603; September 24, 2013; PERLAS-BERNABE, J.:

FACTS: Roque filed a petition for declaratory relief before the RTC, assailing the constitutionality of the
following sections of RA 9372 ("An Act to Secure the State and Protect our People from Terrorism,"
otherwise known as the Human Security Act of 2007): (a) Section 3, for being void for vagueness; (b)
Section 7, for violating the right to privacy of communication and due process and the privileged nature
of priest-penitent relationships; (c )Section 18, for violating due process, the prohibition against ex post
facto laws or bills of attainder, the Universal Declaration of Human Rights, and the International
Covenant on Civil and Political Rights, as well as for contradicting Article 125 of the Revised Penal
Code, as amended; (d) Section 26, for violating the right to travel; and (e) Section 27, for violating the
prohibition against unreasonable searches and seizures. Republic moved to suspend the proceedings,
averring that certain petitions (SC petitions) raising the issue of RA 9372’s constitutionality have been
lodged before the Court.

RTC granted the motion. SC promulgated its Decision in the Southern Hemisphere cases and thereby
dismissed the SC petitions. Republic filed motion to dismiss, contending that Roque, et al. failed to
satisfy the requisites for declaratory relief. Likewise, they averred that the constitutionality of RA 9372
had already been upheld by the Court in the Southern Hemisphere cases. Roque filed
Comment/Opposition. RTC denied motion to dismiss, finding that the Court did not pass upon the
constitutionality of RA 9372 and that Roque’s petition for declaratory relief was properly filed. Hence
Republic filed the Petition for certiorari (65) with the Supreme Court.

ISSUE: Whether or not the petition for declaratory relief was sufficient? Was there compliance to the
requirements of declaratory relief to warrant the case?

RULING: No. The Court, in Southern Hemisphere, did not make any definitive ruling on the
constitutionality of RA 9372. The certiorari petitions in those consolidated cases were dismissed based
solely on procedural grounds, namely: (a) the remedy of certiorari was improper; (b) petitioners therein
lack locus standi; and (c) petitioners therein failed to present an actual case or controversy. Therefore,
there was no grave abuse of discretion (JOV: dismissal was proper in view of the said procedural
grounds).

The following are the requisites for an action for declaratory relief: first , the subject matter of the
controversy must be a deed, will, contract or other written instrument, statute, executive order or
regulation, or ordinance; second , the terms of said documents and the validity thereof are doubtful and
require judicial construction; third, there must have been no breach of the documents in question; fourth,
there must be an actual justiciable controversy or the "ripening seeds" of one between persons whose
interests are adverse; fifth , the issue must be ripe for judicial determination; and sixth, adequate relief is
not available through other means or other forms of action or proceeding.

Based on a judicious review of the records, the Court observes that while the first, second, and third
requirements appear to exist in this case, the fourth, fifth, and sixth requirements, however, remain
wanting.

As to the fourth requisite, no actual justiciable controversy or the “ripening seeds” of one exists in this
case. A justiciable controversy refers to an existing case or controversy that is appropriate or ripe for
judicial determination, not one that is conjectural or merely anticipatory. A perusal of private
respondents’ petition for declaratory relief would show that they have failed to demonstrate how they are
left to sustain or are in immediate danger to sustain some direct injury as a result of the enforcement of
the assailed provisions of RA 9372. Not far removed from the factual milieu in the Southern Hemisphere
cases, private respondents only assert general interests as citizens, and taxpayers and infractions which
the government could prospectively commit if the enforcement of the said law would remain
untrammeled. As their petition would disclose, private respondents’ fear of prosecution was solely based
on remarks of certain government officials which were addressed to the general public. They, however,
failed to show how these remarks tended towards any prosecutorial or governmental action geared
towards the implementation of RA 9372 against them. In other words, there was no particular, real or
imminent threat to any of them. the Court dismissed the SC petitions in the Southern Hemisphere cases
on the basis of, among others, lack of actual justiciable controversy (or the ripening seeds of one), the
RTC should have dismissed private respondents’ petition for declaratory relief all the same.

Remedial Law Review II (CY 2018-2019) – Saturday 1:00pm – 5:00pm: Case Digest Compilation [Atty. Brondial] Page 67
Also, the private respondents lack the required locus standi to mount their constitutional challenge against
the implementation of the above-stated provisions of RA 9372 since they have not shown any direct and
personal interest in the case. [JOV: even though transcendental public importance dispenses with the
locus standi requirement, there must still be a closer judicial scrutiny of locus standi] As to the fifth
requisite for an action for declaratory relief, neither can it be inferred that the controversy at hand is ripe
for adjudication since the possibility of abuse xxx remain highly- speculative and merely theorized. xxx a
question is ripe for adjudication when the act being challenged has had a direct adverse effect on the
individual challenging it. Finally, as regards the sixth requisite, the Court finds it irrelevant to proceed
with a discussion on the availability of adequate reliefs since no impending threat or injury to the private
respondents exists in the first place.

In relation to Southern Hemisphere case, 632 SCRA 10/5/2010

RA 9372 ("An Act to Secure the State and Protect our People from Terrorism," otherwise known as the
Human Security Act of 2007) was signed into law on March 6, 2007. RA 9372 subsequently became
effective. Petitioners filed petitions for certiorari and prohibition.

Whether declaratory actions characterized by “double contingency” are within the ambit of judicial
review. No, the Court is not unaware that a reasonable certainty of the occurrence of a perceived threat to
any constitutional interest suffices to provide a basis for mounting a constitutional challenge. This,
however, is qualified by the requirement that there must be sufficient facts to enable the Court to
intelligently adjudicate the issues. Herein petitioners have failed to show that the challenged provisions of
RA 9372 forbid constitutionally protected conduct or activity that they seek to do. No demonstrable threat
has been established, much less a real and existing one. Petitioners’ obscure allegations of sporadic
"surveillance" and supposedly being tagged as "communist fronts" in no way approximate a credible
threat of prosecution. From these allegations, the Court is being lured to render an advisory opinion,
which is not its function. Without any justiciable controversy, the petitions have become pleas for
declaratory relief, over which the Court has no original jurisdiction. Then again, declaratory actions
characterized by "double contingency," where both the activity the petitioners intend to undertake and the
anticipated reaction to it of a public official are merely theorized, lie beyond judicial review for lack of
ripeness. The possibility of abuse in the implementation of RA 9372 does not avail to take the present
petitions out of the realm of the surreal and merely imagined. Such possibility is not peculiar to RA 9372
since the exercise of any power granted by law may be abused. Allegations of abuse must be anchored on
real events before courts may step in to settle actual controversies involving rights which are legally
demandable and enforceable.

Remedial Law Review II (CY 2018-2019) – Saturday 1:00pm – 5:00pm: Case Digest Compilation [Atty. Brondial] Page 68
B.7. DEPARTMENT OF FINANCE VS DELA CRUZ

G.R. No. 209331; August 24, 2015; CARPIO, J.

FACTS: EO 140 states that the Customs Policy Research Office (CPRO) "shall be responsible for
reviewing the customs administration policies, rules and procedures, and thereafter providing sound
recommendations for the improvement of the same." Section 3 of EO 140 provides that "CPRO shall be
composed of its organic personnel, as approved by the Department of Budget and Management (DBM)
upon recommendation of the DOF Secretary, augmented and reinforced by DOF and BOC personnel as
well as those detailed or seconded from other agencies, whether attached to the DOF or not. x x x."
Section 9 of EO 140 states that it shall "take effect immediately upon publication in two (2) newspapers
of general circulation."

Bureau of Customs (BOC) Commissioner Rozzano Rufino B. Biazon issued Customs Personnel Order
No. B-189-2013 (CPO 189-2013) detailing 27 BOC personnel including respondents in this case to
CPRO "effective immediately and valid until sooner revoked."Respondents filed an action for
Declaratory Relief with Application for Temporary Restraining Order and/or Writ of Preliminary
Injunction before the Regional Trial Court (RTC) of Manila. Petitioners alleged that CPO 189-2013 is an
internal personnel order with application that is limited to and only within BOC and as such, it cannot be
the subject of an action for declaratory relief.

ISSUE: Whether or not the RTC and not the CSC has jurisdiction over the action for declaratory relief
filed by respondents?

RULING:Yes. Regional Trial Court has jurisdiction over the action for declaratory relief. The petition
went beyond questioning the detail of respondents. Respondents further assailed the validity and
constitutionality of CPO 189-2013. Respondents alleged that CPO 189-2013 was issued even before EO
140 became effective.  The issue took the case beyond the scope of the CSC's jurisdiction because the
matter is no longer limited to personnel action.

Respondents assail CPO 189-2013 as patently illegal, arbitrary, and oppressive. This case clearly falls
within the exceptions where exhaustion of administrative remedies need not be resorted to by
respondents.

DOCTRINE: When respondents raised the issue of validity and constitutionality of CPO 189-2013, the
issue took the case beyond the scope of the CSC's jurisdiction because the matter is no longer limited to
personnel action.

Dissenting (J. Leonen): Courts should not directly assume jurisdiction based on allegations of
unconstitutionality and invalidity of government regulations when the question, in essence, involves a
personnel action.

Remedial Law Review II (CY 2018-2019) – Saturday 1:00pm – 5:00pm: Case Digest Compilation [Atty. Brondial] Page 69
B.8. REP. REYNALDO V. UMALI, in his capacity as Chairman of the House of Representatives
Committee on Justice and Ex Officio Member of the JBC, Petitioner vs.
THE JUDICIAL AND BAR COUNCIL, chaired by THE HON. MARIA LOURDES P.A.
SERENO, Chief Justice and Ex Officio Chairperson, Respondent

G.R. No. 228628; July 25, 2017; VELASCO, JR., J.

FACTS: This Court, in a Decision dated July 17, 2012, declared the said practice of having two
representatives from Congress with one vote each in the JBC unconstitutional. This Court enunciated that
the use of the singular letter "a" preceding "representative of the Congress" in the aforequoted provision
is unequivocal and leaves no room for any other construction or interpretation. The same is indicative of
the Framers' intent that Congress may designate only one representative to the JBC. Had it been
otherwise, they could have, in no uncertain terms, so provided. This Court further articulated that in the
context of JBC representation, the term "Congress" must be taken to mean the entire legislative
department as no liaison between the two houses exists in the workings of the JBC. There is no
mechanism required between the Senate and the House of Representatives in the screening and
nomination of judicial officers. Moreover, this Court, quoting the keen observation of Retired Supreme
Court Associate Justice Consuelo Ynares-Santiago, who is also a JBC Consultant, stated that the ex
officio members of the JBC consist of representatives from the three main branches of government, to wit:
the Chief Justice of the Supreme Court representing the judiciary, the Secretary of Justice representing the
executive, and a representative of the Congress representing the legislature. It can be deduced therefrom
that the unmistakable tenor of Section 8(1), Article VIII of the 1987 Constitution was to treat each ex
officio member as representing one co-equal branch of government having equal say in the choice of
judicial nominees. Now, to allow the legislature to have more than one representative in the JBC would
negate the principle of equality among these three branches of the government, which is enshrined in the
Constitution.

In light of these Decision and Resolution, both Houses of Congress agreed on a six-month rotational
representation in the JBC, wherein the House of Representatives will represent Congress from January to
June and the Senate from July to December. This is now the current practice in the JBC. It is by reason of
this arrangement that the votes cast by the petitioner for the selection of nominees for the vacancies of
then retiring Supreme Court Associate Justices Jose P. Perez (Perez) and Arturo Brion (Brion) were not
counted by the JBC during its En Banc deliberations held last December 2 and 9, 2016. Instead, the
petitioner's votes were simply placed in an envelope and sealed subject to any further disposition as this
Court may direct in a proper proceeding. This is the root of the present controversy that prompted the
petitioner to file the instant Petition for Certiorari and Mandamus

ISSUES:
1. Whether the petitioner's direct resort to this Court via a Petition for Certiorari and Mandamus is the
plain, speedy and adequate remedy available to him to assail the JBC's adoption of the rotational
representation leading to the non-counting of his votes in its En Banc deliberations

2. Whether the JBC can be compelled through mandamus to count the petitioner's votes in its En Banc
deliberations

RULING:
1. Yes. Generally, the writ of certiorari can only be availed of in the absence of an appeal or any plain,
speedy and adequate remedy in the ordinary course of law. In Bordomeo v. Court of Appeals, however,
this Court clarified that it is inadequacy that must usually determine the propriety of certiorari and not the
mere absence of all other remedies and the danger of failure of justice without the writ. A remedy is
considered plain, speedy and adequate if it will promptly relieve the petitioner from the injurious effects
of the judgment, order, or resolution of the lower court or agency.

This Court took pains in enumerating the circumstances that would warrant a direct resort to this Court, to
wit: (1) when there are genuine issues of constitutionality that must be addressed at the most immediate
time; (2) when the issues involved are of transcendental importance; (3) cases of first impression as no
jurisprudence yet exists that will guide the lower courts on this matter; (4) the constitutional issues raised
are better decided by this court; (5) the time element presented in this case cannot be ignored; (6) the filed
petition reviews the act of a constitutional organ; (7) petitioners rightly claim that they had no other plain,
speedy, and adequate remedy in the ordinary course of law; and (8) the petition includes questions that
are dictated by public welfare and the advancement of public policy, or demanded by the broader interest

Remedial Law Review II (CY 2018-2019) – Saturday 1:00pm – 5:00pm: Case Digest Compilation [Atty. Brondial] Page 70
of justice, or the orders complained of were found to be patent nullities, or the appeal was considered as
clearly an inappropriate remedy.

While this Court agrees with the JBC that the petitioner's preliminary remedy to question the rotational
arrangement of Congress is to ask the latter to repudiate the same, this, however, cannot be considered
plain, speedy and adequate. This Court is, thus, inclined to sustain the petitioner's direct resort to this
Court not only because it is the plain, speedy and adequate remedy available to him but also by reason of
the constitutional issues involved herein and the urgency of the matter. As correctly pointed out by the
OSG, the Constitution mandates that any vacancy to the office of an Associate Justice of the Supreme
Court must be filled up within the 90-day period from its occurrence. Therefore, the JBC must submit the
list of nominees prior to the start of that period. As the nominations covered by the questioned December
2016 JBC En Banc deliberations were intended for vacancies created by then Associate Justices Perez and
Brion, who respectively retired last December 14 and 29, 2016, hence, any resort to Congress during that
time would already be inadequate since the JBC list of nominees would be submitted any moment to the
Office of the President for the appointment of the next Associate Justices of the Supreme Court. Since
time is of the essence, the petitioner's direct resort to this Court is warranted.

2. No. It is essential to the issuance of a writ of mandamus that the applicant has a clear legal right to the
thing demanded and it must be the imperative duty of the respondent to perform the act required. The
burden is on the petitioner to show that there is such a clear legal right to the performance of the act, and a
corresponding compelling duty on the part of the respondent to perform the act. As an extraordinary
writ, it lies only to compel an officer to perform a ministerial duty, not a discretionary one. A clear
line demarcates a discretionary act from a ministerial one. A purely ministerial act is one which an officer
or tribunal performs in a given state of facts, in a prescribed manner, in obedience to the mandate of legal
authority, without regard to or the exercise of his own judgment upon the propriety or impropriety of the
act done. On the other hand, if the law imposes a duty upon a public officer and gives him the right to
decide how or when the duty shall be performed, such duty is discretionary and not ministerial. The duty
is ministerial only when the discharge of the same requires neither the exercise of official discretion or
judgment. Clearly, the use of discretion and the performance of a ministerial act are mutually exclusive.
Further, the writ of mandamus does not issue to control or review the exercise of discretion or to compel a
course of conduct.

In the case at bench, the counting of votes in the selection of the nominees to the judiciary may only be
considered a ministerial duty of the JBC if such votes were cast by its rightful members and not by
someone, like the petitioner, who is not considered a member during the En Banc deliberations last
December 2 and 9, 2016. For during the questioned period, the lawful representative of Congress to the
JBC is a member of the Senate and not of the House of Representatives as per their agreed rotational
scheme. Considering that a member of the Senate already cast his vote therein, the JBC has the full
discretion not to count the votes of the petitioner for it is mandated by both the Constitution and
jurisprudence to maintain that Congress will only have one representative in the JBC. As the act of the
JBC involves a discretionary one, accordingly, mandamus will not lie.

Remedial Law Review II (CY 2018-2019) – Saturday 1:00pm – 5:00pm: Case Digest Compilation [Atty. Brondial] Page 71
C. Review of Judgments and Final Orders of the COMELEC and CoA (Rule 64)

C.1. Alliance for Nationalism and Democracy (ANAD), petitioner vs. Commission on Elections
(COMELEC), respondent

G. R. No. 206987; September 10, 2013 ; Ponente: Perez, J. (En Banc)

~ The only question that may be raised in a petition for certiorari under Section 2, Rule 64 of the Rules of
Court is whether or not the COMELEC acted with grave abuse of discretion amounting to lack or excess
of jurisdiction. For a petition for certiorari to prosper, there must be a clear showing of caprice and
arbitrariness in the exercise of discretion.

FACTS: COMELEC En Banc promulgated a Resolution cancelling petitioner’s Certificate of


Registration and/or Accreditation.

ANAD went before the Supreme Court challenging the above-mentioned resolution. […] The Court
remanded the case to the COMELEC for re-evaluation in accordance with the parameters prescribed in
the aforesaid decision (Atong Paglaum, Inc. v. COMELEC).

COMELEC affirmed the cancellation of petitioner’s Certificate of Registration and/or Accreditation and
disqualified it from participating in the 2013 Elections.

Petitioners, again, went to the Supreme Court assailing that COMELEC gravely abused its discretion by
promulgating its Resolution cancelling petitioners Certificate of Registration and/or Accreditation and
disqualifying them from participating in the 2013 Elections.

ISSUE:
1. WON COMELEC gravely abused its discretion in promulgating the assailed Resolution
without the benefit of a summary evidentiary hearing mandated by the due process clause.

RULING:

1. NO.

The only question that may be raised in a petition for certiorari under Section 2, Rule 64 of the
Rules of Court is whether or not the COMELEC acted with grave abuse of discretion amounting to lack
or excess of jurisdiction. For a petition for certiorari to prosper, there must be a clear showing of caprice
and arbitrariness in the exercise of discretion.

“Grave abuse of discretion,” under Rule 65, has a specific meaning. Itis the arbitrary or despotic
exercise of power due to passion, prejudice orpersonal hostility; or the whimsical, arbitrary, or capricious
exercise ofpower that amounts to an evasion or a refusal to perform a positive dutyenjoined by law or to
act at all in contemplation of law. For an act to bestruck down as having been done with grave abuse of
discretion, the abuseof discretion must be patent and gross.

Herein, ANAD was already afforded a summary hearing, during which Mr. Domingo M. Balang,
ANAD’s president, authenticated documents and answered questions from the members of the
COMELEC pertinent to ANAD’s qualifications.

COMELEC need not have called another summary hearing. The Comelec could […] readily
resort to documents and other pieces of evidence previously submitted by petitioners in re-appraising
ANAD’s qualifications. After all, it can be presumed that the qualifications, or lack thereof, which were
established during the summary hearing continued until Election Day and even thereafter.

Remedial Law Review II (CY 2018-2019) – Saturday 1:00pm – 5:00pm: Case Digest Compilation [Atty. Brondial] Page 72
D. Certiorari, Prohibition and Mandamus

D.1.1. OSCAR R. AMPIL vs. THE HON. OFFICE OF THE OMBUDSMAN, et al.

G.R. No. 192685; July 31, 2013; Ponente: Perez, J.

FACTS: ASB Realty Corporation (ASB) and Malayan Insurance Company (MICO) entered into a Joint
Project Development Agreement (JPDA) for the construction of "The Malayan Tower." wherein the latter
shall provide the real property while former would construct and shoulder the cost of construction and
development of the condominium building. Due to financial difficulties, ASB was unable to perform its
obligations to MICO. Thus, MICO and ASB executed a Memorandum of Agreement (MOA), allowing
MICO to assume the entire responsibility for the development and completion of The Malayan Tower.
The MOA specifies the entitlement of both ASB and MICO to net saleable areas of The Malayan Tower
representing their investments.

On 11 March 2005, two sets of Condominium Certificates of Title (CCTs) were issued by Espenesin for
38 units and the allotted parking spaces in Malayan Tower. The first was in the name of MICO and the
second in the name of ASB. The second set of CCTs was issued upon the instruction of Serrano an officer
of MICO. Ampil, unsecured creditor of the ASB charged Espenesin with violation of Sections 3(a) and
(e) of Republic Act No. 3019 before the Office of the Ombudsman
.
Ombudsman dismissed Ampil’s complaint on the ground of lack of probable cause for the alleged
commission of falsification. Thereafter, Ampil filed a petition for review under Rule 43 of the Rules of
Court before the appellate court. And as already stated, the appellate court affirmed the Ombudsman’s
resolution.

ISSUE: Whether or not Ombudsman’s discretionary power to determine the existence of probable cause
may be assailed via petition for certiorari under Rule 65 of the Rules of Court

RULING: The Supreme Court have consistently hewed to the policy of non-interference with the
Ombudsman’s exercise of its constitutionally mandated powers. The Ombudsman’s finding to proceed or
desist in the prosecution of a criminal case can only be assailed through certiorari proceedings before this
Court on the ground that such determination is tainted with grave abuse of discretion which contemplates
an abuse so grave and so patent equivalent to lack or excess of jurisdiction.

However, on several occasions, the court have interfered with the Ombudsman’s discretion in
determining probable cause: (a) To afford protection to the constitutional rights of the accused; (b) When
necessary for the orderly administration of justice or to avoid oppression or multiplicity of actions; (c)
When there is a prejudicial question which is sub judice; (d) When the acts of the officer are without or
in excess of authority; (e) Where the prosecution is under an invalid law, ordinance or regulation; (f)
When double jeopardy is clearly apparent; (g) Where the court has no jurisdiction over the offense; (h)
Where it is a case of persecution rather than prosecution; (i) Where the charges are manifestly false and
motivated by the lust for vengeance.

The fourth circumstance is present in this case. Despite the admission by Espenesin that he had altered the
CCTs and the Ombudsman’s findings thereon, the Ombudsman abruptly dismissed Ampil’s complaint-
affidavit. A finding of probable cause needs only to rest on evidence showing that more likely than not a
crime has been committed and there is enough reason to believe that it was committed by the accused. It
need not be based on clear and convincing evidence of guilt, neither on evidence establishing absolute
certainty of guilt.

As Registrar of Deeds, Espenesin was duty bound to inquire and ascertain the reason for Serrano’s new
instruction on those specific set of CCTs and not just heed Serrano’s bidding. He heads the Office of
Register of Deeds which is constituted by law as "a public repository of records of instruments affecting
registered or unregistered lands x xx in the province or city wherein such office is situated." He should
not have so easily taken Serrano’s word that the amendment Serrano sought was to correct simple and
innocuous error.

Espenesin could have then easily asked, as he is obliged to, for a contract or an authenticated writing to
ascertain which units and parking slots were really allotted for ASB and MICO. His actions would then
be based on what is documented and not merely by a lame claim of bona fides mistake.

Remedial Law Review II (CY 2018-2019) – Saturday 1:00pm – 5:00pm: Case Digest Compilation [Atty. Brondial] Page 73
D.1.2 A.L. ANG NETWORK, INC., Petitioner, vs. EMMA MONDEJAR, accompanied by her
husband, EFREN MONDEJAR, Respondent

G.R. No. 200804; January 22, 2014

FACTS: On March 23, 2011, petitioner filed a complaint for sum of money under the Rule of Procedure
for Small Claims Cases before the MTCC, seeking to collect from respondent the amount of ₱23,111.71
which represented her unpaid water bills for the period June 1, 2002 to September 30, 2005.

Petitioner claimed that it was duly authorized to supply water to and collect payment therefor from the
homeowners of Regent Pearl Subdivision, one of whom is respondent who owns and occupies Lot 8,
Block 3 of said subdivision. From June 1, 2002 until September 30, 2005, respondent and her family
consumed a total of 1,150 cubic meters (cu. m.) of water, which upon application of the agreed rate of
₱113.00 for every 10 cu. m. of water, plus an additional charge of ₱11.60 for every additional cu. m. of
water, amounted to ₱28,580.09.8 However, respondent only paid the amount of ₱5,468.38, thus, leaving a
balance of ₱23,111.71 which was left unpaid despite petitioner’s repeated demands.

In defense, respondent contended that since April 1998 up to February 2003, she religiously paid
petitioner the agreed monthly flat rate of ₱75.00 for her water consumption. Notwithstanding their
agreement that the same would be adjusted only upon prior notice to the homeowners, petitioner
unilaterally charged her unreasonable and excessive adjustments (at the average of 40 cu. m. of water per
month or 1.3 cu. m. of water a day) far above the average daily water consumption for a household of
only 3 persons. She also questioned the propriety and/or basis of the aforesaid ₱23,111.71 claim.

In the interim, petitioner disconnected respondent’s water line for not paying the adjusted water charges
since March 2003 up to August 2005.

ISSUE: WON RTC is correct in denying petitioner’s recourse under Rule 65 assailing the MTCC
decision in the small claims case.

RULING: YES.Section 23 of the Rule of Procedure for Small Claims Cases states that:

SEC. 23.Decision. — After the hearing, the court shall render its decision on the same day, based on the
facts established by the. The decision shall immediately be entered by the Clerk of Court in the court
docket for civil cases and a copy thereof forthwith served on the parties.

The decision shall be final and unappealable.

Considering the final nature of a small claims case decision under the above-stated rule, the remedy of
appeal is not allowed, and the prevailing party may, thus, immediately move for its execution.
Nevertheless, the proscription on appeals in small claims cases, similar to other proceedings where appeal
is not an available remedy, does not preclude the aggrieved party from filing a petition for certiorari under
Rule 65 of the Rules of Court. This general rule has been enunciated in the case of Okada v. Security
Pacific Assurance Corporation, wherein it was held that:

In a long line of cases, the Court has consistently ruled that "the extraordinary writ of certiorari is always
available where there is no appeal or any other plain, speedy and adequate remedy in the ordinary course
of law."In Jaca v. Davao Lumber Co.In view of the foregoing, the Court thus finds that petitioner
correctly availed of the remedy of certiorari to assail the propriety of the MTCC Decision in the subject
small claims case, contrary to the RTC’s ruling.

Likewise, the Court finds that petitioner filed the said petition before the proper forum. To be sure, the
Court, the Court of Appeals and the Regional Trial Courts have concurrent jurisdiction to issue a writ of
certiorari. Such concurrence of jurisdiction, however, does not give a party unbridled freedom to choose
the venue of his action lest he ran afoul of the doctrine of hierarchy of courts. Instead, a becoming regard
for judicial hierarchy dictates that petitions for the issuance of writs of certiorari against first level courts
should be filed with the Regional Trial Court, and those against the latter, with the Court of Appeals,
before resort may be had before the Court. This procedure is also in consonance with Section 4, Rule 65
of the Rules of Court.

Hence, considering that small claims cases are exclusively within the jurisdiction of the Metropolitan
Trial Courts, Municipal Trial Courts in Cities, Municipal Trial Courts, and Municipal Circuit Trial

Remedial Law Review II (CY 2018-2019) – Saturday 1:00pm – 5:00pm: Case Digest Compilation [Atty. Brondial] Page 74
Courts, certiorari petitions assailing its dispositions should be filed before their corresponding Regional
Trial Courts. This petitioner complied with when it instituted its petition for certiorari before the RTC
which, as previously mentioned, has jurisdiction over the same. In fine, the RTC erred in dismissing the
said petition on the ground that it was an improper remedy, and, as such, RTC Case No. 11-13833 must
be reinstated and remanded thereto for its proper disposition.

Remedial Law Review II (CY 2018-2019) – Saturday 1:00pm – 5:00pm: Case Digest Compilation [Atty. Brondial] Page 75
D.1.3. MARK JEROME S. MAGLALANG, PETITIONER VS.PHILIPPINE AMUSEMENT AND
GAMING CORPORATION (PAGCOR), AS REPRESENTEDBY ITS INCUMBENT
CHAIRMAN EFRAIM GENUINO, RESPONDENT.

G.R. No. 190566; December 11, 2013

FACTS: Mark Maglalang was a teller at the Casino Filipino operated by PAGCOR. In December 2008,
he committed an error counting the money of a lady customer. Due to tension that arose between the two,
they were invited to the casino’s Internal Security Office in order to air their respective sides. He was
required to file an Incident Report. By January 2009, he was issued a memo charging him with
Discourtesy. He was later on found guilty of the same and 30-day suspension was imposed. He filed MR
seeking reversal of the decision and also Motion for Production to be furnished with documents relative
to the case. Both were denied. He then filed petition for certiorari under Rule 65 before the CA. He
ascribed grave abuse of discretion amounting to lack or excess of jurisdiction to the acts of PAGCOR in
adjudging him guilty of the charge, in failing to observe the proper procedure in the rendition of its
decision and in imposing the harsh penalty of a 30-day suspension. He further explained that he did not
appeal to the Civil Service Commission because the penalty imposed on him was only a 30-day
suspension which is not within the CSC’s appellate jurisdiction. CA outrightly dismissed the petition for
certiorari for being premature as petitioner failed to exhaust administrative remedies before seeking
recourse from the CA.

ISSUE: WON CA was correct in outrightly dismissing the petition for certiorari filed before it on the
ground of non-exhaustion of administrative remedies.

RULING: CA’s outright dismissal of the petition for certiorari on the basis of non-exhaustion of
administrative remedies is bereft of any legal standing

Under the doctrine of exhaustion of administrative remedies, before a party is allowed to seek the
intervention of the court, he or she should have availed himself or herself of all the means of
administrative processes afforded him or her.

The case falls squarely under exception “where no administrative review is provided by law” since the
law per se provides no administrative review for administrative cases whereby an employee like
petitioner is covered by Civil Service law, rules and regulations and penalized with a suspension for not
more than 30 days.
The judicial recourse petitioner availed of in this case before the CA is a special civil action for certiorari
ascribing grave abuse of discretion, amounting to lack or excess of jurisdiction on the part of PAGCOR,
not an appeal.

As there being no appeal or any plain, speedy, and adequate remedy in the ordinary course of law in view
of petitioner’s allegation that PAGCOR has acted without or in excess of jurisdiction, or with grave abuse
of discretion amounting to lack or excess of jurisdiction, the CA’s outright dismissal of the petition for
certiorari on the basis of non-exhaustion of administrative remedies is bereft of any legal standing and
should therefore be set aside.
Finally, as a rule, a petition for certiorari under Rule 65 is valid only when the question involved is an
error of jurisdiction, or when there is grave abuse of discretion amounting to lack or excess of jurisdiction
on the part of the court or tribunals exercising quasi-judicial functions. Occasionally, however, they are
constrained to wade into factual matters when the evidence on record does not support those factual
findings; or when too much is concluded, inferred or deduced from the bare or incomplete facts appearing
on record. Considering the circumstances and since this Court is not a trier of facts, remand of this case to
the CA for its judicious resolution is in order.

DOCTRINE: An appeal and a special civil action such as certiorari under Rule 65 are entirely distinct
and separate from each other. One cannot file petition for certiorari under Rule 65 of the Rules where
appeal is available, even if the ground availed of is grave abuse of discretion. A special civil action for
certiorari under Rule 65 lies only when there is no appeal, or plain, speedy and adequate remedy in the
ordinary course of law. Certiorari cannot be allowed when a party to a case fails to appeal a judgment
despite the availability of that remedy, as the same should not be a substitute for the lost remedy of
appeal. The remedies of appeal and certiorari are mutually exclusive and not alternative or successive.

Remedial Law Review II (CY 2018-2019) – Saturday 1:00pm – 5:00pm: Case Digest Compilation [Atty. Brondial] Page 76
D.1.4. PEOPLE OF THE PHILIPPINES, Petitioner, vs. THE HONORABLE JUANITO C.
CASTANEDA, JR., HONORABLE CAESAR A. CASANOVA, HONORABLE CIELITO N.
MINDARO-GRULLA, AS ASSOCIATE JUSTICES OF THE SPECIAL SECOND DIVISION,
COURT OF TAX APPEALS; and MYRNA M. GARCIA AND CUSTODIO MENDOZA
VESTIDAS, JR., Respondents.

G.R. No. 208290; December 11, 2013

FACTS: Myrna M. Garcia and Custodio Mendoza Vestidas, Jr. as owner/proprietress and broker of
Plinth Enterprise were charged with violation of the Tariff and Customs Code of the Philippines when
they imported into the Port of Manila, 858 cartons containing 17,160 pieces of Anti-Virus Software
Kaspersky Internet Security Premium 2012, and falsely declaring them to contain 40 pallets/1,690 cartons
of CD kit cleaner and plastic CD case, said imported items (Kaspersky) having customs duties amounting
to Three Million Three Hundred Forty One Thousand Two Hundred Forty Five Pesos (Php 3,341,245) of
which only the amount of One Hundred Thousand Three Hundred Sixty Two Pesos (Php100,362) was
paid.

Garcia and Vestidas, Jr. filed their Omnibus Motion to File Demurrer to Evidence with Leave of Court to
Cancel Hearing Scheduled on January 21, 2013,which was granted by the CTA. Thereafter, they filed the
Demurrer to Evidence, dated January 13, 2012, claiming that the prosecution failed to prove their guilt
beyond reasonable doubt.

On July 24, 2013, the Run After the Smugglers (RATS) Group, Revenue Collection Monitoring Group
(RCMG), as counsel for the BOC, received a copy of the July 15, 2013 Resolution of the CTA ordering
the entry of judgment in the case.

The BOC then filed a petition for certiorari, ascribing grave abuse of discretion on the part of the CTA.

At the outset, it should be noted that the petition was filed beyond the reglementary period for the filing
thereof under Rule 65. The petition itself stated that a copy of the May 15, 2013 Resolution was received
by the BOC two (2) days after its promulgation, or on May 17, 2013. Nonetheless, the RATS was only
alerted by the developments in the case on July 24, 2013, when Atty. Danilo M. Campos Jr. (Atty.
Campos) received the July 15, 2013 Resolution of the CTA ordering the entry of judgment in the case,
considering that no appeal was taken by any of the parties. According to Atty. Campos, it was only on
that occasion when he discovered the May 15, 2013 Resolution of the CTA.

ISSUE: Whether the petition for certiorari should be given due course despite its late filing.

RULING: No, Section 4, Rule 65 of the 1997 Rules of Civil Procedure is explicit in stating that certiorari
should be instituted within a period of 60 days from notice of the judgment, order or resolution sought to
be assailed. The 60-day period is inextendible to avoid any unreasonable delay that would violate the
constitutional rights of parties to a speedy disposition of their case. While there are recognized exceptions
to such strict observance, there should be an effort on the part of the party invoking liberality to advance a
reasonable or meritorious explanation for his/her failure to comply with the rules.

In the case at bench, no convincing justification for the belated filing of the petition was advanced to
warrant the relaxation of the Rules. Notably, the records show that the petition was filed only on August
12, 2013, or almost a month late from the due date which fell on July 16, 2013. To excuse this grave
procedural lapse will not only be unfair to the other party, but it will also sanction a seeming rudimentary
attempt to circumvent standing rules of procedure. Suffice it to say, the reasons proffered by the petitioner
do not carry even a tinge of merit that would deserve leniency. The late filing of the petition was borne
out of the petitioner’s failure to monitor incoming court processes that needed to be addressed by the
office. Clearly, this is an admission of inefficiency, if not lack of zeal, on the part of an office tasked to
effectively curb smuggling activities which rob the government of millions of revenue every year.

The display of patent violations of even the elementary rules leads the Court to suspect that the case
against Garcia and Vestidas Jr. was doomed by design from the start. The failure to present the certified
true copies of documentary evidence; the failure to competently and properly identify the misdeclared
goods; the failure to identify the accused in court; and, worse, the failure to file this petition on time
challenging a judgment of acquittal, are tell-tale signs of a reluctant and subdued attitude in pursuing the

Remedial Law Review II (CY 2018-2019) – Saturday 1:00pm – 5:00pm: Case Digest Compilation [Atty. Brondial] Page 77
case. This stance taken by the lawyers in government service rouses the Court’s vigilance against
inefficiency in the administration of justice.

Even the error committed by the RATS in filing a motion for reconsideration with the CTA displays gross
ignorance as to the effects of an acquittal in a criminal case and the constitutional proscription on double
jeopardy. Had the RATS been eager and keen in prosecuting the respondents, it would have, in the first
place, presented its evidence with the CTA in strict compliance with the Rules.

In any case, even if the Court decides to suspend the rules and permit this recourse, the end result would
remain the same. While a judgment of acquittal in a criminal case may be assailed in a petition for
certiorari under Rule 65 of the Rules of Court,it must be shown that there was grave abuse of discretion
amounting to lack or excess of jurisdiction or a denial of due process.In this case, a perusal of the
challenged resolutions of the CTA does not disclose any indication of grave abuse of discretion on its
partor denial of due process.The records are replete with indicators that the petitioner actively participated
during the trial and, in fact, presented its offer of evidence and opposed the demurrer.

Grave abuse of discretion is defined as capricious or whimsical exercise of judgment as is equivalent to


lack of jurisdiction. The abuse of discretion must be patent and gross as to amount to an evasion of a
positive duty or a virtual refusal to perform a duty enjoined by law, or to act at all in contemplation of
law, as where the power is exercised in an arbitrary and despotic manner by reason of passion and
hostility. Here, the subject resolutions of the CTA have been issued in accordance with the rules on
evidence and existing jurisprudence.

DOCTRINE: Section 4, Rule 65 of the 1997 Rules of Civil Procedure is explicit in stating that
certiorari should be instituted within a period of 60 days from notice of the judgment, order or resolution
sought to be assailed. The 60-day period is inextendible to avoid any unreasonable delay that would
violate the constitutional rights of parties to a speedy disposition of their case.

Remedial Law Review II (CY 2018-2019) – Saturday 1:00pm – 5:00pm: Case Digest Compilation [Atty. Brondial] Page 78
D.1.5. UP Board of Regents vs. Ligot-Teylan

G.R. No. 110280; October 12, 1993; Romero, J.

FACTS: Ramon P. Nadal, a student enrolled in the UP College of Law, applied forSocialized Tuition Fee
and Assistance Program (STFAP).

U.P. charged Nadal before the Student Disciplinary Tribunal (SDT). SDTrendered a decision finding him
guilty of "wilfully and deliberately withholdinginformation in his STFAP application about the income of
his mother, who isliving abroad, in support of the studies of his brothers Antonio and Federico,which is
tantamount to acts of dishonesty in relation to his studies.

As such, the SDT imposed upon Nadal the penalty of expulsion from theUniversity and required him to
reimburse all STFAP benefits he had receivedbut if he does not voluntarily make reimbursement. The
Executive Committeeaffirmed the decision of the SDT. Nadal appealed to the Board of Regents(BOR).
BOR affirmed the decision of the SDT. Nadal forthwith filed a motionfor reconsideration of the BOR
decision.

In the morning of March 29, 1993, the BOR found Nadal guilty and imposedupon him the penalties of
suspension for one (1) year effective March 29, 1993,non-issuance of any certificate of good moral
character during the suspensionand/or as long as Nadal has not reimbursed the STFAP benefits he had
receivedwith 12% interest per annum from march 30, 1993 and non-issuance of histranscript of records
until he has settled his financial obligations with theuniversity.

Nadal filed with the Regional Trial Court of Quezon City a petition formandamus with preliminary
injunction and prayer for a temporary restrainingorder against President Abueva, the BOR, Oscar M.
Alfonso, Cesar A.Buenaventura, Armand V. Fabella and Olivia C. Caoili.

Petitioners filed the instant petition for certiorari and prohibition with prayer forthe issuance of an
injunction and alleged that RTC judge gravely abused herdiscretion in issuing a writ of preliminary
injunction thereby preventing theBOR from implementing the suspension penalty it had imposed on
Nadal.

Private respondent opposed the petition and argued that Dr. Caoili, not havingbeen authorized by the
Board of Regents as a collegial body to file the instantpetition, and Dr. Abueva, who verified the petition,
not being the "Board ofRegents" nor "the University of the Philippines," they are not real parties
ininterest who should file the same.

ISSUE:
I. Whether or not a respondent to a petition for certiorari may assail thelegal standing of the
petitioner when the former had specifically named the latterin a petition for mandamus for which
the injunction was issued and is assailedin the certiorari petition.
II. Whether or notmandamus shall be applicable in this case.

RULING:
1) The answer is in the negative. A real party in interest is one "who stands to bebenefited or injured
by the judgment or the party entitled to the avails of thesuit. 'Interest' within the meaning of the
rule means material interest, an interestin issue and to be affected by the decree, as distinguished
from mere interest inthe question involved, or a mere incidental interest."

Undoubtedly, the U.P. Board of Regents has an interest to protect inasmuch aswhat is in issue here
is its power to impose disciplinary action against a studentwho violated the Rules and Regulations
on Student Conduct and Discipline bywithholding information in connection with his application
for STFAP benefits,which information, if disclosed, would have sufficed to disqualify him
fromreceiving the financial assistance he sought. Such dishonesty, if left unpunished,would have
the effect of subverting a commendable program into which theUniversity officials had devoted
much time and expended precious resources,from the conceptualization to the implementation
stage, to rationalize thesocialized scheme of tuition fee payments in order that more students
maybenefit from the public funds allocated to the State University.

Having specifically named Drs. Abueva and Caoili as respondents in thepetition for mandamus that he
filed below, Nadal is now estopped fromquestioning their personality to file the instant petition.

Remedial Law Review II (CY 2018-2019) – Saturday 1:00pm – 5:00pm: Case Digest Compilation [Atty. Brondial] Page 79
Moreover, under Sec.7 of the U.P. Charter (Act 1870) and Sec. 11 of the University Code "all
process"against the BOR shall be served on "the president or secretary thereof'." It is inaccordance
with these legal provisions that Dr. Caoili is named as a petitioner.Necessarily, Dr. Abueva, the
University President and member of the BOR, hasto verify the petition. It is not mandatory,
however, that each and every memberof the BOR be named petitioners. As the Court has time and
again held, anaction may be entertained, notwithstanding the failure to include anindispensable
party where it appears that the naming of the party would be buta formality. Mandamus is never
issued in doubtful cases, a showing of a clearand certain right on the part of the petitioner being
required. It is of no availagainst an official or government agency whose duty requires the exercise
ofdiscretion or judgment.

2) Mandamus is not applicable since UP exercise a valid academic freedom. SC reiterated the
doctrine in Arellano vs Cui that school has an academic freedom to whom they will accept in their
institution. UP has therefore the discretion in this case that the court should respect therefore
Mandamus shall not prosper.

Doctrine: A school, in its exercise academic freedom, has the right to choose whom to accept and whom
not to accept. Considering that such exercise of academic freedom is a discretionary function, the special
civil action for mandamus, which requires that the function of the respondent must be ministerial, cannot
be applied.

Remedial Law Review II (CY 2018-2019) – Saturday 1:00pm – 5:00pm: Case Digest Compilation [Atty. Brondial] Page 80
D.1.6. ROMAN C. TUASON and REMEDIOS V. TUASON, by attorney-in-fact Trinidad S.
Viado, petitioners, vs. REGISTER OF DEEDS, CALOOCAN City, MINISTRY OF JUSTICE, and
the NATIONAL TREASURER, respondents. TOMASA BARTOLOME, in her own behalf and in
behalf of the other members of the "Consuelo Heights Homeowners Association," petitioners-
intervenors.

G.R. No. 70484; January 29, 1988; NARVASA, J.

FACTS: Petitioner spouses, the Tuasons, were retired public school teachers. They bought a piece of
land in Caloocan from Carmel Farms8 yrs there after; they woke up one morning to discover that by
presidential flat, they were no longer the owners of the land.Their land and the other lots in the
subdivision had been "declared open for disposition and sale to the members of the Malacanang
Homeowners Association, Inc., the present bona fide occupants thereof." A year after the declaration of
martial law, Marcos sued PD 293 with immediate effect. The decree invalidated inter alia the title of the
Tuasons' vendor, Carmel Farms, which had earlier purchased from the Government the land. The land
bought by Carmel Farms was part of the Tala Estate (one of the so-called "Friar Lands"). Carmel had
bought the land under Act No. 1120 and C.A. No. 32, as amended.

Presidential Decree No. 293 made the findingthat Carmel had failed to complete payment of the price.
According to the records of the Bureau of Lands, neither the original purchasers nor their subsequent
transferees have made full payment of all installments of the purchase money and interest on the lots
claimed by the Carmel Farms, Inc., including those on which the dwellings of the members of said
Association stand. Hence, title to said land has remained with the Government, and the land  now
occupied by the members of said association   has never ceased to form part of the property of the
Republic of the Philippines, any and all acts affecting said land and purporting to segregate it from the
said property of the Republic of the Philippines being therefore null and void  ab initio  as against the law
and public policy.

Upon this adjudgment, Mr. Marcos:


a. Invalidated the titles of Carmel Farms, Inc. and all those derived therefrom, and
b. Declared as aforestated "the members of the Malacanang Homeowners Association, Inc.
the present bona fide occupants" of the lots 

It seems to have completely escaped Mr. Marcos' attention that his decree contained contradictory
declarations. While acknowledging on the one hand that the lots in the Carmel Subdivision  were
occupied by the buyers thereof, and in fact the latter's dwellings stood thereon, he states on the other that
the "members of the Malacanang Homeowners Association, Inc. (are) the present bona fide occupants" of
all said lots. On the strength of this presidential decree, the Register of Deeds of Caloocan City caused
the inscription on the Tuasons' title, TCT No. 8314.

SC: The Tuason Spouses thereupon filed a petition for certiorari assailing the Marcos decree as an
arbitrary measure which deprived them of their property in favor of a selected group, in violation of the:
a. Constitutional provisions on due process and eminent domain; and
b. Provisions of the Land Registration Act on the indefeasibility of Torrens titles; 

ISSUE: W/N Certiorari under Rule 65 was proper.

RULING: YES. It is true that the extraodinary writ of certiorari  may properly issue to nullify
only judicial or quasi-judicial acts, unlike the writ of prohibition which may be directed against acts
either judicial or ministerial. Section 1, Rule 65 of the Rules of Court deals with the writ of certiorari in
relation to "any tribunal, board or officer exercising judicial functions, while Section 2 of the same Rule
treats of the writ of prohibition in relation to "proceedings of any tribunal, corporation, board, or person ...
exercising functions judicial or ministerial." But the petition will be shown upon analysis to be in reality
directed against an unlawful exercise of judicial power.

The decree reveals that Mr. Marcos exercised an obviously judicial function. He made a determination
of facts, and applied the law to those facts, declaring what the legal rights of the parties were in the
premises. These acts essentially constitute a judicial function,  or an exercise of jurisdiction — which is
the power and authority to hear or try and decide or determine a cause.  He adjudged it to be an
established fact that neither the original purchasers nor their subsequent transferees have made full
payment of all installments of the purchase money and interest on the lots claimed by Carmel Farms, Inc.,
including those on which the dwellings of the members of ... (the) Association (of homeowners) stand."

Remedial Law Review II (CY 2018-2019) – Saturday 1:00pm – 5:00pm: Case Digest Compilation [Atty. Brondial] Page 81
And applying the law to that situation, he made the adjudication that "title to said land has remained with
the Government, and the land now occupied by the members of said association has never ceased to form
part of the property of the Republic of the Philippines," and that 'any and all acts affecting said land and
purporting to segregate it from the said property of the Republic ... (were) null and void ab initio as
against the law and public policy.

These acts may thus be properly struck down by the writ of certiorari, because done by an officer in
the performance of what in essence is a judicial function, if it be shown that the acts were done without or
in excess of jurisdiction, or with grave abuse of discretion. Since Mr. Marcos was never vested with
judicial power, such power, as everyone knows, being vested in the Supreme Court and such
inferior courts as may be established by law  — the judicial acts done by him were in the
circumstances indisputably perpetrated without jurisdiction. The acts were completely alien to his
office as chief executive, and utterly beyond the permissible scope of the legislative power that he had
assumed as head of the martial law regime.

Moreover, he had assumed to exercise power — i.e. determined the relevant facts and applied the law
thereto without a trial at which all interested parties were accorded the opportunity to adduce evidence to
furnish the basis for a determination of the facts material to the controversy. He made the finding
ostensibly on the basis of "the records of the Bureau of Lands." Prescinding from the fact that there is no
indication whatever the nature and reliability of these records and that they are in no sense conclusive, it
is undeniable that the petitioner Tuasons (and the petitioners in intervention) were never confronted with
those records and afforded a chance to dispute their trustworthiness and present countervailing evidence.
This is yet another fatal defect. The adjudication was patently and grossly violative of the right to due
process to which the petitioners are entitled in virtue of the Constitution. Mr. Marcos, in other words, not
only arrogated unto himself a power never granted to him by the Constitution or the laws but had in
addition exercised it unconstitutionally.

In any event, this Court has it in its power to treat the petition for certiorari as one for prohibition if
the averments of the former sufficiently made out a case for the latter.  Considered in this wise, it
will also appear that an executive officer had acted without jurisdiction — exercised judicial power not
granted to him by the Constitution or the laws — and had furthermore performed the act in violation of
the constitutional rights of the parties thereby affected. The Court will grant such relief as may be proper
and efficacious in the premises even if not specifically sought or set out in the prayer of the appropriate
pleading, the permissible relief being determined after all not by the prayer but by the basic averments of
the parties' pleadings. 

WHEREFORE, Presidential Decree No. 293 is declared to be unconstitutional and void ab initio.

Separate Opinion: I concur fully in the main opinion depicting the unparalleled "despotic, capricious,
oppressive and unjustifiable exercise of government power" by the deposed President Ferdinand E.
Marcos, as struck down by the Court's unanimous judgment in the case at bar. These arbitrary, capricious
and oppressive decrees, tailored to suit the deposed President's every wish and whim, were the product of
unrestrained power, as the deposed President took over the entire government with the imposition of
martial

DOCTRINE:Certiorari, in reality, is directed against an unlawful exercise of power. It is proper when


the doneby an officer in the performance of what in essence is a judicial function, if it be shown that the
acts were done without or in excess of jurisdiction, or with grave abuse of discretion. Certiorari may also
be treated as a Prohibition if the averments are sufficient.

Remedial Law Review II (CY 2018-2019) – Saturday 1:00pm – 5:00pm: Case Digest Compilation [Atty. Brondial] Page 82
D.1.7. Province of Leyte versus Energy Development Corporation (EDC)

G.R. No. 203124; June 22, 2015; Perlas-Bernabe, J.

FACTS: The Province of Leyte issued separate franchise tax assessments against EDC which the latter,
in turn, protested separately. When the Province of Leyte effectively denied all protests, EDC appealed
such denials with the RTC of Tacloban City. Notwithstanding the pendency of the cases with the RTC,
the Province of Leyte continued to issued franchise tax assessments against EDC which prompted the
latter to file a Motion for Issuance of Writ of Preliminary Injunction praying that the Province of Leyte be
ordered to enjoin from further assessing and collecting franchise taxes against EDC until the RTC shall
have resolved the cases with finality. In support thereof, EDC averred that it does not have a franchise,
thus, the Province of Leyte’s assessment of franchise taxes against it is contrary to law. It further claimed
that should the Province of Leyte’s action continue, EDC’s operations will cease resulting in the losses of
substantial revenues and jobs for its employees. Also, the damage that it stands to suffer from the
Province of Leyte’s acts is irreparable as there is no assurance that it will be able to recover such losses.

The RTC initially denied the foregoing motion for preliminary injunction but on motion of EDC, the RTC
issued another order granting the issuance of writ of preliminary injunction ratiocinating that in case EDC
will be held liable for the said franchise taxes, the injunction bond would answer for the claim of the
Province of Leyte whereas if EDC is compelled to pay such taxes pending resolution of the cases with the
RTC and is subsequently adjudged to be not liable to pay the same, there is no assurance that it could
recover its operational losses. This, prompted the Province of Leyte to file a Petition for Certiorari under
Rule 65 with the CA.

The CA, then, dismissed the Petition for Certiorari on the ground that “there was no proper proof of
service of the Petition to the adverse party” as the registry receipts can hardly be considered as sufficient
and proper proof of receipt by the addressee of registered mail. Hence, this Petition for Review on
Certiorari under Rule 45 by the Province of Leyte.

ISSUE: Whether or not the CA is correct in dismissing the Petition for Certiorari on the ground that there
was no sufficient proof of service of the Petition to the private respondent.

RULING: NO. The instant case was elevated to the CA via a petition for certiorari which is, by nature,
an original and independent action, and therefore, not considered as part of the trial that had resulted in
the rendition of the judgment or order complained of. Being an original action, there is a need for the CA
to acquire jurisdiction over the person of the parties to the case before it can resolve the same on the
merits. Thus, the CA acquired jurisdiction over the person of the petitioner (Province of Leyte) upon the
filing of the certiorari petition. On the other hand, the CA acquires jurisdiction over the person of the
respondent upon: (a) the service of the order or resolution indicating the CA's initial action on the petition
to the respondent; or (b) the voluntary submission of the respondent to the CA's jurisdiction.

In the case at bar, records reveal that the CA served its Resolution indicating its initial action on the
Province of Leyte's certiorari petition before it, i.e., directing EDC to file a comment to the petition,
among others. In fact, the EDC complied with such directive by filing its comment to such petition. When
EDC filed its comment to the Petition for Certiorari, it voluntarily submitted itself to the jurisdiction of
the CA. Thus, the CA had already acquired jurisdiction over both parties to the instant case.

Hence, the Petition for Review is GRANTED. Accordingly, the pertinent Resolutions of the CA are
hereby REVERSED and SET ASIDE.

Remedial Law Review II (CY 2018-2019) – Saturday 1:00pm – 5:00pm: Case Digest Compilation [Atty. Brondial] Page 83
D.1.8. GIL G. CAWAD, MARIO BENEDICT P. GALON, DOMINGO E. LUSAYA, JEAN V.
APOLINARES, MA. LUISA S. OREZCA, JULIO R. GARCIA, NESTOR M. INTIA, RUBEN C.
CALIWATAN, ADOLFO Q. ROSALES, MA. LUISA NAVARRO, and the PHILIPPINE PUBLIC
HEALTH ASSOCIATION, INC. vs.FLORENCIO B. ABAD, in his capacity as Secretary of the
Department of Budget and Management (DBM); ENRIQUE T. ONA, in his capacity as Secretary
of the Department of Health (DOH); and FRANCISCO T. DUQUE III, in his capacity as Chairman
of the Civil Service Commission (CSC)

G.R. No. 207145; July 28, 2015; PERALTA, J.:

FACTS: Petitioner, Philippine Public Health Association, Inc., filed a petition for certiorari against the
Secretaries of the Department of Budget and Management& of the Department of Health, and the
Chairman of the Civil Service Commission, alleging that the latter group acted with grave abuse of
discretion upon the issuance of two Joint Circulars prescribing certain requirements on the grant of
benefits that are not required bythe Magna Carta of Public Health Workers,. Among others, petitioners
claim that the subject circulars are void for being an undue exercise of legislative power by the said
administrative bodies.

Through the Solicitor General, the respondents commented that the assailed circulars were issued within
the scope of their authority, and are therefore valid and binding. Moreover, the respondents controverted
that the remedies of Certiorari and Prohibition are unavailing because the assailed circulars were done in
the exercise of their quasi-legislative, and not of their judicial or quasi-judicial functions.

ISSUES: W/N the circulars are void for being an undue exercise of legislative or quasi-legislative
function.

W/N petition for certiorari is the correct remedy against an undue exercise of legislative or quasi-
legislative function.

RULING: No, the circulars are not void because theissuance and enforcement by the Secretaries of the
DBM, CSC and DOH of the questioned joint circulars were done in the exercise of their quasi-legislative
and administrative functions. It was in the nature of subordinate legislation, promulgated by them in
their exercise of delegated power. Quasi-legislative power is exercised by administrative agencies
through the promulgation of rules and regulations within the confines of the granting statute and the
doctrine of non-delegation of powers from the separation of the branches of the government

In the same vein, a petition for certiorari is not the proper remedy against an undue exercise of legislative
or quasi-legislative function. Petitions for certiorari and prohibition may be invoked only against
tribunals, corporations, boards, officers, or persons exercising judicial, quasi-judicial or ministerial
functions, and not against their exercise of legislative or quasi-legislative functions.

In this case, respondents did not act in any judicial, quasi-judicial, or ministerial capacity in their
issuance of the assailed joint circulars.In issuing and implementing the subject circulars, respondents
were not called upon to adjudicate the rights of contending parties to exercise, in any manner, discretion
of a judicial nature.

Be that as it may, for proper guidance, the Supreme Court still proceeded with the resolution of the
Circulars’ validity as to its substance whether or not the same are contrary to law.

DOCTRINE: It is beyond the province of certiorari to declare administrative issuances illegal because
petitions for certiorari seek solely to correct defects in jurisdiction, anderrors committed by a court,
board, or officer exercising judicial or quasi-judicial functionswithout or in excess of jurisdiction, or with
grave abuse of discretion amounting to lack of jurisdiction.

It is likewise beyond the territory of a writ of prohibition to declare administrative issuances illegal
becausethe purpose of the same is to keep a lower court within the limits of its jurisdiction in order to
maintain the administration of justice in orderly channels. It affords relief against usurpation of
jurisdiction by an inferior court; or when, in the exercise of jurisdiction, the inferior court transgresses
the bounds prescribed by the law, and/or where there is no adequate remedy available in the ordinary
course of law.

Remedial Law Review II (CY 2018-2019) – Saturday 1:00pm – 5:00pm: Case Digest Compilation [Atty. Brondial] Page 84
PLEASE DISREGARD THIS PAGE FOR SYLLABUS PURPOSES although you may also read it, it
may help in some way.

Thus, on the one hand, certiorari as a special civil action is available only if: (1) it is directed against a
tribunal, board, or officer exercising judicial or quasi-judicial functions; (2) the tribunal, board, or
officer acted without or in excess of jurisdiction, or with grave abuse of discretion amounting to lack or
excess of jurisdiction; and (3) there is no appeal nor any plain, speedy, and adequate remedy in the
ordinary course of law.11

On the other hand, prohibition is available only if: (1) it is directed against a tribunal, corporation,
board, officer, or person exercising functions, judicial, quasi-judicial, or ministerial; (2) the tribunal,
corporation, board or person acted without or in excess of its jurisdiction, or with grave abuse of
discretion amounting to lack or excess of jurisdiction; and (3) there is no appeal or any other plain,
speedy, and adequate remedy in the ordinary course of law.

Judicial functions involve the power to determine what the law is and what the legal rights of the parties
are, and then undertaking to determine these questions and adjudicate upon the rights of the parties.Quasi
judicial functions apply to the actions and discretion of public administrative officers or bodies required
to investigate facts, hold hearings, and draw conclusions from them as a basis for their official action , in
their exercise of discretion of a judicial nature.Ministerial functionsare those which an officer or tribunal
performs in the context of a given set of facts, in a prescribed manner and without regard to the exercise
of his own judgment upon the propriety or impropriety of the act done.

Before a tribunal, board, or officer may exercise judicial or quasi-judicial acts, it is necessary that there be
a law that gives rise to some specific rights under which adverse claims are made, and the controversy
ensuing therefrom is brought before a tribunal, board, or officer clothed with authority to determine the
law and adjudicate the respective rights of the contending parties.

Remedial Law Review II (CY 2018-2019) – Saturday 1:00pm – 5:00pm: Case Digest Compilation [Atty. Brondial] Page 85
D.2.1. ALFEO D. VIVAS vs. THE MONETARY BOARD OF THE BANGKO
SENTRAL NG PILIPINAS AND THE PHILIPPINE DEPOSIT
INSURANCE CORPORATION

G.R. No. 191424, August 7, 2013

FACTS: On March 4, 2010, the MB of BSP issued Resolution No. 27623 placing Eurocredit Bank under
receivership. Assailing MB Resolution No. 276, Vivas (member ofEurocredit Bank new management
team) filed this petition for prohibitionbefore this Court, ascribing grave abuse of discretion to the MB for
prohibitingEurocredit Bank from continuing its banking business and for placing it underreceivership.

ISSUE: Whether a petition for prohibition is the proper remedy to assail an order of the BSP to place a
bank under receivership.

RULING: NO. The answer is in the negative. Vivas Availed of the Wrong Remedy.To begin with,
Vivas availed of the wrong remedy. The MB issued ResolutionNo. 276, dated March 4, 2010, in the
exercise of its power under R.A. No. 7653.

Under Section 30 thereof, any act of the MB placing a bank underconservatorship, receivership or
liquidation may not be restrained or set asideexcept on a petition for certiorari. Pertinent portions of
Section 30, R.A. 7653read: x x x x The actions of the Monetary Board taken under this section orunder
Section 29 of this Act shall be final and executory, and may not berestrained or set aside by the court
except on petition for certiorari on theground that the action taken was in excess of jurisdiction or with
such graveabuse of discretion as to amount to lack or excess of jurisdiction. The petitionfor certiorari
may only be filed by the stockholders of record representing themajority of the capital stock within ten
(10) days from receipt by the board ofdirectors of the institution of the order directing receivership,
liquidation or conservatorship.

Prohibition is already unavailing. Granting that a petition for prohibition isallowed, it is already an
ineffective remedy under the circumstances obtaining.

Prohibition or a "writ of prohibition" is that process by which a superior court prevents inferior courts,
tribunals, officers, or persons from usurping orexercising a jurisdiction with which they have not been
vested by law, andconfines them to the exercise of those powers legally conferred. Its office is torestrain
subordinate courts, tribunals or persons from exercising jurisdictionover matters not within its cognizance
or exceeding its jurisdiction in matters ofwhich it has cognizance.

Indeed, prohibition is a preventive remedy seeking that a judgment be rendered which would direct the
defendant to desist from continuing with the commissionof an act perceived to be illegal. As a rule, the
proper function of a writ ofprohibition is to prevent the doing of an act which is about to be done. It
isnot intended to provide a remedy for acts already accomplished.

Though couched in imprecise terms, this petition for prohibition apparently seeks to prevent the acts of
closing of ECBI and placing it under receivership.Resolution No. 276, however, had already been issued
by the MB and theclosure of ECBI and its placement under receivership by the PDIC were already
accomplished. Apparently, the remedy of prohibition is no longer appropriate.

Settled is the rule that prohibition does not lie to restrain an act that is already a fait accompli.

Remedial Law Review II (CY 2018-2019) – Saturday 1:00pm – 5:00pm: Case Digest Compilation [Atty. Brondial] Page 86
D.2.2. ROSENDO R. CORALES, IN HIS OFFICIAL CAPACITY AS MUNICIPAL MAYOR OF
NAGCARLAN, LAGUNA, AND DR. RODOLFO R. ANGELES, IN HIS OFFICIAL CAPACITY
AS MUNICIPAL ADMINISTRATOR OF NAGCARLAN, LAGUNA, PETITIONERS, vs.
REPUBLIC OF THE PHILIPPINES, REPRESENTED BY THE COMMISSION ON AUDIT, AS
REPRESENTED BY PROVINCIAL STATE AUDITOR OF LAGUNA MAXIMO L.
ANDAL, RESPONDENT.

G.R. No. 186613; August 27, 2013; PEREZ, J.

FACTS: Petitioner Corales was the duly elected Municipal Mayor of Nagcarlan, Laguna for three (3)
consecutive terms. In his first term as local chief executive, petitioner Corales appointed petitioner Dr.
Angeles to the position of Municipal Administrator, whose appointment was unanimously approved by
the Sangguniang Bayan of Nagcarlan, Laguna (Sangguniang Bayan). During his second and third terms
as municipal mayor, petitioner Corales renewed the appointment of petitioner Dr. Angeles. But, on these
times, the Sangguniang Bayan disapproved petitioner Dr. Angeles’ appointment on the ground of
nepotism, as well as the latter’s purported unfitness and unsatisfactory performance. Even so, petitioner
Dr. Angeles continued to discharge the functions and duties of a Municipal Administrator.

MaximoAndal (Andal), the Provincial State Auditor of Laguna, issued an Audit Observation
Memorandum (AOM) to petitioner Corales who was asked to comment/reply. The aforesaid AOM states
it is not the Municipality of Nagcarlan that should be made liable to pay for petitioner Dr. Angeles’
salary; instead, it is petitioner Corales, being the appointing authority, that it is recommended that an
appropriate Notice of Disallowance be issued for the payment of salary expenses incurred without legal
basis by the Municipality of Nagcarlan in the aforestated amount.

Instead of submitting his comment/reply thereon, petitioner Corales, together with petitioner Dr. Angeles,
opted to file a Petition for Prohibition and Mandamus against Andal and the then members of the
Sangguniang Bayan before the RTC of San Pablo City, Laguna. Petitioners sought, by way of prohibition,
to require the Office of the Provincial Auditor, through Andal, to recall its AOM and to eventually desist
from collecting reimbursement from petitioner Corales for the salaries paid to and received by petitioner
Dr. Angeles for the latter’s services as Municipal Administrator. Petitioners similarly sought, by way of
mandamus, to compel the then members of the Sangguniang Bayan, as a collegial body, to recall its
Resolutions denying confirmation to petitioner Dr. Angeles’ appointment as Municipal Administrator and
in their stead to confirm the validity and legitimacy of such appointment.

The (OSG), on Andal’s behalf, filed a Motion to Dismiss petitioners’ Petition for Prohibition and
Mandamus grounded on lack of cause of action, prematurity and non-exhaustion of administrative
remedies. It was specifically contended therein that: (1) the issuance of the AOM was merely an initiatory
step in the administrative investigation of the Commission on Audit (COA) to allow petitioner Corales to
controvert the findings and conclusions of the Sangguniang Bayan; (2) it was only after the completion of
the said investigation that a resolution will be issued as regards the propriety of the disbursements made
by the Municipality of Nagcarlan in the form of salaries paid to petitioner Dr. Angeles during his tenure
as Municipal Administrator; and (3) instead of resorting to judicial action, petitioner Corales should have
first responded to the AOM and, in the event of an adverse decision against him, elevate the matter for
review to a higher authorities in the COA. With these, petitioners’ petition should be dismissed, as
petitioner Corales has no cause of action against Andal - his resort to judicial intervention is premature
and he even failed to avail himself of, much less exhaust, the administrative remedies available to him.

The RTC denied the said Motion to Dismiss on the ground that Andal was merely a nominal party. The
subsequent motion for its reconsideration was also denied. Respondent Republic, as represented by COA,
as represented by Andal, consequently filed a Petition for Certiorari with the Court of Appeals. The Court
of Appeals rendered its now assailed Decision granting respondent’s Petition for Certiorari, thereby
annulling and setting aside the RTC Orders dismissing petitioners’ Petition for Prohibition with the court
a quo.
Hence, this petition.

ISSUE: Whether or not the Petition for Prohibition was the proper remedy to assail the Audit
Observation Memorandum

RULING: NO. Settled is the rule that for the courts to exercise the power of judicial review, the
following must be extant: (1) there must be an actual case calling for the exercise of judicial power; (2)
the question must be ripe for adjudication; and (3) the person challenging must have the "standing." An

Remedial Law Review II (CY 2018-2019) – Saturday 1:00pm – 5:00pm: Case Digest Compilation [Atty. Brondial] Page 87
actual case or controversy involves a conflict of legal rights, an assertion of opposite legal claims,
susceptible of judicial resolution as distinguished from a mere hypothetical or abstract difference or
dispute.

Moreover, prohibition, being a preventive remedy to seek a judgment ordering the defendant to
desist from continuing with the commission of an act perceived to be illegal, may only be resorted to
when there is "no appeal or any other plain, speedy, and adequate remedy in the ordinary course of
law."

In this case, petitioners insist that it is no longer necessary to exhaust administrative remedies considering
that there is no appeal or any other plain, speedy and appropriate remedial measure to assail the
imposition under the AOM aside from an action for prohibition.
Petitioners’ action for prohibition was premature. The audit investigative process was still in its initial
phase. There was yet no Notice of Disallowance issued. And, even granting that the AOM issued to
petitioner Corales is already equivalent to an order, decision or resolution of the Auditor or that such
AOM is already tantamount to a directive for petitioner Corales to reimburse the salaries paid to
petitioner Dr. Angeles, still, the action for prohibition is premature since there are still many
administrative remedies available to petitioners to contest the said AOM.

WHEREFORE, premises considered, the Decision and Resolution dated 15 September 2008 and 20
February 2009, respectively, of the Court of Appeals in CA-G.R. SP No. 101296 are hereby
AFFIRMED. Costs against petitioners.

Remedial Law Review II (CY 2018-2019) – Saturday 1:00pm – 5:00pm: Case Digest Compilation [Atty. Brondial] Page 88
D.2.3. Tan vs. Court of Appeals

G.R. No. 164966; June 8, 2007; YNARES-SANTIAGO, J.

FACTS: In 2002, James L. King (King) charged Roderick Lim-Go, et al. with violation B.P. 22 and
Estafa involving two checks both dated June 21, 2002.Subsequently, King filed a Supplemental
Complaint-Affidavit involving five additional checks. King filed a Second Supplemental Complaint-
Affidavit for Estafa impleading Grace Tan-Go, and herein petitioners Rolando Tan, Elena Tan, and
Lamberto Tan, as additional respondents.

King averred that in 2002, the spouses Go, together with herein petitioners Rolando Tan (father of Grace
Tan-Go), Elena Tan (mother of Grace Tan-Go), asked ₱100 Million from King allegedly for the
renovation of their movie houses in Butuan City. However, King could only accommodate ₱40 Million,
in exchange for which, Roderick Go issued several checks to King in the amount of ₱61.28 Million,
inclusive of the interest for three months.

At first, the checks issued by Go were honored by the drawee bank when presented. However, when
several of the checks he issued were about to fall due, Roderick Go requested King for a meeting. While
at the agreed meeting place, Roderick Go allegedly attacked King with a box cutter and told him that all
the checks that he issued would be dishonored and for this reason he had to injure, kidnap and kill him.
Thereafter, all the checks dated June 21, 23 and 24, 2002 issued by Roderick Go were dishonored for
having been drawn against insufficient funds. Despite repeated demands, no payment was made; hence,
King filed a complaint for violation of BP Blg. 22 and Estafa.

On November 11, 2002, five informations for estafa were filed against Roderick L. Go et al., and herein
petitioners, and was raffled to the RTC, Branch 5 of Cebu City.On November 18, 2002, before any
warrant of arrest could be issued, petitioners posted bail. The following day, they were arraigned and
pleaded not guilty.

On December 17, 2002, petitioners filed a Petition for Prohibition and Injunction with Preliminary
Injunction and Prayer for Temporary Restraining Order before the Court of Appeals. They sought to
restrain the trial court from proceeding with the subject criminal cases against them and prayed that the
same be dismissed.
On November 24, 2003, the Court of Appeals issued the assailed Decision dismissing the petition for lack
of merit. It found, among others, that: petitioners failed to avail themselves of other plain, speedy and
adequate remedies to challenge the public prosecutor’s finding of probable cause; the petition failed to
establish that it falls under any of the exceptions to the general rule that the court will not issue writs of
prohibition or injunction, preliminary or final, to enjoin or restrain a criminal prosecution; and petitioners’
claims contesting the public prosecutor’s finding of probable cause are matters of defense that should be
threshed out during the trial of the criminal cases and not through the extraordinary remedy of
prohibition.

ISSUE: Whether petitioners’ resort to the extraordinary remedy of prohibition was proper.

RULING: No. Basic is the rule that the writ of prohibition is an extraordinary remedy to prevent the
unlawful and oppressive exercise of legal authority and to provide for a fair and orderly administration of
justice. It is available only when there is no appeal or any plain, speedy and adequate remedy in the
ordinary course of law, and when the proceedings are done without or in excess of jurisdiction or with
grave abuse of discretion. The petitioner must allege in his petition and establish facts to show that any
other existing remedy is not speedy or adequate. A remedy is plain, speedy and adequate if it will
promptly relieve the petitioner from the injurious effects of that judgment and the acts of the tribunal or
inferior court. 

Further, the writ will not lie to correct errors of judgment but only errors of jurisdiction. As long as the
tribunal acts within its jurisdiction, any alleged errors committed in the exercise of its discretion will
amount to nothing more than mere errors of judgment which are correctible by a timely appeal. In
determining whether a tribunal acted in grave abuse of discretion, mere abuse of discretion is not enough.
There must be grave abuse of discretion as where the tribunal exercised its power in an arbitrary or
despotic manner, by reason of passion or personal hostility, and it must be so patent or gross as would
amount to an evasion, or virtual refusal to perform the duty enjoined, or to act in contemplation of law.

Remedial Law Review II (CY 2018-2019) – Saturday 1:00pm – 5:00pm: Case Digest Compilation [Atty. Brondial] Page 89
Petitioners admitthat they received a copy of the Joint Resolution dated November 8, 2002 as early as
November 13, 2002. However, from the time they received the copy of the aforesaid Resolution to the
time they were arraigned on November 19, 2002, petitioners did not take steps to move for
reconsideration, or appeal the aforesaid Resolution to the Secretary of Justice.

In fine, the arguments raised in their petition for prohibition ineluctably shows that petitioners are
principally questioning the factual and legal bases of the finding of probable cause against them. This is
but a veiled attempt to litigate issues which should have been timely appealed to the Secretary of Justice
via a petition for review. However, petitioners, through their own fault, failed to avail themselves of this
remedy. Countless times we have ruled that the extraordinary remedy of certiorari or prohibition is not a
substitute for a lost appeal. This case is no different.

DOCTRINE: Basic is the rule that the writ of prohibition is an extraordinary remedy to prevent the
unlawful and oppressive exercise of legal authority and to provide for a fair and orderly administration
of justice. It is available only when there is no appeal or any plain, speedy and adequate remedy in the
ordinary course of law, and when the proceedings are done without or in excess of jurisdiction or with
grave abuse of discretion.

Remedial Law Review II (CY 2018-2019) – Saturday 1:00pm – 5:00pm: Case Digest Compilation [Atty. Brondial] Page 90
D.3.1. Nilo Hipos Sr. et al vs. Hon. RTC Judge Teodoro A. Bay

G.R. No. 174813-15; March 17, 2009; J. Chico-Nazario

FACTS: On 15 December 2003, two Informations for the crime of rape and one Information for the
crime of acts of lasciviousness were filed against petitioners Darryl Hipos, Jaycee Corsio, Arthur
Villaruel and two others before Branch 86 of the Regional Trial Court of Quezon City, acting as a Family
Court, presided by respondent Judge Bay.  The Informations were signed by Assistant City Prosecutor
Ronald C. Torralba.

The private complainants filed a Motion for Reinvestigation asking Judge Bay to order the City
Prosecutor of Quezon City to study if the proper Informations had been filed against petitioners and their
co-accused. Judge Bay granted the Motion and ordered a reinvestigation of the cases.

On August 2004, the Office of the City Prosecutor issued a Resolution on the reinvestigation affirming
the Informations filed against petitioners and their co-accused in Criminal Cases. However,the Public
Prosecutor treated the Joint Memorandum to Dismiss the Case filed by accused as an appeal of the  10
August 2004 Resolution, thus resolution was reversed, holding that there was lack of probable cause.  On
the same date, the City Prosecutor filed a Motion to Withdraw Informations before Judge Bay.

But, Judge Bay denied the Motion to Withdraw Informations in an Order of even date. thus prompted the
petitioners to file the case of mandamus.

ISSUE: Whether or not SC may compel Judge Bay to dismiss the case through writ of mandamus by
virtue of a resolution by public prosecutor’s finding that there was no probable cause?

RULING: Mandamus is an extraordinary writ commanding a tribunal, corporation, board, officer or


person, immediately or at some other specified time, to do the act required to be done, when the
respondent unlawfully neglects the performance of an act which the law specifically enjoins as a duty
resulting from an office, trust, or station; or when the respondent excludes another from the use and
enjoyment of a right or office to which the latter is entitled, and there is no other plain, speedy and
adequate remedy in the ordinary course of law.

As an extraordinary writ, the remedy of mandamus lies only to compel an officer to perform a ministerial
duty, not a discretionary one; mandamus will not issue to control the exercise of discretion by a public
officer where the law imposes upon him the duty to exercise his judgment in reference to any manner in
which he is required to act, because it is his judgment that is to be exercised and not that of the court.

In the case at bar, the act which petitioners pray that we compel the trial court to do is to grant the Office
of the City Prosecutors Motion for Withdrawal of Informations against petitioners. In effect, petitioners
seek to curb Judge Bays exercise of judicial discretion.

There is indeed an exception to the rule that matters involving judgment and discretion are beyond the
reach of a writ of mandamus, for such writ may be issued to compel action in those matters, when
refused. However, mandamus is never available to direct the exercise of judgment or discretion in a
particular way or the retraction or reversal of an action already taken in the exercise of either. In other
words, while a judge refusing to act on a Motion to Withdraw Informations can be compelled
by mandamus to act on the same, he cannot be compelled to act in a certain way, i.e., to grant or deny
such Motion. 

In the case at bar, Judge Bay did not refuse to act on the Motion to Withdraw Informations; he had
already acted on it by denying the same. Accordingly, mandamus is not available anymore. If petitioners
believed that Judge Bay committed grave abuse of discretion in the issuance of such Order denying the
Motion to Withdraw Informations, the proper remedy of petitioners should have been to file a Petition
for Certiorari against the assailed Order of Judge Bay.

Doctrine:A judge refusing to act on a Motion to Withdraw Informations can be compelled


by mandamus to act on the same he cannot be compelled to act in a certain way.

Remedial Law Review II (CY 2018-2019) – Saturday 1:00pm – 5:00pm: Case Digest Compilation [Atty. Brondial] Page 91
D.3.2. EX-C1C JIMMY B. SANCHEZ and EX-C2C SALVADOR A. METEORO, Petitioners,
vs.ROBERTO T. LASTIMOSO, in his capacity as DIRECTOR GENERAL OF THE PHILIPPINE
NATIONAL POLICE,Respondent

G.R. No. 161735; September 25, 2007

FACTS: Petitioners Sanchez and Meteoro are police officers discharged from the Philippine
Constabulary. Their respective cases were dismissed on appeal and charges against them were cleared.
They then applied for reinstatement but their applications were not acted upon even up to the integration
of the PC into the Philippine National Police (PNP).

NAPOLCOM issued a resolution considering as absorbed into the police force those who had been
discharged by virtue of pending administrative or criminal cases but who were later acquitted or had their
cases dismissed, and who subsequently filed petitions for reinstatement that were not acted upon by the
PNP.

NAPOLCOM issued a memo for the issuance of absorption orders by the PNP but no absorption order
was issued by the PNP. Petitioners then sought assistance of the DILG which issued a memo.

There was no order from PNP thus, petitioners filed a case for mandamus.

ISSUE: Is mandamus a correct remedy?

RULING: No. The remedy of mandamus is employed only to compel the performance, when refused, of
a ministerial duty, but not to require anyone to fulfill a discretionary one. The issuance of the writ is
simply a command to exercise a power already possessed and to perform a duty already imposed.

In order that a writ of mandamus may aptly issue, it is essential that, on the one hand, petitioner has a
clear legal right to the claim that is sought and that, on the other hand, respondent has an imperative duty
to perform that which is demanded of him.

The principal function of the writ of mandamus is to command and to expedite, not to inquire and to
adjudicate. Thus, it is neither the office nor the aim of the writ to secure a legal right but to implement
that which is already established. Unless the right to relief sought is unclouded, mandamus will not issue.

the PNP Chief’s issuance of the orders for the absorption of petitioners in the police force is not
compellable by a writ of mandamus precisely because the same does not involve a performance of a
ministerial duty.

Let it be noted that petitioners were discharged from the PC service, subsequently cleared of the charges
against them, applied for reinstatement but their applications were not acted upon until the integration of
the PC into the PNP in 1990 when R.A. No. 6975was enacted. Thus, we no longer speak of the
reinstatement of the petitioners to the service because the Philippine Constabulary no longer exists, but of
their employment in the PNP which istechnically an issuance of a new appointment.

The power to appoint is essentially discretionary to be performed by the officer in which it is vested
according to his best lights, the only condition being that the appointee should possess the qualifications
required by law.Consequently, it cannot be the subject of an application for a writ of mandamus.

* NAPOLCOM issued Resolution No. 99-061 recalling the approval of absorption thus trial court should
have dismissed the case for being moot.

*On indispensable parties “The absence of an indispensable party renders all subsequent actions of the
court null and void for want of authority to act, not only as to the absent parties but even as to those
present.”

Remedial Law Review II (CY 2018-2019) – Saturday 1:00pm – 5:00pm: Case Digest Compilation [Atty. Brondial] Page 92
D.3.3. SOCIAL JUSTICE SOCIETY (SJS) et al. vs. HON. JOSE L. ATIENZA, JR.

G.R. No. 156052; March 7, 2007; Ponente: Corona, J.

FACTS: SangguniangPanlungsod of Manila enacted Ordinance No. 8027 which becameeffective on


December 28, 2001. Ordinance No. 8027 reclassified the areadescribed therein from industrial to
commercial and directed the owners andoperators of businesses disallowed under Section 1 to cease and
desist fromoperating their businesses within six months from the date of effectivity of theordinance.
Among the businesses situated in the area are the so-called"Pandacan Terminals" of the oil companies
Caltex (Philippines), Inc., PetronCorporation and Pilipinas Shell Petroleum Corporation.

However, implementation of the ordinance was repeatedly postponed. Thus,petitioners filed this original
action for mandamus on December 4, 2002 prayingthat Mayor Atienza be compelled to enforce
Ordinance No. 8027 and order theimmediate removal of the terminals of the oil companies. Petitioners
contendthat respondent has the mandatory legal duty, under Section 455 (b) (2) of theLocal Government
Code (RA 7160), to enforce Ordinance No. 8027 and orderthe removal of the Pandacan Terminals of the
oil companies. Instead, he hasallowed them to stay.

ISSUE: Whether or not mandamus may be issued in order to compel theenforcement of an ordinance.

RULING: The answer is in the affirmative. Mandamus is an extraordinary writ that isemployed to
compel the performance, when refused, of a ministerial duty thatis already imposed on the respondent and
there is no other plain, speedy andadequate remedy in the ordinary course of law. The petitioner should
have awell-defined, clear and certain legal right to the performance of the act and itmust be the clear and
imperative duty of respondent to do the act required to bedone.

When a mandamus proceeding concerns a public right and its object is tocompel a public duty, the people
who are interested in the execution of the lawsare regarded as the real parties in interest and they need not
show any specificinterest. Besides, as residents of Manila, petitioners have a direct interest in
theenforcement of the city’s ordinances. Respondent never questioned the right ofpetitioners to institute
this proceeding.

On the other hand, the Local Government Code imposes upon respondent theduty, as city mayor, to
"enforce all laws and ordinances relative to thegovernance of the city." One of these is Ordinance No.
8027. As the chiefexecutive of the city, he has the duty to enforce Ordinance No. 8027 as long asit has not
been repealed by the Sanggunian or annulled by the courts. He hasno other choice. It is his ministerial
duty to do so.

These officers cannot refuse to perform their duty on the ground of an allegedinvalidity of the statute
imposing the duty. The reason for this is obvious. Itmight seriously hinder the transaction of public
business if these officers wereto be permitted in all cases to question the constitutionality of statutes
andordinances imposing duties upon them and which have not judicially beendeclared unconstitutional.
Officers of the government from the highest to thelowest are creatures of the law and are bound to obey
it.

Remedial Law Review II (CY 2018-2019) – Saturday 1:00pm – 5:00pm: Case Digest Compilation [Atty. Brondial] Page 93
D.3.4.DENNIS A.B. FUNA, Petitioner, vs. MANILA ECONOMIC AND CULTURAL OFFICE and
the COMMISSION ON AUDIT, Respondents.

G.R. No. 193462 | February 4, 2014 | EN BANC; PEREZ, J.:

FACTS:
Context: There used to be a power struggle in China, one government is called Peoples’ Republic of
China based in mainland China while the other is called Republic of China based in Taiwan. The former
won and consequently the International Community revoked their recognition of the latter, the
Philippines followed suit. However, the Philippines continued to deal with Taiwan and to facilitate its
dealings, Manila Economic and Cultural Office (MECO) was created as such.

The MECO12 was organized as a non-stock, non-profit corporation under the Corporation Code. MECO
became the corporate entity "entrusted" by the Philippine government with the responsibility of fostering
"friendly" and "unofficial" relations with the people of Taiwan, particularly in the areas of trade,
economic cooperation, investment, cultural, scientific and educational exchanges. Also, it was
"authorized" by the government to perform certain "consular and other functions" that relates to the
promotion, protection and facilitation of Philippine interests in Taiwan.

On 23 August 2010, petitioner Atty. Dennis Funa (petitioner) sent a letter to the COA requesting for a
"copy of the latest financial and audit report" of the MECO invoking, for that purpose, his "constitutional
right to information on matters of public concern." The petitioner made the request on the belief that the
MECO, being under the "operational supervision" of the Department of Trade and Industry (DTI), is a
government owned and controlled corporation (GOCC) and thus subject to the audit jurisdiction of the
COA.

Petitioner’s letter was received by COA Assistant Commissioner Naranjo and issued a
memorandum referring the request to COA Assistant Commissioner Espina for "further disposition." In
said memorandum, Naranjo revealed that MECO was "not among the agencies audited. Subsequently,
petitioner learned about the contents of Naranjo’s memorandum and petitioner took it as an admission
that COA had never audited and examined the accounts of the MECO. Consequently, petitioner filed
the instant petitionfor mandamus before the SC in his capacities as "taxpayer, concerned citizen, a
member of the Philippine Bar and law book author.
Petitioner posits that by failing to audit the accounts of the MECO, the COA is neglecting its duty under
Section 2(1), Article IX-D of the Constitution to audit the accounts of an otherwise bona fide GOCC or
government instrumentality. It is the adamant claim of the petitioner that the MECO is a GOCC without
an original charter or, at least, a government instrumentality, the funds of which partake the nature of
public funds.

MECOargues that a cause of action for mandamus to compel the performance of a ministerial duty
required by law only ripens once there has been a refusal by the tribunal, board or officer concerned to
perform such a duty.34The MECO claims that there was, in this case, no such refusal either on its part or
on the COA’s because the petitioner never made any demand for it to submit to an audit by the COA or
for the COA to perform such an audit, prior to filing the instant mandamus petition. 35 

The COA argues that the mandamus petition ought to be dismissed on the ground of mootness because
COA Chairperson Maria Gracia M. Pulido-Tan (Pulido-Tan) issued Office Order No. 2011-698directing
a team of auditors to proceed to Taiwan to audit certain accounts of MECO. Further, it argued that
petitioner lacks locus standi to bring the suit and the petition was filed in violation of the doctrine of
hierarchy of courts.

ISSUES:
1. Whether the mandamus petition should be dismissed for being moot and academic, in light of COA’s
Office Order No. 2011-698, an order directing to audit certain accounts of MECO.
2. Whether petitioner Funa has standing to file the mandamus petition.
3. Whether petitioner Funa in filing the mandamus petitiondirectly with SC violated the Principle of
Hierarchy of Courts.

RULING:
1. No. In this case, we find that the issuance by the COA of Office Order No. 2011-698 indeed qualifies
as a supervening event that effectively renders moot and academic the main prayer of the instant

Remedial Law Review II (CY 2018-2019) – Saturday 1:00pm – 5:00pm: Case Digest Compilation [Atty. Brondial] Page 94
mandamus petition. A writ of mandamus to compel the COA to audit the accounts of the MECO would
certainly be a mere superfluity, when the former had already obliged itself to do the same.

Be that as it may, this Court refrains from dismissing outright the petition. We believe that the mandamus
petition was able to craft substantial issues presupposing the commission of a grave violation of the
Constitution and involving paramount public interest, which need to be resolved nonetheless:

First. The petition makes a serious allegation that the COA had been remiss in its constitutional or
legal duty to audit and examine the accounts of an otherwise auditable entity in the MECO.

Second. There is paramount public interest in the resolution of the issue concerning the failure of the
COA to audit the accounts of the MECO. The propriety or impropriety of such a refusal is
determinative of whether the COA was able to faithfully fulfill its constitutional role as the guardian of
the public treasury, in which any citizen has an interest.

Third. There is also paramount public interest in the resolution of the issue regarding the legal status of
the MECO; a novelty insofar as our jurisprudence is concerned. We find that the status of the MECO
—whether it may be considered as a government agency or not—has a direct bearing on the country’s
commitment to the One China policy of the PROC.

An allegation as serious as a violation of a constitutional or legal duty, coupled with the pressing public
interest in the resolution of all related issues, prompts this Court to pursue a definitive ruling thereon, if
not for the proper guidance of the government or agency concerned, then for the formulation of
controlling principles for the education of the bench, bar and the public in general. 68 For this purpose, the
Court invokes its symbolic function.

If the foregoing reasons are not enough to convince, We still add another: Assuming that the allegations
of neglect on the part of the COA were true, Office Order No. 2011-698 does not offer the strongest
certainty that they would not be replicated in the future. In the first place, Office Order No. 2011-698 did
not state any legal justification as to why, after decades of not auditing the accounts of the MECO, the
COA suddenly decided to do so. Neither does it state any determination regarding the true status of the
MECO. The justifications provided by the COA, in fact, only appears in the memorandum 70 it submitted
to this Court for purposes of this case.

Thus, the inclusion of the MECO in Office Order No. 2011-698 appears to be entirely dependent upon the
judgment of the incumbent chairperson of the COA; susceptible of being undone, with or without reason,
by her or even her successor. Hence, the case now before this Court is dangerously capable of being
repeated yet evading review.

2. Yes. The rules regarding legal standing in bringing public suits, or locus standi, are already well-
defined in our case law. Again, We cite David, which summarizes jurisprudence on this point:

By way of summary, the following rules may be culled from the cases decided by this Court.Taxpayers,
voters, concerned citizens, and legislators may be accorded standing to sue, provided that the following
requirements are met:

(1) the cases involve constitutional issues;


(2) for taxpayers, there must be a claim of illegal disbursement of public funds or that the tax
measure is unconstitutional;
(3) for voters, there must be a showing of obvious interest in the validity of the election law in
question;
(4) for concerned citizens, there must be a showing that the issues raised are of transcendental
importance which must be settled early; and
(5) for legislators, there must be a claim that the official action complained of infringes upon their
prerogatives as legislators.

We rule that the instant petition raises issues of transcendental importance, involved as they are with the
performance of a constitutional duty, allegedly neglected, by the COA. Hence, We hold that the
petitioner, as a concerned citizen, has the requisite legal standing to file the instant mandamus petition.

To be sure, petitioner does not need to make any prior demand on the MECO or the COA in order to
maintain the instant petition. The duty of the COA sought to be compelled by mandamus, emanates from

Remedial Law Review II (CY 2018-2019) – Saturday 1:00pm – 5:00pm: Case Digest Compilation [Atty. Brondial] Page 95
the Constitution and law, which explicitly require, or "demand," that it perform the said duty. To the mind
of this Court, petitioner already established his cause of action against the COA when he alleged that the
COA had neglected its duty in violation of the Constitution and the law.

3. No. In view of the transcendental importance of the issues raised in the mandamus petition, as earlier
mentioned, this Court waives this last procedural issue in favor of a resolution on the merits. 76

NOTE: On the substantive issue of whether MECO is a government instrumentality or GOCC. SC ruled
it is not. Nevertheless, COA has jurisdiction to audit non-governmental entities such as MECO because
MECO

Remedial Law Review II (CY 2018-2019) – Saturday 1:00pm – 5:00pm: Case Digest Compilation [Atty. Brondial] Page 96
D.3.5. Cudia vs. Superintendent of PMA

February 24, 2015

FACTS: Cadet 1CL Cudia was a member of Siklab Diwa Class of 2014 of the PMA. He was supposed to
graduate with honors as the class salutatorian, receive the Philippine Navy Saber as the top Navy Cadet
graduate, and be commissioned as an ensign of the Philippine Navy. On November 14, 2013, the
combined classes of the Navy and Air Force 1CL cadets had a lesson examination (LE) on Operations
Research (OR432) under Dr. Costales at the PMAFI room. OR432 was from 1:30-3:00 while ENG412
was from 3:05 to 4:05. Prof. Berong of ENG412 issued a Delinquency Report (DR) against Cudia
because he was late for 2 minutes.

Cudia received his DR. His Explanation of Report dated December 8, 2013, he reasoned out that:”I came
directly from OR432. We were dismissed a bit late by our instructor Sir”. Major Hindang, the Company
Tactical Officer of Cudia, meted out to him the penalty of 11 demerits and 13 touring hours. Major
Hindang’s basis of the punishment was the result of his conversation with Dr. Costales, who responded
she never dismissed the class late, and the protocol to dismiss class 10-15 minutes earlier than scheduled.
Cudia addressed his Request for reconsideration of Meted Punishment to Maj. Leander, Senior Tactical
Officer. Maj. Leander sustained the penalty imposed. Major Hindang reported him to the Honor
Committee (HC) in violation of the Honor Code for lying. Cudia submitted his letter of explanation on
the honor report. The HC constituted a team to conduct the preliminary investigation on the violation, it
recommended the case be formalized. Cudia pleaded not guilty. The result was 8-1 guilty verdict and
upon the order of the Chairman, the HC reconvened in the chambers, after, the Presiding Officer
announced a 9-0 guilty verdict. The HC denied Cudia’s appeal.

The Headquarters Tactics Group conducted a formal review and checking of findings. Special orders
were issued placing Cudia on indefinite leave of absence and pending approval of separation from the
Armed Forces of the Philippines. Cudia submitted a letter to the Office of the Commandant of Cadets
requesting his reinstatement. The matter was referred to Cadet Review and Appeals Board and it upheld
the decision. Cudia wrote a letter to President Aquino but the President sustained the findings of the
CRAB. CHR-CAR issued a resolution finding probable cause for Human Rights Violations. Six days
prior to the March 16, 2014 graduation ceremonies of the Philippine Military Academy (PMA),
petitioners Renato P. Cudia, acting for himself and in behalf of his son, Cadet First Class Aldrin Jeff P.
Cudia (Cadet 1CL Cudia), and Berteni Cataluña Causing filed this petition for certiorari, prohibition, and
mandamus with application for extremely urgent temporary restraining order (TRO).

ISSUE: Whether or not the petition for mandamus is proper ?

RULING: No, Under Section 3, Rule 65 of the Rules of Civil Procedure, a petition for mandamus may
be filed when any tribunal, corporation, board, officer, or person unlawfully neglects neglects the
performance of an act which the law specifically enjoins as a duty resulting from an office, trust, or
station. It may also be filed when any tribunal, corporation, board, officer, or person unlawfully excludes
another from the use and enjoyment of a right or office to which such other is entitled. For mandamus to
lie, the act sought to be enjoined must be a ministerial act or duty. An act is ministerial if the act should
be performed "[under] a given state of facts, in a prescribed manner, in obedience to the mandate of a
legal authority, without regard to or the exercise of [the tribunal or corporation's] own judgment upon the
propriety or impropriety of the act done." The tribunal, corporation, board, officer, or person must have
no choice but to perform the act specifically enjoined by law. This is opposed to a discretionary act
whereby the officer has the choice to decide how or when to perform the duty. Anent the plea to direct the
PMA to include Cadet 1CL Cudia in the list of graduates of Siklab Diwa Class of 2014 and to allow him
to take part in the commencement exercises, the same was rendered moot and academic when the
graduation ceremonies pushed through on March 16, 2014 without including Cadet 1CL Cudia in the roll
of graduates. With respect to the prayer directing the PMA to restore Cadet 1CL Cudia’s rights and
entitlements as a full-fledged graduating cadet, including his diploma, awards, and commission as a new
Philippine Navy ensign, the same cannot be granted in a petition for mandamus on the basis of academic
freedom. Suffice it to say at this point that these matters are within the ambit of or encompassed by the
right of academic freedom; therefore, beyond the province of the Court to decide. The powers to confer
degrees at the PMA, grant awards, and commission officers in the military service are discretionary acts
on the part of the President as the AFP Commander-in-Chief. For a writ to issue, petitioners should have a
clear legal right to the thing demanded, and there should be an imperative duty on the part of respondents
to perform the act sought to be mandated.

Remedial Law Review II (CY 2018-2019) – Saturday 1:00pm – 5:00pm: Case Digest Compilation [Atty. Brondial] Page 97
D.3.6. FERDINAND R. VILLANUEVA, PRESIDING JUDGE, MCTC, COMPOSTELA-NEW
BATAAN, COMPOSTELA VALLEY PROVINCE,v. JUDICIAL AND BAR COUNCIL

G.R. No. 211833; April 07, 2015; REYES, J.:

FACTS: The petitioner was appointed as the Presiding Judge of the Municipal Circuit Trial Court,
Compostela-New Bataan, Poblacion, Compostela Valley Province, Region XI, which is a first-level court.
More thana year later, he applied for the vacant position of Presiding Judge in various Regional Trial
Courts.

Petitioner was not included in the list of applicants due to the JBC's long-standing policy of opening the
chance for promotion to second-level courts to, among others, incumbent judges who have served in their
current position for at least five years, and since the petitioner has been a judge only for more than a year,
he was excluded from the list. This caused the petitioner to take recourse to this Court.

ISSUES:
1. Whether or not the remedies of certiorari and prohibition are tenable.
2. Whether or not the remedy of mandamus is tenable.

RULING:
One. The remedies of certiorari and prohibition are tenable. "The present Rules of Court uses two special
civil actions for determining and correcting grave abuse of discretion amounting to lack or excess of
jurisdiction. These are the special civil actions for certiorari and prohibition, and both are governed by
Rule 65."As discussed in the case of Maria Carolina P. Araullo, etc., et al. v. Benigno Simeon C. Aquino
III, etc., et al., this Court explained that:

With respect to the Court, however, the remedies of certiorari and prohibition are necessarily broader in
scope and reach, and the writ of certiorari or prohibition may be issued to correct errors of jurisdiction
committed not only by a tribunal, corporation, board or officer exercising judicial, quasi-judicial or
ministerial functions but also to set right, undo and restrain any act of grave abuse of discretion
amounting to lack or excess of jurisdiction by any branch or instrumentality of the Government, even if
the latter does not exercise judicial, quasi-judicial or ministerial functions. This application is expressly
authorized by the text of the second paragraph of Section 1, supra.

Thus, petitions for certiorari and prohibition are appropriate remedies to raise constitutional issues and to
review and/or prohibit or nullify the acts of legislative and executive officials. (Citation omitted)

In this case, it is clear that the JBC does not fall within the scope of a tribunal, board, or officer exercising
judicial or quasi-judicial functions. In the process of selecting and screening applicants, the JBC neither
acted in any judicial or quasi-judicial capacity nor assumed unto itself any performance of judicial or
quasi-judicial prerogative. However, since the formulation of guidelines and criteria, including the policy
that the petitioner now assails, is necessary and incidental to the exercise of the JBC's constitutional
mandate, a determination must be made on whether the JBC has acted with grave abuse of discretion
amounting to lack or excess of jurisdiction in issuing and enforcing the said policy.

Two. The remedy of mandamus cannot be availed of by the petitioner in assailing JBC's policy.

It is essential to the issuance of a writ of mandamus that the applicant should have a clear legal right to
the thing demanded and it must be the imperative duty of the respondent to perform the act required.The
petitioner bears the burden to show that there is such a clear legal right to the performance of the act, and
a corresponding compelling duty on the part of the respondent to perform the act. The remedy of
mandamus, as an extraordinary writ, lies only to compel an officer to perform a ministerial duty, not a
discretionary one.Clearly, the use of discretion and the performance of a ministerial act are mutually
exclusive.

The writ of mandamus does not issue to control or review the exercise of discretion or to compel a course
of conduct, which, it quickly seems to us, was what the petitioner would have the JBC do in his favor.
The function of the JBC to select and recommend nominees for vacant judicial positions is discretionary,
not ministerial. Moreso, the petitioner cannot claim any legal right to be included in the list of nominees
for judicial vacancies. Possession of the constitutional and statutory qualifications for appointment to the
judiciary may not be used to legally demand that one's name be included in the list of candidates for a
judicial vacancy. One's inclusion in the list of the candidates depends on the discretion of the JBC. XXX

Remedial Law Review II (CY 2018-2019) – Saturday 1:00pm – 5:00pm: Case Digest Compilation [Atty. Brondial] Page 98
E. E. Republic of the Philippines v. Maria Lourdes Sereno

G.R. No. 237428; 11 May 2018

FACTS: The Republic of the Philippines, through the OSG, filed a Petition for the issuance of the
extraordinary writ of quo warranto to declare as void Sereno’s (respondent) appointment as Chief Justice
of the Supreme Court and to oust and altogether exclude respondent therefrom.

For a period of 20 years, from 1986 to 2006, respondent served as a member of the faculty of UP-College
of Law. Also, while being employed at the UP College of Law, from 2003 to 2006, she was employed as
legal counsel of the Republic in the PIATCO cases. After serving as a professor in UP, she served as
counsel in various government agencies. In July 2010, respondent submitted her application for the
position of Associate Justice of the Supreme Court.

Despite its 20-year employment with UP and its employment as legal counsel in various government
agencies, records from the UP Human Resources Development Office, Central Records Division of the
Office of the Ombudsman, and the Office of Recruitment Selection and Nomination (ORSN) of the
Judicial and Bar Council (JBC) show that the only SALNs available on record and filed by respondent
were those for the calendar years 1985, 1989, 1990, 1991, 1993, 1994, 1995, 1996, 1997, 1998, and 2002
or eleven (11) SALNs filed in her 20-year government service in U.P. No SALNs were filed from 2003 to
2006 when she was employed as legal counsel for the Republic. Neither was there SALN filed when she
resigned from U.P. College of Law as of June 1, 2006 and when she supposedly re-entered government
service as of August 16, 2010.

In August 2010, respondent was appointed by President Benigno Aquino III as Associate Justice.

When the position of the Chief Justice was declared vacant in 2012, the JBC announced the opening for
application and required the applicants for the Chief Justice position to submit, instead of the usual
submission of the SALNs for the last two years of public service, all previous SALNs up to December 31,
2011 for those in government service. The JBC announcement further provided that "applicants with
incomplete or out-of-date documentary requirements will not be interviewed or considered for
nomination.

Respondent accepted several nominations from the legal and the evangelical community for the position
of Chief Justice and in support of her nomination, respondent submitted to the ORSN her SALNs for the
years 2009, 2010, and 2011. Respondent also executed waiver of confidentialityof her local and foreign
bank accounts.

The JBC in its Special En Banc Meeting, deliberated on the candidates for the position of Chief Justice
with incomplete documentary requirements. In particular, the JBC examined the list of candidates and
their compliance with the required submission of SALNs. The Executive Officer informed the Council
that respondent had not submitted her SALNs for period of ten (10) years, that is, from 1986 to 2006.
Senator Escudero mentioned that Justice Sereno was his professor at U.P. and that they were required to
submit SALNs during those years.

There were other candidates with incomplete documentary requirements.

Hence, the ORSN, through its then Chief Atty. Pascual, inquired as to respondent's SALNs for the years
1995, 1996, 1997 and 1999. Respondent responded through a letter stating that “Considering that most of
my government records in the academe are more than fifteen years old, it is reasonable to consider it
infeasible to retrieve all of those files. In any case, the University of the Philippines has already cleared
me of all academic/administrative responsibilities, money and property accountabilities and from
administrative charges.”

The letter, however, was neither examined by the JBC regular members nor was it deliberated upon either
by the JBC En Banc or the Execom. Despite respondent's submission of only 3 SALNs, respondent was
listed as applicant No. 14 with an opposite annotation that she had "COMPLETE REQUIREMENTS".

The JBC proceeded to interview the candidates, including respondent who was interviewed. ORSN
prepared list of the 20 candidates, respondent included, vis-a-vis their SALN submissions. Opposite
respondent's name was an enumeration of the SALNs she submitted (2009, 2010 and 2011) and an

Remedial Law Review II (CY 2018-2019) – Saturday 1:00pm – 5:00pm: Case Digest Compilation [Atty. Brondial] Page 99
excerpt from her letter that "considering that respondent's government records in the academe are more
than 15 years old, it is reasonable to consider it infeasible to retrieve all those files”.

Respondent was appointed by President Benigno Aquino III on 25 August 2012. Five years later, an
impeachment complaint was filed by Atty. Larry Gadon with the House Committee of Justice. Included in
the complaint was the allegation that Respondent failed to make a truthful statement of her SALNs. A
letter was submitted by Atty. Eligio Mallari to the OSG requesting that the latter, in representation of the
Republic, initiate quo warranto proceeding against respondent.

Republic’s Contention

1. An action for quo warranto is the proper remedy to question the validity of respondent's
appointment.
2. Petition is seasonably filed within the one-year reglementary period under Section 11, Rule 66, of
the Rules of Court since respondent's transgressions only came to light during the proceedings of
the House Committee on Justice on the allegations of the impeachment complaint filed against
her.
3. Quo warranto is available as remedy even as against impeachable officers, like respondent.
4. Petition for quo warranto is different from the impeachment proceedings because the writ of quo
warranto is being sought to question the validity of her appointment, while the impeachment
complaint accuses her of committing culpable violation of the Constitution and betrayal of public
trust while in office.
5. Respondent's failure to submit her SALNs as required by the JBC disqualifies her, at the outset,
from being candidate for the position of Chief Justice. Lacking her SALNs, respondent has not
proven her integrity which is requirement under the Constitution. The Republic thus concludes
that since respondent is ineligible for the position of Chief Justice for lack of proven integrity, she
has no right to hold office and may therefore be ousted via quo warranto.

Respondent’s Contention

 Chief Justice may be ousted from office only by impeachment.


 The use of the phrase "may be removed from office" in Section 2, Article XI of the Constitution
does not signify that Members of the Supreme Court may be removed through modes other than
impeachment. The clear intention of the framers of the Constitution was to create an exclusive
category of public officers who can be removed only by impeachment and not otherwise.
 Since petition for quo warranto may be filed before the RTC, such would result to conundrum
because judge of lower court would have effectively exercised disciplinary power and
administrative supervision over an official of the Judiciary much higher in rank and is contrary to
Sections and 11, Article VIII of the Constitution.
 Present petition is time-barred as Section 11, Rule 66 provides that petition for quo
warranto must be filed within one (1) year from the "cause of ouster" and not from the
"discovery" of the disqualification. The supposed "failure" to file the required SALNs allegedly
took place for several years from 1986 to 2006, thus, the "cause of ouster" existed even before the
respondent was appointed as Chief Justice

ISSUES:

1. Whether the Court can assume jurisdiction and give due course to the instant petition for quo
warranto against respondent who is an impeachable officer and against whom an impeachment
complaint has already been filed with the House of Representatives
2. Whether the petition is outrightly dismissible on the ground of prescription
3. Whether respondent is eligible for the position of Chief Justice

RULING:
1. Yes. SC has jurisdiction over an action for quo warranto

a. Section 5, Article VIII of the Constitution provides that the Supreme Court shall exercise
original jurisdiction over petitions for certiorari, prohibition, mandamus, quo warranto,
and habeas corpus. SC, the Court of Appeals and the Regional Trial Courts have concurrent
jurisdiction to issue the extraordinary writs, including quo warranto.

Also, Section 7, Rule 66 of the Rules of Court provides that the venue of an action for quo

Remedial Law Review II (CY 2018-2019) – Saturday 1:00pm – 5:00pm: Case Digest Compilation [Atty. Brondial] Page 100
warranto, when commenced by the Solicitor General, is either the Regional Trial Court in the
City of Manila, in the Court of Appeals, or in the Supreme Court.

While the hierarchy of courts serves as general determinant of the appropriate forum for
petitions for the extraordinary writs, direct invocation of the Supreme Court's original
jurisdiction to issue such writs is allowed when there are special and important reasons therefor,
clearly and specifically set out in the petition.

The instant petition is case of transcendental importance. It is apparent that the instant petition is
one of first impression and of paramount importance to the public in the sense that the
qualification, eligibility and appointment of an incumbent Chief Justice, the highest official of
the Judiciary, are being scrutinized through an action for quo warranto. The Court's action on
the present petition has far-reaching implications, and it is paramount that the Court make
definitive pronouncements on the issues herein presented for the guidance of the bench, bar, and
the public in future analogous cases. Thus, the questions herein presented merit serious
consideration from the Court and should not be trifled on.

b. On the response of respondent that as she is an impeachable officer, and thus quo
warranto proceeding, which may result in her ouster, cannot be lodged against her, especially
when there is an impending impeachment case against her, the Court ruled that while both
impeachment and quo warranto may result in the ouster of the public official, the two
proceedings materially differ. Impeachment is proceeding exercised by the legislative, as
representatives of the sovereign, to vindicate the breach of the trust reposed by the people in the
hands of the public officer by determining the public officer's fitness to stay in the office.
Meanwhile, an action for quo warranto, involves judicial determination of the eligibility or
validity of the election or appointment of public official based on predetermined rules.

Quo warranto and impeachment can proceed independently and simultaneously as these
remedies are distinct as to (1) jurisdiction (2) grounds, (3) applicable rules pertaining to
initiation, filing and dismissal, and (4) limitations.

Quo warranto proceeding is the proper legal remedy to determine the right or title to the
contested public office or to oust the holder from its enjoyment. In quo warranto proceedings
referring to offices filled by election, what is to be determined is the eligibility of the candidates
elected, while in quo warranto proceedings referring to offices filled by appointment, what is
determined is the legality of the appointment.

An action for quo warranto may be commenced by the Solicitor General or public prosecutor, or


by any person claiming to be entitled to the public office or position usurped or unlawfully held
or exercised by another.

That usurpation of public office is treated as public wrong and carries with it public interest in
our jurisdiction is clear when Section 1, Rule 66 provides that where the action is for the
usurpation of public office, position or franchise, it shall be commenced by verified petition
brought in the name of the Republic of the Philippines through the Solicitor General or public
prosecutor.

The remedies available in quo warranto judgment do not include correction or reversal of acts
taken under the ostensible authority of an office or franchise. Judgment is limited to ouster or
forfeiture and may not be imposed retroactively upon prior exercise of official or corporate
duties.

In sum, Quo warranto and impeachment are, thus, not mutually exclusive remedies and may
even proceed simultaneously. The existence of other remedies against the usurper does not
prevent the State from commencing quo warranto proceeding.

c. Impeachment is not an exclusive remedy by which an invalidly appointed or invalidly elected


impeachable official may be removed from office

The language of Section 2, Article XI of the Constitution does not foreclose a quo warranto
action against impeachable officers: xxx… may be removed from office ...”

Remedial Law Review II (CY 2018-2019) – Saturday 1:00pm – 5:00pm: Case Digest Compilation [Atty. Brondial] Page 101
The provision uses the permissive term “may” which, in statutory construction, denotes
discretion and cannot be construed as having a mandatory effect. An option to remove by
impeachment admits of an alternative mode of effecting the removal. Further, that the
enumeration of "impeachable offenses" is made absolute, that is, only those enumerated offenses
are treated as grounds for impeachment, is not equivalent to saying that the enumeration likewise
purport to be complete statement of the causes of removal from office. 

That the enumeration of “impeachable offenses” is made absolute such that only those
enumerated offenses are treated as grounds for impeachment does not mean that it is to be taken
as a complete statement of the causes of removal from office. The word “may” cannot also be
understood to qualify only the imposable penalties because it would lead to the conclusion that
other lesser penalties may be imposed — a situation not contemplated in the language of the
Constitutional provision.

The courts should be able to inquire into the validity of appointments even of impeachable
officers. To hold otherwise is to allow an absurd situation where the appointment of an
impeachable officer cannot be questioned, on the basis of citizenship or membership in the Bar,
for example. Unless such an officer commits any of the grounds for impeachment and is actually
impeached, he can continue discharging the functions of his office even when he is clearly
disqualified from holding it. Such would result in permitting unqualified and ineligible public
officials to continue occupying key positions, exercising sensitive sovereign functions until they
are successfully removed from office through impeachment.

d. The Supreme Court's exercise of its jurisdiction over quo warranto petition is not violative of
the doctrine of separation of powers

The fact that the violation of the SALN requirement formed part of the impeachment complaint
does not justify shifting responsibility to the Congress, no matter how noble the respondent and
the intervenors’ portray such act to be. The fact remains that the Republic raised an issue as to
respondent's eligibility to occupy the position of Chief Justice, an obviously legal question,
which can be resolved through review of jurisprudence and pertinent laws. To abdicate from
resolving legal controversy simply because of perceived availability of another remedy, in this
case impeachment, would be to sanction the initiation of process specifically intended to be long
and arduous and compel the entire membership of the Legislative branch to momentarily
abandon their legislative duties to focus on impeachment proceedings for the possible removal
of public official, who at the outset, may clearly be unqualified under existing laws and case law.
Evidently, this scenario would involve waste of time, not to mention unnecessary disbursement
of public funds.

Further, as an impeachment court, the Senate is tribunal composed of politicians who are
indubitably versed in pragmatic decision making and cognizant of political repercussions of acts
purported to have been committed by impeachable officials. As representatives of the Filipino
people, they determine whether the purported acts of highest ranking officials of the country
constitute as an offense to the citizenry. Following this premise, the impeachment tribunal
cannot be expected to rule on the validity or constitutionality of the Chief Justice's appointment,
nor can their ruling be of jurisprudential binding effect to this Court. To authorize Congress to
rule on public officials' eligibility would disturb the system of checks and balances as it would
dilute the judicial power of courts, upon which jurisdiction is exclusively vested to rule on
actions for quo warranto.

2. No.
a. Prescription does not lie against the State

The one-year limitation is NOT applicable when the petitioner is not mere private individual
pursuing private interest, but the government itself seeking relief for public wrong and suing for
public interest.

In People ex rel. Moloney v. Pullmans Palace Car Co., “It is the general rule that laches,
acquiescence, or unreasonable delay in the performance of duty on the part of the officers of the
state, is not imputable to the state when acting in its character as sovereign. There are exceptions
to this general rule, but we are unable to see that the allegations of the plea bring the case within
the principles of any such exceptions.”

Remedial Law Review II (CY 2018-2019) – Saturday 1:00pm – 5:00pm: Case Digest Compilation [Atty. Brondial] Page 102
The doctrine of laches, furthermore, does not apply when cause of action is brought by the State
seeking to protect the public. Quo warranto proceedings seeking ouster of public official are
governmental function.  No statute of limitations is, therefore, applicable. 

b. Circumstances obtaining in this case preclude the application of the prescriptive period
That prescription does not lie in this case can also be deduced from the very purpose of an action
for quo warranto. In this case, the Republic cannot be faulted for questioning respondent's
qualification for office only upon discovery of the cause of ouster.

Respondent was never forthright as to whether or not she filed her SALNs covering the period of
her employment in U.P. Recall that during her application for the Chief Justice position, the JBC
required the submission of her previous SALNs. In response to the JBC, respondent never
categorically stated that she filed the required SALNs. Instead, she cleverly hid the fact of non-
filing by stating that she should not be required to submit the said documents as she was
considered to be coming from private practice; that it was not feasible to retrieve most of her
records in the academe considering that the same are more than fifteen years old; and that U.P.
already cleared her of "all academic/administrative responsibilities, money and property
accountabilities and from administrative charges

3. No
a. Compliance with the Constitutional and statutory requirement of filing of SALN intimately
relates to person's integrity.

The filing SALN is an essential requirement to one's assumption of public post. It has
Constitutional, legal and jurisprudential bases.

Section 17, Article XI of the Constitution on the Accountability of Public Officers states:

Public officer or employee shall, upon assumption of office and as often thereafter as may
be required by law, submit declaration under oath of his assets, liabilities, and net worth. In
the case of the President, the Vice-President, the Members of the Cabinet, the Congress, the
Supreme Court, the Constitutional Commissions and other constitutional offices, and officers of
the armed forces with general or flag rank, the declaration shall be disclosed to the public in the
manner provided by law.

The requirement of filing SALN is enshrined in the Constitution to promote transparency in the
civil service and serves as deterrent against government officials bent on enriching themselves
through unlawful means. By mandate of law, every government official or employee must make
complete disclosure of his assets, liabilities and net worth in order to avoid any issue regarding
questionable accumulation of wealth. The failure to file truthful SALN reasonably puts in doubts
the integrity of the officer and normally amounts to dishonesty.

Faithful compliance with the requirement of the filing of SALN is rendered even more exacting
when the public official concerned is member of the Judiciary.

b. Compliance with the SALN requirement indubitably reflects on person's integrity

Failure to file the SALN is clearly violation of the law. The offense is penal in character and is
clear breach of the ethical standards set for public officials and employees. It disregards the
requirement of transparency as deterrent to graft and corruption. For these reasons, public official
who has failed to comply with the requirement of filing the SALN cannot be said to be of proven
integrity and the Court may consider him/her disqualified from holding public office.

The requirement to file SALN is not trivial or formal requirement. Neither is it something over
which public officials can exercise discretion. It is mandated by Our Constitution and laws. It is
meant to forge transparency and accountability in the government and as measure meant to curb
corruption.

c. Respondent chronically failed to file her SALNs and thus violated the Constitution, the law
and the Code of Judicial Conduct, member of the Judiciary who commits such violations
cannot be deemed to be person of proven integrity

Remedial Law Review II (CY 2018-2019) – Saturday 1:00pm – 5:00pm: Case Digest Compilation [Atty. Brondial] Page 103
Respondent could have easily dispelled doubts as to the filing or non-filing of the unaccounted
SALNs by presenting them before the Court. Yet, Respondent opted to withhold such information
or such evidence, if at all, for no clear reason. Her defenses do not lie: 1) The Doblada doctrine
does not persuade because in that case Doblada was able to present contrary proof that the
missing SALNs were, in fact, transmitted to the OCA, thus rendering inaccurate the OCA report
that she did not file SALNs for a number of years, as opposed to the present case where no proof
of existence and filing were presented; 2) Being on leave from government service is not
equivalent to separation from service such that she was still required to submit SALNs during her
leave; 3) While Respondent is not required by law to keep a record of her SALNs, logic dictates
that she should have obtained a certification to attest to the fact of filing; 4) That UP HRDO
never asked Respondent to comply with the SALN laws holds no water as the duty to comply
with such is incumbent with the Respondent, and because there was no duty for the UP HRDO to
order compliance under the rules implemented at that time; 5) That Respondent’s compliance
with the SALN requirement was reflected in the matrix of requirements and shortlist prepared by
the JBC is dispelled by the fact that the appointment goes into her qualifications which were
mistakenly believed to be present, and that she should have been disqualified at the outset.

d. Respondent's inclusion in the matrix of candidates with complete requirements and in the
shortlist nominated by the JBC confirms or ratifies her compliance with the SALN
requirement.

The invalidity of respondent's appointment springs from her lack of qualifications. Her inclusion
in the shortlist of candidates for the position of Chief Justice does not negate, nor supply her with
the requisite proof of integrity. She should have been disqualified at the outset. It must be
underscored that the JBC En Banc included respondent in the shortlist for the position of Chief
Justice without deliberating her July 23, 2012 Letter. Without prejudice to this Court's ruling in
A.M No. 17-11-12-SC and A.M. No. 17-11-17-SC, the JBC En Banc cannot be deemed to have
considered respondent eligible because it does not appear that respondent's failure to submit her
SALNs was squarely addressed by the body. Her inclusion in the shortlist of nominees and
subsequent appointment to the position do not estop the Republic or this Court from looking into
her qualifications. Verily, no estoppel arises where the representation or conduct of the party
sought to be estopped is due to ignorance founded upon an innocent mistake.

e. Respondent failed to properly and promptly file her SALNs, again in violation of the
Constitutional and statutory requirements. Hence, Respondent failed to submit the required
SALNs as to qualify for nomination pursuant to the JBC rules

The JBC required the submission of at least ten SALNs from those applicants who are incumbent
Associate Justices, absent which, the applicant ought not to have been interviewed, much less
been considered for nomination

Further compounding respondent's woes is the established and undisputed fact that she failed to
submit the required number of SALNs in vjolation of the rules set by the JBC itself during the
process of nomination.

Respondent was specifically singled out from the rest of the applicants for having failed to submit
single piece of SALN for her years of service in the U.P. College of Law. This is in obvious
contrast with the other shortlisted applicants who submitted SALNs, or whose years in
government service correspond to the period prior to the effectivity of R.A. No. 6713. Be that as
it may, records clearly show that the only remaining applicant-incumbent Justice who was not
determined by the JBC En Banc to have substantially complied was respondent, who submitted
only SALNs, i.e., 2009, 2010 and 2011, even after extensions of the deadline for the submission
to do so.

Instead of complying, respondent offered, by way of her letter dated July 23,
2012, justifications why she should no longer be required to file the SALNs: that she resigned
from U.P. in 2006 and then resumed government service only in 2009, thus her government
service is not continuous; that her government records are more than 15 years old and thus
infeasible to retrieve; and that U.P. cleared her of all academic and administrative responsibilities
and charges.

Remedial Law Review II (CY 2018-2019) – Saturday 1:00pm – 5:00pm: Case Digest Compilation [Atty. Brondial] Page 104
These justifications, however, did not obliterate the simple fact that respondent submitted only
SALNs in her 20-year service in U.P., and that there was nary an attempt on respondent's part to
comply.

Indubitably, respondent not only failed to substantially comply with the submission of the SALNs
but there was no compliance at all. The contents of respondent's Letter dated July 23, 2012 itself
betray an exercise of dishonesty and disposition to deceive in an attempt to secure for herself the
appointment as Chief Justice.

f. Respondent's failure to submit to the JBC her SALNs for several years means that her
integrity was not established at the time of her application

Respondent is presumed to know of the importance of the filing of the SALN together with the bank
waiver. The waiver which respondent executed under oath clearly provides:

This waiver is executed on the condition that the JBC or its duly authorized representatives shall make
use of it, as well as any and all information or data obtained by virtue thereof, for the exclusive and sole
purpose of evaluating my qualifications for the position of Chief Justice of the Supreme Court.
Conclusively then, respondent's failure to submit her SALNs to the JBC means that she was not able to
prove her integrity at the time of her application as Chief Justice.

g. Respondent's disposition to commit deliberate acts and omissions demonstrating dishonesty


and lack of forthrightness is discordant with any claim of integrity
The Court cannot play blind against the manifest inconsistencies, lack of forthrightness and dishonesty
committed by respondent as government official prior to and at the time of her application as Chief
Justice. In addition to the suspicious and highly questionable circumstances surrounding the execution of
her SALNs, there were untruthful statements and dishonest acts which were committed by the respondent
which ultimately negate respondent's claim that she is person of proven integrity

Again, while concedingly the foregoing acts as revealed during the Congressional hearings on the
impeachment are not proper subjects of the instant quo warranto petition, these acts are nevertheless
reflective and confirmatory of respondent's lack of integrity at the time of her nomination and
appointment as Chief Justice and her inability to possess such continuing requirement of integrity.

h. Respondent's ineligibility for lack of proven integrity cannot be cured by her nomination
and subsequent appointment as Chief Justice

Such failure to file and to submit the SALNs to the JBC, is clear violation not only of the JBC rules, but
also of the law and the Constitution. The discordance between respondent's non-filing and non-
submission of the SALNs and her claimed integrity as person is too patent to ignore. For lack of proven
integrity, respondent ought to have been disqualified by the JBC and ought to have been excluded from
the list of nominees transmitted to the President. As the qualification of proven integrity goes into the
barest standards set forth under the Constitution to qualify as Member of the Court, the subsequent
nomination and appointment to the position will not qualify an otherwise excluded candidate. In other
words, the inclusion of respondent in the shortlist of nominees submitted to the President cannot override
the minimum Constitutional qualifications.

Well-settled is the rule that qualifications for public office must be possessed at the time of appointment
and assumption of office and also during the officer's entire tenure as continuing requirement. When the
law requires certain qualifications to be possessed or that certain disqualifications be not possessed by
persons desiring to serve as public officials, those qualifications must be met before one even becomes
candidate.

The voidance of the JBC nomination as necessary consequence of the Court's finding that respondent is
ineligible, in the first place, to be candidate for the position of Chief Justice and to be nominated for said
position follows as matter of course. The Court has ample jurisdiction to do so without the necessity of
impleading the JBC as the Court can take judicial notice of the explanations from the JBC members and
the OEO, as regards the circumstances relative to the selection and nomination of respondent submitted to
this Court in A.M. No. 17-11-12 and A.M. No. 17-11-17-SC. Relatedly, the Court, in quo
warrantoproceeding, maintains the power to issue such further judgment determining the respective rights
in and to the public office, position or franchise of all the parties to the action as justice requires.

Remedial Law Review II (CY 2018-2019) – Saturday 1:00pm – 5:00pm: Case Digest Compilation [Atty. Brondial] Page 105
Questions:

1. What principles in remedial law have been retained by the case?

a. Concurrent jurisdiction in Quo Warranto Cases

Supreme Court shall exercise original jurisdiction over petitions for certiorari, prohibition, mandamus,
quo warranto, and habeas corpus. This Court, the Court of Appeals and the Regional Trial Courts have
concurrent jurisdiction to issue the extraordinary writs, including quo warranto.

b. The principle of hierarchy of courts (exception: transcendental importance)

While the hierarchy of courts serves as general determinant of the appropriate forum for petitions for the
extraordinary writs, direct invocation of the Supreme Court's original jurisdiction to issue such writs is
allowed when there are special and important reasons therefor, clearly and specifically set out in the
petition. In the instant case, direct resort to the Court is justified considering that the action for quo
warranto questions the qualification of no less than Member of the Court. The issue of whether person
usurps, intrudes into, or unlawfully holds or exercises public office is matter of public concern over which
the government takes special interest as it obviously cannot allow an intruder or impostor to occupy
public position. It is of transcendental importance.

c. Venue in quo warranto cases

Section 7, Rule 66 of the Rules of Court provides that the venue of an action for quo warranto, when
commenced by the Solicitor General, is either the Regional Trial Court in the City of Manila, in the Court
of Appeals, or in the Supreme Court.

d. Supreme Court is not a trier of facts

e. Seeking affirmative relief from the Court is tantamount to voluntary appearance

2. What principles in remedial law have been altered?

1. The one-year limitation is not equally applicable when the petitioner is not mere private
individual pursuing private interest, but the government itself seeking relief for public wrong and
suing for public interest

Section 11, Rule 66, provides:


Limitations. Nothing contained in this Rule shall be construed to authorize an action against public officer
or employee for his ouster from office unless the same be commenced within one (1) year after the
cause of such ouster, or the right of the petitioner to hold such office or position, arose; nor to
authorize an action for damages in accordance with the provisions of the next preceding section unless the
same be commenced within one (1) year after the entry of the judgment establishing the petitioner's right
to the office in question

2. Mode of removal

Impeachment is not an exclusive remedy by which an invalidly appointed or invalidly elected


impeachable official may be removed from office

The provision uses the permissive term "may" which, in statutory construction, denotes discretion and
cannot be construed as having mandatory effect.We have consistently held that the term "may" is
indicative of mere possibility, an opportunity or an option. The grantee of that opportunity is vested with
right or faculty which he has the option to exercise. An option to remove by impeachment admits of an
alternative mode of effecting the removal.

3. Difference in origin and nature of impeachment and quo warranto

While both impeachment and quo warranto may result in the ouster of the public official, the two
proceedings materially differ. At its most basic, impeachment proceedings are political in nature, while an
action for quo warranto is judicial or proceeding traditionally lodged in the courts.

Remedial Law Review II (CY 2018-2019) – Saturday 1:00pm – 5:00pm: Case Digest Compilation [Atty. Brondial] Page 106
Impeachment is proceeding exercised by the legislative, as representatives of the sovereign, to vindicate
the breach of the trust reposed by the people in the hands of the public officer by determining the public
officer's fitness to stay in the office. Meanwhile, an action for quo warranto, involves judicial
determination of the eligibility or validity of the election or appointment of public official based on
predetermined rules.

Likewise, the reliefs sought in the two proceedings are different. Under the Rules on quo warranto,
"when the respondent is found guilty of usurping, intruding into, or unlawfully holding or exercising
public office, x xx, judgment shall be rendered that such respondent be ousted and altogether excluded
therefrom, x xx." In short, respondent in quo warranto proceeding shall be adjudged to cease from holding
public office, which he/she is ineligible to hold. On the other hand, in impeachment, conviction for the
charges of impeachable offenses shall result to the removal of the respondent from the public office that
he/she is legally holding. It is not legally possible to impeach or remove person from an office that he/she,
in the first place, does not and cannot legally hold or occupy.

4. The Supreme Court's exercise of its jurisdiction over quo warranto petition is not violative of the
doctrine of separation of powers

Section 3(1) and 3(6), Article XI, of the Constitution respectively provides that the House of
Representatives shall have the exclusive power to initiate all cases of impeachment while the Senate shall
have the sole power to try and decide all cases of impeachment. Thus, there is no argument that the
constitutionally-defined instrumentality which is given the power to try impeachment cases is the Senate.

Nevertheless, the Court's assumption of jurisdiction over an action for quo warranto involving person who
would otherwise be an impeachable official had it not been for disqualification, is not violative of the core
constitutional provision that impeachment cases shall be exclusively tried and decided by the Senate.

Again, an action for quo warranto tests the right of person to occupy public position. It is direct
proceeding assailing the title to public office. The issue to be resolved by the Court is whether or not the
defendant is legally occupying public position which goes into the questions of whether defendant was
legally appointed, was legally qualified and has complete legal title to the office. If defendant is found to
be not qualified and without any authority, the relief that the Court grants is the ouster and exclusion of
the defendant from office. In other words, while impeachment concerns actions that make the officer unfit
to continue exercising his or her office, quo warranto involves matters that render him or her ineligible to
hold the position to begin with.

Remedial Law Review II (CY 2018-2019) – Saturday 1:00pm – 5:00pm: Case Digest Compilation [Atty. Brondial] Page 107
E.1. PEDRO MENDOZA, petitioner, vs. RAY ALLAS and GODOFREDO OLORES, respondents.

G.R. No. 131977; February 4, 1999; PUNO, J.:

FACTS: Petitioner joined the Bureau of Customs in 1972. In 1988, he was appointed Customs Service
Chief of the Customs Intelligence and Investigation Service (CIIS), reclassified by the Civil Service and
categorized as "Director III, CIIS" in 1980. In 1993, petitioner was temporarily designated as Acting
District Collector, Collection District X, Cagayan de Oro City, continuously receiving the salary and
benefits of a Director III. In his place, respondent Ray Allas was appointed as "Acting Director III" of the
CIIS.

In 1994, petitioner received a letter from Deputy Customs, informing him of his termination from the
Bureau, in view of respondent Allas' appointment as Director III by President Fidel V. Ramos. The
petitioner wrote the Customs Commissioner demanding his reinstatement with full back wages and
without loss of seniority rights, but no reply was made, prompting him to file a petition for  quo
warranto against respondent Allas before the RTC.  The case was tried and a decision was rendered,
granting the petition. Respondent Allas appealed to the Court of Appeals. In 1996, while the case was
pending before said court, respondent Allas was promoted by the President to the position of Deputy
Commissioner of Customs for Assessment and Operations, resulting to dismissal of the appeal and
rendering the decision final and executory.
On May 9, 1996, petitioner filed with the court a quo a Motion for Execution of its decision, which was
denied on the ground that the contested position vacated by respondent Allas was now being occupied by
respondent Godofredo Olores who was not a party to the quo warranto petition. Thus, Petitioner filed a
special civil action for certiorari and mandamus with the Court of Appeals questioning the order of the
trial court, which was dismissed.

ISSUE: Whether or not the Court of Appeals grossly erred in holding that a writ of execution may no
longer be issued on the ground that the contested position vacated was now occupied by a person who
was not a party to the quo warranto petition.

RULING: NO. The instant petition arose from a special civil action for quo warranto under Rule 66 of
the Revised Rules of Court. Quo warranto is a demand made by the state upon some individual or
corporation to show by what right they exercise some franchise or privilege appertaining to the state
which, according to the Constitution and laws of the land, they cannot legally exercise except by virtue of
a grant or authority from the state. In other words, a petition for quo warranto is a proceeding to
determine the right of a person to the use or exercise of a franchise or office and to oust the holder from
its enjoyment, if his claim is not well-founded, or if he has forfeited his right to enjoy the privilege.   The
action may be commenced for the Government by the Solicitor General or the fiscal  against individuals
who usurp a public office, against a public officer whose acts constitute a ground for the forfeiture of his
office, and against an association which acts as a corporation without being legally incorporated.  The
action may also be instituted by an individual in his own name who claims to be entitled to the
public office or position usurped or unlawfully held or exercised by another. 

Where the action is filed by a private person, he must prove that he is entitled to the controverted
position; otherwise respondent has a right to the undisturbed possession of the office.  If the court finds
for the respondent, the judgment should simply state that the respondent is entitled to the office. 
If it is found that the respondent or defendant is usurping or intruding into the office, or unlawfully
holding the same, the court may order:
(1) The ouster and exclusion of the defendant from office;
(2) The recovery of costs by plaintiff or relator;
(3) The determination of the respective rights in and to the office, position, right, privilege or
franchise of all the parties to the action as justice requires. 

The decision of the trial court had long become final and executory, and petitioner prays for its execution.
He alleges that he should have been reinstated despite respondent Olores' appointment because the subject
position was never vacant to begin with. Petitioner's removal was illegal and he was deemed never to
have vacated his office when respondent Allas was appointed to the same. Respondent Allas' appointment
was null and void and this nullity allegedly extends to respondent Olores, his successor-in-interest. 

Ordinarily, a judgment against a public officer in regard to a public right binds his successor in office.
This rule, however, is not applicable in quo warranto cases.  A judgment in quo warranto does not bind
the respondent's successor in office, even though such successor may trace his title to the same source.

Remedial Law Review II (CY 2018-2019) – Saturday 1:00pm – 5:00pm: Case Digest Compilation [Atty. Brondial] Page 108
This follows from the nature of the writ of quo warranto itself. It is never directed to an officer as such,
but always against the person — to determine whether he is constitutionally and legally authorized to
perform any act in, or exercise any function of the office to which he lays claim.  In the case at bar, the
petition for quo warranto was filed by petitioner solely against respondent Allas. What was threshed out
before the trial court was the qualification and right of petitioner to the contested position as against
respondent Ray Allas, not against Godofredo Olores. The Court of Appeals did not err in denying
execution of the trial court's decision.

DOCTRINE: A judgment in quo warranto does not bind the respondent's successor in office, even
though such successor may trace his title to the same source.

Remedial Law Review II (CY 2018-2019) – Saturday 1:00pm – 5:00pm: Case Digest Compilation [Atty. Brondial] Page 109
E.2. MA. LUTGARDA P. CALLEJA, JOAQUIN M. CALLEJA, JR., JADELSON PETER P.
CALLEJA, MA. JESSICA T. FLORES, MERCIE C. TIPONES and PERFECTO NIXON C.
TABORA vs. JOSE PIERRE A. PANDAY, AUGUSTO R. PANDAY and MA. THELNA P.
MALLARI.

G.R. No. 168696; February 28, 2006; AUSTRIA-MARTINEZ, J.:

FACTS: Respondents, members of the board and officers of St John hospital filed with RTC San Jose,
Cam Sur (RTC Branch 58) for quo warranto against petitioners, who are also among the incorporators
and stockholders, who forcibly and with the aid of armed men usurped the powers which supposedly
belonged to respondents.
Petitioners filed a motion to dismiss.

RTC 58 issued an order transferring the case to RTC Naga, residence of the respondents. RTC Naga
refused to receive the case folder since venue is not a ground for transferring quo warranto case. RTC 58
issued and served summons, and issued the assailed order denying the motion to dismiss and remanded
the case to RTC Naga, designated as special commercial court. Petitioners elevated the case to the CA via
petition for review on certiorari under Rule 45.

ISSUE 1: Whether or not Panday, et al. correctly resorted to a quo warranto under Rule 66 of the Rules
of Court?

RULING 1: The present Rule 66 only applies to actions of quo warranto against persons who usurp a
public office, position or franchise; public officers who forfeit their office; and associations which act as
corporations without being legally incorporated. It is, therefore, the Interim Rules of Procedure
Governing Intra-Corporate Controversies under R.A. No. 8799 that applies to the petition for quo
warranto filed by respondents before the trial court since what is being questioned is the authority of the
petitioners to assume the office and act as board of directors of St. John Hospital.

ISSUE2: Which court or tribunal may validly take cognizance of the present case?

RULING 2: RTC has jurisdiction over the principal office of the corporation. It is undisputed that the
principal office of the corporation is situated at Goa, Camarines Sur. Regional Trial Court designated as
Special Commercial Courts in Camarines Sur which shall have jurisdiction over the petition for quo
warranto filed by herein respondents.
RTC-Br. 58 did not have jurisdiction and should have dismissed the petition for lack of jurisdiction.

Since the official station of the designated Special Commercial Court for Camarines Sur is the Regional
Trial Court in Naga City, respondents should have filed their petition with said court.

Remedial Law Review II (CY 2018-2019) – Saturday 1:00pm – 5:00pm: Case Digest Compilation [Atty. Brondial] Page 110
E.3. LUIS K. LOKIN, JR., as the second nominee of CITIZENS BATTLE AGAINST
CORRUPTION (CIBAC), Petitioner, vs. COMMISSION ON ELECTIONS and the HOUSE OF
REPRESENTATIVES, Respondents.

G.R. Nos. 179431-32; June 22, 2010

EN BANC.LUIS K. LOKIN, JR., Petitioner, vs. COMMISSION ON ELECTIONS (COMELEC),


EMMANUEL JOEL J. VILLANUEVA, CINCHONA C. GONZALES and ARMI JANE R.
BORJE, Respondents.

G.R. No. 180443

FACTS: Citizens’ Battle Against Corruption (CIBAC) manifested its intent to participate in the May
2007 elections It then submitted the names of its 5 nominees:
(1) Emmanuel Joel J. Villanueva; (2) herein petitioner Luis K. Lokin, Jr.; (3) Cinchona C. Cruz-Gonzales;
(4) Sherwin Tugna; and (5) Emil L. Galang. CIBAC then filed a motion to amend the names of its
nominees, among others withdrew the nominations of Lokin, Tugna and Galang and substituted Armi
Jane R. Borje as one of the nominees. The amended list of nominees of CIBAC thus included: (1)
Villanueva, (2) Cruz-Gonzales, and (3) Borje. CIBAC then submitted signed petitions of more than 81%
of the CIBAC members, in order to confirm the withdrawal of the nomination of Lokin, Tugna and
Galang and the substitution of Borje, averred that Lokin and Tugna were not among the nominees
presented and proclaimed by CIBAC in its proclamation rally held in May 2007; and that Galang had
signified his desire to focus on his family life. CIBAC filed before the COMELEC En Banc a motion
seeking the proclamation of Lokin as its second nominee.

The right of CIBAC to a second seat as well as the right of Lokin to be thus proclaimed were purportedly
based on Party-List Canvass Report No. 26, which showed CIBAC to have garnered a grand total of
744,674 votes. Using all relevant formulas, the motion asserted that CIBAC was clearly entitled to a
second seat and Lokin to a proclamation.

Villanueva and Cruz-Gonzales opposed the motion. COMELEC failed to act on the matter. COMELEC
En Banc then declared that CIBAC was entitled to an additional seat. Then resolved E.M. No. 07-054:

WHEREFORE, considering the above discussion, the Commission hereby approves the withdrawal of the
nomination of Atty. Luis K. Lokin, Sherwin N. Tugna and Emil Galang as second, third and fourth
nominees respectively and the substitution thereby with Atty. Cinchona C. Cruz-Gonzales as second
nominee and Atty. Armi Jane R. Borje as third nominee for the party list CIBAC. The new order of
CIBAC's nominees therefore shall be: (1). Emmanuel Joel J. Villanueva; (2.) Cinchona C. Cruz-Gonzales;
(3.) Armi Jane R. Borje. THUS, COMELEC en banc proclaimed Cruz-Gonzales as the official second
nominee of CIBAC and the latter took oath and assumed office.

The COMELEC posits that once the proclamation of the winning party-list organization has been done
and its nominee has assumed office, any question relating to the election, returns and qualifications of the
candidates to the House of Representatives falls under the jurisdiction of the HRET pursuant to Section
17, Article VI of the 1987 Constitution. Thus, Lokin should raise the question he poses herein either in an
election protest or in a special civil action for quo warranto in the HRET, not in a special civil action for
certiorari in this Court. We do not agree. Lokin filed a Rule 65

The COMELEC posits that Lokin should raise the question he poses herein either in an election protest or
in a special civil action for quo warranto in the HRET, not in a special civil action for certiorari in this
Court.

ISSUE: Whether COMELEC is correct in its position that a quo warranto may be a correct remedy for
Lokin, and not a petition for certiorari.

RULING: NO.
An election protest proposes to oust the winning candidate from office. It is strictly a contest between the
defeated and the winning candidates, based on the grounds of electoral frauds and irregularities, to
determine who between them has actually obtained the majority of the legal votes cast and is entitled to
hold the office. It can only be filed by a candidate who has duly filed a certificate of candidacy and has
been voted for in the preceding elections.

Remedial Law Review II (CY 2018-2019) – Saturday 1:00pm – 5:00pm: Case Digest Compilation [Atty. Brondial] Page 111
A special civil action for quo warranto refers to questions of disloyalty to the State, or of ineligibility of
the winning candidate. The objective of the action is to unseat the ineligible person from the office, but
not to install the petitioner in his place. Any voter may initiate the action, which is, strictly speaking, not a
contest where the parties strive for supremacy because the petitioner will not be seated even if the
respondent may be unseated.

The controversy involving Lokin is neither an election protest nor an action for quo warranto, for it
concerns a very peculiar situation in which Lokin is seeking to be seated as the second nominee of
CIBAC. Although an election protest may properly be available to one party-list organization seeking to
unseat another party-list organization to determine which between the defeated and the winning party-list
organizations actually obtained the majority of the legal votes, Lokin’s case is not one in which a
nominee of a particular party-list organization thereby wants to unseat another nominee of the same party-
list organization. Neither does an action for quo warranto lie, considering that the case does not involve
the ineligibility and disloyalty of Cruz-Gonzales to the Republic of the Philippines, or some other cause
of disqualification for her.
Lokin has correctly brought this special civil action for certiorari against the COMELEC to seek the
review of the September 14, 2007 resolution of the COMELEC in accordance with Section 7 of Article
IX-A of the 1987 Constitution, notwithstanding the oath and assumption of office by Cruz-Gonzales. The
constitutional mandate is now implemented by Rule 64 of the 1997 Rules of Civil Procedure, which
provides for the review of the judgments, final orders or resolutions of the COMELEC and the
Commission on Audit. As Rule 64 states, the mode of review is by a petition for certiorari in accordance
with Rule 65 to be filed in the Supreme Court within a limited period of 30 days. Undoubtedly, the Court
has original and exclusive jurisdiction over Lokin’s petitions for certiorari and for mandamus against the
COMELEC.

Remedial Law Review II (CY 2018-2019) – Saturday 1:00pm – 5:00pm: Case Digest Compilation [Atty. Brondial] Page 112
E.4. EFREN RACEL ARATEA vs. COMMISSiON ON ELECTIONS and
ESTELA D. ANTlPOLO

G.R. No. 195229; October 9, 2012

FACTS: Lonzanida and Antipolo were candidates for 2010 Mayoralty candidates. Lonzanidawas
disqualify pursuant to Rodolfo’s petition under Section 78 of the OmnibusElection Code. The ground for
the disqualification is to the fact that Lonzanida waselected, and had served, as mayor of San Antonio,
Zambales for four (4)consecutive terms immediately prior to the term for the May 2010 elections.

Lonzanida filed an MR. Pending the resolution of the MR, 2010 Election resulted to Lonzanida and
Aratea garnered the highest number of votes and wererespectively proclaimed Mayor and Vice-Mayor.

Pursuant to a DILG opinion that in light of Lonzanida’s pending MR, the office of the mayor is
considered vacant, Aratea took his oath of office as Acting Mayor.On 11 August 2010, the COMELEC
En Banc resolved to deny Lonzanida’s MR,thereby upholding his disqualification. On 25 August 2010,
Antipolo, contender in2010 mayoralty election who garnered the second highest votes, filed a Motion
forLeave to Intervene and to Admit Attached Petition-in-Intervention. She claimed herright to be
proclaimed as Mayor.

In his Comment filed, Aratea opposed the Antipolo’s motion. However, COMELEC allowed the
intervention and ruled that Lonzanida’s disqualificationand ineligibility to hold public office is
established both in fact and in law onelection day itself. COMELEC further ruled that it should be
Antipolo and notAratea that should be the mayor.

ISSUE: Whether the second placer should in election should be declared as mayor in the event that the
winning candidate was disqualify.

RULING: In eligibility requirements and disqualification issues of election candidates, thereare three
remedies available, namely: (1) cancellation of a CoC; (2) disqualificationfrom candidacy or from
holding office; and (3) quo warranto. These three (3)remedies may be differentiated from each other into
three ways, i.e., as to grounds,period of filing and effects.

As to the grounds

In cancellation of a CoC, the ground is essentially lack of eligibility under the pertinent constitutional
and statutory provisions on qualifications or eligibility forpublic office; the governing provisions are
Sections 78 and 69 of the OEC. In adisqualification case, as mentioned above, the grounds are traits,
conditions, characteristics or acts of disqualification, individually applicable to a candidate.

The grounds for disqualification are different from, and have nothing to do with, a candidate’s CoC.
Section 78 of the Omnibus Election Code states that a certificateof candidacy may be denied or cancelled
when there is false material representation of the contents of the certificate of candidacy. Material
representation may relate tothe qualifications required of the public office he/she is running for.

In a quo warranto petition, the grounds to oust an elected official from his office are ineligibility and
disloyalty to the Republic of the Philippines. This is providedunder Section 253 of the OEC and governed
by the Rules of Court as to procedures.

While quo warranto and cancellation share the same ineligibility grounds, they differ as to the time these
grounds are cited. Under section 78, the qualifications forelective office are misrepresented in the
certificate of candidacy and theproceedings must be initiated before the elections, whereas a petition for
quowarranto under section 253 may be brought on the basis of two grounds - (1)ineligibility or (2)
disloyalty to the Republic of the Philippines, and must be initiatedwithin ten days after the proclamation
of the election results. Under section 253, acandidate is ineligible if he is disqualified to be elected to
office, and he is disqualified if he lacks any of the qualifications for elective office.

As to the period for filing:

The period to file a petition to deny due course to or cancel a CoC depends on the provision of law
invoked. If the petition is filed under Section 78 of the OEC, thepetition must be filed within twenty-five
(25) days from the filing of the CoC.

Remedial Law Review II (CY 2018-2019) – Saturday 1:00pm – 5:00pm: Case Digest Compilation [Atty. Brondial] Page 113
However, if the petition is brought under Section 69 of the same law, the petition must be filed within
five (5) days from the last day of filing the CoC.On the other hand, the period to file a disqualification
case is at any time before theproclamation of a winning candidate, as provided in COMELEC Resolution
No.8696.

As to the effects of a successful suit:

A candidate whose CoC was denied due course or cancelled is not considered a candidate at all. A
"candidate" whose CoC has been cancelled or denied due coursecannot be substituted for lack of a CoC,
to all intents and purposes. Similarly, asuccessful quo warranto suit results in the ouster of an already
elected official fromoffice; substitution, for obvious reasons, can no longer apply.

On the other hand, a candidate who was simply disqualified is merely prohibited from continuing as a
candidate or from assuming or continuing to assume thefunctions of the office; substitution can thus take
place under the terms of Section77 of the OEC.32 However, a three-term candidate with a valid and
subsisting CoCcannot be substituted if the basis of the substitution is his disqualification on accountof his
three-term limitation. Disqualification that is based on a breach of the threetermlimit rule cannot be
invoked as this disqualification can only take place afterelection where the three-term official emerged as
winner. As in a quo warranto, any substitution is too late at this point.

As to the effects of a successful suit on the right of the second placer in the elections:
In any of these three remedies, the doctrine of rejection of the second placer applies.
As an exceptional situation, however, the candidate with the second highest numberof votes (second
placer) may be validly proclaimed as the winner in the electionsshould the winning candidate be
disqualified by final judgment before the elections.

In a CoC cancellation proceeding, the law is silent on the legal effect of a judgment cancelling the CoC
and does not also provide any temporal distinction. Given,however, the formal initiatory role a CoC plays
and the standing it gives to apolitical aspirant, the cancellation of the CoC based on a finding of its
invalidityeffectively results in a vote for an inexistent "candidate" or for one who is deemednot to be in
the ballot. Although legally a misnomer, the "second placer" should beproclaimed the winner as the
candidate with the highest number of votes for thecontested position. This same consequence should
result if the cancellation case becomes final after elections, as the cancellation signifies non-candidacy
from thevery start, i.e., from before the elections.

In this case, the SC ruled that Lonzanida's certificate of candidacy was cancelled because he was
ineligible or not qualified to run for Mayor. Whetherhis certificate of candidacy is cancelled before
or after the elections isimmaterial because the cancellation on such ground means he was never
acandidate from the very beginning, his certificate of candidacy being void abinitio. There was only
one qualified candidate for Mayor in the May 2010elections - Anti polo, who therefore received the
highest number of Svotes.

NB: note that the petition filed against Lonzanida is not for the cancellation of hisCoC but for
disqualification for false misrepresenatation.

Remedial Law Review II (CY 2018-2019) – Saturday 1:00pm – 5:00pm: Case Digest Compilation [Atty. Brondial] Page 114
E.5. De Castro vs. Carlos, 696 SCRA 400.

G.R. No. 194994; April 16, 2013; Ponente:SERENO, C. J.

FACTS: On 29 July 2009, then President Gloria Macapagal-Arroyo appointed Emmanuel A. de Castro
(De Castro) as assistant general manager for operations (AGMO) of the Metropolitan Manila
Development Authority (MMDA). His appointment was concurred and thereafter took his oath.
Meanwhile, on 29 July 2010, Executive Secretary Paquito Ochoa issued Office of the President (OP)
Memorandum Circular No. 2, Series of 2010, amending OP Memorandum Circular No. 1, Series of 2010.
OP Memorandum Circular No. 2 states:
2. All non-Career Executive Service Officials (non-CESO) occupying Career
Executive Service (CES) positions in all agencies of the executive branch shall remain in
office and continue to perform their duties and discharge their responsibility until October
31, 2010 or until their resignations have been accepted and/or until their respective
replacements have been appointed or designated, whichever comes first, unless they are
reappointed in the meantime.

On 30 July 2010, Atty. Francis N. Tolentino, chairperson of the MMDAdesignated Corazon B. Cruz as
officer-in-charge (OIC) of the Office of the AGMO. De Castro was then reassigned to the Legal and
Legislative Affairs Office, Office of the General Manager.

Subsequently, Chairperson Tolentino designated respondentEmerson S. Carlos (Carlos) as OIC of the


Office of the AGMO. Thereafter, the name of De Castrowas stricken off the MMDA payroll, and he was
no longer paid his salary beginning November 2010.

De Castrosent a letter on demanding payment of his salary and reinstatement in the monthly payroll. For
his failure to obtain an action or a response from MMDA, he then made a formal demand for his
reinstatement as AGMO through a letter addressed to the Office of the President on 17 December 2010.

However, on 4 January 2011, President Benigno S. Aquino III (President Aquino) appointed Castro as the
new AGMO of the MMDA. On 10 January 2011, the latter took his oath of office.
Hence, De Castrofiled a Petition for the issuance of a writ of quo warranto under Rule 66 seeking to oust
respondent Carlos from the position of AGMOof the MMDA.

De Castro, being a non-CESO,contends that Section 2 (3), Article IX (B) of the 1987 Constitution
guarantees the security of tenure of employees in the civil service. He further argues that his appointment
as AGMO is not covered by OP Memorandum Circular No. 2, since it is not a CES position as
determined by the CESB.

ISSUE: Whether petitioner Emmanuel A. de Castro’s direct recourse to the Supreme Court was correct
and entitled to the position of AGMO, thus warrants the filing of the petition.

RULING: Petition denied.Although Section 5 (1) of Article VIII of the 1987 Constitution explicitly
provides that the Supreme Court has original jurisdiction over petitions for certiorari, prohibition,
mandamus, quo warranto, and habeas corpus, the jurisdiction of this Court is not exclusive but is
concurrent with that of the Court of Appeals and regional trial court and does not give De Castro
unrestricted freedom of choice of court forum. The hierarchy of courts must be strictly observed.

A direct invocation of this Court's jurisdiction is allowed only when there are special and important
reasons that are clearly and specifically set forth in a petition. The rationale behind this policy arises from
the necessity of preventing (1) inordinate demands upon the time and attention of the Court, which is
better devoted to those matters within its exclusive jurisdiction; and (2) further overcrowding of the
Court's docket.

In this case, De Castro justified his act of directly filing with this Court only when he filed his Reply and
after respondent had already raised the procedural infirmity that may cause the outright dismissal of the
present Petition. De Castro likewise cites stability in the civil service and protection of the rights of civil
servants as rationale for disregarding the hierarchy of courts.
De Castro's excuses are not special and important circumstances that would allow a direct recourse to this
Court. More so, mere speculation and doubt to the exercise of judicial discretion of the lower courts are
not and cannot be valid justifications to hurdle the hierarchy of courts. Thus, the Petition must be
dismissed.

Remedial Law Review II (CY 2018-2019) – Saturday 1:00pm – 5:00pm: Case Digest Compilation [Atty. Brondial] Page 115
Even assuming that De Castro's direct resort to this Court is permissible, the Petition must still be
dismissed for lack of merit.

"A petition for quo warranto is a proceeding to determine the right of a person to use or exercise a
franchise or an office and to oust the holder from the enjoyment, thereof, if the claim is not well-founded,
or if his right to enjoy the privilege has been forfeited." Where the action is filed by a private person, in
his own name, he must prove that he is entitled to the controverted position, otherwise, respondent has a
right to the undisturbed possession of the office.

An AGMO is a career position that enjoys security of tenure by virtue of the MMDA Charter, it is
undisputed that the position of AGMO is above the division chief level, which is equivalent to the rank of
assistant secretary with Salary Grade 29 and a perusal of the MMDA Charter readily reveals that the
duties and responsibilities of the position require the performance of executive and managerial
functions.Thus, the position of AGMO is within the coverage of the CES.
An AGMO should possess all the qualifications required by third-level career service within the CES. In
this case, De Castro does not have the required eligibility. Therefore, we find that his appointment to the
position of AGMO was merely temporary.

De Castro undisputedly lacked CES eligibility. Thus, he did not hold the position of AGMO in a
permanent capacity or acquire security of tenure in that position. Otherwise stated, his appointment was
temporary and "co-terminus with the appointing authority."
Even granting for the sake of argument that the position of AGMO is yet to be classified by the CESB, De
Castro's appointment is still deemed coterminous pursuant to CESB Resolution No. 945 issued on 14 June
2011.

Therefore, considering that De Castro is an appointee of then President Arroyo whose term ended on 30
June 2010, De Castro's term of office was also deemed terminated upon the assumption of President
Aquino.

Likewise, it is inconsequential that De Castro was allegedly replaced by another non-CESO eligible. In a
quo warranto proceeding, the person suing must show that he has a clear right to the office allegedly held
unlawfully by another. Absent a showing of that right, the lack of qualification or eligibility of the
supposed usurper is immaterial.

Remedial Law Review II (CY 2018-2019) – Saturday 1:00pm – 5:00pm: Case Digest Compilation [Atty. Brondial] Page 116
E.6. LORD ALLAN JAY Q. VELASCO, Petitioner, vs. HON. SPEAKER FELICIANO R.
BELMONTE, JR., SECRETARY GENERAL MARILYN B. BARUA-YAP AND REGINA
ONGSIAKO REYES, Respondents.

G.R. No. 211140; January 12, 2016; LEONARDO-DE CASTRO, J.

FACTS: Joseph Tan, a registered voter and resident of Marinduque, filed a petition to cancel the
Certificate of Candidacy (COC) of Regina Reyesdue to material misrepresentations in her COC i.e. (i)that
she is a resident of Brgy. Lupac, Boac, Marinduque; (ii) that she is a natural-born Filipino citizen; (iii)
that she is not a permanent resident of, or an immigrant to, a foreign country among other, as candidate as
a Representativefor the Lone District of Marinduque.COMELEC First Division and later on COMELEC
En Banc affirmed the decision to cancel Reyes’COC. Reyes won the said election whilethe said decision
of COMELEC En Bancbecame final and executory. Despite the receipt of COMELEC Resolution, the
Marinduque Provincial Board of Canvassers (PBOC) proclaimed Reyes as the winner.Hence, Lord Allan
Jay Q. Velasco filed an Election Protest Ad Cautelamand Petition for Quo Warranto Ad Cautelamagainst
Reyes in the House of Representatives Electoral Tribunal (HRET). The same was later on dismissed
HRET due to lack of jurisdiction. HRET’s jurisdiction begins only after Reyes is considered as a member
of House of Representatives, however, since her COC was declared cancelled by COMELEC, her
proclamation is invalid.

Despite the same, Speaker Belmonte, Jr. administered the oath of office to Reyes.Velasco filed a Petition
for Certiorari before the COMELEC assailing the proceedings of the PBOC and the proclamation of
Reyes. Said petition was denied while Reyes assumed office and started discharging the functions of a
Member of the House of Representatives. Upon Motion for Reconsideration of Velasco, COMELEC En
Banc reversed the denial and declared null and void the proclamation of Reyes and proclaimed Velasco as
the winning candidate and was later proclaimed for the position by the newly constituted PBOC. Velasco
then sent letters to Reyes essentially demanding that she vacate the office; to Speaker Belmonte, Jr.
requesting that he be allowed to assume the position, and to Sec. Gen. Barua-Yap to delete the name of
Reyes from the Roll of Members and register his name in her place. However, his requests were not
heeded.
Hence, Velasco field a Petition for Mandamus against Speaker Belmonte, Jr.; Sec. Gen. Barua-Yap; and
Reyes.

On her Comment, Reyes contends that:


1. The Petition is actually one for quo warrantoand not mandamus given that it essentially seeks a
declaration that she usurped the subject office;
2. The Petition is a collateral attack on a title to public office and thus must be dismissed.
3. The Court has no jurisdiction over Quo Warrantocases involving Members of the House of
Representatives as it is the HRET that has the sole and exclusive jurisdiction over all contests
relating to the election, returns and qualifications of Members of the House of Representatives.
4. Velasco is not the first choice of the people in the election, and a second placer cannot take place
the position even if it was proven that Reyes usurped the position.
The Office of the Solicitor General (OSG), arguing for Speaker Belmonte, Jr. and Sec. Gen. Barua-Yap,
opposed Velasco's petition stating the same arguments as Reyes.

ISSUE: Whether or not the petition is one for quo warranto?

RULING: No. A petition for quo warrantois a proceeding to determine the right of a person to the use or
exercise of a franchise or office and to oust the holder from its enjoyment, if his claim is not well-
founded, or if he has forfeited his right to enjoy the privilege. Where the action is filed by a private
person, he must prove that he is entitled to the controverted position; otherwise, respondent has a right to
the undisturbed possession of the office.In this case, the final and executory resolutions of COMELEC in
cancelling Reyes's COC; in declaring null and void the proclamation of Reyes and proclaiming Velasco
as the winning candidate cannot be claimed that the present petition is one for the determination of the
right of Velasco to the claimed office.To be sure, what is prayed for herein is merely the enforcement of
clear legal duties and not to try disputed title. That the respondents make it appear so will not convert this
petition to one for quo warranto.

The present Petition for Mandamus seeks the issuance of a writ of mandamus to compel respondents
Speaker Belmonte, Jr. and Sec. Gen. Barua-Yap to acknowledge and recognize the final and executory
Decisions and Resolution of this Court and of the COMELEC by administering the oath of office to
Velasco and entering the latter's name in the Roll of Members of the House of Representatives. The

Remedial Law Review II (CY 2018-2019) – Saturday 1:00pm – 5:00pm: Case Digest Compilation [Atty. Brondial] Page 117
administration of oath and the registration of Velasco in the Roll of Members of the House of
Representatives for the Lone District of the Province of Marinduqueare no longer a matter of discretion
or judgment on the part of Speaker Belmonte, Jr. and Sec. Gen. Barua-Yap. They are legally duty-bound
to recognize Velasco as the duly elected Member of the House of Representatives for the Lone District of
Marinduque in view of the ruling rendered by this Court and the COMELEC'S compliance with the said
ruling, now both final and executory.

DOCTRINE: A PETITION FOR QUO WARRANTO will not prosper if the issue is the enforcement of
one’s clear right to the claimed office which has already been held as final and executoryand not merely
a determination of the right to the claimed office.

Remedial Law Review II (CY 2018-2019) – Saturday 1:00pm – 5:00pm: Case Digest Compilation [Atty. Brondial] Page 118
F. Expropriation (Rule 67)

F.1. City of Manila, petitioner vs. Oscar, Felicitas, Jose, Benjamin, Estelita, Leonora and Adelaida,
all surnamed SERRANO, respondents

G.R. No. 142304; June 20, 2001; Mendoza, J.

FACTS: On December 21, 1993, the City Council of Manila enacted the Ordinance No. 7833,
authorizing the expropriation of certain properties in Manila 's First District in Tondo whichare to be sold
and distributed to qualified occupants pursuant to the Land Use Development Program of the City of
Manila. Upon motion by petitioner, the trial court issued an order, dated October 9, 1998, directing
petitioner to deposit the amount of Pl,825,241.00 equivalent to the assessed value of the properties. After
petitioner had made the deposit, the trial court issued another order, dated December 15, 1998, directing the
issuance of a writ of possession in favor of petitioner. Respondents opposedthe propriety of the
expropriation, contending that petitioners have not resorted to other modes of acquisition as required
under Sections 9-10 of RA 7279, which was denied by the trial court but upon elevation to the CA was
granted by the latter.

ISSUE: Whether or not it the issuance of the order of a writ of possession is tantamount to an order of
condemnation?

RULING: No.Rule 67, Section 2 provides that "Upon the filing of the complaint or at any time thereafter
and after due notice to the defendant, the plaintiff shall have the right to take or enter upon the
possession of the real property involved if he deposits with the authorized government depositary an
amount equivalent to the assessed value of the property for purposes of taxation to be held by such bank
subject to the orders of the court. Such deposit shall be in money, unless in lieu thereof the court
authorizes the deposit of a certificate of deposit of a government bank of the Republic of the Philippines
payable on demand to the authorized government depositary. If personal property is involved, its value
shall be provisionally ascertained and the amount to be deposited shall be fixed by the court. After such
deposit is made the court shall order the sheriff or other proper officer to forthwith place the plaintiff in
possession of the property involved and promptly submit a report thereof to the court with service of
copies to the parties."Thus,a writ of execution may be issued by a court upon the filing by the government
of a complaint for expropriation sufficient in form and substance and upon deposit made by the
government of the amount equivalent to the assessed value of the property subject toexpropriation. Upon
compliance with these requirements, the issuance of the writ of possession becomes ministerial. In this
case, these requirements were satisfied and, therefore, it became the ministerial duty of the court to issue
the writ of possession. However, this does not amount to an order of condemnation. Whether petitioner has
complied with these provisions requires the presentation of evidence, although in its amended complaint
petitioner did allege that it had complied with the requirements. The determination of this question must
await that hearing on the complaint for expropriation, particularly the hearing for the condemnation of the
properties sought to be expropriated. Expropriation proceedings consist of two stages: first, condemnation
of the property after it is determined that its acquisition will be for a public purpose or public use and,
second, the determination of just compensation to be paid for the taking of the private property to be
made by the court with the assistance of not more than three commissioners.

DOCTRINE:The entry of the plaintiff upon the property in an expropriation proceeding pursuant to a
writ of possession is not an order of condemnation. It merely gives the plaintiff the right of possession.

Remedial Law Review II (CY 2018-2019) – Saturday 1:00pm – 5:00pm: Case Digest Compilation [Atty. Brondial] Page 119
F.2. NATIONAL POWER CORPORATION, petitioner, vs. COURT OF APPEALS and
MACAPANTON MANGONDATO, respondents.

G.R. No. 113194; March 11, 1996; PANGANIBAN, J.:

FACTS: 1978, NAPOCORtook possession of a 21,995 square meter land which is a portion of Lot 1 of
the subdivision plan situated in Marawi City, owned by Mangondatounder the mistaken belief that it
forms part of the public land reserved for use by NAPOCOR for hydroelectric power purposes.
NAPOCOR alleged that the subject land was until then possessed and administered by Marawi City so
that in exchange for the city’s waiver and quitclaim of any right over the property, NAPOCOR had paid
the city a financial assistance of P40.00 per square meter.

When NAPOCOR started buildingHydroelectric Plant, Mangondato demanded compensation from


NAPOCOR. NAPOCOR refused to compensate insisting that the property is public land and that it had
already paid financial assistance to Marawi City in exchange for the rights over the property.Mangondato
claimed that the subject land is his duly registered private property that he is not privy to any agreement
between NAPOCOR and Marawi City and that any payment made to said city cannot be considered as
payment to him.
More than a decade later NAPOCOR acceded to the fact that the property belongs to
Mangondato.NAPOCOR passed a resolution resolving to pay Mangondato100.00/SQM as payment for
the land of Mangondato.

Mangondato disagree with such determination and insist that the FMV of his property is higher than 100.
In his manifestation in lieu of Answer Mangondato prayed that he be compensated in damages for the
unauthorized taking and continued possession of his land from 1978 until the filing of the Complaiant in
the expropriation case; that should the lower court order the expropriation of the subject property, that the
just compensation for the land be reckoned from the time of the filing of the expropriation case; that the
expropriation case be consolidated with the recovery of possession case; that the restraining order issued
in the recovery of possession case be maintained and a writ of preliminary injunction be at once issued
against NAPOCOR; and that NAPOCOR be ordered to deposit the value of the land as provisionally
determined by the lower court.

ISSUES:
1. At what point in time should the value of the land subject of expropriation be computed, is it at
the date of the taking or the date of the filing of the complaint for eminent domain? 
2. When is There Taking of Property?

RULING: The petition of NAPOCOR is dismissed. SC affirmed the appellate court decision but amend
the legal interest from 12 % to 6 %.
1. General rule, in determining just compensation in eminent domain the value of the property as
of the date of the filing of the complaint
Sec. 4. Order of Condemnation. When such a motion is overruled or when any party fails to
defend as required by this rule, the court may enter an order of condemnation declaring that the
plaintiff has a lawful right to take the property sought to be condemned, for the public use or
purpose described in the complaint, upon the payment of just compensation to be determined as
of the date of the filing of the complaint 

Normally, the time of the taking coincides with the filing of the complaint for expropriation. Hence, many
rulings of this Court have equated just compensation with the value of the property as of the time of filing
of the complaint consistent with the above provision of the Rules.

The general rule, however, admits of an exception: where the Court fixed the value of the property as of
the date it was taken and not at the date of the commencement of the expropriation proceedings.

Simply stated, the exception finds application where the owner would be given undue incremental
advantages arising from the use to which the government devotes the property expropriated -as for
instance, the extension of a main thoroughfare as was the case in Caro de Araullo. In the instant case,
however, it is difficult to conceive of how there could have been an extra-ordinary increase in the value
of the owner’s land arising from the expropriation, as indeed the records do not show any evidence that
the valuation of P1,000.00 reached in 1992 was due to increments directly caused by petitioners use of the
land. Since the petitioner is claiming an exception to Rule 67, Section 4, it has the burden of proving its

Remedial Law Review II (CY 2018-2019) – Saturday 1:00pm – 5:00pm: Case Digest Compilation [Atty. Brondial] Page 120
claim that its occupancy and use - not ordinary inflation and increase in land values - was the direct cause
of the increase in valuation from 1978 to 1992.

The fair market value as held by the respondent Court, is the amount of P1,000.00 per square meter. In an
expropriation case where the principal issue is the determination of just compensation, as is the case here,
a trial before Commissioners is indispensable to allow the parties to present evidence on the issue of just
compensation.Inasmuch as the determination of just compensation in eminent domain cases is a judicial
function and factual findings of the Court of Appeals are conclusive on the parties and reviewable only
when the case falls within the recognized exceptions, which is not the situation obtaining in this petition,
we see no reason to disturb the factual findings as to valuation of the subject property. As can be gleaned
from the records, the court-and-the-parties-appointed commissioners did not abuse their authority in
evaluating the evidence submitted to them nor misappreciate the clear preponderance of evidence. The
amount fixed and agreed to by the respondent appellate Court is not grossly exorbitant.

In sum, the petitioner has failed to show why it should be granted an exemption from the general rule in
determining just compensation provided under Section 4 of Rule 67. On the contrary, private respondent
has convinced the court that indeed, such general rule should in fact be observed in this case.

2. the elements of taking as the main ingredient in the exercise of power of eminent domain;
a. the expropriator must enter a private property;
b. the entrance into private property must be for more than a momentary period;
c. the entry into the property should be under warrant or color of legal authority; 
d. the property must be devoted to a public use or otherwise informally appropriated or
injuriously affected; and
e. the utilization of the property for public use must be in such a way to oust the owner and
deprive him of all beneficial enjoyment of the property;

Clearly, this is not the intent, nor the expropriation contemplated by law. This is a simple attempt at a
voluntary purchase and sale. Obviously, the petitioner neglected and/or refused to exercise the power of
eminent domain.

In the instant case, petitioner effectively repudiated the deed of sale it entered into with the private
respondent when it passed Resolution on May 25, 1992 authorizing its president to negotiate, inter
alia, that payment shall be effected only after Agus I HE project has been placed in operation. It was only
then that petitioner’s intent to expropriate became manifest as private respondent disagreed and, barely a
month after, filed suit.

DOCTRINES:
 General rule,it is the value of the property as of the date of the filing of the complaint is
the basis in determining just compensation in eminent domain;

Except, where the owner would be given undue incremental advantages arising from the
use to which the government devotes the property expropriated in which case the date it
was taken shall be the basis;

 Elements of taking as the main ingredient in the exercise of power of eminent domain;

Remedial Law Review II (CY 2018-2019) – Saturday 1:00pm – 5:00pm: Case Digest Compilation [Atty. Brondial] Page 121
F.3. REPUBLIC OF THE PHILIPPINES VS ISMAEL ANDAYA

G.R. NO. 160656; June 15, 2007; Quisumbing, J.:

FACTS: Respondent Andaya is the registered owner of two parcels of land in Bading, Butuan City. The
Republic negotiated with Andaya to enforce the 60-m easement of right-of-way. The easement was for
concrete levees and floodwalls for Phase 1, stage 1 of the Lower Agusan Development Project. The
parties, however, failed to reach an agreement.

On December 13, 1995, the Republic instituted an action to enforce the easement of right-of-way or
eminent domain, the Trial Court issued a writ of possession on April 26, 1996. It also constituted a Board
of Commissioners (Board) to determine the just compensation. Eventually, the trial court issued an Order
of Expropriation upon payment of just compensation.

On December 10, 1998, the Board reported that the project would affect a total of 10,380 square meters of
Andaya’s properties, 4,443 square meters of which will be for 60-meter easement. The Board reported
that the easement would diminish the value of the remaining 5,937 square meter. As a result, it
recommended the payment of consequential damages amounting to ₱2,820,430.00 for the remaining area.

The trail court then rendered its decision granting the 2,820,430.00 as fair and reasonable severance
damages and other fees.

Both parties appealed to the Court of Appeals. Republic contested the awards of severance and damages
and attorney’s fees while Andaya demanded just compensation for his entire property minus the
easement. For he alleged that the easement would prevent ingress and egress to his property and turn it
into a catch basin for the floodwaters coming from Agusan River. As a result, his entire property would
be rendered unusable and uninhabitable. Hence, the instant petition

ISSUE: Whether or not the Republic is liable for just compensation if in enforcing the legal easement of
right-of-way on a property, the remaining area would be rendered unusable and uninhabitable. (Indirect
expropriation)

RULING: The answer is in the affirmative. “Taking” in the exercise of eminent domain, occurs not only
when the government actually deprives or dispossesses the property owner of his property or of its
ordinary use, but also when there is practical destruction or material impairment of the valueof the
property. Using this standard, there was undoubtedly a taking of the remaining area of Andaya’s property.
But, as correctly observed by the Board and affirmed by the court a quo, the nature and effect of the flood
walls would deprive Andaya of the normal use of the remaining areas. It would prevent ingress and egress
to the property and turn it into a catch basin for the floodwaters coming from the Agusan River.

For this reason, in our view, Andaya is entitled to payment of just compensation, which must be neither
more nor less than the monetary equivalent of the land.

DOCTRINE: An essential element of due process is that there must be just compensation whenever
private property is taken for public use.

Remedial Law Review II (CY 2018-2019) – Saturday 1:00pm – 5:00pm: Case Digest Compilation [Atty. Brondial] Page 122
F.4. ASIA'S EMERGING DRAGON CORPORATION, petitioner, vs. DOTC, SEC. LEANDRO R.
MENDOZA and MIAA, respondents.

G.R. No. 169914; April 18, 2008

FACTS: The present case is a Petition for Mandamus and Prohibition with Application for Temporary
Restraining Order filed by herein petitioner before the SC. This petition sprouted from 2separate
decisions of the SC regarding the same subject matter, which is the NAIA IPT III.

In August 1989, the Department of Trade and Communications (DOTC) engaged the services of Aeroport
de Paris (ADP) to conduct a comprehensive study of the Ninoy Aquino International Airport (NAIA) and
determine whether the present airport can cope with the traffic development up to the year 2010. Some
time in 1993, six business leaders consisting of John Gokongwei, Andrew Gotianun, Henry Sy, Sr., Lucio
Tan, George Ty and Alfonso Yuchengco met with then President Fidel V.Ramos to explore the possibility
of investing in the construction and operation of a new international airport terminal. To signify their
commitment to pursue the project, they formed the Asia's Emerging Dragon Corp. (AEDC), which was
registered, with the Securities and Exchange Commission (SEC) on September 15, 1993.

On October 5, 1994, AEDC submitted an unsolicited proposal to the Government through the
DOTC/Manila International Airport Authority (MIAA) for the development of NAIA International
Passenger Terminal III (NAIA IPT III) under a build-operate-and-transfer arrangement pursuant to RA
6957 as amended by RA 7718 (BOT Law).

On December 2, 1994, the DOTC issued Dept. Order No. 94-832 constituting the Prequalification Bids
and Awards Committee (PBAC) for the implementation of the NAIA IPT III project.

From the DOTC, then DOTC Secretary Jose Garcia endorsed the proposal of AEDCto the National
Economic and Development Authority (NEDA) which on 13 february1996 by virtue of its Board
Resolution No.2 approved the NAIA IPT III.

From then on, DOTC/MIAA caused the publication in two daily newspapers of an invitation for
competitive or comparative proposals on AEDC's unsolicited proposal for the said project. Among the
bidders was then People’s Air Cargo & Warehousing Co., Inc. (Pair Cargo), which on February 27, 1997
incorporated into Philippines International Airport Terminals Co., Inc. otherwise known as PIATCO.

AEDC failed to match PIATCO’s bid hence PIATCO won the bidding. This paved way to the filing of
the first case entitled Aganvs. PIATCO. AEDC filed before the RTC Pasig petitioning for the Declaration
of Nullity of the Proceedings, Mandamus and Injunction on 16 April 1997. Meanwhile, several
employees of the MIAA likewise filed a petition assailing the legality of various agreements
entered into by the Government with PIATCO (Concession Agreements) on the ground that they
stand to lose their employment upon the implementation of the questioned agreements.

1st Case: Agan, Jr. vs PIATCO (Null and Void Contract)

SC held that the contract for the construction, operation and maintenance of the NAIA IPT III is null and
void for the absence of the requisite financial capacity of therein-respondent PIATCO. However, since
the structures of the NIAA IPT III facility are almost complete, the government has to compensate
PIATCO as builder, just compensation in order to take over the facility, in accordance with the law and
equity.

This pronouncement in Agan case gave rise to the 2nd case, which is a complaint for expropriation.

2nd case: Republic v. Gingoyon (Complaint for Expropriation) On 21 December 2004, the Government
filed a Complaint for expropriation with the Pasay City RTC. The petition prayed for issuance of a writ of
possession in favor ofthe Government. The case was raffled to Branch 117 of the Pasay City RTC,
presided by respondent judge Hon. Henrick F. Gingoyon (Hon. Gingoyon). On the same day that the
Complaint was filed, the RTC issued an Order directing the issuance of a writ of possession to the
Government, authorizing it to "take or enter upon the possession" of the NAIA 3 facilities.

However, on 4 January 2005, the RTC issued another Order designed to supplement its 21 December2004
Order and the Writ of Possession. In the 4 January 2005 Order, now assailed in the present petition, the
RTC noted that its earlier issuance of its writ of possession was pursuant to Section 2, Rule 67 of the

Remedial Law Review II (CY 2018-2019) – Saturday 1:00pm – 5:00pm: Case Digest Compilation [Atty. Brondial] Page 123
1997Rules of Civil Procedure. However, it was observed that Republic Act No. 8974, otherwise known as
"An Act to Facilitate the Acquisition of Right-of-Way, Site or Location for National Government
Infrastructure Projects and For Other Purposes" and its Implementing Rules and Regulations
(Implementing Rules) had amended Rule 67 in many respects.

There are at least two crucial differences between the respective procedures under Rep. Act No. 8974 and
Rule 67. Under the statute, the Government is required to make immediate payment to the property owner
upon the filing of the complaint to been titled to a writ of possession, whereas in Rule 67, the Government
is required only to make an initial deposit with an authorized government depositary. Moreover, Rule 67
prescribes that the initial deposit be equivalent to the assessed value of the property for purposes of
taxation, unlike Rep. Act No. 8974 which provides, as the relevant standard for initial compensation, the
market value of the property as stated in the tax declaration or the current relevant zonal valuation of the
Bureau of Internal Revenue (BIR), whichever is higher, and the value of the improvements and/or
structures using the replacement cost method.

In Gingoyon, the Court ruled that Agan case could be construed as mandating the full payment of the just
compensation before the Government may be permitted to take over NAIA 3 which is in accord with
Republic Act No. 8974 and with equitable standard that allowed the government to take possession of
NAIA 3 after payment of the proffered value of the facilities to PIATCO.

ISSUE: Whether the filing by the Government of the complaint for expropriation is proper.

RULING: Yes. The Court then, in Gingoyon, directly addressed the issue on the appropriateness of
the Republic's resort to expropriation proceedings.

We appreciate that the case at bar is a highly unusual case, whereby the Government seeks to expropriate
a building complex constructed on land which the State already owns.

The right of eminent domain extends to personal and real property, and the NAIA 3 structures, adhered as
they are to the soil are considered as real property. It shouldbe noted that Section 1 of Rule 67 (on
Expropriation) recognizes the possibility that the property sought to be expropriated may be titled in the
name of the Republic of the Philippines, although occupied by private individuals, and in such case an
averment to that effect should be made in the complaint. The instant expropriation complaint did aver that
the NAIA 3 complex "stands on a parcel of land owned by the Bases Conversion Development Authority,
another agency of [the Republic of the Philippines]."

Eminent domain though may be the most effective, as well as the speediest means by which such goals
may be accomplished. Not only does it enable immediate possession after satisfaction of the requisites
under the law, it also has a built-in procedure through which just compensation may be ascertained.

PIATCO's ownership over the structures it had built in NAIA IPT III:
In Gingoyon, the court said that the 2004 Resolution squarely recognized that right when it mandated the
payment of just compensation to PIATCO prior to the takeover by the Government of NAIA 3.

The expropriation proceedings shall be held in accordance with Republic Act No. 8974, thus:Again
in Gingoyon, the court stated that the 2004 Resolution does not particularize the extent such payment
must be effected before the take over, but it unquestionably requires at least some degree of payment to
the private property owner before a writ of possession may issue. The utilization of Rep. Act No. 8974
guarantees compliance with this bare minimum requirement, as it assures the private property owner the
payment of, at the very least, the proffered value of the property to be seized. Such payment of the
proffered value to the owner, followed by the issuance of the writ of possession in favor of the
Government, is precisely the schematic under Rep. Act No. 8974, one which facially complies with the
prescription laid down in the 2004 Resolution.

And finally, as to the determination of the amount due PIATCO, this Court ruled in Gingoyon
that:Under Rep. Act No. 8974, the Government is required to "immediately pay" the owner of the
property the amount equivalent to the sum of:
(1) One hundred percent (100%) of the value of the property based on the current relevant zonal valuation
of the [BIR]; and
(2) The value of the improvements and/or structures as determined under Section 7.
Since, the BIR zonal valuation cannot apply in this case, the amount subject to immediate payment should
be limited to "the value of the improvements and/or structures as determined under Section 7, of the IRR.

Remedial Law Review II (CY 2018-2019) – Saturday 1:00pm – 5:00pm: Case Digest Compilation [Atty. Brondial] Page 124
Under the present IRR, the valuations of the improvement/structure are to be based using "the
replacement cost method. In addition to the statute, the 2004 Resolution in Agan also mandated that the
payment of just compensation should be in accordance with equity as well.

From the forgoing there is no way, to immediately ascertain the value of the improvements and structures
since such valuation is a matter for factual determination. Yet Rep. Act No. 8974 permits an expedited
means by which the Government can immediately take possession of the property without having to await
precise determination of the valuation. Section 4(c) of Rep. Act No. 8974 states that" in case the
completion of a government infrastructure project is of utmost urgency and importance, and there
is no existing valuation of the area concerned, the implementing agency shall immediately pay the
owner of the property its proffered value, taking into consideration the standards prescribed in
Section 5 [of the law]."The "proffered value" may strike as a highly subjective standard based solely on
the intuition of the government, but Rep. Act No. 8974 does provide relevant standards by
which"proffered value" should be based, as well as the certainty of judicial determination of the propriety
of the proffered value.

In filing the complaint for expropriation, the Government alleged to have deposited the amount of P3
Billion earmarked for expropriation, representing the assessed value of the property. The making of the
deposit, including the determination of the amount of the deposit, was undertaken under the erroneous
notion that Rule 67, and not Rep. Act No. 8974, is the applicable law. Still, as regards the amount, the
Court sees no impediment to recognize this sum of P3 Billion as the proffered value under Section 4(b) of
Rep. Act No. 8974. After all, in the initial determination of the proffered value, the Government is
not strictly required to adhere to any predetermined standards, although its proffered value may
later be subjected to judicial review using the standards enumerated under Section 5 of Rep. Act No.8974.

Remedial Law Review II (CY 2018-2019) – Saturday 1:00pm – 5:00pm: Case Digest Compilation [Atty. Brondial] Page 125
F.5. Spouses Abad vs. Fil-Homes Realty and Development Corporation and
Magdiwang Realty Corporation

G.R. No. 189239; November 24, 2010; Carpio Morales, J.

FACTS: Fil-Homes Realty and Development Corporation and Magdiwang Realty Corporation, co-
owners of two lots situated in Sucat, Parañaque filed a complaint for unlawful detainer on May 7, 2003
against above-named petitioners before the Parañaque Metropolitan Trial Court (MeTC).

Respondents alleged that petitioners, through tolerance, had occupied the subject lots since 1980 but
ignored their repeated demands to vacate them. Petitioners countered that there is no possession by
tolerance for they have been in adverse, continuous and uninterrupted possession of the lots for more than
30 years; and that respondent’s predecessor-in-interest, Pilipinas Development Corporation, had no title
to the lots. In any event, they contend that the question of ownership must first be settled before the issue
of possession may be resolved.

During the pendency of the case, the City of Parañaque filed expropriation proceedings covering the lots
before the Regional Trial Court of Parañaque with the intention of establishing a socialized housing
project therein for distribution to the occupants including petitioners. A writ of possession was
consequently issued and a Certificate of Turn-over given to the City.

Branch 77 of the MeTC rendered judgment in the unlawful detainer case against petitioners. The MeTC
held that as no payment had been made to respondents for the lots, they still maintain ownership thereon.
It added that petitioners cannot claim a better right by virtue of the issuance of a Writ of Possession for
the project beneficiaries have yet to be named. On appeal, the RTC, reversed the MeTC decision and
dismissed respondents’ complaint, ruling that the unlawful detainer requires the significant element of
"tolerance". Tolerance of the occupation of the property must be present right from the start of the
defendants’ possession. The phrase "from the start of defendants’ possession" is significant. When there is
no "tolerance" right from the start of the possession sought to be recovered, the case of unlawful detainer
will not prosper.

Then, RTC ruled that the issuance of a writ of possession in favor of the City bars the continuation of the
unlawful detainer proceedings, and since the judgment had already been rendered in the expropriation
proceedings which effectively turned over the lots to the City, the MeTC has no jurisdiction to "disregard
the final judgment and writ of possession" due to non-payment of just compensation.

Before the Court of Appeals where respondents filed a petition for review, they maintained that
respondents’ "act of allowing several years to pass without requiring them to vacate nor filing an
ejectment case against them amounts to acquiescence or tolerance of their possession." CA granted the
petition. Hence, the filing of this Petition to the SC.

ISSUE: Whether or not an ejectment proceeding due to its summary nature are NOT suspended or their
resolution held in abeyance despite the pendency of a civil action regarding ownership.

RULING: In the exercise of the power of eminent domain, the State expropriates private property for
public use upon payment of just compensation. A socialized housing project falls within the ambit of
public use as it is in furtherance of the constitutional provisions on social justice.

As a general rule, ejectment proceedings, due to its summary nature, are not suspended or their resolution
held in abeyance despite the pendency of a civil action regarding ownership.
Section 1 of Commonwealth Act No. 538 enlightens, however:

Section 1. When the Government seeks to acquire, through purchase or expropriation proceedings, lands
belonging to any estate or chaplaincy (cappellania), any action for ejectment against the tenants
occupying said lands shall be automatically suspended, for such time as may be required by the
expropriation proceedings or the necessary negotiations for the purchase of the lands, in which latter case,
the period of suspension shall not exceed one year. To avail himself of the benefits of the suspension, the
tenants shall pay to the landowner the current rents as they become due or deposit the same with the court
where the action for ejectment has been instituted.

Petitioners did not comply with any of the acts mentioned in the law to avail of the benefits of the
suspension. They nevertheless posit that since the lots are the subject of expropriation proceedings,

Remedial Law Review II (CY 2018-2019) – Saturday 1:00pm – 5:00pm: Case Digest Compilation [Atty. Brondial] Page 126
respondents can no longer assert a better right of possession; and that the City Ordinance authorizing the
initiation of expropriation proceedings designated them as beneficiaries of the lots, hence, they are
entitled to continue staying there. Petitioners’ position does not lie. The exercise of expropriation by a
local government unit is covered by Section 19 of the Local Government Code (LGC):

SEC. 19. Eminent Domain. – A local government unit may, through its chief executive and acting
pursuant to an ordinance, exercise the power of eminent domain for public use, or purpose, or welfare for
the benefit of the poor and the landless, upon payment of just compensation, pursuant to the provisions of
the Constitution and pertinent laws: Provided, however, That the power of eminent domain may not be
exercised unless a valid and definite offer has been previously made to the owner, and such offer was not
accepted: Provided, further, That the local government unit may immediately take possession of the
property upon the filing of the expropriation proceedings and upon making a deposit with the proper court
of at least fifteen percent (15%) of the fair market value of the property based on the current tax
declaration of the property to be expropriated: Provided, finally, That the amount to be paid for the
expropriated property shall be determined by the proper court, based on the fair market value of the
property.

The Lintag v. National Power Corporation clearly outlines the stages of expropriation, viz: Expropriation
of lands consists of two stages: The first is concerned with the determination of the authority of the
plaintiff to exercise the power of eminent domain and the propriety of its exercise in the context of the
facts involved in the suit. It ends with an order, if not of dismissal of the action, "of condemnation
declaring that the plaintiff has a lawful right to take the property sought to be condemned, for the public
use or purpose described in the complaint, upon the payment of just compensation to be determined as of
the date of the filing of the complaint x x x.

The second phase of the eminent domain action is concerned with the determination by the court of "the
just compensation for the property sought to be taken." This is done by the court with the assistance of not
more than three (3) commissioners x x x .

It is only upon the completion of these two stages that expropriation is said to have been completed. The
process is not complete until payment of just compensation. Accordingly, the issuance of the writ of
possession in this case does not write finis to the expropriation proceedings. To effectuate the transfer of
ownership, it is necessary for the NPC to pay the property owners the final just compensation.

In the present case, the mere issuance of a writ of possession in the expropriation proceedings did not
transfer ownership of the lots in favor of the City. Such issuance was only the first stage in expropriation.
There is even no evidence that judicial deposit had been made in favor of respondents prior to the City’s
possession of the lots, contrary to Section 19 of the LGC. Respecting petitioners’ claim that they have
been named beneficiaries of the lots, the city ordinance authorizing the initiation of expropriation
proceedings does not state so. Petitioners cannot thus claim any right over the lots on the basis of the
ordinance. Even if the lots are eventually transferred to the City, it is non sequitur for petitioners to claim
that they are automatically entitled to be beneficiaries thereof. For certain requirements must be met and
complied with before they can be considered to be beneficiaries.

In another vein, petitioners posit that respondents failed to prove that their possession is by mere
tolerance. This too fails. Apropos is the ruling in Calubayan v. Pascual:
In allowing several years to pass without requiring the occupant to vacate the premises nor filing an
action to eject him, plaintiffs have acquiesced to defendant’s possession and use of the premises. It has
been held that a person who occupies the land of another at the latter’s tolerance or permission, without
any contract between them, is necessarily bound by an implied promise that he will vacate upon demand,
failing which a summary action for ejectment is the proper remedy against them. The status of the
defendant is analogous to that of a lessee or tenant whose term of lease has expired but whose occupancy
continued by tolerance of the owner. In such a case, the unlawful deprivation or withholding of
possession is to be counted from the date of the demand to vacate.

Respondents bought the lots from Pilipinas Development Corporation in 1983. They stepped into the
shoes of the seller with respect to its relationship with petitioners. Even if early on respondents made no
demand or filed no action against petitioners to eject them from the lots, they thereby merely maintained
the status quo – allowed petitioners’ possession by tolerance.

Remedial Law Review II (CY 2018-2019) – Saturday 1:00pm – 5:00pm: Case Digest Compilation [Atty. Brondial] Page 127
F.6. NATIONAL POWER CORPORATION vs. COURT OF APPEALS and ANTONINO POBRE

G.R. No. 106804; August 12, 2004; CARPIO, J.

FACTS: NPC is authorized by law to acquire property and exercise the right of eminent domain. Private
respondent Pobre is the owner of a 68,969 square-meter land located in Tiwi, Albay. In 1963, Pobre
began developing the Property as a resort-subdivision. On 18 February 1972 when Pobre leased to NPC
for one year eleven lots from the approved subdivision plan. Subsequently, NPC filed two expropriation
case over a portion of the said property, i.e., first for the 8,311.60 square-meter portion of the Property
and second for the 5,554 square-meter lot (1 September 1979).

On 2 January 1985, NPC filed a motion to dismiss the second expropriation case on the ground that NPC
had found an alternative site. The trial court granted NPC’s motion to dismiss but the trial court allowed
Pobre to adduce evidence on his claim for damages. Thereafter, trial court issued a judgment ordering the
plaintiff to pay the defendant P3,448,450.00 PESOS which is the fair market value of the subdivision of
defendant with an area of sixty eight thousand nine hundred sixty nine (68,969) square meters, plus legal
rate of interest per annum from September 6, 1979 and damages.

The Court of Appeals affirmed the decision of the trial court. NPC insists that at the time that it moved for
the dismissal of its complaint, Pobre had yet to serve an answer or a motion for summary judgment on
NPC. Thus, NPC as plaintiff had the right to move for the automatic dismissal of its complaint. NPC
relies on Section 1, Rule 17 of the 1964 Rules of Court, the Rules then in effect. NPC argues that the
dismissal of the complaint should have carried with it the dismissal of the entire case including Pobre’s
counterclaim.

ISSUE: Whether or not the rule on dismissal of action as a matter of right is applicable to a special civil
action of expropriation.

RULING: No. Section 1, Rule 17 of the 1964 Rules of Court provided the exception to the general rule
that the dismissal of the complaint is addressed to the sound discretion of the court. For as long as all of
the elements of Section 1, Rule 17 were present the dismissal of the complaint rested exclusively on the
plaintiff’s will. The defending party and even the courts were powerless to prevent the dismissal. The
courts could only accept and record the dismissal. A plain reading of Section 1, Rule 17 of the 1964 Rules
of Court makes it obvious that this rule was not intended to supplement Rule 67 of the same Rules.
Section 1, Rule 17 of the 1964 Rules of Court, provided that:

SECTION 1.Dismissal by the plaintiff. — An action may be dismissed by the plaintiff without order of
court by filing a notice of dismissal at any time before service of the answer or of a motion for summary
judgment. Unless otherwise stated in the notice, the dismissal is without prejudice, exceptthat a notice
operates as an adjudication upon the merits when filed by a plaintiff who has once dismissed in a
competent court an action based on or including the same claim. A class suit shall not be dismissed or
compromised without approval of the court

While Section 1, Rule 17 spoke of the “service of answer or summary judgment,” the Rules then did not
require the filing of an answer or summary judgment in eminent domain cases. In lieu of an answer,
Section 3 of Rule 67 required the defendant to file a single motion to dismiss where he should present all
of his objections and defenses to the taking of his property for the purpose specified in the complaint. In
short, in expropriation cases under Section 3 of Rule 67, the motion to dismiss took the place of the
answer. The records show that Pobre had already filed and served on NPC his “motion to dismiss/answer”
even before NPC filed its own motion to dismiss. NPC filed its notice of dismissal of the complaint on 2
January 1985.

In expropriation cases, there is no such thing as the plaintiff’s matter of right to dismiss the complaint
precisely because the landowner may have already suffered damages at the start of the taking. The
plaintiff’s right in expropriation cases to dismiss the complaint has always been subject to court approval
and to certain conditions. The exceptional right that Section 1, Rule 17 of the 1964 Rules of Court
conferred on the plaintiff must be understood to have applied only to other civil actions. The 1997 Rules
of Civil Procedure abrogated this exceptional right.

Thus, NPC’s theory that the dismissal of its complaint carried with it the dismissal of Pobre’s claim for
damages is baseless. There is nothing in Rule 67 of the 1964 Rules of Court that provided for the
dismissal of the defendant’s claim for damages, upon the dismissal of the expropriation case. Case law

Remedial Law Review II (CY 2018-2019) – Saturday 1:00pm – 5:00pm: Case Digest Compilation [Atty. Brondial] Page 128
holds that in the event of dismissal of the expropriation case, the claim for damages may be made either
in a separate or in the same action, for all damages occasioned by the institution of the expropriation case.
The dismissal of the complaint can be made under certain conditions, such as the reservation of the
defendant’s right to recover damages either in the same or in another action. The trial court in this case
reserved Pobre’s right to prove his claim in the same case, a reservation that has become final due to
NPC’s own fault

Remedial Law Review II (CY 2018-2019) – Saturday 1:00pm – 5:00pm: Case Digest Compilation [Atty. Brondial] Page 129
F.7. Limkaichong vs. LBP

Remedial Law Review II (CY 2018-2019) – Saturday 1:00pm – 5:00pm: Case Digest Compilation [Atty. Brondial] Page 130
Remedial Law Review II (CY 2018-2019) – Saturday 1:00pm – 5:00pm: Case Digest Compilation [Atty. Brondial] Page 131
G. Foreclosure of Real Estate Mortgage (Rule 68)

G.1. JOSE T. RAMIREZ vs.THE MANILA BANKING CORPORATION

G.R. No. 198800; December 11, 2013

FACTS: Jose Ramirez mortgaged two parcels of land located atBayanbayan, Marikina Cityin favor of
respondent The Manila Banking Corporation to secure his ₱265,000 loan.The real estate mortgage
provides:

N) All correspondence relative to this MORTGAGE, including demand letters, summons, subpoenas or
notifications of any judicial or extrajudicial actions shall be sent to the MORTGAGOR at the
address given above or at the address that may hereafter be given in writing by the MORTGAGOR
to the MORTGAGEE xxx

Manila Banking filed a request for extrajudicial foreclosure of real estate mortgage before Atty. Sañez on
the ground that Ramirez failed to pay his loan despite demands. During the auction sale Manila Banking
was the only bidder for the mortgaged properties. Thereafter, a certificate of sale was issued in its favor as
the highest bidder.

Ramirez sued respondent for annulment of sale and prayed that the certificate of sale be annulled on the
ground, among others, that paragraph N of the real estate mortgage was violated for he was not notified of
the foreclosure and auction sale.

RTC – ruled that the foreclosure proceedings were null and void and the certificate of sale is invalid.

CA – revered ruling of the RTC and ruled that absence of personal notice of foreclosure to Ramirez as
required by paragraph N of the real estate mortgage is not a ground to set aside the foreclosure sale.
MR was denied.

ISSUE: WON the foreclosure proceeding is null and void on the ground that no personal notice of
foreclosure was provided to Ramirez.

RULING: Yes. In Carlos Lim, et al. v. Development Bank of the Philippines, unless the parties
stipulate, personal notice to the mortgagor in extrajudicial foreclosure proceedings is not necessary
because Section 3 of Act No. 3135 only requires the posting of the notice of sale in three public
places and the publication of that notice in a newspaper of general circulation. In this case, the
parties stipulated in paragraph N of the real estate mortgage that all correspondence relative to the
mortgage including notifications of extrajudicial actions shall be sent to mortgagor Ramirez at his given
address. Respondent had no choice but to comply with this contractual provision it has entered into with
Ramirez. The contract is the law between them.

Section 3, Act No. 3135 reads:


"Sec. 3. Notice shall be given by posting notices of the sale for not less than twenty days in at least three
public places of the municipality or city where the property is situated, and if such property is worth more
than four hundred pesos, such notice shall also be published once a week for at least three consecutive
weeks in a newspaper of general circulation in the municipality and city."
The Act only requires (1) the posting of notices of sale in three public places, and (2) the publication of
the same in a newspaper of general circulation. Personal notice to the mortgagor is not
necessary. Nevertheless, the parties to the mortgage contract are not precluded from exacting additional
requirements. In this case, petitioner and respondent in entering into a contract of real estate mortgage,
agreed inter alia:

"all correspondence relative to this mortgage, including demand letters, summonses, subpoenas, or
notifications of any judicial or extra-judicial action shall be sent to the MORTGAGOR…."
Precisely, the purpose of the foregoing stipulation is to apprise respondent of any action which petitioner
might take on the subject property, thus according him the opportunity to safeguard his rights. When
petitioner failed to send the notice of foreclosure sale to respondent, he committed a contractual breach
sufficient to render the foreclosure sale on November 23,

Remedial Law Review II (CY 2018-2019) – Saturday 1:00pm – 5:00pm: Case Digest Compilation [Atty. Brondial] Page 132
G.2. Spouses Nicasio C. Marquez and Anita J. Marquez vs. Spouses CarlitoAlindog and Carmen
Alindog

G.R. No.184045; January 22, 2014; Perlas-Bernabe, J.:

FACTS: Petitioner Anit) extended a loan to Gutierrez. As security therefor, Gutierrez executed a Deed of
Real Estate Mortgage over a parcel of land covered by TCT (subject property).

Since Gutierrez defaulted in the payment of his loan obligation, Anita sought the extra-judicial
foreclosure of the subject property. At the public auction sale, Anita emerged as the highest bidder. Upon
Gutierrez’s failure to redeem the same property within the prescribed period therefor, title was
consolidated under a new TCTwhich, however, bore an annotation of adverse claim in the names of
respondents-spouses Sps. Alindog. Said annotation was copied from an earlier annotation in theold TCT
made only after the subject property’s mortgage to Sps. Marquez.

Subsequently, Sps. Alindog filed a civil case for annulment of real estate mortgage and certificate of sale
with prayer for damages against Sps. Marquez and a certain Agripina Gonzales before the RTC.In an
Orderhe RTC granted Anita’s ex-parte petition and thereby directed the issuance of a writ of possession
in her favor.

Claiming that they would suffer irreparable injury if the implementation of the writ of possession in favor
of Sps. Marquez would be left unrestrained, Sps. Alindog sought the issuance of a temporary restraining
order (TRO) and/or writ of preliminary injunction with prayer for damages
After further proceedings on the injunction case, the RTC, through an Order, issued a writ of preliminary
injunction enjoining Sps. Marquez from taking possession of the subject property until after the
controversy has been fully resolved on the merits.

Aggrieved, Sps. Marquez moved for reconsideration. In an Order, the RTC denied the motion of Sps.
Marquez, while granted that of Sps. Alindog. Unperturbed, Sps. Marquez elevated the case to the CA on
certiorari.

In a Decision, the CA denied Sps. Marquez’s petition as it found no grave abuse of discretion on the
RTC’s part when it issued the injunctive writ that enjoined Sps. Marquez from taking possession of the
subject property.Dissatisfied, Sps. Marquez moved for reconsideration which was, however, denied in a
Resolution, hence, this petition.

ISSUE: Whether thepurchaser at the public auction sale of an extra-judicially foreclosed real property is
entitled to the possession of said property.

RULING: The petition is meritorious.

WHEREFORE, the petition is GRANTED. The Decision dated February 29, 2008 and Resolution dated
August 6, 2008 of the Court of Appeals in CA-G.R. SP No. 97744, as well as the Orders dated November
14, 2005 and January 17, 2007 of the Regional Trial Court of Tagaytay City, Branch 18 in SCA No. TG-
05-2521 are hereby REVERSED and SET ASIDE. Accordingly, the writ of preliminary injunction in
SCA No. TG-05-2521 is NULLIFIED, while the Writ of Possession in LRC Case No. TG-05-1068 is
REINSTATED.

DOCTRINE: It is an established rule that the purchaser in an extra-judicial foreclosure sale is entitled
to the possession of the property and can demand that he be placed in possession of the same either
during (with bond) or after the expiration (without bond) of the redemption period therefor. To this end,
the Court, in China Banking Corp. v.Sps. Lozada (China Banking Corp.), citing several cases on the
matter, explained that a writ of possession duly applied for by said purchaser should issue as a matter of
course, and thus, merely constitutes a ministerial duty on the part of the court.

In IFC Service Leasing and Acceptance Corporation v. Nera, the Court reasoned that if under Section 7
of Act No. 3135, as amended, the RTC has the power during the period of redemption to issue a writ of
possession on the ex parte application of the purchaser, there is no reason why it should not also have the
same power after the expiration of the redemption period, especially where a new title has already been
issued in the name of the purchaser. Hence, the procedure under Section 7 of Act No. 3135, as amended,
may be availed of by a purchaser seeking possession of the foreclosed property he bought at the public
auction sale after the redemption period has expired without redemption having been made.

Remedial Law Review II (CY 2018-2019) – Saturday 1:00pm – 5:00pm: Case Digest Compilation [Atty. Brondial] Page 133
It is thus settled that the buyer in a foreclosure sale becomes the absolute owner of the property
purchased if it is not redeemed during the period of one year after the registration of the sale. As such, he
is entitled to the possession of the said property and can demand it at any time following the
consolidation of ownership in his name and the issuance to him of a new transfer certificate of title. The
buyer can in fact demand possession of the land even during the redemption period except that he has to
post a bond in accordance with Section 7 of Act No. 3135, as amended. No such bond is required after
the redemption period if the property is not redeemed. Possession of the land then becomes an absolute
right of the purchaser as confirmed owner. Upon proper application and proof of title, the issuance of the
writ of possession becomes a ministerial duty of the court.

Remedial Law Review II (CY 2018-2019) – Saturday 1:00pm – 5:00pm: Case Digest Compilation [Atty. Brondial] Page 134
G.3. LZK Holdings vs. Planters Dev. Bank

G.R. No. 187973; January 20, 2014 ; Reyes, J.

FACTS: LZK Holdings obtained a ₱40,000,000.00 loan from Planters Bank on December 16, 1996 and
secured the same with a Real Estate Mortgage over its lot located in La Union. On September 21, 1998,
the lot was sold at a public auction after Planters Bank extra-judicially foreclosed the real estate mortgage
thereon due to LZK Holdings' failure to pay its loan. Planters Bank emerged as the highest bidder during
the auction sale and its certificate of sale was registered on March 16, 1999.

On April 5, 1999, LZK Holdings filed before the RTC of Makati City, Branch 150, a complaint for
annulment of extra judicial foreclosure, mortgage contract, promissory note and damages. LZK Holdings
also prayed for the issuance of a temporary restraining order (TRO) or a writ of preliminary injunction to
enjoin the consolidation of title over the lot by Planters Bank. On December 27, 1999, Planters Bank filed
an ex-parte motion for the issuance of a writ of possession with the RTC-San Fernando. On March 13,
2000 or three (3) days before the expiration of LZK Holdings' redemption period, the RTC-Makati issued
a TRO effective for 20 days enjoining Planters Bank from consolidating its title over the property. On
April 3, 2000, the RTC-Makati ordered the issuance of a writ of preliminary injunction for the same
purpose but the writ was issued only on June 20, 2000 upon LZK Holdings' posting of a ₱40,000.00 bond.

In the meantime, Planters Bank succeeded in consolidating its ownership over the property on April 24,
2000. However, the proceedings for its ex-parte motion for the issuance of a writ of possession was
suspended by the RTC-San Fernando in an Order dated May 11, 2000 in view of the TRO and writ of
preliminary injunction issued by the RTC-Makati. Meanwhile, upon motion of LZK Holdings, the RTC-
Makati declared as null and void the consolidated title of Planters Bank in an Order dated June 2, 2000.
Such ruling was affirmed by the CA. Planters Bank also appealed the May 11, 2000 Order of the RTC-
San Fernando which held in abeyance the resolution of its ex parte motion for the issuance of a writ of
possession. This time, Planters Bank was victorious. The CA granted the appeal and annulled the assailed
order of the RTC-San Fernando.

Aggrieved, LZK Holdings sought recourse with the Court in a petition for review. In Our Decision dated
April 27, 2007, we affirmed the CA's ruling and decreed that Planters Bank may apply for and is entitled
to a writ of possession as the purchaser of the property in the foreclosure sale. Armed with the above
ruling, Planters Bank filed before the RTC-San Fernando a motion to set ex-parte hearing for the issuance
of a writ of possession. The RTC-San Fernando issued an Order declaring the scheduled hearing moot
and academic and granting Planter Bank's ex-parte motion for the issuance of a writ of possession which
was filed as early as December 27, 1999. The CA affirmed the foregoing ruling and dismissed LZK
Holdings' petition for certiorari.

LZK Holdings claims that the writ of possession issued to Planters Bank should be annulled for the
following reasons, to wit: (a) with the cancellation of Planters Bank's consolidated title, LZK Holdings
remain to be the registered owner of the property and as such, the former had no right to apply for a writ
of possession pursuant to PNB v. Sanao Marketing Corporation, which held that right of possession is
based on the ownership of the subject property by the applicant; (b) LZK Holdings was deprived of due
process because the RTC did not conduct a hearing on Planter Bank's motion for the issuance of a writ of
possession; (c) the P.2,000,000.00 bond posted by LZK Holdings does not conform with Section 7 of Act
No. 3135 which mandates that the bond amount shall be equivalent to "twelve (12) months use of the
subject property" which in this case amounted to P.7,801,4 72.28 at the time the writ was issued.

ISSUE: Whether or not the writ of possession issued to Planters Bank should be annulled.

RULING: No, the writ cannot be annulled.

Under the principle of conclusiveness of judgment, the right of Planter's Bank to a writ of possession as
adjudged in G.R. No. 167998 is binding and conclusive on the parties. The doctrine of res judicata by
conclusiveness of judgment postulates that "when a right or fact has been judicially tried and determined
by a court of competent jurisdiction, or when an opportunity for such trial has been given, the judgment
of the court, as long as it remains unreversed, should be conclusive upon the parties and those in privity
with them." All the elements of the doctrine are present in this case.

The final judgment in G.R. No. 167998 was rendered by the Court pursuant to its jurisdiction over the
review of decisions and rulings of the CA. It was a judgment on the merits of Planters Banks's right to

Remedial Law Review II (CY 2018-2019) – Saturday 1:00pm – 5:00pm: Case Digest Compilation [Atty. Brondial] Page 135
apply for and be issued a writ of possession. Lastly, the parties in G.R. No. 167998 are the same parties
involved in the present case. Hence, LZK Holdings can no longer question Planter Bank's right to a writ
of possession over the subject property because the doctrine of conclusiveness of judgment bars the
relitigation of such particular issue.

Moreover, the authority relied upon by LZK Holdings defeats rather than support its position. The ruling
in PNB echoes the very same rationale of the judgment in G.R. No. 167998 that is - the purchaser in
foreclosure sale may take possession of the property even before the expiration of the redemption period
by filing an ex parte motion for such purpose and upon posting of the necessary bond.

The pronouncement in PNB that right of possession is based on the ownership of the subject property by
the applicant pertains to applications for writ of possession after the expiration of the redemption period, a
situation not contemplated within the facts of the present case.

We cannot also uphold the contentions of LZK Holdings that the RTC, in issuing the writ of possession,
transgressed Act No. 3135. No hearing is required prior to the issuance of a writ of possession. This is
clear from the following disquisitions in Espinoza v. United Overseas Bank Phils. which reiterates the
settled rules on writs of possession, to wit:

The proceeding in a petition for a writ of possession is ex parte and summary in nature. It is a judicial
proceeding brought for the benefit of one party only and without notice by the court to any person adverse
of interest. It is a proceeding wherein relief is granted without giving the person against whom the relief is
sought an opportunity to be heard. By its very nature, an ex parte petition for issuance of a writ of
possession is a non-litigious proceeding. It is a judicial proceeding for the enforcement of one's right of
possession as purchaser in a foreclosure sale. It is not an ordinary suit filed in court, by which one party
sues another for the enforcement of a wrong or protection of a right, or the prevention or redress of a
wrong.

Anent the correct amount of surety bond, it is well to emphasize that our task in an appeal by petition for
review on certiorari is limited, as a jurisdictional matter, to reviewing errors of law that might have been
committed by the CA The allegations of incorrect computation of the surety bond involve factual matters
within the competence of the trial court to address as this Court is not a trier of facts.

DOCTRINE: The purchaser in foreclosure sale may take possession of the property even before the
expiration of the redemption period by filing an ex parte motion for such purpose and upon posting of the
necessary bond.

Remedial Law Review II (CY 2018-2019) – Saturday 1:00pm – 5:00pm: Case Digest Compilation [Atty. Brondial] Page 136
G.4. GOLDENWAY MERCHANDISING CORPORATION, Petitioner, vs. EQUITABLE PCI
BANK, Respondent.

G.R. No. 195540; March 13, 2013; VILLARAMA, JR., J.:

FACTS: Goldenway Merchandising Corporation (petitioner) executed a Real Estate Mortgage in favor of
Equitable PCI Bank (respondent) over its real properties situated in Valenzuela, Bulacan. The mortgage
secured the Two Million Pesos (₱2,000,000.00) loan granted by respondent to petitioner and was duly
registered.As petitioner failed to settle its loan obligation, respondent extrajudicially foreclosed the
mortgage. During the public auction, the mortgaged properties were sold for ₱3,500,000.00 to respondent.
Petitioner’s counsel offered to redeem the foreclosed properties by tendering a check in the amount of
₱3,500,000.00 exercising the right of redemption. Petitioner was told that such redemption is no longer
possible because the certificate of sale had already been registered.

Petitioner filed a complaint for specific performance and damages against the respondent with the
Regional Trial Court of Valenzuela City (RTC), asserting that it is the one-year period of redemption
under Act No. 3135 which should apply and not the shorter redemption period provided in Republic Act
(R.A.) No. 8791. In its Answer with Counterclaim, respondent pointed out that petitioner cannot claim
that it was unaware of the redemption price which is clearly provided in Section 47 of R.A. No. 8791, and
that petitioner had all the opportune time to redeem the foreclosed properties from the time it received the
letter of demand and the notice of sale before the registration of the certificate of sale.The trial court
rendered its decision dismissing the complaint as well as the counterclaim. It noted that the issue of
constitutionality of Sec. 47 of R.A. No. 8791 was never raised by the petitioner during the pre-trial and
the trial. Aside from the fact that petitioner’s attempt to redeem was already late, there was no valid
redemption made because petitioner’s council was not properly authorized by petitioner’s Board of
Directors to transact for and in its behalf.

Aggrieved, petitioner appealed to the CA which affirmed the trial court’s decision. According to the
CA, petitioner failed to justify why Section 47 of R.A. No. 8791 should be declared unconstitutional.
Furthermore, the appellate court concluded that a reading of Section 47 plainly reveals the intention to
shorten the period of redemption for juridical persons and that the foreclosure of the mortgaged properties
in this case when R.A. No. 8791 was already in effect clearly falls within the purview of the said
provision.Hence, the petitioner filed a petition for review on certiorari with the Supreme Court.
Petitioner contended that the right of redemption is part and parcel of the Deed of Real Estate
Mortgage itself and attaches thereto upon its execution, a vested right flowing out of and made
dependent upon the law governing the contract of mortgage and not on the mortgagee’s act of
extrajudicially foreclosing the mortgaged properties.

ISSUE: Whether or not the foregoing amendment be validly applied in this case when the real estate
mortgage contract was executed in 1985 and the mortgage foreclosed when R.A. No. 8791 was already in
effect?

RULING: Yes, we answer in the affirmative. We agree with the CA that the legislature clearly intended
to shorten the period of redemption for juridical persons whose properties were foreclosed and sold in
accordance with the provisions of Act No. 3135.

The difference in the treatment of juridical persons and natural persons was based on the nature of the
properties foreclosed – whether these are used as residence, for which the more liberal one-year
redemption period is retained, or used for industrial or commercial purposes, in which case a shorter term
is deemed necessary to reduce the period of uncertainty in the ownership of property and enable
mortgagee-banks to dispose sooner of these acquired assets. It must be underscored that the General
Banking Law of 2000, crafted in the aftermath of the 1997 Southeast Asian financial crisis, sought to
reform the General Banking Act of 1949 by fashioning a legal framework for maintaining a safe and
sound banking system.In this context, the amendment introduced by Section 47 embodied one of such
safe and sound practices aimed at ensuring the solvency and liquidity of our banks.1âwphi1 It cannot
therefore be disputed that the said provision amending the redemption period in Act 3135 was based on a
reasonable classification and germane to the purpose of the law.

DOCTRINE: The right of redemption being statutory, it must be exercised in the manner prescribed by
the statute,and within the prescribed time limit, to make it effective.

Remedial Law Review II (CY 2018-2019) – Saturday 1:00pm – 5:00pm: Case Digest Compilation [Atty. Brondial] Page 137
Juridical persons whose property is being sold pursuant to an extrajudicial foreclosure, shall have the
right to redeem the property within one year after the sale of the real estate, but not after, the registration
of the certificate of foreclosure sale with the applicable Register of Deeds which in no case shall be more
than three (3) months after foreclosure, whichever is earlier.

Remedial Law Review II (CY 2018-2019) – Saturday 1:00pm – 5:00pm: Case Digest Compilation [Atty. Brondial] Page 138
G.5. ALLIED BANKING CORPORATION, Petitioner, vs. RUPERTO JOSE H. MATEO,
represented by WARLITA MATEO, as Attorney-in-Fact, Respondent.

G.R. No. 167420; June 5, 2009

Facts: Ruperto Jose Mateo (respondent) obtained a loan from Allied Banking in the amount of
₱950,000.00. To secure the payment of the loan, respondent executed in favor of petitioner a deed of real
estate mortgage over a parcel of land registered in Mateo’s name. Mateo likewise executed a promissory
note in the amount of ₱950,000.00. Subsequently, Mateo incurred default in the payment of his loan
prompting Allied Banking to cause the extrajudicial foreclosure of the mortgage constituted on the
subject property. The property was then sold at public auction for ₱1,531,474.53 with Allied Banking as
the sole and highest bidder. The Certificate of Sale was issued to Allied Banking, and was registered with
the Register of Deeds on July 21, 1999.

Mateo, through her attorney-in-fact, Warlita N. Mateo (Warlita), sent, on several dates, faxed letters to
petitioner signifying his desire to redeem the foreclosed property for ₱1.1 million pesos.
On July 21, 2000, or on the last day of the period for redemption, Mateo, represented by Warlita, filed a
case for legal redemption with prayer for temporary restraining order and preliminary injunction with the
RTC of Isabela.

RTC: On October 21, 2004, the RTC rendered its Decision, in favor of Mateo and against the Allied
Banking, ALLOWING the Mateo to redeem from Allied Banking the property now covered by TCT No.
T-311043, upon payment of the amount of ₱1,531,474.53, plus one (1) percent as interest for one (1)
month only, and ORDERING Allied Banking to accept the tender of redemption of the plaintiff and to
deliver the proper certificate of redemption to the latter and finally, ordering Allied Banking to indemnify
the plaintiff ₱30,000.00 as attorney’s fees and cost of the suit.

Allied Banking then filed an MR against the decision of the RTC but was later on denied.
Dissatisfied, Allied Banking filed a petition for review on certiorari with the Court alleging that:
(1) the RTC erred in considering the various offers made by respondent to redeem the subject property for
the amount of ₱1.1 million as sufficient tender of payment for purposes of redemption;
(2) the tender to be legally sufficient must be for the amount of the purchase price, plus the agreed interest
rate on the principal obligation;
(3) the RTC erred in considering the deposit of ₱1.1 million with Land Bank as sufficient consignation,
since the amount should have been deposited in court and not anywhere else;
(4) the offer to redeem in the amount of ₱1,531,474.53 was made only during the pre-trial conference,
which was already way past the redemption period; and
(5) the redemption price should be based on Section 47 of the General Banking Act.

ISSUE: Was there valid redemption?

RULING: No, it was already stipulated upon by the parties that respondent offered ₱1.1 million as
redemption price before the filing of this action; thus, the issue is not the amount of redemption price, but
the sufficiency of the amount offered by respondent that would warrant the redemption of the foreclosed
property.

The general rule in redemption is that it is not sufficient that a person offering to redeem manifests his
desire to do so. The statement of intention must be accompanied by an actual and simultaneous tender of
payment. This constitutes the exercise of the right to repurchase.
In several cases decided by the Court where the right to repurchase was held to have been properly
exercised, there was an unequivocal tender of payment for the full amount of the repurchase price.
Otherwise, the offer to redeem is ineffectual. Bona fide redemption necessarily implies a reasonable and
valid tender of the entire repurchase price, otherwise the rule on the redemption period fixed by law can
easily be circumvented.

In this case, it was stipulated upon by the parties that the real estate mortgage over respondent’s property
was foreclosed in the amount of ₱1,531,474.53, and that respondent offered the amount of ₱1.1 million as
redemption price before the filing of the complaint. It has been held that the tender of payment must be
for the full amount of the purchase price, i.e., the amount fixed by the court in the order of execution or
the amount due under the mortgage deed, as the case may be, with interest thereon at the rate specified in
the mortgage; and all the costs, and judicial and other expenses incurred by the bank or institution
concerned by reason of the execution and sale and as a result of the custody of said property less the

Remedial Law Review II (CY 2018-2019) – Saturday 1:00pm – 5:00pm: Case Digest Compilation [Atty. Brondial] Page 139
income received from the property. Thus, the amount of ₱1.1 million offered by respondent was
ineffective, since not only did the amount not include the interest but it was even below the purchase
price. Such offer did not effect a valid redemption, and petitioner was justified in refusing to accept such
offer.

Doctrine:It is not sufficient that a person offering to redeem manifests his desire to do so. The statement
of intention must be accompanied by an actual and simultaneous tender of payment. This constitutes the
exercise of the right to repurchase, otherwise, the offer to redeem is ineffectual.

Remedial Law Review II (CY 2018-2019) – Saturday 1:00pm – 5:00pm: Case Digest Compilation [Atty. Brondial] Page 140
G.6. ROLANDO ROBLES, REPRESENTED BY ATTY. CLARA C.
ESPIRITU, Petitioner, v. FERNANDO FIDEL YAPCINCO, PATROCINIO B. YAPCINCO,
MARIA CORAZON B. YAPCINCO, AND MARIA ASUNCION B. YAPCINCO-
FRONDA, Respondents.

G.R. No. 169568; October 22, 2014; BERSAMIN J;

FACTS: Yapcinco constituted a mortgage on the property in favor of Marcelo to secure the performance
of his obligation. Marcelo transferred his rights to Apolinario Cruz. When Yapcinco did not pay the
obligation, Apolinario Cruz brought an action for judicial foreclosure of the mortgage in the CFI w/c
ordered Kelly (the administratrix of the estate of Yapcinco who died during the pendency of the action) to
pay Apolinario Cruz the indebtedness secured by the mortgage plus interest and in case of the failure to
pay after 90 days from the date of the decision, the property would be sold at a public auction. Apolinario
Cruz was adjudged the highest bidder in the public auction. In his favor was then issued the certificate of
absolute sale, and he took possession of the property in due course. However, he did not register the
certificate of sale; nor was a judicial confirmation of sale issued. Apolinario Cruz donated the property to
his grandchildren.

Yapcinco filed an action in the RTC for the annulment of TCT. RTC ruled in favor of Yapcinco/later on
(a case was again filed and) RTC ruled in favor of Apolinario Cruz. On Appeal, the CA ruled in favor of
Yapcinco hence this instante petition.

ISSUE:  Is non-registration of property after judicially foreclosure and sale had the effect of invalidating
the foreclosure proceedings, such that ownership reverts to the original owner?

RULING: NO. The effect of the failure of Apolinario Cruz to obtain the judicial confirmation was only
to prevent the title to the property from being transferred to him. For sure, such failure did not give rise to
any right in favor of the mortgagor or the respondents as his successors-in-interest to take back the
property already validly sold through public auction. Nor did such failure invalidate the foreclosure
proceedings. To maintain otherwise would render nugatory the judicial foreclosure and foreclosure sale,
thus unduly disturbing judicial stability. The non-transfer of the title notwithstanding, Apolinario Cruz as
the purchaser should not be deprived of the property purchased at the foreclosure sale. With the
respondents having been fully aware of the mortgage, and being legally bound by the judicial foreclosure
and consequent public sale, and in view of the unquestioned possession by Apolinario Cruz and his
successors-in-interest (including the petitioner) from the time of the foreclosure sale until the present, the
respondents could not assert any better right to the property. It would be the height of inequity to still
permit them to regain the property on the basis alone of the lack of judicial confirmation of the sale. After
all, under the applicable rule earlier cited, the judicial confirmation operated only “to divest the rights of
all the parties to the action and to vest their rights in the purchaser, subject to such rights of redemption as
may be allowed by law.”

Consequently, the late Fernando F. Yapcinco and the respondents as his successors-in-interest were
divested of their right in the property, for they did not duly exercise the equity of redemption decreed in
the decision of the trial court. With Yapcinco having thereby effectively ceased to be the owner of the
property sold, the property was taken out of the mass of the assets of Yapcinco upon the expiration of the
equity of redemption.

DOCTRINE:The registration of the sale is required only in extra-judicial foreclosure sale because the
date of the registration is the reckoning point for the exercise of the right of redemption. In contrast, the
registration of the sale is superfluous in judicial foreclosure because only the equity of redemption is
granted to the mortgagor, except in mortgages with banking institutions. The equity of redemption is the
right of the defendant mortgagor to extinguish the mortgage and retain ownership of the property by
paying the secured debt within the 90-day period after the judgment becomes final, or even after the
foreclosure sale but prior to the confirmation of the sale.

Remedial Law Review II (CY 2018-2019) – Saturday 1:00pm – 5:00pm: Case Digest Compilation [Atty. Brondial] Page 141
G.7. METROPOLITAN BANK AND TRUST COMPANY, Petitioner, v. CPR PROMOTIONS
AND MARKETING, INC. AND SPOUSES CORNELIO P. REYNOSO, JR. AND LEONIZA* F.
REYNOSO, Respondents.

G.R. No. 200567; June 22, 2015; J. Velasco

FACTS: CPR Promotions and Marketing, Inc. (CPR Promotions) obtained loans from petitioner MBTC.
These loans were covered by fifteen (15) promissory notes (PNs) all duly signed by the spouses Reynoso,
as Treasurer and President of the CPR Promotions. The Spouses Reynoso executed two deeds of real
estate mortgage to secure the loan.
To wit:
Amount (PhP) TCT No. Registered in the name of

1st Mortgage 6,500,000 (TCT) No. 624835 Spouses Reynoso

2nd Mortgage 2,500,000 (TCT) No. 565381 CPR Promotions


(TCT) No. 263421 Spouses Reynoso
(TCT) No. 274682 Spouses Reynoso

Thereafter, spouses Reynoso executed a continuing surety agreement binding themselves solidarity with
CPR Promotions to pay any and all loans CPR Promotions may have obtained from petitioner MBTC,
including those covered by the said PNs, but not to exceed PhP 13,000,000.
Upon maturity of the loans, respondents defaulted, prompting MBTC to file a petition for extra-judicial
foreclosure of the real estate mortgages.
The mortgaged properties covered by TCT Nos. 624835 and 565381 were sold at a public auction sale.
MBTC participated therein and submitted the highest bid in the amount of PhP 10,374,000. Petitioner
again participated and won in the public auction sale of the remaining mortgaged properties, having
submitted the highest bid amounting to PhP 3,240,000. As a result, petitioner was issued the
corresponding Certificates of Sale on July 15 and 16, 1998, covering the properties subjected to the first
and second public auctions, respectively.
Notwithstanding the foreclosure of the mortgaged properties for the total amount of PhP 13,614,000,
petitioner MBTC alleged that there remained a deficiency balance of PhP 2,628,520.73, plus interest and
charges as stipulated and agreed upon in the PNs and deeds of real estate mortgages. Despite petitioner's
repeated demands, however, respondents failed to settle the alleged deficiency. Thus, petitioner filed an
action for collection of sum of money against respondents, docketed as Civil Case No. 99-230,
entitled Metropolitan Bank and Trust Company v. CPR Promotions and Marketing, Inc. and Spouses
Cornelio Reynoso, Jr. and Leoniza F. Reynoso.

The RTC ruled in favor of MBTC and held that the respondents are liable for the balance of PhP
2,628,520.73, plus interest and charges as part of their contractual obligation.
The decision was reversed by the CA and instead ordered MBTC   to refund or return to the Spouses
Reynoso the amount of PhP 722,602.22 representing the remainder of the proceeds of the foreclosure
sale, with legal interest of six percent per annum from the date of filing of the answer with counterclaim
on March 26, 1999, until paid.

ISSUE: Whether or not Metropolitan Banking and Trust Company's (MBTC's) can claim for deficiency
payment upon foreclosing respondents' mortgaged properties

RULING: We partially grant the petition. While we fully agree with the CA that MBTC was not able to
prove the amount claimed, we however, find that neither were respondents able to timely setup their
claim for refund.

MBTC failed to prove that there is a deficiency balance of PhP 2,628,520.73.We have already ruled in
several cases that in extrajudicial foreclosure of mortgage, where the proceeds of the sale are insufficient
to pay the debt, the mortgagee has the right to recover the deficiency from the debtor. In ascertaining the
deficit amount, Sec. 4, Rule 68 of the Rules of Court is elucidating, to wit:
Section 4. Disposition of proceeds of sale. The amount realized from the foreclosure sale of the
mortgaged property shall, after deducting the costs of the sale, be paid to the person foreclosing
the mortgage, and when there shall be any balance or residue, after paying off the mortgage debt
due, the same shall be paid to junior encumbrances in the order of their priority, to be
ascertained by the court, or if there be no such encumbrances or there be a balance or residue

Remedial Law Review II (CY 2018-2019) – Saturday 1:00pm – 5:00pm: Case Digest Compilation [Atty. Brondial] Page 142
after payment to them, then to the mortgagor or his duly authorized agent, or to the person
entitled to it. (emphasis added)
Verily, there can only be a deficit when the proceeds of the sale is not sufficient to cover (1) the costs of
foreclosure proceedings; and (2) the amount due to the creditor, inclusive of interests and penalties, if
any, at the time of foreclosure.

Having alleged the existence of a deficiency balance, it behooved petitioner to prove, at the very least, the
amount due at the date of foreclosure against which the proceeds from the auction sale would be applied.
Otherwise, there can be no basis for awarding the claimed deficiency balance. Unfortunately for
petitioner, it failed to substantiate the amount due as of May 5, 1998 as appearing in its Statement of
Account. Also, Petitioner failed to prove the amount of expenses incurred in foreclosing the mortgaged
properties.

In sum, given petitioner's failure to establish the sum due at the time the mortgaged properties were
foreclosed and sold via public auction, as well as the expenses incurred in those foreclosure proceedings,
it would be impossible for the Court to determine whether or not there is, indeed, a deficiency balance
petitioner would have been entitled to.

In demanding payment of a deficiency in an extrajudicial foreclosure of mortgage, proving that there is


indeed one and what its exact amount is, is naturally a precondition thereto. The same goes with a claim
for reimbursement of foreclosure expenses, as here. In this regard, it is elementary that the burden to
prove a claim rests on the party asserting such. Eiincumbitprobatio qui dicit, non qui negat. He who
asserts, not he who denies, must prove.

(Issue on Counterclaim: Having determined that a claim for recovery of an excess in the bid price should
be set up in the action for payment of a deficiency as a compulsory counterclaim, We rule that
respondents failed to timely raise the same. It is elementary that a defending party's compulsory
counterclaim should be interposed at the time he files his Answer, and that failure to do so shall
effectively bar such claim.)

Remedial Law Review II (CY 2018-2019) – Saturday 1:00pm – 5:00pm: Case Digest Compilation [Atty. Brondial] Page 143
H. Partition (Rule 69)

H.1. VDA. DE FIGURACION VS vs.EMILIA FIGURACION-GERILLA

G.R. No. 151334; February 13, 2013; REYES, J.

FACTS: Agripina and Carolina are heirs of Eulalio. As such heirs, they became co-owners of Lot No.
707. Agripina executed a Deed of Quitclaim over the eastern half of Lot No. 707 in favor of her niece,
herein respondent Emilia.

Petitioner Carolina executed an Affidavit of Self-Adjudication, adjudicating unto herself the entire Lot No.
707 as the sole and exclusive heir of her deceased parents, Eulalio and Faustina. Carolina also executed
a Deed of Absolute Sale over Lot No. 707 in favor of petitioners Hilaria and Felipa, who in turn
immediately caused the cancellation of OCT No. 15867 and the issuance of TCT No. 42244 in their
names.

The legal debacle of the Figuracions started in 1994 when Hilaria and her agents threatened to demolish
the house of Emilia who, in retaliation, was prompted to seek the partition of Lot No. 707 as well as Lot
Nos. 2299 and 705.  In 1971, Emilia and her family went to the United States and returned to the
Philippines only in 1981. Upon her return and relying on the Deed of Quitclaim, she built a house on the
eastern half of Lot No. 707.

ISSUE: Whether or not the respondent can compel the partition of Lot No. 707?

RULING: The first stage in an action for partition is the settlement of the issue of ownership. Such an
action will not lie if the claimant has no rightful interest in the subject property. In fact, the parties filing
the action are required by the Rules of Court to set forth in their complaint the nature and the extent of
their title to the property. It would be premature to effect a partition until and unless the question of
ownership is first definitely resolved.

Here, the respondent traces her ownership over the eastern half of Lot No. 707 from the  Deed of
Quitclaim executed by Agripina, who in turn, was the co-owner thereof being one of the legitimate heirs
of Eulalio. It is well to recall that the petitioners failed to categorically dispute the existence of the  Deed
of Quitclaim. Instead, they averred that it has been rendered ineffective by TCT No. 42244 in the name of
Felipa and Hilaria―this contention is, of course, flawed.

Mere issuance of a certificate of title in the name of any person does not foreclose the possibility that the
real property may be under coownership with persons not named in the certificate.  The certificate cannot
always be considered as conclusive evidence of ownership. In this case, co-ownership of Lot No. 707 was
precisely what respondent Emilia was able to successfully establish, as correctly found by the RTC and
affirmed by the CA.

Furthermore, there is no acquisitive prescription. The act of Hilaria and Felipa in effecting the registration
of the entire Lot No. 707 in their names thru TCT No. 42244 did not serve to effectively repudiate the co-
ownership. The respondent built her house on the eastern portion of the lot in 1981 without any
opposition from the petitioners. 

DOCTRINE: The parties filing the action are required by the Rules of Court to set forth in their
complaint the nature and the extent of their title to the property. It would be premature to effect a
partition until and unless the question of ownership is first definitely resolved.

Remedial Law Review II (CY 2018-2019) – Saturday 1:00pm – 5:00pm: Case Digest Compilation [Atty. Brondial] Page 144
H.2. Balus vs. Balus

G.R. No. 168970; January 15, 2010; PERALTA, J.

FACTS: Petitioner Celestino and respondents Saturnino and Leonarda are the children of the spouses
Rufo and SebastianaBalus. In 1979, Rufo mortgaged a parcel of land as security for a loan obtained from
a bank. When Rufo failed to pay the loan, the property was foreclosed and was subsequently sold to the
Bank as the sole bidder at a public auction held for that purpose. The same was not redeemed within the
period allowed by law. Hence, a new title was issued in the name of the Bank. Rufo died on 6 July 1984.

On 10 October, 1989, petitioner and respondents executed an Extrajudicial Settlement of Estate


adjudicating to each of the a specific one-third portion of the subject property. Three years thereafter,
respondents bought the subject property from the Bank and a new title was issued in their name.
Meanwhile, petitioner continued possession of the subject lot. The respondents thus filed a complaint for
recovery of possession. However, petitioner alleged that respondents’ act of buying back the property
without notifying him inures to his benefit as co-owner and that he is entitled to a one-third share of the
property.

The RTC rendered, ordering the plaintiffs to execute a Deed of Sale in favor of the defendant, the one-
third share of the property in question, presently possessed by him. The RTC held that the right of
petitioner to purchase from the respondents his share in the disputed property was recognized by the
provisions of the Extrajudicial Settlement of Estate, which the parties had executed before the
respondents bought the subject lot from the Bank.The CA promulgated the presently assailed Decision,
reversing and setting aside the Decision of the RTC and ordering petitioner to immediately surrender
possession of the subject property to the respondents. The CA ruled that when petitioner and respondents
did not redeem the subject property within the redemption period and allowed the consolidation of
ownership and the issuance of a new title in the name of the Bank, their co-ownership was extinguished.

ISSUE:Whether or not the subject property forms part of the estate of petitioner and respondents’ father

RULING: No. The court ruled that the subject property does not form part of the estate of Rufo
considering that ownership over the same was transferred to the bank prior to the death of Rufo.
Inheritance consists of existing property, as well as accrued property, and transmissible rights and
obligations at the time of death of the decedent. Thus, since Rufo lost ownership over the subject property
during his lifetime, the same no longer forms part of his estate to which his heirs may lay claim at the
time of his death. Consequently, his children never inherited the property. The Court further ruled that
petitioner and respondents are not co-owners of the subject property and there is no property to partition,
as the disputed lot never formed part of the estate of their deceased father.

Remedial Law Review II (CY 2018-2019) – Saturday 1:00pm – 5:00pm: Case Digest Compilation [Atty. Brondial] Page 145
H.3. Eugenio Feliciano, substituted by hiswife Ceferina De Palmafeliciano,Angelina De
Leon,representing the heirs of EstebanFeliciano, Trinidad Valiente,andBasilia Trinidad,
represented byher son Dominador T. Feliciano, Petitioners vs. Pedro Canoza, Delia
Feliciano,Rosauro Feliciano, ElsaFeliciano and PoncianoFeliciano,Respondents.

G.R. No. 161746; September 1, 2010; Ponente: Villaram, Jr., J. (Third Division)

~ A deed of extrajudicial partition executed without including some of the heirs, who had no knowledge
of and consent to the same, is fraudulent and vicious. Hence, an action to set it aside on the ground of
fraud could be instituted. Such action for the annulment of the said partition, however, must be brought
within four (4) years from the discovery of the fraud.

FACTS:On March 28, 1972,Leona, Maria, Pedro and Salina, all surnamed Feliciano, declared
themselves to be the only surviving heirs of Antonio Feliciano, excluding the heirs of Esteban and
Doroteo Feliciano, children of Antonio whom predeceased him.

Leona, Maria, Pedro and Salina, on even date, executed a deed of absolute sale or Kasulatan sa Ganap Na
Bilihan over a property owned by Antonio in favor of the Jacinto Feliciano, Felisa Feliciano and Pedro
Canoza.

During his lifetime, Jacinto Feliciano applied for a free patent over the portion of land hebought,
declaring that the same was a public. Jacinto was issued Free Patenton November 28, 1977, but
unfortunately, it was burned. Pedro Canoza, for his part, also applied for a free patent over theportion of
land which he bought, therein he was issued Free Patentnow covered by Original Certificate of Title
(OCT) No. P-364, on February 23, 1979.

The surviving heir of Esteban and Doroteo Feliciano filed a complaint for the Declaration of Nullity of
Documents and Title, Recovery of Real Property and Damages on October 18, 1993, alleging that the
settlement of the estate and sale were done without their participation and consent as heirs of Esteban and
Doroteo.

[…] the RTC rendered a Decision declaring the extra-judicial settlement of estate of Antonio Feliciano
null and void and declaring the sale of the property in question null and void.

Aggrieved, respondents appealed to the CA which rendered the assailed Decision reversing the trial
court’s decision ruled that prescription had set in explaining that the applicable prescriptive period to
annul a deed of extrajudicial settlement four (4) years from the discovery of the fraud. […]

ISSUE:

1. WON the CA erred in reversing the trialcourt’s decision.

RULING:

1. NO.

[…] the heirs of Doroteo and Esteban did notparticipate in the extrajudicial partition executed by
Salina with the other compulsory heirs,Leona, Maria and Pedro. Undeniably, the said deed was
fraudulently obtained as it deprived theknown heirs of Doroteo and Esteban of their shares in the estate. A
deed of extrajudicialpartition executed without including some of the heirs, who had no knowledge of and
consent tothe same, is fraudulent and vicious.Hence, an action to set it aside on the ground of fraudcould
be instituted. Such action for the annulment of the said partition, however, must bebrought within four (4)
years from the discovery of the fraud.

Citing the case of Gerona vs. De Guzman:

“Inasmuch as petitioners seek to annul the aforementioned deed of extra-judicial


settlementupon the ground of fraud in the execution thereof, the action therefor may
be filed within four(4) years from the discovery of the fraud. Such discovery is
deemed to have taken place, in the case at bar, on June 25, 1948, when
saidinstrument was filed with the Register of Deeds and new certificates of title were

Remedial Law Review II (CY 2018-2019) – Saturday 1:00pm – 5:00pm: Case Digest Compilation [Atty. Brondial] Page 146
issued in the nameof respondents exclusively, for the registration of the deed of
extra-judicial settlement constituteconstructive notice to the whole world.”

Evidently, the applicable prescriptive period to institute the action to annul the deed ofextrajudicial
settlement was four (4) years counted from the discovery of fraud […].However, the records show that
petitioners’ complaint wasfiled only on October 18, 1993, or almost sixteen (16) years after Jacinto
Feliciano was issuedFree Patent No. (IV-4) 012293 on November 28, 1977, and almost fourteen (14)
years from thetime Pedro Canoza was issued OCT No. P-364 on November 28, 1979. As petitioners are
deemed to have obtained constructive notice of the fraud upon the registration of the FreePatent, they
clearly failed to institute the present civil action within the allowable period. Thesame result obtains even
if their complaint is treated as one (1) essentially for reconveyance asmore than ten (10) years have
passed since petitioners cause of action accrued. The CAcommitted no error in dismissing their
complaint.

Remedial Law Review II (CY 2018-2019) – Saturday 1:00pm – 5:00pm: Case Digest Compilation [Atty. Brondial] Page 147
H.4. CARMELA BROBIO MANGAHAS vs. EUFROCINA A. BROBIO

G.R. No. 183852; October 20, 2010; Ponente: Nachura, J.

FACTS: Pacifico died intestate and was survived by his wife, respondent Eufrocina A. Brobio, and four
legitimate and three illegitimate children. Petitioner Carmela Brobio Mangahas is one of the illegitimate
children. The heirs of the deceased executed a Deed of Extrajudicial Settlement of Estate. Respondent
agreed to pay the petitioner for the latter’s share in the estate. Upon demand, Respondent failed to pay but
subsequently issued a Promissory Note in favor of the petitioner.

When the promissory note fell due, respondent failed and refused to pay despite demand. Thus, petitioner
filed a Complaint for Specific Performance with Damages against respondent. Trial court rendered a
decision in favor of petitioner.

CA reversed the trial court decision and held that the waiver of petitioner’s share in the three properties,
as expressed in the deed of extrajudicial settlement, may not be considered as the consideration of the
promissory note, considering that petitioner signed the Deed way back in 2002 and she had already
received the consideration of P150,000.00 for signing the same. The CA went on to hold that if petitioner
disagreed with the amount she received, then she should have filed an action for partition.

ISSUE: Whether or not an action for partition is the proper remedy to contest the terms of the extra-
judicial settlement.

RULING: The remedy suggested by the CA is not the proper one under the circumstances. An action for
partition implies that the property is still owned in common. Considering that the heirs had already
executed a deed of extrajudicial settlement and waived their shares in favor of respondent, the properties
are no longer under a state of coownership; there is nothing more to be partitioned, as ownership had
already been merged in one person

Remedial Law Review II (CY 2018-2019) – Saturday 1:00pm – 5:00pm: Case Digest Compilation [Atty. Brondial] Page 148
I. Forcible Entry and Unlawful Detainer (Rule 70)

I.1. CARMENCITA SUAREZ,Petitioner, - versus – MR. and MRS. FELIX E. EMBOY, JR.and
MARILOU P. EMBOYDELANTAR, Respondents

G.R. No. 187944; MAR 12 2014

FACTS: Respondent spouses Felix and Marilou Emboy inherited a 222 sq.m. lot, Lot No. 1907-A-2
covered by TCT No. T-174880, from their mother, Claudia. The lot was originally part of a bigger
property, Lot No. 1907-A, which was partitioned into 5 different lots for Claudia and her siblings as
inheritance from their parents Carlos Padilla and Asuncion Pacres. In 2004, the respondents were asked
by their cousins, the heirs of Vicente (Claudia's brother) to vacate the subject lot and transfer to Lot No.
1907-A-5 which was a landlocked portion without a right of way. The respondents refused, insisting that
Claudia's inheritance pertained to Lot No. 1907-A-2. Shortly thereafter, the respondents received a
demand letter from counsel of the petitioner, requiring them to vacate the said lot on the information that
petitioner had already purchased the said lot from the respondent's relatives on Feb. 12, 2004.
Respondents refused and went on to file a complaint for nullification of the partition and for the issuance
of new TCTs covering the heir's respective portions of Lot No. 1907-A after examining the records
pertaining to the subject lot and uncovering possible anomalies like forged signatures and alterations in
the execution of a series of deeds of partition relative to the same lot.

The petitioner filed a complaint for unlawful detainer against respondents on Dec. 8, 2004 before the
MTCC alleging that she bought the subject lot from Remedios, Moreno, Veronica and Dionesia, the
registered owners thereof and the persons who allowed the respondents to occupy the same by mere
tolerance.

The MTCC upheld respondent's claims in its 2006 decision and ordered the respondents to vacate the
subject lot and remove at their expence all the improvements they had built thereon. The RTC in its 2008
decision affirmed the MTCC ruling. Through a petition for review filed before the CA, respondents
argued that they have been occupying the subject lot in the concept of owners for several decades while
petitioner was buyer in bad faith for having purchased the property despite the notice of lis pendens
clearly annotated on the subject lot's title. The respondents insisted that the Heirs of Vicente, who had
allegedly sold the subject lot to petitioner, had never physically occupied the same. Hence, there was no
basis at all for petitioner's claim that the respondents' possession of the subject lot was by mere tolerance
of the alleged owners. Respondents also discovered a duly notarized document showing that Vicente and
his spouse, Dionesoa, had waived their hereditary rights to Lot No. 1907-A when he used the same as a
collateral in obtaining a loan from PNB. The loan was paid for by Carlos and Asuncion. Based on this
discoveries, the Heirs of Vicente did not have the rights over the subject lot to convey to petitioner.
Respondents also averred that petitioner's complaint lacked a cause of action because the certification to
file an action was issued in the name of James Tan Suarez (petitioner's brother), who had no real rights or
interests over the subject lot. Also, no demand to vacate was made upon the respondents after the
execution of the deed of sale (April 1, 2004), which served as petitioner's primary basis for her claim to
the property. The absence of such demand rendered the complaint fatally defective, as the date of its
service should be the reckoning point of the one-year period within which the suit can be filed.

ISSUES:
1. Whether or not Carmencita'scomlaintagaints the respondents had sufficiently alleged and
proven a cause of action for unlawful detainer.
2. Whether or not the pendency of the respondents' petition for nullification of partition of
Lot No. 1907-A and for the issuance of new certificates of title can abate Carmencita's
ejectment suit.

RULING:
1. Carmencita had not amply alleged and proven that all the requisites for unlawful detainer are present in
the case at bar. “Without a doubt, the registered owner of real property is entitled to its possession.
However, the owner cannot simply wrest possession thereof from whoever is in actual occupation of the
property. To recover possession, he must resort to the proper judicial remedy and, once he chooses what
action to file, he is required to satisfy the conditions necessary for such action to prosper.”

Remedial Law Review II (CY 2018-2019) – Saturday 1:00pm – 5:00pm: Case Digest Compilation [Atty. Brondial] Page 149
2. As an exception to the general rule, the respondents’ petition for nullification of the partition of Lot No.
1907-A can abate Carmencita’s suit for unlawful detainer. Carmencita’s complaint for unlawful detainer
is anchored upon the proposition that the respondents have been in possession of the subject lot by mere
tolerance of the owners. The respondents, on the other hand, raise the defense of ownership of the subject
lot and point to the pendency of Civil Case No. CEB-30548, a petition for nullification of the partition of
Lot No. 1907-A, in which Carmencita and the Heirs of Vicente were impleaded as parties. Further, should
Carmencita’s complaint be granted, the respondents’ house, which has been standing in the subject lot for
decades, would be subject to demolition. The foregoing circumstances, thus, justify the exclusion of the
instant petition from the purview of the general rule.

Remedial Law Review II (CY 2018-2019) – Saturday 1:00pm – 5:00pm: Case Digest Compilation [Atty. Brondial] Page 150
I.2. Atty. Virgilio Alconera vs. Alfredo Palanan

A.M P – 12-3069; January 20, 2014; Velasco, Jr. J

FACTS: Rafols was a defendant in an unlawful detainer case of which Alconera is his counsel. MTCC
ruled against Rafols. Rafols through Alconera appealed with RTC. While the appeal is pending, RTC
issued a writ of possession upon the motion of the complainant in the ejectment suit. Alconera filed a
motion for reconsideration assailing the writ of possession issued. On March 17, 2011, herein Respondent
Sheriff Palanan enforced the writ of execution against Rafols. During the enforcement of the writ,
Alconera and the Sheriff had a verbal disagreement over the phone as the former is asserting that the writ
should not be enforced pending the resolution of his motion for reconsideration. On April 6, 2011,
Alconera went to RTC Br. 36 with his daughter to confront respondent sheriff. The face-off escalated into
a heated argument was recorded on video.
Alconera filed an administrative complaint for Grave Misconduct and Making Untruthful Statement
against the respondent sheriff for before Supreme Court and was referred to Office of the Court
Administrator on April 6, 2011. Respondent filed his comment where he averred that the duty of a court
sheriff in enforcing a writ of execution is ministerial, and without a TRO enjoining it, a sheriff is duty
bound to implement it.

ISSUE: Whether or not a motion seeking the reconsideration of the court in issuing a writ of possession
in an unlawful detainer suit would stop the enforcement of the writ.

RULING: The answer is in the negative. In ejectment cases, the rulings of the courts are immediately
executory and can only be stayed via compliance with Section 19, Rule 70 of the Rules of Court. Under
said Sec. 19, Rule 70, a judgment on a forcible entry and detainer action is made immediately executory
to avoid further injustice to a lawful possessor. The defendant in such a case may have such judgment
stayed only by
(a) perfecting an appeal; (b) filing a supersedeas bond; and (c) making a periodic deposit of the rental or
reasonable compensation for the use and occupancy of the property during the pendency of the appeal.

The failure of the defendant to comply with any of these conditions is a ground for the outrightexecution
of the judgment, the duty of the court in this respect being ministerial and imperative. Hence, if the
defendant-appellant has perfected the appeal but failed to file a supersedeas bond, the immediate
execution of the judgment would automatically follow. Conversely, the filing of a supersedeas bond will
not stay the execution of the judgment if the appeal is not perfected. Necessarily then, the supersedeas
bond should be filed within the period for the perfection of the appeal.

In the case at bar,complainant lost his client’s case and appealed to the RTC. Hisclient has also been
periodically depositing rental with the court for the use of the property pending appeal. However, as
ruled by the RTC, the bond filed did not meet the legal requirements because first and foremost, the bond
posted was a property bond, not cash nor surety. Furthermore, Rafols did not own the property he posted
as bond and besides, it was also not issued in favour of the plaintiff in the ejectment case. Because of the
non-compliance with the requirements under the above-quoted rule, the execution of the judgment was
not effectively stayed.

The only exceptions to non-compliance are the existence of fraud, accident, mistake or excusable
negligence which prevented the defendant from posting the supersedeas bond or making the monthly
deposit, or the occurrence of supervening events which brought about a material change in the situation of
the parties and which would make the execution inequitable.
But whether or not these obtain in the case at bar is an issue best left to the court that issued the writ of
execution.

DOCTRINE: IN EJECTMENT CASES, THE RULINGS OF THE COURTS ARE IMMEDIATELY


EXECUTORY AND CAN ONLY BE STAYED VIA COMPLIANCE WITH SECTION 19, RULE 70 OF
THE RULES OF COURT.

Under said Sec. 19, Rule 70, a judgment on a forcible entry and detainer action is made immediately
executory to avoid further injustice to a lawful possessor. The defendant in such a case may have such
judgment stayed only by (a) perfecting an appeal; (b) filing a supersedeas bond; and (c) making a
periodic deposit of the rental or reasonable compensation for the use and occupancy of the property
during the pendency of the appeal. The failure of the defendant to comply with any of these conditions is

Remedial Law Review II (CY 2018-2019) – Saturday 1:00pm – 5:00pm: Case Digest Compilation [Atty. Brondial] Page 151
a ground for the outright execution of the judgment, the duty of the court in this respect being ministerial
and imperative. Hence, if the defendant-appellant has perfected the appeal but failed to file a
supersedeas bond, the immediate execution of the judgment would automatically follow. Conversely, the
filing of a supersedeas bond will not stay the execution of the judgment if the appeal is not perfected.
Necessarily then, the supersedeas bond should be filed within the period for the perfection of the appeal.
(ALCONERA VS. PALLANAN [2014]).

Remedial Law Review II (CY 2018-2019) – Saturday 1:00pm – 5:00pm: Case Digest Compilation [Atty. Brondial] Page 152
I.3. TEODORO S. TEODORO (Deceased), Substituted by his heirs/sons NELSON TEODORO and
ROLANDO TEODORO, Petitioners, vs.DANILO ESPINO, ROSARIO SANTIAGO, JULIANA
CASTILLO, PAULINA LITAO, RAQUEL RODRIGUEZ, RUFINA DELA CRUZ, and LEONILA
CRUZ, Respondents.

G.R. No. 189248; February 5, 2014

FACTS:A case for Forcible Entry was filed by the predecessor-in-interest of petitioners heirs of Teodoro
S. Teodoro (TeodoroTeodoro), against respondents a squabble for physical possession of a portion of a
real property, the ownership of which is traceable to GenaroTeodoro (Genaro). 

The subject property is a portion within Cadastral Lot No. 2476 with a total area of 248 square meters,
covered by Tax Declaration No. 99-05003-0246, registered in the name of Genaro, long deceased
ascendant of all the parties. The subject property pertains to the vacant lot where the old ancestral house
of Genaro stood until its demolition in June 2004, at the instance of TeodoroTeodoro.

Of all Genaro’s children, only Petra occupied the subject property, living at the ancestral house. Genaro’s
other children, specifically Santiago, Maria and Mariano were bequeathed, and stayed at, a different
property within the same locality, still from the estate of their father. 

After Petra’s death, her purported will, a holographic will, was probated in Special Proceedings No. 1615-
M before RTC, Branch 8, Malolos, Bulacan, which Decision on the will’s extrinsic validity has become
final and executory. In the will, Petra, asserting ownership, devised the subject property to
TeodoroTeodoro, her nephew. TeodoroTeodoro effected the demolition of the ancestral house, intending
to use the subject property for other purposes. 

Respondents, who resided at portions of Lot No. 2476 that surround the subject property on which
the ancestral house previously stood, erected a fence on the surrounding portion, barricaded its
frontage, and put up a sign thereat, effectively dispossessing TeodoroTeodoro of the property
bequeathed to him by Petra. 

After TeodoroTeodoro’s demand for respondents to vacate the subject property went unheeded, he filed
the complaint for forcible entry against respondents. 

ISSUE 1: Whether the action for unlawful detainershould prosper?

ISSUE 2: Whether petitioner must prove exclusive ownership for the case of unlawful detainer to
prosper?

RULING 1: Yes.In forcible entry cases one employs force, intimidation, threat, strategy or stealth to
deprive another of physical possession of real property. 

The whole of Lot No. 2476 including the portion now litigated is, owing to the fact that it has remained
registered in the name of Genaro who is the common ancestor of both parties herein, co-owned property.
All, or both TeodoroTeodoro and respondents are entitled to exercise the right of possession as co-
owners. Neither party can exclude the other from possession.
 
Although the property remains unpartitioned, the respondents in fact possess specific areas.
TeodoroTeodoro can likewise point to a specific area, which is that which was possessed by Petra.
TeodoroTeodoro cannot be dispossessed of such area, not only by virtue of Petra's bequeathal in his favor
but also because of his own right of possession that comes from his co- ownership of the property. As the
RTC concluded, petitioners, as heirs substituting TeodoroTeodoro in this suit, should be restored in the
lawful possession of the disputed area. 

RULING 2: NO.because such ownership issue is "inutile in an ejectment suit except to throw light on the
question of possession." Exclusive ownership of Lot No. 2476 or a portion thereof is not in this case
required of TeodoroTeodoro for him to be entitled to possession.

DOCTRINE:
The ground rules in forcible entry cases:
(1) One employs force, intimidation, threat, strategy or stealth to deprive another of physical possession
of real property.

Remedial Law Review II (CY 2018-2019) – Saturday 1:00pm – 5:00pm: Case Digest Compilation [Atty. Brondial] Page 153
(2) Plaintiff must allege and prove prior physical possession of the property in litigation until deprived
thereof by the defendant. This requirement implies that the possession of the disputed land by the latter
was unlawful from the beginning.

(3) The sole question for resolution hinges on the physical or material possession (possession de facto) of
the property. Neither a claim of juridical possession (possession de jure) nor an averment of ownership
by the defendant can, at the outset, preclude the court from taking cognizance of the case.

(4) Ejectment cases proceed independently of any claim of ownership, and the plaintiff needs merely to
prove prior possession de facto and undue deprivation thereof.

Remedial Law Review II (CY 2018-2019) – Saturday 1:00pm – 5:00pm: Case Digest Compilation [Atty. Brondial] Page 154
I.4. Ferrer vs. Rabaca

A.M. No. MTJ-05-1580; October 6, 2010; Bersamin, J.

FACTS: This administrative case charges Hon. Romeo A. Rabaca, then the Presiding Judgeof Branch 25
of the Metropolitan Trial Court of Manila (MeTC), with ignorance ofthe law, disregard of the law,
dereliction of duty, knowingly rendering an unjustinterlocutory order, and violation of the Code of
Conduct for Government Officials.

The complainants were the President and the Executive Director of the plaintiff inan ejectment suit. In the
said suit, respondent Judge rendered a decision in favor ofthe plaintiff. Consequently, plaintiff’s counsel
filed a motion for immediateexecution, praying that a writ of execution be issued "for the immediate
executionof the aforesaid Judgment. Respondent Judge denied the motion for immediateexecution on the
ground that notice of appeal had been timely filed hence jurisdiction over the case is now elevated to the
Regional Trial Court.

ISSUE: Whether or not a motion for issuance of writ of execution in an ejectmentsuit may no longer be
issued when notice of appeal had already been filed.

RULING:
No. It is basic rule in ejectment cases that the execution of judgment in favorof the plaintiff is a matter of
right and mandatory. This has been the consistentruling of the Court in a number of cases involving the
same issue posed beforethe respondent judge. Respondent Judge is expected to know this and
hisjustification of erroneous application of the law, although mitigating, could notexculpate him from
liability.

Indeed, respondent Judge should have granted the plaintiff’s motion for immediateexecution considering
that the defendant did not file the sufficient supersedeas bonddespite having appealed. Granting the
plaintiff’s motion for immediate executionbecame his ministerial duty upon the defendant’s failure to file
the sufficientsupersedeas bond. Section 19, Rule 70, of the Rules of Court clearly imposes suchduty, viz:

Section 19. Immediate execution of judgment; how to stay same. — If judgment isrendered against the
defendant, execution shall issue immediately upon motion,unless an appeal has been perfected and the
defendant to stay execution files asufficient supersedeas bond, approved by the Municipal Trial Court
and executedin favor of the plaintiff to pay the rents, damages, and costs accruing down to thetime of the
judgment appealed from, and unless, during the pendency of the appeal,he deposits with the appellate
court the amount of rent due from time to time underthe contract, if any, as determined by the judgment
of the Municipal Trial Court.

In the absence of a contract, he shall deposit with the Regional Trial Court thereasonable value of the
use and occupation of the premises for the precedingmonth or period at the rate determined by the
judgment of the lower court on orbefore the tenth day of each succeeding month or period. The
supersedeas bondshall be transmitted by the Municipal Trial Court, with the other papers, to theclerk of
the Regional Trial Court to which the action is appealed.

Remedial Law Review II (CY 2018-2019) – Saturday 1:00pm – 5:00pm: Case Digest Compilation [Atty. Brondial] Page 155
I.5.CGR CORPORATION, et al. v. ERNESTO L. TREYES, JR.

522 SCRA 765 (2007)

FACTS: The recoverable damages in forcible entry and detainer cases refer to “rents” or “the reasonable
compensation for the use and occupation of the premises” or “fair rental value of the property” and
attorney’s fees and costs. CGR Corporation, owned by Herman M. Benedicto and Alberto R. Benedicto,
leased several hectares of public land, mostly consisting of fishponds, in Negros Occidental. Ernesto L.
Treyes, Jr., with his men, forcibly entered the leased properties and barricaded the entrance to the
fishponds, set up a barbed wire fence along the road going to CRG Corporation‘s fishponds,
and harvested several tons of milkfish, fry and fingerlings.

CGR filed with the Municipal Trial Court (MTC) in Sagay City separate complaints for Forcible Entry
with Temporary Restraining Order with Preliminary Injunction and Damages and reserved a separate civil
action.

The MTC found Treyes and his men guilty of forcible entry. CGR filed a separate complaint alleging
therein that he suffered damages for the actions of Treyes during and after the forcible entry. A claim for
additional damages which arose from incidents occurring after the dispossession by Treyes of the
premises was thereafter prayed for. The MTC awarded the claims of CGR.

ISSUE: Can a complainant in a forcible entry case file an independent action for damages arising after
the act of dispossession had occurred?

RULING: Yes. The recoverable damages in forcible entry and detainer cases thus refer to rents or the
reasonable compensation for the use and occupation of the premises or fair rental value of the property
and attorneys fees and costs. Considering that the only issue raised in ejectment is that of rightful
possession, damages which could be recovered are those which the plaintiff could have sustained as a
mere possessor, or those caused by the loss of the use and occupation of the property, and not the
damages which he may have suffered but which have no direct relation to his loss of material possession.
Other damages must thus be claimed in an ordinary action.

Further, Court held that the ―rents‖ or the ―reasonable compensation for the use of the premises or the
fair rental value of the property and attorney‘s fees may be recovered through a separate action while the
forcible entry case is pending. The recoverable damages in forcible entry and detainer cases refer to
―rents‖ or ―the reasonable compensation for the use and occupation of the premises‖ or ―fair rental
value of the property‖ and attorney‘s fees and costs.

Remedial Law Review II (CY 2018-2019) – Saturday 1:00pm – 5:00pm: Case Digest Compilation [Atty. Brondial] Page 156
I.6. SPOUSES ABAD vs. FIL-HOMES REALTY AND DEVELOPMENT CORPORATION

G.R. No. 189239; November 24, 2010; Carpio-Morales, J.

FACTS: Fil-Homes Realty (Fil-Homes) and Magdiwang Realty are co-owners of two (2) parcels of lot in
Paranaque City covered by separate certificates of title. They filed a complaint for unlawful detainer
against Spouses Leticia and Jose Ervin Abad and others(Sps. Abad) with the MeTC Paranaque in 2003. It
was alleged therein that Sps. Abad, through tolerance, had occupied the subject lots since 1980 but
ignored their repeated demands to vacate it. Sps. Abad countered that there is no possession by tolerance
for they have been in adverse, continuous and uninterrupted possession of the said lots for more than
thirty (30) years, and that Fil-Homes’ predecessor-in-interest (Pilipinas Development Corporation) had no
title to the lots. In any event, Sps. Abad contended that the question of ownership must first be settled
before the issue of possession may be resolved.

During the pendency of the ejectment suit, the City of Paranaque filed an expropriation proceeding with
the RTC of Paranaque involving the lots subject of the ejectment case to be used as socialized housing
project for distribution to the occupants including the Sps. Abad. Then, a writ of possession was issued
and a Certificate of Turn-Over was given to the City of Paranaque.

Thereafter (after two years), the MeTC rendered a Decision in the unlawful detainer case against the Sps.
Abad ordering them to vacate and surrender the possession of the subject lots and to pay reasonable
compensation therefor. The MeTC further held that as no payment (of just compensation) had been made
to Fil-Homes for the subject lots, they still maintain ownership thereon. It added that Sps. Abad cannot
claim a better right by virtue of the issuance of a writ of possession (in favor of the City of Paranaque) for
the project beneficiaries have yet to be named.

Sps. Abad appealed the MeTC Decision to the RTC,which in turn, reversed the lower court and dismissed
the complaint for unlawful detainer filed by Fil-Homes ruling that “it is not the allegations of the
Complaint that finally determine whether a case is unlawful detainer, rather it is the evidence in the case”
and that “when there is no ‘tolerance’ right from the start of the possession sought to be recovered, the
case of unlawful detainer will not prosper.” The RTC further ruled that the issuance of a writ of
possession in favor of the City of Paranaque bars the continuation of the unlawful detainer proceedings,
and since the judgment had already been rendered in the expropriation proceedings which effectively
turned over the lots to the city government, the MeTC has no jurisdiction to “disregard the final judgment
and writ of possession” due to non-payment of just compensation.Aggrieved by the decision of the RTC,
Fil-Homes filed a Petition for Review with the CA alleging that its act of allowing several years to pass
without requiring Sps. Abad to vacate nor filing an ejectment case against them amounts to acquiescence
or tolerance of their possession.

The CA rendered a Decision setting aside the decision of the RTC and reinstating the decision of MeTC.
It ruled that Sps. Abad did not present evidence to rebut the Fil-Homes’ allegation of possession by
tolerance, and considering that the Sps. Abad’s admission that they commenced occupation of the subject
property without the permission of the previous owner (Pilipinas Development Corporation) as indicium
of tolerance by Fil-Homes’ predecessor-in-interest. The CA further ruled that the issuance of a writ of
possession in the expropriation proceedings does not signify the completion of the expropriation
proceedings.

Thus, Sps. Abad filed a Petition for Review on Certiorari with the SC.

ISSUES:
1. WON an ejectment proceeding will be suspended or the resolution thereof be held in abeyance
during the pendency of a civil action on ownership of the subject lot.
2. WON the issuance of writ of possession in an expropriation proceeding signifies the completion
of the said proceeding thereby vesting ownership of thesubject land on the beneficiary thereof.

RULING:
1. As a general rule, ejectment proceedings, due to its summary nature, are not suspended or their
resolution held in abeyance despite the pendency of a civil action regarding ownership. However,
Section 1, Commonwealth Act No. 538 essentially provides that “x x x any action for ejectment
against the tenants occupying said lands shall be automatically suspended, for such time as may be
required by the expropriation proceedings or the necessary negotiations for the purchase of the
lands, in which latter case, the period of suspension shall not exceed one year. To avail himself of

Remedial Law Review II (CY 2018-2019) – Saturday 1:00pm – 5:00pm: Case Digest Compilation [Atty. Brondial] Page 157
the benefits of the suspension, the tenants shall pay to the landowner the current rents as they
become due or deposit the same with the court where the action for ejectment has been instituted.”
Here, Sps. Abad did not comply with any of the acts mentioned in the law to avail of the benefits of
the suspension.Since the Sps. Abad failed to pay or deposit the rents to Fil-Homes, the ejectment
proceeding will not be suspended during the pendency of the expropriation proceeding.

2. NO. The mere issuance of a writ of possession in the expropriation proceedings did not transfer
ownership of the lots in favor of the City of Paranaque.  Such issuance was only the first stage in
expropriation.  There is even no evidence that judicial deposit had been made in favor of Fil-Homes’
prior to the City of Paranaque's possession of the subject lots.Even if the lots are eventually
transferred to the City of Paranaque, it is non sequitur for Sps. Abad to claim that they are
automatically entitled to be beneficiaries thereof.  For certain requirements must be met and complied
with before they can be considered to be beneficiaries.

DOCTRINE: Expropriation of lands consists of two stages:

The first is concerned with the determination of the authority of the plaintiff to exercise the power of
eminent domain and the propriety of its exercise in the context of the facts involved in the suit. It ends
with an order, if not of dismissal of the action, "of condemnation declaring that the plaintiff has a lawful
right to take the property sought to be condemned, for the public use or purpose described in the
complaint, upon the payment of just compensation to be determined as of the date of the filing of the
complaint.

The second phase of the eminent domain action is concerned with the determination by the court of "the
just compensation for the property sought to be taken." This is done by the court with the assistance of
not more than three (3) commissioners.

It is only upon the completion of these two stages that expropriation is said to have been completed. The
process is not complete until payment of just compensation. Accordingly, the issuance of the writ of
possession in this case does not write finis to the expropriation proceedings.

Remedial Law Review II (CY 2018-2019) – Saturday 1:00pm – 5:00pm: Case Digest Compilation [Atty. Brondial] Page 158
I.7. AMADA C. ZACARIASvs.VICTORIA ANACAY, EDNA ANACAY,
CYNTHIAANACAYGUISIC, ANGELITO ANACAY, JERMIL ISRAEL, JIMMY ROY ISRAEL
and all other persons claiming authority under them

G.R. No. 202354; September 24, 2014; VILLARAMA, JR., J.:

FACTS: A complaint for Ejectment with Damages/Unlawful Detainer was filed with the Municipal Trial
Court by petitioner Amada Zacarias against respondents. The MTC dismissed the complaint holding that
the complaint failed to state the essential elements of an action for unlawful detainer; that conversely, the
allegations are more consistent with an action for forcible entry which should be filed within one year.
Moreover, since the complaint was filed after the lapse of one year from her discover, the MCT has no
jurisdiction over the case.

The RTC reversed the decision resolving that the complaint stated that petitioner was in lawful possession
and acceded to respondents’ request to stay in the premises for a certain period of time. The RTC held
that the suit is one for unlawful detainer because the respondents unlawfully withheld the property from
petitioner after she allowed them to stay for a certain period; that the subsequent oral agreement as to
allowance of stay for a certain period can be converted to a contract since it is not prohibited by law nor
contrary to morals or good customs.

In a petition for certiorari by the respondent, the CA re-affirmed the MTC decision, and commenting that
since the reglementary period for the correct action which should be for forcible entry had elapsed, the
petitioner may nevertheless file an ordinary civil action of accion publicianaor accion reivindicatoria with
the proper regional trial court.

ISSUE: W/N the MTC has no jurisdiction over the case because the complaint alleged an action for
forcible entry

RULING: The MTC has no jurisdiction over the case because the complaint alleged an action for
forcible entry. The allegations in the complaint determine the nature of the action. Knowing the nature of
the action determines the court jurisdiction as well (since by it, you can now look at B.P. 129 what nature
of action the complaint falls into). Also, an action for forcible entry or unlawful detainer (ejectment cases)
must show on its face statement of facts to bring it within the class of cases which the rule thereon
provides for a summary procedure so that the court can take cognizance of it without resort to parol
evidence.

The difference between the action of forcible entry and unlawful detainer lies on the nature of the right of
possession held by the defendant over the subject real property (specifically land, building, or both) of the
complaint.

In forcible entry, the defendant acquired physical possession over the property by means of force,
intimidation, strategy, threat or stealth (FISTS) depriving the plaintiff of the same.In illegal detainer, the
defendant acquired lawful physical possession over the property; thereafter, his possession became
unlawful due to his retention of the property in spite of the expiration or termination of his right of
possession.

Corollary, in forcible entry, the defendant has no prior possession of the property or at least his entry
thereon is illegal; on the other hand, in unlawful detainer; the defendant has prior lawful possession which
thereafter became unlawful.

In this case, the complaint substantially alleged that:

The plaintiff is the owner of a certain parcel of land, and was in lawful possession and control over the
said property; that she discovered that the defendants have entered the subject property and occupied the
same; that plaintiff made a demand to the defendants asking them to leave the premises; that the
defendants requested time to leave and the plaintiff acceded to the request; that the defendants committed
to vacate the subject property at a certain date; but, in spite of several repeated demands, the defendants
unjustifiably refused to vacate the subject property, thus prompting the plaintiff to write the defendant of
a formal and final demand to vacate the premises and to pay reasonable compensation for their illegal use
and occupancy of the subject property; finally, plaintiff referred the matter to the Lupon Tagapamayapa

Remedial Law Review II (CY 2018-2019) – Saturday 1:00pm – 5:00pm: Case Digest Compilation [Atty. Brondial] Page 159
of the Barangay for possible conciliation but to no avail, thus the barangay issued a certification to file
action.

The above allegationsdo not contain any averment of fact that that they permitted or tolerated the
occupation of the property by respondents from the very start. Nothing has been said on how respondents’
entry was effected or how and when dispossession started. Admittedly, no express contract existed
between the parties. Conversely, the complaint’s allegation as to defendant’s clandestine entry on the
subject property without the plaintiff’s consent is constitutive of forcible entry, not unlawful detainer.

Per se, the alleged possession could not have been legal from the very start as it was without the
plaintiff’s knowledge or consent, much less was it based on any contract, express or implied. Petitioner’s
toleration of defendant’s stay in the meantime does not give the latter a right to possess the subject
property.To justify an action for unlawful detainer, it is essential that the plaintiff’s supposed acts of
tolerance must have been present right from the start of the possession which is later sought to be
recovered. Otherwise, if the possession was unlawful from the start, an action for unlawful detainer would
be an improper remedy.

Conclusively, the complaint is not for unlawful detainer, and consequently, the MTC shall have no
jurisdiction over the case for failing to satisfy the jurisdictional requirement of a valid cause for unlawful
detainer.

DOCTRINE: The jurisdictional facts must appear on the face of the complaint. If the complaint failed to
state how the entry was effected or how and when the dispossession started, the remedy shall be an
ordinary civil action for accion publiciana or accion reinvindicatoria with the proper Regional Trial
Court.

A complaint sufficiently alleges a cause of action for unlawful detainer if it recites the following:

(1) Initially, possession of property by the defendant was by contract with or by tolerance of the plaintiff;

(2) Eventually, such possession became illegal upon notice by plaintiff to defendant of the termination of
the latter’s right of possession;

(3) Thereafter, the defendant remained in possession of the property and deprived the plaintiff of the
enjoyment thereof; and

(4) Within one year from the last demand on defendant to vacate the property, the plaintiff instituted the
complaint for ejectment.

Forcible entry into the land is an open challenge to the right of the possessor. Violation of that right
authorizes the speedy redress – in the inferior court - provided for in the rules. If one year from the
forcible entry is allowed to lapse before suit is filed, then the remedy ceases to be speedy; and the
possessor is deemed to have waived his right to seek relief in the inferior court.

Second, if a forcible entry action in the inferior court is allowed after the lapse of a number of years, then
the result may well be that no action of forcible entry can really prescribe. No matter how long such
defendant is in physical possession, plaintiff will merely make a demand, bring suit in the inferior court –
upon a plea of tolerance to prevent prescription to set in - and summarily throw him out of the land. Such
a conclusion is unreasonable. Especially if we bear in mind the postulates that proceedings of forcible
entry and unlawful detainer are summary in nature, and that the one year time-bar to suit is but in
pursuance of the summary nature of the action. (Italics and underscoring supplied).

Remedial Law Review II (CY 2018-2019) – Saturday 1:00pm – 5:00pm: Case Digest Compilation [Atty. Brondial] Page 160
I.8. Manalang vs. Bacani

Remedial Law Review II (CY 2018-2019) – Saturday 1:00pm – 5:00pm: Case Digest Compilation [Atty. Brondial] Page 161
I.9.ESPERANZA SUPAPO AND THE HEIRS OF ROMEO SUPAPO, NAMELY: ESPERANZA,
REX EDWARD, RONALD TROY, ROMEO, JR., SHEILA LORENCE, ALL SURNAMED
SUPAPO, AND SHERYL FORTUNE SUPAPO-SANDIGAN, Petitioners, v. SPOUSES ROBERTO
AND SUSAN DE JESUS, MACARIO BERNARDO, AND THOSE PERSONS CLAIMING
RIGHTS UNDER THEM, Respondent.

G.R. No. 198356; April 20, 2015

FACTS: The Spouses Supapo filed a complaint for accion publiciana against Roberto and Susan de
Jesus (Spouses de Jesus), et al. (collectively, the respondents), with the Metropolitan Trial Court (MeTC)
of Caloocan City.

The complaint sought to compel the respondents to vacate a piece of land located in Novaliches, Quezon
City. The subject lot is registered and titled under the Spouses Supapo's names. The land has an assessed
value of thirty-nine thousand nine hundred eighty pesos (39,980.00).

The Spouses Supapo did not reside on the subject lot. They also did not employ an overseer but they
made sure to visit at least twice a year. During one of their visits in 1992, they saw two (2) houses built
on the subject lot. The houses were built without their knowledge and permission. They later learned that
the Spouses de Jesus occupied one house while Macario occupied the other one.

The Spouses Supapo demanded from the respondents the immediate surrender of the subject lot by
bringing the dispute before the appropriate Lupong Tagapamayapa. The Lupon issued a
Katibayan Upang Makadulog sa Hukuman (certificate to file action) for failure of the parties to settle
amicably.

The Spouses Supapo then filed a criminal case against the respondents for violation of Presidential
Decree No. 772 or the Anti-Squatting Law. The MeTC convicted the respondents.
The respondents appealed their conviction to the CA. While the appeal was pending, Congress
enacted Republic Act (RA) No. 8368, otherwise known as "An Act Repealing Presidential Decree No.
772," which resulted to the dismissal of the criminal case. The CA's dismissal of the criminal case
became final.

Notwithstanding the dismissal, the Spouses Supapo moved for the execution of the respondents' civil
liability, praying that the latter vacate the subject lot. The Regional Trial Court (RTC) granted the
motion and issued the writ of execution. The respondents moved for the quashal of the writ but the
RTC denied the same. The RTC also denied the respondents' motion for reconsideration.

The respondents thus filed with the CA a petition for certiorari to challenge the RTC's orders denying
the quashal of the writ and the respondent's motion for reconsideration. The CA granted the petition
and held that with the repeal of the Anti-Squatting Law, the respondents' criminal and civil
liabilities were extinguished. 
The Spouses Supapo thus filed the complaint for action publiciana.

After filing their Answer, the respondents moved to set their affirmative defenses for preliminary
hearing and argued that: (1) there is another action pending between the same parties; (2) the complaint
for accion publiciana is barred by statute of limitations; and (3) the Spouses Supapo's cause of action is
barred by prior judgment.
MeTC: Denied the motion to set the affirmative defenses for preliminary hearing. From the MeTC's
ruling, the respondents filed a petition for certiorari with the RTC.
RTC:Granted the petition for certiorari on two grounds, viz.:
(i) the action has prescribed; and
(ii) accion publiciana falls within the exclusive jurisdiction of the RTC.

The RTC denied the petitioners' motion for reconsideration.

Spouses Supapo appealed to the CA.

CA: Dismissed the appeal and held that the complaint for accion publiciana should have been lodged
before the RTC and that the period to file the action had prescribed.

Remedial Law Review II (CY 2018-2019) – Saturday 1:00pm – 5:00pm: Case Digest Compilation [Atty. Brondial] Page 162
The Spouses Supapo moved but failed to secure a reconsideration of the CA decision; hence, this present
petition.
SPS SUPAPO CONTENTION:

(1) the MeTC exercises exclusive original jurisdiction over accion publiciana where the assessed
value of the property does not exceed P20,000.00, or P50,000.00 if the property is located in
Metro Manila; and that
(2) prescription had not yet set in because their cause of action is imprescriptible under the Torrens
system.

DE JESUS CONTENTION:
The respondents argue that the complaint for accion publiciana was (1) filed in the wrong court; (2)
barred by prescription; and (3) barred by res judicata.

ISSUES:
1. Whether the MeTC properly acquired jurisdiction;
2. Whether the cause of action has prescribed; and 
3. Whether the complaint for accion publiciana is barred by res judicata.

RULING:
1. YES. MeTC properly acquired jurisdiction;
Accion publiciana is an ordinary civil proceeding to determine the better right of possession of realty
independent of title. It refers to an ejectment suit filed after the expiration of one year from the accrual of
the cause of action or from the unlawful withholding of possession of the realty.

In the present case, the Spouses Supapo filed an action for the recovery of possession of the subject lot
but they based their better right of possession on a claim of ownership.

Under Batas Pambansa Bilang 129, the jurisdiction of the RTC over actions involving title to or
possession of real property is plenary.

RA No. 7691, however, divested the RTC of a portion of its jurisdiction and granted the Metropolitan
Trial Courts, Municipal Trial Courts and Municipal Circuit Trial Courts the exclusive and original
jurisdiction to hear actions where the assessed value of the property does not exceed Twenty Thousand
Pesos (P20,000.00), or Fifty Thousand Pesos (P50,000.00), if the property is located in Metro Manila.
In the present case, the Spouses Supapo alleged that the assessed value of the subject lot, located in Metro
Manila, is P39,980.00. This is proven by the tax declaration issued by the Office of the City Assessor of
Caloocan. The respondents do not deny the genuineness and authenticity of this tax declaration.

Given that the Spouses Supapo duly complied with the jurisdictional requirements, we hold that the
MeTC of Caloocan properly acquired jurisdiction over the complaint for accion publiciana.
In this regard, the complaint must allege the assessed value of the real property subject of the complaint
or the interest thereon to determine which court has jurisdiction over the action. This is required because
the nature of the action and the court with original and exclusive jurisdiction over the same is determined
by the material allegations of the complaint, the type of relief prayed for by the plaintiff, and the law in
effect when the action is filed, irrespective of whether the plaintiffs are entitled to some or all of the
claims asserted therein

2. NO. The cause of action has not prescribed;


The respondents argue that the complaint for accion publiciana is dismissible for being filed out of time.
They invoke Article 555 of the Civil Code, which states: Art. 555. A possessor may lose his possession:
xxxx

(4) By the possession of another, subject to the provisions of Article 537, if the new possession has
lasted longer than one year. But the real right of possession is not lost till after the lapse of ten
years. (Emphasis supplied.)

Thus, without solid evidentiary basis, laches cannot be a valid ground to deny the Spouses Supapo's
petition. On the contrary, the facts as culled from the records show the clear intent of the Spouses Supapo
to exercise their right over and recover possession of the subject lot, viz.: (1) they brought the dispute to
the appropriate Lupon; (2) they initiated the criminal complaint for squatting; and (3) finally, they filed
the action publiciana. To our mind, these acts negate the allegation of laches.

Remedial Law Review II (CY 2018-2019) – Saturday 1:00pm – 5:00pm: Case Digest Compilation [Atty. Brondial] Page 163
With these as premises, we cannot but rule that the Spouses Supapo's right to recover possession of the
subject lot is not barred by prescription.

3. NO. The complaint is not barred by res judicata.

"Bar by prior judgment" means that when a right or fact had already been judicially tried on the merits
and determined by a court of competent jurisdiction, the final judgment or order shall be conclusive upon
the parties and those in privity with them and constitutes an absolute bar to subsequent actions involving
the same claim, demand or cause of action.

The requisites for res judicata under the concept of bar by prior judgment are:


(1) The former judgment or order must be final;

(2) It must be a judgment on the merits;

(3) It must have been rendered by a court having jurisdiction over the subject matter and the parties; and

(4) There must be between the first and second actions, identity of parties, subject matter, and
cause of action.

Res judicata is not present in this case.

While requisites one to three may be present, it is obvious that the there is no identity of subject matter,
parties and causes of action between the criminal case prosecuted under the Anti-Squatting Law and
the civil action for the recovery of the subject property.

First, there is no identity of parties. The criminal complaint, although initiated by the Spouses Supapo,
was prosecuted in the name of the people of the Philippines. The accion publiciana, on the other hand,
was filed by and in the name of the Spouses Supapo.

Second, there is no identity of subject matter. The criminal case involves the prosecution of a crime
under the Anti-Squatting Law while the accion publiciana is an action to recover possession of the
subject property.

And third, there is no identity of causes of action. The people of the Philippines filed the criminal case to
protect and preserve governmental interests by prosecuting persons who violated the statute. The Spouses
Supapo filed the accion publiciana to protect their proprietary interests over the subject property and
recover its possession.

DOCTRINE: Accion publiciana is an ordinary civil proceeding to determine the better right of
possession of realty independent of title. It refers to an ejectment suit filed after the expiration of one year
from the accrual of the cause of action or from the unlawful withholding of possession of the realty.

Remedial Law Review II (CY 2018-2019) – Saturday 1:00pm – 5:00pm: Case Digest Compilation [Atty. Brondial] Page 164
I.10. Dela Cruz vs. Hermano

G.R. No. 169914; March 25, 2015; Sereno, CJ.

Facts: On June 13, 2002, Respondent-spouses Antonio and Remedios Hermano sued petitioner before the
MTCC of Tagaytay City for ejectment and damages. In their Complaint, the Respondents alleged that:
They are the registered and lawful owner of a house and lot situated at Lot 2, Block 2, P.B. Constantino
Subd., Tagaytay City; that Petitioner Marcela Dela Cruz occupied and possessed the aforesaid house and
lot sometime on September 1, 2001 pursuant to the alleged Memorandum of Agreement between her and
a certain Don Mario Enciso Benitez, without the authority and consent of the plaintiff; and that On
September 27, 2001, plaintiff through counsel sent a formal demand letter to the defendant for the latter to
vacate and turn over the possession of the property and to pay the rental in the amount of P20,000.00 a
month starting September 1, 2001.

In her Answer with Counterclaim, petitioner admitted the existence of the TCT, but she contended that
the true and actual owner of the property was Don Benitez. Allegedly, the respondents had already sold
the property to Benitez; the latter, in turn, sold it to petitioner by virtue of a Deed of Absolute Sale
executed on 1 March 2001. Petitioner claimed that Antonio knew about the sale and her immediate
occupation of the premises. Moreover, Antonio allegedly knew that her caretakers had been managing the
property since March 2001, and that he never questioned their presence there. Thus, petitioner contended
that estoppel had set in, as he had made her believe that she had the right to occupy and possess the
property.
In 2002, the MTCC rendered a Decision dismissing the case for lack of jurisdiction over the subject
matter of the Complaint. The MTCC found that Antonio still owned the property when Benitez delivered
it to petitioner; Antonio’s proper remedy was an action for recovery, instead of the summary proceeding
of ejectment, because there was no showing of forcible entry or unlawful detainer.

In 2003, the respondents appealed to the RTC, which rendered a Decision affirming en toto the Decision
of the MTCC. The RTC opined that respondents’ Complaint did not clearly show whether it was one for
forcible entry or for unlawful detainer.

Respondents then filed a Petition for Review with the CA. The appellate court rendered a Decision
reversing the RTC Decision declaring Antonio as the lawful possessor of the subject lot, and ordering that
petitioner surrender the possession of the said lot to Antonio. The reversal by the CA of the rulings of the
courts a quo was based on its finding that the case was an ejectment complaint for forcible entry, and that
Antonio had sufficiently alleged and proved prior physical possession, as well as petitioner’s entry and
possession by stealth. Further, the CA found that the case was filed within the one-year time bar for an
ejectment suit, as Antonio came to know of petitioner’s possession only on 1 September 2001

Issue: Whether Respondent Antonio has adequately pleaded and proved a case of forcible entry/ whether
the complaint was timely filed.

RULING: Yes. The Complaint’s allegations sufficiently established the jurisdictional facts required in
forcible entry cases.

Section 1, Rule 70 of the Rules of Court, requires that in actions for forcible entry, it must be alleged that
the complainant was deprived of the possession of any land or building by force, intimidation, threat,
strategy, or stealth, and that the action was filed anytime within one year from the time the unlawful
deprivation of possession took place. This requirement implies that in those cases, possession of the land
by the defendant has been unlawful from the beginning, as the possession was obtained by unlawful
means. Further, the complainant must allege and prove prior physical possession of the property in
litigation until he or she was deprived thereof by the defendant. The one-year period within which to
bring an action for forcible entry is generally counted from the date of actual entry into the land, except
when entry was made through stealth; if so, the one-year period would be counted from the time the
plaintiff learned about it.

The Complaint was timely filed. It is settled that where forcible entry occurred clandestinely, the one-year
prescriptive period should be counted from the time the person who was deprived of possession
demanded that the deforciant desist from dispossession when the former learned about it. The owners or
possessors of the land cannot be expected to enforce their right to its possession against the illegal
occupant and sue the latter before learning of the clandestine intrusion. And to deprive lawful possessors
of the benefit of the summary action under Rule 70 of the Revised Rules, simply because the stealthy

Remedial Law Review II (CY 2018-2019) – Saturday 1:00pm – 5:00pm: Case Digest Compilation [Atty. Brondial] Page 165
intruder managed to conceal the trespass for more than a year, would be to reward clandestine usurpations
even if they are unlawful.

Issue 2: Whether Respondents proved prior possession of the lot.

RULING: No. To prove their claim of having a better right to possession, respondents submitted their
title thereto and the latest Tax Declaration prior to the initiation of the ejectment suit.
Ownership certainly carries the right of possession, but the possession contemplated is not exactly the
same as that which is in issue in a forcible entry case. Possession in a forcible entry suit refers only to
possession de facto, or actual or material possession, and not one flowing out of ownership. These are
different legal concepts under which the law provides different remedies for recovery of possession.
Thus, in a forcible entry case, a party who can prove prior possession can recover the possession even
against the owner. Whatever may be the character of the possession, the present occupant of the property
has the security to remain on that property if the occupant has the advantage of precedence in time and
until a person with a better right lawfully causes eviction.

The Court finds that the proofs submitted by respondents only established possession flowing from
ownership. Although respondents have claimed from the inception of the controversy up to now that they
are using the property as their vacation house, that claim is not substantiated by any corroborative
evidence. On the other hand, petitioner’s claim that she started occupying the property in March 2001,
and not in September of that year as Antonio alleged in his Complaint, was corroborated by the
Affidavit of petitioner’s caretaker. Respondents did not present any evidence to controvert that affidavit.
Therefore, respondents failed to discharge their burden of proving the element of prior physical
possession. Their uncorroborated claim of that fact, even if made under oath, is self-serving.

Remedial Law Review II (CY 2018-2019) – Saturday 1:00pm – 5:00pm: Case Digest Compilation [Atty. Brondial] Page 166
I.11. Sps. Herminio E. Erorita and Editha C. Erorita vs. Sps. Ligaya Dumlao and Antonio Dumlao

G.R.No. 1945477; J. Brion

FACTS: Sps. Dumlao are the registered owner of a parcel land located in Oriental Mindoro covered by a
TCT. Sps.Dumlao bought the property in an extrajudicial foreclosure sale because the former owners,
Sps. Erorita failed to redeem it, the title was consolidated in the buyers’ name.

The San Mariano Academy structures are built on the property. However even after the TCT was
transferred the Sps. Dumlao agreed to allow the petitioners to continue to operate the school on the
property for a consideration that Sps. Erorita will pay a monthly rent of Php20,000 but had failed to pay
rentals since 1990. On 2002, the Spouses Dumlao asked the Sps.Erorita to vacate the property. Although
the Sps. Erorita wanted to comply, they could not immediately close the school without clearance from
the Department of Education, Culture, and Sports to whom they are accountable.

As a result of Sps. Dumlao filed a complaint for recovery of possession before the RTC. Sps. Erorita in its
answer prayed that complaint be dismissed under their factual circumstance. During the pre-trial Sps.
Erorita failed to appear, thus was declared in default and ordered Sps. Dumlao to present evidence ex
parte. The RTC decided in favor of Sps. Dumlao, the court ordered Sps. Erorita to vacate the premises
and pay accumulated rentals and damages.

On appeal, Sps. Erorita alleged that RTC has no jurisdiction over the subject matter but the CA affirmed
the decision of RTC. It held that the law on jurisdiction when the complaint was filed, was Republic Act
No. 7691 it provides that in civil actions involving a real property’s title or possession, jurisdiction
depends on the property’s assessed value and location, if the assessed value pesos Php50,000 in Metro
Manila, and Php20,000 outside of Metro Manila, the RTC has jurisdiction. If the assessed value does not
exceed these amounts, then, the MTC has jurisdiction.

ISSUE: Whether or not RTC has jurisdiction over the case?


Whether or not Sps. Erorita timely raised the issue of jurisdiction?

RULING:
1. NO, RTC has no jurisdiction. The jurisdiction and nature of an action is determined on the
allegations in the complaint, it does not depend on the complaint’s caption nor defenses in the
answer.

To make a case for unlawful detainer, the complaint must allege that:
A. initially, the defendant lawfully possessed the property, either by contract or by
plaintiff’s tolerance;
B. the plaintiff notified the defendant that his right of possession is terminated;
C. the defendant remained in possession and deprived plaintiff of its enjoyment; and
D. the plaintiff filed a complaint within one year from the last demand on defendant to
vacate the property.

A complaint for accion publiciana or recovery of possession of real property will not be


considered as an action for unlawful detainer if any of these special jurisdictional facts is omitted.

In the case-at-bar, the allegation shows that there was agreement between Sps. Dumlao and Sps.
Erorita for the latter to occupy the property; there was demand to vacate but Sps. Erorita refused
to vacate. The allegations made in the complaint clearly show elements of unlawful detainer.
Thus, although the complaint bears the caption "recovery of possession," its allegations contain
the jurisdictional facts for an unlawful detainer case. Under RA 7691, an action for unlawful
detainer is within the MTC’s exclusive jurisdiction regardless of the property’s assessed value.

2. YES, As a general rule, lack of jurisdiction over the subject matter may be raised at any time, or
even for the first time on appeal. An exception to this rule is the principle of estoppel by laches.

In the present case, the applicable rule is laid down in Figueroa vs. People, the court ruled that the
failure to assail jurisdiction during trial is not sufficient for estoppel by laches to apply.
When lack of jurisdiction is raised before the appellate court, no considerable length of time
had elapsed for laches to apply. Laches refers to the "negligence or omission to assert a right

Remedial Law Review II (CY 2018-2019) – Saturday 1:00pm – 5:00pm: Case Digest Compilation [Atty. Brondial] Page 167
within a reasonable length of time, warranting a presumption that the party entitled to assert it
either has abandoned it or declined to assert it."

The Tijam vs. Sibonghanoy will not apply in this case where in it was held that lack of
jurisdiction was raised for the first time after almost fifteen (15) years after the questioned ruling
had been rendered and after the movant actively participated in several stages of the proceedings.
It was only invoked, too, after the CA rendered a decision adverse to the movant.

DOCTRINE:Although the complaint bears the caption "recovery of possession," but its allegations
contain the jurisdictional facts for an unlawful detainer case. Under RA 7691, an action for unlawful
detainer is within theMTC’s exclusive jurisdiction regardless of the property’s assessed value.

Issue of lack of jurisdiction as a general rule may be raised anytime or even for the first time on appeal.
An exception to this rule is the principle of estoppel by laches.

Remedial Law Review II (CY 2018-2019) – Saturday 1:00pm – 5:00pm: Case Digest Compilation [Atty. Brondial] Page 168
J. Contempt (Rule 71)

J.1. SECURITIES AND EXCHANGE COMMISSION CHAIRMAN PERFECTO R. YASAY, JR.,


ASSOCIATECOMMISSIONERS FE ELOISA C. GLORIA, EDIJER MARTINEZ and
ROSALINDA U. CASIGURAN, vs. MANUEL D. RECTO, PELAGIO T. RICALDE and CESAR
P. MANALAYSAY

Facts: SEC issued a temporary restraining order (TRO) enjoining the Interport Resources Corporation
from holding the July 9, 1996 scheduled annual meeting of the stockholders.
Notwithstanding the SEC’s TRO, the stockholders proceeded with the meeting on July 9, 1996, presided
over by respondent Manalaysay in light of the TRO issued by the Court of Appeals restraining the SEC
from enforcing its TRO.
The SEC declared the stockholders meeting of Interport Resources Corporation held, null and void and
directed respondents to appear before the SEC to show cause why no disciplinary action should be taken
against them or why they should not be cited in contempt. SEC declared the respondents in contempt.
Respondents appealed to the Court of Appeals which reversed the order of the SEC. SEC appealed to the
Supreme Court. Respondents argue that contempt is criminal in character and their exoneration from a
charge of contempt amounts to an acquittal from which an appeal would not lie.

ISSUE: Whether or not a judgment of acquittal in a case for contempt appealable?

RULING: The charge of contempt partakes of the nature of a criminal offense.9 The exoneration of the
contemner from the charge amounts to an acquittal from which an appeal would not lie.
“A distinction is made between a civil and criminal contempt. Civil contempt is the failure to do
something ordered by a court to be done for the benefit of a party. A criminal contempt is any conduct
directed against the authority or dignity of the court.
Petitioners argue that the contempt committed by respondents was civil in nature, as the temporary
restraining order the SEC issued was for the benefit of a party to a case. The contention is untenable.
“Civil contempt proceedings are generally held to be remedial and civil in their nature; that is, they are
proceedings for the enforcement of some duty, and essentially a remedy for coercing a person to do the
thing required.”
“In general, civil contempt proceedings should be instituted by an aggrieved party, or his successor, or
someone who has a pecuniary interest in the right to be protected.” If the contempt is initiated by the
court or tribunal exercising the power to punish a given contempt, it is criminal in nature, and the
proceedings are to be conducted in accordance with the principles and rules applicable to criminal cases.
The State is the real prosecutor.
“The real character of the proceedings in contempt cases is to be determined by the relief sought or by the
dominant purpose. The proceedings are to be regarded as criminal when the purpose is primarily
punishment, and civil when the purpose is primarily compensatory or remedial.”
“But whether the first or the second, contempt is still a criminal proceeding in which acquittal, for
instance, is a bar to a second prosecution. The distinction is for the purpose only of determining the
character of punishment to be administered.

Remedial Law Review II (CY 2018-2019) – Saturday 1:00pm – 5:00pm: Case Digest Compilation [Atty. Brondial] Page 169
J.2. SALVADOR SISON vs. JUDGE JOSE F. CAOIBES, JR., Presiding Judge,and TEODORO S.
ALVAREZ, Sheriff IV, Regional Trial Court, Las Piñas City, Branch 253

A.M. No. RTJ-03-1771; May 27, 2004

FACTS: On September 29, 1999, Sison, an MMDA officer apprehended Caoibes III for violating a no
right turn sign while the latter was driving along EDSA. Caoibes III introduce himself to be the son of
RTC Judge Caoibes Jr. and plead that he had an official errand for his father. Notwithstanding such
introduction and plea, Sison confiscated the driver’s license of Caoibies III, even bragging in the process
that he did the same to somebody who introduced himself as a lawyer the day before.

On September 15, 1999, Judge Caoibes Jr. issued an order requiring Sison to appear before him to explain
a traffic incident involving his son and to show cause why he should not be cited as in contempt of court.
Because of the complainant’s failure to appear as directed, Judge Caoibes Jr., issued another Order dated
September 22, 1999 for the complainant’s arrest and commitment, and for the latter to personally appear
for hearing before his sala on September 29, 1999. On the scheduled hearing, the complainant appeared
and executed an affidavit admitting to the court that he made a mistake and that it was all a
misunderstanding. The respondent judge, thereafter, lifted the September 22, 1999 Order.

Sison filed an administrative case against Judge Caoibes, Jr., and alleged the following: (1) that former
was charged for indirect contempt for apprehending the latter’s son for traffic violation; (2) that former
was arrested and was only released upon admitting his mistakes in relation to the apprehension under
duress; and (3) that the acts of respondents in arresting him without any warrant of arrest before a charge
of indirect contempt is heard constitute the gravest ABUSE OF AUTHORITY ever committed by the
respondents.

ISSUE: Whether or not the act of an MMDA officer in apprehending a person, who while not an official
employee of the judiciary was allegedly performing official function for his father who is a judge, is
contumacious act under Section 3, Rule 71.

RULING: The answer is in the negative. The act complained against must be any of those specified in
Sec. 3, Rule 71, 1997 Rules of Civil Procedure; otherwise, there is no contempt of court, which
requires that the person obstructed should be performing a duty connected with judicial functions.

The respondent Judge was not justified to so consider the act and remarks of Sison as thereby displaying
arrogance towards and deliberate disregard of the usual respect, courtesy and accommodation due to a
court of law and its representative.

The refusal of Sison and the supposed remarks should not cause resentment on the part of the respondent
Judge (whom Sison most likely did not yet know at the time) because he knew, as a public official
himself, that Sison was only doing his duty of enforcing evenly the particular traffic regulation against
swerving into a one-way street from the wrong direction, regardless of the office or position of the
violator’s father.

The foregoing renders clear that the respondent Judge had no legitimate basis by which to consider
Sison’s apprehension of his son as indirect contempt.

Remedial Law Review II (CY 2018-2019) – Saturday 1:00pm – 5:00pm: Case Digest Compilation [Atty. Brondial] Page 170
J.3.JUDGE DOLORES L. ESPAÑOL vs. ATTY. BENJAMIN S. FORMOSO and SPOUSES
BENITO SEE and MARLY SEE

G.R. No. 150949; June 21, 2007; Sandoval-Gutierrez, J.

FACTS: On April 15, 1994, Sharcons Builders Philippines, Inc. (Sharcons) bought from Evanswinda
Morales a piece of land purported to be registered under the latter’s name.

Thus, Title in Evanswinda’s name was cancelled and in lieu thereof, TCT No. T-511462 was issued in the
name of Sharcons. However, when the latter’s workers tried to fence and take possession of the lot, they
were prevented by the caretaker of spouses Joseph and Enriqueta Mapua. The caretaker claimed that
spouses Mapua are the owners of the land. Sharcons verified the status of the title and found that TCT No.
T-107163 was indeed registered in the names of spouses Mapua as early as July 13, 1979.

On January 25, 2000, Sharcons filed a complaint for quieting of title and impleaded as defendants spouses
Mapua, Evanswinda Morales, and the Register of Deeds of Trece Martires City. In the course of the
proceedings, Judge Dolores L. Español came across Civil Case No. 623-92 wherein it was adjudged that
Sharcons' TCT and other supporting documents are falsified and that respondents are responsible therefor.
Hence, Judge Español issued an Order declaring respondents guilty of direct contempt of court and
ordered their confinement for ten (10) days in the municipal jail of Dasmariñas, Cavite.

By virtue of a warrant of arrest issued by petitioner, respondents were confined in municipal jail. That
same day, respondents filed a motion for bail and a motion to lift the order of arrest. But they were denied
outright by petitioner. Respondents then filed with the Court of Appeals a petition for a writ of habeas
corpus which was granted.

ISSUE: Whether or not the act of presenting falsified documents before a court constitutes direct
contempt.

RULING: The answer is in the negative. The use of falsified and forged documents is a contumacious
act. However, it constitutes indirect contempt not direct contempt. Pursuant to Section 3, Rule 71, such
act is an improper conduct which degrades the administration of justice. In Santos v. Court of First
Instance of Cebu, Branch VI, we ruled that the imputed use of a falsified document, more so where the
falsity of the document is not apparent on its face, merely constitutes indirect contempt, and as such is
subject to such defenses as the accused may raise in the proper proceedings. Thus, following Section 3,
Rule 71, a contemner may be punished only after a charge in writing has been filed, and an opportunity
has been given to the accused to be heard by himself and counsel. Moreover, settled is the rule that a
contempt proceeding is not a civil action, but a separate proceeding of a criminal nature in which the
court exercises limited jurisdiction. Thus, the modes of procedure and the rules of evidence in contempt
proceedings are assimilated as far as practicable to those adapted to criminal prosecutions. Perforce,
petitioner judge erred in declaring summarily that respondents are guilty of direct contempt and ordering
their incarceration. She should have conducted a hearing with notice to respondent

Remedial Law Review II (CY 2018-2019) – Saturday 1:00pm – 5:00pm: Case Digest Compilation [Atty. Brondial] Page 171
J.4. P/SUPT. Marantan vs. Atty. Diokno

FACTS: This case stemmed from a criminal case entitled “People vs. P/SINSP Marantan, et. al” pending
before the RTC. Marantan and his co-accused were charged with homicide (where persons [including
Anton Cu-Unjieng (son of respondent La’O)] were shot and killed by police in front of AIC Gold Tower
at Ortigas Center which was captured by a TV crew from UNTV). In the meantime, another shooting
incident transpired which also involved Marantan. This allegedly resulted in much negative publicity on
the part of Marantan. Marantan filed a petition before SC to cite La’o (the private complainant in the
homicide charge) and Atty. Diokno (La’o’s counsel therein) in contempt, [contending that the
respondents violated the sub judice rule, making them liable for indirect contempt for their contemptuous
statements and improper conduct tending directly or indirectly to impede, obstruct or degrade the
administration of justice. He argues that their pronouncements and malicious comments delved not only
on the supposed inaction of the Court in resolving the petitions filed, but also on the merits of the criminal
cases before the RTC and prematurely concluded that he and his co-accused are guilty of murder.

It is Maranta’s position that the press conference was organized by the respondents for the sole purpose of
influencing the decision of the Court in the petition nd the Atimonan incident.] La’o and Diokno filed
their comment [arguing that there was no violation of the sub judice rule as their statements were
legitimate expressions of their desires, hopes and opinions which were taken out of context and did not
actually impede, obstruct or degrade the administration of justice in a concrete way; that no criminal
intent was shown as the utterances were not on their face actionable being a fair comment of a matter of
public interest and concern; and that this petition is intended to stifle legitimate speech.

ISSUE: Whether there was a violation of the sub judice rule on the part of La’o and Atty. Diokno so as to
merit their being cited in contempt?

RULING: No. sub judice rule restricts comments and disclosures pertaining to the judicial proceedings in
order to avoid prejudging the issue, influencing the court, or obstructing the administration of justice. A
violation of this rule may render one liable for indirect contempt under Sec. 3(d), Rule 71 of the Rules of
Court, which reads: Section 3. Indirect contempt to be punished after charge and hearing. – x x x a person
guilty of any of the following acts may be punished for indirect contempt: (d) Any improper conduct
tending, directly or indirectly, to impede, obstruct, or degrade the administration of justice.

The proceedings for punishment of indirect contempt are criminal in nature. This form of contempt is
conduct that is directed against the dignity and authority of the court or a judge acting judicially; it is an
act obstructing the administration of justice which tends to bring the court into disrepute or disrespect.
Intent is a necessary element in criminal contempt, and no one can be punished for a criminal contempt
unless the evidence makes it clear that he intended to commit it. For a comment to be considered as
contempt of court "it must really appear" that such does impede, interfere with and embarrass the
administration of justice. As to the conduct of the Court, a review of the respondents' comments reveals
that they were simply stating that it had not yet resolved their petition. There was no complaint, express
or implied, that an inordinate amount of time had passed since the petition was filed without any action
from the Court. There appears no attack or insult on the dignity of the Court either [wala namang reklamo
na matagal desisyunan ung case, no attack/insult also, no criminal intent to impede, obstruct or degrade
the administration of justice can be inferred from the comments of the respondents]. The power to punish
for contempt, being drastic and extraordinary in its nature, should not be resorted to unless necessary in
the interest of justice. In the present case, such necessity is wanting.

Remedial Law Review II (CY 2018-2019) – Saturday 1:00pm – 5:00pm: Case Digest Compilation [Atty. Brondial] Page 172
J.5. JILL M. TORMIS,vs. JUDGE MEINRADO P. PAREDES 

A.M. No. RTJ-13-2366 [Formerly OCA IPI No. 11-3740-RTJ]; February 04, 2015; SECOND
DIVISIONMENDOZA, J.:

FACTS: Jill charged Judge Paredes with grave misconduct.  Jill was a student of Judge Paredes in
Political Law Review during the first semester of school year 2010-2011 at the Southwestern University,
Cebu City.  She averred that sometime in his class discussions, Judge Paredes named her mother, Judge
RosabellaTormis (Judge Tormis as one of the judges involved in the marriage scams in Cebu City.  Judge
Paredes also mentioned in his class that Judge Tormis was abusive of her position as a judge, corrupt, and
ignorant of the law.Judge Paredes was even said to have included in his discussion Jill’s brother, stating
that he was a “court-noted addict.” To avoid humiliation in school, Jill decided to drop the class under
Judge Paredes and transfer to another law school in Tacloban City.

In her memorandum, Jill contended that Judge Paredes’ act of discussing Judge Tormis’ cases in class
where she was present was an open display of insensitivity, impropriety and lack of delicadeza bordering
on oppressive and abusive conduct, which fell short of the exacting standards of behavior demanded of
magistrates. 

Jill also averred that Judge Paredes violated the subjudice rule when he discussed the marriage scam
involving Judge Tormis in 2010 because at that time, the case was still being investigated.

ISSUES: Whether or not Judge Paredes is guilty of misconduct.

RULING: No. Misconduct is defined as a transgression of some established and definite rule of action,
more particularly, unlawful behavior or gross negligence by a public officer.  The misconduct is grave if
it involves any of the additional elements of corruption, willful intent to violate the law, or to disregard
established rules, which must be established by substantial evidence. As distinguished from simple
misconduct, the elements of corruption, clear intent to violate the law, or flagrant disregard of established
rule, must be manifest in a charge of grave misconduct. Corruption, as an element of grave misconduct,
consists in the act of an official or fiduciary person who unlawfully and wrongfully uses his station or
character to procure some benefit for himself or for another person, contrary to duty and the rights of
others.

To constitute misconduct, the act or acts must have a direct relation to and be connected with the
performance of his official duties. Considering that the acts complained of, the remarks against Judge
Tormis and Francis, were made by Judge Paredes in his class discussions, they cannot be considered as
“misconduct.”  They are simply not related to the discharge of his official functions as a judge.  Thus,
Judge Paredes cannot be held liable for misconduct, much less for grave misconduct.

Remedial Law Review II (CY 2018-2019) – Saturday 1:00pm – 5:00pm: Case Digest Compilation [Atty. Brondial] Page 173
J.6. NEMENCIO C. PULUMBARIT, SR., Petitioner, v. THE COURT OF APPEALS (17th Division
Composed of JUSTICE BIENVENIDO L. REYES, PONENTE; JUSTICE ROBERTO A.
BARRIOS, Chairman; AND JUSTICE EDGARDO F. SUNDIAM, Acting Third Member),
LOURDES S. PASCUAL, LEONILA F. ACASIO, AND SAN JUAN MACIAS MEMORIAL
PARK, INC., Respondents. 

G.R. NOS. 153745-46, October 14, 2015

FACTS: San Juan Macias Memorial Park, Inc. (SJMMPI), through its President Lourdes S. Pascual,
authorized Atty. Soledad de Jesus to look for a buyer for the San Juan Memorial Park. They were
introduced to Pulumbarit. The parties eventually came to an agreement, with Pulumbarit issuing eighteen
(18) checks in the name of SJMMPI Secretary-Treasurer Leonila Acasio.

For failure to provide the written copy of the contract as well as the re-issuance of the check, Pascual filed
a Complaint for Rescission of Contract, Damages and Accounting with Prayer for Preliminary Injunction
or Receivership. The Complaint alleged that they entered into a contract of management with option to
buy the Memorial Park with Pulumbarit, with the latter allegedly agreeing to pay Pascual a sum of
P750,000.00 on staggered installments.
Pulumbarit filed a Motion praying for the dismissal of the Complaint for lack of cause of action, attaching
a copy of the Memorandum of Agreement (MOA). Pending the Motion, trial court declared Pulumbarit in
default but this was reversed by the CA. Thus, trial court admitted Pulumbarit’s answer. Pascual
presented evidence to prove that Pulumbarit falsified the MOA.
Trial court decided in favor of Pascual and declared that the MOA is null and void.
Pulumbarit filed a notice of appeal (CA-G.R. CV No. 69931.)
Before transmittal of the records, Pascual filed with the trial court motions praying for (1) the issuance of
a writ of injunction against Pulumbaritand (2) the execution of the decision pending appeal. The trial
court granted these motions on Pulumbarit's subsequent motion for reconsideration 3of this Order
(directing discretionary execution) was denied.
Pulumbarit filed a Petition for Certiorari with the CA to nullify the writs of execution and injunction
issued by the trial court. (CA-G.R. SP No. 61873)
The CA (G.R. SP No. 61873) issued a TRO and thereafter a writ of preliminary injunction.
Despite the issuance of TRO, Pascual filed a motion in CA-G.R. CV No. 69931 seeking execution of the
trial court's Decision pending Pulumbarit's appeal.
CA granted Pascual's motion for execution pending appeal and, as a consequence, dismissed CA-G.R. SP
No. 61873 for being moot and academic. However, upon Petition for Review on Certiorari under Rule
45, the CA resolved to suspend its Resolution granting Pascual's motion for execution pending appeal.
CA issued its Decision reversing the trial court's ruling in Civil Case No. 7250-M. Pascual et al.'s motion
for reconsideration was denied by the CA. Aggrieved, Pascual et al. filed a petition seeking the review of
this Decision.

ISSUE: 1) WON Pascual committed abuse of Court processes.


2) Whether Pascual committed forum shopping

RULING:
1.) Yes. Here, the CA in CA-G.R. SP No. 61873 issued the TRO and the writ of preliminary injunction
against the discretionary execution. Pulumbarit posted the required bond amounting to P500,000.00.
Pascual on the other hand, filed their motion for execution pending appeal in CA-G.R. CV No. 69931,
nearly four months after the issuance of the TRO, two months after the writ of injunction and almost a
month from Pulumbarit's posting of the bond.

Said motion is clearly an attempt on Pascual et al.'s part to undermine the TRO and writ of preliminary
injunction earlier issued in CA-G.R. SP No. 61873 in Pulumbarit's favor. (Notably, Pascual et al. do not
appear to have sought the reconsideration of the issuance of said injunctive orders.) Pascual's filing of the
motion in CA-G.R. CV No. 69931 demonstrates defiance of, if not lack of due respect for, the authority
of the CA which earlier issued injunctive writs against the execution by the trial court of the
appealed Decision.

2) No.
Pascual et al. did not commit forum shopping. Forum shopping exists when the elements of litis
pendentia are present, or when a final judgment in one case will amount to res judicatain another. Here,
any action by the CA on Pascual et al.'s motion in CA-G.R. CV No. 69931 is provisional in nature, such

Remedial Law Review II (CY 2018-2019) – Saturday 1:00pm – 5:00pm: Case Digest Compilation [Atty. Brondial] Page 174
that it can in no way constitute as res judicata in CA-G.R. SP No. 61873. Moreover, forum shopping
requires the identity of parties, rights or causes of action, and reliefs sought in two or more pending
cases. Here, there is no identity of relief and/or cause of action. CA-G.R. SP No. 61873 is limited to a
determination of whether grave abuse of discretion was committed by the trial court in granting execution
pending appeal while Pascual et al.'s motion in CA-G.R. CV No. 69931 involves a determination by the
CA whether there are "good reasons" warranting the grant of discretionary execution.

Remedial Law Review II (CY 2018-2019) – Saturday 1:00pm – 5:00pm: Case Digest Compilation [Atty. Brondial] Page 175
J.7. MAYOR ANWAR BERUA BALINDONG, LT. COL. JALANDONI COTA, MAYOR AMER
ODEN BALINDONG, AND ALI BALINDONG, Petitioners, v.COURT OF APPEALS, STATE
PROSECUTOR LEAH ARMAMENTO, OFFICE OF THE SOLICITOR GENERAL AND
ZENAIDA LIMBONA, Respondents.

G.R. No. 177600, October 19, 2015

ZENAIDA M. LIMBONA, Petitioner, v.HON. JUDGE ALEXANDER S. BALUT OF THE


REGIONAL TRIAL COURT OF QUEZON CITY, BRANCH 76, Respondent.

G.R. No. 178684;BERSAMIN, J.:

FACTS:The instant petition was instigated by a shooting incident that took place in Lanao del Sur that
resulted in the death of Dante Limbona and Ante Maguindanao. In the course of the preliminary
investigation the investigating prosecutor found probable cause to charge private respondents Lt. Col.
Jalandoni D. Cota, Anwar Berua Balindong, PO1 Kennedy Balindong, Amer Oden Balindong and Ali
Sarip Balindong with Double Murder with Multiple Frustrated Murder.

Motions were filed which resulted to downgrading the crime to 2 information for frustrated murder. CA
granted the certiorari and ordered RTC to change the information to murder with attempted murder and
frustrated murder with attempted murder. Warrant of arrest was issued. SC denied the certiorari
subsequently filed. There were subsequent inhibition of judges. When Judge Lee handled the case, he
granted the motion to determine the existence of probable cause and again to downgrade the crime. After
granting the motion, Judge Lee inhibited himself. The case was transferred to Judge Genilo who
reinstated the original crime and ordered the issuance of alias warrants of arrest. She inhibited and the
case was transferred to Judge Balut who deferred action on the Motion for Reconsideration and/or Recall
Suspend Order of Arrest of Balindong, et al., and suspended the enforcement of the alias warrants of
arrest. Hence, Limbona commenced G.R. No. 178684.

Limbona raises as the sole ground for her petition to cite in contempt of court Judge Balut and Balindong,
et al. that:

Petitioner respectfully submits the foregoing acts of Respondent in willfully disobeying the
decision and resolutions issued by the Hon. Supreme Court in G.R. No. 159962 and G.R.
173290, which tend to impede upon or obstruct the administration of justice, constitutes an
indirect contempt which ought to be punished.

ISSUE: Whether or not Judge Balut can be cited in contempt?

RULING: No. While awaiting resolution of the said petition for review assailing the validtity of issuance
of the original warrants, Judge Balut has justified his actions by invoking judicial courtesy and asserting
his judicial discretion.

Judge Balut did not thereby disobey the decisions of the Court in G.R. No. 159962 and G.R. No. 173290.
To start with, there was no indication in his Order that bad faith had moved him to suspend the
implementation of the warrants of arrest against Balindong, et al., or that he had thereby acted with a
willful and deliberate intent to disobey or to ignore the Court's bidding, or to cause injustice to any of the
parties. In the absence of the clear showing of bad faith on his part, his being prudent could only be an
error of judgment, for which he could not be held to account.

DOCTRINE: The power of the courts to punish for contempt is to be exercised cautiously, sparingly,
and judiciously.36 Self-restraint in wielding contempt powers should be the rule unless the act complained
of is clearly contumacious. An act, to be contumacious, must manifest willfulness, bad faith, or deliberate
intent to cause injustice.

Remedial Law Review II (CY 2018-2019) – Saturday 1:00pm – 5:00pm: Case Digest Compilation [Atty. Brondial] Page 176
J.8. SSSSBRO. BERNARD OCA v. LAURITA CUSTODIO

GR No. 199825; Jul 26, 2017; LEONEN, J.:

FACTS: This indirect contempt case stemmed from an intra-corporate controversy among the Board of
Trustees of petitioner St. Francis School of General Trias, Cavite, Inc. (St. Francis School). St. Francis
School was established with the assistance of the La Salle brothers without a written agreement. On
September 8, 1988, the incorporators and the La Salle brothers formalized their arrangement in a
Memorandum of Agreement, under which De La Salle Greenhills (La Salle) would supervise the
academic affairs of St. Francis School to increase enrollment. La Salle appointed supervisors to sit in the
Board of Trustees without voting rights.

In 1998, petitioner Bro. Bernard Oca (Bro. Oca) became a member of St. Francis School as a La Salle-
appointed supervisor. He sat in the Board of Trustees and was later elected as its Chairman and St.
Francis School's President. In 2000, petitioner Bro. Dennis Magbanua (Bro. Magbanua) was also admitted
as a La Salle-appointed supervisor. He sat as a trustee and was later elected as Treasurer of St. Francis
School.

Sometime in August 2001, the members of the Board of Trustees came into a disagreement regarding the
school's administrative structure and La Salle's supervision over the school. After several incidents
relating to the disagreement, Custodio, a Curriculum Administrator and a member of the Board of
Trustees , filed a Motion for Clarification among others, praying that the trial court clarify to whom the
school's fees should be paid. Custodio manifested that prior to October 8, 2002, the school cashier was
Ms. Herminia Reynante (Reynante).

On October 21, 2002, the Regional Trial Court issued an Order designating Reynante to act as school
cashier "with authority to collect all fees" and, together with Custodio, "to pay all accounts.” The trial
court also directed all parties in the case to submit a report on and to turn over to Reynante all money
previously collected. Plaintiff and defendants, as well as Mr. Al Mojica, are directed to turn-over to Ms.
Herminia Reynante all money previously collected and to submit a report on what have been collected,
how much, from whom, and the dates collected. Effective October 22, 2002, Ms. Herminia Reynante
shall submit to the Court, to the plaintiff and to all the defendants made.

On March 24, 2003, the Regional Trial Court issued another Order directing petitioners to fully comply
with its earlier order to submit a report and to turn over to Reynante all the money they had collected:
Still, petitioners refused to comply.

On August 21, 2003, the trial court granted Custodio's Manifestation and Motion dated October 9, 2002.
It issued a status quo order allowing Custodio to discharge her functions as school director and
curriculum administrator because it found that petitioners had already established a new school.
However, petitioners still did not comply despite this Order. Instead, they filed their September 1, 2003
Motion for Clarification, raising questions on Custodio's use of the turned over money, Custodio's and
Reynante's bonds as guaranty to the money's exclusive use as teachers' retirement fund, and petitioners'
liability in case of Custodio's misuse of this amount.

On September 2, 2003, Custodio filed the Petition to Cite Respondents in Contempt of Court under Rule
71 of the Rules of Court. She likewise prayed that an order be issued reiterating the Orders dated October
21, 2002, March 24, 2003, and August 5, 2003.

The trial court responded to petitioners' Motion for Clarification dated September 1, 2003 and issued its
October 8, 2003 Order, agreeing that the retirement fund would be merely held in trust by Custodio and
Reynante. It also directed Custodio and Reynante to file a bond of P300,000.00 each. Again, it ordered
petitioners to comply with the mandate in its March 24, 2003 and August 5, 2003 Orders and directed
them to inform the court the total amount of the money deposited and reserved for teachers' retirement
and its bank account details. Nonetheless, petitioners still did not comply. Instead, they argued in the
contempt proceeding that the March 24, 2003 and August 5, 2003 Orders were unlawful and were being
questioned in G.R. No. 174996. They claimed that their availnient of legal remedies showed their good
faith.

Branch 90, Regional Trial Court, Dasmariñas, Cavite found petitioners guilty of indirect contempt for
failing to comply with the Orders dated October 21, 2002 and March 24, 2003 and ordered them to jointly
and severally pay a fine of P30,000.00.

Remedial Law Review II (CY 2018-2019) – Saturday 1:00pm – 5:00pm: Case Digest Compilation [Atty. Brondial] Page 177
The Court of Appeals affirmed the trial court Decision. The Court of Appeals ruled that defying the trial
court orders amounted to contumacious conduct, which "tended to prejudice St. Francis School's
operations due to lack of operational funds."

ISSUE: Whether petitioners are guilty of indirect contempt.

RULING: This Court rules that petitioners Oca, Magbanua, Cirila, and Josefina are guilty of indirect
contempt. There is a contumacious refusal on their part to comply with the Regional Trial Court Orders.

Contempt of court is willful disobedience to the court and disregard or defiance of its authority, justice,
and dignity. It constitutes conduct which "tends to bring the authority of the court and the administration
of law into disrepute or in some manner to impede the due administration of justice" or "interfere with or
prejudice parties['] litigant or their witnesses during litigation." In Sison v. Caoibes, Jr.: Thus, the power
to declare a person in contempt of court and in dealing with him accordingly is an inherent power lodged
in courts of justice, to be used as a means to protect and preserve the dignity of the court, the solemnity of
the proceedings therein, and the administration of justice from callous misbehavior, offensive
personalities, and contumacious refusal to comply with court orders. Indeed, the power of contempt is
power assumed by a court or judge to coerce cooperation and punish disobedience, disrespect or
interference with the court's orderly process by exacting summary punishment. The contempt power was
given to the courts in trust for the public, by tradition and necessity, in as much as respect for the courts,
which are ordained to administer the laws which are necessary to the good order of society, is as
necessary as respect for the laws themselves.

There are two (2) types of contempt of court: (a) Direct contempt consists of "misbehavior in the
presence of or so near a court as to obstruct or interrupt the proceedings before [it]." It includes: (i)
disrespect to the court, (ii) offensive behavior against others, (iii) refusal, despite being lawfully required,
to be sworn in or to answer as a witness, or to subscribe an affidavit or deposition. It can be punished
summarily without a hearing. (b) Indirect contempt is committed through any of the acts enumerated
under Rule 71, Section 3 of the Rules of Court: (i)Misbehavior of an officer of a court in the performance
of his [or her] official duties or in his [or her] official transactions; (ii) Disobedience of or resistance to
a lawful writ, process, order, or judgment of a court, including the act of a person who, after being
dispossessed or ejected from any real property by the judgment or process of any court of competent
jurisdiction, enters or attempts or induces another to enter into or upon such real property, for the
purpose of executing acts of ownership or possession, or in any manner disturbs the possession given to
the person adjudged to be entitled thereto; (iii) Any abuse of or any unlawful interference with the
processes or proceedings of a court not constituting direct contempt under Section 1 of this Rule; (iv) Any
improper conduct tending, directly or indirectly, to impede, obstruct, or degrade the administration of
justice; (v) Assuming to be an attorney or an officer of a court, and acting as such without authority; (vi)
Failure to obey a subpoena duly served; (vii) The rescue, or attempted rescue, of a person or property in
the custody of an officer by virtue of an order or process of a court held by him [or her]. (viii) indirect
contempt is only punished after a written petition is filed and an opportunity to be heard is given to the
party charged.

In the case at bar, petitioners were charged with indirect contempt through "disobedience of or resistance
to a lawful writ, process, order, or judgment of a court." Plaintiff and defendants, as well as Mr. Al
Mojica, are directed to turn-over to Ms. Herminia Reynante alt money previously collected and to submit
a report on what have been collected, how much, from whom and the dates collected. Effective October
22, 2002, Ms. Herminia Reynante shall submit to the Court, to the plaintiff and to all the defendants a
monthly report of all receivables collected and all disbursements made. The trial court reiterated the
orders to turn over the amounts at least thrice. The wording of the Order is clear. However, despite its
clear wording, petitioners still did not comply. Petitioners' filing of numerous pleadings reveals their
contumacious refusal to comply and their abuse of court processes.

In intra-corporate controversies, all orders of the trial court are immediately executory: Section 4.
Executory nature of decisions and orders. — All decisions and orders issued under these Rules shall
immediately be executory except the awards for moral damages, exemplary damages and attorney's fees,
if any. No appeal or petition taken therefrom shall stay the enforcement or implementation of the decision
or order, unless restrained by an appellate court. Interlocutory orders shall not be subject to appeal.
Questioning the trial court orders does not stay its enforcement or implementation. There is no showing
that the trial court orders were restrained by the appellate court. Hence, petitioners could not refuse to
comply with the trial court orders just because they opined that they were invalid. It is not for the parties
to decide whether they should or should not comply with a court order. Petitioners did not obtain any

Remedial Law Review II (CY 2018-2019) – Saturday 1:00pm – 5:00pm: Case Digest Compilation [Atty. Brondial] Page 178
injunction to stop the implementation of the trial court orders nor was there an injunction to prevent the
trial court from hearing and ruling on the contempt case. Petitioners' stubborn refusal cannot be excused
just because they were convinced of its invalidity. Their resort to the processes of questioning the orders
does not show that they are in good faith.

This Court has acknowledged the trial court's power to cite parties in indirect contempt for their refusal to
follow its orders, although the validity of the orders is being questioned in another proceeding. The RTC
acted on the basis of the unjustified refusal of petitioners to abide by its lawful order. It is of no moment
that private respondents may have filed several pleadings to urge the RTC to cite petitioners in contempt.
Petitioners utterly violated an order issued by the trial court which act is considered contemptuous.

In this case, petitioners were given several opportunities to comply with the trial court orders. Even after
the trial court clarified which funds to turn over, they still refused to obey. While petitioners questioned
the legality of these orders, they are immediately executory. Moreover, the parties do not have the power
to determine for themselves what should and should not be excluded from the orders. Their failure to turn
over the amounts showed petitioners' defiance and disregard for the authority of the trial court.

Remedial Law Review II (CY 2018-2019) – Saturday 1:00pm – 5:00pm: Case Digest Compilation [Atty. Brondial] Page 179
III. Special Proceedings (Rules 72 – 109)

A. Settlement of Estate

A.1. RICARDO C. SILVERIO, SR., Petitioner, vs. RICARDO S. SILVERIO, JR., CITRINE


HOLDINGS, INC., MONICA P. OCAMPO and ZEE2 RESOURCES, INC.,Respondents.

G.R. Nos. 208828-29; August 13, 2014

FACTS:The late Beatriz S. Silverio died without leaving a will on October 7, 1987.  She was survived by
her legal heirs, namely: Ricardo C. Silverio, Sr. (husband), Edmundo S. Silverio (son), Edgardo S.
Silverio (son), Ricardo S. Silverio, Jr. (son), Nelia S. Silverio-Dee (daughter), and Ligaya S. Silverio
(daughter).  Subsequently, an intestate proceeding (SP PROC. NO. M-2629) for the settlement of her
estate was filed by SILVERIO, SR.

In the course of the proceedings, the parties filed different petitions and appeal challenging several orders
of the intestate court that went all the way up to the Supreme Court.

CA-G.R. SP No. 121172

The first petition of the three consolidated petitions is CA-G.R. SP No. 121172 wherein petitioner,
RICARDO S. SILVERIO JR. ("SILVERIO JR.") assails the Order of the intestate court dated 16 June
2011 reinstating RICARDO SILVERIO SR. ("SILVERIO SR.") as administrator to the... estate of the late
Beatriz Silverio. The administrator first appointed by the Court was EDGARDO SILVERIO
("EDGARDO"), but by virtue of a Joint Manifestation dated 3 November 1999 filed by the heirs of
BEATRIZ D. SILVERIO, the motion to withdraw as administrator filed by EDGARDO was approved by
the intestate court... and in his stead, SILVERIO SR. was appointed as the new administrator. Thereafter,
an active exchange of pleadings to remove and appoint a new administrator ensued between SILVERIO
SR. and SILVERIO JR.

CA-G.R. SP No. 121173

On 15 March 2011, heirs SILVERIO JR., EDMUNDO and LIGAYA represented by her legal guardian
moved for the disqualification and/or inhibition of JUDGE GUANLAO, JR. based on the following
grounds: (1) Absence of the written consent of all parties in interest allowing JUDGE GUANLAO,... JR.
to continue hearing the case considering that he appeared once as counsel in the intestate proceedings; (2)
JUDGE GUANLAO, JR. has shown bias and partiality in favor of SILVERIO SR. by allowing the latter
to pursue several motions and even issued a TRO in violation of the... rules against forum shopping; (3)
Heir LIGAYA's Petition for Support and Release of Funds for Medical Support has not been resolved;
and (4) It is in the best interest of all the heirs that the proceedings be presided and decided by the cold
neutrality of an impartial judge. On 23 March 2011, JUDGE GUANLAO, JR. issued an order denying the
Motion for Disqualification and/or Inhibition.

CA-G.R. SP NO. 122024

The intestate court in its Omnibus Order dated 31 October 2006, ordered  among others, the sale of
certain properties belonging to the estate. By virtue of the aforesaid Order, SILVERIO, JR. on 16 October
2007 executed a Deed of Absolute Sale in favor of CITRINE HOLDINGS, Inc. ("CITRINE") over the
property located at No. 3 Intsia Road, Forbes Park, Makati City. CITRINE became the registered owner
thereof on 06September 2010 as evidenced by TCT No. 006-201000063. A Deed of Absolute Sale was
likewise executed in favor of Monica P.  Ocampo (notarized on September 16, 2010) for the lot located at
No. 82 Cambridge Circle, Forbes Park, Makati City.  On 23 December 2010, TCT No. 006-2011000050
was issued to Monica P. Ocampo. The... latter subsequently sold said property to ZEE2 Resources, Inc.
(ZEE2) and TCT No. 006-2011000190 was issued on 11 February 2011 under its name.

On 04 February 2011 SILVERIO SR. filed an Urgent Application for the Issuance of Temporary
Restraining Order/Preliminary Prohibitory Injunction (With Motion For the Issuance of Subpoena Ad 
Testificandum and Subpoena Duces Tecum) praying among others, that aTRO be issued restraining
and/or preventing SILVERIO, JR., MONICA OCAMPO, CITRINE HOLDINGS, INC. and their
successors-in-interest from committing any act that would affect the titles to the three properties.

Remedial Law Review II (CY 2018-2019) – Saturday 1:00pm – 5:00pm: Case Digest Compilation [Atty. Brondial] Page 180
On 14 February 2011, SILVERIO SR. filed an Urgent Omnibus Motion (a) To Declare as Null and Void
the Deed of Absolute Sale dated 16 September 2010; (b) To cancel the Transfer Certificate of Title No.
006-2011000050; and  (c) To reinstate the Transfer Certificate of TitleNo. 2236121 in the name of
Ricardo C. Silverio Sr. and the Intestate Estate of the late Beatriz S. Silverio.

On 28 February 2011 the Intestate Court issued an Order granting a Temporary Restraining Order.

The consolidated petitions for certiorari  filed by respondent Ricardo S. Silverio, Jr. ("Silverio, Jr.")
before the CA questioned the following issuances of the intestate court: CA-G.R. SP No. 121172 Order
dated June 16, 2011 reinstating Silverio, Sr. asAdministrator;  CA-G.R. SP No. 121173 (1) Order dated
March 23, 2011 granting Silverio, Sr.'s application for preliminary injunction enjoining Silverio, Jr. or
anyone acting on their behalf from committing any act that would affect the titles to the subject...
properties and enjoining the Register of Deeds of Makati City from accepting, admitting, approving,
registering, annotating or in any way giving due course to whatever deeds, instruments or any other
documents involving the Cambridge and Intsia properties, (2) Order dated March23, 2011 which denied
Silverio, Jr.'s motion or disqualification and/or inhibition of Judge Guanlao, Jr., and (3) Order dated June
14, 2011 denying the motion for reconsideration of the March 23, 2011 Order (granting application for
preliminary injunction);  and inCA-G.R. SP No. 122024 Order dated August 18, 2011 declaring the Deed
of Absolute Sale, TCT and all derivative titles over the Cambridge and Intsia properties as null and void.

On March 8, 2013, the CA rendered its Decision, the fallo of which reads:
1.  The petition in CA G.R. SP No. 121172 is DENIED for lack of merit.
2.  The petition in CA GR. S.P. No. 121173 is partly DENIED for lack of merit insofar as it questions the
23 March 2011 Order denying RICARDO SILVERIO, JR's Motion for Disqualification and/or Inhibition
of Judge Honorio E. Guanlao, Jr.
3.  The petition in CA G.R.-S.P. No. 122024 is GRANTED.  Accordingly, the 18 August 2011 Order
declaring the Deed of Absolute Sale, Transfer Certificate of Title and all derivative titles over the
Cambridge and IntsiaProperty null and void is hereby
REVERSED and SET ASIDE.

ISSUES: The CA committed a reversible error in upholding the validity of the Intsia and Cambridge
properties upon the ground that the intestate court cannot annul the sales as it has a limited jurisdiction
only and which does not include resolving... issues of ownership.  It is asserted that the CA should not
have stopped there and looked into the nature of the properties sold, which formed part of the conjugal
partnership of Ricardo Silverio, Sr. and Beatriz S. Silverio.

RULING: The probate court having jurisdiction over properties under administration has the authority
not only to approve any disposition or conveyance, but also to annul an unauthorized sale by the
prospective heirs or administrator.

In this case, the sale of the subject properties was executed by respondent Silverio, Jr. with prior approval
of the intestate court under its Omnibus Order dated October 31, 2006.  Subsequently, however, the sale
was annulled by the said court on motion bypetitioner.
In reversing the intestate court's order annulling the sale of the subject properties, the CA noted that said
ruling is anchored on the fact that the deeds of sale were executed at the time when the TRO and writ of
preliminary injunction issued in CA-G.R. SP No. 97196 was still... in effect. It then concluded that the
eventual decision in the latter case making the writ of preliminary injunction permanent only with respect
to the appointment of petitioner as administrator and not to the grant of authority to sell mooted the issue
of whether the sale was... executed at the time when the TRO and writ of preliminary injunction were in
effect.

The CA's ruling on this issue is hereunder quoted:The more crucial question that needs to be addressed is:
Whether the authority to sell the properties in question granted under the October 31, 2006 Omnibus
Order, was nullified by the decision of the Court of Appeals in CA-G.R. SP No. 97196.  A look at the...
dispositive portion of the decision in CA-G.R. SP No. 97196 would lead us to reasonably conclude that
the grant of authority to sell is still good and valid.

The October 31, 2006 Omnibus Order of the testate [sic] court in so far as it authorizes the sale of the
three properties in question was not declared by the Court of Appeals, Seventh Division as null and void.

Remedial Law Review II (CY 2018-2019) – Saturday 1:00pm – 5:00pm: Case Digest Compilation [Atty. Brondial] Page 181
the injunction order which was made permanent by the Court of Appeals (Seventh Division) was declared
to be limited only to the portion of the Omnibus Order that upheld the grant of letters of administration by
SILVERIO, JR. and the... removal of SILVERIO, SR. as administrator and nothing else.

When the preliminary injunction was issued on 23 March 2011 new titles over the disputed properties
were already issued to CITRINE HOLDINGS, INC. and ZEE2 RESOURCES INC.

While it is true that petitioner was eventually reinstated as Administrator pursuant to the August 28, 2008
decision in CA-G.R. SP No. 97196 (petition for certiorari filed by Nelia Silverio-Dee), we agree with the
CA that the permanent injunction issued under the said decision,... as explicitly stated in its fallo,
pertained only to the portions of the October 31, 2006 Omnibus Order upholding the grant of letters of
administration to and taking of an oath of administration by respondent Silverio, Jr., as otherwise the CA
would have expressly set... aside as well the directive in the same Omnibus Order allowing the sale of the
subject properties.  Moreover, the CA Decision attained finality only on February 11, 2011 when this
Court denied with finality respondent Silverio, Jr.'s motion for reconsideration of the
February 11, 2009 Resolution denying his petition for review (G.R. No. 185619).

The CA therefore did not err in reversing the August 18, 2011 Order of the intestate court annulling the
sale of the subject properties grounded solely on the injunction issued in CA-G.R. SP No. 97196. 
Respondents Ocampo, Citrine and ZEE2 should not be prejudiced by the... flip-flopping appointment of
Administrator by the intestate court, having relied in good faith that the sale was authorized and with
prior approval of the intestate court under its Omnibus Order dated October 31, 2006 which remained
valid and subsisting insofar as it... allowed the aforesaid sale.

Remedial Law Review II (CY 2018-2019) – Saturday 1:00pm – 5:00pm: Case Digest Compilation [Atty. Brondial] Page 182
A.2. San Luis vs. San Luis

G.R. No. 133743; February 06, 2007, 514 SCRA 294; YNARES-SANTIAGO, J.

Facts: The instant case involves the settlement of the estate of Felicisimo T. San Luis (Felicisimo), who
was the former governor of the Province of Laguna. During his lifetime, Felicisimo contracted three
marriages. His first marriage was with Virginia Sulit on March 17, 1942 out of which were born six
children, namely: Rodolfo, Mila, Edgar, Linda, Emilita and Manuel. On August 11, 1963, Virginia
predeceased Felicisimo.

Five years later, Felicisimo married Merry Lee Corwin, with whom he had a son, Tobias. However,
Merry Lee, an American citizen, thereafter filed a Complaint for Divorce before the Family Court of the
First Circuit, State of Hawaii, United States of America (U.S.A.), which issued a Decree Granting
Absolute Divorce and Awarding Child Custody on December 14, 1973.

On June 20, 1974, Felicisimo married respondent Felicidad San Luis in Los Angeles, California, U.S.A.
He had no children with respondent but lived with her for 18 years from the time of their marriage up to
his death on December 18, 1992.

Thereafter, Felicidad sought the dissolution of their conjugal partnership assets and the settlement of
Felicisimo's estate. On December 17, 1993, she filed a petition for letters of administration before the
Regional Trial Court of Makati City.

Felicidad alleged that she is the widow of Felicisimo; that, at the time of his death, the decedent was
residing at 100 San Juanico Street, New Alabang Village, Alabang, Metro Manila; that the decedent's
surviving heirs are respondent as legal spouse, his six children by his first marriage, and son by his
second marriage; that the decedent left real properties, both conjugal and exclusive, valued at
P30,304,178.00 more or less; that the decedent does not have any unpaid debts. Felicidad prayed that the
conjugal partnership assets be liquidated and that letters of administration be issued to her.

On February 4, 1994, petitioner Rodolfo San Luis, one of the children of Felicisimo by his first marriage,
filed a motion to dismiss on the grounds of improper venue and failure to state a cause of action. Rodolfo
claimed that the petition for letters of administration should have been filed in the Province of Laguna
because this was Felicisimo's place of residence prior to his death. He further claimed that respondent has
no legal personality to file the petition because she was only a mistress of Felicisimo since the latter, at
the time of his death, was still legally married to Merry Lee.

Later Linda invoked the same grounds and joined her brother Rodolfo in seeking the dismissal of the
petition. The trial court issued an Order denying the two motions to dismiss.
Thereafter, Linda, Rodolfo and herein petitioner Edgar San Luis, separately filed motions for
reconsideration from the Order denying their motions to dismiss, which were denied.
Mila filed a motion for inhibition against Judge Tensuan and on the same day, Edgar also filed a motion
for reconsideration from the Order denying their motion for reconsideration arguing that it does not state
the facts and law on which it was based. Judge Tensuan issued an Order granting the motion for
inhibition. The case was re-raffled to Branch 134 presided by Judge Paul T. Arcangel.

Thereafter the trial court dismissed the petition for letters of administration.
The Court of Appeals reversed and set aside the orders of the trial court.
Edgar, Linda, and Rodolfo filed separate motions for reconsideration which were denied
Edgar appealed to the Supreme Court via the instant petition for review on certiorari. Rodolfo later filed
a manifestation and motion to adopt the said petition which was granted.

ISSUES: (1) Whether or not the petition for letters of administration was properly filed in Makati City;
and (2) whether respondent has legal capacity to file the subject petition for letters of administration..

RULING: Petition denied. The subject petition was validly filed before the Regional Trial Court of
Makati City, however the case should be remanded to the trial court for further reception of evidence on
the divorce decree obtained by Merry Lee and the marriage of respondent Felicidad and Felicisimo.

Under Section 1, Rule 73 of the Rules of Court, the petition for letters of administration of the estate of
Felicisimo should be filed in the Regional Trial Court of the province "in which he resides at the time of
his death."

Remedial Law Review II (CY 2018-2019) – Saturday 1:00pm – 5:00pm: Case Digest Compilation [Atty. Brondial] Page 183
The rulings in Nuval v. Guray and Romualdez v. RTC, are inapplicable to the instant case because they
involve election cases. In election cases, "residence" and "domicile" are treated as synonymous terms, that
is, the fixed permanent residence to which when absent, one has the intention of returning. However, for
purposes of fixing venue under the Rules of Court, the "residence" of a person is his personal, actual or
physical habitation, or actual residence or place of abode, which may not necessarily be his legal
residence or domicile provided he resides therein with continuity and consistency. Hence, it is possible
that a person may have his residence in one place and domicile in another.

In the instant case, while petitioners established that Felicisimo was domiciled in Sta. Cruz, Laguna,
respondent proved that he also maintained a residence in Alabang, Muntinlupa from 1982 up to the time
of his death. Respondent submitted in evidence the Deed of Absolute Sale dated January 5, 1983 showing
that the deceased purchased the aforesaid property. She also presented billing statements from the
Philippine Heart Center and Chinese General Hospital for the period August to December 1992 indicating
the address of Felicisimo at "100 San Juanico, Ayala Alabang, Muntinlupa." Respondent also presented
proof of membership of the deceased in the Ayala Alabang Village Association and Ayala Country Club,
Inc., letter-envelopes from 1988 to 1990 sent by the deceased's children to him at his Alabang address,
and the deceased's calling cards stating that his home/city address is at "100 San Juanico, Ayala Alabang
Village, Muntinlupa" while his office/provincial address is in "Provincial Capitol, Sta. Cruz, Laguna."

From the foregoing, we find that Felicisimo was a resident of Alabang, Muntinlupa for purposes of fixing
the venue of the settlement of his estate. Consequently, the subject petition for letters of administration
was validly filed in the Regional Trial Court which has territorial jurisdiction over Alabang, Muntinlupa.
The subject petition was filed on December 17, 1993. At that time, Muntinlupa was still a municipality
and the branches of the Regional Trial Court of the National Capital Judicial Region which had territorial
jurisdiction over Muntinlupa were then seated in Makati City as per Supreme Court Administrative Order
No. 3. Thus, the subject petition was validly filed before the Regional Trial Court of Makati City.

Anent the issue of respondent Felicidad's legal personality to file the petition for letters of administration,
we must first resolve the issue of whether a Filipino who is divorced by his alien spouse abroad may
validly remarry under the Civil Code, considering that Felicidad's marriage to Felicisimo was solemnized
on June 20, 1974, or before the Family Code took effect on August 3, 1988. In resolving this issue, we
need not retroactively apply the provisions of the Family Code, particularly Art. 26, par. (2) considering
that there is sufficient jurisprudential basis allowing us to rule in the affirmative.

Applying the doctrine in Alonzo v. Intermediate Appellate Court to the instant case, the divorce decree
allegedly obtained by Merry Lee which absolutely allowed Felicisimo to remarry, would have vested
Felicidad with the legal personality to file the present petition as Felicisimo's surviving spouse. However,
the records show that there is insufficient evidence to prove the validity of the divorce obtained by Merry
Lee as well as the marriage of respondent and Felicisimo under the laws of the U.S.A. In Garcia v. Recio,
the Court laid down the specific guidelines for pleading and proving foreign law and divorce judgments.
It held that presentation solely of the divorce decree is insufficient and that proof of its authenticity and
due execution must be presented. Under Sections 24 and 25 of Rule 132, a writing or document may be
proven as a public or official record of a foreign country by either (1) an official publication or (2) a copy
thereof attested by the officer having legal custody of the document. If the record is not kept in the
Philippines, such copy must be (a) accompanied by a certificate issued by the proper diplomatic or
consular officer in the Philippine foreign service stationed in the foreign country in which the record is
kept and (b) authenticated by the seal of his office.

With regard to Felicidad's marriage to Felicisimo allegedly solemnized in California, U.S.A., she
submitted photocopies of the Marriage Certificate and the annotated text of the Family Law Act of
California which purportedly show that their marriage was done in accordance with the said law. As
stated in Garcia, however, the Court cannot take judicial notice of foreign laws as they must be alleged
and proved.

Therefore, this case should be remanded to the trial court for further reception of evidence on the divorce
decree obtained by Merry Lee and the marriage of respondent and Felicisimo.
Even assuming that Felicisimo was not capacitated to marry respondent in 1974, nevertheless, we find
that the latter has the legal personality to file the subject petition for letters of administration, as she may
be considered the co-owner of Felicisimo as regards the properties that were acquired through their joint
efforts during their cohabitation.

Remedial Law Review II (CY 2018-2019) – Saturday 1:00pm – 5:00pm: Case Digest Compilation [Atty. Brondial] Page 184
Section 6, Rule 78 of the Rules of Court states that letters of administration may be granted to the
surviving spouse of the decedent. However, Section 2, Rule 79 thereof also provides in part:
SEC. 2. Contents of petition for letters of administration. — A petition for letters of
administration must be filed by an interested person and must show, as far as known to
the petitioner: . . . .
An "interested person" has been defined as one who would be benefited by the estate,
such as an heir, or one who has a claim against the estate, such as a creditor. The interest
must be material and direct, and not merely indirect or contingent.
In the instant case, respondent Felicidad would qualify as an interested person who has a direct interest in
the estate of Felicisimo by virtue of their cohabitation, the existence of which was not denied by
petitioners. If she proves the validity of the divorce and Felicisimo's capacity to remarry, but fails to prove
that her marriage with him was validly performed under the laws of the U.S.A., then she may be
considered as a co-owner under Article 144 of the Civil Code. This provision governs the property
relations between parties who live together as husband and wife without the benefit of marriage, or their
marriage is void from the beginning. It provides that the property acquired by either or both of them
through their work or industry or their wages and salaries shall be governed by the rules on co-ownership.
In a co-ownership, it is not necessary that the property be acquired through their joint labor, efforts and
industry. Any property acquired during the union is prima facie presumed to have been obtained through
their joint efforts. Hence, the portions belonging to the co-owners shall be presumed equal, unless the
contrary is proven.

Meanwhile, if respondent fails to prove the validity of both the divorce and the marriage, the applicable
provision would be Article 148 of the Family Code which has filled the hiatus in Article 144 of the Civil
Code by expressly regulating the property relations of couples living together as husband and wife but are
incapacitated to marry. In Saguid v. Court of Appeals, we held that even if the cohabitation or the
acquisition of property occurred before the Family Code took effect, Article 148 governs.

In view of the foregoing, we find that respondent's legal capacity to file the subject petition for letters of
administration may arise from her status as the surviving wife of Felicisimo or as his co-owner under
Article 144 of the Civil Code or Article 148 of the Family Code.

Remedial Law Review II (CY 2018-2019) – Saturday 1:00pm – 5:00pm: Case Digest Compilation [Atty. Brondial] Page 185
A.3. EDUARDO G. AGTARAP,Petitioner, vs.SEBASTIAN AGTARAP, JOSEPH AGTARAP,
TERESA AGTARAP, WALTER DE SANTOS, and ABELARDO DAGORO,Respondents.

G.R. No. 177099; June 8, 2011; NACHURA, J.

FACTS:Joaquin Agtarap contracted two marriages, first with Lucia Garcia (Lucia), and second with
Caridad Garcia (Caridad) whom he married after Lucia died. Joaquin had 3 children with Lucia: Jesus
(died without issue), Milagros, and Jose (survived by 3 children namely, Gloria, Joseph, and Teresa). He
also had 3 children with Caridad: Eduardo, Sebastian, and Mercedes (survived by her daughter Cecile).
Later on, Joaquin died (see Figure below).

On September 1994, Eduardo Agtarap (Eduardo) filed with the RTC, Pasay City, a verified petition for
the judicial settlement of the estate of his deceased father Joaquin Agtarap (Joaquin) on November 21,
1964. The petition alleged that Joaquin died intestate and without any known debts or obligations and that
the estate is composed of 2 parcels of land with improvements covered by TCTs. Eduardo also prayed to
be appointed as a special administrator to take possession and charge of the estate assets and their civil
fruits, pending the appointment of a regular administrator. Sebastian filed a comment conceding to the
appointment of Eduardo as special administrator.

Joseph, Gloria, and Teresa filed their answer/opposition alleging that a) the 2 subject lots belong to the
conjugal partnership of Joaquin with Lucia, and that, upon Lucia’s death in 1924, they became the pro
indiviso owners of the subject properties and b) Opposed the appointment of Eduardo as administrator on
the following grounds: (1) he is not physically and mentally fit to do so; (2) his interest in the lots is
minimal; and (3) he does not possess the desire to earn. On 1995, the RTC issued a resolution appointing
Eduardo as regular administrator and issued him letters of administration and later on issued an Order of
Partition. Eduardo, Sebastian, and oppositors Joseph and Teresa filed their respective motions for
reconsideration to which only Joseph and Teresa’s motion were granted. RTC declared that the real
estate properties belonged to the conjugal partnership of Joaquin and Lucia and ordered for
modification of partition. However, before the RTC could issue a new order of partition, Eduardo and
Sebastian both appealed to the CA but both were dismissed for lack of merit. Hence, both Eduardo and
Sebastian filed a consolidated petition for review on certiorari alleging that a probate court has no power
to determine the ownership of the property.

ISSUE:Whether or not RTC, acting as an intestate court with limited jurisdiction, can determine
questions of ownership, which properly belongs to another court with general jurisdiction?

RULING: Yes. RTC, as an intestate court, had jurisdiction to resolve the issues of ownership.

The general rule is that the jurisdiction of the trial court, either as a probate or an intestate court, relates
only to matters having to do with the probate of the will and/or settlement of the estate of deceased
persons, but does not extend to the determination of questions of ownership that arise during the
proceedings. The patent rationale for this rule is that such court merely exercises special and limited
jurisdiction. As held in several cases, a probate court or one in charge of estate proceedings, whether
testate or intestate, cannot adjudicate or determine title to properties claimed to be a part of the estate and
which are claimed to belong to outside parties, not by virtue of any right of inheritance from the deceased
but by title adverse to that of the deceased and his estate. All that the said court could do as regards said
properties is to determine whether or not they should be included in the inventory of properties to be
administered by the administrator. If there is no dispute, there poses no problem, but if there is, then the
parties, the administrator, and the opposing parties have to resort to an ordinary action before a court
exercising general jurisdiction for a final determination of the conflicting claims of title.
 
However, this general rule is subject to exceptions as justified by expediency and convenience. First,
the probate court may provisionally pass upon in an intestate or a testate proceeding the question of
inclusion in, or exclusion from, the inventory of a piece of property without prejudice to the final
determination of ownership in a separate action. Second, if the interested parties are all heirs to the estate,
or the question is one of collation or advancement, or the parties consent to the assumption of jurisdiction
by the probate court and the rights of third parties are not impaired, then the probate court is competent to
resolve issues on ownership. Verily, its jurisdiction extends to matters incidental or collateral to the
settlement and distribution of the estate, such as the determination of the status of each heir and whether
the property in the inventory is conjugal or exclusive property of the deceased spouse.

Remedial Law Review II (CY 2018-2019) – Saturday 1:00pm – 5:00pm: Case Digest Compilation [Atty. Brondial] Page 186
We hold that the general rule does not apply to the instant case considering that the parties are all
heirs of Joaquin and that no rights of third parties will be impaired by the resolution of the
ownership issue. More importantly, the determination of whether the subject properties are conjugal is
but collateral to the probate court’s jurisdiction to settle the estate of Joaquin. Section 2, Rule 73 of the
Rules of Court provides that when the marriage is dissolved by the death of the husband or the wife, the
community property shall be inventoried, administered, and liquidated, and the debts thereof paid; in the
testate or intestate proceedings of the deceased spouse, and if both spouses have died, the conjugal
partnership shall be liquidated in the testate or intestate proceedings of either. Thus, the RTC had
jurisdiction to determine whether the properties are conjugal as it had to liquidate the conjugal partnership
to determine the estate of the decedent.

DOCTRINE: The general rule is that the jurisdiction of the trial court, either as a probate or an
intestate court, relates only to matters having to do with the probate of the will and/or settlement
of the estate of deceased persons, but does not extend to the determination of questions of
ownership that arise during the proceedings. The patent rationale for this rule is that such court
merely exercises special and limited jurisdiction.

However, the general rule is subject to exceptions as justified by expediency and convenience Verily, its
jurisdiction extends to matters incidental or collateral to the settlement and distribution of the estate,
such as the determination of the status of each heir and whether the property in the inventory is conjugal
or exclusive property of the deceased spouse.

JOSE+ Gloria

JOAQUIN LUCIA GARCIA+ Teresa


AGTARAP+ (1st marriage)

MILAGROS
JESUS+ Joseph
(died without issue)

EDUARDO
(petitioner)

JOAQUIN CARIDAD GARCIA+


AGTARAP+ (2nd marriage) SEBASTIAN

MERCEDES+ Cecile

Remedial Law Review II (CY 2018-2019) – Saturday 1:00pm – 5:00pm: Case Digest Compilation [Atty. Brondial] Page 187
A.4.Emilio M. Suntay III vs. Isabel Coangco-Suntay

G.R. No. 183053; October 10, 2012; PEREZ J.

FACTS: Christina Aguinaldo-Sunaty (Cristina), the decedent, married to Dr. Federico Suntay (Federico),
diedinstestate in 1990. In 1979, however, their only son, Emilio Aguinaldo Suntay (Emilio I),
predeceasedboth Cristrina and Federico. At the time of her death, Cristina wassurvived by her husband,
Federico, and several grandchildren, including therein petitioner, Emilio Suntay III and respondent Isabel
Cojuangco-Suntay. Emilio I was married to Isabel Cojuangco and had three children, respondent Isabel,
Margarita, and Emilio II. Emilio’s I marriage was subsequently annulled and Emilio I had two children
out of wedlock, Emilio III, herein petitioner and Nenita Suntay. Consequently, respondent and her
siblings Margarita and Emilio II live with their mother, separately from their father and paternal
grandparents. After the death of Emilio I, Federico filed petition for visitation rights over his
grandchildren but was altogether stopped because of a manifestation filed by respondent, Isabel
articulating her sentiments on the unwanted visits of her grandparents. After her spouses’s death, Federico
legally adopted his illegitmate grandchildren, Nenita and Emilio III. Reposndent later filed a petition for
theissuance of letters administration over decedent’s property but was opposed by Federico, alleging that
being the spouse of Cristina, he is capable of administering her state and he should be the one to appoint
an adminsitatotr, in the person of Emilio II and that as part owner of the mass of the conjugal properties,
he must be accorded legal preference in the administration. In the course of the proceedings, Federico
died. The trial court rendered a decision of appointing petitioner Emilio III as administrator of decedent
Cristina’s intestate estate. The Court believes that to appoint Isabel Cojuangco-Suntay would go against
the wishes of the decedent who raise Emilio III from infancy as her own child and would likewise go
against the wishes of Federico, who nominated Emilio III for appointnment as administrator.

On appeal, the Court of Appeals reversed and set aside the decision of the Regional Trial Court and
revoked the Letters of Administration issued to Emilio III. The Court of Appeals focused on Emilio III’s
status as an illegitimate child of Emilio I and therefore barred from representing his deceased father in the
estate of the latter’s legitimate mother and that he cannot be appointed as administrator because the
appointment of Emilio III was subject to a suspensive condition, that between the legitimatye offspring
(respondent) and illegitimate one (petitioner) respondent is preferred, being the next of kin referred to by
Section 6, Rule 78 of the Rules of Court, and jurisprudence has consistently held that Art. 992 of the Civil
Code bars the illegitimate child from inheriting ab instestado from the legitimate children and relative of
his father and mother.

ISSUES:
A. In the appointemnt of an administator of the estate under Sec. 6 of Rule 78 of the Rules of Court,
whether Art. 992 of the Civil Code applies?
B. Who as between Emilio III and respondent Isabel is better qualified to act as administrator of the
decedent’s estate?

RULING: The basis for Art. 992 of the Civil Code, referred to as the iron curtail bar rule, is opposite to
the scenario in the facts obtaining the actual relationship between Federico and Cristina, on one hand, and
Emilio III, on the other, was akin to the normal relationship of legitimate relatives. In the appointment of
an administrator, the principla consideration is the interest of the estate of the one to be appointed. The
order of preference does not rule our the appoitnment of co-administrators, especially in cases where
justice and equity demand that opposing parties of faction be represented in the management of the
estates.

In the case at bar, Emilio III and Nenita were legally adopted by Federico, putting them in equal footing
with that of legitimate children and were treated by the decedent and her husband as their own, reared
from infancy, educated and trained in their business,while the relationship of the respondent was trained.
The factual antecedents of this case accurately reflect the basis of intestate succession, “love first
descends”, for the decedent Cristina, did not distinguish between her legitimate and illegitimate
grandchildren. Neither did her husband, Federico, who legally raised the statusof Emilio III from an
illegitimate granchildren to that of a legitimate child. The law of intestacy is founded on the presumed
will of the deceased. Love, it is said, first descends, then descends, and finally spreads sideways.

Letters of Administration over the estate of decedent Christina Aguinaldo-Suntay shall be issued to both
the petitioner, Emilio III respondent, Isabel Cojuangco-Suntay.

Remedial Law Review II (CY 2018-2019) – Saturday 1:00pm – 5:00pm: Case Digest Compilation [Atty. Brondial] Page 188
DOCTRINE:The paramount consideration in the appointment of an administratorover the estate of a
decedent is the prospective administrator’sinterest in the estate.―The paramount consideration in the
appointment of an administrator over the estate of a decedent is the prospective administrator’s interest
in the estate. This is the same consideration which Section 6, Rule 78 takes into account in establishing
the order of preference in the appointment of administrator for the estate. The rationale behind the rule is
that those who will reap the benefit of a wise, speedy and economicaladministration of the estate, or, in
the alternative, suffer theconsequences of waste, improvidence or mismanagement, havethe highest
interest and most influential motive to administer theestate correctly. In all, given that the rule speaks of
an order ofpreference, the person to be appointed administrator of adecedent’s estate must demonstrate
not only an interest in theestate, but an interest therein greater than any other candidate.

In a number of cases, the Supreme Courthas sanctioned the appointment of more than one administrator
for the benefit of the estate and those interested therein.―In anumber of cases, we have sanctioned the
appointment of more than one administrator for the benefit of the estate and those interested therein. We
recognized that the appointment of administrator of the estate of a decedent or the determination of
aperson’s suitability for the office of judicial administrator rests, toa great extent, in the sound judgment
of the court exercising the power of appointment. Under certain circumstances and for various reasons
well settled in Philippine and American jurisprudence, we have upheld the appointment of
coadministrators: (1) to have the benefits of their judgment and perhaps at all times to have different
interests represented; (2)where justice and equity demand that opposing parties or factionsbe
represented in the management of the estate of the deceased;(3) where the estate is large or, from any
cause, an intricate and perplexingone to settle; (4) to have all interested persons satisfied and
therepresentatives to work in harmony for the best interests of theestate; and when a person entitled to
the administration of anestate desires to have another competent person associated withhim in the office.

Remedial Law Review II (CY 2018-2019) – Saturday 1:00pm – 5:00pm: Case Digest Compilation [Atty. Brondial] Page 189
A.5. JOSE C. LEE AND ALMA AGGABAO, in their capacities as President and Corporate
Secretary, respectively, of Philippines Internationl Life Insurance Company, and FILIPINO LOAN
ASSISTANCE GROUP, petitioners, vs. REGIONAL TRIAL COURT OF QUEZON CITY
BRANCH 85 presided by JUDGE PEDRO M. AREOLA, BRANCH CLERK OF COURT JANICE
Y. ANTERO, DEPUTY SHERIFFS ADENAUER G. RIVERA and PEDRO L. BORJA, all of the
Regional Trial Court of Quezon City Branch 85, MA. DIVINA ENDERES claiming to be Special
Administratrix, and other persons/ public officers acting for and in their behalf, respondents.

G.R. No. 146006; February 23, 2004; CORONA, J.:

This is a petition for review under Rule 45 of the Rules of Court seeking to reverse and set aside the
decision of the Court of Appeals, First Division, dated July 26, 2000, in CA G.R. 59736, which dismissed
the petition for certiorari filed by petitioners Jose C. Lee and Alma Aggabao (in their capacities as
president and secretary, respectively, of Philippine International Life Insurance Company) and Filipino
Loan Assistance Group.

FACTS: Dr.Juvencio P. Ortaez incorporated the Philippine International Life Insurance Company, Inc.
on July 6, 1956, owning 90% of the subscribed capital stock. He died, leaving his wife (Juliana Salgado
Ortaez), three legitimate children (Rafael, Jose and Antonio Ortaez) and five illegitimate children by
LigayaNovicio (herein private respondent Ma. DivinaOrtaez-Enderes and her siblings Jose, Romeo,
Enrico Manuel and Cesar, all surnamed Ortaez).

Rafael filed before the CFI of Rizal, (RTC of Quezon City) a petition for letters of administration, which
was opposed by private respondent Ma. Divina - who, in a subsequent urgent motion, prayed that the
intestate court appoint a special administrator. The presiding judge of Branch 85, appointed Rafael and
Jose as joint special administrators while hearings continued for the appointment of a regular
administrator. As a result an inventory, which included, among other properties, 2,029 shares of stock in
Philippine International Life Insurance Company (hereafter Philinterlife), representing 50.725% of the
companys outstanding capital stock, was submitted, to which half of said shares was being claimed by the
Wife as her conjugal share. She, then, sold them with right to repurchase in favor of herein petitioner
Filipino Loan Assistance Group (FLAG). She failed to repurchase it within the stipulated period, resulting
to the consolidation of the ownership in FLAG’s name.

Special Administrator Jose, acting in his personal capacity and claiming that he owned the remaining
1,011 Philinterlife shares of stocks as his inheritance share in the estate, sold said shares with right to
repurchase also in favor of FLAG, which after one year, consolidated in its name the ownership of the
said Philinterlife shares upon failure to repurchase. It appears that several years before (but already during
the pendency of the intestate proceedings at the Regional Trial Court of Quezon City, Branch 85), Juliana
and her two children, Special Administrators Rafael and Jose, entered into a memorandum of agreement
for the extrajudicial settlement and partitioning the estate (including the Philinterlife shares of stock)
among themselves. This was the basis of the number of shares separately sold by Juliana (1,014 shares)
and by Jose (1,011 shares) in favor of herein petitioner FLAG.

Private respondent Ma.Divina and her siblings (hereafter referred to as private respondents Enderes et al.)
filed a motion for appointment of special administrator of Philinterlife shares of stock, which was
opposed by Jose; but which, was granted by the intestate court, appointing private respondent Enderes as
special administratrix of the Philinterlife shares of stock. She then filed an urgent motion to declare
void ab initio the memorandum of agreement and another, to declare the partial nullity of the extrajudicial
settlement. Jose opposed. AdministratrixEnderes filed an urgent motion to declare void ab initio the deeds
of sale of Philinterlife shares of stock, which again was opposed by Jose, who filed an omnibus motion
for (1) the approval of the deeds of sale of the Philinterlife shares of stock and (2) the release of Ma.
DivinaOrtaez-Enderes as special administratrix of the Philinterlife shares of stock on the ground that there
were no longer any shares of stock for her to administer, which was eventually denied by the intestate
court, which further issued another order granting the motion of Special AdministratrixEnderes for the
annulment of the memorandum of agreement or extrajudicial partition of estate.

ISSUE:
1. Whether the sale of the shares of stock is valid.
2. Wheherthe intestate or probate court execute its order nullifying the invalid sale.

RULING: No. What we have here is a situation where some of the heirs of the decedent without securing
court approval have appropriated as their own personal property the properties of [the] Estate, to the

Remedial Law Review II (CY 2018-2019) – Saturday 1:00pm – 5:00pm: Case Digest Compilation [Atty. Brondial] Page 190
exclusion and the extreme prejudice of the other claimant/heirs. In other words, these heirs, without court
approval, have distributed the asset of the estate among themselve and proceeded to dispose the same to
third parties even in the absence of an order of distribution by the Estate Court. As admitted by petitioners
counsel, there was absolutely no legal justification for this action by the heirs. There being no legal
justification, petitioner has no basis for demanding that public respondent [the intestate court] approve the
sale of the Philinterlife shares of the Estate by Juliana and Jose Ortaez in favor of the Filipino Loan
Assistance Group.

It is clear that Juliana Ortaez, and her three sons, Jose, Rafael and Antonio, all surnamed Ortaez, invalidly
entered into a memorandum of agreement extrajudicially partitioning the intestate estate among
themselves, despite their knowledge that there were other heirs or claimants to the estate and before final
settlement of the estate by the intestate court. Since the appropriation of the estate properties by Juliana
Ortaez and her children (Jose, Rafael and Antonio Ortaez) was invalid, the subsequent sale thereof by
Juliana and Jose to a third party (FLAG), without court approval, was likewise void.

An heir can sell his right, interest, or participation in the property under administration under Art. 533 of
the Civil Code which provides that possession of hereditary property is deemed transmitted to the heir
without interruption from the moment of death of the decedent. However, an heir can only alienate such
portion of the estate that may be allotted to him in the division of the estate by the probate or intestate
court after final adjudication, that is, after all debtors shall have been paid or the devisees or legatees shall
have been given their shares. This means that an heir may only sell his ideal or undivided share in the
estate, not any specific property therein. In the present case, Juliana Ortaez and Jose Ortaez sold specific
properties of the estate (1,014 and 1,011 shares of stock in Philinterlife) in favor of petitioner FLAG. This
they could not lawfully do pending the final adjudication of the estate by the intestate court because of the
undue prejudice it would cause the other claimants to the estate, as what happened in the present case.
Juliana Ortaez and Jose Ortaez sold specific properties of the estate, without court approval. It is well-
settled that court approval is necessary for the validity of any disposition of the decedents estate. In the
early case of Godoy vs. Orellano,[22] we laid down the rule that the sale of the property of the estate by an
administrator without the order of the probate court is void and passes no title to the purchaser.

Our jurisprudence is therefore clear that (1) any disposition of estate property by an administrator or
prospective heir pending final adjudication requires court approval and (2) any unauthorized disposition
of estate property can be annulled by the probate court, there being no need for a separate action to annul
the unauthorized disposition.

2. Can the intestate or probate court execute its order nullifying the invalid sale?
We see no reason why it cannot. The intestate court has the power to execute its order with regard to the
nullity of an unauthorized sale of estate property, otherwise its power to annul the unauthorized or
fraudulent disposition of estate property would be meaningless. In other words, enforcement is a
necessary adjunct of the intestate or probate courts power to annul unauthorized or fraudulent transactions
to prevent the dissipation of estate property before final adjudication.

We are not dealing here with the issue of inclusion or exclusion of properties in the inventory of the estate
because there is no question that, from the very start, the Philinterlife shares of stock were owned by the
decedent, Dr.JuvencioOrtaez. Rather, we are concerned here with the effect of the sale made by the
decedents heirs, Juliana Ortaez and Jose Ortaez, without the required approval of the intestate
court. This being so, the contention of petitioners that the determination of the intestate court was merely
provisional and should have been threshed out in a separate proceeding is incorrect.

The petitioners Jose Lee and Alma Aggabao next contend that the writ of execution should not be
executed against them because they were not notified, nor they were aware, of the proceedings nullifying
the sale of the shares of stock.

We are not persuaded. The title of the purchaser like herein petitioner FLAG can be struck down by the
intestate court after a clear showing of the nullity of the alienation. This is the logical consequence of our
ruling in Godoy and in several subsequent cases. The sale of any property of the estate by an
administrator or prospective heir without order of the probate or intestate court is void and passes
no title to the purchaser. Thus, in Juan Lao et al. vs. Hon. MelencioGeneto, G.R. No. 56451, June 19,
1985, we ordered the probate court to cancel the transfer certificate of title issued to the vendees at the
instance of the administrator after finding that the sale of real property under probate proceedings was
made without the prior approval of the court.

Remedial Law Review II (CY 2018-2019) – Saturday 1:00pm – 5:00pm: Case Digest Compilation [Atty. Brondial] Page 191
WHEREFORE, the petition is hereby DENIED. The decision of the Court of Appeals in CA-G.R. S.P.
No. 59736 dated July 26, 2000, dismissing petitioners petition for certiorari and affirming the July 6,
2000 order of the trial court which ordered the execution of its (trial courts) August 11 and 29, 1997
orders, is hereby AFFIRMED. SO ORDERED.

Remedial Law Review II (CY 2018-2019) – Saturday 1:00pm – 5:00pm: Case Digest Compilation [Atty. Brondial] Page 192
A.6. THE ESTATE OF HILARIO RUIZ, EDMOND RUIZVS. COURT OF APPEALS, MARIA
PILAR MONTES, MA. CATHRYN RUIZ, etc.

G.R. No. 118671; JANUARY 29, 1996; PUNO, J,:

FACTS:Hilario Ruiz executed a holographic will naming his heirs his only son, Edmond Ruiz, his
adopted daughter, private respondent Ma. Pilar Ruiz Montes, and his three grand daughter, Ma. Cathryn,
Candace Albertine and Ma.Angeline, all children of Edmond Ruiz. On April 12, 1988, Hilario Ruiz died.
Immediately thereafter, the cash component of his estate was distributed among Edmond Ruiz and private
respondents. Edmond, the named executor did not take action for the probate of the will. Four years after
the testator’s death, private respondent, Pilar, filed before the RTC, Branch 156, Pasig, a petition for the
probate and approval of the Hilario Ruiz’s will and for the issuance of the letters testamentary to Edmond
Ruiz. This was opposed by the latter.

One of the properties of the Estate bequeathed by the testator to his grandchildren was leased by Edmond.
He was then ordered by the court to deposit with the branch Clerk of Court the rental deposit and
payment. On March, 1993, he moved for the release of 50,000.00 to pay for the real estate taxes on the
Real properties of the Estate. On July 1993, Petitioner filed an Ex-Parte Motion for release of funds which
was opposed by respondent and prayed for the release of the said payments to the granddaughters instead.

The Probate court granted the motion of respondent. Petitioner moved for reconsideration thus the probate
court ordered the release of the funds to Edmond but only “such amount as may be necessary to cover the
expenses of administration and allowance for support” of the testator’s three granddaughters subject to
collation and deductible from their share of the inheritance. The court, however, held in abeyance the
release of the titles to respondents Montes and the three granddaughters until the lapse of six months from
the date of publication of the notice of the creditor.

This decision was sustained by the CA upon appeal by the petitioner.

ISSUES:Whether or not the probate court, after admitting the will to probate but before the payment of
the estate’s debts and obligations, has the authority:
3) To grant an allowance from the funds of the estate for the support of the testator’s grandchildren
4) To order the release of the titles to certain heirs, and
5) To grant possession of all properties of the estate to the executor of the will

RULING:

1st ISSUE:On the matter of allowance, Section 3 of Rule 83 of the Revised Rules of Court provides: The
widow and minor or incapacitated children of a deceased person, during the settlement of the estate, shall
receive therefrom under the direction of the court, such allowance as are provided by law.

It is settles that allowances for support under Sec.3 of Rule 83 should not be limited to the minor or
incapacitated children of the deceased. Art 188 of the Civil Code of The Philippines, the substantive law
in force at the time of the testator’s death, provides that during the liquidation of the conjugal partnership,
the deceased’s legitimate spouse and children, regardless of their age, civil status or gainful employment,
are entitled to provisional support from the funds of the Estate. The law is rooted on the fact that the right
and duty to support, especially the right to education, subsist even beyond the age of majority.

Be that as it may, grandchildren are NOT entitled to provisional support from the funds of the decedent’s
estate. The law clearly limits the allowance to “widow and children”

2nd ISSUE:No. Sec 1 of Rule 90 provides that: When the debts, funeral charges, and expenses of
administration the allowance to the widow, and inheritance tax if any, chargeable to the estate in
accordance with law, have been paid, the court, on the application of the executor or administrator, or of a
person interested in the estate, and after hearing upon notice shall assign the residue of the estate to the
persons entitled to the same, xxx

In settlement of estate proceedings, the distribution of the estate properties can only be made: 1) after all
the debts, funeral, charges, expenses of administration, allowance to the widow and estate tax have been
paid; or 2) before payment of said obligations only if the distributes or any of them gives a bond in sum
fixed by the court conditioned upon the payment of said obligations within such time as the court directs,
or when provision is made to meet those obligation.

Remedial Law Review II (CY 2018-2019) – Saturday 1:00pm – 5:00pm: Case Digest Compilation [Atty. Brondial] Page 193
In the case at bar, the probate court ordered the release of the titles to private respondents after the lapse
of six months from the date of publication of notice to the creditor. The questioned order speaks of notice
to creditor and NOT payment of debts and obligations. Notably, at the time the order was issued the
properties of the estate had not yet been inventoried.

3rd ISSUE:The petitioner cannot correctly claim that the assailed order deprived him of his right to take
possession of all real and personal properties of the estate. The right of the executor or administrator to
the possession and management of the real and personal properties of the deceased is not absolute and can
only be exercised “so long as it is necessary for the payment of the debts and expenses of administration,
“Sec. 3 of Rule 84 of the Revised Rules of Court explicitly provides:An executor or administrator shall
have the right to the possession and management of the real as well as the personal estate of the deceased
as long as it is necessary for the payment of the debts and expenses for administration.

It was correct for the probate court to require him to submit an accounting of the necessary expenses for
administration before releasing any further money in his favor.

Remedial Law Review II (CY 2018-2019) – Saturday 1:00pm – 5:00pm: Case Digest Compilation [Atty. Brondial] Page 194
A.7. Unionbank vs. Santibanez

452 SCRA 228

FACTS: On May 31, 1980, the First Countryside Credit Corporation (FCCC) and EfraimSantibañez
entered into a loan agreement in the amount of P128,000.00. The amount was intended for the payment of
one (1) unit Ford 6600 Agricultural Tractor. In view thereof, Efraim and his son, Edmund, executed a
promissory note in favor of the FCCC, the principal sum payable in five equal annual amortizations. On
Dec. 1980, FCCC and Efraim entered into another loan agreement for the payment of another unit of Ford
6600 and one unit of a Rotamotor. Again, Efraim and Edmund executed a promissory note and a
Continuing Guaranty Agreement for the later loan. In 1981, Efraim died, leaving a holographic will.
Testate proceedings commenced before the RTC of Iloilo City. Edmund was appointed as the special
administrator of the estate. During the pendency of the testate proceedings, the surviving heirs, Edmund
and his sister Florence, executed a Joint Agreement, wherein they agreed to divide between themselves
and take possession of the three (3) tractors: (2) tractors for Edmund and (1) for Florence. Each of them
was to assume the indebtedness of their late father to FCCC, corresponding to the tractor respectively
taken by them. In the meantime, a Deed of Assignment with Assumption of Liabilities was executed by
and between FCCC and Union Bank, wherein the FCCC assigned all its assets and liabilities to Union
Bank.

Demand letters were sent by Union Bank to Edmund, but the latter refused to pay. Thus, on February 5,
1988, Union Bank filed a Complaint for sum of money against the heirs of EfraimSantibañez, Edmund
and Florence, before the RTC of Makati City. Summonses were issued against both, but the one intended
for Edmund was not served since he was in the United States and there was no information on his address
or the date of his return to the Philippines. Florence filed her Answer and alleged that the loan documents
did not bind her since she was not a party thereto. Considering that the joint agreement signed by her and
her brother Edmund was not approved by the probate court, it was null and void; hence, she was not liable
to Union Bank under the joint agreement.

Union Bank asserts that the obligation of the deceased had passed to his legitimate heirs (Edmund and
Florence) as provided in Article 774 of the Civil Code; and that the unconditional signing of the joint
agreement estopped Florence, and that she cannot deny her liability under the said document.

In her comment to the petition, Florence maintains that Union Bank is trying to recover a sum of money
from the deceased EfraimSantibañez; thus the claim should have been filed with the probate court. She
points out that at the time of the execution of the joint agreement there was already an existing probate
proceedings. She asserts that even if the agreement was voluntarily executed by her and her brother
Edmund, it should still have been subjected to the approval of the court as it may prejudice the estate, the
heirs or third parties.

ISSUE: Whether the agreement entered by Edmund and Florence is valid? Whether the heirs’ assumption
of indebtedness is binding?

RULING:
1. No. In our jurisdiction, the rule is that there can be no valid partition among the heirs until after the will
has been probated: “In testate succession, there can be no valid partition among the heirs until after the
will has been probated. The law enjoins the probate of a will and the public requires it, because unless a
will is probated and notice thereof given to the whole world, the right of a person to dispose of his
property by will may be rendered nugatory. The authentication of a will decides no other question than
such as touch upon the capacity of the testator and the compliance with those requirements or solemnities
which the law prescribes for the validity of a will” This, of course, presupposes that the properties to be
partitioned are the same properties embraced in the will. In the present case, the deceased,
EfraimSantibañez, left a holographic will which contained, inter alia, the provision which reads as
follows: (e) All other properties, real or personal, which I own and may be discovered later after my
demise, shall be distributed in the proportion indicated in the immediately preceding paragraph in favor of
Edmund and Florence, my children.

We agree with the appellate court that the above-quoted is an all-encompassing provision embracing all
the properties left by the decedent which might have escaped his mind at that time he was making his
will, and other properties he may acquire thereafter. Included therein are the three (3) subject tractors.
This being so, any partition involving the said tractors among the heirs is not valid. The joint agreement
executed by Edmund and Florence, partitioning the tractors among themselves, is invalid, specially so

Remedial Law Review II (CY 2018-2019) – Saturday 1:00pm – 5:00pm: Case Digest Compilation [Atty. Brondial] Page 195
since at the time of its execution, there was already a pending proceeding for the probate of their late
father’s holographic will covering the said tractors.

2. No. The filing of a money claim against the decedent’s estate in the probate court is mandatory. As we
held in the vintage case of PyEng Chong v. Herrera: This requirement is for the purpose of protecting the
estate of the deceased by informing the executor or administrator of the claims against it, thus enabling
him to examine each claim and to determine whether it is a proper one which should be allowed. The
plain and obvious design of the rule is the speedy settlement of the affairs of the deceased and the early
delivery of the property to the distributees, legatees, or heirs. `The law strictly requires the prompt
presentation and disposition of the claims against the decedent's estate in order to settle the affairs of the
estate as soon as possible, pay off its debts and distribute the residue. Perusing the records of the case,
nothing therein could hold private respondent Florence S. Ariola accountable for any liability incurred by
her late father. The documentary evidence presented, particularly the promissory notes and the continuing
guaranty agreement, were executed and signed only by the late EfraimSantibañez and his son Edmund.
As the petitioner failed to file its money claim with the probate court, at most, it may only go after
Edmund as co-maker of the decedent under the said promissory notes and continuing guaranty, of course,
subject to any defenses Edmund may have as against the petitioner. As the court had not acquired
jurisdiction over the person of Edmund, we find it unnecessary to delve into the matter further.

Remedial Law Review II (CY 2018-2019) – Saturday 1:00pm – 5:00pm: Case Digest Compilation [Atty. Brondial] Page 196
A.8. Garcia-Quiazon vs. Belen

G.R. NO. 189121; July 31, 2013; Perez, J.;

FACTS:Eliseo died intestate on December 12, 1992. On September 12, 1994, Elise filed a Petition for
Letters of Administration and claimed that she is the natural child of Eliseo, having been conceived and
born at the time when her parents were both capacitated to marry each other. Insisting on the legal
capacity of Eliseo and Lourdes to marry, Elise impugned the validity of Eliseo’s marriage to Amelia by
claiming that it was bigamous for having been contracted during the subsistence of the latter’s marriage
with FilipitoSandico. To prove her filiation to the decedent, Elise attached to the Petition for Letters of
Administration her Certificate of Live Birth, signed by Eliseo as her father.

Claiming that the venue of the petition was improperly laid, Amelia, together with her children, opposed
the issuance of the letters of administration by filing an Opposition/Motion to Dismiss. The petitioners
asserted that as shown in the Death Certificate, Eliseo was a resident of Capas, Tarlac and not of Las
Piñas City at the time of his death. Pursuant to Sec1, R73, the petition for settlement of the estate should
have been filed in Capas and not in Las Piñas. In addition to their claim of improper venue, the petitioners
averred that there are no factual and legal bases for Elise to be administratix of Eliseo’s estate.

The lower court ruled that the venue of the petition was properly laid in Las Piñas and directed the
issuance of the Letters of Administration to Elise upon posting the necessary bond. On appeal, the
decision of the trial court was affirmed in toto.

ISSUE:
A. Whether or not the residence of the decedent as indicated in the death certificate should be taken into
account for purposes of determining the venue for the probate of the will.

B. Whether or not the natural child of the decedent may be appointed as an administrator.

RULING:
A. Under Section 1, Rule 73 of the Rules of Court, the petition for letters of administration of the estate of
a decedent should be filed in the RTC of the province where the decedent resides at the time of his death.

"Resides" should be viewed or understood in its popular sense, meaning, the personal, actual or physical
habitation of a person, actual residence or place of abode. It signifies physical presence in a place and
actual stay thereat. Venue for ordinary civil actions and that for special proceedings have one and the
same meaning. As thus defined, "residence," in the context of venue provisions, means nothing more than
a person’s actual residence or place of abode, provided he resides therein with continuity and consistency.

Moreover, the Court of Appeals cannot be faulted for affirming the ruling of the RTC that the venue for
the settlement of the estate of Eliseo was properly laid in Las Piñas City. It is evident from the records
that during his lifetime, Eliseo resided in Las Piñas City. For this reason, the venue for the settlement of
his estate may be laid in the said city. While the recitals in death certificates can be considered proofs of a
decedent’s residence at the time of his death, the contents thereof, however, is not binding on the courts.
Both the RTC and the Court of Appeals found that Eliseo had been living with Lourdes, deporting
themselves as husband and wife, from 1972 up to the time of his death in 1995. This finding is consistent
with the fact that in 1985, Eliseo filed an action for judicial partition of properties against Amelia before
the RTC of Quezon City, Branch 106, on the ground that their marriage is void for being bigamous. That
Eliseo went to the extent of taking his marital feud with Amelia before the courts of law renders
untenable petitioners’ position that Eliseo spent the final days of his life in Tarlac with Amelia and her
children. It disproves rather than supports petitioners’ submission that the lower courts’ findings arose
from an erroneous appreciation of the evidence on record. Factual findings of the trial court, when
affirmed by the appellate court, must be held to be conclusive and binding upon this Court.

B. Section 2 of Rule 79 provides that a petition for Letters of Administration must be filed by an
interested person and no defect in the petition shall render void the issuance of letters of administration.
An "interested party," in estate proceedings, is one who would be benefited in the estate, such as an heir,
or one who has a claim against the estate, such as a creditor. Also, in estate proceedings, the phrase "next
of kin" refers to those whose relationship with the decedent Is such that they are entitled to share in the
estate as distributees.

Remedial Law Review II (CY 2018-2019) – Saturday 1:00pm – 5:00pm: Case Digest Compilation [Atty. Brondial] Page 197
In the instant case, Elise, as a compulsory heir who stands to be benefited by the distribution of Eliseo’s
estate, is deemed to be an interested party. With the overwhelming evidence on record produced by Elise
to prove her filiation to Eliseo, the petitioners’ pounding on her lack of interest in the administration of
the decedent’s estate, is just a desperate attempt to sway this Court to reverse the findings of the Court of
Appeals. Certainly, the right of Elise to be appointed administratix of the estate of Eliseo is on good
grounds. It is founded on her right as a compulsory heir, who, under the law, is entitled to her legitimate
after the debts of the estate are satisfied.Having a vested right in the distribution of Eliseo’s estate as one
of his natural children, Elise can rightfully be considered as an interested party within the purview of the
law.

DOCTRINES: While the recitals in death certificates can be considered proofs of a decedent’s
residence at the time of his death, the contents thereof, however, is not binding on the courts.

An "interested party," in estate proceedings, is one who would be benefited in the estate, such as an heir,
or one who has a claim against the estate, such as a creditor. Also, in estate proceedings, the phrase
"next of kin" refers to those whose relationship with the decedent Is such that they are entitled to share in
the estate as distributes.

Remedial Law Review II (CY 2018-2019) – Saturday 1:00pm – 5:00pm: Case Digest Compilation [Atty. Brondial] Page 198
A.9.PILAPIL and HEIRS OF DONATA ORTIZ BRIONES vs. HEIRS OF MAXIMINO R.
BRIONES

G.R. No. 150175; February 5, 2007

FACTS: Maximino was married to Donata but their union did not produce any children. When
Maximino died on 1 May 1952, Donata instituted intestate proceedings to settle her husband’s estate. CFI
issued Letters of Administration appointing Donata as the administratrix of Maximino’s estate.
Subsequently, CFI issue an Order, dated 2 October 1952, awarding ownership of the aforementioned real
properties to Donata.

Donata died on 1 November 1977. Erlinda, one of Donata’s nieces, instituted with the RTC a petition for
the administration of the intestate estate of Donata. Erlinda and her husband, Gregorio, were appointed by
the RTC as administrators of Donata’s intestate estate.

On 3 March 1987, the heirs of Maximino filed a Complaint with the RTC against the heirs of Donata for
the partition, annulment, and recovery of possession of real property. They alleged that Donata, as
administratrix of the estate of Maximino, through fraud and misrepresentation, in breach of trust, and
without the knowledge of the other heirs, succeeded in registering in her name the real properties
belonging to the intestate estate of Maximino.

After trial in due course, the RTC rendered its Decision, dated 8 April 1986, in favor of the heirs of
Maximino. The heirs of Donata appealed the RTC Decision before the Court of Appeals but the latter
court affirmed the decision. Unsatisfied the Decision of the Court of Appeals, the heirs of Donata
elevated the case before the SC. SC reversed the decisions of CA and RTC and dismissed the Complaint
for partition, annulment, and recovery of possession of real property filed by the heirs of Maximino in
Civil Case No. CEB-5794.

ISSUE: Whether or not a judgment awarding ownership of the properties included in the decedent’s
estate to his surviving wife may be assailed on the ground of fraud after more than 30 years had lapse
from the promulgation of the said judgment.

RULING: The answer is in the negative. The heirs of Maximino failed to prove by clear and convincing
evidence that Donata managed, through fraud, to have the real properties, belonging to the intestate estate
of Maximino, registered in her name. In the absence of fraud, no implied trust was established between
Donata and the heirs of Maximino under Article 1456 of the New Civil Code. Donata was able to register
the real properties in her name, not through fraud or mistake, but pursuant to an Order, dated 2 October
1952, issued by the CFI in Special Proceedings No. 928-R. The CFI Order, presumed to be fairly and
regularly issued, declared Donata as the sole, absolute, and exclusive heir of Maximino; hence, making
Donata the singular owner of the entire estate of Maximino, including the real properties, and not merely
a co-owner with the other heirs of her deceased husband.

The CFI Order, dated 2 October 1952, issued in Special Proceedings No. 928-R, effectively settled the
intestate estate of Maximino by declaring Donata as the sole, absolute, and exclusive heir of her deceased
husband. The issuance by the CFI of the said Order, as well as its conduct of the entire Special
Proceedings No. 928-R, enjoy the presumption of validity pursuant to the Section 3(m) and (n) of Rule
131 of the Revised Rules of Court, reproduced below –(m) That official duty has been regularly
performed; (n) That a court, or judge acting as such, whether in the Philippines or elsewhere, was acting
in the lawful exercise of jurisdiction.

By reason of the foregoing provisions, this Court must presume, in the absence of any clear and
convincing proof to the contrary, that the CFI in Special Proceedings No. 928-R had jurisdiction of the
subject matter and the parties, and to have rendered a judgment valid in every respect; and it could not
give credence to the following statements made by the Court of Appeals in its Decision.

While it is true that since the CFI was not informed that Maximino still had surviving siblings and so the
court was not able to order that these siblings be given personal notices of the intestate proceedings, it
should be borne in mind that the settlement of estate, whether testate or intestate, is a proceeding in
rem, and that the publication in the newspapers of the filing of the application and of the date set
for the hearing of the same, in the manner prescribed by law, is a notice to the whole world of the
existence of the proceedings and of the hearing on the date and time indicated in the publication.
The publication requirement of the notice in newspapers is precisely for the purpose of informing all

Remedial Law Review II (CY 2018-2019) – Saturday 1:00pm – 5:00pm: Case Digest Compilation [Atty. Brondial] Page 199
interested parties in the estate of the deceased of the existence of the settlement proceedings, most
especially those who were not named as heirs or creditors in the petition, regardless of whether such
omission was voluntarily or involuntarily made.

Remedial Law Review II (CY 2018-2019) – Saturday 1:00pm – 5:00pm: Case Digest Compilation [Atty. Brondial] Page 200
A.10. RODOLFO C. SABIDONG, Complainant, vs. NICOLASITO S. SOLAS (Clerk of Court
IV), Respondent.

A.M.No. P-01-1448 (Formerly OCA IPI No. 99-664-P); June 25, 2013; VILLARAMA, JR., J.:

The present administrative case stemmed from a sworn letter-complaintdated May 29, 1999 filed before
this Court by Rodolfo C. Sabidong (complainant) charging respondent Nicolasito S. Solas, Clerk of Court
IV, Municipal Trial Court in Cities (MTCC), Iloilo City with grave and serious misconduct, dishonesty,
oppression and abuse of authority.

FACTS: Trinidad Sabidong, Complainant, Rodolfo’s mother, is one of the longtime occupants of the
subject property - parcel of land. The Sabidongs are in possession of one-half portion of Lot 11, subject of
an ejectment suit filed by the Hodges Estate in the MTCC Iloilo City Br. 4. On May 31, 1983, a decision
was rendered, with an order to immediately vacate and to pay the plaintiff (Hodges Estate) rentals due.  

Sometime in October 1984, Solas (Respondent), who was then the Clerk of Court, submitted an Offer to
Purchase on instalment Lots 11 and 12. The Administratrix of the Hodges Estate rejected respondent’s
offer in view of an application to purchase already filed by the actual occupant of Lot 12. The check for
initial down payment tendered by Solas was returned to him, but nevertheless informed that he may file
an offer to purchase Lot 11 and that if he could put up a sufficient down payment, the Estate could
immediately endorse it for approval of the Probate Court so that the property can be awarded to him
"should the occupant fail to avail of the priority given to them,” to this, Solas immediately complied. The
probate court (Regional Trial Court of Iloilo, Branch 27) approved, upon observation that the occupants
of the subject lots "have not manifested their desire to purchase the lots they are occupying up to this date
and considering time restraint and considering further, that the sales in favor of the x xxofferors are most
beneficial to the estate x xx". Eventually, writ of possession and title of the lot was transferred to Solas.
Later, a writ of demolition was issued by the probate court against all adverse occupants of Lot 11.

In 1999, a complaint was initiated against the Solas in the Supreme Court, alleging the prohibition for
court personnel to buy properties in litigation, which likewise alleged that the respondent committed
deception, dishonesty, oppression and grave abuse of authority. Criminal case for Estafa and Civil case
for Annulment of title were also filed but were both dismissed. SC referred the Administrative case of
Judge Hortillo (RTC Iloilo) who required Solas to file a memorandum, who, in the said memorandum
asserted that he is a buyer in good faith and for value. Sola, retired (compulsory) without his benefits
being released.

OCA found respondent Solas liable for serious and grave misconduct and dishonesty and recommended
the forfeiture of his salary for six months, to be deducted from his retirement benefits.

ISSUE/S:
1. Whether the subject property is no longer a thing in litigation and hence among those prohibited under
Art. 1491 of the Civil Code.
2. Whether Clerk of Court violated the rule on disqualification to purchase property in litigation

RULING:
1. No. In the case at bar, when respondent purchased Lot 11-A on November 21, 1994, the Decision in
Civil Case No. 14706 which was promulgated on May 31, 1983 had long become final. Be that as it may,
it can not be said that the property is no longer "in litigation" at that time considering that it was part of
the Hodges Estate then under settlement proceedings (Sp. Proc. No. 1672)

A thing is said to be in litigation not only if there is some contest or litigation over it in court, but also
from the moment that it becomes subject to the judicial action of the judge. A property forming part of the
estate under judicial settlement continues to be subject of litigation until the probate court issues an order
declaring the estate proceedings closed and terminated. The rule is that as long as the order for the
distribution of the estate has not been complied with, the probate proceedings cannot be deemed closed
and terminated. The probate court loses jurisdiction of an estate under administration only after the
payment of all the debts and the remaining estate delivered to the heirs entitled to receive the same. Since
there is no evidence to show that Sp. Proc. No. 1672 in the RTC of Iloilo, Branch 27, had already been
closed and terminated at the time of the execution of the Deed of Sale With Mortgage dated November
21, 1994, Lot 11 is still deemed to be "in litigation" subject to the operation of Article 1491 (5) of the
Civil Code.

Remedial Law Review II (CY 2018-2019) – Saturday 1:00pm – 5:00pm: Case Digest Compilation [Atty. Brondial] Page 201
This notwithstanding, we hold that the sale of Lot 11 in favor of respondent did not violate the rule on
disqualification to purchase property because Sp. Proc. No. 1672 was then pending before another court
(RTC) and not MTCC where he was Clerk of Court.

2. No. Article 1491, paragraph 5 of the Civil Code prohibits court officers such as clerks of court from
acquiring property involved in litigation within the jurisdiction or territory of their courts. Said provision
reads: Article 1491. The following persons cannot acquire by purchase, even at a public or judicial
auction, either in person or through the mediation of another:
x xxx
(5) Justices, judges, prosecuting attorneys, clerks of superior and inferior courts, and other officers and
employees connected with the administration of justice, the property and rights in litigation or levied
upon an execution before the court within whose jurisdiction or territory they exercise their respective
functions; this prohibition includes the act of acquiring by assignment and shall apply to lawyers, with
respect to the property and rights which may be the object of any litigation in which they may take part by
virtue of their profession.
x xxx (Emphasis supplied.)

The rationale advanced for the prohibition is that public policy disallows the transactions in view of the
fiduciary relationship involved, i.e., the relation of trust and confidence and the peculiar control exercised
by these persons. "In so providing, the Code tends to prevent fraud, or more precisely, tends not to give
occasion for fraud, which is what can and must be done."

For the prohibition to apply, the sale or assignment of the property must take place during the pendency
of the litigation involving the property. Where the property is acquired after the termination of the case,
no violation of paragraph 5, Article 1491 of the Civil Code attaches.

WHEREFORE, the Court finds respondent Nicolasito S. Solas, retired Clerk of Court IV, Municipal Trial
Court in Cities, Iloilo City, LIABLE FOR GRAVE MISCONDUCT AND DISHONESTY. Respondent is
FINED in an amount equivalent to his salary for six (6) months to be deducted from his retirement
benefits. SO ORDERED.

DOCTRINE: Article 1491 paragraph 5 of the Civil Code prohibits court officers such as clerks of court
from acquiring property involved in litigation within the jurisdiction or territory of their courts.

On probate proceedings – The rule is that as long as the order for the distribution of the estate has not
been complied with, the probate proceedings cannot be deemed closed and terminated.

Remedial Law Review II (CY 2018-2019) – Saturday 1:00pm – 5:00pm: Case Digest Compilation [Atty. Brondial] Page 202
A.11. THELMA M. ARANAS,Petitioner, v.TERESITA V. MERCADO, FELIMON V.
MERCADO, CARMENCITA M. SUTHERLAND, RICHARD V. MERCADO, MA. TERESITA
M. ANDERSON, AND FRANKLIN L. MERCADO, Respondents.

G.R. No. 156407, January 15, 2014; Bersamin, J.

FACTS: Emigdio S. Mercado (Emigdio) died intestate in 1991, survived by his second wife, Teresita
(appointed administrator), and their five children, namely: Allan, Felimon, Carmencita, Richard, and
Maria Teresita; and his two children by his first marriage, namely: respondent Franklin and petitioner
Thelma M. Aranas (Thelma).

Emigdio inherited and acquired real properties during his lifetime.

Teresita submitted an inventory of the estate for the consideration and approval by the RTC, indicating
that at the time of his death, Emigdio had “left no real properties but only personal properties” worth
P6,675,435.25 in all. Claiming that Emigdio had owned other properties, excluded from the inventory,
Thelma moved for the amendment of the inventory, which the RTC granted. Teresita complied with the
order, supporting her inventory with copies of three certificates of stocks covering Mervir Realty shares
of stock; deed of assignments, involving real properties; and the certificate of stock of Cebu Emerson.

The RTC issued an order expressing the need for the parties to present evidence and for Teresita to be
examined to enable the court to resolve the motion for approval of the inventory. Thelma opposed. With
the parties agreeing to submit themselves to the jurisdiction of the court on the issue of what properties
should be included in or excluded from the inventory, the RTC set dates for the hearing on that issue.

Ruling of the RTC: After a series of hearings that ran for almost eight years, the RTC issued an order
finding and holding that the inventory submitted excluded properties that should be included, denying the
approval of inventory and orders the administratrix to re–do the inventory of properties

Teresita, joined by other heirs, timely sought the reconsideration (opposed by Thelma) on the ground that
one of the real properties affected, had already been sold to Mervir Realty, and that the parcels of land
covered by the deed of assignment had already come into the possession of and registered in the name of
Mervir Realty.The RTC denied the MR.

The CA partly granted the petition for certiorari, in favour of Teresita, et al.

ISSUES:
1. Whether certiorari was the proper recourse to assail the questioned orders of the RTC.
2. Whether the RTC commit grave abuse of discretion in directing the inclusion of the properties in
the estate of the decedent?
3. Whether the properties should be included in the inventory.

RULING:
1. Was certiorari the proper recourse to assail the questioned orders of the RTC?

Yes. The propriety of the special civil action for certiorari as a remedy depended on whether the assailed
orders of the RTC were final or interlocutory in nature.

The assailed order of March 14, 2001 denying Teresita’s motion for the approval of the inventory and the
order dated May 18, 2001 denying her motion for reconsideration were interlocutory. This is because the
inclusion of the properties in the inventory was not yet a final determination of their ownership. Hence,
the approval of the inventory and the concomitant determination of the ownership as basis for inclusion or
exclusion from the inventory were provisional and subject to revision at anytime during the course of the
administration proceedings.

2. Did the RTC commit grave abuse of discretion in directing the inclusion of the properties in the
estate of the decedent?

No. The determination of which properties should be excluded from or included in the inventory of estate
properties was well within the authority and discretion of the RTC as an intestate court. In making its
determination, the RTC acted with circumspection, and proceeded under the guiding policy that it was
best to include all properties in the possession of the administrator or were known to the administrator to

Remedial Law Review II (CY 2018-2019) – Saturday 1:00pm – 5:00pm: Case Digest Compilation [Atty. Brondial] Page 203
belong to Emigdio rather than to exclude properties that could turn out in the end to be actually part of the
estate. As long as the RTC commits no patent grave abuse of discretion, its orders must be respected as
part of the regular performance of its judicial duty. Grave abuse of discretion means either that the
judicial or quasi–judicial power was exercised in an arbitrary or despotic manner by reason of passion or
personal hostility, or that the respondent judge, tribunal or board evaded a positive duty, or virtually
refused to perform the duty enjoined or to act in contemplation of law, such as when such judge, tribunal
or board exercising judicial or quasi–judicial powers acted in a capricious or whimsical manner as to be
equivalent to lack of jurisdiction.

It is unavoidable to find that the CA, in reaching its conclusion, ignored the law and the facts that had
fully warranted the assailed orders of the RTC.

Under Section 6(a), Rule 78 of the Rules of Court, the letters of administration may be granted at the
discretion of the court to the surviving spouse, who is competent and willing to serve when the person
dies intestate. Upon issuing the letters of administration to the surviving spouse, the RTC becomes duty–
bound to direct the preparation and submission of the inventory of the properties of the estate, and the
surviving spouse, as the administrator, has the duty and responsibility to submit the inventory within three
months from the issuance of letters of administration pursuant to Rule 83 of the Rules of Court, viz:

Section 1.Inventory and appraisal to be returned within three months. – Within three (3) months after his
appointment every executor or administrator shall return to the court a true inventory and appraisal of all
the real and personal estate of the deceased which has come into his possession or knowledge. In the
appraisement of such estate, the court may order one or more of the inheritance tax appraisers to give his
or their assistance.

3. Whether the properties should be included in the inventories.

Yes. The usage of the word all in Section 1, supra, demands the inclusion of all the real and personal
properties of the decedent in the inventory. However, the word all is qualified by the phrase which has
come into his possession or knowledge, which signifies that the properties must be known to the
administrator to belong to the decedent or are in her possession as the administrator. Section 1 allows no
exception, for the phrase true inventory implies that no properties appearing to belong to the decedent can
be excluded from the inventory, regardless of their being in the possession of another person or entity.

It is clear to us that the RTC took pains to explain the factual bases for its directive for the inclusion of the
properties in question in its assailed order of March 14, 2001, viz:

In the first place, the administratrix of the estate admitted that Emigdio Mercado was one of the heirs of
Severina Mercado who, upon her death, left several properties as listed in the inventory of properties
submitted in Court in Special Proceedings No. 306–R which are supposed to be divided among her heirs.
The administratrix admitted, while being examined in Court by the counsel for the petitioner, that she did
not include in the inventory submitted by her in this case the shares of Emigdio Mercado in the said estate
of Severina Mercado. Certainly, said properties constituting Emigdio Mercado’s share in the estate of
Severina Mercado should be included in the inventory of properties required to be submitted to the
Court in this particular case.

In the second place, the administratrix of the estate of Emigdio Mercado also admitted in Court that she
did not include in the inventory shares of stock of Mervir Realty Corporation which are in her name and
which were paid by her from money derived from the taxicab business which she and her husband had
since 1955 as a conjugal undertaking. As these shares of stock partake of being conjugal in character,
one–half thereof or of the value thereof should be included in the inventory of the estate of her
husband.

In the third place, the administratrix of the estate of Emigdio Mercado admitted, too, in Court that she had
a bank account in her name at Union Bank which she opened when her husband was still alive. Again,
the money in said bank account partakes of being conjugal in character, and so, one–half thereof
should be included in the inventory of the properties constituting as estate of her husband.

In the fourth place, it has been established during the hearing in this case that Lot No. 3353 of Pls–657–D
located in Badian, Cebu containing an area of 53,301 square meters as described in and covered by
Transfer Certificate of Title No. 3252 of the Registry of Deeds for the Province of Cebu is still registered
in the name of Emigdio S. Mercado until now. When it was the subject of Civil Case No. CEB–12690

Remedial Law Review II (CY 2018-2019) – Saturday 1:00pm – 5:00pm: Case Digest Compilation [Atty. Brondial] Page 204
which was decided on October 19, 1995, it was the estate of the late Emigdio Mercado which claimed to
be the owner thereof. Mervir Realty Corporation never intervened in the said case in order to be the
owner thereof. This fact was admitted by Richard Mercado himself when he testified in Court. x xxSo the
said property located in Badian, Cebu should be included in the inventory in this case.

Fifthly and lastly, it appears that the assignment of several parcels of land by the late Emigdio S. Mercado
to Mervir Realty Corporation on January 10, 1991 by virtue of the Deed of Assignment signed by him on
the said day (Exhibit N for the petitioner and Exhibit 5 for the administratrix) was a transfer in
contemplation of death. It was made two days before he died on January 12, 1991. A transfer made in
contemplation of death is one prompted by the thought that the transferor has not long to live and made in
place of a testamentary disposition (1959 Prentice Hall, p. 3909). Section 78 of the National Internal
Revenue Code of 1977 provides that the gross estate of the decedent shall be determined by including the
value at the time of his death of all property to the extent of any interest therein of which the decedent has
at any time made a transfer in contemplation of death. So, the inventory to be approved in this case
should still include the said properties of Emigdio Mercado which were transferred by him in
contemplation of death. Besides, the said properties actually appeared to be still registered in the name
of Emigdio S. Mercado at least ten (10) months after his death, as shown by the certification issued by the
Cebu City Assessor’s Office on October 31, 1991 (Exhibit O).28

The objective of the Rules of Court in requiring the inventory and appraisal of the estate of the decedent
is “to aid the court in revising the accounts and determining the liabilities of the executor or the
administrator, and in making a final and equitable distribution (partition) of the estate and otherwise to
facilitate the administration of the estate.” Hence, the RTC that presides over the administration of an
estate is vested with wide discretion on the question of what properties should be included in the
inventory.

WHEREFORE, the Court GRANTS the petition for review on certiorari; REVERSES and SETS ASIDE
the decision promulgated on May 15, 2002; REINSTATES the orders issued on March 14, 2001 and May
18, 2001 by the Regional Trial Court in Cebu; DIRECTS the Regional Trial Court in Cebu to proceed
with dispatch in Special Proceedings No. 3094–CEB entitled Intestate Estate of the late Emigdio
Mercado, Thelma Aranas, petitioner, and to resolve the case; and ORDERS the respondents to pay the
costs of suit.

DOCTRINES:

1. The assailed order of March 14, 2001 denying Teresita’s motion for the approval of the inventory and
the order dated May 18, 2001 denying her motion for reconsideration were interlocutory. This is because
the inclusion of the properties in the inventory was not yet a final determination of their ownership.
Hence, the approval of the inventory and the concomitant determination of the ownership as basis for
inclusion or exclusion from the inventory were provisional and subject to revision at anytime during the
course of the administration proceedings.

2. An appeal would not be the correct recourse for Teresita, et al. to take against the assailed orders.
The final judgment rule embodied in the first paragraph of Section 1, Rule 41, Rules of Court, which
also governs appeals in special proceedings, stipulates that only the judgments, final orders (and
resolutions) of a court of law “that completely disposes of the case, or of a particular matter therein
when declared by these Rules to be appealable” may be the subject of an appeal in due course. The
same rule states that an interlocutory order or resolution (interlocutory because it deals with
preliminary matters, or that the trial on the merits is yet to be held and the judgment rendered) is
expressly made non-appealable.

Multiple appeals are permitted in special proceedings as a practical recognition of the possibility that
material issues may be finally determined at various stages of the special proceedings. Section 1, Rule
109 of the Rules of Court enumerates the specific instances in which multiple appeals may be resorted
to in special proceedings, viz.: Section 1. Orders or judgments from which appeals may be taken.—An
interested person may appeal in special proceedings from an order or judgment rendered by a Court of
First Instance or a Juvenile and Domestic Relations Court, where such order or judgment: (a) Allows or
disallows a will; (b) Determines who are the lawful heirs of a deceased person, or the distributive share
of the estate to which such person is entitled; (c) Allows or disallows, in whole or in part, any claim
against the estate of a deceased person, or any claim presented on behalf of the estate in offset to a claim
against it; (d) Settles the account of an executor, administrator, trustee or guardian; (e) Constitutes, in
proceedings relating to the settlement of the estate of a deceased person, or the administration of a

Remedial Law Review II (CY 2018-2019) – Saturday 1:00pm – 5:00pm: Case Digest Compilation [Atty. Brondial] Page 205
trustee or guardian, a final determination in the lower court of the rights of the party appealing, except
that no appeal shall be allowed from the appointment of a special administrator; and (f) Is the final order
or judgment rendered in the case, and affects the substantial rights of the person appealing, unless it be
an order granting or denying a motion for a new trial or for reconsideration.

3. Under Section 6(a), Rule 78 of the Rules of Court, the letters of administration may be granted at the
discretion of the court to the surviving spouse, who is competent and willing to serve when the person
dies intestate. Upon issuing the letters of administration to the surviving spouse, the RTC becomes duty-
bound to direct the preparation and submission of the inventory of the properties of the estate, and the
surviving spouse, as the administrator, has the duty and responsibility to submit the inventory within
three months from the issuance of letters of administration pursuant to Rule 83 of the Rules of Court.
4. There is no dispute that the jurisdiction of the trial court as an intestate court is special and limited.
The trial court cannot adjudicate title to properties claimed to be a part of the estate but are claimed to
belong to third parties by title adverse to that of the decedent and the estate, not by virtue of any right of
inheritance from the decedent. All that the trial court can do regarding said properties is to determine
whether or not they should be included in the inventory of properties to be administered by the
administrator. Such determination is provisional and may be still revised

Remedial Law Review II (CY 2018-2019) – Saturday 1:00pm – 5:00pm: Case Digest Compilation [Atty. Brondial] Page 206
A.12. SPOUSES MARIA BUTIONG and VILLAFRlA, DR. RUEL B. SPOUSES MARIA
FRANCISCO substituted by VILLAFRIA, Petitioners, vs. MA. GRACIA RINOZA PLAZO and
MA. FE RINOZA ALARAS, Respondents.

G.R. No. 187524; August 5, 2015; PERALTA, J.:

FACTS: Pedro L. Rifioza died intestate, leaving several heirs, including his children with his first wife,
respondents Ma. Gracia R. Plazo and Ma.Fe Alaras, as well as several properties including a resort, and a
family home. In their Amended Complaint for Judicial Partition with Annulment of Title and Recovery of
Possession, respondents alleged that they discovered that their co-heirs, Pedro’s second wife,
Benita"Tenorio and other children, had sold the subject properties to petitioners, spouses Francisco
Villafria and Maria Butiong, who are now deceased and substituted by their son, Dr.Ruel B. Villafria,
without their knowledge and consent. When confronted about the sale, Benita acknowledged the same,
showing respondents a document she believed evidenced receipt of her share in the sale, which, however,
did not refer to any sort of sale but to a previous loan obtained by Pedro and Benita from a bank. Upon
inquiry, the Register of Deeds of Nasugbu informed respondents that he has no record of any transaction
involving the subject properties, giving them certified true copies of the titles to the same.

Subsequently, respondents learned a notice of an extra-judicial settlement of estate of their late father was
published in a tabloid called Balita. Because of this, they caused the annotation of their adverse claims
over the subject properties before the Register of Deeds of Nasugbu and filed their complaint praying,
among others, for the annulment of all documents conveying the subject properties to the petitioners and
certificates of title issued pursuant thereto.

The trial court nullified the transfer of the subject Properties to petitioners and spouses Bondoc due to
irregularities in the Documents of conveyance offered by petitioner’s as well as the circumstances
Surrounding the execution of the same. Specifically, the Extra-Judicial Settlement was notarized by a
notary public that was not duly commissioned as such on the date it was executed. The Deed of Sale was
Undated, the date of the acknowledgment therein was left blank, and the Typewritten name "Pedro
Rifioza, Husband" on the left side of the document was not signed. The trial court also observed that both
documents were never presented to the Office of the Register of Deeds for registration and that the titles
to the subject properties were still in the names of Pedro and His second wife Benita. In addition, the
supposed notaries and buyers of the Subject properties were not even presented as witnesses whom
supposedly witnessed the signing and execution of the documents of conveyance. On The basis thereof,
the triaI court ruled in favor of respondents. On appeal, the CA affirmed the trial ‘court’s Judgment.

Aggrieved, petitioners, substituted by their son RuelVillafria, filed a Motion for Reconsideration dated
November 24, 2006 raising the trial court’s lack of jurisdiction. It was alleged that when the Complaint
for Judicial Partition with Annulment of Title and Recovery of Possession was filed, there was yet no
settlement of Pedro's estate, determination as to the nature thereof, nor was there an identification of the
number of legitimate heirs. As such, the trial court ruled on the settlement of the intestate estate of Pedro
in its ordinary· jurisdiction when the action filed was for Judicial Partition. Considering that the instant
action is really one for settlement of intestate estate, the trial court, sitting merely in its probate
jurisdiction, exceeded its jurisdiction when it ruled upon the issues of forgery and ownership. Thus,
petitioner argued that. Said ruling is void and has no effect for having been rendered without jurisdiction.
The Motion for Reconsideration was, however, denied by the appellate court. When the appellate court
denied Petitioner’s Motion for Reconsideration in its Resolution, petitioner filed the instant Petition for
Review on Certiorari.

ISSUE: Whether the respondents committed no error in filing an action for judicial partition instead of a
special proceeding for the settlement of estate.

RULING: The petition is bereft of merit. Petitioner is mistaken. It is true that some of respondents'
causes of action pertaining to the properties left behind by the decedent Pedro, his known heirs, and the
nature and extent of their interests thereon may fall under an action for settlement of estate. However, a
complete reading of the complaint would readily show that, based on the nature of the suit, the allegations
therein, and the relief’s prayed for, the action, is clearly one for judicial partition with annulment of title
and recovery of possession.

DOCTRINE:
General Rule: When a person dies intestate, or, if testate, failed to name an executor in his will or the
executor named is incompetent, or refuses the trust, or fails to furnish the bond equipped by the Rules of

Remedial Law Review II (CY 2018-2019) – Saturday 1:00pm – 5:00pm: Case Digest Compilation [Atty. Brondial] Page 207
Court, then the decedent's estate shall be judicially administered and the competent court shall appoint a
qualified administrator the order established in Section 6 of Rule 78 of the Rules of Court.

Exception: Section 1 of Rule 74 wherein the heirs of a decedent, who left no will and no debts due from
the estate, may divide the estate either extrajudicially or in an ordinary action or partition without
submitting the same for judicial administration nor applying for the appointment of an administrator by
the court. The reasons that where the deceased dies without pending obligations, there is no necessity for
the appointment of an administrator to administer the estate for them and to deprive the real owners of
their possession to which they are immediately entitled.

Section 1, Rule 74 of the Revised Rules of Court, however, does not preclude the heirs from instituting
administration proceedings, even if the estate has no debts or obligations, if they do not desire to resort
for good reasons to an ordinary action for partition. While Section 1 allows the heirs to divide the estate
among themselves as they may see fit, or to resort to an ordinary action for partition, the said provision
does not compel them to do so if they have good reasons to take a different course of action. It should be
noted that recourse to an administration proceeding even if the estate has no debts is sanctioned only if
the heirs have good reasons for not resorting to an action for partition. Where partition is possible, either
in or out of court, the estate should not be burdened with an administration proceeding without good and
compelling reasons.

Thus, it has been repeatedly held that when a person dies without leaving pending obligations to be paid,
his heirs, whether of age or not, are not bound to submit the property to a judicial administration, which
is always long and costly, or to apply for the appointment of an administrator by the Court. It has been
uniformly held that in such case the judicial administration and the appointment of an administrator are
superfluous and unnecessary proceedings.

Thus, respondents committed no error in filing an action for judicial partition instead of a special
proceeding for the settlement of estate as law expressly permits the same. That the complaint contained
allegations inherent in an action for settlement of estate does not mean that there was a prohibited joined
of causes of action for questions as to the estate's properties as well as a determination of the heirs, their
status as such, and the nature and extent of their titles to the estate, may also be properly ventilated in
partition proceedings alone. In fact, a complete inventory of the estate may likewise be done during the
partition proceedings, especially since the estate has no debts. Indeed, where the more expeditious
remedy of partition is available to the heirs, then they may not be compelled to submit to administration
proceedings, dispensing of the risks of delay and of the properties being dissipated.

Remedial Law Review II (CY 2018-2019) – Saturday 1:00pm – 5:00pm: Case Digest Compilation [Atty. Brondial] Page 208
B. Escheats (Rule 91)

B.1. CASTORIO ALVARICO vs. AMELITA L. SOLA

G.R. No. 138953; June 6, 2002; QUISUMBING, J.:

FACTS: Petitioner CastorioAlvarico is the natural father of respondent Amelita Sola while Fermina
Lopez is petitioner’s aunt, and also Amelita’s adoptive mother.On June 17, 1982, the Bureau of Lands
approved and granted the Miscellaneous Sales Application (MSA) of Fermina over Lot 5, SGS-3451,
with an area of 152 sq. m. at the Waterfront, Cebu City.

On May 28, 1983, Fermina executed a Deed of Self-Adjudication and Transfer of Rights over Lot 5 in
favor of Amelita, who agreed to assume all the obligations, duties, and conditions imposed upon Fermina
under MSA Application No. V-81066. The document of transfer was filed with the Bureau of Lands.
Amelita assumed payment of the lot to the Bureau of Lands.

On April 7, 1989, the Bureau of Lands issued an order approving the transfer of rights and granting the
amendment of the application from Fermina to Amelita. On May 2, 1989, Original Certificate of Title
(OCT) No. 3439 was issued in favor of Amelita.

On June 24, 1993, herein petitioner filed a civil case for reconveyance against Amelita. He claimed that
on January 4, 1984, Fermina donated the land to him and immediately thereafter, he took possession of
the same. He averred that the donation to him had the effect of withdrawing the earlier transfer to
Amelita. For her part, Amelita maintained that the donation to petitioner is void because Fermina was no
longer the owner of the property when it was allegedly donated to petitioner, the property having been
transferred earlier to her. She added that the donation was void because of lack of approval from the
Bureau of Lands, and that she had validly acquired the land as Fermina’s rightful heir. She also denied
that she is a trustee of the land for petitioner.

RTC: Rendered a decision in favor of petitioner. Lot 5, Sgs-3451, is hereby declared as lawfully owned
by plaintiff and defendant is directed to reconvey the same to the former.
CA: Reversed the RTC.

ISSUE: In an action for reconveyance, who between petitioner and respondent has a better claim to the
land?

RULING: Given the circumstances in this case and the contentions of the parties, we find that no
reversible error was committed by the appellate court in holding that herein petitioner’s complaint against
respondent should be dismissed. The evidence on record and the applicable law indubitably favor
respondent.

Petitioner claims that respondent was in bad faith when she registered the land in her name and, based on
Articles 744 and 1544 of the New Civil Code, he has a better right over the property because he was first
in material possession in good faith. However, this allegation of bad faith on the part of Amelita Sola in
acquiring the title is devoid of evidentiary support. For one, the execution of public documents, as in the
case of Affidavits of Adjudication, is entitled to the presumption of regularity, hence convincing evidence
is required to assail and controvert them. Second, it is undisputed that OCT No. 3439 was issued in 1989
in the name of Amelita. It requires more than petitioners bare allegation to defeat the Original Certificate
of Title which on its face enjoys the legal presumption of regularity of issuance. A Torrens title, once
registered, serves as notice to the whole world. All persons must take notice and no one can plead
ignorance of its registration.

Even assuming that respondent Amelita Sola acquired title to the disputed property in bad faith, only the
State can institute reversion proceedings under Sec. 101 of the Public Land Act. Thus:
Sec. 101.All actions for reversion to the Government of lands of the public domain or
improvements thereon shall be instituted by the Solicitor General or the officer acting in his
stead, in the proper courts, in the name of the Republic of the Philippines.

In other words, a private individual may not bring an action for reversion or any action which would have
the effect of canceling a free patent and the corresponding certificate of title issued on the basis thereof,
such that the land covered thereby will again form part of the public domain. Only the Solicitor General
or the officer acting in his stead may do so. Since AmelitaSolas’ title originated from a grant by the

Remedial Law Review II (CY 2018-2019) – Saturday 1:00pm – 5:00pm: Case Digest Compilation [Atty. Brondial] Page 209
government, its cancellation is a matter between the grantor and the grantee. Clearly then, petitioner has
no standing at all to question the validity of Amelita’s title. It follows that he cannot recover the property
because, to begin with, he has not shown that he is the rightful owner thereof.

Anent petitioner’s contention that it was the intention of Fermina for Amelita to hold the property in trust
for him, we held that if this was really the intention of Fermina, then this should have been clearly stated
in the Deed of Self-Adjudication executed in 1983, in the Deed of Donation executed in 1984, or in a
subsequent instrument. Absent any persuasive proof of that intention in any written instrument, we are not
prepared to accept petitioner’s bare allegation concerning the donor’s state of mind. The decision of the
CA is AFFIRMED. The complaint filed by petitioner against respondent is declared properly
DISMISSED.

DOCTRINE: Reversion; Only the State can institute reversion proceedings under Section 101 of the
Public Land Act.— Even assuming that respondent Amelita Sola acquired title to the disputed property
in bad faith, only the State can institute reversion proceedings under Sec. 101 of the Public Land Act.
Thus: Sec. 101.—All actions for reversion to the Government of lands of the public domain or
improvements thereon shall be instituted by the Solicitor General or the officer acting in his stead, in the
proper courts, in the name of the Republic of the Philippines.

A private individual may not bring an action for reversion or any action which would have the effect of
canceling a free patent and the corresponding certificate of title issued on the basis thereof, such that
the land covered thereby will again form part of the public domain.—In other words, a private
individual may not bring an action for reversion or any action which would have the effect of canceling a
free patent and the corresponding certificate of title issued on the basis thereof, such that the land
covered thereby will again form part of the public domain. Only the Solicitor General or the officer
acting in his stead may do so. Since AmelitaSola’s title originated from a grant by the government, its
cancellation is a matter between the grantor and the grantee. Clearly then, petitioner has no standing at
all to question the validity of Amelita’s title. It follows that he cannot “recover” the property because, to
begin with, he has not shown that he is the rightful owner thereof.

Remedial Law Review II (CY 2018-2019) – Saturday 1:00pm – 5:00pm: Case Digest Compilation [Atty. Brondial] Page 210
B.2. Maltos vs. Heirs of EusebioBorromeo

G.R. No. 172720; September, 14, 2015; Leonen, J.

FACTS: On February 13, 1979, Eusebio Borromeo (Borromeo) was issued Free Patent over a piece of
agricultural land located in San Francisco, Agusan del Sur. On June 15, 1983, well within the five-year
prohibitory period, Borromeo sold the land to Eliseo Maltos (Maltos). Borromeo died on January 16,
1991. His heirs claimed that prior to his death, he allegedly told his wife, Norberta and his children to
nullify the sale made to Maltos because the sale was within the five-year prohibitory period. On June 23,
1993, Norberta and her children (heirs of Borromeo) filed a Complaint for Nullity of Title and
Reconveyance of Title against Maltos.

Eliseo Maltos and Rosita Maltos (Maltos Spouses) filed their Answer, arguing that the sale was made in
good faith and that in purchasing the property, they relied on Eusebio Borromeo's title. Further, the parties
were in pari delicto. Since the sale was made during the five-year prohibitory period, the land would
revert to the public domain and the proper party to institute reversion proceedings was the Office of the
Solicitor General.

The Register of Deeds of Agusan del Sur also filed an Answer, arguing that the deed of sale was
presented for Registration after the five-year prohibitory period, thus, it was ministerial on its part to
register the deed. The heirs of Borromeo countered that good faith was not a valid defense because the
prohibitory period appeared on the face of the title of the property.

The trial court dismissed the Complaint on the ground of failure to state a cause of action. Also, the heirs
of Borromeo did not have a right of action because they were unable to establish their status as heirs of
the late Eusebio Borromeo. They may have declared themselves the legal heirs of Eusebio Borromeo, but
they did not present evidence to prove their allegation. Further, the determination of their rights to
succession must be established in special proceedings.

The trial court also ruled that "[t]he sale was null and void because it was within the five (5) year
prohibitionary [sic] period" under the Public Land Act. The defense of indefeasibility of title was
unavailing because the title to the property stated that it was "subject to the provisions of Sections 118,
119, 121, 122 and 124" of the Public Land Act. Since the property was sold within the five-year
prohibitory period, such transfer "result[ed] in the cancellation of the grant and the reversion of the land
to the public domain."

The Court of Appeals reversed the Decision of the trial court and held that since Eusebio Borromeo sold
his property within the five-year prohibitory period, the property should revert to the state. However, the
government has to file an action for reversion because "reversion is not automatic." While there is yet no
action for reversion instituted by the Office of the Solicitor General, the property should be returned to
the heirs of Borromeo.

On May 10, 2006, the Maltos Spouses |filed a Petition for Review before this court, questioning the
Decision and Resolution of the Court of Appeals.

ISSUES:
(1)Whether the Court of Appeals erred in reversing the Decision of the trial court and ordering the
reconveyance of the property from petitioners Spouses Eliseo Maltos and Rosita Maltos to respondents
heirs of Eusebio Borromeo.

(2)Whether the Court of Appeals erred in ruling that petitioners Spouses Eliseo Maltos and Rosita Maltos
are not entitled to reimbursement for the improvements they introduced on the land.

RULING:
(1)Whether the Court of Appeals erred in reversing the Decision of the trial court and ordering the
reconveyance of the property from petitioners Spouses Eliseo Maltos and Rosita Maltos to respondents
heirs of Eusebio Borromeo.
No, the CA did not err.

Remedial Law Review II (CY 2018-2019) – Saturday 1:00pm – 5:00pm: Case Digest Compilation [Atty. Brondial] Page 211
[T]he main purpose in the grant of a free patent of homestead is to preserve and keep in the family of the
homesteader that portion of public land which the State has given to him so he may have a place to live
with his family and become a happy citizen and a useful member of the society. In Jocson v. Soriano, we
held that the conservation of a family home is the purpose of homestead laws. The policy of the state is to
foster, families as the foundation of society, and thus promote general welfare.

The effect of violating the five-year prohibitory period is provided under Section 124 of the Public Land
Act, which provides:

SECTION 124. Any acquisition, conveyance, alienation, transfer, or other contract made or executed in
violation of any of the provisions of sections one hundred and eighteen, one hundred and twenty, one
hundred and twenty-one, one hundred and twenty-two, and one hundred and twenty-three of this. Act
shall be unlawful and null and void from its execution and shall produce the effect of annulling and
cancelling the grant, title, patent, or permit originally issued, recognized or confirmed, actually or
presumptively, and cause the reversion of the property and its improvements to the State.

In this case, Section 10187 of the Public Land Act is applicable since title already vested in Eusebio
Borromeo's name. Both the trial court and the Court of Appeals found that the sale was made within the
five-year prohibitory period. Thus, there is sufficient cause to revert the property in favor of the state.
However, this court cannot declare reversion of the property in favor of the state in view of the limitation
imposed by Section 101 that an action for reversion must first be filed by the Office of the Solicitor
General.

Moreover, this court held that:

Section 124 of the Public Land Act indeed provides that any acquisition, conveyance or transfer executed
in violation of any of its provisions shall be null and void and shall produce the effect of annulling and
cancelling the grant or patent and cause the reversion of the property to the State, and the principle of
pari delicto has been applied by this Court in a number of cases wherein the parties to a transaction have
proven to be guilty of effected the transaction with knowledge of the cause of its invalidity. But we doubt
if these principles can now be invoked considering the philosophy and the policy behind the approval of
the Public Land Act. The principle underlying pari delicto as known here and in the United States is not
absolute in its application. It recognizes certain exceptions one of them being when its enforcement or
application runs counter to an avowed fundamental policy or to public interest. As stated by us in the
Rellosa case, "This doctrine is subject to one important limitation, namely, [']whenever public policy is
considered advanced by allowing either party to sue for relief against the transaction[']"

The case under consideration comes within the exception above adverted to. Here appellee desires to
nullify a transaction which was done in violation of the law. Ordinarily the principle of pari delicto
would apply to her because her predecessor-in-interest has carried out the sale with the presumed
knowledge of its illegality, but because the subject of the transaction is a piece of public land, public
policy requires that she, as heir, be not prevented from re-acquiring it because it was given by law to her
family for her home and cultivation. This is the policy on which our homestead law is predicated. This
right cannot be waived. "It is not within the competence of any citizen to barter away what public policy
by law seeks to preserve." We are, therefore, constrained to hold that appellee can maintain the present
action it being in furtherance of this fundamental aim of our homestead law.(Emphasis supplied, citations
omitted)

As the in pari delicto rule is not applicable, the question now arises as to who between the parties have a
better right to possess the subject parcel of land. This issue was addressed in Santos:

What is important to consider now is who of the parties is the better entitled to the possession of the land
while the government does not take steps to assert its title to the homestead. Upon annulment of the sale,
the purchaser's claim is reduced to the purchase price and its interest. As against the vendor or his heirs,
the purchaser is no more entitled to keep the land than any intruder. Such is the situation of the
appellants. Their right to remain in possession of the land is no better than that of appellee and,
therefore, they should not be allowed to remain in it to the prejudice of appellee during and until the
government takes steps toward its reversion to the State. (Emphasis supplied, citation omitted)

In Binayug v. Ugaddan, which involved the sale of two properties covered by a homestead patent, this
court cited jurisprudence showing that in cases involving the sale of a property covered by the five-year
prohibitory period, the property should be returned to the grantee.

Remedial Law Review II (CY 2018-2019) – Saturday 1:00pm – 5:00pm: Case Digest Compilation [Atty. Brondial] Page 212
Applying the ruling in Santos and Binayug, this court makes it clear that petitioners have no better right to
remain in possession of the property against respondents.

Hence, the Court of Appeals did not err in ruling that while there is yet no action for reversion filed by the
Office of the Solicitor General, the property should be conveyed by petitioners to respondents.

(2)Whether the Court of Appeals erred in ruling that petitioners Spouses Eliseo Maltos and Rosita Maltos
are not entitled to reimbursement for the improvements they introduced on the land.
No, the CA did not err.

With regard to the claim for reimbursement, respondents argue that it was not raised as a counterclaim in
the Answer to the Complaint.

During trial, petitioner Eliseo Maltos testified that when he entered the land, there were around 100 trees,
including coconut trees and a few banana trees. He then planted additional coconut trees which, at the
time of the trial, were already bearing fruit. Petitioner Eliseo Maltos' testimony was not rebutted by
respondents.

The general rule is that "[a] compulsory counterclaim . . . not set up shall be barred. Further, the
computation of the value of the improvements on the land entails findings of fact.

In any case, the Court of Appeals did not err when it stated in its Resolution dated April 7, 2006 that:

With respect to Appellees' claim for the reimbursement of the improvements on the land in question, they
are hereby declared to have lost and forfeited the value of the necessary improvements that they made
thereon in the same manner that Appellants should lose the value of the products gathered by the
Appellees from the said land.

Restating the rulings in Angeles and Arsenal, this court finds that while the rule on in pari delicto does
not apply policy, if its effect is to violate public policy it is applicable with regard to value of the
improvements introduced by petitioner Eliseo Maltos. Petitioners had been in possession of the land for
20 years before the heirs of Borromeo filed a Complaint. The expenses incurred by petitioners in
introducing improvements on the land for which they seek reimbursement should already be compensated
by the fruits they received from the improvements.

DOCTRINE: The sale of a parcel of agricultural land covered by a free patent during the five-year
prohibitory period under the Public Land Act is void. Reversion of the parcel of land is proper. However,
reversion under Section 101 of the Public Land Act is not automatic. The Office of the Solicitor General
must first file an action for reversion.

Remedial Law Review II (CY 2018-2019) – Saturday 1:00pm – 5:00pm: Case Digest Compilation [Atty. Brondial] Page 213
B.3. AURELIA NARCISE, GLORIA A. DELA CRUZ, MARITESS O. GARCIA, PHILIP
FALCON, ENRICO M. VITUG, LYNETTE C. PONTRERAS, BONIFACIO BARRAMEDA,
RAMON S. MORADA, MANUEL G. VIOLA, ZENAIDA LANUZA, CIRILO G. SALTO,
TEODORO DEL ROSARIO, NANCY G. INSIGNE, MELANIE G. VIANA, ROMEO TICSAY,
AMY J. FRANCISCO, MARIE J. FRANCISCO, ZENAIDA LANUZA, MIGUELITO B.
MARTINEZ, APOLONIO SANTOS, MARIVIC TAN, JANE CLOR DILEMA, VALENTINO
DILEMA, JOSE L. PANGAN, ANTONIA M. MANGELEN, IMELDA MANALASTAS,
TEODORICO N. ANDRADE, AIDA L. CRUZ, MANUEL YAMBOT, JAIME SERDENA, ARIEL
PALACIOS, EVE BOLNEO, LIBETINE MODESTO, MA. AILEEN VERDE, BENNY ILAGAN,
MICHELLE ROMANA, DANILO VILLANUEVA, LEO NALUGON, ROSSANA MARASIGAN,
NELIE BINAY AND ISABELITA MENDOZA, Petitioners, v. VALBUECO, INC., Respondent.

G.R. No. 196888; July 19, 2017; TIJAM, J.:

FACTS:Respondent Valbueco alleged that it is the possessor of the subject lots in an actual, peaceful,
adverse and peaceful possession since 1970. Respondent averred that from 1977 until 1999, Original
Certificates of Title, Free Patents and Transfer Certificates of Title covering the lots in question were
issued in the name of petitioners. Thus, Valbueco filed an action for Annulment of the Free Patents,
Certificates of Title and Damages against petitioners Narcise, et al., the Department of Natural
Resources (DENR) and the Register of Deeds of Bataan before the Regional Trial Court (RTC).

Petitioner Narcise et al. filed several Motions to Dismiss on the ground of lack of cause of action, failure
to state cause of action, defect in the certificate of non-forum shopping and prescription. The RTC
granted the motion of Narcise. The RTC ruled that the instant case is an action for reversion because
petitioners are not qualified to be issued said free patents. As such, the land must revert back to the State.
Thus, it is the Office of the Solicitor General (OSG) who is the real party-in-interest, and not the
respondent.

Respondent filed a motion for reconsideration, which was denied by the RTC. Undaunted,
respondent filed an appeal before the Court of Appeals (CA). The CA reversed and set aside the
ruling of the RTC. Thus the Narcise filed a Petition for Review on Certiorari under Rule 45.

ISSUES:
1. Whether or not the instant case is actually a reversion case, and not a case for annulment of free patents
and certificates of title?
2. Whether or not respondent is the real party-in-interest?

RULING:
1. No. We hold that the action is one of annulment of patents and titles. The allegations in the complaint
show that respondent asserts its ownership over the subject properties by acquisitive prescription.
A careful perusal of respondent's complaint reads:

“3. That the herein plaintiff has been in the actual, peaceful, adverse, continuous and
peaceful possession since sometime in 1970 and up to the present time, by itself and its
predecessor-in-interest, some of which it acquired by transfer of rights, claims, interest as
evidence [sic] by the documents x xx and the rest by occupation and planting of root crops
and other including trees x xx.

4. That the plaintiff and its workers and employees of its ranches and the cultivation and
planting of different root crops and trees were always in the premises since 1970 or
thereabouts, and their presence were never disturbed nor molested by anybody until
sometime in the year 2000 x x x.(Emphasis ours)”

An action for reversion, a remedy provided under Commonwealth Act No. 141, seeks to cancel the
original certificate of registration, and nullify the original certificate of title, including the transfer of
certificate of title of the successors-in-interest because the same were all procured through fraud and
misrepresentation. In cancelling and nullifying such title, it restores the public land fraudulently awarded
and disposed of to private individuals or corporations to the mass of public domain. Such action is filed
by the OSG pursuant to its authority under the Administrative Code.

On the other hand, an action for annulment of free patents and certificates of title also seeks for the
cancellation and nullification of the certificate of title, but once the same is granted, it does not operate to

Remedial Law Review II (CY 2018-2019) – Saturday 1:00pm – 5:00pm: Case Digest Compilation [Atty. Brondial] Page 214
revert the property back to the State, but to its lawful owner. In such action, the nullity arises not from
fraud or deceit, but from the fact that the director of the Land Management Bureau had no jurisdiction to
bestow title; hence, the issued patent or certificate of title was void ab initio.

Thus, the difference between them lies in the allegations as to the character of ownership of the realty
whose title is sought to be nullified. In an action for reversion, the pertinent allegations in the complaint
would admit State ownership of the disputed land, while in an action for annulment of patent and
certificate of title, pertinent allegations deal with plaintiffs ownership of the contested land prior to the
issuance of the same as well as defendant's fraud or mistake in successfully obtaining these documents of
title over the parcel of land claimed by the plaintiff.

2. YES. Being an action for annulment of patents and titles, it is the respondent who is the real party-in-
interest for it is the one claiming title or ownership adverse to that of the registered owner.

DOCTRINE:Action for REVERSIONvs Action for ANNULMENT of Patent and Certificate of Title:

In an action for reversion, the pertinent allegations in the complaint would admit State ownership of the
disputed land, while in an action for annulment of patent and certificate of title, pertinent allegations deal
with plaintiffs ownership of the contested land prior to the issuance of the same as well as defendant's
fraud or mistake in successfully obtaining these documents of title over the parcel of land claimed by the
plaintiff.

Remedial Law Review II (CY 2018-2019) – Saturday 1:00pm – 5:00pm: Case Digest Compilation [Atty. Brondial] Page 215
C. Guardians and Guardianship (R-92 to 97)

C.1. Goyena vs Ledesma-Gustilo

GR No 147148 13; January 2003

FACTS: This is an appeal, a petition for review on certiorari, from the CA’s Decision which affirmed the
RTC’s Decision in a Special Proceeding appointing herein respondent Amparo Ledesma-Gustilo,
respondent, as guardian over the person and property of her sister Julieta Ledesma, Pilar Y. Goyena,
petitioner, Julieta’s close friend and companion of more than 60 years.
Respondent filed with the RTC of Makati a Petition for Letters of Guardianship over the person and
property of her sister Julieta alleging that Julieta’s health is under medical attention and has been in and
out of the hospital; Julieta is not in a position to care for herself and needs the assistance of a guardian to
manage her interests in on-going corporate and agricultural enterprises.
Petitioner filed an opposition to the petition for letters of guardianship. Petitioner avers that such petition
lacked factual and legal basis because Julieta is competent and sane, thus, guardianship over is absolutely
not needed. Also, petitioner is not fit to be appointed as guardian of Julieta since their interests are
antagonistic.

ISSUE: Whether petitioner should be the guardian of Julieta?

RULING: No. The qualification of respondent to act as guardian over the person and properties of Julieta
has been duly established. As a sister, she can best take care of Julieta’s concern and wellbeing. Now that
Julieta is in the twilight of her life, her family should be given the opportunity to show their love and
affection for her without however denying petitioner access to her considering the special bond of
friendship between the two.

Needless to say, the oppositor at 90 years of age could not be said to be physically fit to attend to all the
needs of Julieta.

Remedial Law Review II (CY 2018-2019) – Saturday 1:00pm – 5:00pm: Case Digest Compilation [Atty. Brondial] Page 216
C.2. The Incompetent, CARMEN CANIZA, represented by her legal guardian, AMPARO
EVANGELISTA v. COURT OF APPEALS(SPECIAL FIRST DIVISION), PEDRO ESTRADA
and his wife, LEONORA ESTRADA

G.R. No. 110427; February 24, 1997; NARVASA, C.J.

FACTS: Being then ninety-four (94) years of age, Carmen Caniza was declared incompetent by
judgment in a guardianship proceeding instituted by her niece, Amparo A. Evangelista. Caniza was the
owner of a house and lot. Her guardian Amparo Evangelista commenced a suit to eject the spouses Pedro
and Leonora Estrada from said premises.

The complaint was later amended to identify the incompetent Caniza as plaintiff, suing through her legal
guardian, Amparo Evangelista. The amended Complaint pertinently alleged that plaintiff Caniza was the
absolute owner of the property in question; that out of kindness, she had allowed the Estrada Spouses,
their children, grandchildren, and sons-in-law to temporarily reside in her house, rent-free; that Caniza
already had urgent need of the house on account of her advanced age and failing health, “so funds could
be raised to meet her expenses for support, maintenance and medical treatment;” among others.
The defendants declared that they had been living in Caniza’s house since the 1960’s; that in
consideration of their faithful service they had been considered by Caniza as her own family, and the
latter had in fact executed a holographic will by which she “bequeathed” to the Estradas the house and lot
in question. The Estradas insist that the devise of the house to them by Caiza clearly denotes her intention
that they remain in possession thereof, and legally incapacitated her judicial guardian, Amparo
Evangelista, from evicting them therefrom, since their ouster would be inconsistent with the ward’s will.
Such will has not been submitted for probate.

ISSUE(S):
1. Whether or not Evangelista, as Cañiza's legal guardian had authority to bring said action;
2. Whether or not Evangelista may continue to represent Cañiza after the latter's death.

RULING:
1. Yes, a will is essentially ambulatory, it may be changed or revoked prior to testator’s death and until
admitted to probate, it has no effect, owner can still take back possession in the meantime Carmen
needed income from house due to his age, etc. Because of Amparo’s appointment as general guardian of
both the person and estate of her aunt – Carmen, it became her duty to care for her aunt’s person, to attend
to her physical and spiritual needs, to assure her well-being, with right to custody of her person in
preference to relatives and friends. It also became her right and duty to get possession of, and exercise
control over, Caiza's property, both real and personal, it being recognized principle that the ward has no
right to possession or control of his property during her incompetency. That right to manage the ward's
estate carries with it the right to take possession thereof and recover it from anyone who retains it, and
bring and defend such actions as may be needful for this purpose. She has full authority to take possession
of the property of said incompetent in any province or provinces in which it may be situated and to
perform all other acts necessary for the management of her properties. Actually, in bringing the action of
desahucio, Evangelista was merely discharging the duty to attend to "the comfortable and suitable
maintenance of the ward" explicitly imposed on her by Section 4, Rule 96 of the Rules of Court.

2. Yes, Evangelista may still pursue the case not as a guardian but in substitution for the deceased. While
it is indeed well-established rule that the relationship of guardian and ward is necessarily terminated by
the death of either the guardian or the ward, the rule affords no advantage to the Estradas. Amparo
Evangelista, as niece of Carmen Caiza, is one of the latter's only two (2) surviving heirs, the other being
Caiza's nephew, Ramon C. Nevado. On their motion and by Resolution of this Court of June 20, 1994,
they were in fact substituted as parties in the appeal at bar in place of the deceased, in accordance with
Section 17, Rule 3 of the Rules of Court.

"SEC. 18. Death of a party. After a party dies and the claim is not thereby extinguished, the court shall
order, upon proper notice, the legal representative of the deceased to appear and be substituted for the
deceased within a period of thirty (30) days, or within such time as may be granted. If the legal
representative fails to appear within said time, the court may order the opposing party to procure the
appointment of a legal representative of the deceased within a time to be specified by the court, and the
representative shall immediately appear for and on behalf of the interest of the deceased. The court
charges involved in procuring such appointment, if defrayed by the opposing party, may be recovered as
costs. The heirs of the deceased may be allowed to be substituted for the deceased, without requiring the
appointment of an executor or administrator and the court may appoint guardian ad litem for the minor

Remedial Law Review II (CY 2018-2019) – Saturday 1:00pm – 5:00pm: Case Digest Compilation [Atty. Brondial] Page 217
heirs. An ejectment case survives the death of a party. Carmen’s demise did not extinguish the desahucio
suit instituted by her through her guardian. That action, not being a purely personal one, survived her
death; her heirs have taken her place and now represent her interests in the appeal at bar.

DOCTRINE: While it is indeed well-established rule that the relationship of guardian and ward is
necessarily terminated by the death of either the guardian or the ward, the rule affords no advantage to
the Estradas. The action not being personal one, survived her death, her heirs have taken her place and
now represent her interests in the appeal at bar.

Remedial Law Review II (CY 2018-2019) – Saturday 1:00pm – 5:00pm: Case Digest Compilation [Atty. Brondial] Page 218
C.3. NERI VS HEIRS OF HADJI YUSOP UY

G.R. No. 194366; October 10, 2012; PERLAS-BERNABE, J.:

FACTS: AnunciacionNeri had seven children, two (2) from her first marriage with Gonzalo Illut,
namely: Eutropia and Victoria, and five (5) from her second marriage with Enrique Neri, namely:
Napoleon, Alicia, Visminda, Douglas and Rosa. In 1977, Anunciacion died intestate. Her husband,
Enrique as natural guardian of his minor children Rosa and Douglas, together with the 3 elder siblings,
executed an Extra-Judicial Settlement of the Estate with Absolute Deed of Sale of the said homestead
properties, and conveying them to the late spouses HadjiYusopUyChildren of Enrique filed a complaint
for annulment of sale of the said homestead properties against spouses Uy (later substituted by their heirs)
before the RTC. Heirs of Uy countered that the sale took place beyond the 5-year prohibitory period from
the issuance of the homestead patents. They also denied knowledge of Eutropia and Victoria’s exclusion
from the extrajudicial settlement and sale of the subject properties, and interposed further the defenses of
prescription and laches.

ISSUE: Whether the sale made by Enrique was valid considering that the said sale was purportedly made
in behalf of the latter’s children.

RULING: YES except insofar as the shares of Eutropia, Victoria and Douglas are concerned because (1)
Eutropia and Victoria were not under Enrique’s guardianship AND (2) although Douglas was under
Enrique’s guardianship, he did not ratify the said sale upon reaching majority age.

Section 7, Rule 93 of the Rules of Court provides:


SEC. 7. Parents as Guardians. – When the property of the child under parental authority is worth two
thousand pesos or less, the father or the mother, without the necessity of court appointment, shall be his
legal guardian. When the property of the child is worth more than two thousand pesos, the father or the
mother shall be considered guardian of the child’s property, with the duties and obligations of guardians
under these Rules, and shall file the petition required by Section 2 hereof. For good reasons, the court
may, however, appoint another suitable persons.

DOCTRINE:Administration includes all acts for the preservation of the property and the receipt of
fruits according to the natural purpose of the thing. Any act of disposition or alienation, or any reduction
in the substance of the patrimony of child, exceeds the limits of administration.   Thus, a father or mother,
as the natural guardian of the minor under parental authority, does not have the power to dispose or
encumber the property of the latter. Such power is granted by law only to a judicial guardian of the
ward’s property and even then only with courts’ prior approval secured in accordance with the
proceedings set forth by the Rules of Court.

Remedial Law Review II (CY 2018-2019) – Saturday 1:00pm – 5:00pm: Case Digest Compilation [Atty. Brondial] Page 219
C.4. Oropesa vs. Oropesa

G.R. No. 184528; April 25, 2012

FACTS: On January 23, 2004, the (petitioner) filed with the Regional Trial Court of Paraaque City, a
petition for him and a certain Ms. Louie Ginez to be appointed as guardians over the property of his
father, the (respondent) CiriloOropesa. In the said petition, it is alleged among others that the
(respondent) has been afflicted with several maladies and has been sickly for over ten (10) years already
having suffered a stroke on April 1, 2003 and June 1, 2003, that his judgment and memory [were]
impaired and such has been evident after his hospitalization; that even before his stroke, the (respondent)
was observed to have had lapses in memory and judgment, showing signs of failure to manage his
property properly; that due to his age and medical condition, he cannot, without outside aid, manage his
property wisely, and has become an easy prey for deceit and exploitation by people around him,
particularly Ms. Ma. Luisa Agamata, his girlfriend.

In an Order, the presiding judge of the court a quo set the case for hearing, and directed the court social
worker to conduct a social case study and submit a report thereon. Pursuant to the abovementioned order,
the Court Social Worker conducted her social case study, interviewing the (petitioner) and his witnesses.
The Court Social Worker subsequently submitted her report but without any finding on the (respondent)
who refused to see and talk to the social worker. On July 6, 2004, the (respondent) filed his Opposition to
the petition for guardianship. The (petitioner) failed to file his written formal offer of evidence. Thus, the
(respondent) filed his Omnibus Motion (1) to declare the petitioner to have waived the presentation of his
Offer of Exhibits and the presentation of his Evidence Closed since they were not formally offered; (2) To
Expunge the Documents of the Petitioner from the Record; and (3) To Grant leave to the Oppositor to
File Demurrer to Evidence.

The trial court granted respondents demurrer to evidence. Considering that the petitioner has failed to
provide sufficient evidence to establish that Gen. Cirilo O. Oropesa is incompetent to run his personal
affairs and to administer his properties, Oppositors Demurrer to Evidence is GRANTED, and the case
was DISMISSED. Petitioner elevated the case to the Court of Appeals but his appeal was dismissed.

ISSUE: Whether respondent is considered an incompetent person as defined under section 2, rule 92 of
the rules of court who should be placed under guardianship.

RULING: No. A guardianship is a trust relation of the most sacred character, in which one person, called
a guardian acts for another called the ward whom the law regards as incapable of managing his own
affairs. A guardianship is designed to further the wards well-being, not that of the guardian. It is intended
to preserve the wards property, as well as to render any assistance that the ward may personally require. It
has been stated that while custody involves immediate care and control, guardianship indicates not only
those responsibilities, but those of one in loco parentis as well.

In a guardianship proceeding, a court may appoint a qualified guardian if the prospective ward is proven
to be a minor or an incompetent. A reading of Section 2, Rule 92 of the Rules of Court tells us that
persons who, though of sound mind but by reason of age, disease, weak mind or other similar causes, are
incapable of taking care of themselves and their property without outside aid are considered as
incompetents who may properly be placed under guardianship.

We have held in the past that a finding that a person is incompetent should be anchored on clear, positive
and definite evidence. We consider that evidentiary standard unchanged and, thus, must be applied in the
case at bar. With the failure of petitioner to formally offer his documentary evidence, his proof of his
father’s incompetence consisted purely of testimonies given by himself and his sister (who were claiming
interest in their fathers real and personal properties) and their fathers former caregiver (who admitted to
be acting under their direction). These testimonies, which did not include any expert medical testimony,
were insufficient to convince the trial court of petitioners cause of action and instead lead it to grant the
demurrer to evidence that was filed by respondent.

The Court noted the absence of any testimony of a medical expert which states that Gen. Cirilo O.
Oropesa does not have the mental, emotional, and physical capacity to manage his own affairs. On the
contrary, Oppositors evidence includes a Neuropsychological Screening Report which states that Gen.
Oropesa, (1) performs on the average range in most of the domains that were tested; (2) is capable of
mental calculations; and (3) can provide solutions to problem situations. The Report concludes that Gen.

Remedial Law Review II (CY 2018-2019) – Saturday 1:00pm – 5:00pm: Case Digest Compilation [Atty. Brondial] Page 220
Oropesa possesses intact cognitive functioning, except for mildly impaired abilities in memory, reasoning
and orientation. It is the observation of the Court that oppositor is still sharp, alert and able.

As a general rule, only questions of law may be raised in a petition for review on certiorari because the
Court is not a trier of facts. We only take cognizance of questions of fact in certain exceptional
circumstances; however, we find them to be absent in the instant case. It is also long settled that factual
findings of the trial court, when affirmed by the Court of Appeals, will not be disturbed by this Court. As
a rule, such findings by the lower courts are entitled to great weight and respect, and are deemed final and
conclusive on this Court when supported by the evidence on record. We therefore adopt the factual
findings of the lower court and the Court of Appeals and rule that the grant of respondent’s demurrer to
evidence was proper under the circumstances obtaining in the case at bar.

Remedial Law Review II (CY 2018-2019) – Saturday 1:00pm – 5:00pm: Case Digest Compilation [Atty. Brondial] Page 221
C.5. Eduardo T. Abad, Petitioner vs. Leonardo Biason and Gabriel A. Magno, Respondent

G. R. No. 191993; December 5, 2012; Reyes, J.

FACTS: Abad filed a petition for guardianship over the person and properties of Maura B. Abad (Maura)
with the RTC of Dagupan on the ground that Maura is already sickly and can no longer manage to take
care of herself and her properties unassisted thus becoming an easy prey of deceit and exploitation.
Nobody entered an opposition and Abad was allowed to present evidence ex parte. After formal of
Plaintiff’s evidence and the case was submitted for decision, Atty. Magno filed a Motion for Leave to
Intervene, together with an Opposition-in-Intervention. Subsequently, Biason filed a Motion for Leave to
File Opposition to the Petition. Specifically, Defendant alleged that he is also a nephew of Maura and that
he was not notified of the pendency of the petition for the appointment of the Maura’s guardian.

RTC rendered a Decision, denying Abad’s petition and appointing Biason as Maura’s guardian.

Plaintiff filed an appeal to the CA, wherein the CA decided to affirm the decision of the RTC.

Abad filed a Petition for Review on Certiorari with the Supreme Court.

Pending the resolution of the instant petition, Biason died.Maura averred that Biason’sdeath rendered
moot and academic the issues raised in the petition. She thusprayed that the petition be dismissed and the
guardianship be terminated.

The Court issued a Resolution, requiring Abad tocomment on the manifestation filed by Maura. Abad
filed his Comment and expressed hisacquiescence to Maura’s motion to dismiss the petition. He
asseverated thatthe issues raised in the petition pertain to the irregularity in the appointmentof Biason as
guardian which he believed had been rendered moot andacademic by the latter’s death. He also supported
Maura’s prayer for thetermination of the guardianship by asseverating that her act of filing of apetition-in-
intervention is indicative of the fact that she is of sound mind andthat she can competently manage her
business affairs.

ISSUE: WON Biason’sdeath rendered moot and academic the issues raised in the Petition for Review in
SC filed by Abad.

RULING: YES. An issue or a case becomes moot and academic when it ceases to present a justiciable
controversy, so that a determination of the issue wouldbe without practical use and value. In such cases,
there is no actualsubstantial relief to which the petitioner would be entitled and which wouldbe negated
by the dismissal of the petition.

In the petition, Plaintiff was challenging Biason’s qualifications and the procedure by which theRTC
appointed him as guardian for Maura. However, with Biason’s demise, it has become impractical and
futile to proceed with resolving the merits of the petition. It is a well-established rule that the relationship
of guardian andward is necessarily terminated by the death of either the guardian or the ward. The
supervening event of death rendered it pointless to delve intothe propriety of Biason’s appointment since
the juridical tie between him and Maura has already been dissolved. The petition, regardless of
itsdisposition, will not afford Abad, or anyone else for that matter, anysubstantial relief.

DOCTRINE: ~ It is a well-established rule that the relationship of guardian and ward is necessarily
terminated by the death of either the guardian or the ward. The supervening event of death rendered it
pointless to delve into the propriety of Biason’s appointment since the juridical tie between him and
Maura has already been dissolved. The petition, regardless of its disposition, will not afford Abad, or
anyone else for that matter, any substantial relief.

Remedial Law Review II (CY 2018-2019) – Saturday 1:00pm – 5:00pm: Case Digest Compilation [Atty. Brondial] Page 222
D. Trustees (R-98)

D.1. LAND BANK OF THE PHILIPPINES vs. LAMBERTO C. PEREZ, et al.

G.R. No. 166884; June 13, 2012

FACTS: LBP extended a credit accommodation to ACDC through the execution of a CreditLine
Agreement. ACDC used the Letters of Credit/Trust Receipts Facility of theAgreement to buy construction
materials. The respondents, as officers andrepresentatives of ACDC, executed trust receipts in connection
with theconstruction materials. The trust receipts matured, but ACDC failed to return toLBP the proceeds
of the construction projects or the construction materials subjectof the trust receipts.

When ACDC failed to comply with the demands of LBP, the latter filed theaffidavit-complaint for estafa
before the City Prosecutor’s Office.

ACDC argued that its clients for the construction projects have not yet paid them;thus, ACDC had yet to
receive the proceeds of the materials that were the subjectof the trust receipts and were allegedly used for
these constructions. As there wereno proceeds received from these clients, no misappropriation thereof
could havetaken place.

ISSUE: Whether or not a criminal complaint for estafa is the proper remedy for analleged violation of the
Trust Receipt Law by the Trustee.

RULING: The answer is in the affirmative. However, the transaction between the parties tothis case is
not a trust receipt. Violations of Trust Receipts Law are criminallypunishable, but no criminal complaint
for violation of Article 315, paragraph 1(b)of the Revised Penal Code, in relation with P.D. 115, should
prosper against aborrower who was not part of a genuine trust receipt transaction.

It is fundamental in a trust receipt transaction that the person who advancedpayment for the merchandise
becomes the absolute owner of said merchandise andcontinues as owner until he or she is paid in full, or
if the goods had already beensold, the proceeds should be turned over to him or to her. In all trust
receipttransactions, the trustee has the obligation to either the return of the proceeds of thesale or the
return or recovery of the goods, whether raw or processed.

When both parties enter into an agreement knowing that the return of the goods subject of the trust receipt
is not possible even without any fault on the part of thetrustee, it is not a trust receipt transaction
penalized under Section 13 of P.D. 115;the only obligation actually agreed upon by the parties would be
the return of theproceeds of the sale transaction. This transaction becomes a mere loan, where
theborrower is obligated to pay the bank the amount spent for the purchase of thegoods.

The fact that LBP had knowingly authorized the delivery of construction materialsto a construction site
ACDC’s projects, repudiates the idea that LBP intended to bethe owner of those construction materials.
As a government financial institution,LBP should have been aware that the materials were to be used for
the constructionof an immovable property, as well as a property of the public domain. As animmovable
property, the ownership of whatever was constructed with thosematerials would presumably belong to the
owner of the land, under Article 445 ofthe Civil Code.

It should be noted line of work that the borrowers were engaged in was construction. In the case of
materials used in the manufacture of finished products, these finishedproducts – if not the raw materials
or their components – similarly remain in thepossession of the trustee until they are sold. But the goods
and the materials that areused for a construction project are often placed under the control and custody of
theclients employing the contractor, who can only be compelled to return the materialsif they fail to pay
the contractor and often only after the requisite legal proceedings.The contractor’s difficulty and
uncertainty in claiming these materials (or thebuildings and structures which they become part of),
as soon as the bankdemands them, disqualify them from being covered by trust receiptagreements.

Since these transactions are not trust receipts, an action for estafa should not bebrought against the
respondents, who are liable only for a loan. As the law standstoday, there can be no violation of the right
against imprisonment for non-paymentof a debt.

Remedial Law Review II (CY 2018-2019) – Saturday 1:00pm – 5:00pm: Case Digest Compilation [Atty. Brondial] Page 223
Furthermore, the proceedings regarding the criminal aspect of this case, should bedismissed as it does not
appear from the records that the complaint was filed withthe participation or consent of the Office of the
Solicitor General (OSG).

Remedial Law Review II (CY 2018-2019) – Saturday 1:00pm – 5:00pm: Case Digest Compilation [Atty. Brondial] Page 224
E. Adoption and Custody of Minors (R-99-100)

E.1. ROSARIO MATA CASTRO AND JOANNE BENEDICTA CHARISSIMA M. CASTRO,


A.K.A. "MARIA SOCORRO M. CASTRO" AND "JAYROSE M. CASTRO," Petitioners, v. JOSE
MARIA JED LEMUEL GREGORIO AND ANA MARIA REGINA GREGORIO, Respondents.

G.R. No. 188801; October 15, 2014

FACTS: This is a petition for review on Certiorari assailing the decision of the CA which denied the
petition for annulment of judgment filed by petitioners. The petition before the appellate court sought to
annul the judgment of the trial court that granted Rs’ decree of adoption.

Atty. Castro was allegedly married to Rosario Castro (Petitioner). Unfortunately, they separated later on
due to their incompatibilities and Jose’s alleged homosexual tendencies. Their marriage bore two
daughters: Rose Marie, who succumbed to death after nine days from birth due to congenital heart
disease, and Joanne Benedicta Charissima Castro (Petitioner).

On August 2000, A petition for adoption of Jose Maria Jed Gregorio (Jed) and Ana Maria Regina
Gregorio (Regina) was instituted by Atty. Jose Castro. Atty. Castro alleged that Jed and Regina were his
illegitimate children with Lilibeth Gregorio (Rosario’s housekeeper). After a Home Study Report
conducted by the Social Welfare Officer of the TC, the petition was granted.

A disbarment complaint was filed against Atty. Castro by Rosario. She alleged that Jose had been remiss
in providing support to his daughter Joanne for the past 36 year; that she single-handedly raised and
provided financial support to Joanne while Jose had been showering gifts to his driver and allege lover,
Larry, and even went to the extent of adopting Larry’s two children, Jed and Regina, without her and
Joanne knowledge and consent. Atty. Castro denied the allegation that he had remiss his fatherly duties to
Joanne. He alleged that he always offered help but it was often declined. He also alleged that Jed and
Regina were his illegitimate children that’s why he adopted them.  Later on Atty. Castro died.
Rosario and Joanne filed a petition for annulment of judgment seeking to annul the decision of the TC
approving Jed and Regina’s adoption.

Petitioner allege that Rosario’s consent was not obtained and the document purporting as Rosario’s
affidavit of consent was fraudulent. P also allege that Jed and Regina’s birth certificates shows disparity.
One set shows that the father to is Jose, while another set of NSO certificates shows the father to be
Larry.  P further alleged that Jed and Regina are not actually Jose’s illegitimate children but the legitimate
children of Lilibeth and Larry who were married at the time of their birth. CA denied the petition.

CA held that while no notice was given by the TC to Rosario and Joanne of the adoption, it ruled that
there is “no explicit provision in the rules that spouses and legitimate child of the adopter. . . should be
personally notified of the hearing.”

CA also ruled that the alleged fraudulent information contained in the different sets of birth certificates
required the determination of the identities of the persons stated therein and was, therefore, beyond the
scope of the action for annulment of judgment. The alleged fraud could not be classified as extrinsic
fraud, which is required in an action for annulment of judgment.

ISSUE:Whether consent of the spouse and legitimate children 10 years or over of the adopter is required?

RULING: YES. RA 8552 requires that the adoption by the father of a child born out of wedlock obtain
not only the consent of his wife but also the consent of his legitimate children. (Art. III, Sec. 7, RA 8552)

As a rule, the husband and wife must file a joint petition for adoption. The law, however, provides for
several exceptions to the general rule, as in a situation where a spouse seeks to adopt his or her own
children born out of wedlock. In this instance, joint adoption is not necessary. But, the spouse seeking to
adopt must first obtain the consent of his or her spouse.

In the absence of any decree of legal separation or annulment, Jose and Rosario remained legally married
despite their de facto separation. For Jose to be eligible to adopt Jed and Regina, Rosario must first
signify her consent to the adoption. Since her consent was not obtained, Jose was ineligible to adopt.

Remedial Law Review II (CY 2018-2019) – Saturday 1:00pm – 5:00pm: Case Digest Compilation [Atty. Brondial] Page 225
The law also requires the written consent of the adopter’s children if they are 10 years old or older (ART.
III, Sec. 9, RA 8552).

For the adoption to be valid, petitioners’ consent was required by Republic Act No. 8552. Personal
service of summons should have been effected on the spouse and all legitimate children to ensure that
their substantive rights are protected. It is not enough to rely on constructive notice as in this case.
Surreptitious use of procedural technicalities cannot be privileged over substantive statutory rights.

Since the trial court failed to personally serve notice on Rosario and Joanne of the proceedings, it never
validly acquired jurisdiction.

DOCTRINE:The policy of the law is clear. In order to maintain harmony, there must be a showing of
notice and consent. This cannot be defeated by mere procedural devices. In all instances where it appears
that a spouse attempts to adopt a child out of wedlock, the other spouse and other legitimate children
must be personally notified through personal service of summons. It is not enough that they be deemed
notified through constructive service.

Remedial Law Review II (CY 2018-2019) – Saturday 1:00pm – 5:00pm: Case Digest Compilation [Atty. Brondial] Page 226
E.2. Herbert Cang vs. CA

G.R. No. 105308; September 25 1998; J. Romero

FACTS: Anna Marie filed a petition for legal separation upon learning of her husband's extramarital
affairs, which the trial court approved the petition as result of legal separation the children granted
monthly support of P 1,000. Herbert went to US sought a divorce from Anna Marie in the United States.
The court granted sole custody of the 3 minor children to Anna, reserving the rights of visitation to
Herbert, while Herbert became naturalized US citizen and married an American wife and later divorced,
while in the US a portion of his salaries are being deposited for his children expenses.

The brother and sister-in-law of Anna filed for the adoption of the 3 minor children because Anna Marie
left also for US for job seeking venture and leaving a children would hamper her job seeking
opportunities and Herbert already long forfeited his parental rights. Herbert upon learning, contest the
adoption, but the petition was already granted by the trial court. CA affirmed the decree of adoption,
holding that Art. 188 of the FC requires the written consent of the natural parents of the children to be
adopted, but the consent of the parent who has abandoned the child is not necessary. It held that Herbert
failed to pay monthly support to his children. Herbert elevated the case to the Supreme Court.

ISSUE:
1. Whether or not the Herbert may be considered as having abandoned the children.
2. Whether or not the requirement of written consent for petition for adoption can be dispense with
if the parent has abandoned the child

RULING: Article 188 amended the statutory provision on consent for adoption, the written consent of
the natural parent to the adoption has remained a requisite for its validity. Rule 99 of the Rules of the
Court requires a written consent to the adoption signed by the child, xxx and by each of its known living
parents who is not insane or hopelessly intemperate or has not abandoned the child.

Article 256 of the Family Code requires the written consent of the natural parent for the decree of
adoption to be valid unless the parent has abandoned the child or that the parent is "insane or hopelessly
intemperate."

In reference to abandonment of a child by his parent, the act of abandonment imports "any conduct of the
parent which evinces a settled purpose to forego all parental duties and relinquish all parental claims to
the child." It means "neglect or refusal to perform the natural and legal obligations of care and support
which parents owe their children."

In this case, however, Herbert did not manifest any conduct that would forego his parental duties and
relinquish all parental claims over his children as to, constitute abandonment. Physical abandonment
alone, without financial and moral desertion, is not tantamount to abandonment. While Herbert was
physically absent, he was not remiss in his natural and legal obligations of love, care and support for his
children. The Court find pieces of documentary evidence that he maintained regular communications with
his wife and children through letters and telephone, and send them packages catered to their whims.

Remedial Law Review II (CY 2018-2019) – Saturday 1:00pm – 5:00pm: Case Digest Compilation [Atty. Brondial] Page 227
E.3. TOMASA VDA. DE JACOB, as Special Administratrix of the Intestate Estate of Deceased
Alfredo E. Jacob, petitioner, vs. COURT OF APPEALS, PEDRO PILAPIL, THE REGISTER OF
DEEDS for the Province of Camarines Sur, and JUAN F. TRIVINO as publisher of “Balalong,–
respondents.

G.R. No. 135216; August 19, 1999; PANGANIBAN, J.:

FACTS: Tomasa Vda. De Jacob claimed to be the surviving spouse of deceased Dr. Alfredo E. Jacob and
was appointed Special Administratix for the various estates of the deceased by virtue of a reconstructed
Marriage Contract between herself and the deceased. During the proceeding for the settlement of the
estate of the deceased Alfredo, Pedro Pilapil, claiming to be the legally-adopted son of Alfredo, sought to
intervene to claim his share of the estate as Alfredo’s adopted son and sole surviving heir. In support of
his claim, he presented an Order issued by then Presiding Judge Jose L. Moya, CFI, granting the petition
for adoption filed by Alfredo in favor of him. He also questioned the validity of the marriage
between appellant Tomasa and his adoptive father Alfredo. Tomasa claimed that the marriage between
her and Alfredo was solemnized by one Msgr. Florencio C. Yllana, CBCP, Intramuros, Manila sometime
in 1975. She could not however present the original copy of the Marriage Contract stating that the
original document was lost when Msgr. Yllana allegedly gave it to Mr. Jose Centenera for registration. In
lieu of the original, Tomasa presented as secondary evidence a reconstructed Marriage Contract issued in
1978. During the trial, the following irregularities in the execution of the reconstructed Marriage Contract
were observed. As of the adoption of Pedro, Tomasa questioned the authenticity of the signature of Judge
Moya. According to Judge Moya, he could no longer remember the facts in judicial proceedings taken
about twenty-nine (29) years ago when he was then presiding judge since he was already 79 years old and
was suffering from “glaucoma”. According Bienvenido Albacea, NBI Document Examiner, the
questioned and the standard signatures “JOSE L. MOYA” were NOT written by one and the same person.
On the other hand, Pedro presented comparative findings of former NBI Chief Document Examiner Atty.
Desiderio A. Pagui that the signature of Judge Moya granting the petition for adoption was indeed
genuine. Triacl Court ruled that Pedro was legally adopted. CA affirmed the decision of the trial court.

ISSUE: Whether defendant Pedro Pilapil is the legally adopted son of Alfredo E. Jacob?

RULING: No, because he was not able to sufficiently establish his adoption. As a rule, the factual
findings of the trial court are accorded great weight and respect by appellate courts, because it had the
opportunity to observe the demeanor of witnesses and to note telltale signs indicating the truth or the
falsity of a testimony. The rule, however, is not applicable to the present case, because it was Judge
Augusto O. Cledera, not the ponente, who heard the testimonies of the two expert witnesses. Thus, the
Court examined the records and found that the Court of Appeals and the trial court failed to notice certain
relevant facts which, if properly considered, will justify a different conclusion. Hence, the present case is
an exception to the general rule that only questions of law may be reviewed in petitions under Rule 45.As
to the authenticity of Judge Moya’s signature which is the present question, two expert witnesses were
presented, one for petitioner and one for respondent Pilapil in which the trial court relied on and brushed
aside the desposition of Judge Moya himself wherein the Pilapil justified the action of the court by
arguing that the deposition was ambiguous and contends that Judge Moya could not remember whether
the signature on the Order was his. As what Judge Moya already stated that he could not remember
issuing an Order of Adoption and when the signature was shown to him, he positively declared that it was
not his. The fact that he had glaucoma when his Deposition was taken does not discredit his statements.
At the time, he could with medication still read the newspapers; upon the request of the defense counsel,
he even read a document shown to him. Indeed, there is no reason and the respondent has not presented
any to disregard the Deposition of Judge Moya in which his testimony was supported by the expert
testimony of NBI Document Examiner Bienvenido Albacea which is a disinterested party. The alleged
Order was purportedly made in open court. In his Deposition, however, Judge Moya declared that he did
not dictate decisions in adoption cases. The only decisions he made in open court were criminal cases, in
which the accused pleaded guilty. Moreover, Judge Moya insisted that the branch where he was assigned
was always indicated in his decisions and orders; yet the questioned Order did not contain this
information. Furthermore, Pilapils conduct gave no indication that he recognized his own alleged
adoption, as shown by the documents that he signed and other acts that he performed thereafter. In the
same vein, no proof was presented that Dr. Jacob had treated him as an adopted child. Likewise, both the
Bureau of Records Management in Manila and the Office of the Local Civil Registrar of Tigaon,
Camarines Sur, issued Certifications that there was no record that Pedro Pilapil had been adopted by Dr.
Jacob. The burden of proof in establishing adoption is upon the person claiming such relationship in
which the Respondent Pilapil failed to do.

Remedial Law Review II (CY 2018-2019) – Saturday 1:00pm – 5:00pm: Case Digest Compilation [Atty. Brondial] Page 228
DOCTRINE: The burden of proof in establishing adoption is upon the person claiming such
relationship.

Remedial Law Review II (CY 2018-2019) – Saturday 1:00pm – 5:00pm: Case Digest Compilation [Atty. Brondial] Page 229
E.4. REPUBLIC OF THE PHILIPPINES, petitioner, vs.THECOURT OF APPEALS, JAIME B.
CARANTO, and ZENAIDA P. CARANTO, respondents.

G.R. No. 103695; March 15, 1996; MENDOZA, J.:

FACTS: The petition below was filed by private respondents spouses Jaime B. Caranto andZenaida P.
Caranto for the adoption of Midael C. Mazon, then fifteen years old, whohad been living with private
respondent Jaime B. Caranto since he was seven yearsold. When private respondents were married on
January 19, 1986, the minor MidaelC. Mazon stayed with them under their care and custody. Private
respondentsprayed that judgment be rendered:

1. Declaring the child Michael C. Mazon the child of petitioners for all intents andpurposes;
2. Dissolving the authority vested in the natural parents of the child; and
3. That the surname of the child be legally changed to that of the petitioners andthat the first name
which was mistakenly registered as "MIDAEL" be corrected to"MICHAEL."RTC set the case for
hearing, giving notice thereof by publication in a newspaper ofgeneral circulation and by service
of the order upon the Department of SocialWelfare and Development and the Office of the
Solicitor General.

The Solicitor General opposed the petition insofar as it sought the correction of thename of the child from
"Midael" to "Michael." He argued that although thecorrection sought concerned only a clerical and
innocuous error, it could not begranted because the petition was basically for adoption, not the correction
of anentry in the civil registry under Rule 108 of the Rules of Court.RTC dismissed the opposition of the
Solicitor General on the ground that Rule 108 ofthe Rules of Court (Cancellation or Correction of Entries
in the Civil Registry) appliesonly to the correction of entries concerning the civil status of persons. It
cited Rule108, Section 1, which provides that any person interested in an act, event, order or
decreeconcerning the civil status of persons which has been recorded in the civil register,may file a
verified petition for the cancellation or correction of any entry relatingthereto." It held that the correction
of names in the civil registry is not one of thematters enumerated in Rule 108, Section 2 as "entries
subject to cancellation orcorrection." According to the trial court, the error could be corrected in the
sameproceeding for adoption to prevent multiplicity of actions and inconvenience to thepetitioners.The
Solicitor General appealed to the Court of Appeals reiterating his contentionthat the correction of names
cannot be effected in the same proceeding foradoption. As additional ground for his appeal, he argued
that the RTC did notacquire jurisdiction over the case for adoption because in the notice published inthe
newspaper, the name given was "Michael," instead of "Midael," which is thename of the minor given in
his Certificate of Live Birth.

Court of Appeals affirmed in toto the decision of the RTC. Hence this petition forreview.

ISSUES:
A. Whether or not a petition for adoption may be denied on the ground that thepublication thereof
contains the misspelled name of the person to be adopted.
B. Whether or not Rule 108 of the Rules of Court applies to a correction of nameprayed for in a
petition for adoption.

RULING:
1) The answer is in the affirmative. The present case involves an obvious clerical errorin the name of the
child sought to be adopted. In this case the correction involvesmerely the substitution of the letters
"ch" for the letter "d," so that what appears as"Midael" as given name would read "Michael." Even
the Solicitor General admitsthat the error is a plainly clerical one. The purpose of the publication
requirementis to give notice so that those who have any objection to the adoption can make
theirobjection known. That purpose has been served by publication of notice in this case.

2) The answer is in the affirmative. Rule 108 of the Rules of Court applies to this caseand because its
provision was not complied with, the decision of the trial court,insofar as it ordered the correction of
the name of the minor, is void and withoutforce or effect. This case falls under letter "(o)," of Rule
108, referring to "changeof name."Now Section 3 of this Rule further provides that “the civil registrar
and all persons whohave or claim any interest which would be affected thereby shall be made parties
tothe proceeding.

Remedial Law Review II (CY 2018-2019) – Saturday 1:00pm – 5:00pm: Case Digest Compilation [Atty. Brondial] Page 230
The local civil registrar is thus an indispensable party, without whom no finaldetermination of the case
can be had. As he was not impleaded in this case muchless given notice of the proceeding, the decision of
the trial court, insofar as itgranted the prayer for the correction of entry, is void. The absence of
anindispensable party in a case renders ineffectual all the proceedings subsequent tothe filing of the
complaint including the judgment.

Remedial Law Review II (CY 2018-2019) – Saturday 1:00pm – 5:00pm: Case Digest Compilation [Atty. Brondial] Page 231
E.5. EUGENIO R. REYES, joined by TIMOTHY JOSEPH M. REYES, MA.GRACIA S. REYES,
ROMAN GABRIEL M. REYES, and MA. ANGELA S. REYES, petitioners,
vs. LIBRADA F. MAURICIO (deceased) and LEONIDA F. MAURICIO, respondents.

G.R. No. 175080; November 24, 2010;

FACTS:
1. Eugenio owns a parcel of land in Turo, Bocaue, Bulacan (4,527 square meters, more or less, and
covered by a TCT --- property was adjudicated to Eugenio by virtue of an extrajudicial settlement
among the heirs following the death of his parents).
2. Librada F. Mauricio (Librada, DECEASED) and her daughter Leonida F. Mauricio (Leonida)
filed a complaint before the DARAB of Malolos, Bulacan alleging that they’re the legal heirs of
Godofredo Mauricio who was the lawful and registered tenant of Eugenio through his
predecessors-in-interest to the subject land( that from 1936 until his death in May 1994,
Godofredo had been working on the subject land and introduced improvements consisting of
fruit-bearing trees, seasonal crops, a residential house and other permanent improvements; that
through fraud, deceit, strategy and other unlawful means, Eugenio caused the preparation of a
document denominated as Kasunduan dated 28 September 1994 to eject respondents from the
subject property, and had the same notarized by Notary Public Ma. Sarah G. Nicolas in Pasig,
Metro Manila; that Librada never appeared before the Notary Public; that Librada was illiterate
and the contents of the Kasunduan were not read nor explained to her; that Eugenio took undue
advantage of the weakness, age, illiteracy, ignorance, indigence and other handicaps of Librada in
the execution of the Kasunduanrendering it void for lack of consent; and that Eugenio had been
employing all illegal means to eject respondents from the subject property). 
3. Leonida and Librada prayed for the declaration of nullity of the Kasunduan and for an order for
Eugenio to maintain and place them in peaceful possession and cultivation of the subject
property.
4. According to Eugenio: Godofredo’s occupation of the subject premises was based on the former’s
mere tolerance and accommodation.  Eugenio denied signing a tenancy agreement, nor
authorizing any person to sign such an agreement.  He maintained that Librada, accompanied by a
relative, voluntarily affixed her signature to the Kasunduan and that she was fully aware of the
contents of the document.  Moreover, Librada receivedP50,000.00 from Eugenio on the same day
of the execution of the Kasunduan.  Eugenio also questioned the jurisdiction of the DARAB since
the principal relief sought by respondents is the annulment of the contract, over which jurisdiction
is vested on the regular courts. 

Provincial Adjudicator –Godofredo was the tenant of Eugenio, and Librada, being the surviving
spouse, should have peaceful possession of the land.

DARAB - Mauricios are former tenants of Spouses Reyes.

CA - affirmed the decision and resolution of the DARAB.

!!! ON APPEAL, Leonida’s legal standing as a party was also assailed by Eugenio.  Eugenio submitted
that the complaint was rendered moot with the death of Librada, Godofredo’s sole compulsory
heir.  Eugenio contended that Leonida is a mere ward of Godofredo and Librada, thus, not a legal heir.

ISSUE: WON Eugenio can question the filiation of Leonida in a case regarding land dispute.

RULING: NO.

RATIO: We are in full accord with the Court of Appeals when it ruled that Eugenio cannot collaterally
attack the status of Leonida in the instant petition.
 
          It is settled law that filiation cannot be collaterally attacked.  Well-known civilista Dr. Arturo M.
Tolentino, in his book “Civil Code of the Philippines, Commentaries and Jurisprudence,” noted that the
aforecited doctrine is rooted from the provisions of the Civil Code of the Philippines. He explained thus:
 
The legitimacy of the child cannot be contested by way of defense or as a collateral issue
in another action for a different purpose.  The necessity of an independent action directly
impugning the legitimacy is more clearly expressed in the Mexican code (article 335)
which provides: “The contest of the legitimacy of a child by the husband or his heirs must

Remedial Law Review II (CY 2018-2019) – Saturday 1:00pm – 5:00pm: Case Digest Compilation [Atty. Brondial] Page 232
be made by proper complaint before the competent court; any contest made in any other
way is void.”  This principle applies under our Family Code.  Articles 170 and 171 of the
code confirm this view, because they refer to “the action to impugn the legitimacy.” This
action can be brought only by the husband or his heirs and within the periods fixed in the
present articles.
 
In Braza v. City Civil Registrar of Himamaylan City, Negros Occidental, the Court stated that legitimacy
and filiation can be questioned only in a direct action seasonably filed by the proper party, and not
through collateral attack.
 
The same rule is applied to adoption such that it cannot also be made subject to a collateral
attack.  In Reyes v. Sotero, this Court reiterated that adoption cannot be assailed collaterally in a
proceeding for the settlement of a decedent’s estate.  Furthermore, in Austria v. Reyes, the Court declared
that the legality of the adoption by the testatrix can be assailed only in a separate action brought for that
purpose and cannot be subject to collateral attack.

Remedial Law Review II (CY 2018-2019) – Saturday 1:00pm – 5:00pm: Case Digest Compilation [Atty. Brondial] Page 233
E.6.IN THE MATTER OF THE ADOPTION OF STEPHANIE NATHY ASTORGA GARCIA
HONORATO B. CATINDIG, Petitioner,
Digested by: JESUS FERNAN MONTEMAYOR

G.R. No. 148311, March 31, 2005; Sandoval-Gutierrez, J.

FACTS: Herein petitioner Honorato B. Catindig filed a Petition to adopt his minor illegitimate daughter
STEPHANIE NATHY ASTORGA GARCIA. It was alleged therein that Stephanie was born on June 26,
1994 and her mother is Gemma Astorga Garcia. Stephanie has been using her mother’s middle name and
surname, and since he is now a widower, he is qualified to be her adopting parent. And it was prayed
therein that Stephanie’s middle name “Astorga” be changed to “Garcia”, her mother’s surname, and that
her surname “Garcia” be changed to “Catindig”, his surname.

Then, the trial court rendered a Decision granting the Petition. Accordingly, Stephanie shall be known as
STEPHANIE NATHY CATINDIG. Thereafter, petitioner moved for clarification and/or reconsideration
of the said decision praying that Stephanie should be allowed to use the surname of her natural mother,
Garcia, as her middle name. The trial court the said motion holding that there is no law or jurisprudence
allowing an adopted child to use the surname of his biological mother as his middle name. Hence, this
petition.

ISSUE: WON an illegitimate child may use the surname of her mother as her middle name after having
been adopted by her natural father.

RULING: YES.Although the law is silent as to what middle name an adoptee may use as Article 365 of
the Civil Code merely provides that “an adopted child shall bear the surname of the adopted”, and that
Article 189 of the Family Code provides that “the adopted shall be deemed to be a legitimate child of the
adopter”, the committees which drafted the pertinent laws recognized the Filipino custom of adding the
surname of the child’s mother as his middle name. Being a legitimate child by virtue of her adoption, it
follows that Stephanie is entitled to all the rights provided by law to a legitimate child without
discrimination of any kind, including the right to bear the surname of her father and her mother.Since
there is no law prohibiting an illegitimate child adopted by her natural father, like Stephanie, to use, as
middle name her mother’s surname, there is no reason why she should not be allowed to do so.

Remedial Law Review II (CY 2018-2019) – Saturday 1:00pm – 5:00pm: Case Digest Compilation [Atty. Brondial] Page 234
E.7. IN RE: PETITION FOR ADOPTION OF MICHELLE P. LIM,MONINA P. LIM, Petitioner.

G.R. Nos. 168992-93; May 21, 2009; CARPIO, J.:

FACTS:Monina P. Lim filed with the RTC a petition for adoption of Michelle P. Lim and Michael Jude
P. Lim availing the amnesty given under the Domestic Adoption Act of 1998 rectifying simulation of
births. Prior to this case, Monina, with her first husband, now deceased, took in the said adoptees as their
own children, registering the fact of their birth making it appear that they were petitioner and his prior
husband’s natural children.

When Monina remarried, she decided to adopt the children getting the consent of his new husband who is
an American citizen. During this time, the children attained the age of emancipation.

The trial court dismissed the petition declaring that it should have been filed jointly with the petitioner’s
spouse for it is a mandatory requirement under the Domestic Adoption Act and the Family Code; that
mere consent of the other is not enough for the petition to prosper because the circumstance does not fall
within the exception provided by the said Act; that joint adoption of an emancipated child is not merely
for the purpose of exercising parental authority because the same acquires certain rights from his parents
and assumes certain obligations and responsibilities.

Now, in this petition for review on certiorari, petitioner contends that joint parental authority is not
necessary in the adoption of emancipated children. She also insisted that her foreign husband already
initiated a case for the dissolution of their marriage, thus joint adoption is not anymore possible.

ISSUE: W/N an individual who has remarried, can singly adopt, because joint parental authority is not
necessary in the adoption of emancipated children.

RULING: No, an individual who has remarried cannot singly adopt, because joint parental authority is
not necessary in the adoption of emancipated children. Domestic Adoption Act provides that spouses
shall jointly adopt except if one of them seeks to adopt the legitimate child of the other; if one of them
seeks to adopt own illegitimate child, with the other’s consent; or, if the spouse are legally separated from
each other.

The above instances are not present in this case, thus the petitioner and her husband is mandated to file
joint petition to adopt the children concerned who are neither the legitimate children nor the illegitimate
children of the petitioner.

Second, it is true that upon attaining the age of emancipation, parental authority over the person and
property a child shall be terminated; however, joint parental authority is only one of the effects of
adoption.

Below are the specific effects of adoption: (1) it severs all legal ties between the biological parent(s) and
the adoptee, except when the biological parent is the spouse of the adopter; (2) it deems the adoptee as a
legitimate child of the adopter; and (3) it gives the adopter and adoptee reciprocal rights and obligations
arising from the relationship of parent and child, including but not limited to: (i) the right of the adopter to
choose the name the child is to be known; and (ii) the right of the adopter and adoptee to be legal and
compulsory heirs of each other.

Even if emancipation terminates parental authority, the adoptee is still considered a legitimate child of the
adopter with all the rights of a legitimate child such as: (1) to bear the surname of the father and the
mother; (2) to receive support from their parents; and (3) to be entitled to the legitime and other
successional rights. Conversely, the adoptive parents shall, with respect to the adopted child, enjoy all the
benefits to which biological parents are entitled such as support and successional rights.

Lastly, the supervening fact of an institution of a case for dissolution of marriage is of no moment as well.
Only a judicial decree of marriage can sever marital relations.

DOCTRINE: The Domestic Adoption Act provides that spouses shall jointly adopt except if one of them
seeks to adopt the legitimate child of the other; if one of them seeks to adopt own illegitimate child, with
the other’s consent; or, if the spouse are legally separated from each other.

Remedial Law Review II (CY 2018-2019) – Saturday 1:00pm – 5:00pm: Case Digest Compilation [Atty. Brondial] Page 235
Specific effects of adoption: (1) it severs all legal ties between the biological parent(s) and the adoptee,
except when the biological parent is the spouse of the adopter; (2) it deems the adoptee as a legitimate
child of the adopter; and (3) it gives the adopter and adoptee reciprocal rights and obligations arising
from the relationship of parent and child, including but not limited to: (i) the right of the adopter to
choose the name the child is to be known; and (ii) the right of the adopter and adoptee to be legal and
compulsory heirs of each other.

Even if emancipation terminates parental authority, the adoptee is still considered a legitimate child of
the adopter with all the rights of a legitimate child such as: (1) to bear the surname of the father and the
mother; (2) to receive support from their parents; and (3) to be entitled to the legitime and other
successional rights. Conversely, the adoptive parents shall, with respect to the adopted child, enjoy all
the benefits to which biological parents are entitled such as support and successional rights.

Remedial Law Review II (CY 2018-2019) – Saturday 1:00pm – 5:00pm: Case Digest Compilation [Atty. Brondial] Page 236
E.8. MELODY NERY v. ATTY SAMPANA

A.C. No. 10196; September 9, 2014

FACTS:Nery engaged the services of Sampana for the annulment of her marriage and for her adoption
by an alien adopter. The petition for annulment was eventually granted, and Nery paid ₱200,000.00 to
Sampana. Thereafter, Nery paid Sampana ₱100,000.00, in installment: (a) ₱10,000.00 on 10 September
2008; (b) ₱50,000.00 on 2 October 2008; and (c) ₱40,000.00 on 17 November 2008. Nery no longer
asked for receipts since she trusted Sampana. On 14 February 2009, Sampana sent a text message
informing Nery that he already filed the petition for adoption and it was already published. Sampana
further informed Nery that they needed to rehearse before the hearing. Subsequently, Sampana told Nery
that the hearing was set on 5 March 2010 in Branch 11 of Malolos, Bulacan. When Nery asked why she
did not receive notices from the court, Sampana claimed that her presence was no longer necessary
because the hearing was only jurisdictional. Nery inquired from Branch 11 of Malolos, Bulacan about the
status of the petition for adoption and discovered that there was no such petition filed in the court. Thus,
in the afternoon of the same day, Nery met Sampana and sought the reimbursement of the ₱100,000.00
she paid him. Sampana agreed, but said that he would deduct the filing fee worth ₱12,000.00. Nery
insisted that the filing fee should not be deducted, since the petition for adoption was never filed.
Thereafter, Nery repeatedly demanded for the reimbursement of the ₱100,000.00 from Sampana, but the
demands were left unheeded.

Nery thereafter filed a case before the IBP. Sampana denied that he misled Nery as to the filing of the
petition for adoption. Sampana claimed that Nery could have mistaken the proceeding for the annulment
case with the petition for adoption, and that the annulment case could have overshadowed the adoption
case. In any case, Sampana committed to refund the amount Nery paid him, after deducting his legal
services and actual expenses. The IBP found Sampana guilty of malpractice for making Nery believe that
he already filed the petition for adoption and for failing to file the petition despite receiving his legal fees.
Thus, Commissioner Antiquiera recommended a penalty of three (3) months suspension from the practice
of law.

Issue: Whether Sampana is guilty of malpractice of law

Ruling: Yes.

Acceptance of money from a client establishes an attorney-client relationship and gives rise to the duty of
fidelity to the client’s cause. Every case accepted by a lawyer deserves full attention, diligence, skill and
competence, regardless of importance. A lawyer also owes it to the court, their clients, and other lawyers
to be candid and fair.

In the present case, Sampana admitted that he received "one package fee" for both cases of annulment and
adoption. Despite receiving this fee, he unjustifiably failed to file the petition for adoption and fell short
of his duty of due diligence and candor to his client. Sampana’s proffered excuse of waiting for the
certification before filing the petition for adoption is disingenuous and flimsy. Inhis position paper, he
suggested to Nery that if the alien adopter would be married to her close relative, the intended adoption
could be possible. Under the Domestic Adoption Act provision, which Sampana suggested, the alien
adopter can jointly adopt a relative within the fourth degree of consanguinity or affinity of his/her Filipino
spouse, and the certification of the alien’s qualification to adopt is waived.

Having no valid reason not to file the petition for adoption, Sampana misinformed Nery of the status of
the petition. He then conceded that the annulment case overshadowed the petition for adoption. Verily,
Sampana neglected the legal matter entrusted tohim. He even kept the money given him, in violation of
the Code’s mandate to deliver the client’s funds upon demand. A lawyer’s failure to return upon demand
the funds held by him gives rise to the presumption that he has appropriated the same for his own use, in
violation of the trust reposed in him by his client and of the public confidence in the legal profession.

Since this is not the first case filed against Sampana, the High Court modified the IBP’s recommendation
as to the penalty imposed and increased it to three (3) years suspension from the practice of law.

Remedial Law Review II (CY 2018-2019) – Saturday 1:00pm – 5:00pm: Case Digest Compilation [Atty. Brondial] Page 237
E.9. BERNARDINA Petitioner, vsP. BARTOLOME, SOCIAL SECURITY SYSTEM and
SCANMAR MARITIME SERVICES, INC., Respondents.

G.R. No. 192531; November 12, 2014

FACTS: John Colcol was employed as electrician by Scanmar Maritime Services, Inc., on board the
vessel Maersk Danville. He was enrolled under the government's Employees' Compensation Program
(ECP).Unfortunately, an accident occurred on board the vessel whereby steel plates fell on John, which
led to his untimely death.

John was, at the time of his death, childless and unmarried. Thus, petitioner Bernardina P. Bartolome,
John’s biological mother and, allegedly, sole remaining beneficiary, filed a claim for death benefits under
PD 626 with the Social Security System (SSS). However, the SSS La Union office, in a letter addressed
to petitioner, denied the claim, stating:

We regret to inform you that wecannot give due course to your claim because you are no longer
considered as the parent of JOHN COLCOL as he was legally adopted by CORNELIO COLCOL based
on documents you submitted to us.

The denial was appealed tothe Employees’ Compensation Commission (ECC), which affirmed the ruling
of the SSS La Union Branch through the assailed Decision.

In denying the claim, both the SSS La Union branch and the ECC ruled against petitioner’s entitlement to
the death benefits sought after under PD 626 on the ground she can no longer be considered John’s
primary beneficiary. As culled from the records, John and his sister Elizabeth were adopted by their great
grandfather, petitioner’s grandfather, Cornelio Colcol (Cornelio). Consequently, as argued by the
agencies, it is Cornelio who qualifies as John’s primary beneficiary, not petitioner.

ECC Contention:Under Article 167 (j) of P.D. 626, as amended, provides that beneficiaries are the
"dependent spouse until he remarries and dependent children, who are the primary beneficiaries. In their
absence, the dependent parentsand subject to the restrictions imposed on dependent children, the
illegitimate children and legitimate descendants who are the secondary beneficiaries; Provided; that the
dependent acknowledged natural child shall be considered as a primary beneficiary when there are no
other dependent children who are qualified and eligible for monthly income benefit."
The dependent parent referred to by the above provision relates to the legitimate parent of the covered
member, as provided for by Rule XV, Section 1 (c) (1) of the Amended Rules on Employees’
Compensation. This Commission believes that the appellant is not considered a legitimate parent of the
deceased, having given up the latter for adoption to Mr. Cornelio C. Colcol. Thus, in effect, the adoption
divested her of the statusas the legitimate parent of the deceased.
x xxx

In effect, the rights which previously belong to the biological parent of the adopted child shall now be
upon the adopting parent. Hence, in this case, the legal parent referred to by P.D. 626, as amended, as the
beneficiary, who has the right to file the claim, is the adoptive father of the deceased and not herein
appellant.

ISSUE: Whether or not the biological parents of the covered, but legally adopted, employee considered
secondary beneficiaries and, thus, entitled, in appropriate cases, to receive the benefits under the ECP?
(Whether or not the parental authority of the biological parent is restored when the adoptive parent dies
and when the adoptee is still a minor)

RULING: Yes. The biological parent is entitled.


True, when Cornelio, in 1985, adoptedJohn, then about two (2) years old, petitioner’s parental authority
over John was severed. However, lest it be overlooked, one key detail the ECC missed, aside from
Cornelio’s death, was that when the adoptive parent died less than three (3) years after the adoption
decree, John was still a minor, at about four (4) years of age.

John’s minority at the time of his adopter’s death is a significant factor in the case at bar. Under such
circumstance, parental authority should be deemed to have reverted in favor of the biological
parents. Otherwise, taking into account our consistent ruling that adoption is a personal
relationship and that there are no collateral relatives by virtue of adoption, who was then left to
care for the minor adopted child if the adopter passed away?

Remedial Law Review II (CY 2018-2019) – Saturday 1:00pm – 5:00pm: Case Digest Compilation [Atty. Brondial] Page 238
To be sure, reversion of parental authority and legal custody in favor of the biological parents is not a
novel concept. Section 20 of Republic Act No. 8552 (RA 8552), otherwise known as the Domestic
Adoption Act, provides:
Section 20. Effects of Rescission.– If the petition [for rescission of adoption] is granted, the parental
authority of the adoptee's biological parent(s), if known, or the legal custody of the Department shall be
restored if the adoptee is still a minoror incapacitated. The reciprocal rights and obligations of the
adopter(s) and the adoptee to each other shall be extinguished.

The provision adverted to is applicable herein by analogy insofar as the restoration of custody is
concerned. The manner herein of terminating the adopter’s parental authority, unlike the grounds for
rescission,justifies the retention of vested rights and obligations between the adopter and the adoptee,
while the consequent restoration of parental authority in favor of the biological parents, simultaneously,
ensures that the adoptee, who is still a minor, is not left to fend for himself at such a tender age.

To emphasize, We can only apply the rule by analogy, especially since RA 8552 was enacted after
Cornelio’s death. Truth be told, there is a lacuna in the law as to which provision shall govern
contingencies in all fours with the factual milieu of the instant petition. Nevertheless, We are guided by
the catena of cases and the state policies behind RA 8552 wherein the paramount consideration is the
best interest of the child, which We invoke to justify this disposition. It is, after all, for the best interest
of the child that someone will remain charged for his welfare and upbringing should his or her
adopter fail or is rendered incapacitated to perform his duties as a parent at a time the adoptee
isstill in his formative years, and, to Our mind, in the absence or, as in this case, death of the
adopter, no one else could reasonably be expected to perform the role of a parent other than the
adoptee’s biological one.

Moreover, this ruling finds support on the fact that even though parental authority is severed by virtue of
adoption, the ties between the adoptee and the biological parents are not entirely eliminated. To
demonstrate, the biological parents, insome instances, are able to inherit from the adopted, as can be
gleaned from Art. 190 of the Family Code:
Art. 190. Legal or intestate succession to the estate of the adopted shall be governed by the following
rules:
xxx
(2) When the parents, legitimate or illegitimate, or the legitimate ascendants of the adopted concur
withthe adopter, they shall divide the entire estate, one-half tobe inherited by the parents or ascendants
and the other half, by the adopters;
xxx
(6) When only collateral blood relatives of the adopted survive, then the ordinary rules of legal or
intestate succession shall apply.
Similarly, at the time of Cornelio Colcol’s death, which was prior to the effectivity of the Family Code,
the governing provision is Art. 984 of the New Civil Code, which provides:

Art. 984. In case of the death of an adopted child, leaving no children or descendants, his parents and
relatives by consanguinity and not by adoption, shall be his legal heirs.

From the foregoing, it is apparent that the biological parents retain their rights of succession to the estate
of their child who was the subject of adoption. While the benefits arising from the death of an SSS
covered employee do not form part of the estateof the adopted child, the pertinent provision on legal or
intestate succession at least reveals the policy on the rights of the biological parents and those by adoption
vis-à-vis the right to receive benefits from the adopted. In the same way that certain rights still attach by
virtue of the blood relation, so too should certain obligations, which, We rule, include the exercise of
parental authority, in the event of the untimely passing of their minor offspring’s adoptive parent. We
cannot leave undetermined the fate of a minor child whose second chance ata better life under the care of
the adoptive parents was snatched from him by death’s cruel grasp. Otherwise, the adopted child’s quality
of life might have been better off not being adopted at all if he would only find himself orphaned in the
end. Thus, We hold that Cornelio’s death at the time of John’sminority resulted in the restoration of
petitioner’s parental authority over the adopted child.

Following Cornelio’s death in 1987, so records reveal, both petitioner and John repeatedly reported
"Brgy. Capurictan, Solsona, Ilocos Norte" as their residence. In fact, this veryaddress was used in John’s
Death Certificate executed in Brazil, and in the Report of Personal Injury or Loss of Life accomplished by
the master of the vessel boarded by John. Likewise, this is John’s known address as per the ECC’s

Remedial Law Review II (CY 2018-2019) – Saturday 1:00pm – 5:00pm: Case Digest Compilation [Atty. Brondial] Page 239
assailed Decision.Similarly, this same address was used by petitioner in filing her claim before the SSS
La Union branch and, thereafter, in her appeal with the ECC. Hence, it can be assumed that aside from
having been restored parental authority over John, petitioner indeed actually exercised the same, and that
they lived together under one roof.

Moreover, John, in his SSS application, named petitioner as one of his beneficiaries for his benefits under
RA 8282, otherwise known as the "Social Security Law." While RA 8282 does not cover compensation
for work-related deaths or injury and expressly allows the designation of beneficiaries who are not related
by blood to the member unlike in PD 626, John’s deliberate act of indicating petitioner as his beneficiary
at least evinces that he, in a way, considered petitioner as his dependent. Consequently, the confluence of
circumstances – from Cornelio’s death during John’s minority, the restoration ofpetitioner’s parental
authority, the documents showing singularity of address, and John’s clear intention to designate petitioner
as a beneficiary - effectively made petitioner, to Our mind, entitled to death benefit claims as a secondary
beneficiary under PD 626 as a dependent parent.

Cornelio’s adoption of John, without more, does not deprive petitioner of the right to receive the benefits
stemming from John’s death as a dependent parent given Cornelio’s untimely demise during John’s
minority. Since the parent by adoption already died, then the death benefits under the Employees'
Compensation Program shall accrue solely to herein petitioner, John's sole remaining beneficiary.

DOCTRINE: Section 20. Effects of Rescission.– If the petition [for rescission of adoption] is granted,
the parental authority of the adoptee's biological parent(s), if known, or the legal custody of the
Department shall be restored if the adoptee is still a minor or incapacitated. The reciprocal rights and
obligations of the adopter(s) and the adoptee to each other shall be extinguished.

Remedial Law Review II (CY 2018-2019) – Saturday 1:00pm – 5:00pm: Case Digest Compilation [Atty. Brondial] Page 240
F. Habeas Corpus (R-102)

F.1. Ilusorio vs. Bildner

G.R. No. 139789; May 12, 2000; Pardo, J.

FACTS: In 1942, Petitioner Erlinda (Erlinda)Kalaw and PotencianoIlusorio contracted matrimony and
lived together for a period of 30 years. In 1972, they separated from bed and board for undisclosed
reasons. Potenciano lived in Makati City when he was in Manila and at Ilusorio Penthouse, Baguio
Country Club when he was in. On the other hand, Erlinda lived in Antipolo City.Out of their marriage,
the spouses had 6 children, including hereinRespondentsErlinda (Lin)IlusorioBildner and SylviaIlusorio.

In 1997, upon Potenciano’s arrival from the US, he stayed with Erlinda for about 5months in Antipolo
City. The children, Sylvia and Lin, alleged that during this time, their mother gave Potenciano an
overdose of 200 mg instead of 100 mg Zoloft, an antidepressant drug prescribed by his doctor in New
York, U.S.A. As a consequence, Potenciano’s health deteriorated.

In 1998, Erlinda filed with the RTC of Antipolo City a petition for guardianship over the person and
property of Potenciano due to the latter’s advanced age, frail health, poor eyesight and impaired
judgment.

After attending a corporate meeting in Baguio City, Potenciano did not return to Antipolo City and
instead lived at Cleveland Condominium, Makati.

In 1999, Erlinda filed with the CA a petition for habeas corpus to have the custody of Potenciano. She
alleged that respondents refused petitioner’s demands to see and visit her husband and prohibited
Potenciano from returning to Antipolo City.

After due hearing, the CA rendered a decision ordering that the writ of habeas corpus previously issued
be recalled and the petition for habeas corpus be denied. The CA dismissed the petition for lack of
unlawful restraint or detention of the Potenciano.

ISSUE: Whether a spouse may secure a writ of habeas corpus to compel his/her spouse to live with
him/her.

RULING: No. Marital rights including coverture and living in conjugal dwelling may not be
enforced by the extra-ordinary writ of habeas corpus.

The evidence shows that there was no actual and effective detention or deprivation of
PotencianoIlusorio’s liberty that would justify the issuance of the writ. The fact that
PotencianoIlusorio is about 86 years of age, or under medication does not necessarily render him mentally
incapacitated. Soundness of mind does not hinge on age or medical condition but on the capacity of the
individual to discern his actions.

Being of sound mind, he is thus possessed with the capacity to make choices. In this case, the crucial
choices revolve on his residence and the people he opts to see or live with. The choices he made may not
appeal to some of his family members but these are choices which exclusively belong to Potenciano. He
made it clear before the Court of Appeals that he was not prevented from leaving his house or seeing
people. With that declaration, and absent any true restraint on his liberty, we have no reason to reverse the
findings of the Court of Appeals.

DOCTRINE: Habeas corpus is a writ directed to the person detaining another, commanding him to
produce the body of the prisoner at a designated time and place, with the day and cause of his capture
and detention, to do, submit to, and receive whatsoever the court or judge awarding the writ shall
consider in that behalf.

It is a high prerogative, common-law writ, of ancient origin, the great object of which is the liberation of
those who may be imprisoned without sufficient cause.It is issued when one is deprived of liberty or is
wrongfully prevented from exercising legal custody over another person.

Remedial Law Review II (CY 2018-2019) – Saturday 1:00pm – 5:00pm: Case Digest Compilation [Atty. Brondial] Page 241
A writ of habeas corpus extends to all cases of illegal confinement or detention,or by which the rightful
custody of a person is withheld from the one entitled thereto. It is available where a person continues to
be unlawfully denied of one or more of his constitutional freedoms, where there is denial of due process,
where the restraints are not merely involuntary but are unnecessary, and where a deprivation of freedom
originally valid has later become arbitrary.  It is devised as a speedy and effectual remedy to relieve
persons from unlawful restraint, as the best and only sufficient defense of personal freedom.

The essential object and purpose of the writ of habeas corpus is to inquire into all manner of involuntary
restraint, and to relieve a person therefrom if such restraint is illegal.To justify the grant of the petition,
the restraint of liberty must be an illegal and involuntary deprivation of freedom of action.The illegal
restraint of liberty must be actual and effective, not merely nominal or moral.

Remedial Law Review II (CY 2018-2019) – Saturday 1:00pm – 5:00pm: Case Digest Compilation [Atty. Brondial] Page 242
F.2. Atty. Edwardo Serapio vs. Sandiganbayan, People of the Philippines and
PNP Director General Leandro Mendoza

G.R. No. 148468, 148769 & 149116 ; January 28, 2003; Ponente: J. Callejo Sr.

FACTS: Atty. Edwards Serapio filed two petitions for certiorari in the SC for assailing the resolutions of
Sandiganbayan in denying his petition for bail, motion for reinvestigation, motion to quash and petition
for Habeas Corpus.

Atty. Edward was charged with the crime of plunder together with former President Jospeh Estrada and
latter’s son Jinggoy Estrada. Petitioner was a member of Board of Trustees and legal cousel of Erap
Muslim Youth Foundation a non-stock, non-profit foundation established in February 2000 ostensibly for
the purpose of providing educational opportunities for the poor and underprivileged but deserving Muslim
youth and students, and support to research and advance studies of young Muslim educators and
scientists..

Sometime in April 2000, petitioner, as trustee of the Foundation, received on its behalf a donation in the
amount of Php200 Million from Ilocos Sur Governor Luis Chavit Singson through the latters assistant
Mrs. Yolanda Ricaforte. Petitioner received the donation and turned over the said amount to the
Foundations treasurer who later deposited it in the Foundations account with the Equitable PCI Bank.

Singson publicly acused Pre. Estrada and his cohort including the petitioner who allegedly received
millions of pesos on behalf of the said foundation, coming from illegal activities known as Jueteng.

The Ombusdman recommended the filing of a case against the petitioner, Pre. Estrada and et al. before
the Sandiganbayan. A warrant of arrest for Atty. Serapio was issued. Upon learning of the said warrant,
the petitioner voluntyarily surrendered himself to the PNP. He then filed an Urgent Motion for Bail but it
was opposed by the prosection for the reason that the petitioner should be arrainged first before he can
avail of bail. the bail hearing was reset several times due to various reasons, this prompted the petitioner
to file a petitioner for writ of habeas corpus. Atty. Serapio alleged that the prosecution have waived his
right to present evidence in oppositio to his petition for bail by launching endless barrage of obstructive
and dilatory moves to prevent the conduct of the bail hearings and presecutor’s failure to adduce strong
eveidence of his guilt, thus he is deprived of his liberty.

ISSUE: Whether of not Petition for Habeas Corpus is proper?

RULING: NO, the Supreme Court ruled that the general rule applies. The petition for Habeas Corpus is
not an appropriate remedy for asserting one’s right to bail. It cannot be availed where accused is entitled
to bail not as a matter of right but on the discretion of the court. Also the writ of habeas corpus will not
issue where the person alleged to be restrained of his liberty in custody of an officer under a process
issued by the court which jurisdiction to do so. 

In exceptional circumstances, habeas corpus may be granted by the courts even when the person
concerned is detained pursuant to a valid arrest or his voluntary surrender, for this writ of liberty is
recognized as the fundamental instrument for safeguarding individual freedom against arbitrary and
lawless state action due to its ability to cut through barriers of form and procedural mazes. In previous
cases decided by SC, it was held that the issuance of the writ where the deprivation of liberty, while
initially valid under the law, had later become invalid, and even though the persons praying for its
issuance were not completely deprived of their liberty.

In the above case, the court finds no basis for the issuance of a writ of  habeas corpus in favor of Atty.
Serapio. The general rule that habeas corpus does not lie where the person alleged to be restrained of his
liberty is in the custody of an officer under process issued by a court which had jurisdiction to issue the
same applies, because petitioner is under detention pursuant to the order of arrest issued by the
Sandiganbayan on April 25, 2001 after the filing by the Ombudsman of the amended information for
plunder against petitioner and his co-accused.

The delay in the hearing of petitioners petition for bail cannot be pinned solely on the Sandiganbayan or
on the prosecution for that matter. Atty. Serapio himself is partly to be blamed. Moreover, a petition
for habeas corpus is not the appropriate remedy for asserting ones right to bail It cannot be availed of
where accused is entitled to bail not as a matter of right but on the discretion of the court and the latter has

Remedial Law Review II (CY 2018-2019) – Saturday 1:00pm – 5:00pm: Case Digest Compilation [Atty. Brondial] Page 243
not abused such discretion in refusing to grant bail or has not even exercised said discretion. The proper
recourse is to file an application for bail with the court where the criminal case is pending and to allow
hearings thereon to proceed.

DOCTRINE: The general rule that habeas corpus does not lie where the person alleged to be
restrained of his liberty is in the custody of an officer under process issued by a court which had
jurisdiction to issue the same. Also a petition for habeas corpus is not the appropriate remedy for
asserting ones right to bail. It cannot be availed of where accused is entitled to bail not as a matter of
right but on the discretion of the court and the latter has not abused such discretion in refusing to grant
bail or has not even exercised said discretion. 

Remedial Law Review II (CY 2018-2019) – Saturday 1:00pm – 5:00pm: Case Digest Compilation [Atty. Brondial] Page 244
F.3. PANFILO LACSON, MICHAEL RAY B. AQUINO and CESAR O. MANCAO vs.
SECRETARY HERNANDO PEREZ, P/DIRECTOR LEANDRO MENDOZA, and P/SR. SUPT.
REYNALDO BERROYA

FACTS: President Macapagal-Arroyo, issued Proclamation No. 38 declaring that there was a state of
rebellion in the National Capital Region. She likewise issued General Order No. 1 directing the Armed
Forces of the Philippines and the Philippine National Police to suppress the rebellion in the National
Capital Region. Warrantless arrests of several alleged leaders and promoters of the “rebellion” were
thereafter effected. Aggrieved by the warrantless arrests, and the declaration of a “state of rebellion,”
which allegedly gave a semblance of legality to the arrests, actions for for prohibition, injunction,
mandamus, and habeas corpus (with an urgent application for the issuance of temporary restraining order
and/or writ of preliminary injunction) were filed by several petitioners.

HABEAS CORPUS as discussed in the decision of the court: One of the petitioners are Lacson,
Aquino and Mancao who claim that they will be arrested without a warrant pursuant to the orders issued
by the president. Anent their allegations ex abundante ad caittelamin support of their application for the
issuance of a writ of habeas corpus, it is manifest that the writ is not called for since its purpose is to
relieve petitioners from unlawful restraint, a matter which was speculative.

Discussion on the Dissenting Opinion: [T]he distinction (between the calling out power, on one hand,
and the power to suspend the privilege of the writ of habeas corpus and to declare martial law, on the
other hand) places the calling out power in a different category from the power to declare martial law and
the power to suspend the privilege of the writ of habeas corpus, otherwise, the framers of the Constitution
would have simply lumped together the three powers and provided for their revocation and review
without any qualification. Expressio unius est exclusio altenus.

The reason for the difference in the treatment of the aforementioned powers highlights the intent to grant
the President the widest leeway and broadest discretion in using the “calling out” power because it is
considered as the lesser and more benign power compared to the power to suspend the privilege of the
writ of habeas corpus and the power to impose martial law, both of which involve the curtailment and
suppression of certain basic civil rights and individual freedoms, and thus necessitating affirmation by
Congress and, in appropriate cases, review by this Court.

There is nothing in the Constitution which authorizes the President or any person acting under her
direction to make unwarranted arrests. The existence of “lawless violence, invasion or rebellion” only
authorizes the President to call out the “armed forces to prevent or suppress lawless violence, invasion or
rebellion.”

Not even the suspension of the privilege of the writ of habeas corpus or the declaration of martial law
authorizes the President to order the arrest of any person. The only significant consequence of the
suspension of the writ of habeas corpus is to divest the courts of the power to issue the writ whereby the
detention of the person is put inissue. It does not by itself authorize the President to order the arrest of a
person. And even then, the Constitution in Section 18, Article VII makes the following qualifications:

The suspension of the privilege of the writ shall apply only to persons judicially charged for rebellion or
offenses inherent in or directly connected with invasion.
During the suspension of the privilege of the writ, any person thus arrested or detained shall be judicially
charged within three days, otherwise he shall be released.

In the instant case, the President did not suspend the writ of habeas corpus. Nor did she declare martial
law. A declaration of a “state of rebellion,” at most, only gives notice to the nation that it exists, and that
the armed forces may be called to prevent or suppress it, as in fact she did.

Such declaration does not justify any deviation from the Constitutional proscription against unreasonable
searches and seizures.

Remedial Law Review II (CY 2018-2019) – Saturday 1:00pm – 5:00pm: Case Digest Compilation [Atty. Brondial] Page 245
F.4. ANISAH IMPAL SANGCA vs. THE CITY PROSECUTOR OF CEBUCITY

G.R. No. 175864; June 8, 2007

FACTS: PDEA charged Lovely Impal Adam with violation of RA 9165. The inquestprosecutor
recommended the dismissal of the case but was disapproved by the CityProsecutor. Consequently, an
information charging Adam with violation of Section5, Article 2 of R.A. No. 9165 was filed before RTC.

On petition for review before the Department of Justice, Secretary Raul M.Gonzalez found no probable
cause to hold Adam liable for the offense chargedThe Justice Secretary directed the City Prosecutor of
Cebu City to withdraw theinformation. PDEA filed a motion for reconsideration but was denied by the
JusticeSecretary on December 8, 2006.

On January 4, 2007, petitioner Anisah Impal Sangca filed the instant petitionraying for the issuance of a
writ of habeas corpus and the release of Lovely ImpalAdam.

Finding that Adam could not be held liable for the crime charged, Judge Inglesissued an Order on January
26, 2007 granting the Motion to Withdraw Informationand ordering the release of the accused, unless
otherwise held for another validground.

ISSUE: Whether or not petition for habeas corpus may be availed of in case ofwarrantless arrests where
there is pending motion to withdraw information beforethe trial court.

RULING: Yes. A writ of habeas corpus extends to all cases of illegal confinement or detentionin which
any person is deprived of his liberty, or in which the rightful custody ofanyperson is withheld from the
person entitled to it. Its essential object and purposeis to inquire into all manner of involuntary restraint
and to relieve a person from itif such restraint is illegal. The singular function of a petition for habeas
corpus is toprotect and secure the basic freedom of physical liberty.

Remedial Law Review II (CY 2018-2019) – Saturday 1:00pm – 5:00pm: Case Digest Compilation [Atty. Brondial] Page 246
F.5. ANITA MANGILA, Petitioner, vs. JUDGE HERIBERTO M. PANGILINAN, ASST. CITY
PROSECUTOR II LUCIA JUDY SOLINAP, and NATIONAL BUREAU OF INVESTIGATION
(DIRECTOR REYNALDO WYCOCO), Respondents.

G.R. No. 160739; July 17, 2013; FIRST DIVISIONBERSAMIN, J.:

FACTS: Seven criminal complaints charging petitioner Anita Mangila and four others with syndicated
estafa, and with violations of Section 7(b) of Migrant Workers and Overseas Filipino Act of 1995 were
filed in the Municipal Trial Court in Cities in Puerto Princesa City (MTCC), On the following day,
Presiding Judge Pangilinanconducted a preliminary investigation on the complaints. After examining one
of the complainants, Judge Pangilinan issued a warrant for the arrest of Mangila and her cohorts without
bail.2 On the next day, the entire records of the cases, including the warrant of arrest, were transmitted to
the City Prosecutor of Puerto Princesa City for further proceedings and appropriate action in accordance
with the prevailing rules.3As a consequence, Mangila was arrested on June 18, 2003 and detained at the
NBI headquarters.

Mangilafiled in the Court of Appeals (CA) a petition for habeas corpus claiming that Judge Pangilinan
did not have the authority to conduct the preliminary investigation, that the preliminary investigation he
conducted was not yet completed when he issued the warrant of arrest, and that the issuance of the
warrant of arrest was without sufficient justification or without a prior finding of probable cause. She also
averred that the remedy of habeas corpus was available to her because she could no longer file a motion
to quash or a motion to recall the warrant of arrest considering that Judge Pangilinan had already
forwarded the entire records of the case to the City Prosecutor who had no authority to lift or recall the
warrant.

CA denied the petition for habeas corpus for its lack of merit and that the proper remedy available to
petitioner is for her to file with the Provincial Prosecutor a motion to be released from detention on the
grounds alleged in the instant petition.

MR was denied, hence, the petition for review.

ISSUE: Whether CA erred in ruling that habeas corpus was not the proper remedy to obtain the release of
Mangila from detention?

RULING: No.
Habeas corpus is not in the nature of a writ of error; nor intended as substitute for the trial court’s
function. It cannot take the place of appeal, certiorari or writ of error. The writ cannot be used to
investigate and consider questions of error that might be raised relating to procedure or on the merits. The
inquiry in a habeas corpus proceeding is addressed to the question of whether the proceedings and the
assailed order are, for any reason, null and void. The writ is not ordinarily granted where the law provides
for other remedies in the regular course, and in the absence of exceptional circumstances. Moreover,
habeas corpus should not be granted in advance of trial. The orderly course of trial must be pursued and
the usual remedies exhausted before resorting to the writ where exceptional circumstances are extant.
In another case, it was held that habeas corpus cannot be issued as a writ of error or as a means of
reviewing errors of law and irregularities not involving the questions of jurisdiction occurring during the
course of the trial, subject to the caveat that constitutional safeguards of human life and liberty must be
preserved, and not destroyed. It has also been held that where restraint is under legal process, mere errors
and irregularities, which do not render the proceedings void, are not grounds for relief by habeas corpus
because in such cases, the restraint is not illegal.

The object of the writ of habeas corpus is to inquire into the legality of the detention, and, if the detention
is found to be illegal, to require the release of the detainee. Equally well-settled however, is that the writ
will not issue where the person in whose behalf the writ is sought is out on bail, or is in the custody of an
officer under process issued by a court or judge with jurisdiction or by virtue of a judgment or order of a
court of record.

There is no question that when the criminal complaints were lodged against Mangila and her cohorts on
June 16, 2003, Judge Pangilinan, as the Presiding Judge of the MTCC, was empowered to conduct
preliminary investigations involving "all crimes cognizable by the proper court in their respective
territorial jurisdictions."His authority was expressly provided in Section 2, Rule 112 of the Revised Rules
of Criminal Procedure.

Remedial Law Review II (CY 2018-2019) – Saturday 1:00pm – 5:00pm: Case Digest Compilation [Atty. Brondial] Page 247
Under Section 6(b) of Rule 112of the Revised Rules of Criminal Procedure, the investigating judge could
issue a warrant of arrest during the preliminary investigation even without awaiting its conclusion should
he find after an examination in writing and under oath of the complainant and the witnesses in the form of
searching questions and answers that a probable cause existed, and that there was a necessity of placing
the respondent under immediate custody in order not to frustrate the ends of justice. In the context of this
rule, Judge Pangilinan issued the warrant of arrest against Mangila and her cohorts. Consequently, the CA
properly denied Mangila’s petition for habeas corpus because she had been arrested and detained by
virtue of the warrant issued for her arrest by Judge Pangilinan, a judicial officer undeniably possessing the
legal authority to do so.

With Mangila’s arrest and ensuing detention being by virtue of the order lawfully issued by Judge
Pangilinan, the writ of habeas corpus was not an appropriate remedy to relieve her from the restraint on
her liberty. This is because the restraint, being lawful and pursuant to a court process, could not be
inquired into through habeas corpus.

Accordingly, Section 4, Rule 102 of the Rules of Court explicitly states:


Section 4.When writ not allowed or discharge authorized. — If it appears that the person alleged to be
restrained of his liberty is in the custody of an officer under process issued by a court or judge or by virtue
of a judgment or order of a court of record, and that the court or judge had jurisdiction to issue the
process, render the judgment, or make the order, the writ shall not be allowed; or if the jurisdiction
appears after the writ is allowed, the person shall not be discharged by reason of any informality or defect
in the process, judgment, or order. Nor shall anything in this rule be held to authorize the discharge of a
person charged with or convicted of an offense in the Philippines, or of a person suffering imprisonment
under lawful judgment. (Bold underscoring supplied for emphasis)

Her proper recourse was to bring the supposed irregularities attending the conduct of the preliminary
investigation and the issuance of the warrant for her arrest to the attention of the City Prosecutor, who had
been meanwhile given the most direct access to the entire records of the case, including the warrant of
arrest, following Judge Pangilinan’s transmittal of them to the City Prosecutor for appropriate action.

DOCTRINE:When a person’s arrest and ensuing detention being by virtue of the order lawfully issued
by a Judge, the writ of habeas corpus is not an appropriate remedy to relieve a person from the restraint
on her liberty. This is because the restraint, being lawful and pursuant to a court process, could not be
inquired into through habeas corpus.

Remedial Law Review II (CY 2018-2019) – Saturday 1:00pm – 5:00pm: Case Digest Compilation [Atty. Brondial] Page 248
F.6. Tujan-Militante vs. CadaDeapera

Remedial Law Review II (CY 2018-2019) – Saturday 1:00pm – 5:00pm: Case Digest Compilation [Atty. Brondial] Page 249
F.7. IN THE MATTER OF THE PETITION FOR HABEAS CORPUS OF DATUKAN MALANG
SALIBO, DATUKAN MALANG SALIBO,  v. WARDEN, QUEZON CITY JAIL ANNEX, ET AL.

G.R. No. 197597; April 08, 2015; LEONEN, J.

FACTS:Habeas corpus is the proper remedy for a person deprived of liberty due to mistaken identity. In
such cases, the person is not under any lawful process and is continuously being illegally detained.

Datukan Malang Salibo (Salibo) was mistaken to be Butukan S. Malang, a suspect in the Maguindanao
Massacre.

Salibo presented himself before the police officers of Datu Hofer Police Station to clear his name. There,
he explained that he was not Butukan S. Malang and that he could not have participated in the November
23, 2009 Maguindanao Massacre because he was in Saudi Arabia at that time.

Afterwards, however, the police officers apprehended Salibo and tore off page two of his passport that
evidenced his departure for Saudi Arabia on November 7, 2009. They then detained Salibo at the Datu
Hofer Police Station for about three (3) days.

Salibo was finally transferred to the Quezon City Jail Annex, Bureau of Jail Management and Penology
Building, Camp Bagong Diwa, Taguig City.

On September 17, 2010, Salibo filed before the Court of Appeals the Urgent Petition for Habeas
Corpusquestioning the legality of his detention and deprivation of his liberty. He maintained that he is not
the accused Butukan S. Malang.

ISSUES:

1. Whether the Decision of the Regional Trial Court, Branch 153, Pasig City on petitioner Salibo's
Petition for Habeas Corpus was appealable to the Court of Appeals; and
2. Whether petitioner Salibo's proper remedy is to file a Petition for Habeas Corpus.

RULING:

1. Contrary to petitioner Salibo's claim, respondent Warden correctly appealed before the Court of
Appeals.

An application for a writ of habeas corpus may be made through a petition filed before this court or any
of its members, the Court of Appeals or any of its members in instances authorized by law, or the
Regional Trial Court or any of its presiding judges. The court or judge grants the writ and requires the
officer or person having custody of the person allegedly restrained of liberty to file a return of the writ.A
hearing on the return of the writ is then conducted.

The return of the writ may be heard by a court apart from that which issued the writ.  Should the court
issuing the writ designate a lower court to which the writ is made returnable, the lower court shall proceed
to decide the petition of habeas corpus. By virtue of the designation, the lower court "acquire[s] the power
and authority to determine the merits of the [petition for habeas corpus.]" Therefore, the decision on the
petition is a decision appealable to the court that has appellate jurisdiction over decisions of the lower
court.

2. Called the "great writ of liberty[,]" the writ of habeas corpus "was devised and exists as a speedy
and effectual remedy to relieve persons from unlawful restraint, and as the best and only sufficient
defense of personal freedom." The remedy of habeas corpus is extraordinary and summary in nature,
consistent with the law's "zealous regard for personal liberty."

Under Rule 102, Section 1 of the Rules of Court, the writ of habeas corpus "shall extend to all cases of
illegal confinement or detention by which any person is deprived of his liberty, or by which the rightful
custody of any person is withheld from the person entitled thereto." The primary purpose of the writ "is to
inquire into all manner of involuntary restraint as distinguished from voluntary, and to relieve a person
therefrom if such restraint is illegal." "Any restraint which will preclude freedom of action is sufficient."

The nature of the restraint of liberty need not be related to any offense so as to entitle a person to the

Remedial Law Review II (CY 2018-2019) – Saturday 1:00pm – 5:00pm: Case Digest Compilation [Atty. Brondial] Page 250
efficient remedy of habeas corpus. It may be availed of as a post-conviction remedy or when there is an
alleged violation of the liberty of abode. In other words, habeas corpus effectively substantiates the
implied autonomy of citizens constitutionally protected in the right to liberty in Article III, Section 1 of
the Constitution. Habeas corpus being a remedy for a constitutional right, courts must apply a
conscientious and deliberate level of scrutiny so that the substantive right to liberty will not be further
curtailed in the labyrinth of other processes.

XXX

It is true that a writ of habeas corpus may no longer be issued if the person allegedly deprived of liberty is
restrained under a lawful process or order of the court. The restraint then has become legal, and the
remedy of habeas corpus is rendered moot and academic.

XXXX
It is to be noted that, in all the petitions here considered, criminal charges have been filed in the proper
courts against the petitioners. The rule is, that if a person alleged to be restrained of his liberty is in the
custody of an officer under process issued by a court or judge, and that the court or judge had jurisdiction
to issue the process or make the order, or if such person is charged before any court, the writ of habeas
corpus will not be allowed. (Emphasis in the original)

XXXX

XXX. Petitioner Salibo was not arrested by virtue of any warrant charging him of an offense. He was not
restrained under a lawful process or an order of a court. He was illegally deprived of his liberty, and,
therefore, correctly availed himself of a Petition for Habeas Corpus.

XXXX

It is undisputed that petitioner Salibo presented himself before the Datu Hofer Police Station to clear his
name and to prove that he is not the accused Butukan S. Malang. When petitioner Salibo was in the
presence of the police officers of Datu Hofer Police Station, he was neither committing nor attempting to
commit an offense. The police officers had no personal knowledge of any offense that he might have
committed. Petitioner Salibo was also not an escapee prisoner.

The police officers, therefore, had no probable cause to arrest petitioner Salibo without a warrant. They
deprived him of his right to liberty without due process of law, for which a petition for habeas corpus may
be issued.

XXX Respondent Warden, Quezon City Jail Annex, Bureau of Jail Management and Penology
Building,Camp Bagong Diwa, Taguig, is ORDERED to immediately RELEASE petitioner Datukan
Maiang Salibo from detention.

Remedial Law Review II (CY 2018-2019) – Saturday 1:00pm – 5:00pm: Case Digest Compilation [Atty. Brondial] Page 251
F.8. ALEXANDER A. PADILLA v. CONGRESS OF PHILIPPINES

GR No. 231671, July 25, 2017

FACTS:P.S. Resolution No. 388 was approved, after receiving 17 affirmative votes as against 5 negative
votes, and was adopted as Senate Resolution No. 49 entitled "Resolution Expressing the Sense of the
Senate Not to Revoke, at this Time, Proclamation No. 216, Series of 2017, Entitled 'Declaring a State of
Martial Law and Suspending the Privilege of the Writ of Habeas Corpus in the Whole of Mindanao.
P.S. Resolution No. 390, on the other hand, garnered only 9 votes from the senators who were in favor of
it as opposed to 12 votes from the senators who were against its approval and adoption. On May 31, 2017,
the House of Representatives, having previously constituted itself as a Committee of the Whole House,[8]
was briefed by Executive Secretary Salvador C. Medialdea, Secretary Lorenzana, and other security
officials for about 6 hours. After the closed-door briefing, the House of Representatives resumed its
regular meeting and deliberated on House Resolution No. 1050 entitled "Resolution Expressing the Full
Support of the House of Representatives to President Rodrigo Duterte as it Finds No Reason to Revoke
Proclamation No. 216, Entitled 'Declaring a State of Martial Law and Suspending the Privilege of the
Writ of Habeas Corpus in the Whole of Mindanao.'"

The House of Representatives proceeded to divide its members on the matter of approving said resolution
through viva voce voting. The result shows that the members who were in favor of passing the subject
resolution secured the majority vote. The House of Representatives also purportedly discussed the
proposal calling for a joint session of the Congress to deliberate and vote on President Duterte's
Proclamation No. 216. After the debates, however, the proposal was rejected. These series of events led to
the filing of the present consolidated petitions.

Petitioners raise the question of "[w]hether Congress is required to convene in joint session, deliberate,
and vote jointly under Article VII, [Section] 18 of the Constitution"

Issues:Whether or not the Congress has the mandatory duty to convene jointly upon the President's
proclamation of martial law or the suspension of the privilege of the writ of habeas corpus under Article
VII, Section 18 of the 1987 Constitution?

Ruling:No. The Congress is not constitutionally mandated to convene in joint session except to vote
jointly to revoke the President's declaration or suspension.By the language of Article VII, Section 18 of
the 1987 Constitution, the Congress. is only required to vote jointly to revoke the President's proclamation
of martial law and/or suspension of the privilege of the writ of habeas corpus.

Within forty-eight hours from the proclamation of martial law or the suspension of the privilege of the
writ of habeas corpus, the President shall submit a report in person or in writing to the Congress. The
Congress, voting jointly, by a vote of at least a majority of all its Members in regular or special session,
may revoke such proclamation or suspension which revocation shall not be set aside by the President.
Upon the initiative of the President, the Congress may, in the same manner, extend such proclamation or
suspension for a period to be determined by the Congress, if the invasion or rebellion shall persist and
public safety requires it.The Congress, if not in session, shall, within twenty-four hours following such
proclamation or suspension, convene in accordance with its rules without need of a call.

The provision in question is clear, plain, and unambiguous. In its literal and ordinary meaning, the
provision grants the Congress the power to revoke the President's proclamation of martial law or the
suspension of the privilege of the writ of habeas corpus and prescribes how the Congress may exercise
such power, i.e., by a vote of at least a majority of all its Members, voting jointly, in a regular or special
session. The use of the word "may" in the provision - such that "[t]he Congress x x x may revoke such
proclamation or suspension x x x" - is to be construed as permissive and operating to confer discretion on
the Congress on whether or not to revoke,[71] but in order to revoke, the same provision sets the
requirement that at least a majority of the Members of the Congress, voting jointly, favor revocation.

It is worthy to stress that the provision does not actually refer to a "joint session." While it may be
conceded, subject to the discussions below, that the phrase "voting jointly" shall already be understood to
mean that the joint voting will be done "in joint session," notwithstanding the absence of clear language in
the Constitution,[72] still, the requirement that "[t]he Congress, voting jointly, by a vote of at least a
majority of all its Members in regular or special session, x x x" explicitly applies only to the situation
when the Congress revokes the President's proclamation of martial law and/or suspension of the privilege

Remedial Law Review II (CY 2018-2019) – Saturday 1:00pm – 5:00pm: Case Digest Compilation [Atty. Brondial] Page 252
of the writ of habeas corpus. Simply put, the provision only requires Congress to vote jointly on the
revocation of the President's proclamation and/or suspension.

Hence, the plain language of the subject constitutional provision does not support the petitioners'
argument that it is obligatory for the Congress to convene in joint session following the President's
proclamation of martial law and/or suspension of the privilege of the writ of habeas corpus, under all
circumstances.

The deliberations of the 1986 ConCom reveal the framers' specific intentions to (a) remove the
requirement of prior concurrence of the Congress for the effectivity of the President's proclamation of
martial law and/or suspension of the privilege of the writ of habeas corpus; and (b) grant to the Congress
the discretionary power to revoke the President's proclamation and/or suspension by a vote of at least a
majority of its Members, voting jointly.

As the Court established in its preceding discussion, the clear meaning of the relevant provision in Article
VII, Section 18 of the 1987 Constitution is that the Congress is only required to vote jointly on the
revocation of the President's proclamation of martial law and/or suspension of the privilege of the writ of
habeas corpus. Based on the Civil Liberties Union case, there is already no need to look beyond the plain
language of the provision and decipher the intent of the framers of the 1987 Constitution.

That the Congress will vote on the revocation of the President's proclamation and/or suspension in a joint
session can only be inferred from the arguments of the Commissioners who pushed for the "voting
jointly" amendment that the Members of the House of Representatives will benefit from the advice,
opinion, and/or wisdom of the Senators, which will be presumably shared during a joint session of both
Houses. Such inference is far from a clear mandate for the Congress to automatically convene in joint
session, under all circumstances, when the President proclaims martial law and/or suspends the privilege
of the writ of habeas corpus, even when Congress does not intend to revoke the President's proclamation
and/or suspension.

There was no obligation on the part of the Congress herein to convene in joint session as the provision on
revocation under Article VII, Section 18 of the 1987 Constitution did not even come into operation in
light of the resolutions, separately adopted by the two Houses of the Congress in accordance with their
respective rules of procedure, expressing support for President Duterte's Proclamation No. 216.
The provision in Article VII, Section 18 of the 1987 Constitution requiring the Congress to vote jointly in
a joint session is specifically for the purpose of revocation of the President's proclamation of martial law
and/or suspension of the privilege of the writ of habeas corpus. In the petitions at bar, the Senate and
House of Representatives already separately adopted resolutions expressing support for President
Duterte's Proclamation No. 216. Given the express support of both Houses of the Congress for
Proclamation No. 216, and their already evident lack of intent to revoke the same, the provision in Article
VII, Section 18 of the 1987 Constitution on revocation did not even come into operation and, therefore,
there is no obligation on the part of the Congress to convene in joint session.

It cannot be disputed then that the Senate and House of Representatives placed President Duterte's
Proclamation No. 216 under serious review and consideration, pursuant to their power to revoke such a
proclamation vested by the Constitution on the Congress.

The Court highlights the particular circumstance herein that both Houses of Congress already separately
expressed support for President Duterte's Proclamation No. 216, so revocation was not even a possibility
and the provision on revocation under Article VII, Section 18 of the 1987 Constitution requiring the
Congress to vote jointly in a joint session never came into operation. It will be a completely different
scenario if either of the Senate or the House of Representatives, or if both Houses of the Congress,
resolve/s to revoke the President's proclamation of martial law and/or suspension of the privilege of the
writ of habeas corpus, in which case, Article VII, Section 18 of the 1987 Constitution shall apply and the
Congress must convene in joint session to vote jointly on the revocation of the proclamation and/or
suspension.

Petitioners invoke the following provision also in Article VII, Section 18 of the 1987 Constitution: "The
Congress, if not in session, shall, within twenty-four hours following such proclamation or suspension
convene in accordance with its rules without call." Petitioners reason that if the Congress is not in session,
it is constitutionally mandated to convene within twenty-four (24) hours from the President's
proclamation of martial law and/or suspension of the privilege of the writ of habeas corpus, then it is with
all the more reason required to convene immediately if in session.

Remedial Law Review II (CY 2018-2019) – Saturday 1:00pm – 5:00pm: Case Digest Compilation [Atty. Brondial] Page 253
The Court is not persuaded.
First, the provision specially addresses the situation when the President proclaims martial law and/or
suspends the privilege of the writ of habeas corpus while the Congress is in recess. To ensure that the
Congress will be able to act swiftly on the proclamation and/or suspension, the 1987 Constitution
provides that it should convene within twenty-four (24) hours without need for call. It is a whole different
situation when the Congress is still in session as it can readily take up the proclamation and/or suspension
in the course of its regular sessions, as what happened in these cases.

Second, the provision only requires that the Congress convene without call, but it does not explicitly state
that the Congress shall already convene in joint session. In fact, the provision actually states that the
Congress "convene in accordance with its rules," which can only mean the respective rules of each House
as there are no standing rules for joint sessions. And third, it cannot be said herein that the Congress failed
to convene immediately to act on Proclamation No. 216. Both Houses of the Congress promptly took
action on Proclamation No. 216, with the Senate already issuing invitations to executive officials even
prior to receiving President Duterte's Report, except that the two Houses of the Congress acted separately.
There is likewise no basis for petitioners' assertion that without a joint session, the public cannot hold the
Senators and Representatives accountable for their respective positions on President Duterte's
Proclamation No. 216. Senate records completely chronicled the deliberations and the voting by the
Senators on Senate Resolution No. 49 (formerly P.S. Resolution No. 388) and P.S. Resolution No. 390.
While it is true that the House of Representatives voted on House Resolution No. 1050 viva voce, this is
only in accordance with its rules. Per the Rules of the House of Representatives... ince no one moved for
nominal voting on House Resolution No. 1050, then the votes of the individual Representatives cannot be
determined. It does not render though the proceedings unconstitutional or invalid.

The Congress did not violate the right of the public to information when it did not convene in joint
session.

Petitioners contend that the Constitution requires a public deliberation process on the proclamation of
martial law: one that is conducted via a joint session and by a single body. They insist that the Congress
must be transparent, such that there is an "open and robust debate," where the evaluation of the
proclamation's factual bases and subsequent implementation shall be openly discussed and where each
member's position on the issue is heard and made known to the public.

However, based on their internal rules, each House has the discretion over the manner by which
Congressional proceedings are to be conducted. Verily, sessions are generally open to the public,[84] but
each House may decide to hold an executive session due to the confidential nature of the subject matter to
be discussed and deliberated upon.

It is clear that matters affecting the security of the state are considered confidential and must be discussed
and deliberated upon in an executive session, excluding the public therefrom.

That these matters are considered confidential is in accordance with settled jurisprudence that, in the
exercise of their right to information, the government may withhold certain types of information from the
public such as state secrets regarding military, diplomatic, and other national security matters.

Thus, to demand Congress to hold a public session during which the legislators shall openly discuss these
matters, all the while under public scrutiny, is to effectively compel them to make sensitive information
available to everyone, without exception, and to breach the recognized policy of preserving these matters'
confidentiality, at the risk of being sanctioned, penalized, or expelled from Congress altogether.
Propriety of the issuance of a writ of mandamus or certiorari... t

It is essential to the issuance of a writ of mandamus that petitioner should have a clear legal right to the
thing demanded and it must be the imperative duty of the respondent to perform the act required.
Mandamus never issues in doubtful cases.

Although there are jurisprudential examples of the Court issuing a writ of mandamus to compel the
fulfillment of legislative duty,[94] we must distinguish the present controversy with those previous cases.
In this particular instance, the Court has no authority to compel the Senate and the House of
Representatives to convene in joint session absent a clear ministerial duty on its part to do so under the
Constitution and in complete disregard of the separate actions already undertaken by both Houses on
Proclamation No. 216, including their respective decisions to no longer hold a joint session, considering
their respective resolutions not to revoke said Proclamation.

Remedial Law Review II (CY 2018-2019) – Saturday 1:00pm – 5:00pm: Case Digest Compilation [Atty. Brondial] Page 254
In the same vein, there is no cause for the Court to grant a writ of certiorari.
To reiterate, the two Houses of the Congress decided to no longer hold a joint session only after
deliberations among their Members and putting the same to vote, in accordance with their respective rules
of procedure. Premises considered, the Congress did not gravely abuse its discretion when it did not
jointly convene upon the President's issuance of Proclamation No. 216 prior to expressing its concurrence
thereto.

Remedial Law Review II (CY 2018-2019) – Saturday 1:00pm – 5:00pm: Case Digest Compilation [Atty. Brondial] Page 255
H. Change of Name vs. Correction/Cancellation of Entries, as amended R.A. 9048 and 10172 (Rule
103 vs. Rule 108)

H.1. MA. LOURDES BARRIENTOS ELEOSIDA, for and in behalf of her minor child, CHARLES
CHRISTIAN ELEOSIDA, petitioner, vs. LOCAL CIVIL REGISTRAR OF QUEZON CITY, and
CARLOS VILLENA BORBON, respondents.

G.R. No. 130277; May 9, 2002; FIRST DIVISION; PUNO, J.:

FACTS: On January 30, 1997, petitioner Ma. Lourdes Eleosida filed a petition before the Regional Trial
Court of Quezon City seeking to correct the following entries in the birth certificate of her son, Charles
Christian: first, the surname "Borbon" should be changed to "Eleosida;" second, the date of the parents'
wedding should be left blank; and third, the informant's name should be "Ma. Lourdes B. Eleosida,"
instead of "Ma. Lourdes E. Borbon." In support of her petition, petitioner alleged that she gave birth to
her son out of wedlock on May 24, 1992; that she and the boy's father, Carlos Borbon, were never
married; and that the child is therefore illegitimate and should follow the mother's surname. The petition
impleaded the Local Registrar of Quezon City and Carlos VillenaBorbon as respondents.
On April 23, 1997, the trial court issued a notice of hearing.

NOTICE IS HEREBY GIVEN, that this petition is set for hearing on June 26, 1997 at 8:30 o'clock in the
morning, in the Session Hall of this Court sitting at the Ground Floor, Room 118, Hall of Justice, Quezon
City, which is ordered published once a week for three (3) consecutive weeks, in a newspaper of general
circulation and published in Metro Manila, to be selected by raffle, at the expense of the petitioner, at
which date, time and place, the petitioner shall appear and prove her petition, in that all other persons
having or claiming any interest thereon shall also appear and show cause why, if any, they have, the
petition shall not be granted.

Let copies of this notice be furnished the petitioner, and together with copies of the petition, respondent
Carlos VillenaBorbon; the Offices of the Local Civil Registrar of Quezon City and the Solicitor General,
who are given fifteen (15) days from notice of the petition, or from the last date of publication of such
notice, within which to file their opposition thereto, if any. In the event that the Solicitor General may not
be able to appear on the scheduled hearing, to designate the City Prosecutor of Quezon City to appear for
and in behalf of the State.SO ORDERED.

On June 26, 1997, the trial court issued another order setting the date for the presentation of evidence on
July 23, 1997. On August 25, 1997, the trial court motuproprio dismissed the petition for lack of merit. It
ruled:It is an established jurisprudence that, only CLERICAL ERRORS OF A HARMLESS AND
INNOCUOUS NATURE like: misspelled name, occupation of the parents, etc., may be the subject of a
judicial order (contemplated under Article 412 of the New Civil Code), authorizing changes or
corrections and: NOT as may affect the CIVIL STATUS, NATIONALITY OR CITIZENSHIP OF THE
PERSONS INVOLVED.In the present case, it is very clear that the changes desired by the petitioner will
ultimately affect the CIVIL STATUS OF CHARLES CHRISTIAN. With the petition's ultimate purpose
on the part of petitioner to secure judicial order, which would authorize a change in the civil status of
CHARLES CHRISTIAN, this Court, finds the action improper. The matters desired to be cancelled
and/or changed by petitioner cannot be considered falling under the ambit of the words clerical errors of a
harmless and innocuous nature.

ISSUE: whether corrections of entries in the certificate of live birth pursuant to Article 412 of the Civil
Code, in relation to Rule 108 of the Rules of Court may be allowed even if the errors to be corrected are
substantial and not merely clerical errors of a harmless and innocuous nature.

RULING: YES. Rule 108 of the Revised Rules of Court provides the procedure for cancellation or
correction of entries in the civil registry. The proceedings under said rule may either be summary or
adversary in nature. If the correction sought to be made in the civil register is clerical, then the procedure
to be adopted is summary. If the rectification affects the civil status, citizenship or nationality of a party, it
is deemed substantial, and the procedure to be adopted is adversary. This is our ruling in Republic vs.
Valencia where we held that even substantial errors in a civil registry may be corrected and the true facts
established under Rule 108 provided the parties aggrieved by the error avail themselves of the appropriate
adversary proceeding. An appropriate adversary suit or proceeding is one where the trial court has
conducted proceedings where all relevant facts have been fully and properly developed, where opposing
counsel have been given opportunity to demolish the opposite party's case, and where the evidence has

Remedial Law Review II (CY 2018-2019) – Saturday 1:00pm – 5:00pm: Case Digest Compilation [Atty. Brondial] Page 256
been thoroughly weighed and considered. The Court further laid down the procedural requirements to
make the proceedings under Rule 108 adversary, thus:

The pertinent sections of Rule 108 provide:


SEC. 3. Parties.--When cancellation or correction of an entry in the civil register is sought, the civil
registrar and all persons who have or claim any interest which would be affected thereby shall be made
parties to the proceeding.
SEC. 4. Notice and publication.--Upon the filing of the petition, the court shall, by an order, fix the time
and place for the hearing of the same, and cause reasonable notice thereof to be given to the persons
named in the petition. The court shall also cause the order to be published once in a week for three (3)
consecutive weeks in a newspaper of general circulation in the province.
SEC. 5. Opposition.--The civil registrar and any person having or claiming any interest under the entry
whose cancellation or correction is sought may, within fifteen (15) days from notice, file his opposition
thereto.

Thus, the persons who must be made parties to a proceeding concerning the cancellation or correction of
an entry in the civil register are--(1) the civil registrar, and (2) all persons who have or claim any interest
which would be affected thereby. Upon the filing of the petition, it becomes the duty of the court to--(1)
issue an order fixing the time and place for the hearing of the petition, and (2) cause the order for hearing
to be published once a week for three (3) consecutive weeks in a newspaper of general circulation in the
province. The following are likewise entitled to oppose the petition:--(1) the civil registrar, and (2) any
person having or claiming any interest under the entry whose cancellation or correction is sought.If all
these procedural requirements have been followed, a petition for correction and/or cancellation of entries
in the record of birth even if filed and conducted under Rule 108 of the Revised Rules of Court can no
longer be described as 'summary.'

It is true in the case at bar that the changes sought to be made by petitioner are not merely clerical or
harmless errors but substantial ones as they would affect the status of the marriage between petitioner and
Carlos Borbon, as well as the legitimacy of their son, Charles Christian. Changes of such nature, however,
are now allowed under Rule 108 in accordance with our ruling in Republic vs. Valencia provided that the
appropriate procedural requirements are complied with. The records show that upon receipt of the
petition, the trial court issued a notice of hearing setting the hearing on June 26, 1997 at 8:30 in the
morning at Room 118, Hall of Justice, Quezon City. The trial court likewise ordered the publication of
said notice once a week for three (3) consecutive weeks in a newspaper of general circulation and its
posting in selected places in Metro Manila. The notice stated that the petitioner shall prove her petition
during said hearing and all other persons having or claiming any interest thereon shall also appear and
show if there is any reason why the petition should not be granted. Respondents Carlos VillenaBorbon,
the Local Civil Registrar of Quezon City and the Solicitor General were all furnished with a copy of the
notice of hearing together with a copy of the petition. On June 26, 1997, the trial court issued a second
order giving the petitioner an opportunity to show compliance with the jurisdictional requirements and to
present evidence during the hearing set on July 23, 1997. The foregoing satisfy all the requirements of
Rule 108 to make it an adversary proceeding. It was therefore an error for the trial court to dismiss the
petition motuproprio without allowing the petitioner to present evidence to support her petition and all the
other persons who have an interest over the matter to oppose the same.

Remedial Law Review II (CY 2018-2019) – Saturday 1:00pm – 5:00pm: Case Digest Compilation [Atty. Brondial] Page 257
H.2. Republic vs. Kho

Remedial Law Review II (CY 2018-2019) – Saturday 1:00pm – 5:00pm: Case Digest Compilation [Atty. Brondial] Page 258
H.3. In Re: Petition for Change of Name and/or Correction of Entry in the Civil Registry of Julian
Lin Carulasan Wang,CHANGE OF NAME VS. CORRECTION/CANCELLATION OF ENTRIES

G.R. No. 159966; March 30, 2005, 454 SCRA 155; TINGA,J.

FACTS: Petitioner Julian Lin Carulasan Wang, a minor, represented by his mother Anna Lisa Wang,
filed a petition for change of name and/or correction/cancellation of entry in the Civil Registry. Petitioner
sought to drop his middle name and have his registered name changed from Julian Lin Carulasan Wang to
Julian Lin Wang.

The RTC established the following facts:


Julian Lin Carulasan Wang was born in Cebu City on February 20, 1998 to parents Anna Lisa Wang and
Sing-Foe Wang who were then not yet married to each other. When his parents subsequently got married
on September 22, 1998, . . . they executed a deed of legitimation of their son so that the child's name was
changed from Julian Lin Carulasan to Julian Lin Carulasan Wang. . . .

The parents of Julian Lin Carulasan Wang plan to stay in Singapore for a long time because they will let
him study there. . . . Since in Singapore middle names or the maiden surname of the mother are not
carried in a person's name, they anticipate that Julian Lin Carulasan Wang will be discriminated against
because of his current registered name which carries a middle name.

The RTC rendered a decision denying the petition. The trial court found that the reason given for the
change of name sought in the petition — that is, that petitioner Julian may be discriminated against when
studies in Singapore because of his middle name — did not fall within the grounds recognized by law.
Petitioner filed a motion for reconsideration of the decision but this was denied.
Petitioner then filed this Petition for Review on Certiorari (Under Rule 45) arguing that the trial court has
decided a question of substance not theretofore determined by the Court, that is: whether or not dropping
the middle name of a minor child is contrary to Article 174 of the Family Code.

ISSUE:Whether or not dropping the middle name of a minor child is contrary to Article 174 7 of the
Family Code.

RULING: Yes. Petition denied. The Supreme Court affirmed the decision of the trial court.
The touchstone for the grant of a change of name is that there be 'proper and reasonable cause' for which
the change is sought. To justify a request for change of name, petitioner must show not only some proper
or compelling reason therefore but also that he will be prejudiced by the use of his true and official name.
Among the grounds for change of name which have been held valid are: (a) when the name is ridiculous,
dishonorable or extremely difficult to write or pronounce; (b) when the change results as a legal
consequence, as in legitimation; (c) when the change will avoid confusion; (d) when one has continuously
used and been known since childhood by a Filipino name, and was unaware of alien parentage; (e) a
sincere desire to adopt a Filipino name to erase signs of former alienage, all in good faith and without
prejudicing anybody; and (f) when the surname causes embarrassment and there is no showing that the
desired change of name was for a fraudulent purpose or that the change of name would prejudice public
interest.

Decided cases in this jurisdiction involving petitions for change of name usually deal with requests for
change of surname. There are only a handful of cases involving requests for change of the given name
and none on requests for changing or dropping of the middle name. Does the law allow one to drop the
middle name from his registered name? We have to answer in the negative.
Our laws on the use of surnames state that legitimate and legitimated children shall principally use the
surname of the father. The Family Code gives legitimate children the right to bear the surnames of the
father and the mother, while illegitimate children shall use the surname of their mother, unless their father
recognizes their filiation, in which case they may bear the father's surname.

Applying these laws, an illegitimate child whose filiation is not recognized by the father bears only a
given name and his mother's surname, and does not have a middle name. The name of the unrecognized
illegitimate child therefore identifies him as such. It is only when the illegitimate child is legitimated by
the subsequent marriage of his parents or acknowledged by the father in a public document or private
handwritten instrument that he bears both his mother's surname as his middle name and his father's
surname as his surname, reflecting his status as a legitimated child or an acknowledged illegitimate child.

Remedial Law Review II (CY 2018-2019) – Saturday 1:00pm – 5:00pm: Case Digest Compilation [Atty. Brondial] Page 259
Accordingly, the registration in the civil registry of the birth of such individuals requires that the middle
name be indicated in the certificate. The registered name of a legitimate, legitimated and recognized
illegitimate child thus contains a given or proper name, a middle name, and a surname.

Weighing petitioner's reason of convenience for the change of his name against the standards set in the
cases he cites to support his contention would show that his justification is amorphous, to say the least,
and could not warrant favorable action on his petition.

In the case at bar, the only reason advanced by petitioner for the dropping his middle name is
convenience. However, how such change of name would make his integration into Singaporean society
easier and convenient is not clearly established. That the continued use of his middle name would cause
confusion and difficulty does not constitute proper and reasonable cause to drop it from his registered
complete name.

In addition, petitioner is only a minor. Considering the nebulous foundation on which his petition for
change of name is based, it is best that the matter of change of his name be left to his judgment and
discretion when he reaches the age of majority. As he is of tender age, he may not yet understand and
appreciate the value of the change of his name and granting of the same at this point may just prejudice
him in his rights under our laws.

Remedial Law Review II (CY 2018-2019) – Saturday 1:00pm – 5:00pm: Case Digest Compilation [Atty. Brondial] Page 260
H.4. MA. CRISTINA TORRES BRAZA, PAOLO JOSEF T. BRAZA and JANELLE ANN T.
BRAZA,Petitioners, v.THE CITY CIVIL REGISTRAR OF HIMAMAYLAN CITY, NEGROS
OCCIDENTAL, minor PATRICK ALVIN TITULAR BRAZA, Respondents.

G.R. NO. 181174; December 4, 2009

FACTS: Ma. Cristina Torres and Pablo Braza were married on January 4, 1978. They had 3 children
named Paolo Jose and Janelle Ann and Gian Carlo.On April 15, 2002, Pablo died in a vehicular accident
in Java, Indonesia. During the wake, Lucille Titular introduced her and Pablo’s alleged child, the minor
Patrick Alvin Titular Braza.Ma. Cristina, to confirm the truth, obtained a copy of Patrick’s birth
certificate from the Civil Registrar of Himamaylan City, Negros Occidental.

Name of Child: PATRICK ALVIN CELESTIAL

TITULAR
Date of Birth: 01 January 1996
Mother: Lucille Celestial Titular
Father: Pablito S. Braza
Date Received at the
Local Civil Registrar: January 13, 1997
Annotation: "Late Registration"

Annotation/Remarks: "Acknowledge (sic) by the father PablitoBraza on January 13,


1997"

Remarks: Legitimated by virtue of subsequent marriage of parents on April 22,


1998 at Manila. Henceforth, the child shall be known as Patrick Alvin Titular Braza 

Furthermore, Ma. Cristina obtained a copy of marriage contract of Lucille and Pablo with a marriage date
of April 22, 1998.

Hence, Ma.Cristina and her 3 children filed a petition to the RTC of Negros Occidental to correct the
entries in the birth record of Patrick in the Local Register as Patrick could not have been legitimated due
to the marriage between Lucille and Pablo as the said marriage was bigamous.

The petition prayed for

1. The correction of entries in Patrick’s birth record with respect to his legitimation,
acknowledgement of Pablo and the use of the surname “Braza”;
2. The directive to submit Patrick to DNA testing;
3. The declaration of nullity of Patrick’s legitimation and consequently, the declaration of marriage
of Lucille and Pablo as bigamous.

RTC dismissed the petition without prejudiced for lack of jurisdiction. In a special proceeding for
correction of entry, the court is not acting as a Family Court; and thus, has no jurisdiction over an action
to annul the marriage between Lucille and Pablo, impugn Patrick’s legitimacy, or order the DNA testing.
Said controversies must be done in an ordinary adversarial action.

Motion for reconsideration was denied and thus this review.

ISSUE: Whether or not in a special proceeding for correction of entry under Rule 108, the court has
jurisdiction to pass upon the validity of marriage and questions on legitimacy?

RULING: NO. Petitioners contends that the court a quo may pass upon the validity of marriage and
questions on legitimacy even in an action to correct entries in the civil registrar as even substantial
errorscan be the subject of a petition under Rule 108. Petition FAILS.
 
In a special proceeding for correction of entry under Rule 108 (Cancellation or Correction of Entries in
the Original Registry), the trial court has no jurisdiction to nullify marriages and rule on legitimacy and
filiation.
 

Remedial Law Review II (CY 2018-2019) – Saturday 1:00pm – 5:00pm: Case Digest Compilation [Atty. Brondial] Page 261
Rule 108 of the Rules of Court vis a vis Article 412 of the Civil Code charts the procedure by which an
entry in the civil registry may be cancelled or corrected. 

The proceeding contemplated therein may generally be used only to correct clerical, spelling,
typographical and other innocuous errors in the civil registry. 

A clerical error is one which is visible to the eyes or obvious to the understanding; an error made by a
clerk or a transcriber; a mistake in copying or writing, or a harmless change such as a correction of name
that is clearly misspelled or of a misstatement of the occupation of the parent. Substantial or contentious
alterations may be allowed only in adversarial proceedings, in which all interested parties are impleaded
and due process is properly observed.
 
The allegations of the petition filed before the trial court clearly show that petitioners seek to nullify the
marriage between Pablo and Lucille on the ground that it is bigamous and impugn Patrick’s filiation and
not just a mere incident to their petition to correct the entries on the birth record.

Petitioners’ causes of action are governed not by Rule 108 but by A.M. No. 02-11-10-SC which took
effect on March 15, 2003, and Art. 171 of the Family Code, respectively, hence, the petition should be
filed in a Family Court as expressly provided in said Code.
 
It is well to emphasize that, doctrinally, validity of marriages as well as legitimacy and filiation can be
questioned only in a direct action seasonably filed by the proper party, and not through collateral attack
such as the petition filed before the court a quo.

DOCTRINE:In a special proceeding for correction of entry under Rule 108 (Cancellation or Correction
of Entries in the Original Registry), the trial court has no jurisdiction to nullify marriages and rule on
legitimacy and filiation.Rule 108 of the Rules of Court vis a vis Article 412 of the Civil Code charts the
procedure by which an entry in the civil registry may be cancelled or corrected. The proceeding
contemplated therein may generally be used only to correct clerical, spelling, typographical and other
innocuous errors in the civil registry. 

A clerical error is one which is visible to the eyes or obvious to the understanding; an error made by a
clerk or a transcriber; a mistake in copying or writing, or a harmless change such as a correction of
name that is clearly misspelled or of a misstatement of the occupation of the parent. Substantial or
contentious alterations may be allowed only in adversarial proceedings, in which all interested parties
are impleaded and due process is properly observed.

Remedial Law Review II (CY 2018-2019) – Saturday 1:00pm – 5:00pm: Case Digest Compilation [Atty. Brondial] Page 262
H.5.ROMMEL JACINTO DANTES SILVERIO, petitionervs.REPUBLIC OF THE
PHILIPPINES, respondent.

G.R. No. 174689;  October 19, 2007; PONENTE: CORONA, J.:

FACTS: On November 26, 2002, Rommel Jacinto Dantes Silverio filed a petition for the change of his
first name and sex in his birth certificate in the RTC of Manila, Branch 8.
 
Rommel was born in the City of Manila to the spouses Melecio Petines Silverio and Anita Aquino Dantes
on April 4, 1962. His name was registered as "Rommel Jacinto Dantes Silverio" in his birth certificate.
His sex was registered as "male."

 Rommel, according to him, is amale transsexualthat is,"anatomically male but feels, thinks and acts
as a female"and that he had always identified himself with girls since childhood.
 
On January 27, 2001 he underwent sex reassignment surgery in Bangkok, Thailand. He was thereafter
examined by Dr. Marcelino Reysio-Cruz, Jr., a plastic and reconstruction surgeon in the Philippines, who
issued a medical certificate attesting that he had in fact undergone the procedure.
 
From then on, he lived as a female and was in fact engaged to be married. He then sought to have his
name in his birth certificate changed from "Rommel Jacinto" to "Mely," and his sex from "male" to
"female."
 
ISSUE: Whether or not Rommel can legally change his name and sex in his birth certificate on the
ground of sex reassignment.

RULING: NO, he cannot legally change his name and sex in his birth certificate on the ground of sex
reassignment.

RATIO: SC gave 3 reasons: (1) A person's first name cannot be changed on the ground of sex
reassignment; (2) No law allows the change of entry in the birth certificate as to sex on the ground of sex
reassignment; and (3) Neither may entries in the birth certificate as to first name or sex be changed on the
ground of equity.

1. A Person's First Name Cannot Be Changed On The Ground Of Sex Reassignment


 
The State has an interest in the names borne by individuals and entities for purposes
of identification.

A change of name is a privilege, not a right.


 
 
RA 9048 does not sanction a change of first name on the ground of sex reassignment. Rather than
avoiding confusion, changing petitioner's first name for his declared purpose may only create
grave complications in the civil registry and the public interest. The grounds allowed by RA 9048
in the change of name does not include sex reassignment.

2. No Law Allows The Change Of Entry In The Birth Certificate As To Sex On The Ground
Of Sex Reassignment

 
RA 9048: Clerical or typographical errors are allowed to be corrected.A correction in the civil
registry involving the change of sex is not a mere clerical or typographical error.It is a
substantial change for which the applicable procedure is Rule 108 of the Rules of Court.

 
The entries foreseen in Article 412 of the Civil Code and correctable under Rule 108 of the Rules
of Court are those provided in Articles 407 and 408 of the Civil Code

3. Neither May Entries In The Birth Certificate As To First Name Or Sex Be Changed On The
Ground Of Equity
 

Remedial Law Review II (CY 2018-2019) – Saturday 1:00pm – 5:00pm: Case Digest Compilation [Atty. Brondial] Page 263
The changes sought by petitioner will have serious and wide-ranging legal and public policy
consequences.

Remedial Law Review II (CY 2018-2019) – Saturday 1:00pm – 5:00pm: Case Digest Compilation [Atty. Brondial] Page 264
H.6. REPUBLIC OF THE PHILIPPINES vs. JENNIFER CAGANDAHAN

GR No. 166676; September 12, 2008; QUISUMBING, J.:

FACTS: Jennifer Cagandahan, registered as female in Certificate of Live Birth, filed before the RTC Br
33 of Siniloan, Laguna a Petition for Correction of Entries in Birth Certificate of her name, from
Jennifer B. Cagandahan to Jeff Cagandahan, and her gender, from female to male. It appeared that
Cagandahan is suffering from Congenital Adrenal Hyperplasia (possessing both male and female
characteristics), a rare medical condition. Jennifer Cagandahan grew up with secondary male
characteristics.

To further her petition, Cagandahan presented in court, medical certificate evidencing that she is suffering
from Congenital Adrenal Hyperplasia, issued by Dr. Michael Sionzon of the Department of Psychiatry,
University of the Philippines-Philippine General Hospital, who, in addition, explained that “Cagandahan
genetically is female but because her body secretes male hormones, her female organs did not develop
normally, thus, possessing organs of both male and female.” The lower court decided in favour of
Cagandahan but the Office of the Solicitor General appealed before the Supreme Court invoking that the
same was a violation of Rules 103 and 108 of the Rules of Court because the said petition did not implead
the local civil registrar.

ISSUE: Whether correction of entries in her birth certificate be granted.

RULING: We agree that there is substantial compliance with Rule 108 when respondent furnished a
copy of the petition to the local civil registrar.

The determination of a persons sex appearing in his birth certificate is a legal issue and the court must
look to the statutes. In this connection, Article 412 of the Civil Code provides:
 
ART. 412. No entry in a civil register shall be changed or corrected without a judicial order.
 
Together with Article 376 of the Civil Code, this provision was amended by Republic Act No. 9048 in so
far as clerical or typographical errors are involved. The correction or change of such matters can now be
made through administrative proceedings and without the need for a judicial order. In effect, Rep. Act
No. 9048 removed from the ambit of Rule 108 of the Rules of Court the correction of such errors. Rule
108 now applies only to substantial changes and corrections in entries in the civil register.

Under Rep. Act No. 9048, a correction in the civil registry involving the change of sex is not a mere
clerical or typographical error. It is a substantial change for which the applicable procedure is Rule 108 of
the Rules of Court.

The entries envisaged in Article 412 of the Civil Code and correctable under Rule 108 of the Rules of
Court are those provided in Articles 407 and 408 of the Civil Code:

ART. 407. Acts, events and judicial decrees concerning the civil status of persons shall be
recorded in the civil register.
 
ART. 408. The following shall be entered in the civil register:
 
(1) Births; (2) marriages; (3) deaths; (4) legal separations; (5) annulments of marriage; (6)
judgments declaring marriages void from the beginning; (7) legitimations; (8) adoptions; (9)
acknowledgments of natural children; (10) naturalization; (11) loss, or (12) recovery of
citizenship; (13) civil interdiction; (14) judicial determination of filiation; (15) voluntary
emancipation of a minor; and (16) changes of name.

The acts, events or factual errors contemplated under Article 407 of the Civil Code include even those
that occur after birth.

Respondent undisputedly has CAH. This condition causes the early or inappropriate appearance of male
characteristics. A person, like respondent, with this condition produces too much androgen, a male
hormone. A newborn who has XX chromosomes coupled with CAH usually has a (1) swollen clitoris
with the urethral opening at the base, an ambiguous genitalia often appearing more male than female; (2)
normal internal structures of the female reproductive tract such as the ovaries, uterus and fallopian tubes;

Remedial Law Review II (CY 2018-2019) – Saturday 1:00pm – 5:00pm: Case Digest Compilation [Atty. Brondial] Page 265
as the child grows older, some features start to appear male, such as deepening of the voice, facial hair,
and failure to menstruate at puberty. About 1 in 10,000 to 18,000 children are born with CAH.

CAH is one of many conditions that involve intersex anatomy. During the twentieth century, medicine
adopted the term intersexuality to apply to human beings who cannot be classified as either male or
female. The term is now of widespread use. According to Wikipedia, intersexuality is the state of a living
thing of a gonochoristic species whose sex chromosomes, genitalia, and/or secondary sex characteristics
are determined to be neither exclusively male nor female. An organism with intersex may have biological
characteristics of both male and female sexes.

Intersex individuals are treated in different ways by different cultures. In most societies, intersex
individuals have been expected to conform to either a male or female gender role. Since the rise of
modern medical science in Western societies, some intersex people with ambiguous external genitalia
have had their genitalia surgically modified to resemble either male or female genitals. More commonly,
an intersex individual is considered as suffering from a disorder which is almost always recommended to
be treated, whether by surgery and/or by taking lifetime medication in order to mold the individual as
neatly as possible into the category of either male or female.

In deciding this case, we consider the compassionate calls for recognition of the various degrees of
intersex as variations which should not be subject to outright denial. It has been suggested that there is
some middle ground between the sexes, a no-mans land for those individuals who are neither truly male
nor truly female. The current state of Philippine statutes apparently compels that a person be classified
either as a male or as a female, but this Court is not controlled by mere appearances when nature itself
fundamentally negates such rigid classification.

Biologically, nature endowed respondent with a mixed (neither consistently and categorically female nor
consistently and categorically male) composition. Respondent has female (XX) chromosomes. However,
respondents body system naturally produces high levels of male hormones (androgen). As a result,
respondent has ambiguous genitalia and the phenotypic features of a male.

Ultimately, we are of the view that where the person is biologically or naturally intersex the determining
factor in his gender classification would be what the individual, like respondent, having reached the age
of majority, with good reason thinks of his/her sex. Respondent here thinks of himself as a male and
considering that his body produces high levels of male hormones (androgen) there is preponderant
biological support for considering him as being male. Sexual development in cases of intersex persons
makes the gender classification at birth inconclusive. It is at maturity that the gender of such persons, like
respondent, is fixed.

In the absence of a law on the matter, the Court will not dictate on respondent concerning a matter so
innately private as ones sexuality and lifestyle preferences, much less on whether or not to undergo
medical treatment to reverse the male tendency due to CAH. The Court will not consider respondent as
having erred in not choosing to undergo treatment in order to become or remain as a female. Neither will
the Court force respondent to undergo treatment and to take medication in order to fit the mold of a
female, as society commonly currently knows this gender of the human species. Respondent is the one
who has to live with his intersex anatomy. To him belongs the human right to the pursuit of happiness and
of health. Thus, to him should belong the primordial choice of what courses of action to take along the
path of his sexual development and maturation. In the absence of evidence that respondent is an
incompetent and in the absence of evidence to show that classifying respondent as a male will harm other
members of society who are equally entitled to protection under the law, the Court affirms as valid and
justified the respondents position and his personal judgment of being a male.

As for respondents change of name under Rule 103, this Court has held that a change of name is not a
matter of right but of judicial discretion, to be exercised in the light of the reasons adduced and the
consequences that will follow. The trial courts grant of respondents change of name from Jennifer to Jeff
implies a change of a feminine name to a masculine name. Considering the consequence that respondents
change of name merely recognizes his preferred gender, we find merit in respondents change of name.
Such a change will conform with the change of the entry in his birth certificate from female to male.

WHEREFORE, the Republics petition is DENIED. The Decision dated January 12, 2005 of the
Regional Trial Court, Branch 33 of Siniloan, Laguna, is AFFIRMED. No pronouncement as to costs.SO
ORDERED.

Remedial Law Review II (CY 2018-2019) – Saturday 1:00pm – 5:00pm: Case Digest Compilation [Atty. Brondial] Page 266
Remedial Law Review II (CY 2018-2019) – Saturday 1:00pm – 5:00pm: Case Digest Compilation [Atty. Brondial] Page 267
H.7.REPUBLIC OF THE PHILIPPINES vs. DR. NORMA S. LUGSANAY UY

G.R. NO. 198010; AUGUST 12, 2013; PERALTA, J.:

FACTS: On March 8, 2004, respondent, Dr. Lugsanay Uy, filed a petition for Correction of Entry in her
Certificate of Live Birth. Impleaded as respondent is the Local Civil Registrar of Gingoog City. She
alleged that she was born on February 8, 1952 and is the illegitimate daughter of Sy Ton and Sotera
Lugsanay. Her Certificate of Live Birth shows that her full name is “Anita Sy” when in fact she is
allegedly known to her friends and family as “Norma S. Lugsanay”. She further claimed that her school
records, Professional Regulation Commission (PRC) Board of Medicine Cerificate, and passport bear the
name “Norma S. Lugsanay.” She also alleged that she is illegitimate child considering that her parents
were never married, so she had to follow the surname of her mother. She also contended that she is a
Filipino Citizen and not Chinese, and all her siblings bear the surname Lugsanay and are all Filipinos.
Respondent allegedly filed earlier petition for correction of entries with the Office of the Local Civil
Registrar of Gingoong City to effect the corrections on her name and Citizenship which was supposedly
granted. However, the National Statistics Office records did not bear such changes. Hence, the petition
before the RTC.

On May 13, 2004, the RTC issued an order finding the petition to be sufficient in form and substance and
setting the case for hearing with the directive that the order be published in a newspaper of general
circulation in the City of Gingoong and Province of Misamis Oriental at least once a week for three
consecutive weeks at the expense of respondent, and that the order of petition be furnished the Office of
the Solicitor General and City Prosecutor’s Office for their information and guidance. The RTC issued an
Order in favor of respondent.

On February 18, 2011, the CA affirmed in toto RTC Order. The CA held that respondent’s failure to
implead other indispensable parties was cured upon the publication of the Order setting the case for
hearing in a newspaper of general circulation for three consecutive weeks and by serving a copy of the
notice to the Local Civil Registrar, the OSG and the City Prosecutor’s Office.

ISSUE:Whether or not the publication of the Order setting the case for hearing cured the failure to
implead other indispensable parties.

RULING: No. the cancellation or correction of entries in the civil registry is governed by Rule 108 of the
Rules of Court, to wit:

Sec. 3. Parties. – when cancellation or correction of an entry in the civil register is sought, the civil
registrar and all persons who have or claim any interest which would be affected thereby shall be made
parties in the proceeding.

Sec. 4. Notice and Publication – Upon the filing of the petition, the court shall by order, fix the time and
place for the hearing of the same, and cause reasonable notice thereof to be given to the persons named in
the petition. The court shall also cause the order to be published once a week for three consecutive weeks
in a newspaper of general circulation in the province.

Sec 5. Opposition – the civil registrar and any person having or claiming any interest under the entry
whose cancellation or correction sought may within fifteen days from the notice of petition, or from the
last date of publication of such notice file his opposition thereto.

In this case, respondent sought the correction of entries in her Birth Certificate, particularly those
pertaining to her first name, surname and citizenship. She sought the correction allegedly to reflect the
name which she had been known since childhood, including her legal documents such as passport and
school professional records. She likewise relied on the birth certificates of her full blood siblings who
bear the same Lugsanay instead of Sy and citizenship of Filipino instead of Chinese. The changes
however, are obviously not mere clerical as they touch on respondent’s filiation and citizenship. She, is in
effect, changes her status from legitimate to illegitimate and in changing her citizenship from Chines to
Filipino, the same affects her rights and obligations in this country. Clearly, changes are substantial.

The fact that the notice of hearing was published in a newspaper of circulation and notice thereof was
served upon the State will not change the nature of the proceedings taken. A reading of Sections 4 and 5,
Rule 108 of the Rules of Court shows that the Rules mandate two sets of notices to different potential
oppositors: one given to the persons named in the petition and another given to other persons who are not

Remedial Law Review II (CY 2018-2019) – Saturday 1:00pm – 5:00pm: Case Digest Compilation [Atty. Brondial] Page 268
named in the petition but nonetheless may be considered interested or affected parties. Summons must,
therefore, be served not for the purpose of vesting the court’s jurisdiction but to comply with the
requirements of fair play and due process to afford the person concerned the opportunity to protect his
interest if he so chooses.

While there may be cases where the Court held that the failure to implead and notify the affected or
interested parties may be cured by the publication of the notice of hearing, earnest efforts were made by
petitioners in bringing to court all possible interested parties. Such failure was likewise excused where the
interested parties themselves initiated the corrections proceedings, when there is no actual or presumptive
awareness of the existence of the interested parties or when a party is inadvertently left out.

DOCTRINE:When a petition for cancellation or correction of an entry in the civil register involves
substantial and controversial alterations, including those citizenship, legitimacy of paternity of filiation,
or legitimacy of marriage, a strict compliance with the requirements of Rule 108 of the Rules of Court is
mandated.

Remedial Law Review II (CY 2018-2019) – Saturday 1:00pm – 5:00pm: Case Digest Compilation [Atty. Brondial] Page 269
H.8. MINORU FUJIKI, vs. MARIA PAZ GALELA MARINAY

G.R.No.196049;.June26,2013;.CARPIO,J.

FACTS: Petitioner Minoru Fujiki (Fujiki) is a Japanese national who married respondent Maria Paz
GalelaMarinay (Marinay) in the Philippines on 23 January 2004. The marriage did not sit well with
petitioner’s parents. Thus, Fujiki could not bring his wife to Japan where he resides. Eventually, they lost
contact with each other.

In 2008, Marinay met another Japanese, Shinichi Maekara (Maekara). Without the first marriage being
dissolved, Marinay and Maekara were married on 15 May 2008 in Quezon City, Philippines. Maekara
brought Marinay to Japan. However, Marinay allegedly suffered physical abuse from Maekara. She left
Maekara and started to contact Fujiki.

Fujiki and Marinay met in Japan and they were able to reestablish their relationship. In 2010, Fujiki
helped Marinay obtain a judgment from a family court in Japan which declared the marriage between
Marinay and Maekara void on the ground of bigamy. On 14 January 2011, Fujiki filed a petition in the
RTC entitled: “Judicial Recognition of Foreign Judgment (or Decree of Absolute Nullity of Marriage).”

The decision of the lower courts (RTC): dismissed the petition for "Judicial Recognition of Foreign
Judgment (or Decree of Absolute Nullity of Marriage)" based on improper venue and the lack of
personality of petitioner, Minoru Fujiki, to file the petition.

ISSUES:
1. Whether the Rule on Declaration of Absolute Nullity of Void Marriages and Annulment of Voidable
Marriages (A.M. No. 02-11-10-SC) is applicable.

2. Whether a husband or wife of a prior marriage can file a petition to recognize a foreign judgment
nullifying the subsequent marriage between his or her spouse and a foreign citizen on the ground of
bigamy.

3. Whether the Regional Trial Court can recognize the foreign judgment in a proceeding for cancellation
or correction of entries in the Civil Registry under Rule 108 of the Rules of Court.

RULING:
1. No. Rule on Declaration of Absolute Nullity of Void Marriages and Annulment of Voidable Marriages
(A.M. No. 02-11-10-SC) does not apply in a petition to recognize a foreign judgment relating to the status
of a marriage where one of the parties is a citizen of a foreign country. Moreover, in Juliano-Llave v.
Republic, this Court held that the rule in A.M. No. 02- 11-10-SC that only the husband or wife can file a
declaration of nullity or annulment of marriage “does not apply if the reason behind the petition is
bigamy.” While the Philippines has no divorce law, the Japanese Family Court judgment is fully
consistent with Philippine public policy, as bigamous marriages are declared void from the beginning
under Article 35(4) of the Family Code. Bigamy is a crime under Article 349 of the Revised Penal Code.
Thus, Fujiki can prove the existence of the Japanese Family Court judgment in accordance with Rule 132,
Sections 24 and 25, in relation to Rule 39, Section 48(b) of the Rules of Court.

2. Yes, the recognition of the foreign divorce decree may be made in a Rule 108 proceeding itself, as the
object of special proceedings (such as that in Rule 108 of the Rules of Court) is precisely to establish the
status or right of a party or a particular fact.”Rule 108, Section 1 of the Rules of Court states:
Sec. 1. Who may file petition. — Any person interested in any act, event, order or decree concerning
the civil status of persons, which has been recorded in the civil register, may file a verified petition for the
cancellation or correction of any entry relating thereto, with the Regional Trial Court of the province
where the corresponding civil registry is located. (Emphasis supplied)

There is no doubt that the prior spouse has a personal and material interest in maintaining the integrity of
the marriage he contracted and the property relations arising from it.

4. Yes, there is neither circumvention of the substantive and procedural safeguards of marriage
under Philippine law, nor of the jurisdiction of Family Courts under R.A. No. 8369. Recognition
of a foreign judgment is not an action to nullify a marriage. It is an action for Philippine courts to
recognize the effectivity of a foreign judgment, which presupposes a case, which was already
tried and decided under foreign law.

Remedial Law Review II (CY 2018-2019) – Saturday 1:00pm – 5:00pm: Case Digest Compilation [Atty. Brondial] Page 270
5. In the recognition of foreign judgments, Philippine courts are incompetent to substitute their
judgment on how a case was decided under foreign law. They cannot decide on the “family rights
and duties, or on the status, condition and legal capacity” of the foreign citizen who is a party to
the foreign judgment. Thus, Philippine courts are limited to the question of whether to extend the
effect of a foreign judgment in the Philippines. In a foreign judgment relating to the status of a
marriage involving a citizen of a foreign country, Philippine courts only decide whether to extend
its effect to the Filipino party, under the rule of lexnationalii expressed in Article 15 of the Civil
Code.

For this purpose, Philippine courts will only determine (1) whether the foreign judgment is inconsistent
with an overriding public policy in the Philippines; and (2) whether any alleging party is able to prove an
extrinsic ground to repel the foreign judgment, i.e. want of jurisdiction, want of notice to the party,
collusion, fraud, or clear mistake of law or fact. If there is neither inconsistency with public policy nor
adequate proof to repel the judgment, Philippine courts should, by default, recognize the foreign judgment
as part of the comity of nations.

DOCTRINE: Court can recognize the foreign judgment in a proceeding for cancellation or correction of
entries in the Civil Registry under Rule 108 of the Rules of Court, Rule 1, Section 3 of such Rule, provides
that "a special proceeding is a remedy by which a party seeks to establish a status, a right, or a
particular fact." Rule 108 creates a remedy to rectify facts of a person’s life which are recorded by the
State pursuant to the Civil Register Law or Act No. 3753. These are facts of public consequence such as
birth, death or marriage, which the State has an interest in recording furthermore, Recognition of a
foreign judgment is not an action to nullify a marriage but an action for Philippine courts to recognize
the effectivity of a foreign judgment, which presupposes a case, which was already tried and decided
under foreign law.

Remedial Law Review II (CY 2018-2019) – Saturday 1:00pm – 5:00pm: Case Digest Compilation [Atty. Brondial] Page 271
H.9. REPUBLIC OF THE PHILIPPINES v. MELINDA OLAYBAR

G.R. NO. 189538; FEBRUARY 10, 2014

FACTS:Olaybar requested from NSO a CENOMAR as one of the requirements for her marriage with her
boyfriend of five years. Upon receipt thereof, she discovered that she was already married to a certain Ye
Son Sune, a Korean national, on June 24, 2002, at the Office of the MTCC, Palace of Justice. She denied
having contracted said marriage and claimed that she did not know the alleged husband; that she did not
appear before the solemnizing officer; and that the signature appearing in the marriage certificate is not
hers. She, thus, filed a Petition for Cancellation of Entries in the Marriage Contract, especially the entries
in the wife portion there of. Olaybar impleaded the Local Registry of Cebu City, as well as her alleged
husband as parties to the case.

During trial, Olaybar testified on her behalf and explained that she could not have appeared before Judge
MamertoCaliflores, the supposed solemnizing officer, at the time the marriage was allegedly celebrated
because she was then in Makati working as a medical distributor in HansoPharma. Olaybar also presented
a witness, EufrocinaNatinga, an employee of the MTCC, Branch 1, who confirmed the marriage had
celebrated in their office but claimed that the alleged wife who appeared was not Olaybar. Lastly, a
document examiner testified that the signature appearing in the marriage contract was forged.

Contrary to petitioner’s stand, the RTC held that it had jurisdiction to take cognizance of cases for
correction of entries even on substantial errors under Rule 108 of the ROC being the appropriate
adversary proceeding required. Considering that an unknown person used Olaybar’s identity to contract
marriage with a Korean national, it would not be feasible to institute an action for declaration of nullity of
marriage since it is not one of the void marriages under Article 35 and 36 of the Family Code. Olaybar
now comes before the court in this Petition for Review on Certiorari under Rule 45.

ISSUE: Whether or not a petition for correction of entry in the civil registry may be availed of to correct
the marriage status of a person who claimed that she had never been married.

RULING: Yes.

While we maintain that Rule 108 cannot be availed of to determine the validity of marriage, we cannot
nullify the proceedings before the trial court where all the parties had been given the opportunity to
contest the allegations of respondent; the procedures were followed, and all the evidence of the parties
had already been admitted and examined. Respondent indeed sought, not the nullification of marriage as
there was no marriage to speak of, but the correction of the record of such marriage to reflect the truth as
set forth by the evidence. Otherwise stated, in allowing the correction of the subject certificate of
marriage by cancelling the wife portion thereof, the trial court did not, in any way, declare the marriage
void as there was no marriage to speak of.

Aside from the certificate of marriage, no such evidence was presented to show the existence of
marriage.Rather, respondent showed by overwhelming evidence that no marriage was entered into and
that she was not even aware of such existence. The testimonial and documentary evidence clearly
established that the only "evidence" of marriage which is the marriage certificate was a forgery

Remedial Law Review II (CY 2018-2019) – Saturday 1:00pm – 5:00pm: Case Digest Compilation [Atty. Brondial] Page 272
H.1. Ondevs Civil Registrar of Las Pinas City,

G.R. No. 197174; September 10, 2014

FACTS: Petitioner filed a petition for correction of entries in his certificate of live birth before the RTC
and named respondent Office of the Local Civil Registrar of Las Pinas City as sole respondent. Petitioner
alleged that he is the illegitimate child of his parents Guillermo A. Onde and Matilde DC Pakingan, but
his birth certificate stated that his parents were married.

His birth certificate also stated that his mother's first name is Tely and that his first name is Franc Ler. He
prayed that the following entries on his birth certificate be corrected

RTC dismissed the petition for correction of entries on the ground thatit is insufficient in form and
substance. It ruled that the proceedings must be adversarial since the first correction is substantial in
nature and would affect petitioner’s status as a legitimate child. It was further held that the correction in
the first name of petitioner and his mother can be done by the city civil registrar under Republic Act
(R.A.) No. 9048.

ISSUES:
(1) whether the RTC erred in ruling that the correction on the first name of petitioner and his mother can
be done by the city civil registrar under R.A. No. 9048;
(2) whether the RTC erred in ruling that correcting the entry on petitioner’s birth certificate that his
parents were married on December 23, 1983 in Bicol to "not married" is substantial in nature requiring
adversarial proceedings;
(3) whether the RTC erred in dismissing the petition for correction of entries; and
(4) whether the RTC erred in ruling that there is no proof that petitioner’s parents were not married on
December 23, 1983. – RULING: It is no longer necessary to dwell on the last issue as petitioner will have
his opportunity to prove his claim that his parents were not married on December 23, 1983 when he files
the new petition for the purpose.

-Petitioner argues that Rule 108 ofthe Rules of Court allows a substantial correction of entries in the civil
registry, stating that in Eleosida v. Local Civil Registrar of Quezon City,the case cited by the RTC, we
have actually ruled that substantial changes in the civil registry are now allowed under Rule 108 of the
Rules of Court. He likewise adds that proof that his parents were not married will be presented during the
trial, not during the filing of the petition for correction of entries.

-Office of the Solicitor General (OSG) contends that the RTC correctly dismissed the petition for
correction of entries. It points out that the first names of petitioner and his mother can be corrected thru
administrative proceedings under R.A. No. 9048. Such correction of the entry on petitioner’s birth
certificate that his parents were married on December 23, 1983 in Bicol to "not married" is a substantial
correction affecting his legitimacy. Hence, it must be dealt with in adversarial proceedings where all
interested parties are impleaded.

RULING:

1. RTC is correct in ruling that the first name of petitioner and his mother as appearing in his birth
certificate can be corrected by the city civil registrar under R.A. No. 9048  Indeed, under Section
15 of R.A. No. 9048, clerical or typographical errors on entries in a civil register can be corrected and
changes of first name can be done by the concerned city civil registrar without need of a judicial order.
Aforesaid Section 1, as amended by R.A. No. 10172, now reads: SECTION 1. Authority to Correct
Clerical or Typographical Error and Change of First Name or Nickname. – No entry in a civil register
shall be changed or correctedwithout a judicial order, except for clerical or typographical errors and
change of first name or nickname, the day and month in the dateof birth or sex of a person where it is
patently clear that there was a clerical or typographical error or mistake in the entry, which can be
corrected or changed by the concerned city or municipal civil registraror consul general in accordance
with the provisions of this Act and its implementing rules and regulations.

2. RTC is correct in ruling that correcting the entry on petitioner’s birth certificate that his parents
were married on December 23, 1983 in Bicol to "not married" is a substantial correction requiring
adversarial proceedings. Said correction is substantial as it will affect his legitimacy and convert him
from a legitimate child to an illegitimate one. In Republic v. Uy, we held that corrections of entries in the
civil register including those on citizenship, legitimacy of paternity or filiation, or legitimacy of marriage,

Remedial Law Review II (CY 2018-2019) – Saturday 1:00pm – 5:00pm: Case Digest Compilation [Atty. Brondial] Page 273
involve substantial alterations. Substantial errors in a civil registry may be corrected and the true facts
established provided the parties aggrieved by the error avail themselves of the appropriate adversary
proceedings.

3. RTC is correct in dismissing the petition for correction of entries. petitioner no longer contested
the RTC ruling that the correction he sought on his and his mother’s first name can be done by the
city civil registrar. Under the circumstances, we are constrained to deny his prayer that the petition for
correction of entries before the RTC bereinstated since the same petition includes the correction he sought
on his and his mother’s first name.RTC’s dismissal is without prejudice. As we said, petitioner can avail
ofthe administrative remedy for the correction of his and his mother’s first name.1âwphi1 He can also file
a new petition before the RTC to correct the alleged erroneous entry on his birth certificate that his
parents were married on December 23, 1983 in Bicol. This substantial correction is allowed under Rule
108 of the Rules of Court.

A petition seeking a substantial correction of an entry in a civil register must implead as parties to the
proceedings not only the local civil registrar, as petitioner did in the dismissed petition for correction of
entries, but also all persons who have or claim any interest which would be affected by the correction.
This is required by Section 3, Rule 108 of the Rules of Court:

SEC. 3.Parties. - When cancellation or correction of an entry in the civil register is sought, the civil
registrar and all persons who have or claim any interest which would be affected thereby shall be made
parties to the proceeding. (Emphasis supplied.)

DENY the petition and AFFIRM the Orders. The dismissal ordered by the Regional Trial Court is,
however, declared to be without prejudice.

Remedial Law Review II (CY 2018-2019) – Saturday 1:00pm – 5:00pm: Case Digest Compilation [Atty. Brondial] Page 274
H.11. Almojuela vs. Republic

Remedial Law Review II (CY 2018-2019) – Saturday 1:00pm – 5:00pm: Case Digest Compilation [Atty. Brondial] Page 275
H.12. EMELITA BASILIO GAN, Petitioner, v.REPUBLIC OF THE PHILIPPINES, Respondent.

G.R. No. 207147, September 14, 2016; REYES, J.:

This is a petition for review on certiorari under Rule 45 of the Rules of Court seeking to annul and set
aside the Decision dated April 26, 2013 issued by the Court of Appeals (CA) in CA-G.R. CV No. 98112.

FACTS: Emelita Basilio Gan (petitioner) was born out of wedlock to a Chinese father, Pia Gan, and a
Filipino mother, Consolacion Basilio, The petitioner's birth certificate, which was registered in the Office
of the Local Civil Registrar (LCR) of Libmanan, Camarines Sur, indicates that her full name is Emelita
Basilio.

The petitioner filed a Petition for correction of name from "Emelita Basilio" to "Emelita Basilio Gan"
with the Regional Trial Court (RTC) of Libmanan, Camarines Sur, claiming that she had been using the
name "Emelita Basilio Gan" in her school records from elementary until college, employment records,
marriage contract, and other government records.

The RTC granted the petition for change of name.


The RTC denied the sought a reconsideration of the respondent.

On appeal, the CA reversed and set aside the RTC Orders dated July 19, 2011 and October 17, 2011.

ISSUE: Whether the RTC erred in granting the petition for the change of name.

RULING: Yes. The petition is denied. A change of name is a privilege and not a matter of right; a proper
and reasonable cause must exist before a person may be authorized to change his name. 21 "In granting or
denying petitions for change of name, the question of proper and reasonable cause is left to the sound
discretion of the court. x x x What is involved is not a mere matter of allowance or disallowance of the
request, but a judicious evaluation of the sufficiency and propriety of the justifications advanced in
support thereof, mindful of the consequent results in the event of its grant and with the sole prerogative
for making such determination being lodged in the courts."

The petitioner's reliance on the cases of Alfon v. Republic of the Philippines,Republic of the Philippines v.
Coseteng-Magpayo, and Republic of the Philippines v. Lim to support her position is misplaced.

In Alfon, the name of the petitioner therein which appeared in her birth certificate was Maria Estrella
Veronica Primitiva Duterte; she was a legitimate child of her father and mother. She filed a petition for
change of name, seeking that she be allowed to use the surname "Alfon," her mother's surname, instead of
"Duterte." The trial court denied the petition, ratiocinating that under Article 364 of the Civil Code,
legitimate children shall principally use the surname of the father. The Court allowed the petitioner
therein to use the surname of her mother since Article 364 of the Civil Code used the word "principally"
and not "exclusively" and, hence, there is no legal obstacle if a legitimate child should choose to use the
mother's surname to which he or she is legally entitled. 27chanrobleslaw

In contrast, Articles 366 and 368 of the Civil Code do not give to an illegitimate child or a natural child
not acknowledged by the father the option to use the surname of the father. Thus, the petitioner cannot
insist that she is allowed to use the surname of her father.

Here, the petitioner filed a petition for change of name under Rule 103 and not a petition for correction of
entries under Rule 108. Unlike in Lim, herein petitioner's birth certificate indicated that she bears the
surname of her mother and not of her father.

WHEREFORE, in consideration of the foregoing disquisitions, the petition is DENIED. SO


ORDERED.

Remedial Law Review II (CY 2018-2019) – Saturday 1:00pm – 5:00pm: Case Digest Compilation [Atty. Brondial] Page 276
I. Prerogative Writs

I.1. DANIEL MASANGKAY TAPUZ, AURORA TAPUZ-MADRIAGA, LIBERTY M.


ASUNCION, LADYLYN BAMOS MADRIAGA, EVERLY TAPUZ MADRIAGA, EXCEL
TAPUZ, IVAN TAPUZ AND MARIAN TIMBAS Petitioners, vs. HONORABLE JUDGE ELMO
DEL ROSARIO, in his capacity as Presiding Judge of RTC Br. 5 Kalibo, SHERIFF NELSON
DELA CRUZ, in his capacity as Sheriff of the RTC, THE PHILIPPINE NATIONAL POLICE
stationed in Boracay Island, represented by the PNP STATION COMMANDER, THE
HONORABLE COURT OF APPEALS IN CEBU 18th DIVISION, SPOUSES GREGORIO
SANSON & MA. LOURDES T. SANSON, Respondents.

G.R. No. 182484, June 17, 2008; BRION, J.:

FACTS: The private respondents spouses Gregorio Sanson and Ma. Lourdes T. Sanson (the private
respondents), filed with the Fifth Municipal Circuit Trial Court of Buruanga-Malay, Aklan (the MCTC) a
complaint dated 24 April 2006 for forcible entry and damages with a prayer for the issuance of a writ of
preliminary mandatory injunction against the petitioners Daniel Masangkay Tapuz, Aurora Tapuz-
Madriaga, Liberty M. Asuncion, Ladylyn Bamos Madriaga, Everly Tapuz Madriaga, Excel Tapuz, Ivan
Tapuz and Marian Timbas (the petitioners) and other John Does numbering about 120. The private
respondents alleged in their complaint that: (1) they are the registered owners under TCT No. 35813 of a
1.0093-hectare parcel of land located at Sitio Pinaungon, Balabag, Boracay, Malay, Aklan (the disputed
land); (2) they were the disputed lands prior possessors when the petitioners armed with bolos and
carrying suspected firearms and together with unidentified persons numbering 120 - entered the disputed
land by force and intimidation, without the private respondents permission and against the objections of
the private respondents security men, and built thereon a nipa and bamboo structure.

In their Answer dated 14 May 2006, the petitioners denied the material allegations of the complaint. They
essentially claimed that: (1) they are the actual and prior possessors of the disputed land; (2) on the
contrary, the private respondents are the intruders; and (3) the private respondents certificate of title to the
disputed property is spurious. They asked for the dismissal of the complaint and interposed a
counterclaim for damages.

ISSUE: Whether a petition for a writ of amparo is appropriate when the issue involved concerns that of
purely property or commercial.

RULING: No.

The Writ of Amparo


To start off with the basics, the writ of amparo was originally conceived as a response to the extraordinary
rise in the number of killings and enforced disappearances, and to the perceived lack of available and
effective remedies to address these extraordinary concerns. It is intended to address violations of or
threats to the rights to life, liberty or security, as an extraordinary and independent remedy beyond those
available under the prevailing Rules, or as a remedy supplemental to these Rules. What it is not, is a writ
to protect concerns that are purely property or commercial. Neither is it a writ that we shall issue
on amorphous and uncertain grounds. Consequently, the Rule on the Writ of Amparo in line with the
extraordinary character of the writ and the reasonable certainty that its issuance demands requires that
every petition for the issuance of the writ must be supported by justifying allegations of fact, to wit:
(a) The personal circumstances of the petitioner;
(b) The name and personal circumstances of the respondent responsible for the threat, act or omission, or,
if the name is unknown or uncertain, the respondent may be described by an assumed appellation;
(c) The right to life, liberty and security of the aggrieved party violated or threatened with violation by an
unlawful act or omission of the respondent, and how such threat or violation is committed with the
attendant circumstances detailed in supporting affidavits;
(d) The investigation conducted, if any, specifying the names, personal circumstances, and addresses of
the investigating authority or individuals, as well as the manner and conduct of the investigation, together
with any report;
(e) The actions and recourses taken by the petitioner to determine the fate or whereabouts of the
aggrieved party and the identity of the person responsible for the threat, act or omission; and
(f) The relief prayed for.
The petition may include a general prayer for other just and equitable reliefs.
The writ shall issue if the Court is preliminarily satisfied with the prima facie existence of the ultimate
facts determinable from the supporting affidavits that detail the circumstances of how and to what extent

Remedial Law Review II (CY 2018-2019) – Saturday 1:00pm – 5:00pm: Case Digest Compilation [Atty. Brondial] Page 277
a threat to or violation of the rights to life, liberty and security of the aggrieved party was or is being
committed.

On the whole, what is clear from these statements - both sworn and unsworn - is the overriding
involvement of property issues as the petition traces its roots to questions of physical possession of the
property disputed by the private parties. If at all, issues relating to the right to life or to liberty can hardly
be discerned except to the extent that the occurrence of past violence has been alleged. The right to
security, on the other hand, is alleged only to the extent of the threats and harassments implied from the
presence of armed men bare to the waist and the alleged pointing and firing of weapons. Notably, none of
the supporting affidavits compellingly show that the threat to the rights to life, liberty and security of the
petitioners is imminent or is continuing.

Under these legal and factual situations, we are far from satisfied with the prima facie existence of the
ultimate facts that would justify the issuance of a writ of amparo. Rather than acts of terrorism that pose a
continuing threat to the persons of the petitioners, the violent incidents alleged appear to us to be purely
property-related and focused on the disputed land. Thus, if the petitioners wish to seek redress and hold
the alleged perpetrators criminally accountable, the remedy may lie more in the realm of ordinary
criminal prosecution rather than on the use of the extraordinary remedy of the writ of amparo.
The writ of amparo, particularly, should not issue when applied for as a substitute for the appeal or
certiorari process, or when it will inordinately interfere with these processes the situation obtaining in the
present case.

While we say all these, we note too that the Rule on the Writ of Amparo provides for rules on the
institution of separate actions, for the effect of earlier-filed criminal actions, and for the consolidation of
petitions for the issuance of a writ of amparo with a subsequently filed criminal and civil action. These
rules were adopted to promote an orderly procedure for dealing with petitions for the issuance of the writ
of amparo when the parties resort to other parallel recourses.

Where, as in this case, there is an ongoing civil process dealing directly with the possessory dispute and
the reported acts of violence and harassment, we see no point in separately and directly intervening
through a writ of amparo in the absence of any clear prima facie showing that the right to life, liberty or
security the personal concern that the writ is intended to protect - is immediately in danger or threatened,
or that the danger or threat is continuing. We see no legal bar, however, to an application for the issuance
of the writ, in a proper case, by motion in a pending case on appeal or on certiorari, applying by analogy
the provisions on the co-existence of the writ with a separately filed criminal case.

WHEREFORE, premises considered, we hereby DISMISS the present petition OUTRIGHT for
deficiencies of form and substance patent from its body and attachments.

Remedial Law Review II (CY 2018-2019) – Saturday 1:00pm – 5:00pm: Case Digest Compilation [Atty. Brondial] Page 278
I.2. Infant JULIAN YUSAY CARAM, represented by his mother, MA. CHRISTINA YUSAY
CARAM, petitioner, vs. Atty. MARIJOY D. SEGUI, Atty. SALLY D. ESCUTIN, VILMA B.
CABRERA, and CELIA YANGCO, respondents.

G.R. No. 193652. August 5, 2014.

FACTS: Constitutional Law; Writs of Amparo; The Amparo Rule was intended to address the intractable
problem of “extralegal killings” and “enforced disappearances,” its coverage, in its present form, is
confined to these two instances or to threats thereof.—Section 1 of the Rule on the Writ of Amparo
provides as follows: SECTION 1. Petition. — The petition for a Writ of Amparo is a remedy available to
any person whose right to life, liberty and security is violated or threatened with violation by an unlawful
act or omission of a public official or employee, or of a private individual or entity. The writ shall cover
extralegal killings and enforced disappearances or threats thereof. In the landmark case of Secretary of
National Defense, et al. v. Manalo, et al., 568 SCRA 1 (2008), this Court RULING: [T]he Amparo Rule
was intended to address the intractable problem of “extralegal killings” and “enforced disappearances,”
its coverage, in its present form, is confined to these two instances or to threats thereof. “Extralegal
killings” are “killings committed without due process of law, i.e., without legal safeguards or judicial
proceedings.” On the other hand, “enforced disappearances” are “attended by the following
characteristics: an arrest, detention or abduction of a person by a government official or organized
groups or private individuals acting with the direct or indirect acquiescence of the government; the
refusal of the State to disclose the fate or whereabouts of the person concerned or a refusal to
acknowledge the deprivation of liberty which places such persons outside the protection of law.

Petitioner Ma. Christina Yusay Caram had an amorous relationship with Marcelino Gicano Constantino
III and eventually became pregnant with the latter’s child without the benefit of marriage. After getting
pregnant, Christina mislead Marcelino into believing that she had an abortion when in fact she proceeded
to complete the term of her pregnancy. During this time, she intended to have the child adopted through
Sun and Moon Home for Children to avoid placing her family in a potentially embarrassing situation for
having a second illegitimate son.

On July 26, 2009, Christina gave birth to Baby Julian. Sun and Moon shouldered all the hospital and
medical expenses. Christina voluntarily surrendered Baby Julian by way of a Deed of Voluntary
Commitment to the DSWD.

On November 26, 2009, Marcelino suffered a heart attack and died without knowing about the birth of his
son. Thereafter, during the wake, Christina disclosed to Marcelino’s family that she and the deceased had
a son that she gave up for adoption due to financial distress and initial embarrassment. Marcelino’s family
was taken aback by the revelation and sympathized with Christina. After the emotional revelation, they
vowed to help her recover and raise the baby. DSWD issued a certificate declaring Baby Julian as
"Legally Available for Adoption." Baby Julian was "matched" with the spouses Vergel and Filomina
Medina.

Christina who had changed her mind about the adoption, wrote a letter to the DSWD asking for the
suspension of Baby Julian’s adoption proceedings. DSWD, through respondent Atty. Marijoy D. Segui,
sent a Memorandum to DSWD Assistant Secretary Vilma B. Cabrera informing her that the certificate
declaring Baby Julian legally available for adoption had attained finality on November 13, 2009, or three
months after Christina signed the Deed of Voluntary Commitment which terminated her parental
authority and effectively made Baby Julian a ward of the State. Assistant Secretary Cabrera further stated
that should Christina wish to reacquire her parental authority over Baby Julian or halt the adoption
process, she may bring the matter to the regular courts as the reglementary period for her to regain her
parental rights had already lapsed under Section 7 of Republic Act (R.A.) No. 9523.
Christina filed a petition for the issuance of a Writ of Amparo before the RTC of Quezon City seeking to
obtain custody of Baby Julian from Atty. Segui, Atty. Escutin, Assistant Secretary Cabrera and Acting
Secretary Celia C. Yangco, all of the DSWD.

RTC issued a Writ of Amparo. 2010, Respondents appeared but did not bring the child, stating that
threats of kidnapping were made on the child and his caregivers. To give respondents another chance, the
RTC reset the hearing. The Office of the Solicitor General (OSG) entered its appearance as representative
of the State and in turn, the RTC acknowledged the appearance of the OSG and allowed its
representatives to actively participate in the arguments raised during the said hearing.

Remedial Law Review II (CY 2018-2019) – Saturday 1:00pm – 5:00pm: Case Digest Compilation [Atty. Brondial] Page 279
RTC dismissed the petition for issuance of a Writ of Amparo without prejudice to the filing of the
appropriate action in court. The RTC held that Christina availed of the wrong remedy to regain custody of
her child Baby Julian. The RTC further stated that Christina should have filed a civil case for custody of
her child as laid down in the Family Code and the Rule on Custody of Minors and Writ of Habeas Corpus
in Relation to Custody of Minors. If there is extreme urgency to secure custody of a minor who has been
illegally detained by another, a petition for the issuance of a Writ of Habeas corpus may be availed of,
either as a principal or ancillary remedy, pursuant to the Rule on Custody of Minors and Writ of Habeas
Corpus in Relation to Custody of Minors.

Christina directly elevated the case before this Court, via a petition for review on certiorari under Rule 45
of the 1997 Rules of Civil Procedure, as amended, in relation to Section 19 of the Rule on the Writ of
Amparo.

ISSUE: Whether a petition for a Writ of Amparo is the proper recourse for obtaining parental authority
and custody of a minor child.

RULING: The Court rejects petitioner’s contentions and denies the petition.

In this case, Christina alleged that the respondent DSWD officers caused her "enforced separation" from
Baby Julian and that their action amounted to an "enforced disappearance" within the context of the
Amparo rule. Contrary to her position, however, the respondent DSWD officers never concealed Baby
Julian's whereabouts. In fact, Christina obtained a copy of the DSWD's May 28, 2010 Memorandum
explicitly stating that Baby Julian was in the custody of the Medina Spouses when she filed her petition
before the RTC. Besides, she even admitted in her petition for review on certiorari that the respondent
DSWD officers presented Baby Julian before the RTC during the hearing held in the afternoon of August
5, 2010. There is therefore, no "enforced disappearance" as used in the context of the Amparo rule as the
third and fourth elements are missing.

Christina's directly accusing the respondents of forcibly separating her from her child and placing the
latter up for adoption, supposedly without complying with the necessary legal requisites to qualify the
child for adoption, clearly indicates that she is not searching for a lost child but asserting her parental
authority over the child and contesting custody over him.37 Since it is extant from the pleadings filed that
what is involved is the issue of child custody and the exercise of parental rights over a child, who, for all
intents and purposes, has been legally considered a ward of the State, the Amparo rule cannot be properly
applied.

To reiterate, the privilege of the Writ of Amparo is a remedy available to victims of extra-judicial killings
and enforced disappearances or threats of a similar nature, regardless of whether the perpetrator of the
unlawful act or omission is a public official or employee or a private individual. It is envisioned basically
to protect and guarantee the right to life, liberty and security of persons, free from fears and threats that
vitiate the quality of life.

WHEREFORE, the petition is DENIED. The August 17, 2010 and September 6, 2010 Orders of the
Regional Trial Court, Branch 106, Quezon City in Sp. Proc. Case No. Q-10-67604 are AFFIRMED
without prejudice to petitioner's right to avail of proper legal remedies afforded to her by law and related
rules.

Remedial Law Review II (CY 2018-2019) – Saturday 1:00pm – 5:00pm: Case Digest Compilation [Atty. Brondial] Page 280
I.3.Vivares et Al vs. St. Therese College et Al

G.R. No. 202666; September 29, 2014; Velasco, Jr., J.

FACTS:Back at the school, Mylene Rheza T. Escudero (Escudero), a computer teacher at STC’s high
school department, learned from her students that some seniors at STC posted pictures online, depicting
themselves from the waist up, dressed only in brassieres. Using STC’s computers, Escudero’s students
logged in to their respective personal Facebook accounts and showed her photos of the identified
students, which include: (a) Nenita Julia V. Daluz (Julia) and Julienne Vida Suzara (Julienne) drinking
hard liquor and smoking cigarettes inside a bar; and (b) Julia and Julienne along the streets of Cebu
wearing articles of clothing that show virtually the entirety of their black brassieres. What is more,
Escudero’s students claimed that there were times when access to or the availability of the identified
students’ photos was not confined to the girls’ Facebook friends, but were, in fact, viewable by any
Facebook user.

Upon discovery, Escudero reported the matter and, through one of her student’s Facebook page, showed
the photos to Kristine Rose Tigol (Tigol), STC’s Discipline-in-Charge, for appropriate action.

On March 1, 2012, Julia, Julienne and the other students in the pictures in question, reported, as required,
to the office of Sr. Celeste Ma. Purisima Pe (Sr. Purisima), STC’s high school principal and ICM
Directress. Sr. Purisima informed their parents the following day that, as part of their penalty, they are
barred from joining the commencement exercises scheduled on March 30, 2012.

On March 23, 2012, Angela’s mother, Dr. Armenia M. Tan (Tan), filed a Petition for Injunction and
Damages before the RTC of Cebu City against STC. In it, Tan prayed that defendants therein be enjoined
from implementing the sanction that precluded Angela from joining the commencement exercises.

On March 25, 2012,petitioner Rhonda Ave Vivares (Vivares), the mother of Julia, joined the fray as an
intervenor. 

The RTC issued a temporary restraining order (TRO) allowing the students to attend the graduation
ceremony, to which STC filed a motion for reconsideration.

Despite the issuance of the TRO,STC, nevertheless, barred the sanctioned students from participating in
the graduation rites, arguing that, on the date of the commencement exercises, its adverted motion for
reconsideration on the issuance ofthe TRO remained unresolved.

Thereafter, petitioners filed before the RTC a Petition for the Issuance of a Writ of Habeas Data.

The RTC rendered a Decision dismissing the petition for habeas data. To the trial court, petitioners failed
to prove the existence of an actual or threatened violation of the minors’ right to privacy, one of the
preconditions for the issuance of the writ of habeas data. Moreover, the court a quo held that the photos,
having been uploaded on Facebook without restrictions as to who may view them, lost their privacy in
some way. Besides, the RTC noted, STC gathered the photographs through legal means and for a legal
purpose, that is, the implementation of the school’s policies and rules on discipline.

Not satisfied with the outcome, petitioners now come before this Court pursuant to Section 19 of the Rule
on Habeas Data.

ISSUE: Whether or not there was indeed an actual or threatened violation of the right to privacy in the
life, liberty, or security of the minors involved which warrants the issuance of the writ of habeas data.

RULING: No violation of such rights was found.

Without an actionable entitlement in the first place to the right to informational privacy, a habeas data
petition will not prosper. Viewed from the perspective of the case at bar, this requisite begs this question:
given the nature of an online social network (OSN)––(1) that it facilitates and promotes real-time
interaction among millions, if not billions, of users, sans the spatial barriers, bridging the gap created by
physical space; and (2) that any information uploaded in OSNs leavesan indelible trace in the provider’s
databases, which are outside the control of the end-users––is there a right to informational privacy in
OSN activities of its users? Before addressing this point, We must first resolve the procedural issues in
this case.

Remedial Law Review II (CY 2018-2019) – Saturday 1:00pm – 5:00pm: Case Digest Compilation [Atty. Brondial] Page 281
Procedural Aspect
a. The writ of habeas data is not only confined to cases of extralegal killings and enforced
disappearances
Contrary to respondents’ submission, the Writ of Habeas Datawas not enacted solely for the purpose of
complementing the Writ of Amparo in cases of extralegal killings and enforced disappearances.

Habeas data, to stress, was designed "to safeguard individual freedom from abuse in the information age."
As such, it is erroneous to limit its applicability to extralegal killings and enforced disappearances only. 

b. Meaning of "engaged" in the gathering, collecting or storing of data or information


Respondents’ contention that the habeas data writ may not issue against STC, it not being an entity
engaged in the gathering, collecting or storing of data or information regarding the person, family, home
and correspondence of the aggrieved party, while valid to a point, is, nonetheless, erroneous.

The provision, when taken in its proper context, as a whole, irresistibly conveys the idea that habeas data
is a protection against unlawful acts or omissions of public officials and of private individuals or entities
engaged in gathering, collecting, or storing data about the aggrieved party and his or her correspondences,
or about his or her family. Such individual or entity need not be in the business of collecting or storing
data.

To "engage" in something is different from undertaking a business endeavour. To "engage" means "to do
or take part in something." It does not necessarily mean that the activity must be done in pursuit of a
business. What matters is that the person or entity must be gathering, collecting or storing said data or
information about the aggrieved party or his or her family. Whether such undertaking carries the element
of regularity, as when one pursues a business, and is in the nature of a personal endeavour, for any other
reason or even for no reason at all, is immaterial and such will not prevent the writ from getting to said
person or entity.

Substantive Aspect

a. The Right to Informational Privacy


The concept of privacy has, through time, greatly evolved, with technological advancements having an
influential part therein. This evolution was briefly recounted in former Chief Justice Reynato S. Puno’s
speech, The Common Right to Privacy, where he explained the three strands of the right to privacy, viz:
(1) locational or situational privacy; (2) informational privacy; and  decisional privacy. Of the three, what
is relevant to the case at bar is the right to informational privacy––usually defined as the right of
individuals to control information about themselves.

With the availability of numerous avenues for information gathering and data sharing nowadays, not to
mention each system’s inherent vulnerability to attacks and intrusions, there is more reason that every
individual’s right to control said flow of information should be protected and that each individual should
have at least a reasonable expectation of privacy in cyberspace. Several commentators regarding privacy
and social networking sites, however, all agree that given the millions of OSN users, "[i]n this [Social
Networking] environment, privacy is no longer grounded in reasonable expectations, but rather in some
theoretical protocol better known as wishful thinking."

b. Facebook’s Privacy Tools: a response to the clamor for privacy in OSN activities
Briefly, the purpose of an OSN is precisely to give users the ability to interact and to stay connected to
other members of the same or different social media platform through the sharing of statuses, photos,
videos, among others, depending on the services provided by the site. It is akin to having a room filled
with millions of personal bulletin boards or "walls," the contents of which are under the control of each
and every user. In his or her bulletin board, a user/owner can post anything––from text, to pictures, to
music and videos––access to which would depend on whether he or she allows one, some or all of the
other users to see his or her posts. Since gaining popularity, the OSN phenomenon has paved the way to
the creation of various social networking sites, including the one involved in the case at
bar, www.facebook.com (Facebook), which, according to its developers, people use "to stay connected
with friends and family, to discover what’s going on in the world, and to share and express what matters
to them."

Facebook connections are established through the process of "friending" another user. By sending a
"friend request," the user invites another to connect their accounts so that they can view any and all

Remedial Law Review II (CY 2018-2019) – Saturday 1:00pm – 5:00pm: Case Digest Compilation [Atty. Brondial] Page 282
"Public" and "Friends Only" posts of the other. Once the request is accepted, the link is established and
both users are permitted to view the other user’s "Public" or "Friends Only" posts, among others.
"Friending," therefore, allows the user to form or maintain one-to-one relationships with other users,
whereby the user gives his or her "Facebook friend" access to his or her profile and shares certain
information to the latter.

In other words, Facebook extends its users an avenue to make the availability of their Facebook activities
reflect their choice as to "when and to what extent to disclose facts about [themselves] – and to put others
in the position of receiving such confidences." Ideally, the selected setting will be based on one’s desire to
interact with others, coupled with the opposing need to withhold certain information as well as to regulate
the spreading of his or her personal information. Needless to say, as the privacy setting becomes more
limiting, fewer Facebook users can view that user’s particular post.

STC did not violate petitioners’ daughters’ right to privacy


Without these privacy settings, respondents’ contention that there is no reasonable expectation of privacy
in Facebook would, in context, be correct. However, such is not the case. It is through the availability of
said privacy tools that many OSN users are said to have a subjective expectation that only those to whom
they grant access to their profile will view the information they post or upload thereto.

This, however, does not mean that any Facebook user automatically has a protected expectation of
privacy in all of his or her Facebook activities.

Before one can have an expectation of privacy in his or her OSN activity, it is first necessary that said
user, in this case the children of petitioners,manifest the intention to keep certain posts private, through
the employment of measures to prevent access thereto or to limit its visibility. And this intention can
materialize in cyberspace through the utilization of the OSN’s privacy tools. In other words, utilization of
these privacy tools is the manifestation,in cyber world, of the user’s invocation of his or her right to
informational privacy.

Therefore, a Facebook user who opts to make use of a privacy tool to grant or deny access to his or her
post or profile detail should not be denied the informational privacy right which necessarily accompanies
said choice. Otherwise, using these privacy tools would be a feckless exercise, such that if, for instance, a
user uploads a photo or any personal information to his or her Facebook page and sets its privacy level at
"Only Me" or a custom list so that only the user or a chosen few can view it, said photo would still be
deemed public by the courts as if the user never chose to limit the photo’s visibility and accessibility.
Such position, if adopted, will not only strip these privacy tools of their function but it would also
disregard the very intention of the user to keep said photo or information within the confines of his or her
private space.

We must now determine the extent that the images in question were visible to other Facebook users and
whether the disclosure was confidential in nature. In other words, did the minors limit the disclosure of
the photos such that the images were kept within their zones of privacy? This determination is necessary
in resolving the issue of whether the minors carved out a zone of privacy when the photos were uploaded
to Facebook so that the images will be protected against unauthorized access and disclosure.

Petitioners, in support of their thesis about their children’s privacy right being violated, insist that
Escudero intruded upon their children’s Facebook accounts, downloaded copies ofthe pictures and
showed said photos to Tigol. To them, this was a breach of the minors’ privacy since their Facebook
accounts, allegedly, were under "very private" or "Only Friends" setting safeguarded with a password.
Ultimately, they posit that their children’s disclosure was only limited since their profiles were not open
to public viewing. Therefore, according to them, people who are not their Facebook friends, including
respondents, are barred from accessing said post without their knowledge and consent. As petitioner’s
children testified, it was Angela who uploaded the subject photos which were only viewable by the five of
them, although who these five are do not appear on the records.

Escudero, on the other hand, stated in her affidavit that "my students showed me some pictures of girls
cladin brassieres. This student [sic] of mine informed me that these are senior high school [students] of
STC, who are their friends in [F]acebook. x x x They then said [that] there are still many other photos
posted on the Facebook accounts of these girls. At the computer lab, these students then logged into their
Facebook account [sic], and accessed from there the various photographs x x x. They even told me that
there had been times when these photos were ‘public’ i.e., not confined to their friends in Facebook."

Remedial Law Review II (CY 2018-2019) – Saturday 1:00pm – 5:00pm: Case Digest Compilation [Atty. Brondial] Page 283
In this regard, We cannot give muchweight to the minors’ testimonies for one key reason: failure to
question the students’ act of showing the photos to Tigol disproves their allegation that the photos were
viewable only by the five of them. Without any evidence to corroborate their statement that the images
were visible only to the five of them, and without their challenging Escudero’s claim that the other
students were able to view the photos, their statements are, at best, self-serving, thus deserving scant
consideration.

It is well to note that not one of petitioners disputed Escudero’s sworn account that her students, who are
the minors’ Facebook "friends," showed her the photos using their own Facebook accounts. This only
goes to show that no special means to be able to view the allegedly private posts were ever resorted to by
Escudero’s students, and that it is reasonable to assume, therefore, that the photos were, in reality,
viewable either by (1) their Facebook friends, or (2) by the public at large.

Considering that the default setting for Facebook posts is "Public," it can be surmised that the
photographs in question were viewable to everyone on Facebook, absent any proof that petitioners’
children positively limited the disclosure of the photograph. If such were the case, they cannot invoke the
protection attached to the right to informational privacy.

That the photos are viewable by "friends only" does not necessarily bolster the petitioners’ contention. In
this regard, the cyber community is agreed that the digital images under this setting still remain to be
outside the confines of the zones of privacy.

It is well to emphasize at this point that setting a post’s or profile detail’s privacy to "Friends" is no
assurance that it can no longer be viewed by another user who is not Facebook friends with the source of
the content. The user’s own Facebook friend can share said content or tag his or her own Facebook friend
thereto, regardless of whether the user tagged by the latter is Facebook friends or not with the former.
Also, when the post is shared or when a person is tagged, the respective Facebook friends of the person
who shared the post or who was tagged can view the post, the privacy setting of which was set at
"Friends."

As applied, even assuming that the photos in issue are visible only to the sanctioned students’ Facebook
friends, respondent STC can hardly be taken to task for the perceived privacy invasion since it was the
minors’ Facebook friends who showed the pictures to Tigol. Respondents were mere recipients of what
were posted. They did not resort to any unlawful means of gathering the information as it was voluntarily
given to them by persons who had legitimate access to the said posts. Clearly, the fault, if any, lies with
the friends of the minors. Curiously enough, however, neither the minors nor their parents imputed any
violation of privacy against the students who showed the images to Escudero.

Furthermore, petitioners failed to prove their contention that respondents reproduced and broadcasted the
photographs. In fact, what petitioners attributed to respondents as an act of offensive disclosure was no
more than the actuality that respondents appended said photographs in their memorandum submitted to
the trial court in connection with Civil Case No. CEB-38594.52 These are not tantamount to a violation of
the minor’s informational privacy rights, contrary to petitioners’ assertion.

In sum, there can be no quibbling that the images in question, or to be more precise, the photos of minor
students scantily clad, are personal in nature, likely to affect, if indiscriminately circulated, the reputation
of the minors enrolled in a conservative institution. However, the records are bereft of any evidence, other
than bare assertions that they utilized Facebook’s privacy settings to make the photos visible only to them
or to a select few. Without proof that they placed the photographs subject of this case within the ambit of
their protected zone of privacy, they cannot now insist that they have an expectation of privacy with
respect to the photographs in question.

Had it been proved that the access tothe pictures posted were limited to the original uploader, through the
"Me Only" privacy setting, or that the user’s contact list has been screened to limit access to a select few,
through the "Custom" setting, the result may have been different, for in such instances, the intention to
limit access to the particular post, instead of being broadcasted to the public at large or all the user’s
friends en masse, becomes more manifest and palpable.

DOCTRINE: It is, thus, incumbent upon internet users to exercise due diligence in their online dealings
and activities and must not be negligent in protecting their rights. Equity serves the vigilant. Demanding
relief from the courts, as here, requires that claimants themselves take utmost care in safeguarding a
right which they allege to have been violated. These are indispensable. We cannot afford protection to

Remedial Law Review II (CY 2018-2019) – Saturday 1:00pm – 5:00pm: Case Digest Compilation [Atty. Brondial] Page 284
persons if they themselves did nothing to place the matter within the confines of their private zone. OSN
users must be mindful enough to learn the use of privacy tools, to use them if they desire to keep the
information private, and to keep track of changes in the available privacy settings, such as those of
Facebook, especially because Facebook is notorious for changing these settings and the site's layout
often.

Remedial Law Review II (CY 2018-2019) – Saturday 1:00pm – 5:00pm: Case Digest Compilation [Atty. Brondial] Page 285
I.4.GEN. AVELINO I. RAZON, JR., Chief, Philippine National Police (PNP); Police Chief
Superintendent RAUL CASTAEDA, Chief, Criminal Investigation and Detection Group (CIDG);
Police Senior Superintendent LEONARDO A. ESPINA, Chief, Police Anti-Crime and Emergency
Response (PACER); and GEN. JOEL R. GOLTIAO, Regional Director of ARMM, PNP, Petitioner
vsMARY JEAN B. TAGITIS, herein represented by ATTY. FELIPE P. ARCILLA, JR., Attorney-
in-Fact, Respondent.

G.R. No. 182498; December 3, 2009; BRION, J.:

DIGEST BY: AUSTRIA, DON RODEL A.

FACTS: Engineer Morced N. Tagitis (Tagitis), a consultant for the World Bank and the Senior Honorary
Counselor for the Islamic Development Bank (IDB) Scholarship Programme, together with Arsimin
Kunnong (Kunnong), an IDB scholar, arrived in Jolo by boat in the early morning of October 31, 2007
from a seminar in Zamboanga City. They immediately checked-in at ASY Pension House. Tagitis asked
Kunnong to buy him a boat ticket for his return trip the following day to Zamboanga. When Kunnong
returned from this errand, Tagitis was no longer around. Kunnong looked for Tagitis and even sent a text
message to the latter’s Manila-based secretary, who advised Kunnong to simply wait for Tagitis’ return.

On November 4, 2007, Kunnong and Muhammad Abdulnazeir N. Matli, a UP professor of Muslim


studies and Tagitis’ fellow student counselor at the IDB, reported Tagitis’ disappearance to the Jolo
Police Station. More than a month later, or on December 28, 2007, the respondent, May Jean Tagitis,
through her attorney-in-fact, filed a Petition for the Writ of Amparo (petition) directed against Lt. Gen.
Alexander Yano, Commanding General, Philippine Army; Gen. Avelino I. Razon, Chief, Philippine
National Police (PNP); Gen. Edgardo M. Doromal, Chief, Criminal Investigation and Detention Group
(CIDG); Sr. Supt. Leonardo A. Espina, Chief, Police Anti-Crime and Emergency Response; Gen. Joel
Goltiao, Regional Director, ARMM-PNP; and Gen. Ruben Rafael, Chief, Anti-Terror Task Force Comet
(collectively referred to as “petitioners”), with the Court of Appeals (CA). On the same day, the CA
immediately issued the Writ of Amparo and set the case for hearing on January 7, 2008.

On March 7, 2008, the CA issued its decision confirming that the disappearance of Tagitis was an
“enforced disappearance” under the United Nations (UN) Declaration on the Protection of All Persons
from Enforced Disappearances. The CA ruled that when military intelligence pinpointed the investigative
arm of the PNP (CIDG) to be involved in the abduction, the missing-person case qualified as an enforced
disappearance. Hence, the CA extended the privilege of the writ to Tagitis and his family, and directed
the petitioners to exert extraordinary diligence and efforts to protect the life, liberty and security of
Tagitis, with the obligation to provide monthly reports of their actions to the CA. At the same time, the
CA dismissed the petition against the then respondents from the military, Lt. Gen Alexander Yano and
Gen. Ruben Rafael, based on the finding that it was PNP-CIDG, not the military, that was involved.

On March 31, 2008, the petitioners moved to reconsider the CA decision, but the CA denied the motion in
its Resolution dated April 9, 2008. Aggrieved, the petitioners filed a petition for review with the Supreme
Court.

ISSUE:
1.) Whether or not the requirement that the pleader must state the ultimate facts is indispensable
in an amparo petition?

2.) Whether or not the presentation of substantial evidence by the petitioner to prove her
allegations is sufficient for the court to grant the privilege of the writ?

3.) Whether or not the writ of amparo determines guilt nor pinpoint criminal culpability for the
alleged enforced disappearance of the subject of the petition for the writ?

RESOLUTION:

1.) NO.
However, it must contain details available to the petitioner under the circumstances, while
presenting a cause of action showing a violation of the victim’s rights to life, liberty and security
through State or private party action.

Remedial Law Review II (CY 2018-2019) – Saturday 1:00pm – 5:00pm: Case Digest Compilation [Atty. Brondial] Page 286
The requirement that the pleader must state the ultimate facts must be read in light of the nature
and purpose of the proceeding, which addresses a situation of uncertainty. The framers of the
Amparo Rule never intended Section 5 (c) to be complete in every detail in stating the threatened
or actual violation of a victim’s rights. As in any other initiatory pleading, the pleader must of
course state the ultimate facts constituting the cause of action, omitting the evidentiary details.

In an Amparo petition, however, this requirement must be read in light of the nature and purpose
of the proceeding, which addresses a situation of uncertainty. The petitioner may not be able to
describe with certainty how the victim exactly disappeared, or who actually acted to kidnap,
abduct or arrest him or her, or where the victim is detained, because these information may
purposely be hidden or covered up by those who caused the disappearance. In this type of
situation, to require the level of specificity, detail and precision that the petitioner apparently
want to read into the Amparo Rule is to make this Rule a token gesture of judicial concern for
violations of the constitutional rights, liberty and security.

To read the Rules of Court requirement on pleadings while addressing the unique Amparo
situation, the test in reading the petition should be to determine whether it contains the details
available to the petitioner under the circumstances, while presenting a cause of action showing a
violation of the victim’s rights to life, liberty and security through State or private party action.
The petition should likewise be read in its totality, rather than in terms of its isolated component
parts, to determine if the required elements – namely, of the disappearance, the State or private
action, and the actual or threatened violations of the rights to life, liberty or security – are present.

2.) YES.
The petitioner needs only to properly comply with the substance and form requirements of a Writ
of Amparo petition, and prove the allegations by substantial evidence. Once a rebuttable case has
been proven, the respondents must then respond and prove their defenses based on the standard of
diligence required. The rebuttable case must show that an enforced disappearance took place
under circumstances showing a violation of the victim’s constitutional rights to life, liberty or
security, and the failure on the part of the investigating authorities to appropriately respond.

Substantial evidence is more than a mere scintilla. It means such relevant evidence as a
reasonable mind might accept as adequate to support a conclusion. The statute provides that the
rules of evidence prevailing in courts of law and equity shall not be controlling.

3.) NO.
The writ of amparo does not determine guilt nor pinpoint criminal culpability for the
disappearance, rather, it determines responsibility, or at least accountability, for the enforced
disappearance for purposes of imposing the appropriate remedies to address the disappearance.
The concept of “enforced disappearance” is neither defined nor penalized in this jurisdiction.

Responsibility refers to the extent the actors have been established by substantial evidence to
have participated in whatever way, by action or omission, in an enforced disappearance, as a
measure of the remedies this Court shall craft, among them, the directive to file the appropriate
criminal and civil cases against the responsible parties in the proper courts.

Accountability, on the other hand, refers to the measure of remedies that should be addressed to
those who exhibited involvement in the enforced disappearance without bringing the level of their
complicity to the level of responsibility defined above; or who are imputed with knowledge
relating to the enforced disappearance and who carry the burden of disclosure; or those who
carry, but failed to discharge, the burden of extraordinary diligence in the investigation of the
enforced disappearance.

DOCTRINE:The characteristics of Amparo proceedings, namely of (1) being summary, and (2) the use
of substantial evidence as required level of proof), reveal the clear intent of the framers of the Amparo
Rule to have the equivalent of an administrative proceeding in addressing Amparo situations.

The writ of amparo does not determine guilt nor pinpoint criminal culpability for the disappearance,
rather, it determines responsibility, or at least accountability, for the enforced disappearance for
purposes of imposing the appropriate remedies to address the disappearance.

The concept of “enforced disappearance” is neither defined nor penalized in this jurisdiction.

Remedial Law Review II (CY 2018-2019) – Saturday 1:00pm – 5:00pm: Case Digest Compilation [Atty. Brondial] Page 287
Remedial Law Review II (CY 2018-2019) – Saturday 1:00pm – 5:00pm: Case Digest Compilation [Atty. Brondial] Page 288
I.5. IN THE MATTER OF THE PETITION FOR THE WRIT OF AMPARO AND THE WRIT OF
HABEAS DATA IN FAVOR OF MELISSA C. ROXAS, MELISSA C. ROXAS, Petitioner, vs.
GLORIA MACAPAGAL-ARROYO, GILBERT TEODORO, GEN. VICTOR S. IBRADO, P/DIR.
GEN. JESUS AME VERZOSA, LT. GEN. DELFIN N. BANGIT, PC/SUPT. LEON NILO A.
DELA CRUZ, MAJ. GEN. RALPH VILLANUEVA, PS/SUPT. RUDY GAMIDO LACADIN, AND
CERTAIN PERSONS WHO GO BY THENAME[S] DEX, RC AND ROSE, Respondents.

G.R. No. 189155; September 7, 2010

FACTS: Petitioner is an American citizen of Filipino descent. While in the United States, enrolled in an
exposure program to the Philippines with the group Bagong Alyansang Makabayan-United States of
America (BAYAN- USA) of which she is a member. After doing survey work on 19 May 2009, petitioner
and her companions rested in the house of Mr. Jesus Paolo in Sitio Bagong Sikat. While Roxas and her
companions were resting, 15 heavily armed menin civilian clothes forcibly entered the house and
dragged theminside a van. When they alighted from the van, she was informed that she is being
detained for being a member of CommunistParty of the Philippines-New People’s Army (CPP-
NPA). She was then separated from her companions and was brought to a room, from where she could
hear sounds of gunfire, noise of planes taking off and landing, and some construction bustle. She
wasinterrogated and tortured for 5 straight days to convince her toabandon her communist beliefs.
She was informed by a person named “RC” that those who tortured her came from the “Special
Operations Group” and that she was abducted because her name is included in the “Order of Battle.”

On 25 May 2009, petitionerwas finally released and returned to her uncle’s house in Quezon
City. Before being released, however, the abductors gave petitioner a cellular phone with a SIM card, a
slip of paper containing an email address with password, a plastic bag containing biscuits and books, the
handcuffs used on her, a blouse and a pair of shoes. Petitioner was also sternly warned not to report the
incident to the group Karapatan or something untoward will happen to her and her family. Sometime after
her release, petitioner continued to receive calls from RC via the cellular phone given to her. Out of
apprehension that she was being monitored and also fearing for the safety of her family, petitioner threw
away the cellularphone with a SIM card. Hence, on 01 June 2009, Roxas filed apetition for the
issuance of Writs of Amparo and Habeas Databefore the Supreme Court, impleading the high-
ranking officialsof military and Philippine National Police (PNP), on the beliefthat it was the
government agents who were behind herabduction and torture. The Court of Appeals was not
convinced that the military or any other person acting under the acquiescence of the government, were
responsible for the abduction and torture of the petitioner. The appellate court stressed that, judging by
her own statements, the petitioner merely "believed" that the military was behind her abduction. Thus, the
Court of Appeals absolved the public respondents from any complicity in the abduction and torture of
petitioner. The petition was likewise dismissed as against public respondentPresident Gloria
Macapagal-Arroyo, in view of her immunityfrom suit. Accordingly, the petitioner’s prayers for the
return of her personal belongings were denied. Petitioner’s prayers for an inspection order and production
order also met the same fate.

Hence, this appeal by the petitioner.

ISSUES:
1. Whether or not the doctrine of command responsibility is applicable in an amparo petition?
2. Whether or not circumstantial evidence with regard to the identity and affiliation of the
perpetrators is enough ground for the issuance of the privilege of the writ of amparo?
3. Whether or not substantial evidence to prove actual or threatened violation of the right to privacy
in life, liberty or security of the victim is necessary before the privilege of the writ

RULING:
1. No, command responsibilityas justification in impleading respondents is legally inaccurate –
if not incorrect. Such doctrine is a rule of substantive law that establishes liability and, by this
account, cannot be a proper legal basis to implead a party-respondent in an amparo petition. As
held in the case of Rubrico v. Arroyo, the writ of amparo is aprotective remedy aimed at
providing judicial relief consistingof the appropriate remedial measures and directives that
maybe crafted by the court, in order to address specific violations orthreats of violation of
the constitutional rights to life, liberty orsecurity. It does not fix liability for such
disappearance, killing orthreats, whether that may be criminal, civil or administrative under the
applicable substantive law. Since the application of command responsibility presupposes an
imputation of individual liability, it is more aptly invoked in a full-blown criminal or

Remedial Law Review II (CY 2018-2019) – Saturday 1:00pm – 5:00pm: Case Digest Compilation [Atty. Brondial] Page 289
administrative case rather than in a summary amparo proceeding. However, the inapplicability
of the doctrine ofcommand responsibility does not preclude impleading militaryor police
commanders on the ground that the complained actsin the petition were committed with
their direct or indirectacquiescence. In which case, commanders may be impleaded — not
actually on the basis of command responsibility—but rather on the ground of their
responsibility, or at least accountability.

2. No, in amparo proceedings, the weight that may be accorded to parallel circumstances as
evidence of military involvement depends largely on the availability or non-availability of other
pieces of evidence that has the potential of directly proving the identity and affiliation of the
perpetrators. Direct evidence of identity, when obtainable, must be preferred over mere
circumstantial evidence based on patterns and similarity, because the former indubitably
offers greater certainty as to the true identity and affiliation of the perpetrators

3. Yes, an indispensable requirement before the privilege of the writ may be extended is the
showing, at least by substantial evidence, of an actual or threatened violation of the right to
privacy in life, liberty or security of the victim. In the case at bar, Roxas failed to show that
there is an actual or threatened violation of such right. Hence, until such time that any of the
respondents were found to be actually responsible for the abduction and torture of Roxas, any
inference regarding the existence of reports being kept in violation of the petitioner’s right to
privacy becomes farfetched, and premature. The Court must, at least in the meantime, strike
down the grant of the privilege of the writ of habeas data.

Remedial Law Review II (CY 2018-2019) – Saturday 1:00pm – 5:00pm: Case Digest Compilation [Atty. Brondial] Page 290
I.6. EDITA T. BURGOS, Petitioner, v. GEN. HERMOGENES ESPERONBRION, J.

FACTS: At around 1:00 in the afternoon of April 28, 2007, Jonas Joseph T. Burgos – a farmer advocate
and a member of Kilusang Magbubukid sa Bulacan was forcibly taken and abducted by (4) men and a
woman from the extension portion of Hapag Kainan Restaurant, located at the ground floor of Ever
Gotesco Mall, Commonwealth Avenue, Quezon City. On April 30, 2007, the petitioner, Edita Burgos,
held a press conference and announced that her son Jonas was missing. That same day, the petitioner
sought confirmation from the guard if the person abducted was her son Jonas. In a subsequent police
investigation and Land Transportation Office (LTO) verification, it was discovered that plate number
TAB 194 was registered to a 1991 Isuzu XLT vehicle owned by a certain Mauro B. Mudlong. The said
vehicle was seized and impounded on June 24, 2006 for transporting timber without permit. However, in
May 2007, right after Jonas’ abduction was made public, it was discovered that plate number TAB 194 of
this 1991 Isuzu XLT vehicle was missing, and the engine and other spare parts were “cannibalized.” The
police was likewise able to generate cartographic sketches of two of the abductors of Jonas based on its
interview of eyewitnesses. (PNPCIDG) presented Emerito Lipio a.k.a. Ka Tibo/Ka Cris, Marlon D.
Manuel a.k.a. Ka Carlo, and Melissa Concepcion Reyes a.k.a. Ka Lisa/Ramil to support the theory that
elements of the New People’s Army (NPA) perpetrated the abduction of Jonas. (CA) dismissed the
petition for the Issuance of the Writ of Habeas Corpus, denied the petitioner’s motion to declare the
respondents in contempt; and partially granted the privilege of the Writ of Amparo in favor of the
petitioner. Essentially, the CA found that the evidence the petitioner presented failed to establish her
claimed direct connection between the abductors of Jonas and the military. It also found that the (AFP)
and the PNP did not fully exert their effort in the conduct of loose ends regarding the identity of Ka
Ramon and the allegation that Ka Ramon is indeed Jonas in the “Order of Battle.” As for the PNPCIDG,
the CA branded its investigation as “rather shallow” and “conducted haphazardly.”

ISSUE: Whether or not the failure of the PNP and AFP to conduct an exhaustive and meaningful
investigation and to exercise extraordinary diligence in the performance of their duties is fatal to the grant
of the privilege of the Writ of Amparo?

RULING: Yes, PRIVILEGE OF THE WRIT OF AMPARO. Considering the findings of the CA and our
review of the records of the present case, we conclude that the PNP and the AFP have so far failed to
conduct an exhaustive and meaningful investigation into the disappearance of Jonas Burgos, and to
exercise the extraordinary diligence (in the performance of their duties) that the Rule on the Writ of
Amparo requires. Because of these investigative shortcomings, we cannot rule on the case until a more
meaningful investigation, using extraordinary diligence, is undertaken. In disposing of the case, the
Supreme Court issued the following directives: DIRECTED the Commission on Human Rights to
conduct appropriate investigative proceedings, including field investigations – acting as the Court’s
directly commissioned agency for purposes of the Rule on the Writ of Amparo, REQUIRE the incumbent
Chiefs of the Armed Forces of the Philippines and the Philippine National Police to make available and to
provide copies, to the Commission on Human Rights, of all documents and records in their possession
and as the Commission on Human Rights may require, relevant to the case of Jonas Joseph T. Burgos,
subject to reasonable regulations consistent with the Constitution and existing laws; DIRECTED the
PNP-CIDG and its incumbent Chief to submit to the Commission on Human Rights the records and
results of the investigation the PNP-CIDG claimed to have forwarded to the Department of Justice, which
were not included in their previous submissions to the Commission on Human Rights, including such
records as the Commission on Human Rights may require, pursuant to the authority granted under this
Resolution; DIRECTED the PNP-CIDG to provide direct investigative assistance to the Commission on
Human Rights as it may require, pursuant to the authority granted under this Resolution; AUTHORIZED
the Commission on Human Rights to conduct a comprehensive and exhaustive investigation that extends
to all aspects of the case (not limited to the specific directives as outlined above), as the extraordinary
measures the case may require under the Rule on the Writ of Amparo; and REQUIRED the Commission
on Human Rights to submit to this Court a Report with its recommendations, copy furnished the
petitioner, the incumbent Chiefs of the AFP, the PNP and the PNP-CIDG, and all the respondents, within
ninety (90) days from receipt of the Resolution. In light of the retirement of Lt. General Alexander Yano
and the reassignment of the other respondents who have all been impleaded in their official capacities, all
subsequent resolutions and actions from the Supreme Court were served on, and directly enforceable by,
the incumbents of the impleaded offices/units whose official action is necessary. The present respondents
shall continue to be personally impleaded for purposes of the responsibilities and accountabilities they
may have incurred during their incumbencies.

Remedial Law Review II (CY 2018-2019) – Saturday 1:00pm – 5:00pm: Case Digest Compilation [Atty. Brondial] Page 291
I.7. MARICRIS D. DOLOT, CHAIRMAN OF THE BAGONG ALYANSANG MAKABAYAN-
SORSOGON, PETITIONER vs.HON. RAMON PAJE, IN HIS CAPACITY AS THE
SECRETARY OF THE DEPARTMENT OF ENVIRONMENT AND NATURAL RESOURCES,
REYNULFO A. JUAN, REGIONAL DIRECTOR, MINES AND GEOSCIENCES BUREAU,
DENR, HON. RAUL R. LEE, GOVERNOR, PROVINCE OF SORSOGON, ANTONIO C.
OCAMPO, JR., VICTORIA A. AJERO, ALFREDO M. AGUILAR, AND JUAN M. AGUILAR,
ANTONES ENTERPRISES, GLOBAL SUMMIT MINES DEV'T CORP., AND TR
ORE, RESPONDENTS.

G.R. No. 199199; August 27, 2013; REYES, J.:

FACTS:Petitioner Maricris D. Dolot (Dolot), together with the parish priest of the Holy Infant Jesus
Parish and the officers of Alyansa Laban sa Mina sa Matnog (petitioners), filed a petition for continuing
mandamus, damages and attorney’s fees with the RTC of Sorsogon. Sometime in 2009, they protested the
iron ore mining operations being conducted by Antones Enterprises, Global Summit Mines Development
Corporation and TR Ore in Barangays Balocawe and Bon-ot Daco. Matnog is susceptible to flooding and
landslides, and confronted with the environmental dangers of flood hazard. After investigation, they
learned that the mining operators did not have the required permit to operate. They allege that the
respondents violated Republic Act (R.A.) No. 7076 or the People’s Small-Scale Mining Act of 1991, R.A.
No. 7942 or the Philippine Mining Act of 1995, and the Local Government Code. Thus, they prayed for
the following reliefs: (1) the issuance of a writ commanding the respondents to immediately stop the
mining operations in the Municipality of Matnog; (2) the issuance of a temporary environment protection
order or TEPO; (3) the creation of an inter-agency group to undertake the rehabilitation of the mining site;
(4) award of damages; and (5) return of the iron ore, among others.

Aside from sustaining the dismissal of the case for lack of jurisdiction, the RTC 11 further ruled that: (1)
there was no final court decree, order or decision yet that the public officials allegedly failed to act on,
which is a condition for the issuance of the writ of continuing mandamus; (2) the case was prematurely
filed as the petitioners therein failed to exhaust their administrative remedies; and (3) they also failed to
attach judicial affidavits and furnish a copy of the complaint to the government or appropriate agency, as
required by the rules

ISSUE:Whether the petition is dismissible on the grounds there is no final court decree, order or decision
that the public officials allegedly failed to act on

RULING: No. None is more well-settled than the rule that jurisdiction, which is the power and authority
of the court to hear, try and decide a case, is conferred by law. It may either be over the nature of the
action, over the subject matter, over the person of the defendants or over the issues framed in the
pleadings. By virtue of Batas Pambansa (B.P.) Blg. 129 or the Judiciary Reorganization Act of 1980,
jurisdiction over special civil actions for certiorari, prohibition and mandamus is vested in the RTC .
Particularly, Section 21(1) thereof provides that the RTCs shall exercise original jurisdiction – in the
issuance of writs of certiorari, prohibition, mandamus, quo warranto, habeas corpus and injunction which
may be enforced in any part of their respective regions. At this juncture, the Court affirms the continuing
applicability of Admin. Circular No. 23-2008 constituting the different "green courts" in the country and
setting the administrative guidelines in the raffle and disposition of environmental cases. While the
designation and guidelines were made in 2008, the same should operate in conjunction with the Rules.
The writ of continuing mandamus is a special civil action that may be availed of "to compel the
performance of an act specifically enjoined by law." The petition should mainly involve an environmental
and other related law, rule or regulation or a right therein. The RTC’s mistaken notion on the need for a
final judgment, decree or order is apparently based on the definition of the writ of continuing mandamus
under Section 4, Rule 1 of the Rules, to wit: (c) Continuing mandamus is a writ issued by a court in an
environmental case directing any agency or instrumentality of the government or officer thereof to
perform an act or series of acts decreed by final judgment which shall remain effective until judgment is
fully satisfied. The final court decree, order or decision erroneously alluded to by the RTC actually
pertains to the judgment or decree that a court would eventually render in an environmental case for
continuing mandamus and which judgment or decree shall subsequently become final.

Doctrine:Continuing mandamus is a writ issued by a court in an environmental case directing any


agency or instrumentality of the government or officer thereof to perform an act or series of acts decreed
by final judgment which shall remain effective until judgment is fully satisfied. The final court decree,
order or decision erroneously alluded to by the RTC actually pertains tothe judgment or decree that

Remedial Law Review II (CY 2018-2019) – Saturday 1:00pm – 5:00pm: Case Digest Compilation [Atty. Brondial] Page 292
acourt would eventually render in an environmental case for continuing mandamus and which judgment
or decree shall subsequently become final.

Remedial Law Review II (CY 2018-2019) – Saturday 1:00pm – 5:00pm: Case Digest Compilation [Atty. Brondial] Page 293
I.8. MANILA ELECTRIC COMPANY v. ROSARIO GOPEZ LIM,

GR No. 184769; 2010-10-05

FACTS:Rosario G. Lim (respondent), also known as Cherry Lim, is an administrative clerk at the Manila
Electric Company (MERALCO). An anonymous letter was posted at the door of the Metering Office of
the Administration building of MERALCO Plaridel, Bulacan  Sector, at which respondent is assigned,
denouncing respondent Cherry Lim. Copies of the letter were also inserted in the lockers of MERALCO
linesmen. Alexander Deyto, Head of MERALCO's Human Resource Staffing, directed the transfer of
respondent to MERALCO's Alabang Sector in Muntinlupa in light of the receipt of "... reports that there
were accusations and threats directed against [her] from unknown individuals and which could possibly
compromise [her] safety and security."

Respondent appealed her transfer and requested for a dialogue so she could voice her concerns and
misgivings on the matter, claiming that... the "punitive" nature of the transfer amounted to a denial of due
process.  Citing the grueling travel from her residence in Pampanga to Alabang and back entails, and
violation of the provisions on job security of their Collective Bargaining Agreement (CBA), respondent...
expressed her thoughts on the alleged threats to her security. No response to her request having been
received, respondent filed a petition for the issuance of a writ of habeas data against petitioners before the
Regional Trial Court (RTC) of Bulacan. By respondent's allegation, petitioners' unlawful act and
omission consisting of their continued failure and refusal to provide her with details or information about
the alleged report which MERALCO purportedly received concerning threats to her safety and security...
amount to a violation of her right to privacy in life, liberty and security, correctible by habeas data.
Additionally, respondent prayed for the issuance of a Temporary Restraining Order (TRO) enjoining
petitioners from effecting her transfer to the MERALCO Alabang Sector. Petitioners moved for the
dismissal of the petition and recall of the TRO on the grounds that, inter alia, resort to a petition for writ
of habeas data was not in order; and the RTC lacked jurisdiction over the case which properly belongs to
the National Labor

The trial court granted the prayers of respondent including the issuance of a writ of preliminary injunction
directing petitioners to desist from implementing respondent's transfer. The trial court justified its ruling
by declaring that, inter alia, recourse to a writ of habeas data should extend not only to victims of extra-
legal killings and political activists but also to ordinary citizens, like respondent whose rights to life and
security... are jeopardized by petitioners' refusal to provide her with information or data on the reported
threats to her person.

ISSUE:Whether the Writ of Hebeas Data was the right remedy for Lim

RULING: No. Section 1 of the Rule on the Writ of Habeas Data provides:
Section 1. Habeas Data. – The writ of habeas data is a remedy available to any person whose right to
privacy in life, liberty or security is violated or threatened by an unlawful act or omissionof a public
official or employee or of a private individual or entity engaged in the gathering, collecting or storing
of data or information regarding the person, family, home and correspondence of the aggrieved party.

The habeas data rule, in general, is designed to protect by means of judicial complaint the image, privacy,
honor, information, and freedom of information of an individual. It is meant to provide a forum to enforce
one’s right to the truth and to informational privacy, thus safeguarding the constitutional guarantees of a
person’s right to life, liberty and security against abuse in this age of information technology.

The writs of amparo and habeas data will NOT issue to protect purely property or commercial concerns
nor when the grounds invoked in support of the petitions therefor are vague or doubtful. Employment
constitutes a property right under the context of the due process clause of the Constitution. It is evident
that respondent’s reservations on the real reasons for her transfer - a legitimate concern respecting the
terms and conditions of one’s employment - are what prompted her to adopt the extraordinary remedy of
habeas data. Jurisdiction over such concerns is inarguably lodged by law with the NLRC and the Labor
Arbiters.

In another vein, there is no showing from the facts presented that petitioners committed any unjustifiable
or unlawful violation of respondent’s right to privacy vis-a-vis the right to life, liberty or security. To
argue that petitioners’ refusal to disclose the contents of reports allegedly received on the threats to
respondent’s safety amounts to a violation of her right to privacy is at best speculative. Respondent in fact
trivializes these threats and accusations from unknown individuals in her earlier-quoted portion of her

Remedial Law Review II (CY 2018-2019) – Saturday 1:00pm – 5:00pm: Case Digest Compilation [Atty. Brondial] Page 294
July 10, 2008 letter as "highly suspicious, doubtful or are just mere jokes if they existed at all." And she
even suspects that her transfer to another place of work "betray[s] the real intent of management]" and
could be a "punitive move." Her posture unwittingly concedes that the issue is labor-related.

Remedial Law Review II (CY 2018-2019) – Saturday 1:00pm – 5:00pm: Case Digest Compilation [Atty. Brondial] Page 295
I.9. Dr. Joy Margate Lee, Petitioner vs. P/Supt. Neri A. Ilagan, Respondent

G. R. No. 203254; October 8, 2014; Ponente: Perlas-Bernabe, J.

FACTS:Respondent filed a petition for Issuance of the Writ of Habeas Data alleging that Petitioner,
former common law partner of respondent, whom filed criminal and administrative complaints against
him by utilizing as evidence a purported sex video by respondent with another woman. Respondent
claims that Petitioner’s acts of reproducing the subject video and threatening to distribute the same to the
upper echelons of the NAPOLCOM and uploading it to the internet violated not only his right to life,
liberty, security, and privacy but also that of the other woman, and thus, the issuance of a writ of habeas
data in his favor is warranted.

RTC issued a Writ of Habeas Data directing Petitioner to appear before the court a quo, and to produce
Respondent’s digital camera, as well as the negative and/or original of the subject video and copies
thereof, and to file a verified written return within five (5) working days from date of receipt thereof.

In Petitioner’s verified complaint, she admitted that she indeed kept the memory card of the digital
camera and reproduced the aforesaid video but averred that she only did so to utilize the same as evidence
in the cases she filed against Petitioner. Respondent contended that Petitioner’s because (a) its filing was
only aimed at suppressing the evidence against Petitioner in the cases she filed; and (b) she is not engaged
in the gathering, collecting, or storing of data regarding the person of Ilagan.

RTC granted the privilege of the writ of habeas data in Petitioner’s favor, and accordingly, ordered the
implementing officer to turn-over copies of the subject video to him, and enjoined Lee from further
reproducing the same

Dissatisfied, Petitioner field before the SC a petitioner for review assailing the decision of the RTC in
extending the privilege of the writ of habeas data in favor of Respondent

ISSUE:
1. WON the RTC correctly extended the privilege of the writ of habeas data in favor of Respondent.

RULING: NO.
Accordingly,the Petition for Issuance of the Writ of Habeas Data filed by respondent must bedismissed
for lack of merit.

[…] in order to support a petition for the issuance ofsuch writ, Section 6 of the Habeas Data Rule
essentially requires that thepetition sufficiently alleges, among others, “the manner the right toprivacy is
violated or threatened and how it affects the right to life,liberty or security of the aggrieved party.” In
other words, the petitionmust adequately show that there exists a nexus between the right toprivacy on the
one hand, and the right to life, liberty or security on the other. Corollarily, the allegations in the petition
must be supported bysubstantial evidence showing an actual or threatened violation of the rightto privacy
in life, liberty or security of the victim. In this relation, it bearspointing out that the writ of habeas data
will not issue to protect purelyproperty or commercial concerns nor when the grounds invoked in
supportof the petitions therefor are vague and doubtful.

In this case, the Court finds that Respondent was not able to sufficientlyallege that his right to privacy in
life, liberty or security was or would beviolated through the supposed reproduction and threatened
dissemination ofthe subject sex video. While Respondent purports a privacy interest in thesuppression of
this video – which he fears would somehow find its way toQuiapo or be uploaded in the internet for
public consumption – he failed toexplain the connection between such interest and any violation of his
right to life, liberty or security. […] alleging and eventually proving the nexus between one’sprivacy right
to the cogent rights to life, liberty or security are crucial inhabeas data cases, so much so that a failure on
either account certainlyrenders a habeas data petition dismissible, as in this case.

[…] Hence, due to the insufficiency of theallegations as well as the glaring absence of substantial
evidence, the Courtfinds it proper to reverse the R TC Decision and dismiss the habeas datapetition.

Remedial Law Review II (CY 2018-2019) – Saturday 1:00pm – 5:00pm: Case Digest Compilation [Atty. Brondial] Page 296
I.10 ARIGO vs SWIFTEn Banc, MARTIN S. VILLARAMA, JR

G.R. No. 206510;735 SCRA 102; September 16, 2014

FACTS: In 2013, the USS Guardian of the US Navy ran aground on an area near the Tubbataha Reefs, a
marine habitat of which entry and certain human activities are prevented and afforded protection by a
Philippine law. The grounding incident prompted the petitioners to seek for issuance of Writ of Kalikasan
with TEPO from the SC.

Among those impleaded are US officials in their capacity as commanding officers of the US Navy. As
petitioners argued, they were impleaded because there was a waiver of immunity from suit between US
and PH pursuant to the VFA terms.

Petitioners claimed that the grounding, salvaging and post-salvaging operations of the USS Guardian
violated their constitutional rights to a balanced and healthful ecology since these events caused and
continue to cause environmental damage of such magnitude as to affect other provinces surrounding the
Tubbataha Reefs. Aside from damages, they sought a directive from the SC for the institution of civil,
administrative and criminal suits for acts committed in violation of environmental laws and regulations in
connection with the grounding incident.

ISSUE: Whether or not the petition for writ of kalikasan is proper?

RULING: No.

The application and non-application of criminal jurisdiction provisions of the VFA to US personnel who
may be found responsible for the grounding of the USS Guardian, would be premature and beyond the
province of the petition for a writ of Kalikasan, in the same vein, damages cannot be granted which have
resulted from a violation of environmental laws, the rules allows the recovery of damages, collection of
administrative fines in a separate civil suits or charging the same violation of an environmental law.

As asserted by the respondents (Philippines Officials) that the petition has become moot in the sense that
the salvage operation sought to be enjoined (under rule 7) or restrained had already been accomplished
when petitioners sought recourse from the court, still the respondents is entitled to the relief sought as far
as the directives of the respondents to protect and rehabilitate the coral reef structures and marine habitat
adversely affected by the grounding incidents, however the the US and Philippine government both
express readiness to negotiate and discuss the matter of compensation for the damage caused by USS
guardian, and the US government is closely coordinating to assessed the extent of damage and
appropriate methods of rehabilitation, more so, exploring avenues for settlement of environmental cases is
not proscribed by the rules.

Remedial Law Review II (CY 2018-2019) – Saturday 1:00pm – 5:00pm: Case Digest Compilation [Atty. Brondial] Page 297
2015 Supreme Court Decisions:

I.11. RAMON JESUS P. PAJE v. TEODORO A. CASIÑO,

GR No. 207257, 2015-02-03

FACTS:In February 2006, Subic Bay Metropolitan Authority (SBMA), a government agency organized
and established under Republic Act No. (RA) 7227,[4] and Taiwan Cogeneration Corporation (TCC)
entered into a Memorandum of Understanding (MOU) expressing their... intention to build a power plant
in Subic Bay which would supply reliable and affordable power to Subic Bay Industrial Park (SBIP).
On July 28, 2006, SBMA and TCC entered into another MOU, whereby TCC undertook to build and
operate a coal-fired power plant. In the said MOU, TCC identified 20 hectares of land at Sitio Naglatore,
Mt. Redondo, Subic Bay Freeport Zone (SBFZ) as... the suitable area for the project and another site of
approximately 10 hectares to be used as an ash pond.[7] TCC intends to lease the property from SBMA
for a term of 50 years with rent fixed at $3.50 per square meter, payable in 10 equal 5-year... installments.
On April 4, 2007, the SBMA Ecology Center issued SBFZ Environmental Compliance Certificate (ECC)
No. EC-SBFZ-ECC-69-21-500 in favor of Taiwan Cogeneration International Corporation (TCIC), a
subsidiary of TCC, for the construction, installation, and... operation of 2x150-MW Circulating Fluidized
Bed (CFB) Coal-Fired Thermal Power Plant at Sitio Naglatore.

On June 6, 2008, TCC assigned all its rights and interests under the MOU dated July 28, 2006 to Redondo
Peninsula Energy, Inc. (RP Energy), a corporation duly organized and existing under the laws of the
Philippines with the primary purpose of building,... owning, and operating power plants in the
Philippines, among others. Accordingly, an Addendum to the said MOU was executed by SBMA and RP
Energy.

On June 8, 2010, RP Energy and SBMA entered into a Lease and Development Agreement (LDA) over a
380,004.456-square meter parcel of land to be used for building and operating the coal-fired power plant.
On July 8, 2010, the DENR-EMB issued an amended ECC (first amendment) allowing the inclusion of
additional components, among others.

Several months later, RP Energy again requested the DENR-EMB to amend the ECC. Instead of
constructing a 2x150-MW coal-fired power plant, as originally planned, it now sought to construct a
1x300-MW coal-fired power plant. In... support of its request, RP Energy submitted a Project Description
Report (PDR) to the DENR-EMB.

On May 26, 2011, the DENR-EMB granted the request and further amended the ECC (second
amendment).

On August 1, 2011, the Sangguniang Panglalawigan of Zambales issued Resolution No. 2011-149,
opposing the establishment of a coal-fired thermal power plant at Sitio Naglatore, Brgy. Cawag, Subic,
Zambales.

On August 11, 2011, the Liga ng mga Barangay of Olongapo City issued Resolution No. 12, Series of
2011, expressing its strong objection to the coal-fired power plant as an energy source.

On July 20, 2012, Hon. Teodoro A. Casiño, Hon. Raymond V. Palatino, Hon. Rafael V. Mariano, Hon.
Emerenciana A. De Jesus, Clemente G. Bautista, Jr., Hon. Rolen C. Paulino, Hon. Eduardo Piano, Hon.
James de los Reyes, Hon. Aquilino Y. Cortez, Jr., Hon. Sarah Lugerna Lipumano-Garcia, Noraida
Velarmino, Bianca Christine Gamboa Espinos, Charo Simons, Gregorio Llorca Magdaraog, Rubelh
Peralta, Alex Corpus Hermoso, Rodolfo Sambajon, Rev. Fr. Gerardo Gregorio P. Jorge, Carlito A. Baloy,
Ofelia D. Pablo, Mario Esquillo, Elle Latinazo, Evangeline Q. Rodriguez, and John Carlo delos Reyes
(Casiño Group) filed before this Court a Petition for Writ of kalikasan against RP Energy, SBMA, and
Hon. Ramon Jesus P. Paje, in his capacity as Secretary of the DENR. [On July 31, 2012, this Court
resolved, among others, to: (1) issue a Writ of kalikasan; and (2) refer the case to the CA for hearing and
reception of evidence and rendition of judgment. [On September 11, 2012, the Petition for Writ of
kalikasan was docketed as CA-G.R. SP No. 00015 and raffled to the Fifteenth Division of the CA. In the
Petition, the Casiño Group alleged, among others, that the power plant project would cause... grave
environmental damage; that it would adversely affect the health of the residents of the municipalities of
Subic, Zambales, Morong, Hermosa, and the City of Olongapo; that the ECC was issued and the LDA
entered into... without the prior approval of the concerned sanggunians as required under Sections 26 and
27 of the Local Government Code (LGC); that the LDA was entered into without securing a prior

Remedial Law Review II (CY 2018-2019) – Saturday 1:00pm – 5:00pm: Case Digest Compilation [Atty. Brondial] Page 298
certification from the National Commission on Indigenous Peoples (NCIP) as required under Section 59
of RA 8371 or the Indigenous Peoples' Rights Act of 1997 (IPRA Law); that Section 8.3 of DENR
Administrative Order No. 2003-30 (DAO 2003-30) which allows amendments of ECCs is ultra vires
because the DENR has no authority to decide on requests for amendments of previously issued ECCs in
the absence of a new EIS;[36] and that due to the nullity of Section 8.3 of DAO 2003-30, all amendments
to RP Energy's ECC are null and void.

On October 29, 2012, the CA conducted a preliminary conference wherein the parties, with their
respective counsels, appeared except for Hon. Teodoro A. Casiño, Hon. Rafael V. Mariano, Hon.
Emerencia A. De Jesus, Clemente G. Bautista, Mario Esquillo, Elle Latinazo, Evangeline Q.

ISSUE: WoN the Casiño Group failed to substantiate its claims that the construction and operation of the
power plant will cause environmental damage
1. Yes. The appellate court correctly ruled that the Casiño Group failed to substantiate its claims
that the construction and operation of the power plant will cause environmental damage of the
magnitude contemplated under the writ of kalikasan. On the other hand, RP Energy presented
evidence to establish that the subject project will not cause grave environmental damage, through
its Environmental Management Plan, which will ensure that the project will operate within the
limits of existing environmental laws and standards;
WoN the ECC is invalid for lack of signature of Mr. Luis Miguel Aboitiz (Mr. Aboitiz), as representative
of RP Energy, in the Statement of Accountability of the ECC

2. No. The appellate court erred when it invalidated the ECC on the ground of lack of signature of
Mr. Aboitiz in the ECC’s Statement of Accountability relative to the copy of the ECC submitted
by RP Energy to the appellate court. While the signature is necessary for the validity of the ECC,
the particular circumstances of this case show that the DENR and RP Energy were not properly
apprised of the issue of lack of signature in order for them to present controverting evidence and
arguments on this point, as the issue only arose during the course of the proceedings upon
clarificatory questions from the appellate court. Consequently, RP Energy cannot be faulted for
submitting the certified true copy of the ECC only after it learned that the ECC had been
invalidated on the ground of lack of signature in the January 30, 2013 Decision of the appellate
court. The certified true copy of the ECC, bearing the signature of Mr. Aboitiz in the Statement
of Accountability portion, was issued by the DENR-EMB, and remains uncontroverted. It
showed that the Statement of Accountability was signed by Mr. Aboitiz on December 24, 2008.
Because the signing was done after the official release of the ECC on December 22, 2008,
wenote that the DENR did not strictly follow its rules, which require that the signing of the
Statement of Accountability should be done before the official release of the ECC. However,
considering that the issue was not adequately argued norwas evidence presented before the
appellate court on the circumstances at the time of signing, there is insufficient basis to conclude
that the procedure adoptedby the DENR was tainted with bad faith or inexcusable negligence.
We remind the DENR, however, to be more circumspect in following its rules. Thus, we rule that
the signature requirement was substantially complied with pro hac vice.

WoN the first and second amendments to the ECC are invalid for failure to undergo a new environmental
impact assessment (EIA) because of the utilization of inappropriate EIA documents.

3. No. The appellate court erred when it ruled that the first and second amendments to the ECC
were invalid for failure to comply with a new EIA and for violating DAO 2003-30 and the
Revised Manual. It failed to properly consider the applicable provisions in DAO 2003-30 and the
Revised Manual for amendment to ECCs. Our own examination of the provisions on
amendments to ECCs in DAO 2003-30 and the Revised Manual, as wellas the EPRMP and PDR
themselves, shows that the DENR reasonably exercised its discretion in requiring an EPRMP and
a PDR for the first and second amendments, respectively. Through these documents, which the
DENR reviewed, a new EIA was conducted relative to the proposed project modifications.
Hence, absent sufficient showing of grave abuse of discretion or patent illegality, relative to both
the procedure and substance of the amendment process, we uphold the validity of these
amendments;

WoN the Certificate of Non-Overlap (CNO), under Section 59 of the IPRA Law, is a precondition to the
issuance of an ECC and the lack of its prior issuance rendered the ECC invalid

Remedial Law Review II (CY 2018-2019) – Saturday 1:00pm – 5:00pm: Case Digest Compilation [Atty. Brondial] Page 299
4. No. The appellate court erred when it invalidated the ECC for failure to comply with Section 59
of the IPRA Law. The ECC is not the license or permit contemplated under Section 59 of the
IPRA Law and its implementing rules. Hence, there is no necessity to secure the CNO under
Section 59 before an ECC may be issued, and the issuance of the subject ECC without first
securing the aforesaid certification does not render it invalid;

WoN the Certificate of Non-Overlap (CNO), under Section 59 of the IPRA Law, is a precondition to the
consummation of the Lease and Development Agreement (LDA) between SBMA and RP Energy and the
lack of its prior issuance rendered the LDA invalid
5. No. The appellate court erred when it invalidated the LDA between SBMA and RP Energy fo
failure to comply with Section 59 of the IPRA Law. While we find that a CNO should have been
secured prior to the consummation of the LDA between SBMA and RP Energy, considering that
this is the first time we lay down the rule of action appropriate to the application of Section 59,
we refrain from invalidating the LDA for reasons of equity;

WoN compliance with Section 27, in relation to Section 26, of the LGC (i.e., approval of the concerned
sanggunian requirement) is necessary prior to the implementation of the power plant project

6. No. The appellate court erred when it ruled that compliance with Section 27, in relation to
Section 26, of the LGC (i.e., approval of the concerned sanggunian requirement) is necessary
prior to issuance of the subject ECC. The issuance of an ECC does not, by itself, result inthe
implementation of the project. Hence, there is no necessity to secure prior compliance with the
approval of the concerned sanggunian requirement, and the issuance of the subject ECC without
first complying with the aforesaid requirement does not render it invalid. The appellate court also
erred when it ruled that compliance with the aforesaid requirement is necessary prior to the
consummation of the LDA. By virtue of the clear provisions of RA 7227, the project is not
subject to the aforesaid requirement and the SBMA’s decision to approve the project prevails
over the apparent objections of the concerned sanggunians. Thus, the LDA entered into between
SBMA and RP Energy suffers from no infirmity despite the lack of approval of the concerned
sanggunians;

WoN the validity of the third amendment to the ECC can be resolved by the Court
7. Yes. The appellate court correctly ruled that the issue as to the validity of the third amendment
to the ECC cannot be resolved in this case because it was not one of the issues set during the
preliminary conference, and would, thus, violate RP Energy’s right to due process

Remedial Law Review II (CY 2018-2019) – Saturday 1:00pm – 5:00pm: Case Digest Compilation [Atty. Brondial] Page 300
I.12. Resident Marine Mammals of the Protected Seascape Tanon Strait vs. Angelo Reyes et Al.
RESIDENT MARINE MAMMALS OF PROTECTED SEASCAPE TANON STRAIT v.
SECRETARY ANGELO REYES,

GR No. 180771, 2015-04-21

FACTS: On June 13, 2002, the Government of the Philippines, acting through the DOE, entered into a
Geophysical Survey and Exploration Contractwith JAPEX. This contract involved geological and
geophysical studies of the Tañon Strait. The studies included surface geology,... sample analysis, and
reprocessing of seismic and magnetic data. JAPEX, assisted by DOE, also conducted geophysical and
satellite surveys, as well as oil and gas sampling in Tañon Strait.On December 21, 2004, DOE and
JAPEX formally converted Geophysical Survey and Exploration Contract into Service contract for the
exploration, development, and production of petroleum resources in a block covering approximately
2,850 square kilometers offshore the Tañon Strait.

JAPEX committed to drill one exploration well during the second sub-phase of the project. Since the well
was to be drilled in the marine waters of Aloguinsan and Pinamungajan, where the Tañon Strait was
declared a protected seascape in 1988,[10] JAPEX... agreed to comply with the Environmental Impact
Assessment requirements pursuant to Presidential Decree No. 1586, entitled "Establishing An
Environmental Impact Statement System, Including Other Environmental Management Related Measures
And For Other Purposes.

On January 31, 2007, the Protected Area Management Board[12] of the Tañon Strait (PAMB-Tañon
Strait) issued Resolution No. 2007-001,[13] wherein it adopted the Initial Environmental Examination
(IEE) commissioned by JAPEX, and... favorably recommended the approval of JAPEX's application for
an ECC.

It was in view of the foregoing state of affairs that petitioners (collectively referred to as the "Resident
Marine Mammals" in who are the toothed whales, dolphins, porpoises, and other cetacean species, which
inhabit the waters in and around the Tañon Strait. They are joined by Gloria Estenzo Ramosand Rose-
Liza Eisma-Osorio as their legal guardians and as friends who allegedly empathize with, and seek the
protection of, the aforementioned marine species. Petitioner applied to this Court for redress, via two
separate original petitions both dated December 17, 2007, wherein they commonly seek that respondents
be enjoined from implementing Service contract forProtesting the adverse ecological impact of JAPEX's
oil exploration activities in the Tañon Strait, petitioners Resident Marine Mammals and Stewards aver
that a study made after the seismic survey showed that the fish catch was reduced drastically by 50 to 70
percent. They claim... that before the seismic survey, the average harvest per day would be from 15 to 20
kilos; but after the activity, the fisherfolk could only catch an average of 1 to 2 kilos a day. They attribute
this "reduced fish catch" to the destruction of the "payao" also known as the"fish aggregating device" or
"artificial reef."

ISSUE: WHETHER OR NOT PETITIONERS (Resident Marine Mammals and Stewards) HAVE
LOCUS STANDI TO FILE THE INSTANT PETITION

Contention of the Petitioner:


The Resident Marine Mammals, through the Stewards, "claim" that they have the legal standing to file
this action since they stand to be benefited or injured by the judgment in this suit, citing Oposa v.
Factoran, Jr. they also assert their right to sue for the faithful performance of international and municipal
environmental laws created in their favor and for their benefit.

Contention of the Oppositor:


Public respondents argue that the Resident Marine Mammals have no standing because Section 1, Rule 3
of the Rules of Court requires parties to an action to be either natural or juridical persons
The public respondents also contest the applicability of Oposa, pointing out that the petitioners therein
were all natural persons, albeit some of them were still unborn.

Decision: Yes.

Under the Rules of Procedure for Environmental Cases,which allow for a "citizen suit," and permit any
Filipino citizen to file an action before our courts for violations of our environmental laws.
Citizen suits filed under R.A. No. 8749 and R.A. No. 9003 shall be governed by their respective
provisions.

Remedial Law Review II (CY 2018-2019) – Saturday 1:00pm – 5:00pm: Case Digest Compilation [Atty. Brondial] Page 301
To further encourage the protection of the environment, the Rules enable litigants enforcing
environmental rights to file their cases as citizen suits. This provision liberalizes standing for all cases
filed enforcing environmental laws and... collapses the traditional rule on personal and direct interest, on
the principle that humans are stewards of nature. The terminology of the text reflects the doctrine first
enunciated in Oposa v. Factoran, insofar as it refers to minors and generations yet... unborn.

Moreover, even before the Rules of Procedure for Environmental Cases became effective, this Court had
already taken a permissive position on the issue of locus standi in environmental cases. In Oposa, we
allowed the suit to be brought in the name of generations yet... unborn "based on the concept of
intergenerational responsibility insofar as the right to a balanced and healthful ecology is
concerned.Furthermore, we said that the right to a balanced and healthful ecology, a right that does not
even need to be... stated in our Constitution as it is assumed to exist from the inception of humankind,
carries with it the correlative duty to refrain from impairing the environment.

In light of the foregoing, the need to give the Resident Marine Mammals legal standing has been
eliminated by our Rules, which allow any Filipino citizen, as a steward of nature, to bring a suit to
enforce our environmental laws. It is worth noting here that the Stewards are... joined as real parties in the
Petition and not just in representation of the named cetacean species. The Stewards, Ramos and Eisma-
Osorio, having shown in their petition that there may be possible violations of laws concerning the habitat
of the Resident Marine Mammals, are... therefore declared to possess the legal standing to file this
petition.

Remedial Law Review II (CY 2018-2019) – Saturday 1:00pm – 5:00pm: Case Digest Compilation [Atty. Brondial] Page 302
I.13. West Tower Condominium vs. Phil. Ind, Corp. (on precautionary principle)
WEST TOWER CONDOMINIUM CORPORATION vs.FIRST PHILIPPINE INDUSTRIAL
CORPORATION

G.R. No. 194239; June 16, 2015; Velasco, Jr., J

FACTS: The respondent FPIC owns and operates two piplelines namely: (1) the White Oil Pipeline
(WOPL) System, 117 kilometers running from Batangas to Pandacan Terminal which transports diesel,
gasoline, kerosene and jet fuel; (2) Black Oil Pipeline (BOPL) System, 105 kilometer running from
Batangas to Sucat Paranaque, transporting bunker fuel. Both pipelines transport 60% of the petroleum
requirement of Manila, Rizal, Laguna and Bulacan. Both pipelines were designed more than the standard
requirements against leakage, which are buried 1.5 meters more that the US Standards of 0.9 meters.

In May 2010, it was discovered that a leak happened from the WOPL (FPIC initially denied this) in
Basement 2 of the West Tower Condominium (West Tower) which started in a two drums per day to 15-
20 drums per day. Unable to control the leak, West Tower reported the same to the Makati Police District
which in turn called the Bureau of Fire Protection. It was in October 28, 2010 when the UP National
Institute of Geological Science which revealed that the leak came from WOPL which is 86 meters from
West Tower. WOPL was closed on October 24, 2010.

On November 15, 2010, West Tower interposed the present Petition for the Issuance of a Writ of
Kalikasan on behalf of the residents of West Tower and in representation of the surrounding communities
in Barangay Bangkal, Makati City. West Tower Corp. also alleged that it is joined by the civil society and
several people's organizations, non-governmental organizations and public interest groups who have
expressed their intent to join the suit because of the magnitude of the environmental issues involved. In
their petition, petitioners prayed that respondents FPIC and its board of directors and officers, and First
Gen Corporation (FGC) and its board of directors and officers be directed to: (1) permanently cease and
desist from committing acts of negligence in the performance of their functions as a common
carrier; (2) continue to check the structural integrity of the whole 117-kilometer pipeline and to
replace the same; (3) make periodic reports on their findings with regard to the 117-kilometer
pipeline and their replacement of the same; (4) rehabilitate and restore the environment, especially
Barangay Bangkal and West Tower, at least to what it was before the signs of the leak became
manifest; and (5) to open a special trust fund to answer for similar and future contingencies in the
future. Furthermore, petitioners pray that respondents be prohibited from opening the pipeline and
allowing the use thereof until the same has been thoroughly checked and replaced, and be temporarily
restrained from operating the pipeline until the final resolution of the case.

On 19 November 2010, the Court issued the Writ of Kalikasan with TEPO. FPIC director in their return
alleged (1) the petitioners has no legal capacity to institute the action; (2) there are no allegation that
environmental damage was caused to two or more cities or provinces; (3) continued operation of the
pipeline should be allowed to maintain adequate supply of petroleum; (4) petition contains no allegation
that respondents FPIC directors and officers acted in such a manner as to allow the piercing of the
corporate veil. FGC directors on the other hand filed a return that they do not operate said pipeline and
that as far as they are concerned, they prayed that the writ and TEPO be declared performed as far as they
are concerned.

On 21 June 2011, FPIC reported to the Court their actions to address the structural integrity and the
preventive maintenance measures undertaken. It was also indicated in their report that it was the
Department of Energy who conducted the test of structural integrity which engaged the services of UP-
NIGS. Coordination with the DPWH and MMDA was done with regard to the preventive maintenance
measures.

Since the issuance of the writ and TEPO, FPIC ceased operation of both pipelines. On 31 May 2011 upon
the clarification from the DOE the Court answered that only WOPL is covered by the writ/TEPO. This
was assailed by the petitioners in its Omnibus Motion and prayed for oral arguments on the opening of
the BOPL. The petitioners in their Manifestation invoked the precautionary principle and asserted
that the possibility of a leak in the BOPL System leading to catastrophic environmental damage is
enough reason to order the closure of its operation. They likewise alleged that the entities contracted
by FPIC to clean and remediate the environment are illegally discharging waste water, which had not
undergone proper treatment, into the Parañaque River. Petitioners, thus, prayed that respondents be
directed to comply with environmental laws in rehabilitating the surroundings affected by the oil leak and

Remedial Law Review II (CY 2018-2019) – Saturday 1:00pm – 5:00pm: Case Digest Compilation [Atty. Brondial] Page 303
to submit a copy of their work plan and monthly reports on the progress thereof. To these omnibus
motions, respondents were directed to file their respective comments.

On September 28 , 2011 , respondent FPIC filed an Urgent Motion for Leave ( To Undertake "Bangkal
Realignment" Project) 13 in order to reduce stress on the WOPL System. FPIC sought to construct a new
realigned segment to replace the old pipe segment under the Magallanes Interchange, which covers the
portion that leaked. Petitioners were directed to file their comment on FPIC's motion.

To expedite resolution of the case, the Court remanded the case to the Court of Appeals for
recommendations.

RECOMMENDATIONS OF THE CA
1. FPIC to submit appropriate certification from the DOE on the safe commercial operation of
BOPL, otherwise its operation must be enjoined.
2. Denied its motion re: Bangkal Realignment Project
3. CA found that the existence of another possible leak alleged by petitioners was not established.
Nonetheless, to prevent such event, the CA ordered FPIC to: (i) review, adopt and strictly observe
appropriate safety and precautionary measures; (ii) closely monitor the conduct of its
maintenance and repair works; and (iii) submit to the DOE regular monthly reports on the
structural integrity and safe commercial operation of the pipeline.
4. As to the merits of the case:
a. People’s organization, NGO and public interest groups who indicated their intention and
submitted proof of juridical personality be allowed to be impleaded as petitioners.
b. FPIC to submit certification from DOE on WOPL’s safe commercial operation.
c. Denied creation of special trust fund.
d. First Gen not liable under the TEPO.

On 30 July 2013, the Court adopted the CA Recommendations and the the WOPL may resume operation
after securing a certification from the DOE that it is safe for commercial operations.

In compliance with the Court's July 30, 2013 Resolution, the DOE Secretary issued on October 25, 2013 a
Certification, 27 attesting that the WOPL is safe to resume commercial operations, subject to monitoring
or inspection requirements, and imposing several conditions that FPIC must comply with.

ISSUES:
1. Whether petitioner West Tower Corp. has the legal capacity to represent the other petitioners and
whether the other petitioners, apart from the residents of West Tower and Barangay Bangkal, are
real parties-in- interest;
2. Propriety of Converting the TEPO to PEPO or its Lifting in light of the DOE Certification of the
WOPL's Commercial Viability
3. Propriety of a special trust fund.
4. Liability of FPIC, FirstGen and their respective boards.

RULING:
1. Yes
As defined, a real party-in-interest is the party who stands to be benefited or injured by the judgment in
the suit, or the party entitled to the avails of the suit. Generally, every action must be prosecuted or
defended in the name of the real parties-in-interest. In other words, the action must be brought by the
person who, by substantive law, possesses the right sought to be enforced. Alternatively, one who has no
right or interest to protect cannot invoke the jurisdiction of the court as party-plaintiff-in-action for it is
jurisprudentially ordained that every action must be prosecuted or defended in the name of the real party-
in-interest. In the case at bar, there can be no quibble that the oil leak from the WOPL affected all the
condominium unit owners and residents of West Tower as, in fact, all had to evacuate their units at the
wee hours in the morning of July 23, 2010, when the condominium’s electrical power was shut down.
Until now, the unit owners and residents of West Tower could still not return to their condominium units.
Thus, there is no gainsaying that the residents of West Tower are real parties-in-interest. There can also
be no denying that West Tower Corp. represents the common interest of its unit owners and residents, and
has the legal standing to file and pursue the instant petition. While a condominium corporation has limited
powers under RA 4726, otherwise known as The Condominium Act, it is empowered to pursue actions in
behalf of its members. In the instant case, the condominium corporation is the management body of West
Tower and deals with everything that may affect some or all of the condominium unit owners or users.

Remedial Law Review II (CY 2018-2019) – Saturday 1:00pm – 5:00pm: Case Digest Compilation [Atty. Brondial] Page 304
The filing of a petition for the issuance of a writ of kalikasan under Sec. 1, Rule 7 of the Rules of
Procedure for Environmental Cases does not require that a petitioner be directly affected by an
environmental disaster. The rule clearly allows juridical persons to file the petition on behalf of persons
whose constitutional right to a balanced and healthful ecology is violated, or threatened with violation.—
Anent the propriety of including the Catholic Bishops’ Conference of the Philippines, Kilusang
Makabansang Ekonomiya, Inc., Women’s Business Council of the Philippines, Inc., Junior Chambers
International Philippines, Inc.-San Juan Chapter, Zonta Club of Makati Ayala Foundations, and the
Consolidated Mansions Condominium Corporation, as petitioners in the case, the Court already granted
their intervention in the present controversy in the adverted July 30, 2013 Resolution. This is so
considering that the filing of a petition for the issuance of a writ of kalikasan under Sec. 1, Rule 7 of the
Rules of Procedure for Environmental Cases does not require that a petitioner be directly affected by an
environmental disaster. The rule clearly allows juridical persons to file the petition on behalf of persons
whose constitutional right to a balanced and healthful ecology is violated, or threatened with violation.
Thus, as parties to the case, they are entitled to be furnished copies of all the submissions to the Court,
including the periodic reports of FPIC and the results of the evaluations and tests conducted on the
WOPL.

2. After a perusal of the recommendations of the DOE and the submissions of the parties, the Court
adopts the activities and measures prescribed in the DOE letter dated August 5, 2014 to be complied with
by FPIC as conditions for the resumption of the commercial operations of the WOPL. The DOE should,
therefore, proceed with the implementation of the tests proposed in the said August 5, 2014 letter.
Thereafter, if it is satisfied that the results warrant the immediate reopening of the WOPL, the DOE shall
issue an order allowing FPIC to resume the operation of the WOPL. On the other hand, should the probe
result in a finding that the pipeline is no longer safe for continued use and that its condition is
irremediable, or that it already exceeded its serviceable life, among others, the closure of the WOPL may
be ordered.

Lastly, any delay in the reopening of the WOPL, if said delay is for the purpose of making sure that the
pipeline is commercially viable, is better than hastily allowing its reopening without an extensive check
on its structural integrity when experience shows that there were and may still be flaws in the pipeline.
Even the DOE, the agency tasked to oversee the supply and distribution of petroleum in the country, is
well aware of this and even recommended the checking of the patched portions of the pipeline, among
others. In this regard, the Court deems it best to take the necessary safeguards, which are not similar to
applying the precautionary principle as previously explained, in order to prevent a similar incident from
happening in the future.

PRECAUTIONARY PRINCIPLE
Section 1, Rule 20 of A.M. No. 09-6-8-SC or the Rules of Procedure for Environmental Cases, on the
Precautionary Principle, provides that “[w]hen there is lack of full scientific certainty in establishing a
causal link between human activity and environmental effect, the court shall apply the precautionary
principle in resolving the case before it.” According to the dissent, the directive for the repetition of the
tests is based on speculations, justified by the application of said principle. This, however, is not the case.
Nowhere did We apply the precautionary principle in deciding the issue on the WOPL’s structural
integrity. The precautionary principle only applies when the link between the cause, that is the human
activity sought to be inhibited, and the effect, that is the damage to the environment, cannot be established
with full scientific certainty. Here, however, such absence of a link is not an issue. Detecting the existence
of a leak or the presence of defects in the WOPL, which is the issue in the case at bar, is different from
determining whether the spillage of hazardous materials into the surroundings will cause environmental
damage or will harm human health or that of other organisms. As a matter of fact, the petroleum leak and
the harm that it caused to the environment and to the residents of the affected areas is not even questioned
by FPIC.

3. No
A reading of the petition and the motion for partial reconsideration readily reveals that the prayer is for
the creation of a trust fund for similar future contingencies. This is clearly outside the limited purpose of a
special trust fund under the Rules of Procedure for Environmental Cases, which is to rehabilitate or
restore the environment that has presumably already suffered. Hence, the Court affirms with concurrence
the observation of the appellate court that the prayer is but a claim for damages, which is prohibited by
the Rules of Procedure for Environmental Cases. As such, the Court is of the considered view that the
creation of a special trust fund is misplaced. The present ruling on petitioners’ prayer for the creation of a
special trust fund in the instant recourse, however, is without prejudice to the judgment/s that may be

Remedial Law Review II (CY 2018-2019) – Saturday 1:00pm – 5:00pm: Case Digest Compilation [Atty. Brondial] Page 305
rendered in the civil and/or criminal cases filed by petitioners arising from the same incident if the
payment of damages is found warranted.

4. No ruling.
The Court will refrain from ruling on the finding of the CA that the individual directors and officers of
FPIC and FGC are not liable due to the explicit rule in the Rules of Procedure for Environmental cases
that in a petition for a writ of kalikasan, the Court cannot grant the award of damages to individual
petitioners under Rule 7, Sec. 15 (e) of the Rules of Procedure for Environmental Cases. As duly noted by
the CA, the civil case and criminal complaint filed by petitioners against respondents are the proper
proceedings to ventilate and determine the individual liability of respondents, if any, on their exercise of
corporate powers and the management of FPIC relative to the dire environmental impact of the dumping
of petroleum products stemming from the leak in the WOPL in Barangay Bangkal, Makati City.

DISSENTING: Leonen
The Writ of Kalikasan has served its functions and, therefore, is functus officio. The leaks have been
found and remedied. The various administrative agencies have identified the next steps that should ensure
a viable level of risk that is sufficiently precautionary. In other words, they have shown that they know
what to do to prevent future leaks. The rest should be left for them to execute.

Remedial Law Review II (CY 2018-2019) – Saturday 1:00pm – 5:00pm: Case Digest Compilation [Atty. Brondial] Page 306
IV. EVIDENCE (RULES 128-134)

A. PRELIMINARY CONSIDERATION

A.1. ONG CHIA vs. REPUBLIC OF THE PHILIPPINES and THE COURT OF APPEALS

G.R. No. 127240; March 27, 2000

FACTS: Petitioner was born in Amoy, China. In 1932, as a nine-year old boy, he arrived at the port of
Manila on board the vessel "Angking." Since then, he has stayed in the Philippines where he found
employment and eventually started his own business, married a Filipina, with whom he had four children.
On July 4, 1989, at the age of 66, he filed a verified petition to be admitted as a Filipino citizen under
C.A. No. 473, otherwise known as the Revised Naturalization Law.

The trial court granted the petition and admitted petitioner to Philippine citizenship. On appeal, CA
reversed the trial court and denied petitioner’s application for naturalization. It ruled that due to the
importance of naturalization cases, the State is not precluded from raising questions not presented in the
lower court and brought up for the first time on appeal.

Petitioner’s principal contention is that the appellate court erred in considering the documents which had
merely been annexed by the State to its appellant’s brief and, on the basis of which, justified the reversal
of the trial court’s decision. Not having been presented and formally offered as evidence during the trial,
they are mere "scraps of paper” devoid of any evidentiary value.

ISSUE: Whether or not the rules on evidence applies to a petition for naturalization.

RULING: The answer is in the negative. Rule 1, Sec. 4 provides that: “These rules shall not apply to land
registration, cadastral and election cases, naturalization and insolvency proceedings, and other cases not
herein provided for, except by analogy or in a suppletory character and whenever practicable and
convenient.”

Prescinding from the above, the rule on formal offer of evidence (Rule 132, §34) now being invoked by
petitioner is clearly not applicable to the present case involving a petition for naturalization. The only
instance when said rules may be applied by analogy or suppletorily in such cases is when it is "practicable
and convenient." That is not the case here, since reliance upon the documents presented by the State for
the first time on appeal, in fact, appears to be the more practical and convenient course of action
considering that decisions in naturalization proceedings are not covered by the rule on res judicata.
Consequently, a final favorable judgment does not preclude the State from later on moving for a
revocation of the grant of naturalization on the basis of the same documents.

Remedial Law Review II (CY 2018-2019) – Saturday 1:00pm – 5:00pm: Case Digest Compilation [Atty. Brondial] Page 307
A.2. ZULUETA VS. COURT OF APPEALS

G.R. No. 107383; February 20, 1996

FACTS: This is a petition to review the decision of the Court of Appeals, affirming the decision of the
Regional Trial Court of Manila (Branch X) which ordered petitioner to return documents and papers
taken by her from private respondent's clinic without the latter's knowledge and consent.

Petitioner Cecilia Zulueta is the wife of private respondent Alfredo Martin. On March 26, 1982, petitioner
entered the clinic of her husband, a doctor of medicine, and in the presence of her mother, a driver and
private respondent's secretary, forcibly opened the drawers and cabinet in her husband's clinic and took
157 documents consisting of private correspondence between Dr. Martin and his alleged paramours,
greetings cards, cancelled checks, diaries, Dr. Martin's passport, and photographs. The documents and
papers were seized for use in evidence in a case for legal separation and for disqualification from the
practice of medicine which petitioner had filed against her husband.

ISSUE:
(1) Whether or not the documents and papers in question are inadmissible in evidence;

RULING:
(1) No.  Indeed the documents and papers in question are inadmissible in evidence. The constitutional
injunction declaring "the privacy of communication and correspondence [to be] inviolable" is no less
applicable simply because it is the wife (who thinks herself aggrieved by her husband's infidelity) who is
the party against whom the constitutional provision is to be enforced. The only exception to the
prohibition in the Constitution is if there is a "lawful order [from a] court or when public safety or order
requires otherwise, as prescribed by law."  Any violation of this provision renders the evidence obtained
inadmissible "for any purpose in any proceeding."
 
The intimacies between husband and wife do not justify any one of them in breaking the drawers and
cabinets of the other and in ransacking them for any telltale evidence of marital infidelity. A person, by
contracting marriage, does not shed his/her integrity or his right to privacy as an individual and the
constitutional protection is ever available to him or to her.
 
The law insures absolute freedom of communication between the spouses by making it privileged.
Neither husband nor wife may testify for or against the other without the consent of the affected spouse
while the marriage subsists.  Neither may be examined without the consent of the other as to any
communication received in confidence by one from the other during the marriage, save for specified
exceptions.  But one thing is freedom of communication; quite another is a compulsion for each one to
share what one knows with the other. And this has nothing to do with the duty of fidelity that each owes
to the other.

Remedial Law Review II (CY 2018-2019) – Saturday 1:00pm – 5:00pm: Case Digest Compilation [Atty. Brondial] Page 308
A.3 People of the Philippines vs. Joel Yatar @ Kawit

G.R. No. 150224; May 19, 2004; Per Curiam

FACTS: On June 30, 1998 at 8:30 in the morning, the 17-year Kathylyn Uba was talking with her first
cousin, Judilyn Pas-a, about the letter sent by their aunt, Luz Yatar, to her husband, Joel Yatar. Kathylyn
handed the said letter to Joel earlier that morning. At 9:00 of the same morning, Judilyn and her husband
left for their farm. Kathylyn was left alone in the house. At around 10 in the morning, her aunt, Anita,
stopped by the house to drink water and saw Joel at the back of the house. She asked Joel what he was
doing there, and he replied that he was getting lumber to bring to the house of his mother. At around
12:30, while Judilyn was on her way home, she saw Joel descend the ladder of the house and run towards
the back of the house. She noticed Joel, who was wearing a white shirt with collar and black pants,
pacing back and forth. By 1:30 in the afternoon, Judilyn again saw Joel, but this time he was wearing a
black shirt without collar and blue pants, and she noticed that Joel’s eyes were “reddish and sharp”. In
that evening, the naked and lifeless body of Kathylyn Uba was found sprawled on the floor of the house
with her intestines protruding out of her stomach.

The people in the vicinity informed the police that Joel was seen going down the ladder of the house at
approximately 12:30 in the afternoon. The police likewise discovered the victim’s panties, brassiere,
denim pants, bag and sandals beside her cadaver, and they also found a dirty white shirt splattered with
blood within 50 meters from the house. Joel denied having any knowledge of Kathylyn’s death but when
he was placed under police custody, he tried to escape but was recaptured. Thus, Joel was indicted with
Rape with Homicide.

After the trial, Joel was convicted and was sentenced to death. Hence, the automatic review pursuant to
Article 47 of the Revised Penal Code.

ISSUES:

WON the evidence of the prosecution is sufficient to warrant the conviction of Joel Yatar considering that
there was no eyewitness during the crime.

WON Deoxyribonucleic acid (DNA) evidence is admissible vis-à-vis the constitutional right of the
accused against self-incrimination.

WON the findings of the trial court on the credibility of the witnesses should be given great weight on
appeal.

RULING:

YES. The weight of the prosecution’s evidence must be appreciated in light of the well-settled rule which
provides that an accused can be convicted even if no eyewitness is available, as long as sufficient
circumstantial evidence is presented by the prosecution to prove beyond doubt that the accused
committed the crime.

In assessing the probative value of DNA evidence, courts should consider, inter alia, the following
factors: how the samples were collected, how they were handled, the possibility of contamination of the
samples, the procedure followed in analyzing the samples, whether the proper standards and procedures
were followed in conducting the tests, and the qualification of the analyst who conducted the tests.
Significantly, subsequent testing showed that the Deoxyribonucleic acid (DNA) of the sperm specimen
from the vagina of the victim was identical the semen to be that of Joel’s gene type.

Further, the kernel of the right against self-incrimination is not against all compulsion, but against
testimonial compulsion. The right against self-incrimination is simply against the legal process of
extracting from the lips of the accused an admission of guilt. It does not apply where the evidence sought
to be excluded is not an incrimination but as part of object evidence. Hence, a person may be compelled
to submit to fingerprinting, photographing, paraffin, blood and DNA, as there is no testimonial
compulsion involved.

Furthermore, the findings of the trial court on credibility of witnesses are entitled to great weight on
appeal unless cogent reasons are presented necessitating a reexamination if not the disturbance of the
same; the reason being that the former is in a better and unique position of hearing first hand the

Remedial Law Review II (CY 2018-2019) – Saturday 1:00pm – 5:00pm: Case Digest Compilation [Atty. Brondial] Page 309
witnesses and observing their deportment, conduct and attitude. Absent any showing that the trial judge
overlooked, misunderstood, or misapplied some facts or circumstances of weight which would affect the
result of the case, the trial judge’s assessment of credibility deserves the appellate court’s highest respect.
Where there is nothing to show that the witnesses for the prosecution were actuated by improper motive,
their testimonies are entitled to full faith and credit.

In the case at bar, the following circumstantial evidence were sufficient to sustain a conviction beyond
reasonable doubt: (1) Joel and his wife were living in the house together with the victim, Kathylyn Uba;
(2) In June 1998, Joel’s wife left the house because of their frequent quarrels; (3) Joel received from the
victim, Kathylyn Uba, a letter from his estranged wife in the early morning on June 30, 1998; (4) Joel was
seen at 1:00 p.m. of June 30, 1998 near the kitchen of the house acting strangely and wearing a dirty
white shirt with collar; (5) Judilyn Pas-a saw Joel going down the ladder of the house at 12:30 p.m.,
wearing a dirty white shirt, and again at 1:30 p.m., this time wearing a black shirt; (6) Joel hurriedly left
when the husband of Judilyn Pas-a was approaching; (7) Joel was seen wearing a dirty white shirt coming
down the ladder of the house on the day Kathylyn Uba was found dead; (8) The door leading to the
second floor of the house was tied by a rope; (9) The victim, Kathylyn Uba, lay naked in a pool of blood
with her intestines protruding from her body on the second floor of the house with her stained pants, bra,
underwear and shoes scattered along the periphery; (10) Laboratory examination revealed sperm in the
victim’s vagina; (11) The stained or dirty white shirt found in the crime scene was found to be positive
with blood; (12) DNA taken at the scene of the crime compared with the DNA profile of the Joel are
identical; and (13) Joel escaped two days after he was detained but was subsequently apprehended, such
flight being indicative of his guilt.

Remedial Law Review II (CY 2018-2019) – Saturday 1:00pm – 5:00pm: Case Digest Compilation [Atty. Brondial] Page 310
A.3 NENA LAZALITA* TATING vs. FELICIDAD TATING MARCELLA, represented by
SALVADOR MARCELLA, CARLOS TATING, and the COURT OF APPEALS

G.R. No. 155208; March 27, 2007; AUSTRIA-MARTINEZ, J.:

FACTS: Carlos Tating and Felicidad Tating Marcella, represented by her son Salvador, filed a complaint
with the RTC against Nena Tating for, among others, the nullification of a Deed of Absolute Sale
executed by the late Daniela Solano Vda. de Tating in favor of the defendant, Nena Tating, when she was
alive.

Respondents are children of the late Daniela while the defendant was a granddaughter. When Daniela was
alive, she sold a parcel of land to the Petitioner. The sale was contained in the contested Deed which has
been duly notarized. Consequently, the title over the land was transferred to defendant. While the
Petitioner declared the property in her name for tax purposes and paid the real estate taxes due thereon,
the land remained in possession of Daniela.

Later on, Daniela executed a sworn statement claiming that she had no intention of selling the property,
and that the true agreement between them was simply to transfer the title in favor of the Petitioner so that
she could obtain a loan by mortgaging the land for the purpose of helping her defray some business
expenses; thus she wanted the reconveyance of the land to her. She died 10 years thereafter.

In the RTC, the sworn statement was admitted as part of the respondents’ evidence and gave credence to
it. On appeal, the CA also accorded great probative weight to the said document.

Hence, petitioner files a petition for certiorari (under Rule 65) averring that the decisions of the lower
courts on the instant case was not supported by and is even against the evidence on recordsconsidering
that the sole evidence of the respondents was the alleged sworn statement executed by Daniela which
should have been rejected outright because Daniela, the affiant, has long been dead when the document
was offered in evidence, thereby denying the petitioner the right to cross-examine her. Moreover,
petitioner asserts that clear, strong and convincing evidence beyond mere preponderance is required to
show the falsity or nullity of a notarial document.

ISSUE:
W/N the sworn statement should not have been admitted in the first place;
W/N the sworn statement alone is not enough to falsify a notarial document.

RULING: The Supreme Court said that there is no issue on the admissibility of the sworn statement (I do
not know why, but maybe because it was not properly objected and the court has the discretion to admit
pieces of evidence).

However, the sworn statement, alone, is not enough to falsify a notarial document. The admissibility of
evidence should not be equated with weight of evidence. The admissibility of evidence depends on
its relevance and competence while the weight of evidence pertains to the tendency of evidence
already admitted to convince and persuade. Thus, a particular item of evidence may be admissible,
but its evidentiary weight depends on judicial evaluation within the guidelines provided by the rules
of evidence.

It is settled that affidavits are classified as hearsay evidence since they are not generally prepared by
the affiant but by another who uses his own language in writing the affiant’s statements, which may
thus be either omitted or misunderstood by the one writing them. Moreover, the adverse party would be
deprived of the opportunity to cross-examine the affiant. For this reason, affidavits are generally
rejected for being hearsay, unless the affiants themselves are placed on the witness stand to testify
thereon.

In this case, any sworn statement or affidavit executed by Daniela has no probative value for being
hearsay. Naturally, none of the parties could be able to cross-examine the deceased affiant on her
declarations on the said documents.

Moreover, the respondents failed to present even a single tax receipt or declaration showing that Daniela
paid taxes on the disputed land as proof of ownership. Actually, what the respondents presented was a tax
declaration REFERRING ONLY TO THE HOUSE on the contested lot. It even contained a notation that

Remedial Law Review II (CY 2018-2019) – Saturday 1:00pm – 5:00pm: Case Digest Compilation [Atty. Brondial] Page 311
petitioner own the lot upon which the house was built (so the petitioner is correct when they alleged that
“the case was decided against the evidence on records”).

Finally, the respondents failed to prove by clear, strong and convincing evidence beyond mere
preponderance of evidence that the contract of sale between Daniela and petitioner was simulated. The
legal presumption is in favor of the validity of contracts and the party who impugns its regularity has the
burden of proving its simulation. The respondents failed to overturn the presumption of regularity.

Thus, the SupremeCourt finds that both the trial court and the CA committed error in giving the sworn
statement probative weight. Since Daniela is no longer available to take the witness stand as she is
already dead, the RTC and the CA should not have given probative value on Daniela’s sworn statement.

DOCTRINE:The admissibility of evidence should not be equated with weight of evidence. The
admissibility of evidence depends on its relevance and competence while the weight of evidence
pertains to the tendency of evidence already admitted to convince and persuade. Thus, a particular item
of evidence may be admissible, but its evidentiary weight depends on judicial evaluation within the
guidelines provided by the rules of evidence.

It is settled that affidavits are classified as hearsay evidence since they are not generally prepared by
the affiant but by another who uses his own language in writing the affiant’s statements, which may
thus be either omitted or misunderstood by the one writing them. Moreover, the adverse party would be
deprived of the opportunity to cross-examine the affiant. For this reason, affidavits are generally
rejected for being hearsay, unless the affiants themselves are placed on the witness stand to testify
thereon.

While a petition is denominated as a Petition for Certiorari under Rule 65 of the Rules of Court without
allegation of grave abuse of discretion, but actually avers errors of judgment, rather than of jurisdiction,
which are proper subjects of a petition for review on certiorari, the Supreme Court may invoke liberal
construction of the Rules of Court, pervading the treatment of a petition for certiorari as having been
filed under Rule 45, in the interest of justice, considering that it was filed within the reglementary period
for filing the same.

Remedial Law Review II (CY 2018-2019) – Saturday 1:00pm – 5:00pm: Case Digest Compilation [Atty. Brondial] Page 312
A.4 People of the Philippines vs. Rodrigo Salafranca

G.R. No. 173476 February 22, 2012

FACTS: Past midnight on July 31, 1993 Bolanon was stabbed near the Del PanSports Complex in
Binondo, Manila; that after stabbing Bolanon, his assailant ranaway; that Bolanon was still able to walk
to the house of his uncle Rodolfo B.Estaño in order to seek help; that his uncle rushed him to the
Philippine GeneralHospital by taxicab; that on their way to the hospital Bolanon told Estaño that itwas
Salafranca who had stabbed him; that Bolanon eventually succumbed at thehospital at 2:30 am despite
receiving medical attention; and that the stabbing ofBolanon was personally witnessed by Augusto
Mendoza, then still a minor of 13years, who was in the complex at the time.

As stated, Salafranca fled after stabbing Bolanon. He evaded arrest for a longperiod, despite the warrant
for his arrest being issued. He was finally arrested onApril 23, 2003, and detained at the Manila City Jail.
After trial, the RTC convicted Salafranca for murder.

On appeal, the CA affirmed the findings and conclusions of the RTC, citing thedying declaration made to
his uncle pointing to Salafranca as his assailant, andSalafranca’s positive identification as the culprit by
Mendoza. It stressed thatSalafranca’s denial and his alibi of being in his home during the incident did
notovercome the positive identification, especially as his unexplained flight after thestabbing, leaving his
home and employment, constituted a circumstance highlyindicative of his guilt.

Presently, Salafranca reiterates his defenses, and insists that the State did notprove his guilt beyond
reasonable doubt.

ISSUE: Whether the ante-mortem declaration of Bolanon meets the conditions ofadmissibility either as a
dying declaration or as a part of the res gestae, or both.

RULING: YES.
A dying declaration, although generally inadmissible as evidence due to its hearsaycharacter, may
nonetheless be admitted when the following requisites concur,namely: (a) that the declaration must
concern the cause and surroundingcircumstances of the declarant’s death; (b) that at the time the
declaration ismade, the declarant is under a consciousness of an impending death; (c) that thedeclarant is
competent as a witness; and (d) that the declaration is offered in acriminal case for homicide, murder, or
parricide, in which the declarant is a victim.

All the requisites were met herein. Bolanon communicated his ante-mortemstatement to Estaño,
identifying Salafranca as the person who had stabbed him. Atthe time of his statement, Bolanon was
conscious of his impending death, havingsustained a stab wound in the chest and, according to Estaño,
was thenexperiencing great difficulty in breathing. Bolanon succumbed in the hospital
emergency room a few minutes from admission, which occurred under three hoursafter the stabbing.
There is ample authority for the view that the declarant’s beliefin the imminence of his death can be
shown by the declarant’s own statements orfrom circumstantial evidence, such as the nature of his
wounds, statements madein his presence, or by the opinion of his physician. Bolanon would have
beencompetent to testify on the subject of the declaration had he survived. Lastly, thedying declaration
was offered in this criminal prosecution for murder in whichBolanon was the victim.

A declaration or an utterance is deemed as part of the res gestaeand thusadmissible in evidence as an


exception to the hearsay rule when the followingrequisites concur, to wit: (a) the principal act, the res
gestae, is a startlingoccurrence; (b) the statements are made before the declarant had time to contriveor
devise; and (c) the statements must concern the occurrence in question and itsimmediately attending
circumstances.

The requisites for admissibility of a declaration as part of the res gestaeconcur herein. Surely, when he
gave the identity of the assailant to Estaño, Bolanonwas referring to a startling occurrence, i.e., his
stabbing by Salafranca. Bolanon wasthen on board the taxicab that would bring him to the hospital, and
thus had notime to contrive his identification of Salafranca as the assailant. His utteranceabout Salafranca
having stabbed him was made in spontaneity and only in reactionto the startling occurrence. The
statement was relevant because it identifiedSalafranca as the perpetrator.
The term res gestaehas been defined as “those circumstances which are theundesigned incidents of a
particular litigated act and which are admissible whenillustrative of such act.” In a general way, res
gestaerefers to the circumstances,facts, and declarations that grow out of the main fact and serve to

Remedial Law Review II (CY 2018-2019) – Saturday 1:00pm – 5:00pm: Case Digest Compilation [Atty. Brondial] Page 313
illustrate itscharacter and are so spontaneous and contemporaneous with the main fact as toexclude the
idea of deliberation and fabrication. The rule on res gestaeencompasses the exclamations and statements
made by either the participants,victims, or spectators to a crime immediately before, during, or
immediately afterthe commission of the crime when the circumstances are such that the statementswere
made as a spontaneous reaction or utterance inspired by the excitement ofthe occasion and there was no
opportunity for the declarant to deliberate and tofabricate a false statement. The test of admissibility of
evidence as a part of the resgestaeis, therefore, whether the act, declaration, or exclamation is so
intimatelyinterwoven or connected with the principal fact or event that it characterizes as tobe regarded as
a part of the transaction itself, and also whether it clearly negativesany premeditation or purpose to
manufacture testimony.

DOCTRINE: An ante-mortem declaration of a victim of murder, homicide, or parricide thatmeets the


conditions of admissibility under the Rules of Court and pertinentjurisprudence is admissible either as a
dying declaration or as a part of the resgestae, or both.

Remedial Law Review II (CY 2018-2019) – Saturday 1:00pm – 5:00pm: Case Digest Compilation [Atty. Brondial] Page 314
A.5 SCC CHEMICALS CORPORATION, petitioner,  vs.THE HONORABLE COURT OF
APPEALS, STATE INVESTMENT HOUSE, INC., DANILO ARRIETA and LEOPOLDO
HALILI, respondent.

G.R. No. 128538;  February 28, 2001; QUISUMBING, J.:

FACTS: On December 13, 1983, SCC Chemicals Corporation (SCC for brevity) through its chairman,
private respondent Danilo Arrieta and vice president, Pablo (Pablito) Bermundo, obtained a loan from
State Investment House Inc., (hereinafter SIHI) in the amount of P129,824.48. The loan carried an annual
interest rate of 30% plus penalty charges of 2% per month on the remaining balance of the principal upon
non-payment on the due date-January 12, 1984. To secure the payment of the loan, Danilo Arrieta and
private respondent Leopoldo Halili executed a Comprehensive Surety Agreement binding themselves
jointly and severally to pay the obligation on the maturity date. SCC failed to pay the loan when it
matured. SIHI then sent demand letters to SCC, Arrieta and Halili, but notwithstanding receipt thereof, no
payment was made.

SIHI filed a Civil Case for a sum of money with a prayer for preliminary attachment against SCC,
Arrieta, and Halili with the Regional Trial Court of Manila.

In its answer, SCC asserted SIHI's lack of cause of action. Petitioner contended that the promissory note
upon which SIHI anchored its cause of action was null, void, and of no binding effect for lack or failure
of consideration.

SIHI presented one witness to prove its claim. The cross-examination of said witness was postponed
several times due to one reason or another at the instance of either party. The case was calendared several
times for hearing but each time, SCC or its counsel failed to appear despite notice. SCC was finally
declared by the trial court to have waived its right to cross-examine the witness of SIHI and the case was
deemed submitted for decision.

The lower court promulgated its decision in favor of SIHI.

On appeal, SCC contended that SIHI had failed to show, by a preponderance of evidence, that the latter
had a case against it. SCC argued that the lone witness presented by SIHI to prove its claim was
insufficient as the competency of the witness was not established and there was no showing that he had
personal knowledge of the transaction. SCC further maintained that no proof was shown of the
genuineness of the signatures in the documentary exhibits presented as evidence and that these signatures
were neither marked nor offered in evidence by SIHI. Finally, SCC pointed out that the original copies of
the documents were not presented in court. The appellate court affirmed in toto the judgment appealed
from. SCC filed its motion for reconsideration, which the Court of Appeals denied in its resolution.
Hence, petitioner's recourse to this Court.

ISSUE: Whether or not the Court of Appeals made an error of law in holding that private respondent
SIHI had proved its cause of action by preponderant evidence

RULING: Petitioner contends that SIHI introduced documentary evidence through the testimony of a
witness whose competence was not established and whose personal knowledge of the truthfulness of the
facts testified to was not demonstrated. It argues that the same was in violation of Sections 36 and
48, Rule 130 of the Rules of Court and it was manifest error for the Court of Appeals to have ruled
otherwise. In addition, SCC points out that the sole witness of SIHI did not profess to have seen the
document presented in evidence executed or written by SCC. Thus, no proof of its genuineness was
adduced. SIHI thus ran afoul of Section 2, Rule 132 of the Rules of Court, which requires proof of due
execution and authenticity of private documents before the same can be received as evidence. Petitioner
likewise submits that none of the signatures affixed in the documentary evidence presented by SIHI were
offered in evidence. It vehemently argues that such was in violation of the requirement of Section
34, Rule 132 of the Rules of Court. It was thus an error of law on the part of the appellate court to
consider the same. Finally, petitioner posits that the non-production of the originals of the documents
presented in evidence allows the presumption of suppression of evidence provided for in Section 3
(e), Rule 131 of the Rules of Court, to come into play.

Petitioner's arguments lack merit; they fail to persuade us.

Remedial Law Review II (CY 2018-2019) – Saturday 1:00pm – 5:00pm: Case Digest Compilation [Atty. Brondial] Page 315
We note that the Court of Appeals found that SCC failed to appear several times on scheduled hearing
dates despite due notice to it and counsel. On all those scheduled hearing dates, petitioner was supposed
to cross-examine the lone witness offered by SIHI to prove its case. Petitioner now charges the appellate
court with committing an error of law when it failed to disallow the admission in evidence of said
testimony pursuant to the "hearsay rule" contained in Section 36, Rule 130 of the Rules of Court.

Rule 130, Section 36 reads:


SEC. 36. Testimony generally confined to personal knowledge; hearsay excluded. – A witness can testify
only to those facts which he knows of his personal knowledge; that is, which are derived from his own
perception, except as otherwise provided in these rules.

Petitioner's reliance on Section 36, Rule 130 of the Rules of Court is misplaced. As a rule, hearsay
evidence is excluded and carries no probative value. However, the rule does admit of an exception. Where
a party failed to object to hearsay evidence, then the same is admissible. The rationale for this exception
is to be found in the right of a litigant to cross-examine. It is settled that it is the opportunity to cross-
examine which negates the claim that the matters testified to by a witness are hearsay.  However, the right
to cross-examine may be waived. The repeated failure of a party to cross-examine the witness is an
implied waiver of such right. Petitioner was afforded several opportunities by the trial court to cross-
examine the other party's witness. Petitioner repeatedly failed to take advantage of these opportunities. No
error was thus committed by the respondent court when it sustained the trial court's finding that petitioner
had waived its right to cross-examine the opposing party's witness. It is now too late for petitioner to be
raising this matter of hearsay evidence.

Nor was the assailed testimony hearsay. The Court of Appeals correctly found that the witness of SIHI
was a competent witness as he testified to facts, which he knew of his personal knowledge. Thus, the
requirements of Section 36, Rule 130 of the Rules of Court as to the admissibility of his testimony were
satisfied.

Respecting petitioner's other submissions, the same are moot and academic. As correctly found by the
Court of Appeals, petitioner's admission as to the execution of the promissory note by it through private
respondent Arrieta and Bermundo at pre-trial sufficed to settle the question of the genuineness of
signatures. The admission having been made in a stipulation of facts at pre-trial by the parties, it must be
treated as a judicial admission. Under Section, 4 Rule 129 of the Rules of Court, a judicial admission
requires no proof.

Nor will petitioner's reliance on the "best evidence rule" advance its cause. Respondent SIHI had no need
to present the original of the documents as there was already a judicial admission by petitioner at pre-trial
of the execution of the promissory note and receipt of the demand letter. It is now too late for petitioner to
be questioning their authenticity. Its admission of the existence of these documents was sufficient to
establish its obligation.

DOCTRINE: Petitioner's reliance on Section 36, Rule 130 of the Rules of Court is misplaced. As a rule,
hearsay evidence is excluded and carries no probative value. However, the rule does admit of an
exception. Where a party failed to object to hearsay evidence, then the same is admissible. The rationale
for this exception is to be found in the right of a litigant to cross-examine. It is settled that it is the
opportunity to cross-examine which negates the claim that the matters testified to by a witness are
hearsay. However, the right to cross-examine may be waived. The repeated failure of a party to cross-
examine the witness is an implied waiver of such right.

Remedial Law Review II (CY 2018-2019) – Saturday 1:00pm – 5:00pm: Case Digest Compilation [Atty. Brondial] Page 316
B. WHAT NEED NOT BE PROVED

B.1 LBP vs. Banal

FACTS:Spouses Vicente and Leonidas Banal, respondents, are the registered owners of 19.3422 hectares
of agricultural land situated in San Felipe, Basud, Camarines Norte covered by TCT No. T-6296. A
portion of the land consisting of 6.2330 hectares (5.4730 of which is planted to coconut and 0.7600
planted to palay) was compulsorily acquired by the Department of Agrarian Reform (DAR) pursuant to
R.A. No. 6657, as amended, otherwise known as the Comprehensive Agrarian Reform Law of 1988.

In accordance with the formula prescribed in DAR Administrative Order No. 6, Series of 1992, as
amended by DAR Administrative Order No. 11, Series of 1994, the Land Bank of the Philippines
(Landbank), petitioner valued the coconut land at P148,675.19, while the riceland was valued at
25,243.36, for a total of P173,918.55.

Respondents rejected the above valuation. Thus, pursuant to Section 16(d) of R.A. 6657, a summary
administrative proceeding was conducted before the Provincial Agrarian Reform Adjudicator (PARAD)
to determine the valuation of the land. The PARAD rendered its Decision affirming the Landbank's
valuation.

Respondents filed with the RTC of Daet, Camarines Norte, designated as a Special Agrarian Court, a
petition for determination of just compensation. Impleaded as respondents were the DAR and the
Landbank. The Spouses prayed for a compensation of P100,000.00 per hectare for both coconut land and
riceland, or an aggregate amount of P623,000.00.

In its Decision, the trial court computed the just compensation for the coconut land at  P657,137.00 and
for the riceland at P46,000.00, or a total of P703,137.00, which is beyond respondents' valuation
of P623,000.00. The court further awarded compounded interest at P79,732.00 in cash.

In determining the valuation of the land, the trial court based the same on the facts established in another
case pending before it (Civil Case No. 6679, "Luz Rodriguez vs. DAR, et al."), and used the formula in
the said case.

The Landbank filed with the Court of Appeals a petition for review. On March 20, 2000, the Appellate
Court rendered a Decision affirming in toto the judgment of the trial court. The Landbank's motion for
reconsideration was likewise denied.

ISSUE:Whether the trial court was correct in taking judicial notice of the production figures in the case
of Rodriguez vs. DAR, et al.

RULING: No. The RTC, in concluding that the valuation of respondents' property is P703,137.00,
merely took judicial notice of the average production figures in the Rodriguez case pending before
it and applied the same to this case without conducting a hearing and worse, without the knowledge or
consent of the parties.

Well-settled is the rule that courts are not authorized to take judicial notice of the contents of the records
of other cases even when said cases have been tried or are pending in the same court or before the same
judge. They may only do so "in the absence of objection" and "with the knowledge of the opposing
party," which are not obtaining here.

Furthermore, as earlier stated, the Rules of Court shall apply to all proceedings before the Special
Agrarian Courts. In this regard, Section 3, Rule 129 of the Revised Rules on Evidence is explicit on the
necessity of a hearing before a court takes judicial notice of a certain matter, thus:
"SEC. 3. Judicial notice, when hearing necessary. – During the trial, the court, on its own initiative, or on
request of a party, may announce its intention to take judicial notice of any matter and allow the
parties to be heard thereon.

"After the trial, and before judgment or on appeal, the proper court, on its own initiative or on request of a
party, may take judicial notice of any matter and allow the parties to be heard thereon if such matter is
decisive of a material issue in the case." (emphasis added)
The RTC failed to observe the above provisions.

Remedial Law Review II (CY 2018-2019) – Saturday 1:00pm – 5:00pm: Case Digest Compilation [Atty. Brondial] Page 317
While the determination of just compensation involves the exercise of judicial discretion, however, such
discretion must be discharged within the bounds of the law. Here, the RTC wantonly disregarded R.A.
6657, as amended, and its implementing rules and regulations. (DAR Administrative Order No. 6, as
amended by DAR Administrative Order No.11).

DOCTRINE: Well-settled is the rule that courts are not authorized to take judicial notice of the contents
of the records of other cases even when said cases have been tried or are pending in the same court or
before the same judge. They may only do so "in the absence of objection" and "with the knowledge of the
opposing party," which are not obtaining here.

Section 3, Rule 129 of the Revised Rules on Evidence is explicit on the necessity of a hearing before a
court takes judicial notice of a certain matter.

Remedial Law Review II (CY 2018-2019) – Saturday 1:00pm – 5:00pm: Case Digest Compilation [Atty. Brondial] Page 318
B.2 People of the Philippines vs. Jailon Kulais, Awalon Kamlon Hassan, Majid Samson, and et. al

G.R. No. 100901-08; July 16, 1998; J. Panganiban

FACTS: On December 12, 1988Felix Rosario a public officer in the City government of Zamboanga.
Felix was brought to the mountainous part of Zamboanga del Sur   was kidnapped by all the accuse, they
were conspiring and confederating together, mutually aiding and assisting one another, with threats to kill
his person. Felix was detained, held hostage and deprived of his liberty until February 2, 1989, the day
when he was released only after payment of the ransom was made to herein accused, to the damage and
prejudice of said victim; there being present an aggravating circumstance in that the aforecited offense
was committed with the aid of armed men or persons who insure or afford impunity. Thus on August 22,
1990 five information for kidnapping for ransom was charged against the accused. There were also, three
Informations for kidnapping, also under Article 267 of the Revised Penal Code, likewise alleged identical
facts and circumstances, except the names of the victims.

The prosecution alleged on December 12, 1988 a group of public officials from various government
agencies, organized themselves as a monitoring team to inspect government projects in Zamboanga City.
The group was composed of Virginia Gara, as the head of the team; Armando Bacarro, representing the
Commission on Audit; Felix del Rosario, representing the non-government; Edilberto Perez, representing
the City Assessors Office; Jessica Calunod and Allan Basa of the City Budget Office and Monico
Saavedra, the driver from the City Engineers Office. The group headed to the Lincomo Elementary
School to check on two of its classrooms. After inspecting the same, they proceeded to the Talaga
Footbridge. The group was not able to reach the place because on their way, they were stopped by nine
armed men who pointed their guns at them. They were then ordered to walk to the mountain by the leader
of the armed men who introduced himself as Commander Falcasantos. The kidnappers held their captives
for fifty-four days in the forest. During their captivity, the victims were able to recognize their captors
who were at all times armed with guns. The wives of the kidnappers performed the basic chores like
cooking. Commander Falcasantos also ordered their victims to sign the ransom notes which demanded a
ransom of P100.000.00 and P14,000.00 in exchange for twenty sets of uniform. After, the kidnappers
received the ransom money the hostage were released at around 12:00 o clock midnight of February 03,
1989.

The facts according to the defense that on May 28, 1990 Jumatiya Amlani while doing weeding their
farm, a group of soldiers picked her up and brought to a place where one army battalion was
stationed. Where she met the other accused inclusind Kulais. Amlani alleged that At the time Amlani was
picked up by the military, she had just escaped from the captivity of Carlos Falcasantos and company
who in 1988 kidnapped and brought her to the mountains. Against their will, she stayed with Falcasantos
and his two wives for two months.

The trial court held Kulais and et al. guilty of kidnapping. The seven accused positively identified to have
been present during the course of the captivity of the five kidnap-victims-complainants are: (1) Jumatiya
Amlani; (2) Jaliha Hussin; (3) Norma Sahiddan; (4) Jailon Kulais; (5) Hadjirul Plasin; (6) Salvador
Mamaril and (7) Jainuddin Hassan.

On appeal the accused alleged that the RTC erred in taking judicial notice of a material testimony given
in another case by Lt. Melquiades Feliciano, who allegedly was the team leader of the government troops
which allegedly captured the accused-appellants in an encounter; thereby, depriving the accused-
appellants their right to cross-examine him.

ISSUE:
1) Whether or not the RTC erred in taking judicial notice the testimony of Lt. Feliciano given in
another case but pending in the same trial court, does denied him of his righ to due process to cross-
examine the witness?
2) Whether or not the erronous taking of judicial notice of witness testimony ina nother case will
affect the conviction of the accused?

RULING:
6) YES, the RTC erred in taking judicial notice. as a general rule, courts should not take judicial
notice of the evidence presented in other proceedings, even if these have been tried or are pending in the
same court, or have been heard and are actually pending before the same judge. This is especially true in
criminal cases, where the accused has the constitutional right to confront and cross-examine the witnesses
against him.

Remedial Law Review II (CY 2018-2019) – Saturday 1:00pm – 5:00pm: Case Digest Compilation [Atty. Brondial] Page 319
However, that even if the court a quo did take judicial notice of the testimony of Lieutenant Feliciano, it
did not use such testimony in deciding the cases against the appellant. Hence, Appellant Kulais was not
denied due process. His conviction was based mainly on the positive identification made by some of
the kidnap victims, namely, Jessica Calunod, Armando Bacarro and Edilberto Perez. These witnesses
were subjected to meticulous cross-examinations conducted by appellants counsel. At best, then, the trial
courts mention of Lieutenant Felicianos testimony is a decisional surplusage which neither affected the
outcome of the case nor substantially prejudiced Appellant Kulais.

7) NO, The trial courts erroneous taking of judicial notice of a witness testimony in another case,
also pending before it, does not affect the conviction of the appellant, whose guilt is proven beyond
reasonable doubt by other clear, convincing and overwhelming evidence, both testimonial and
documentary.

DOCTRINE:No judicial notice of the evidence presented in other proceedings, even if these have been
tried or are pending in the same court, or have been heard and are actually pending before the same
judge.

Remedial Law Review II (CY 2018-2019) – Saturday 1:00pm – 5:00pm: Case Digest Compilation [Atty. Brondial] Page 320
B.3 MENANDRO B. LAUREANO vs. COURT OP APPEALS AND SINGAPORE AIRLINES
LIMITED

FACTS: Menandro B. Laureano was employed by Singapore Airlines. His employment contract was for
until 1984. In 1982, Singapore Airlines was hit by recession and therefore applied cost-cutting measures
by terminating excess personnel including Laureano.

Laureano initially filed an illegal dismissal case against the airline company before the Labor Arbiter in
1983 which was withdrawn. Laureano then filed a complaint before the RTC for damages due to illegal
termination of contract of services.

Singapore Airlines, in 1987, filed a motion to dismiss alleging inter alia: (1) that the court has no
jurisdiction over the subject matter of the case, and (2) that Philippine courts have no jurisdiction over the
instant case and that, since Laureano was employed in Singapore, all other aspects of his employment
contract and/or documents were executed in Singapore thus, postulating that Singapore laws should apply
and courts thereat shall have jurisdiction.

ISSUE: Whether the Singapore Laws should be applied in the present case.

RULING:No, the trial court rightly ruled on the application of Philippine law, thus:
“Neither can the Court determine whether the termination of the Laureano is legal under the Singapore
Laws because of the airline company’s failure to show which specific laws of Singapore Laws apply to
this case. As substantially discussed in the preceding paragraphs, the Philippine Courts do not take
judicial notice of the laws of Singapore. The defendant that claims the applicability of the Singapore
Laws to this case has the burden of proof. The defendant has failed to do so. Therefore, the Philippine law
should be applied.”

Remedial Law Review II (CY 2018-2019) – Saturday 1:00pm – 5:00pm: Case Digest Compilation [Atty. Brondial] Page 321
B.4 CASAN MACODE MAQUILING vs COMELEC, ROMMEL ARNADO, LINOG BALUA

G.R. Nos. 195649; 02 July 2013; Sereno, J.

FACTS: Rommel Arnado is a natural born Filipino citizen who lost his citizenship upon his
naturalization as a citizen of the United States. Subsequently, he availed of the benefits of RA 9225, the
Citizenship Retention and Re-acquisition Act of 2003 and ran as Mayor of Kauswagan, Lanao del Norte
in the 2010 local elections.

Linog C. Balua (Balua), another mayoralty candidate, filed a petition to disqualify Arnado, contending
that Arnado is a foreigner. It turned out that Arnado has been using his US Passport in entering and
departing the Philippines.

The Commission on Elections First Division (COMELEC) annulled the proclamation of Arnado and
consequently directed that that order of succession under Section 44 of the Local Government Code of
1991 be followed. It ruled that Arnado's act of consistently using his US passport after renouncing his US
citizenship negated his Affidavit of Renunciation.

Petitioner Casan Macode Maquiling (Maquiling), another candidate for mayor of Kauswagan, and who
garnered the second highest number of votes in the 2010 elections, intervened in the case. Maquiling
argued that while the First Division correctly disqualified Arnado, the order of succession under Section
44 of the Local Government Code is not applicable in this case. Maquiling claims that due to the
cancellation of Arnado's candidacy and the nullification of the latter's proclamation, he should be
proclaimed as the winner.

The Court ruled that Arnado is declared to be not a candidate at all in the May 2010 elections. This leaves
Maquiling as the qualified candidate who obtained the highest number of votes. Therefore, Maquiling is
declared the duly elected Mayor of Kauswagan, Lanao del Norte.

A Motion for Reconsideration was filed by respondent on May 10, 2013 and the Supplemental Motion for
Reconsideration filed on May 20, 2013.
Arnado cites Section 349 of the Immigration and Naturalization Act of the United States as having the
effect of expatriation when he executed his Affidavit of Renunciation of American Citizenship on April 3,
2009 and thus claims that he was divested of his American citizenship.

ISSUES: Whether or not respondent was divested of all the rights of an American citizen?

RULING: No. The fact that he was still able to use his US passport after executing his Affidavit of
Renunciation repudiates this claim.

The Court cannot take judicial notice of foreign laws,which must be presented as public documentsof a
foreign country and must be "evidenced by an official publication thereof."Mere reference to a foreign
law in a pleading does not suffice for it to be considered in deciding a case.

What is an OFFICAL RECORD?


Rule 132, Sec. 24: OFFICIAL RECORD – ….Official publication or by a copy attested by the officer
having legal custody of the record.

American law does not govern in this jurisdiction. Instead, Section 40(d) of the Local Government Code
calls for application in the case before us, given the fact that at the time Arnado filed his certificate of
candidacy, he was not only a Filipino citizen but, by his own declaration, also an American citizen. It is
the application of this law and not of any foreign law that serves as the basis for Arnado’s disqualification
to run for any local elective position.

Remedial Law Review II (CY 2018-2019) – Saturday 1:00pm – 5:00pm: Case Digest Compilation [Atty. Brondial] Page 322
B.5 PEOPLE OF THE PHILIPPINES, Plaintiff-Appellee, vs. KHADDAFY JANJALANI, GAMAL
B. BAHARAN a.k.a. Tapay, ANGELO TRINIDAD a.k.a. Abu Khalil, GAPPAL BANNAH ASALI
a.k.a. Maidan or Negro, JAINAL SALI a.k.a. Abu Solaiman, ROHMAT ABDURROHIM a.k.a.
Jackie or Zaky, and other JOHN and JANE DOES, Accused, GAMAL B. BAHARAN a.k.a. Tapay,
ANGELO TRINIDAD a.k.a. Abu Khalil, and ROHMAT ABDURROHIM a.k.a. Abu Jackie or
Zaky, Accused-Appellants.

G.R. No. 188314; January 10, 2011; SERENO, J.:

FACTS: On Valentines Day of 2005, accused Baharan and Trinidad boarded an RRCG bus. According to
Elmer Andales, the bus conductor, he immediately became wary of the two men, because, even if they got
on the bus together, the two sat away from each other – one sat two seats behind the driver, while the
other sat at the back of the bus. Afterwards, Andales said he became more suspicious because both men
kept on asking him if the bus was going to stop at Ayala Avenue. The two men insisted to alight at Ayala
Avenue and moments after, the bus exploded killing and wounding some passengers.

The prosecution presented documents confirming that shortly before the explosion, the spokesperson of
the Abu Sayyaf Group – Abu Solaiman – announced over radio station DZBB that the group had a
Valentine’s Day "gift" for former President Macapagal-Arroyo. After the bombing, he again went on
radio and warned of more bomb attacks. As stipulated during pretrial, accused Trinidad gave ABS-CBN
News an exclusive interview some time after the incident, confessing his participation in the Valentine’s
Day bombing incident while Baharan, likewise, admitted his role in the bombing incident on a separate
interview. Finally, accused Asali gave a television interview, confessing that he had supplied the
explosive devices for the 14 February 2005 bombing. The bus conductor identified the accused Baharan
and Trinidad, and confirmed that they were the two men who had entered the RRCG bus.

Members of the Abu Sayyaf Group – namely Khaddafy Janjalani, Gamal B. Baharan, Angelo Trinidad,
Gappal Bannah Asali, Jainal Asali, Rohmat Abdurrohim a.k.a. Abu Jackie or Zaky, and other "John" and
"Jane Does" – were then charged with multiple murder and multiple frustrated murder. Only Baharan,
Trinidad, Asali, and Rohmat were arrested, while the other accused remain at-large.

On their arraignment for the multiple murder charge, Baharan, Trinidad, and Asali all entered a plea of
guilty. On the other hand, upon arraignment for the multiple frustrated murder charge, accused Asali pled
guilty. Accused Trinidad and Baharan pled not guilty. Rohmat pled not guilty to both charges.

After being discharged as state witness, accused Asali testified that the night before the Valentine’s Day
bombing, Trinidad and Baharan got another two kilos of TNT (bomb) from him. Late in the evening of 14
February, he received a call from Abu Solaiman. The latter told Asali not to leave home or go to crowded
areas, since the TNT taken by Baharan and Trinidad had already been exploded in Makati. Thirty minutes
later, Trinidad called Asali, repeating the warning of Abu Solaiman. The next day, Asali allegedly
received a call from accused Rohmat, congratulating the former on the success of the mission. 3 According
to Asali, Abu Zaky specifically said, "Sa wakas nag success din yung tinuro ko sayo."

During the trial, the trial court judge asked the defense counsel to clarify the conflicting pleas of the
accused because Trinidad and Baharan pleaded guilty to multiple murder but not guilty to frustrate
murder charges. Said accused, after conferring with counsel, pleaded guilty to the frustrated murder
charges.

ISSUE:Whether the trial court gravely erred in accepting accused-appellants’ plea of guilt despite
insufficiency of searching inquiry into the voluntariness and full comprehension of the consequences of
the said plea.

RULING: No.
Section 3, Rule 116 of the Rules of Court:
SEC. 3. Plea of guilty to capital offense; reception of evidence. — When the accused pleads guilty to a
capital offense, the court shall conduct a searching inquiry into the voluntariness and full comprehension
of the consequences of his plea and shall require the prosecution to prove his guilt and the precise degree
of culpability. The accused may also present evidence in his behalf. (Emphasis supplied)

We have reiterated in a long line of cases that the conduct of a searching inquiry remains the duty of
judges, as they are mandated by the rules to satisfy themselves that the accused had not been under

Remedial Law Review II (CY 2018-2019) – Saturday 1:00pm – 5:00pm: Case Digest Compilation [Atty. Brondial] Page 323
coercion or duress; mistaken impressions; or a misunderstanding of the significance, effects, and
consequences of their guilty plea.10 This requirement is stringent and mandatory.11
Nevertheless, we are not unmindful of the context under which the re-arraignment was conducted or of
the factual milieu surrounding the finding of guilt against the accused. The Court observes that accused
Baharan and Trinidad previously pled guilty to another charge – multiple murder – based on the same act
relied upon in the multiple frustrated murder charge. The Court further notes that prior to the change of
plea to one of guilt, accused Baharan and Trinidad made two other confessions of guilt – one through an
extrajudicial confession (exclusive television interviews, as stipulated by both accused during pretrial),
and the other via judicial admission (pretrial stipulation). Considering the foregoing circumstances, we
deem it unnecessary to rule on the sufficiency of the "searching inquiry" in this instance. Remanding the
case for re-arraignment is not warranted, as the accused’s plea of guilt was not the sole basis of the
condemnatory judgment under consideration.12

Convictions based on an improvident plea of guilt are set aside only if such plea is the sole basis of the
judgment. If the trial court relied on sufficient and credible evidence to convict the accused, the
conviction must be sustained, because then it is predicated not merely on the guilty plea of the accused
but on evidence proving his commission of the offense charged. 14
Accused contend that the testimony of Asali is inadmissible pursuant to Sec. 30, Rule 130 of the Rules of
Court. It is true that under the rule, statements made by a conspirator against a co-conspirator are
admissible only when made during the existence of the conspiracy. However, as the Court ruled in People
v. Buntag, if the declarant repeats the statement in court, his extrajudicial confession becomes a judicial
admission, making the testimony admissible as to both conspirators. 27 Thus, in People v. Palijon, the
Court held the following:

… [W]e must make a distinction between extrajudicial and judicial confessions. An extrajudicial
confession may be given in evidence against the confessant but not against his co-accused as they are
deprived of the opportunity to cross-examine him. A judicial confession is admissible against the
declarant’s co-accused since the latter are afforded opportunity to cross-examine the former. Section 30,
Rule 130 of the Rules of Court applies only to extrajudicial acts or admissions and not to testimony at
trial where the party adversely affected has the opportunity to cross-examine the declarant. Mercene’s
admission implicating his co-accused was given on the witness stand. It is admissible in evidence against
appellant Palijon. Moreover, where several accused are tried together for the same offense, the testimony
of a co-accused implicating his co-accused is competent evidence against the latter. 28

Doctrine: An extrajudicial confession may be given in evidence against the confessant but not against his
co-accused as they are deprived of the opportunity to cross-examine him. A judicial confession is
admissible against the declarant’s co-accused since the latter are afforded opportunity to cross-examine
the former. Section 30, Rule 130 of the Rules of Court applies only to extrajudicial acts or admissions
and not to testimony at trial where the party adversely affected has the opportunity to cross-examine the
declarant.

Remedial Law Review II (CY 2018-2019) – Saturday 1:00pm – 5:00pm: Case Digest Compilation [Atty. Brondial] Page 324
B.6 Republic v Sandiganbayan

FACTS:In 1993, SB ordered the consolidation cases related to the recovery of the ill-gotten wealth of
Marcos Family and cronies. At the trial of Civil Case No. 0009, the petitioner filed a Motion stating that
petitioner wishes to adopt in Civil Case No. 0009 testimonies and the documentary exhibits presented and
identified by them in another related case. This motion partly denied insofar as the adoption of
testimonies on oral deposition of Maurice V. Bane and Rolando Gapud for the reason that said deponents
are not available for cross-examination in this Court by the respondents. Petitioner then filed another
motion asking SB to take judicial notice of the facts established by the Bane deposition, together with the
marked exhibits appended thereto. This was again denied by the SB, to wit: Judicial notice is found under
Rule 129. This provision refers to the Court’s duty to consider admissions made by the parties in the
pleadings, or in the course of the trial or other proceedings in resolving cases before it.

The duty of the Court is mandatory and in those cases where it is discretionary, the initiative is upon the
Court. Such being the case, the Court finds the Urgent Motion and/or Request for Judicial Notice as
something which need not be acted upon as the same is considered redundant. Petitioner’s 3rd motion was
again denied by SB. Hence, petitioner filed the instant motion alleging grave abuse of discretion on the
part of SB’s refusal to take judicial notice of or to admit the Bane deposition as part of its evidence.
Petitioner asserts that the case where the Bane deposition was originally taken, introduced and admitted in
evidence is but a "child" of the "parent" case, Civil Case No. 0009; under this relationship, evidence
offered and admitted in any of the "children" cases should be considered as evidence in the "parent" case.

ISSUE: Whether or not courts in trying consolidated cases may take judicial notice of testimony and
evidence presented in one of the cases consolidated.

RULING: In adjudicating a case on trial, generally, courts are not authorized to take judicial notice of the
contents of the records of other cases, even when such cases have been tried or are pending in the same
court, and notwithstanding that both cases may have been tried or are actually pending before the same
judge. This rule though admits of exceptions. As a matter of convenience to all the parties, a court may
properly treat all or any part of the original record of a case filed in its archives as read into the record of a
case pending before it, when, with the knowledge of, and absent an objection from, the adverse party,
reference is made to it for that purpose, by name and number or in some other manner by which it is
sufficiently designated; or when the original record of the former case or any part of it, is actually
withdrawn from the archives at the court's direction, at the request or with the consent of the parties, and
admitted as a part of the record of the case then pending. Courts must also take judicial notice of the
records of another case or cases, where sufficient basis exists in the records of the case before it,
warranting the dismissal of the latter case. The petitioner itself admits that the present case has generated
a lot of cases, which, in all likelihood, involve issues of varying complexity. If we follow the logic of the
petitioner’s argument, we would be espousing judicial confusion by indiscriminately allowing the
admission of evidence in one case, which was presumably found competent and relevant in another case,
simply based on the supposed lineage of the cases.

Remedial Law Review II (CY 2018-2019) – Saturday 1:00pm – 5:00pm: Case Digest Compilation [Atty. Brondial] Page 325
B.7 MONICO LIGTAS, v. PEOPLE OF THE PHILIPPINES

G.R. No. 200751, August 17, 2015; LEONEN, J.:

FACTS: The uncontested declaration of the Department of Agrarian Reform Adjudication Board that
Monico Ligtas was a tenant negates a finding of theft beyond reasonable doubt. Tenants having rights to
the harvest cannot be deemed to have taken their own produce.

Ligtas was charged with the crime of theft under Article 308 of the Revised Penal Code.

Meanwhile, Ligtas filed a Complaint before the Department of Agrarian Reform Adjudication Board
(DARAB) of Sogod, Southern Leyte for Maintenance of Peaceful Possession. The DARAB rendered the
Decision ruling that Ligtas was a bona fide tenant of the land.

The Court of Appeals declared that Ligtas' reliance on the DARAB Decision "declaring him as a bonafide
tenant of the . . . land is irrelevant in the case at bar."

ISSUE: Whether or not DARAB’s Decision ruling that Ligtas was a bona fide tenant of the land negates
criminal liability for theft.

RULING: Yes. The Petition is meritorious.


Findings of fact of administrative agencies in the exercise of their quasi-judicial powers are entitled to
respect if supported by substantial evidence. This court is not tasked to weigh again "the evidence
submitted before the administrative body and to substitute its own judgment [as to] the sufficiency of
evidence."
XXXX

In VHJ Construction and Development Corporation v. Court of Appeals, this court ruled that tenancy
relationship must be duly proven:

[A] tenancy relationship cannot be presumed. There must be evidence to prove this allegation. The
principal factor in determining whether a tenancy relationship exists is intent. Tenancy is not a purely
factual relationship dependent on what the alleged tenant does upon the land. It is also a legal
relationship. (Citation omitted)

XXXX

According to petitioner, the elements of theft under Article 308 of the Revised Penal Code were not
established since he was a bona fide tenant of the land. The DARAB's recognition of petitioner as a
legitimate tenant necessarily "implie[d] that he ha[d] the authority to harvest the abaca hemp from
[private complainant's land]." This shows that petitioner had no criminal intent.

As to the existence of another element of theft—that the taking was done without the consent of the
owner—petitioner argues that this, too, was negated by his status as private complainant's tenant:

The purported lack of consent on the part of the private complainant as alleged by the prosecution, is
misplaced. In fact, it was even improper for Anecita Pacate to stop or prevent petitioner from harvesting
the produce of the landholding because as tenant, petitioner is entitled to security of tenure. This right
entitled him to continue working on his landholding until the leasehold relation is terminated or until his
eviction is authorized by the DARAB in a judgment that is final and executory. 135 (Citation omitted)

Tenants have been defined as:

persons who — in themselves and with the aid available from within their immediate farm households —
cultivate the land belonging to or possessed by another, with the latter's consent, for purposes of
production, sharing the produce with the landholder under the share tenancy system, or paying to the
landholder a price certain or ascertainable in produce or money or both under the leasehold tenancy
system.138 (Citation omitted)

Under this definition, a tenant is entitled to the products of the land he or she cultivates. The landowner's
share in the produce depends on the agreement between the parties. Hence, the harvesting done by the
tenant is with the landowner's consent.

Remedial Law Review II (CY 2018-2019) – Saturday 1:00pm – 5:00pm: Case Digest Compilation [Atty. Brondial] Page 326
The existence of the DARAB Decision adjudicating the issue of tenancy between petitioner and private
complainant negates the existence of the element that the taking was done without the owner's consent.
The DARAB Decision implies that petitioner had legitimate authority to harvest the abaca. The
prosecution, therefore, failed to establish all the elements of theft.

In this case, petitioner harvested the abaca, believing that he was entitled to the produce as a legitimate
tenant cultivating the land owned by private complainant. Personal property may have been taken, but it
is with the consent of the owner.

No less than the Constitution provides that the accused shall be presumed innocent of the crime until
proven guilty. "[I]t is better to acquit ten guilty individuals than to convict one innocent person."Thus,
courts must consider "[e]very circumstance against guilt and in favor of innocence[.]"Equally settled is
that "[w]here the evidence admits of two interpretations, one of which is consistent with guilt, and the
other with innocence, the accused must be given the benefit of doubt and should be acquitted."

Remedial Law Review II (CY 2018-2019) – Saturday 1:00pm – 5:00pm: Case Digest Compilation [Atty. Brondial] Page 327
C. RULES OF ADMISSIBILITY

C.1.1 People vs. Mallilin

G.R. No. 172953; April 30, 2008; Tinga, J.:

FACTS: On the strength of a warrant of search and seizure a team of five police officers raided the
residence of petitioner in Sorsogon City on 4 February 2003. DelfinLicup as well as petitioner himself,
his wife Sheila and his mother, Norma allegedly yielded two (2) plastic sachets of shabuand five (5)
empty plastic sachets containing residual morsels of the said substance, the prosecution presented
Bolanos, Arroyo and Esternon as witnesses.

Taking the witness stand, Bolanos, the leader of the raiding team, testified on the circumstances
surrounding the search as follows: that he and his men were allowed entry into the house by petitioner
after the latter was shown the search warrant; that upon entering the premises, he ordered Esternon and
barangay kagawadLicup, whose assistance had previously been requested in executing the warrant, to
conduct the search. Esternon testified that the denim bag containing the empty plastic sachets was found
behind the door of the bedroom and not inside the cabinet; that he then found the two filled sachets under
a pillow on the bed and forthwith called on Gallinera to have the items recorded and marked.

Petitioner‘s contention was that during search he was asked by a police officer to buy cigarettes at a
nearby store and when he returned from the errand. Petitioner asserted that on his return from the errand,
he was summoned by Esternon to the bedroom and once inside, the officer closed the door and asked him
to lift the mattress on the bed. And as he was doing as told, Esternon stopped him and ordered him to lift
the portion of the headboard. In that instant, Esternon showed him sachet of shabu which according to
him came from a pillow on the bed. Supt. Lorlie Arroyo (Arroyo), the forensic chemist who administered
the examination on the seized items. She admitted that all seven sachets were delivered to the laboratory
by Esternon in the afternoon of the same day that the warrant was executed except that it was not she but
rather a certain Mrs. Ofelia Garcia who received the items from Esternon at the laboratory.

On 20 June 2004 the trial court rendered its Decision declaring petitioner guilty beyond reasonable doubt
of the offense charged.
Aggrieved, petitioner filed a Notice of Appeal. In his Appeal Brief filed with the Court of Appeals,
petitioner called the attention of the court to certain irregularities in the manner by which the search of his
house was conducted. The Court of Appeals rendered the assailed decision affirming the judgment of the
trial court but modifying the prison sentence.

ISSUE: whether or not the search and seizure was conducted in a regular manner.

RULING: No

Prosecutions for illegal possession of prohibited drugs necessitates that the elemental act of possession of
a prohibited substance be established with moral certainty, together with the fact that the same is not
authorized by law. The dangerous drug itself constitutes the very corpus delictiof the offense and the fact
of its existence is vital to a judgment of conviction. Essential therefore in these cases is that the identity of
the prohibited drug be established beyond doubt. Be that as it may, the mere fact of unauthorized
possession will not suffice to create in a reasonable mind the moral certainty required to sustain a finding
of guilt.

The chain of custody requirement performs this function in that it ensures that unnecessary doubts
concerning the identity of the evidence are removed. As a method of authenticating evidence, the chain of
custody rule requires that the admission of an exhibit be preceded by evidence sufficient to support a
finding that the matter in question is what the proponent claims it to be. It would include testimony about
every link in the chain, from the moment the item was picked up to the time it is offered into evidence, in
such a way that every person who touched the exhibit would describe how and from whom it was
received, where it was and what happened to it while in the witness possession, the condition in which it
was received and the condition in which it was delivered to the next link in the chain. These witnesses
would then describe the precautions taken to ensure that there had been no change in the condition of the
item and no opportunity for someone not in the chain to have possession of the same.

While testimony about a perfect chain is not always the standard because it is almost always impossible to
obtain, an unbroken chain of custody becomes indispensable and essential when the item of real evidence

Remedial Law Review II (CY 2018-2019) – Saturday 1:00pm – 5:00pm: Case Digest Compilation [Atty. Brondial] Page 328
is not distinctive and is not readily identifiable, or when its condition at the time of testing or trial is
critical, or when a witness has failed to observe its uniqueness. The same standard likewise obtains in
case the evidence is susceptible to alteration, tampering, contamination and even substitution and
exchange. In other words, the exhibits level of susceptibility to fungibility, alteration or tamperingwithout
regard to whether the same is advertent or otherwise notdictates the level of strictness in the application of
the chain of custody rule.

It ruled that unless the state can show by records or testimony, the continuous whereabouts of the exhibit
at least between the time it came into the possession of police officers until it was tested in the laboratory
to determine its composition, testimony of the state as to the laboratory findings is inadmissible.
The prosecution was thus unsuccessful in discharging its burden of establishing the identity of the seized
items because it failed to offer not only the testimony of Gallinera and Garcia but also any sufficient
explanation for such failure. In effect, there is no reasonable guaranty as to the integrity of the exhibits
inasmuch as it failed to rule out the possibility of substitution of the exhibits, which cannot but inure to its
own detriment. This holds true not only with respect to the two filled sachets but also to the five sachets
allegedly containing morsels of shabu.

Moreover, Section 21 of the Implementing Rules and Regulations of R.A. No. 9165 clearly outlines the
post-seizure procedure in taking custody of seized drugs. In a language too plain to require a different
construction, it mandates that the officer acquiring initial custody of drugs under a search warrant must
conduct the photographing and the physical inventory of the item at the place where the warrant has been
served. Esternon deviated from this procedure. It was elicited from him that at the close of the search of
petitioners house, he brought the seized items immediately to the police station for the alleged purpose of
making a true inventory thereof, but there appears to be no reason why a true inventory could not be made
in petitioners house when in fact the apprehending team was able to record and mark the seized items and
there and then prepare a seizure receipt therefor.

Given the foregoing deviations of police officer Esternon from the standard and normal procedure in the
implementation of the warrant and in taking post-seizure custody of the evidence, the blind reliance by
the trial court and the Court of Appeals on the presumption of regularity in the conduct of police duty is
manifestly misplaced. The presumption of regularity is merely just that a mere presumption disputable by
contrary proof and which when challenged by the evidence cannot be regarded as binding truth. Suffice it
to say that this presumption cannot preponderate over the presumption of innocence that prevails if not
overthrown by proof beyond reasonable doubt. In the present case the lack of conclusive identification of
the illegal drugs allegedly seized from petitioner, coupled with the irregularity in the manner by which the
same were placed under police custody before offered in court, strongly militates a finding of guilt.
In our constitutional system, basic and elementary is the presupposition that the burden of proving the
guilt of an accused lies on the prosecution which must rely on the strength of its own evidence and not on
the weakness of the defense. The rule is invariable whatever may be the reputation of the accused, for the
law presumes his innocence unless and until the contrary is shown. In dubio pro reo.When moral
certainty as to culpability hangs in the balance, acquittal on reasonable doubt inevitably becomes a matter
of right.

Remedial Law Review II (CY 2018-2019) – Saturday 1:00pm – 5:00pm: Case Digest Compilation [Atty. Brondial] Page 329
C.1.2 People vs. Pagaduan

GR No 179029; 12 August 2010; BRION, J.:

FACTS: Buy-bust operation was conducted by PO3 Almarez, SPO1 Balido and Captain de Vera. Ruper
Pagaduan was arrested and plastic sachet of what appears to be shabu was marked, request for laboratory
examination was done the same day. The plastic sachet was turned over to PNP Crime Laboratory two
days after. He was found guilty by the court and the same was affirmed by CA

The appellant contends that the prosecution failed to show an unbroken chain of custody in the handling
of the seized drug. He claims that there was no evidence to show when the markings were done.
Moreover, a period of two days had elapsed from the time the shabu was confiscated to the time it was
forwarded to the crime laboratory for examination.

The Office of the Solicitor General (OSG) counters with the argument that the chain of custody of the
shabu was sufficiently established. It explained that the shabu was turned over by the police officers to
the PNP Crime Laboratory, where it was found by the forensic chemist to be positive for the presence of
shabu. The OSG likewise claimed that the appellant failed to rebut the presumption of regularity in the
performance of official duties by the police. The OSG further added that a prior surveillance is not
indispensable to a prosecution for illegal sale of drugs.

ISSUE: WON there was a valid chain of custody that can be admitted as Real or Object Evidence.

RULING: None. The SC resolve to acquit the appellant for the prosecutions failure to prove his guilt
beyond reasonable doubt. Specifically, the prosecution failed to show that the police complied with
paragraph 1, Section 21, Article II of R.A. No. 9165, and with the chain of custody requirement of this
Act.

ILLEGAL SALE OF DRUGS UNDER SECTION 5 VIS--VIS THE INVENTORY AND


PHOTOGRAPH
In a prosecution for illegal sale of a prohibited drug under Section 5 of R.A. No. 9165, the prosecution
must prove the following elements: (1) the identity of the buyer and the seller, the object, and the
consideration; and (2) the delivery of the thing sold and the payment therefor.

The required procedure on the seizure and custody of drugs is implemented by Section 21(a), Article II of
the Implementing Rules and Regulations of R.A. No. 9165, which reads: (a) The apprehending
officer/team having initial custody and control of the drugs shall, immediately after seizure and
confiscation, physically inventory and photograph the same in the presence of the accused or the
person/s from whom such items were confiscated and/or seized, or his/her representative or counsel, a
representative from the media and the Department of Justice (DOJ), and any elected public official
who shall be required to sign the copies of the inventory and be given a copy thereof: Provided, that the
physical inventory and photograph shall be conducted at the place where the search warrant is served;
or at the nearest police station or at the nearest office of the apprehending officer/team, whichever is
practicable, in case of warrantless seizures; Provided, further, that non-compliance with these
requirements under justifiable grounds, as long as the integrity and the evidentiary value of the seized
items are properly preserved by the apprehending officer/team, shall not render void and invalid such
seizures of and custody over said items.

Strict compliance with the prescribed procedure is required because of the illegal drug's unique
characteristic rendering it indistinct, not readily identifiable, and easily open to tampering, alteration or
substitution either by accident or otherwise.[28] The records of the present case are bereft of evidence
showing that the buy-bust team followed the outlined procedure despite its mandatory terms. It is evident
that the apprehending team, upon confiscation of the drug, immediately brought the appellant and the
seized items to the police station, and, once there, made the request for laboratory examination. No
physical inventory and photograph of the seized items were taken in the presence of the accused or his
counsel, a representative from the media and the Department of Justice, and an elective official. PO3
Almarez, on cross-examination, was unsure and could not give a categorical answer when asked whether
he issued a receipt for the shabu confiscated from the appellant. At any rate, no such receipt or certificate
of inventory appears in the records.

Noncompliance with the strict directive of Section 21 of R.A. No. 9165 is not necessarily fatal to the
prosecutions case; police procedures in the handling of confiscated evidence may still have some lapses,

Remedial Law Review II (CY 2018-2019) – Saturday 1:00pm – 5:00pm: Case Digest Compilation [Atty. Brondial] Page 330
as in the present case. These lapses, however, must be recognized and explained in terms of their
justifiable grounds, and the integrity and evidentiary value of the evidence seized must be shown to have
been preserved.

In the present case, the prosecution did not bother to offer any explanation to justify the failure of the
police to conduct the required physical inventory and photograph of the seized drugs. The apprehending
team failed to show why an inventory and photograph of the seized evidence had not been made either in
the place of seizure and arrest or at the nearest police station (as required by the Implementing Rules in
case of warrantless arrests). We emphasize that for the saving clause to apply, it is important that the
prosecution explain the reasons behind the procedural lapses, and that the integrity and value of the seized
evidence had been preserved.[40] In other words, the justifiable ground for noncompliance must be
proven as a fact. The court cannot presume what these grounds are or that they even exist.

THE CHAIN OF CUSTODY REQUIREMENT


Section 1(b) of Dangerous Drugs Board Regulation No. 1, Series of 2002 which implements R.A. No.
9165 defines chain of custody as follows: Chain of Custody means the duly recorded authorized
movements and custody of seized drugs or controlled chemicals or plant sources of dangerous drugs or
laboratory equipment of each stage, from the time of seizure/confiscation to receipt in the forensic
laboratory to safekeeping to presentation in court for destruction. Such record of movements and
custody of seized item shall include the identity and signature of the person who held temporary
custody of the seized item, the date and time when such transfer of custody were made in the course of
safekeeping and use in court as evidence, and the final disposition.

In the present case, the prosecutions evidence failed to establish the chain that would have shown that the
shabu presented in court was the very same specimen seized from the appellant.

The first link in the chain of custody starts with the seizure of the heat-sealed plastic sachet from the
appellant. PO3 Almarez mentioned on cross-examination that he placed his initials on the confiscated
sachet after apprehending the appellant. Notably, this testimony constituted the totality of the
prosecutions evidence on the marking of the seized evidence. PO3 Almarezs testimony, however, lacked
specifics on how he marked the sachet and who witnessed the marking. In People v. Sanchez, we ruled
that the marking of the seized items to truly ensure that they are the same items that enter the chain and
are eventually the ones offered in evidence should be done (1) in the presence of the apprehended violator
(2) immediately upon confiscation. In the present case, nothing in the records gives us an insight on the
manner and circumstances that attended the marking of the confiscated sachet. Whether the marking had
been done in the presence of the appellant is not at all clear from the evidence that merely mentioned that
the evidence had been marked after the appellants apprehension.

The second link in the chain of custody is its turnover from the apprehending team to the police station.
PO3 Almarez testified that the appellant was brought to the Diadi Police Station after his arrest. However,
he failed to identify the person who had control and possession of the seized drug at the time of its
transportation to the police station. In the absence of clear evidence, we cannot presume that PO3
Almarez, as the poseur buyer, handled the seized sachet to the exclusion of others - during its transfer
from the place of arrest and confiscation to the police station. The prosecution likewise failed to present
evidence pertaining to the identity of the duty desk officer who received the plastic sachet containing
shabu from the buy-bust team. This is particularly significant since the seized specimen was turned over
to the PNP Crime Laboratory only after two days. It was not, therefore, clear who had temporary custody
of the seized items during this significant intervening period of time. Although the records show that the
request for laboratory examination of the seized plastic sachet was prepared by Captain de Vera, the
evidence does not show that he was the official who received the marked plastic sachet from the buy-bust
team.

As for the subsequent links in the chain of custody, the records show that the seized specimen was
forwarded by PO3 Almarez to the PNP Crime Laboratory on December 29, 2003, where it was received
by PO2 Dulnuan, and later examined by PSI Quintero. However, the person from whom PO3 Almarez
received the seized illegal drug for transfer to the crime laboratory was not identified. As earlier
discussed, the identity of the duty desk officer who received the shabu, as well as the person who had
temporary custody of the seized items for two days, had not been established.

The procedural lapses mentioned above show the glaring gaps in the chain of custody, creating a
reasonable doubt whether the drugs confiscated from the appellant were the same drugs that were brought
to the crime laboratory for chemical analysis, and eventually offered in court as evidence. In the absence

Remedial Law Review II (CY 2018-2019) – Saturday 1:00pm – 5:00pm: Case Digest Compilation [Atty. Brondial] Page 331
of concrete evidence on the illegal drugs bought and sold, the body of the crime the corpus delicti has not
been adequately proven. In effect, the prosecution failed to fully prove the elements of the crime charged,
creating reasonable doubt on the appellants criminal liability.

PRESUMPTION OF REGULARITY IN THE PERFORMANCE OF OFFICIAL DUTIES


In sustaining the appellants conviction, the CA relied on the evidentiary presumption that official duties
have been regularly performed. This presumption, it must be emphasized, is not conclusive.[45] It cannot,
by itself, overcome the constitutional presumption of innocence. Any taint of irregularity affects the
whole performance and should make the presumption unavailable. In the present case, the failure of the
apprehending team to comply with paragraph 1, Section 21, Article II of R.A. No. 9165, and with the
chain of custody requirement of this Act effectively negates this presumption.

Remedial Law Review II (CY 2018-2019) – Saturday 1:00pm – 5:00pm: Case Digest Compilation [Atty. Brondial] Page 332
C.1.3 NARCISO SALAS v. ANNABELLE MATUSALEM,

GR No. 180284, September 11, 2013

FACTS:Annabelle Matusalem (respondent) filed a complaint for Support/Damages against Narciso Salas
(petitioner) in the Regional Trial Court.

Respondent claimed that petitioner is the father of her son Christian Paulo Salas who was born on
December 28, 1994. Petitioner, already 56 years old at the time, enticed her as she was then only 24 years
old, making her believe that he is a widower. Petitioner rented an... apartment where respondent stayed
and shouldered all expenses in the delivery of their child, including the cost of caesarian operation and
hospital confinement. However, when respondent refused the offer of petitioner's family to take the child
from her, petitioner abandoned... respondent and her child and left them to the mercy of relatives and
friends. Respondent thus prayed for support pendente lite and monthly support

Petitioner filed his answer with special and affirmative defenses and counterclaims. He described
respondent as a woman of loose morals

Petitioner... denied paternity of the child Christian Paulo; he was motivated by no other reason except
genuine altruism when he agreed to shoulder the expenses for the delivery of said child, unaware of
respondent's chicanery and deceit designed to "scandalize" him in exchange for financial... favor.

Respondent testified that she first met petitioner at the house of his "kumadre" Felicisima de Guzman.

During their subsequent meeting, petitioner told her he is already a widower and he has no more
companion in life because his children... are all grown-up. She also learned that petitioner owns a rice
mill, a construction business and a housing subdivision

Petitioner at the time already knows that she is a single mother as she had a child... by her former
boyfriend in Italy. He then brought her to a motel, promising that he will take care of her and marry her.
She believed him and yielded to his advances, with the thought that she and her child will have a better
life. Thereafter, they saw each other weekly and... petitioner gave her money for her child. When she
became pregnant with petitioner's child, it was only then she learned that he is in fact not a widower. She
wanted to abort the baby but petitioner opposed it because he wanted to have another child.

On the fourth month of her pregnancy, petitioner rented an apartment where she stayed with a housemaid;
he also provided for all their expenses. She gave birth to their child on December 28, 1994.

Before delivery, petitioner even... walked her at the hospital room and massaged her stomach, saying he
had not done this to his wife. She filled out the form for the child's birth certificate and wrote all the
information supplied by petitioner himself. It was also petitioner who paid the hospital bills and drove...
her baby home. He was excited and happy to have a son at his advanced age who is his "look-alike," and
this was witnessed by other boarders, visitors and Grace Murillo, the owner of the apartment unit
petitioner rented. However, on the 18th day after the... baby's birth, petitioner went to Baguio City for a
medical check-up. He confessed to her daughter and eventually his wife was also informed about his
having sired an illegitimate child. His family then decided to adopt the baby and just give respondent
money so she can go... abroad. When she refused this offer, petitioner stopped seeing her and sending
money to her. She and her baby survived through the help of relatives and friends.

Murillo corroborated respondent's testimony as to the payment by petitioner of apartment rental, his
weekly visits to respondent and financial support to her, his presence during and after delivery of
respondent's baby, respondent's attempted suicide through sleeping pills... overdose and hospitalization
for which she paid the bill, her complaint before the police authorities and meeting with petitioner's wife
at the headquarters.

On April 5, 1999, the trial court rendered its decision in favor of respondent... the CA dismissed
petitioner's appeal.

On the paternity issue, the CA affirmed the trial court's ruling that respondent satisfactorily established
the illegitimate filiation of her son Christian Paulo, and consequently no error was committed by the trial
court in granting respondent's prayer for support.

Remedial Law Review II (CY 2018-2019) – Saturday 1:00pm – 5:00pm: Case Digest Compilation [Atty. Brondial] Page 333
ISSUES:whether the trial and appellate courts erred in ruling that respondent's evidence sufficiently
proved that her son Christian Paulo is the illegitimate child of petitioner.

RULING: We grant the petition.

Under Article 175 of the Family Code of the Philippines, illegitimate filiation may be established in the
same way and on the same evidence as legitimate children.
Article 172 of the Family Code of the Philippines states:

The filiation of legitimate children is established by any of the following:


(1) The record of birth appearing in the civil register or a final judgment; or
(2) An admission of legitimate filiation in a public document or a private handwritten instrument and
signed by the parent concerned.

In the absence of the foregoing evidence, the legitimate filiation shall be proved by:
(1) The open and continuous possession of the status of a legitimate child; or
(2) Any other means allowed by the Rules of Court and special laws.

Respondent presented the Certificate of Live Birth. (Exhibit "A-1") of Christian Paulo Salas in which the
name of petitioner appears as his father but which is not signed by him. Admittedly, it was only
respondent who filled up the entries and signed the... said document though she claims it was petitioner
who supplied the information she wrote therein.

We have held that a certificate of live birth purportedly identifying the putative father is not competent
evidence of paternity when there is no showing that the putative father had a hand in the preparation of
the certificate. Thus, if the father did... not sign in the birth certificate, the placing of his name by the
mother, doctor, registrar, or other person is incompetent evidence of paternity. Neither can such birth
certificate be taken as a recognition in a public instrument and it has no probative value to establish
filiation to the alleged father

As to the Baptismal Certificate (Exhibit "B") of Christian Paulo Salas also indicating petitioner as the
father, we have ruled that while baptismal certificates may be considered public documents, they can only
serve as evidence of the administration of... the sacraments on the dates so specified. They are not
necessarily competent evidence of the veracity of entries therein with respect to the child's paternity.

The rest of respondent's documentary evidence consists of handwritten notes and letters, hospital bill and
photographs taken of petitioner and respondent inside their rented apartment unit.
Pictures taken of the mother and her child together with the alleged father are inconclusive evidence to
prove paternity… Exhibits "E" and "F" showing petitioner and respondent inside the rented apartment
unit thus have scant... evidentiary value. The Statement of Account (Exhibit "C") from the Good
Samaritan General Hospital where respondent herself was indicated as the payee is likewise incompetent
to prove that petitioner is the father of her child notwithstanding... petitioner's admission in his answer
that he shouldered the expenses in the delivery of respondent's child as an act of charity.

As to the handwritten notes (Exhibits "D" to "D-13") of petitioner and respondent showing their exchange
of affectionate words and romantic trysts, these, too, are not sufficient to establish Christian Paulo's
filiation to petitioner as they were not... signed by petitioner and contained no statement of admission by
petitioner that he is the father of said child. Thus, even if these notes were authentic, they do not qualify
under Article 172 (2) vis-à- vis Article 175 of the Family Code which admits as competent evidence of...
illegitimate filiation an admission of filiation in a private handwritten instrument signed by the parent
concerned.

An illegitimate child is now also allowed to establish his claimed filiation by "any other means allowed
by the Rules of Court and special laws," like his baptismal certificate, a judicial admission, a family Bible
in which his name has been entered, common reputation respecting... his pedigree, admission by silence,
the testimonies of witnesses, and other kinds of proof admissible under Rule 130 of the Rules of Court.
[38] Reviewing the records, we find the totality of respondent's evidence insufficient to establish that...
petitioner is the father of Christian Paulo.

The testimonies of respondent and Murillo as to the circumstances of the birth of Christian Paulo,
petitioner's financial support while respondent lived in Murillo's apartment and his regular visits to her at

Remedial Law Review II (CY 2018-2019) – Saturday 1:00pm – 5:00pm: Case Digest Compilation [Atty. Brondial] Page 334
the said apartment, though replete with details, do not approximate... the "overwhelming evidence,
documentary and testimonial" presented in Ilano.

In sum, we hold that the testimonies of respondent and Murillo, by themselves are not competent proof of
paternity and the totality of respondent's evidence failed to establish Christian Paulo's filiation to
petitioner.

Time and again, this Court has ruled that a high standard of proof is required to establish paternity and
filiation. An order for recognition and support may create an unwholesome situation or may be an irritant
to the family or the lives of the parties so that it must be issued... only if paternity or filiation is
established by clear and convincing evidence.

Remedial Law Review II (CY 2018-2019) – Saturday 1:00pm – 5:00pm: Case Digest Compilation [Atty. Brondial] Page 335
C.1.4 People vs. Eric Rosauro

G.R. No. 209588; February 18, 2015, 751 SCRA 204; PEREZ, J.

FACTS: In an Amended Information dated 21 February 2005, Eric Rosauro y Bongcawil (accused-
appellant) was charged with violation of Sec. 5, Art. II of R.A. No. 9165, to wit:

That on the 3rd day of July, 2004 at about 5:30 o'clock in the afternoon, more or less, at Purok 3,
Barangay Poblacion, Municipality of Villanueva, Province of Misamis Oriental, Republic of the
Philippines, and within the jurisdiction of this Honorable Court, the above-named accused, not
being authorized by law to possess and to sell any dangerous drugs, knowingly, willfully and
feloniously, did then and there, sell and convey to a third person, who acted as a decoy in a buy
bust operation, one (1) sachet of shabu, containing 0.04 grams (sic) of shabu, which when
examined gave POSITIVE result to test for the presence of Methamphetamine Hydrochloride
(Shabu), a dangerous drug.

Upon re-arraignment, accused-appellant pleaded not guilty to the crime charged. Thereafter, pre-trial and
trial on the merits ensued. Accused-appellant claims that he was merely a victim of instigation.

Finding the evidence of the prosecution sufficient to establish the guilt of accused-appellant, the Regional
Trial Court (RTC), Cagayan de Oro City, Branch 25, finding accused-appellant guilty beyond reasonable
doubt of illegal sale of shabu under Sec. 5, Article II of Republic Act No. 9165 (R.A. No. 9165) or the
Comprehensive Dangerous Drugs Act of 2002, sentencing him to suffer the penalty of life imprisonment
and ordering him to pay a fine of P500,000.00.

Accused-appellant appealed before the CA that the RTC committed grave error in convicting him.

The CA affirmed the RTC Judgment. The appellate court ruled that what transpired in the case at bar was
an entrapment and not an instigation; that all the elements of illegal sale of regulated or prohibited drugs
were duly proven; that the non-presentation of the confidential agent in court is not fatal; that the
inconsistencies in the testimony of the lone witness of the prosecution do not affect the result of the case;
and that the apprehending team was able to preserve the integrity of the subject drug and that the
prosecution was able to present the required unbroken chain in the custody of the subject drug. Hence
this appeal.

ISSUE: Whether or not the RTC and CA erred in convicting accused-appellant.

RULING: No. Appeal is DISMISSED. The RTC and the CA both found the arrest of accused-appellant
to be the result of a legitimate entrapment procedure, and we find nothing in the records as to warrant a
contrary finding.

The solicitation of drugs from appellant by the informant utilized by the police merely furnishes evidence
of a course of conduct. The police received an intelligence report that appellant has been habitually
dealing in illegal drugs. They duly acted on it by utilizing an informant to effect a drug transaction with
appellant. There was no showing that the informant induced the appellant to sell illegal drugs to him.

Similarly, the presentation of an informant as witness is not regarded as indispensable to the success of a
prosecution of a drug-dealing accused.

As to whether accused-appellant's guilt was established beyond reasonable doubt, we rule in the
affirmative.

In a catena of cases, this Court laid down the essential elements to be duly established for a successful
prosecution of offenses involving the illegal sale of dangerous or prohibited drugs, like shabu, under
Section 5, Article II of R.A. No. 9165, to wit: (1) the identity of the buyer and the seller, the object of the
sale, and the consideration; and (2) the delivery of the thing sold and payment therefor. Briefly, the
delivery of the illicit drug to the poseur-buyer and the receipt of the marked money by the seller
successfully consummate the buy-bust transaction. What is material, therefore, is the proof that the
transaction or sale transpired, coupled with the presentation in court of the corpus delicti.

Verily, all the elements for a conviction of illegal sale of dangerous or prohibited drugs were proven by
the prosecution: the identity of accused-appellant as the seller, and that of the confidential informant as

Remedial Law Review II (CY 2018-2019) – Saturday 1:00pm – 5:00pm: Case Digest Compilation [Atty. Brondial] Page 336
poseur-buyer were established, as well as the exchange of the sachet of shabu and the marked money. It
was also ascertained that the seized item was positive for shabu, a dangerous drug, and that the same item
was properly identified in open court by SPO4 Larot. Moreover, the P100.00 bill with serial number
YZ712579, or the subject marked money, as well as the living body of the accused-appellant revealed a
positive result for ultraviolet fluorescent powder.

Accused-appellant avers that the prosecution was not able to prove the corpus delicti, and that the
statutory safeguards provided for in Sec. 21 of R.A. No. 9165 were not followed.

Indeed, as we held in People v. Torres, equally important in every prosecution for illegal sale of
dangerous or prohibited drugs is the presentation of evidence of the seized drug as the corpus delicti. The
identity of the prohibited drug must be proved with moral certainty. It must also be established with the
same degree of certitude that the substance bought or seized during the buy-bust operation is the same
item offered in court as exhibit. In this regard, paragraph 1, Section 21, Article II of R.A. No. 9165 (the
chain of custody rule) provides for safeguards for the protection of the identity and integrity of dangerous
drugs seized.

However, this Court has, in many cases, held that while the chain of custody should ideally be perfect, in
reality it is "almost always impossible to obtain an unbroken chain." The most important factor is the
preservation of the integrity and the evidentiary value of the seized items as they will be used to
determine the guilt or innocence of the accused. Hence, the prosecution's failure to submit in evidence the
physical inventory and photograph of the seized drugs as required under Article 21 of R.A. No. 9165, will
not render the accused's arrest illegal or the items seized from him inadmissible.

The chain of custody is not established solely by compliance with the prescribed physical inventory and
photographing of the seized drugs in the presence of the enumerated persons. The Implementing Rules
and Regulations of R.A. No. 9165 on the handling and disposition of seized dangerous drugs states:
. . . Provided, further, that non-compliance with these requirements under justifiable grounds, as
long as the integrity and evidentiary value of the seized items are properly preserved by the
apprehending officer/team, shall not render void and invalid such seizures of and custody over
said items. 22 (Italics, emphasis, undescoring omitted)

In the case at bar, after the sale was consummated, the confidential informant gave the seized item to
SPO4 Larot who placed tape on the sachet and marked it "Exhibit A." Upon reaching the police station,
SPO4 Larot executed the Certificate of Inventory, as well as the request for laboratory examination. The
request, the specimen, as well as the marked money and accused-appellant were then brought to the PNP
Crime Laboratory for examination. They were received by SPO2 Ricardo Maisog, the Receiving Clerk of
the PNP Crime Laboratory Office, who then forwarded them to Police Inspector Ma. Leocy Jabonillo
Mag-abo, the Forensic Chemical Officer of the PNP Crime Laboratory. Moreover, the seized item was
duly identified by SPO4 Larot in open cou

Remedial Law Review II (CY 2018-2019) – Saturday 1:00pm – 5:00pm: Case Digest Compilation [Atty. Brondial] Page 337
C.1.5 PEOPLE OF THE PHILIPPINES, Plaintiff-Appellee, vs. MEDARIO CALANTIAO y
DIMALANTA, Accused-Appellant.

LEONARDO-DE CASTRO, J.

FACTS: Edwin Lojera asked for police assistance regarding a shooting incident that happened while he
was driving a towing truck and traversing along EDSA. A traffic dispute occurred between him and a
white taxi cab prompting him to follow said vehicle until reaching 8th Avenue corner C3-road of
Caloocan. One of the accused, Calantiao who was a passenger of the said taxi alighted and fired a gun.
Lojera who could not do anything continued to drive until reaching the police station nearby.

PO1 Nelson Mariano and PO3 Eduardo Ramirez were on duty that day. Mariano stated that responding
from the complaint, they proceeded to find the white taxi wherein while they were approaching, two men
alighted and fired 3 shots towards them and runaway. The police chased after them and they were
subdued. Mariano recovered a black bag from Calantiao containing 2 bricks of dried marijuana fruiting
tops and magazine of super 38 stainless with ammos.

The suspects and pieces of evidence were turned over to SPO3 Pablo Temena wherein Mariano marked
the bricks with his initials NM. The specimen was then forwarded to the Crime Laboratory for chemical
analysis. The result of the chemical analysis by P/SINSP. JESSE DELA ROSA was that the specimen is
positive for marijuana.

Mariano's testimony was corroborated by Ramirez and the taxi driver, Crisendo Amansec.
Aside from oral testimony, documentary evidences were also offered such as Request for Laboratory
Examination, Physical Sciences Report Exh., Picture of First brick of marijuana fruiting tops, Picture of
2nd brick of marijuana fruiting tops, Pinagsamang Sinumpaang Salaysay of PO3 Eduardo Ramirez and
PO1 Nelson Mariano, Their respective signatures and Sinumpaang Salaysay of Crisendo Amansec
(Erroneously marked as Exh. "E").

RTC ruled that Calantiao was guilty beyond reasonable doubt of illegal possession of 997.9 grams of
marijuana fruiting tops. RTC held that the illegal drug seized was admissible in evidence as it was
discovered during a body search after Calantiao was caught in flagrante delicto of possessing a gun and
firing at the police officers

ISSUE: Whether or not the chain of custody is established?

RULING: YES. Calantiao claims that even if the search and seizure were validly effected, the
marijuana is still inadmissible as evidence against him for failure of the apprehending officers to comply
with the rules on chain of custody, as the item was marked at the police station is untenable.

This Court has held that the failure to strictly comply with Section 21, Article II of Republic Act No.
9165, such as immediately marking seized drugs, will not automatically impair the integrity of chain of
custody because what is of utmost importance is the preservation of the integrity and the evidentiary
value of the seized items, as these would be utilized in the determination of the guilt or innocence of the
accused.

Section 21 and its IRR do not even mention "marking." What they require are (1) physical inventory, and
(2) taking of photographs.

What Section 21 of R.A. No. 9165 and its implementing rule do not expressly specify is the matter of
"marking" of the seized items in warrantless seizures to ensure that the evidence seized upon
apprehension is the same evidence subjected to inventory and photography when these activities are
undertaken at the police station rather than at the place of arrest. Consistency with the "chain of custody"
rule requires that the "marking" of the seized items – to truly ensure that they are the same items that enter
the chain and are eventually the ones offered in evidence – should be done (1) in the presence of the
apprehended violator (2) immediately upon confiscation.

The prosecution was able to establish the chain of custody of the seized marijuana from the time the
police officers confiscated it, to the time it was turned over to the investigating officer, up to the time it
was brought to the forensic chemist for laboratory examination. This Court has no reason to overrule the
RTC and the Court of Appeals, which both found the chain of custody of the seized drugs to have not
been broken so as to render the marijuana seized from Calantiao inadmissible in evidence.

Remedial Law Review II (CY 2018-2019) – Saturday 1:00pm – 5:00pm: Case Digest Compilation [Atty. Brondial] Page 338
Furthermore, unless it can be shown that there was bad faith, ill will, or tampering of the evidence, the
presumption that the integrity of the evidence has been preserved will remain. The burden of showing the
foregoing to overcome the presumption that the police officers handled the seized drugs with regularity,
and that they properly discharged their duties is on Calantiao. Unfortunately, Calantiao failed to discharge
such burden

DOCTRINE: This Court has held that the failure to strictly comply with Section 21, Article II of
Republic Act No. 9165, such as immediately marking seized drugs, will not automatically impair the
integrity of chain of custody because what is of utmost importance is the preservation of the integrity and
the evidentiary value of the seized items, as these would be utilized in the determination of the guilt or
innocence of the accused.
Section 21 and its IRR do not even mention "marking." What they require are (1) physical inventory, and
(2) taking of photographs.

Remedial Law Review II (CY 2018-2019) – Saturday 1:00pm – 5:00pm: Case Digest Compilation [Atty. Brondial] Page 339
C.1.6 PEOPLE OF THE PHILIPPINES vs. HERMANOS CONSTANTINO, JR. y BINAYUG,
a.k.a. “JOJIT,"

G.R. No. 199689; March 12, 2014

FACTS:On January 20, 2005, at around 2:00 in the afternoon, Police Superintendent Mariano Rodriguez,
the Chief of Police of Tuguegarao City, received a report from a confidential informant (CI) that a certain
Jojit was selling illegal drugs in the said city. P/Supt. Rodriguez immediately formed a buy-bust group.
The buy-bust money, consisting of one ₱500.00 bill and five ₱100.00 bills, were dusted with fluorescent
powder and their respective serial numbers were recorded in the police blotter.

Around 8:00 in the evening of the same day, the team proceeded to Reynovilla St., Caritan Centro,
Tuguegarao City, the place where, according to the CI, Jojit was selling shabu. Thereafter, Constantino
was brought to the police station where the recovered drugs and money were turned over to the
investigator, SPO2 Tamang. The recovered drugs were then marked with the initials "A-1" and "A-2."
The incident was recorded in the police blotter with an inventory of the recovered drugs and money.
Later that evening, at around ten o’clock, P/Supt. Rodriguez and SPO2 Tamang submitted to the
Philippine National Police (PNP) Crime Laboratory Services, Camp Marcelo Adduru, Tuguegarao City, a
request for laboratory examination of two plastic sachets with white crystalline substance marked as "A-
1" and "A-2" to determine the presence of dangerous drugs; as well as both hands of Constantino, one
piece ₱500.00 bill, and five pieces ₱100.00 bills, to determine the presence of the ultra violet powder. Per
Chemistry Report No. D-08-200511 and Physical Identification Report No. PI-04-2005,12 prepared by
Police Senior Inspector (P/ SInsp.) Mayra Matote Madria,Forensic Chemist, the contents of the two
plastic sachets tested positive for Methamphetamine Hydrochloride; while the other specimens tested
positive for the presence of bright-yellow ultraviolet fluorescent powder.

Constantino denied the accusation against him and asserted that he was merely framed-up.
The following day, January 21, 2005, the police officers again brought Constantino to the PNP Crime
Laboratory. Along the way, one of the police escorts forced Constantino to hold a certain amount of
money. Constantino tried to resist but he could not really do anything because he was handcuffed. After
his examination, Constantino was detained and was told that he was suspected of selling shabu.
RTC found Constantino guilty as charged. It was affirmed by the Court of Appeals.
Constantino contests his conviction, averring inconsistencies in the testimonies of the prosecution
witnesses, particularly, on the circumstances of the marking of the two plastic sachets containing shabu
allegedly confiscated from him. Different people claim to have made the marking "NBT" on the two
plastic sachets and gave various explanations as to what the initials "NBT" stand for.

ISSUE: W/N the prosecution failed to establish a crucial link in the chain of custody of the shabu.

RULING: YES. The following links must be established in the chain of custody in a buy-bust situation:
first, the seizure and marking, if practicable, of the illegal drug recovered from the accused by the
apprehending officer; second, the turn over of the illegal drug seized by the apprehending officer to the
investigating officer; third, the turn over by the investigating officer of the illegal drug to the forensic
chemist for laboratory examination; and fourth, the turn over and submission of the marked illegal drugs
seized from the forensic chemist to the court. As held in People vs. Zakaria, crucial in proving the chain
of custody is the marking of the seized dangerous drugs or other related items immediately after they are
seized from the accused, for the marking upon seizure is the starting point in the custodial link that
succeeding handlers of the evidence will use as reference point. Moreover, the value of marking of the
evidence is to separate the marked evidence from the corpus of all other similar or related evidence from
the time of seizure from the accused until disposition at the end of criminal proceedings, obviating
switching, “planting” or contamination of evidence. A failure to mark at the time of taking of initial
custody imperils the integrity of the chain of custody that the law requires.

The failure of the prosecution to establish the evidence’s chain of custody is fatal to its case as the Court
can no longer consider or even safely assume that the integrity and evidentiary value of the confiscated
dangerous drug were properly preserved. In light of the foregoing, Constantino is acquitted of the crime
charged, not because the Court accords credence to his defense of frame-up, but because the prosecution
failed to discharge its burden of proving his guilt beyond reasonable doubt.

Remedial Law Review II (CY 2018-2019) – Saturday 1:00pm – 5:00pm: Case Digest Compilation [Atty. Brondial] Page 340
C.1.7PEOPLE OF THE PHILIPPINES vs. MERCURY DELA CRUZ ALIAS “DEDAY"

G.R. No. 212171, September 07, 2016

FACTS: At around 7:15 o'clock in the evening of November 10, 2006, PO3 Batobalonos, PO1 Reales,
PO1 Bullido andtheir civilian asset proceeded to Sitio Cogon, A. Lopez St., Barangay Labangon. When
the team went inside the interior portion of Sitio Cogon, PO1 Reales together with the civilian asset
approached the house of Dela Cruz, while PO3 Batobalonos and PO1 Bullido were strategically hidden
more or less ten (10) meters away. The civilian asset called Dela Cruz and told her that they will buy
shabu worth P200.00. Thereafter, Dela Cruz handed PO1 Reales a small plastic containing white
crystalline substance and in exchange he handed to the former the P200.00 bills. Upon getting hold of the
money, PO3 Batobalonos and PO1 Bullido, who saw the consummation of the transaction rushed to the
scene. When PO3 Batobalonos got hold of Dela Cruz, the latter shouted for help and resisted arrest. Dela
Cruz was able to run and so the team chased her, however, her neighbor Arthur Tabasa Ortega ("Ortega")
blocked their way. The team introduced themselves as policemen but Ortega did not listen, so PO3
Batobalonos fired a warning shot as the people likewise started to gather around them. Meanwhile, Dela
Cruz was able to evade arrest. The team then arrested Ortega for obstruction of justice.
On their way to the police station aboard their patrol car, PO1 Reales handed to PO3 Batobalonos the
small plastic containing white crystalline substance which he purchased from Dela Cruz. Thereafter, upon
arrival at the police station, PO3 Batobalonos marked the seized item with "DDM 11/10/06."
Afterwards, a Request for Laboratory Examination of the seized item was prepared by PO3 Batobalonos.
The Request and the seized item were delivered to the Regional Crime Laboratory Office-7, Camp Sotero
Cabahug, Gorordo Avenue, Cebu City by PO1 Reales at around 1:10 o'clock in the morning of November
11, 2006.

Thereafter Forensic Chemist PCI Salinas issued Chemistry Report No. D-1771-2006," with the finding
that the specimen gave positive result for the presence of Methamphetamine hydrochloride.
Regional Trial Court (RTC), Branch 58, Cebu City, found the accused-appellant guilty of illegal sale of
shabu under Sections 5, Article II of Republic Act (R.A.) No. 91653 and sentenced him to suffer the
penalty life imprisonment and to pay a fine of P500,000.00.

ISSUE: W/N appellant’s case should be dismissed due to failure of the police officers to comply with the
procedure in the custody and disposition of seized drugs (chain of custody).

RULING: We agree with the lower courts that in the absence of any intent or ill motive on the part of the
police officers to falsely impute commission of a crime against the accused-appellant, the presumption of
regularity in the performance of official duty is entitled to great respect and deserves to prevail over the
bare, uncorroborated denial and self-serving claim of the accused of frame-up. Also, we reject the
appellant’s contention that the police officers failed to comply with the provisions of Section 21,
paragraph 1 of R.A. No. 9165, which provides for the procedure in the custody and disposition of seized
drugs.

Although ideally the prosecution should offer a perfect chain of custody in the handling of evidence,
“substantial compliance with the legal requirements on the handling of the seized item” is sufficient. This
Court has consistently ruled that even if the arresting officers failed to strictly comply with the
requirements under Section 21 of R.A. No. 9165, such procedural lapse is not fatal and will not render the
items seized inadmissible in evidence. What is of utmost importance is the preservation of the integrity
and evidentiary value of the seized items, as the same would be utilized in the determination of the guilt
or innocence of the accused. In other words, to be admissible in evidence, the prosecution must be able to
present through records or testimony, the whereabouts of the dangerous drugs from the time these were
seized from the accused by the arresting officers; turned over to the investigating officer; forwarded to the
laboratory for determination of their composition; and up to the time these are offered in evidence. For as
long as the chain of custody remains unbroken, as in this case, even though the procedural requirements
provided for in Sec. 21 of R.A. No. 9165 were not faithfully observed, the guilt of the accused will not be
affected.

The integrity of the evidence is presumed to have been preserved unless there is a showing of bad faith, ill
will, or proof that the evidence has been tampered with. Accused-appellant bears the burden of showing
that the evidence was tampered or meddled with in order to overcome the presumption of regularity in the
handling of exhibits by public officers and the presumption that public officers properly discharged their
duties. Accused appellant in this case failed to present any plausible reason to impute ill motive on the
part of the arresting officers. Thus, the testimonies of the apprehending officers deserve full faith and

Remedial Law Review II (CY 2018-2019) – Saturday 1:00pm – 5:00pm: Case Digest Compilation [Atty. Brondial] Page 341
credit. In fact, accused-appellant did not even question the credibility of the prosecution witnesses. She
simply anchored her defense on denial and alibi.

Remedial Law Review II (CY 2018-2019) – Saturday 1:00pm – 5:00pm: Case Digest Compilation [Atty. Brondial] Page 342
C.1.8 PEOPLE OF THE PHILIPPINES vs MANUEL DELA ROSA Y LUMANOG

G.R. No. 230228; DECEMBER 13, 2017; GESMUNDO, J.:

FACTS: On March 28, 2009, at around 9:00 o’clock in the morning, a confidential informant reported to
PCI Marijane Ojasttro of PDEA IV-B Office of Calapan City, that accused-appellant, Manuel Dela Rosa,
was selling marijuana at White Beach, Puerto Galera, Oriental Mindoro. Based on the said informant, PCI
Ojastro directed the conduct of the buy-bust operation against accused-appellant.

Arriving at Puerto Galera, informant introduced IOI Briguel to accused-appellant.IOI Briguel then asked
the tattoo artist, accused-appellant, to put henna tattoo on his right shoulder. As accused-appellant was
doing the tattoo, IOI Briguel asked him; “Manny, pwede bang umiskor?” to which he replied:
“Meron”.IOI Briguel told accused-appellant that he was going to buy Php 300.00 worth of drugs, and
handed the marked money to accused-appellant, who, in turn, handed IOI Briguel the folded dried banana
leaves containing suspected dried marijuana leaves. Thus, IOI Briguel made the pre-arranged signal of
removing the handkerchief wrapped around his head. Immediately, IOI Jabano and IOI Echavaria arrived
and arrested accused-appellant. IOI Briguel frisked him and the marked money was recovered from him.
Subsequently, the accused-appellant was boarded into the service vehicle of the PDEA to avoid any
commotion at the shop. While inside the vehicle, IOI Briguel marked the seized marijuana with his
initials and the date of the arrest. He then testified that he placed the suspected dried marijuana leaves in
his pocket. The team then proceeded back to the PDEA IV-B Officce at Calapan City, which was 54 km
away from Puerto Galera. There, IOI Briguel conducted the inventory which was witnessed by the Brgy.
Chairperson Anacleto Vergara and media representative Dennis Nebrejo. Photographs were likewise
taken during the marking and inventory of the seized item. Then, the suspected marijuana was brought to
the Laboratory for examination and sample was tested positive for marijuana. The defense on the other
hand stated a different version of the story.

The RTC ruled that the accused appellant was guilty beyond reasonable doubt for the crime of violation
of Section 5, Article II of RA No. 9165. The RTC held that the prosecution was able to prove the identity
of the buyer, the seller, the object and consideration in the illegal sale of marijuana. It also held that the
delivery of drugs and payment thereof during the buy-bust operation was duly established. The RTC
further ruled that it was reasonable for PDEA to conduct the inventory of the seized item at their office in
Calapan, Mindoro to prevent a commotion at the place of arrest.

Aggrieved, accused-appellant appealed before the CA arguing that the integrity and evidentiary value of
the confiscated item. However, in its decision, CA dismissed the appeal and held that the RTC correctly
ruled that all the elements of the crime were duly proven.
Hence, this appeal

ISSUE: Whether or not the chain of custody rule was sufficiently complied with.

RULING: The court finds that the prosecution failed to sufficiently comply with the chain of custody
rule.

Chain of custody means duly recorded authorized movements and custody of seized drugs or controlled
chemicals or plant sources of dangerous drugs or controlled chemicals or plant sources of dangerous
drugs or laboratory equipment of each stage, from the time of seizure/confiscation to receipt in the
forensic laboratory to the safekeeping to presentation in court for destruction.

As the means of ensuring the establishment of the chain of custody, Section 21 (1) of RA No. 9165
signifies that:

The apprehending team having initial custody and control of the drugs shall, immediately after seizure
and confiscation, physically inventory and photograph the same in the presence of the accused of person/s
from whom such items were confiscated and/or seized, or his/her representative or counsel, a
representative from the media and the Department of Justice, and any elected public official who shall be
required to sign the copies of the inventory and be given a copy thereof.

In addition, Section 21 of the IRR of RA No. 9165 provides that the physical inventory and photograph
shall be conducted at the place where the search warrant is served; or at the nearest police station r nearest
office of the apprehending officer/team, whichever is practicable, in case of warrantless seizures. It

Remedial Law Review II (CY 2018-2019) – Saturday 1:00pm – 5:00pm: Case Digest Compilation [Atty. Brondial] Page 343
further states that non-compliance with these requirements shall not render void and invalid such seizures
and custody over the confiscated items provided that such non-compliance were under justifiable grounds
and the integrity and evidentiary value of the seized items are properly preserved by the apprehending
officer or team.

The apprehending team did not comply with Section 21 of RA No. 9165 and it’s IRR. The records show
that the physical inventory of the confiscated drug and photographs of the same where only done in the
presence of the accused-appellant, Brgy. Captain Vergara and media representative Nebrejo. Clearly, a
representative of the DOJ as required was not present during the inventory of the seized item. More
importantly, the apprehending team did not immediately conduct the physical inventory and taking of the
photographs at the time the suspected drug was confiscated or at the nearest police station. Instead, they
travelled 54km from Puerto Galera, the place of seizure, to Calapan City before they conducted the
inventory of the seized drug.

Remedial Law Review II (CY 2018-2019) – Saturday 1:00pm – 5:00pm: Case Digest Compilation [Atty. Brondial] Page 344
C.2.1 Sps. Silos vs. PNB

FACTS: Petitioners secured a revolving credit line of P150,000 from PNB which was increased to
P1.5million then subsequently increased to P2.5 million. These were secured by real estate mortgage of
petitioner’s properties and in addition, petitioners issued eight Promissory Notes8 and signed a Credit
Agreement. This July 1989 Credit Agreement contained a stipulation on interest which provides as
follows:

1.03. Interest. (a) The Loan shall be subject to interest at the rate of 19.5% per annum. Interest shall be
payable in advance every one hundred twenty days at the rate prevailing at the time of the renewal.

(b) The Borrower agrees that the Bank may modify the interest rate in the Loan depending on whatever
policy the Bank may adopt in the future, including without limitation, the shifting from the floating
interest rate system to the fixed interest rate system, or vice versa. Where the Bank has imposed on the
Loan interest at a rate per annum, which is equal to the Bank’s spread over the current floating interest
rate, the Borrower hereby agrees that the Bank may, without need of notice to the Borrower, increase or
decrease its spread over the floating interest rate at any time depending on whatever policy it may adopt
in the future. (Emphases supplied)

The eight Promissory Notes, on the other hand, contained a stipulation granting PNB the right to increase
or reduce interest rates "within the limits allowed by law or by the Monetary Board."
The Real Estate Mortgage agreement provided the same right to increase or reduce interest rates "at any
time depending on whatever policy PNB may adopt in the future."

Petitioners claim that interest rates imposed by it are null and void for the reasons that 1) the Credit
Agreements and the promissory notes were signed in blank; 2) interest rates were at short periods; 3) no
interest rates could be charged where no agreement on interest rates was made in writing; 4) PNB fixed
interest rates on the basis of arbitrary policies and standards left to its choosing; and 5) interest rates based
on prime rate plus applicable spread are indeterminate and arbitrary

Respondent argued that this issue (signed in blank) was never raised in the lower court and besides,
documentary evidence prevails over testimonial evidence; Lydia Silos’ testimony in this regard is self-
serving, unsupported and uncorroborated, and for being the lone evidence on this issue. The fact remains
that these documents are in proper form, presumed regular, and endure, against arbitrary claims by Silos –
who is an experienced business person – that she signed questionable loan documents whose provisions
for interest rates were left blank, and yet she continued to pay the interests without protest for a number of
years.

ISSUE: W/N the interest rates are null and void.

RULING: YES. In a number of decided cases, the Supreme Court (SC) struck down provisions in credit
documents issued by Philippine National Bank (PNB) to, or required of, its borrowers which allow the
bank to increase or decrease interest rates “within the limits allowed by law at any time depending on
whatever policy it may adopt in the future.”

It is basic that there can be no contract in the true sense in the absence of the element of agreement, or of
mutual assent of the parties. If this assent is wanting on the part of the one who contracts, his act has no
more efficacy than if it had been done under duress or by a person of unsound mind. Similarly, contract
changes must be made with the consent of the contracting parties. The minds of all the parties must meet
as to the proposed modification, especially when it affects an important aspect of the agreement. In the
case of loan contracts, it cannot be gainsaid that the rate of interest is always a vital component, for it can
make or break a capital venture. Thus, any change must be mutually agreed upon, otherwise, it is bereft of
any binding effect.Any modification in the contract, such as the interest rates, must be made with the
consent of the contracting parties. By requiring the petitioners to sign the credit documents and the
promissory notes in blank, and then unilaterally filling them up later on, respondent violated the Truth in
Lending Act, and was remiss in its disclosure obligations.

Remedial Law Review II (CY 2018-2019) – Saturday 1:00pm – 5:00pm: Case Digest Compilation [Atty. Brondial] Page 345
C.2.a.1 MCMP CONST. CORP v MONARK EQUIPMENT CORP.

G.R. No. 201001; November 10, 2014

FACTS: MCMP leased heavy equipment from Monark. The lease covered by a Rental Equipment
Contract. Thus, Monark delivered five (5) pieces of heavy equipment to the project site of MCMP in
Tanay, Rizal and Llavac, Quezon, the delivery evidenced by invoices as well as Documents
Acknowledgment Receipts, received and signed by representatives of MCMP, namely, Jorge Samonte
and Rose Takahashi.

Despite the lapse of the thirty (30)-day period indicated in the invoices, MCMP failed to pay the rental
fees. Upon demands made upon MCMP to pay the amount due, partial payments were made in the
amount of PhP100,000.00 on April 15, 2001 and PhP100,000.00 on August 15, 2001. Further demands
went unheeded. As of April 30, 2002, MCMP owed Monark the amount of PhP1,282,481.83.

Thus, on June 18, 2002, Monark filed a suit for a Sum of Money with the RTC docketed.  In its Answer,
MCMP alleged in defense that the complaint was premature as Monark has refused to give a detailed
breakdown of its claims. MCMP further averred that it had an agreement with Monark that it would not
be charged for the whole time that the leased equipment was in its possession but rather only for the
actual time that the equipment was used although still on the project site. MCMP, however, admitted that
this agreement was not contained in the Contract.

During trial, Monark presented as one of its witnesses, Peregrino, its Senior Account Manager. Peregrino
testified that there were two original copies of the Contract, one retained by Monark, while the other was
given to MCMP. He further testified that Monark's copy had been lost and that diligent efforts to recover
the copy proved futile. Instead, Peregrino presented a photocopy of the Contract which he personally had
on file. MCMP objected to the presentation of secondary evidence to prove the contents of the Contract
arguing that there were no diligent efforts to search for the original copy. Notably, MCMP did not present
its copy of the Contract notwithstanding the directive of the trial court to produce the same.

The RTC issued its Decision finding for Monark. MCMP appealed with the appellated court but the same
was denied. Hence, the case was elevated to the Supreme Court.

ISSUE: Whether the appellate court should have allowed the presentation of secondary evidence to prove
the existence of the contract, following the Best Evidence Rule.

Held:
Petitioner's contention is erroneous.The Best Evidence Rule, a basic postulate requiring the production of
the original document whenever its contents are the subject of inquiry.Relative thereto, Sections 5 and 6
of Rule 130 provide the relevant rules on the presentation of secondary evidence to prove the contents of
a lost document. Before a party is allowed to adduce secondary evidence to prove the contents of the
original, the offeror must prove the following: (1) the existence or due execution of the original; (2) the
loss and destruction of the original or the reason for its non-production in court; and (3) on the part of the
offeror, the absence of bad faith to which the unavailability of the original can be attributed. The correct
order of proof is as follows: existence, execution, loss, and contents.

In the instant case, the CA correctly ruled that the above requisites are present. Both the CA and the RTC
gave credence to the testimony of Peregrino that the original Contract in the possession of Monark has
been lost and that diligent efforts were exerted to find the same but to no avail. Such testimony has
remained uncontroverted. As has been repeatedly held by this Court, "findings of facts and assessment of
credibility of witnesses are matters best left to the trial court." Hence, the Court will respect the evaluation
of the trial court on the credibility of Peregrino.

Remedial Law Review II (CY 2018-2019) – Saturday 1:00pm – 5:00pm: Case Digest Compilation [Atty. Brondial] Page 346
C.2.a.2 WILGEN LOON et al. vs. POWER MASTER, INC.et al.

G.R. No. 189404; December 11, 2013

FACTS: Respondents employed and assigned the petitioners as janitors and leadsmen in various PLDT
offices in Metro Manila area. Subsequently, the petitioners filed a complaint for money claims and illegal
dismissal. Labor Arbiter (LA) partially ruled in favor of the petitioners. Both parties appealed the LA’s
ruling with the NLRC.

6 months after filing their notice of appeal, Respondents filed an unverified supplemental appeal. They
attached photocopied and computerized copies of list of employees with automated teller machine (ATM)
cards to the supplemental appeal. This list also showed the amounts allegedly deposited in the employees’
ATM cards. On the other hand, petitioners filed an Urgent Manifestation and Motion where they asked
for the deletion of the supplemental appeal from the records because it allegedly suffered from infirmities.
First, the supplemental appeal was not verified. Second, it was belatedly filed six months from the filing
of the respondents’ notice of appeal with memorandum on appeal. The petitioners pointed out that they
only agreed to the respondents’ filing of a responsive pleading until December 18, 2002. Third¸ the
attached documentary evidence on the supplemental appeal bore the petitioners’ forged signatures. NLRC
giving weight to the photocopy of computerized payroll records ruled in favor of respondent. It
maintained that the absence of the petitioners’ signatures in the payrolls was not an indispensable factor
for their authenticity. The CA affirmed the NLRC’s ruling.

ISSUE: Whether or not mere photocopies as documentary evidence filed 6 months from notice of appeal
are admissible in evidence where there is an allegation of forgery by the adverse party.

RULING: The answer is in the negative.

While strict adherence to the technical rules of procedure is not required in labor cases, the liberality of
procedural rules is qualified by two requirements:
(1) a party should adequately explain any delay in the submission of evidence; and
(2) a party should sufficiently prove the allegations sought to be proven.

Respondents, in this case, failed to sufficiently prove the allegations sought to be proven. Why the
respondents’ photocopied and computerized copies of documentary evidence were not presented at the
earliest opportunity is a serious question that lends credence to the petitioners’ claim that the respondents
fabricated the evidence for purposes of appeal.

While courts generally admit in evidence and give probative value to photocopied documents in
administrative proceedings, allegations of forgery and fabrication should prompt the adverse party
to present the original documents for inspection. It was incumbent upon the respondents topresent the
originals, especially in this case where the petitioners had submittedtheir specimen signatures. Instead, the
respondents effectively deprived thepetitioners of the opportunity to examine and controvert the alleged
spuriousevidence by not adducing the originals. Failure to present the originals raisesthe presumption that
evidence willfully suppressed would be adverse ifproduced.

Remedial Law Review II (CY 2018-2019) – Saturday 1:00pm – 5:00pm: Case Digest Compilation [Atty. Brondial] Page 347
C.2.a.3 THERESITA DIMAGUILA et al. vs. JOSE and SONIA A. MONTEIRO

G.R. No. 201011; January 27, 2014

FACTS: Respondent spouses filed their Complaint for Partition and Damages against the petitioners.
Later on thecomplaint was amended to an action for recovery. Respondent alleged: (1) that the disputed
property wasoriginally owned by Buenaseda; (2) had long been partitioned between her two sons,
Perfecto and Vitaliano Dimaguila; and (3) that owned 1/3 of the portion of Perfecto’s share by virtue of a
deed of sale executed between them and one of Perfecto’s heir, Pedro. While in their original answer,
petitioner alleged that the subject property had already been extra-judicially partitioned between the heirs
of Vitaliano and Perfecto, they subsequently changed their position when the complaint was amended.
They now claimed that the property was not partitioned with specific metes and bounds. What is cleared
among the heirs is their respective shares thereof.

RTC ruled in favor of Spouses Monteiro. CA affirmed the ruling of the RTC. The CA found that Spouses
Monteiro had established their case by a preponderance of evidence thru their presentation of the Deed of
Extrajudicial Partition, the certified true copy of cadastral map and the municipal assessor's records.
Hence, this petition for review on certiorari where petitioners argued that the cadastral map, which serves
as the basis of the alleged partition, is inadmissible in violation of the best evidence rule and hearsay rule.

ISSUE: Whether or not a certified true copy of cadastral map is inadmissible in evidence on the ground
that it violates the best evidence rule and hearsay rule.

RULING: The answer is in the negative. Anent violation of Best Evidence Rule, Section 3(d) of Rule
130 of the Rules of Court provides that when the subject of inquiry is the contents of a document, no
evidence shall be admissible other than the original document itself, except when the original is a public
record in the custody of a public officer or is recorded in a public office. Section 7 of the same Rule
provides that when the original of a document is in the custody of a public officer or is recorded in a
public office, its contents may be proved by a certified copy issued by the public officer in custody
thereof. Section 24 of Rule 132 provides that the record of public documents may be evidenced by a copy
attested by the officer having the legal custody or the record. Certified true copies of the cadastral map of
Liliw and the corresponding list of claimants of the area covered by the map were presented by two
public officers.

As to the Hearsay Rule, Section 44 of Rule 130 of the Rules of Court similarly provides that entries in
official records are an exception to the rule. The rule provides that entries in official records made in the
performance of the duty of a public officer of the Philippines, or by a person in the performance of a duty
specially enjoined by law, are prima facie evidence of the facts therein stated. The necessity of this rule
consists in the inconvenience and difficulty of requiring the official's attendance as a witness to testify to
the innumerable transactions in the course of his duty. The document's trustworthiness consists in the
presumption of regularity of performance of official duty.

Cadastral maps are the output of cadastral surveys. The DENR is the department tasked to execute,
supervise and manage the conduct of cadastral surveys. It is, therefore, clear that the cadastral map and
the corresponding list of claimants qualify as entries in official records as they were prepared by the
DENR, as mandated by law. As such, they are exceptions to the hearsay rule and are prima facie evidence
of the facts stated therein.

Remedial Law Review II (CY 2018-2019) – Saturday 1:00pm – 5:00pm: Case Digest Compilation [Atty. Brondial] Page 348
C.2.a.4 REPUBLIC OF THE PHILIPPINES, represented by Executive Secretary Eduardo R.
Ermita, the DEPARTMENT OF TRANSPORTATION AND COMMUNICATIONS, AND
MANILA INTERNATIONAL AIRPORT AUTHORITY, Petitioners, vs. HON. JESUS M.
MUPAS, in his capacity as Acting Presiding Judge of the Regional Trial Court, National Capital
Judicial Region, Branch 117, Pasay City, AND PHILIPPINE INTERNATIONAL AIR
TERMINALS CO., INC., Respondents.

G.R. No. 181892; September 8, 2015


x-----------------------x

REPUBLIC OF THE PHILIPPINES, represented by Executive Secretary Eduardo Ermita,


DEPARTMENT OF TRANSPORTATION AND COMMUNICATIONS, AND MANILA
INTERNATIONAL AIRPORT AUTHORITY, Petitioners, vs. PHILIPPINE INTERNATIONAL AIR
TERMINALS COMPANY, INC., TAKENAKA CORPORATION AND ASAHIKOSAN
CORPORATION, Respondents.

G.R. No. 209917


x-----------------------x

TAKENAKA CORPORATION AND ASAHIKOSAN CORPORATION, Petitioners, vs. REPUBLIC


OF THE PHILIPPINES, represented by Executive Secretary Eduardo Ermita, DEPARTMENT OF
TRANSPORTATION AND COMMUNICATIONS, MANILA INTERNATIONAL AIRPORT
AUTHORITY, AND PHILIPPINE INTERNATIONAL AIR TERMINALS COMPANY, INC.
Respondents.

G.R. No. 209696


x-----------------------x

PHILIPPINE INTERNATIONAL AIR TERMINALS CO., INC. Petitioner, vs. REPUBLIC OF THE
PHILIPPINES, as represented by EXECUTIVE SECRETARY EDUARDO ERMITA,
DEPARTMENT OF TRANSPORTATION AND COMMUNICATIONS, MANILA
INTERNATIONAL AIRPORT AUTHORITY, TAKENAKA CORPORATION, AND ASAHIKOSAN
CORPORATION, Respondents.

G.R. No. 209731; BRION, J.:

Before the Court are the consolidated petitions for review on certiorari assailing several decisions.

FACTS: On October 5, 1994, under a build operate-and-transfer (BOT) arrangement, Asia's Emerging
Dragon Corp. (AEDC) submitted an unsolicited proposal to the Government for the construction and
development of the NAIA-IPT III, through the Department of Transportation and Communications
(DOTC) and the Manila International Airport Authority (MIAA).

The DOTC and the MIAA invited the public to submit competitive and comparative proposals to AEDC's
unsolicited proposal in accordance with the BOT Law and its implementing rules. In response, On
September 20, 1996, Paircargo Consortium - composed of People's Air Cargo and Warehousing Co., Inc.
(Paircargo), Philippine Air and Grounds Services, Inc. (PAGS), and Security Bank Corporation (Security
Bank) - submitted its competitive proposal to the Prequalification Bids and Awards Committee (PBAC)

After finding that Paircargo Consortium submitted a bid superior to the AEDC's unsolicited proposal and
after the AEDC's failure to match the competitive bid, the DOTC awarded, through a notice of award, the
NAIA-IPT III project to the Paircargo Consortium (that later organized itself as PIATCO), resulting to the
execution of a Concession Agreement between the government and PIATCO, authorizing the latter the
construction, development, and operation of the NAIA-IPT III during the concession period of twenty-
five (25) years. Thus, PIATCO engaged the services of Takenaka, as well as, Asahikosan, both foreign
corporations organized in Japan, for the construction of the NAIA-IPT. However, PIATCO defaulted on
its obligations, and to settle the problem Takenaka and Asahikosan agreed to defer PIATCO’s payments
until June 2003, conditioned on their receipt of adequate security from PIATCO as stipulated in the
Fourth Supplemental Agreement (relating to the Onshore Construction Contract) and the Fourth
Supplement Agreement (relating to the Offshore Procurement Contract), respectively.

Remedial Law Review II (CY 2018-2019) – Saturday 1:00pm – 5:00pm: Case Digest Compilation [Atty. Brondial] Page 349
On November 29, 2002, President Gloria Macapagal Arroyo declared in her speech that the Government
would not honor the PIATCO contracts. On the same day, Takenaka and Asahikosan notified PIATCO
that they were suspending the construction of the NAIA-IPT III for PIATCO's failure to provide adequate
security.

Trial ensued and PIATCO was required to submit the original documents to the court. PIATCO attached
to its Compliance dated December 14, 2010, the photocopies of numerous documents, and the validation
of PIATCO's computation of attendant costs prepared by Reyes Tacandong & Co., among others.
PIATCO justifies the non-presentment of original documents pursuant to Section 3 (c), Rule 130 of the
Rules of Court.

ISSUE: Whether the non-submission of original documents is justified.

RULING: No.

However, as a condition precedent to the admission of a summary of numerous documents, the proponent
must lay a proper foundation for the admission of the original documents on which the summary is based.
The proponent must prove that the source documents being summarized are also admissible if presented
in court.

Furthermore, when a party invokes Section 3 (c), Rule 130 of the Rules of Court, he does not similarly
invoke Section 3 (a), (b), and/or (d), Rule 130 of the Rules of Court. He does not likewise claim that the
original documents have been lost or destroyed. The party merely asserts that the numerous documents
cannot be examined in court without great loss of time and that the fact sought to be established from
these documents is only the general result of the whole. Whenever a party seeks an exemption under the
best evidence rule pursuant to Section 3 (c), Rule 130 of the Rules of Court, he asks permission from the
trial court to produce a summary of numerous documents, whose originals are available to the adverse
party for inspection. He does not ask permission from the trial court to present in evidence the numerous
non-original documents. Otherwise, the very purpose of Section 3 (c), Rule 130 of the Rules of Court
would be defeated. In that case, every exhibit of non-original documents would be identified,
authenticated, and cross-examined, leading to a tedious and protracted litigation.

Thus, if a party desires to present photocopies of the original documents, he must first establish that the
presentation of photocopies is justified under Section 3 (a), (b), and/or (d), Rule 130 of the Rules of
Court. He must establish the presence of all the elements under these provisions. In the case of lost or
destroyed documents, the offeror of non-original documents must first prove the following elements
before secondary evidence is admitted before the court: (a) the existence or due execution of the original;
(b) the loss and destruction of the original, or the reason for its non-production in court; and (c) the
absence of bad faith on the part of the offeror to which the unavailability of the original can be attributed.
To conclude otherwise is to allow the party to circumvent the best evidence rule and the requirements
under Section 3 (a), (b), and (d), Rule 130 of the Rules of Court by merely invoking Section 3 (c), Rule
130 of the Rules of Court.

Under the best evidence rule, when the subject of inquiry relates to the contents of a document, no
evidence shall be admissible other than the original document itself. In proving the terms of a written
document, the original of the document must be produced in court.

We affirm the lower courts' uniform findings that PIATCO failed to establish its attendant costs. PIATCO
failed to establish that the photocopied documents fall under Section 3 (a), (b), and/or (d), Rule 130 of the
Rules of Court. These photocopied documents are hearsay evidence. They are mere scraps of paper and
have no weight as basis for the attendant costs of the NAIA-IPT III. We likewise cannot give weight to
the summary prepared by Reyes Tacandong & Co. for being double hearsay. Reyes Tacandong & Co.,
whose letter was addressed to PIATCO and not to the trial court, did not state in its report that it
examined the original documents allegedly proving attendant costs. Moreover, in a letter dated December
14, 2010, Reyes Tacandong & Co stated it does not "express any assurance on the attendant costs:"

DOCTRINES: Equiponderance of Evidence Rule —Under the equiponderance of evidence rule, when
the scale of justice shall stand on equipoise and nothing in the evidence inclines a conclusion to one side
or the other, the court will find for the defendant. If the facts and circumstances are capable of two or
more explanations, one of which is consistent with the allegations of the plaintiff and the other consistent
with the defense of the defendant, the evidence does not fulfill the requirement of preponderance of
evidence. When the evidence of the parties is in equipoise, or when there is a doubt as to where the

Remedial Law Review II (CY 2018-2019) – Saturday 1:00pm – 5:00pm: Case Digest Compilation [Atty. Brondial] Page 350
preponderance of evidence lies, the party with the burden of proof fails. The reason for this rule is that
the plaintiff must rely on the strength of his evidence and not on the weakness of the defendant’s claim.
Thus, even if the evidence of the plaintiff may be stronger than that of the defendant, there is no
preponderance of evidence on his side when this evidence is insufficient in itself to establish his cause of
action.

Documentary Evidence; Best Evidence Rule —Under the best evidence rule, when the subject of
inquiry relates to the contents of a document, no evidence shall be admissible other than the original
document itself. In proving the terms of a written document, the original of the document must be
produced in court. The best evidence rule ensures that the exact contents of a document are brought
before the court. In deeds, wills, and contracts, a slight variation in words may mean a great difference in
the rights and obligations of the parties. A substantial hazard of inaccuracy exists in the human process
of making a copy by handwriting or typewriting. Moreover, with respect to oral testimony purporting to
give the terms of a document from memory, a special risk of error is present, greater than in the case of
attempts at describing other situations generally. The best evidence rule likewise acts as an insurance
against fraud. If a party is in the possession of the best evidence and withholds it, and seeks to substitute
inferior evidence in its place, the presumption naturally arises that the better evidence is withheld for
fraudulent purposes that its production would expose and defeat. The rule likewise protects against
misleading inferences resulting from the intentional or unintentional introduction of selected portions of
a larger set of writings.

Voluminous Exhibits —Section 3(c), Rule 130 of the Rules of Court does away with the item-by-item
court identification and authentication of voluminous exhibits which would only be burdensome and
tedious for the parties and the court.

Best Evidence Rule —Whenever a party seeks an exemption under the best evidence rule pursuant to
Section 3(c), Rule 130 of the Rules of Court, he asks permission from the trial court to produce a
summary of numerous documents, whose originals are available to the adverse party for inspection. He
does not ask permission from the trial court to present in evidence the numerous non-original documents.
Otherwise, the very purpose of Section 3(c), Rule 130 of the Rules of Court would be defeated. In that
case, every exhibit of non-original documents would be identified, authenticated, and cross-examined,
leading to a tedious and protracted litigation. Thus, if a party desires to present photocopies of the
original documents, he must first establish that the presentation of photocopies is justified under Section
3(a), (b), and/or (d), Rule 130 of the Rules of Court. He must establish the presence of all the elements
under these provisions.

Lost or Destroyed Documents; Elements that Must be Proved Before Secondary Evidence of Lost or
Destroyed Documents are Admitted.—In the case of lost or destroyed documents, the offeror of non-
original documents must first prove the following elements before secondary evidence is admitted before
the court: (a) the existence or due execution of the original; (b) the loss and destruction of the original,
or the reason for its non-production in court; and (c) the absence of bad faith on the part of the offeror to
which the unavailability of the original can be attributed. To conclude otherwise is to allow the party to
circumvent the best evidence rule and the requirements under Section 3(a), (b), and (d), Rule 130 of the
Rules of Court by merely invoking Section 3(c), Rule 130 of the Rules of Court.

Remedial Law Review II (CY 2018-2019) – Saturday 1:00pm – 5:00pm: Case Digest Compilation [Atty. Brondial] Page 351
C.2.b.1 RAFAEL S. ORTAÑES vs.THE COURT OF APPEALS, OSCAR INOCENTES AND
ASUNCION LLANES INOCENTES

G.R. No. 107372; January 23, 1997; FRANCISCO, J.:

FACTS: Private respondents sold to petitioner two (2) parcels of registered land in Quezon City for a
consideration of P35,000.00 and P20,000.00, respectively.

Private respondents received the payments for the above-mentioned lots, but failed to deliver the titles to
petitioner. The latter demanded from the former the delivery of said titles. Private respondents, however,
refused on the ground that the title of the first lot is in the possession of another person, and petitioner's
acquisition of the title of the other lot is subject to certain conditions.

Offshoot, petitioner sued private respondents for specific performance before the RTC. In their answer
with counterclaim private respondents merely alleged the existence of the oral conditions which were
never reflected in the deeds of sale.

During trial, private respondent Oscar Inocentes, a former judge, orally testified that the sale was subject
to the above conditions, although such conditions were not incorporated in the deeds of sale. Despite
petitioner's timely objections on the ground that the introduction of said oral conditions was barred by the
parol evidence rule, the lower court nonetheless, admitted them and eventually dismissed the complaint as
well as the counterclaim. On appeal, CA affirmed the court a quo. Hence, this petition.

Issue: Whether theparol evidence is admissible to establish the alleged oral conditions-precedent to a
contract of sale, when the deeds of sale are silent on such conditions.

RULING: The parol evidence herein introduced is inadmissible. First, private respondents' oral
testimony on the alleged conditions, coming from a party who has an interest in the outcome of the case,
depending exclusively on human memory, is not as reliable as written or documentary evidence. Spoken
words could be notoriously unreliable unlike a written contract which speaks of a uniform language.
Thus, under the general rule in Section 9 of Rule 130 of the Rules of Court, when the terms of an
agreement were reduced to writing, as in this case, it is deemed to contain all the terms agreed upon and
no evidence of such terms can be admitted other than the contents thereof. Considering that the written
deeds of sale were the only repository of the truth, whatever is not found in said instruments must have
been waived and abandoned by the parties. Examining the deeds of sale, we cannot even make an
inference that the sale was subject to any condition. As a contract, it is the law between the parties.

Secondly, to buttress their argument, private respondents rely on the case of Land Settlement
Development, Co. vs. Garcia Plantation where the Court ruled that a condition precedent to a contract
may be established by parol evidence. However, the material facts of that case are different from this
case. In the former, the contract sought to be enforced expressly stated that it is subject to an agreement
containing the conditions-precedent which were proven through parol evidence. Whereas, the deeds of
sale in this case, made no reference to any pre-conditions or other agreement. In fact, the sale is
denominated as absolute in its own terms.

Third, the parol evidence herein sought to be introduced would vary, contradict or defeat the operation of
a valid instrument, hence, contrary to the rule that:

The parol evidence rule forbids any addition to . . . the terms of a written instrument by testimony
purporting to show that, at or before the signing of the document, other or different terms were orally
agreed upon by the parties.

Although parol evidence is admissible to explain the meaning of a contract, "it cannot serve the purpose
of incorporating into the contract additional contemporaneous conditions which are not mentioned at all
in the writing unless there has been fraud or mistake." No such fraud or mistake exists in this case.

Fourth, we disagree with private respondents' argument that their parol evidence is admissible under the
exceptions provided by the Rules, specifically, the alleged failure of the agreement to express the true
intent of the parties. Such exception obtains only in the following instance:

[W]here the written contract is so ambiguous or obscure in terms that the contractual intention of the
parties cannot be understood from a mere reading of the instrument. In such a case, extrinsic evidence of

Remedial Law Review II (CY 2018-2019) – Saturday 1:00pm – 5:00pm: Case Digest Compilation [Atty. Brondial] Page 352
the subject matter of the contract, of the relations of the parties to each other, and of the facts and
circumstances surrounding them when they entered into the contract may be received to enable the court
to make a proper, interpretation of the instrument.

In this case, the deeds of sale are clear, without any ambiguity, mistake or imperfection, much less
obscurity or doubt in the terms thereof.

Fifth, we are not persuaded by private respondents' contention that they "put in issue by the pleadings" the
failure of the written agreement to express the true intent of the parties. Record shows that private
respondents did not expressly plead that the deeds of sale were incomplete or that it did not reflect the
intention of the buyer (petitioner) and the seller (private respondents). Such issue must be, "squarely
presented." Private respondents merely alleged that the sale was subject to four (4) conditions which they
tried to prove during trial by parol evidence. Obviously, this cannot be done, because they did not plead
any of the exceptions mentioned in the parolevidence rule. Their case is covered by the general rule that
the contents of the writing are the only repository of the terms of the agreement. Considering that private
respondent Oscar Inocentes is a lawyer (and former judge) he was "supposed to be steeped in legal
knowledge and practices" and was "expected to know the consequences" of his signing a deed of absolute
sale. Had he given an iota's attention to scrutinize the deeds, he would have incorporated important
stipulations that the transfer of title to said lots were conditional.

One last thing, assuming arguendo that the parol evidence is admissible, it should nonetheless be
disbelieved as no other evidence appears from the record to sustain the existence of the alleged
conditions. Not even the other seller, Asuncion Inocentes, was presented to testify on such conditions.

ACCORDINGLY, the appealed decision is REVERSED and the records of this case REMANDED to the
trial court for proper disposition in accordance with this ruling.

Remedial Law Review II (CY 2018-2019) – Saturday 1:00pm – 5:00pm: Case Digest Compilation [Atty. Brondial] Page 353
C.2.b.2 LAPU LAPU FOUNDATION, INC. VS. COURT OF APPEALS, 421 SCRA 328,

G.R. NO. 126006; JANUARY 29, 2004

FACTS: Petitioner Elias Q. Tan, then President of the co-petitioner Lapulapu Foundation, Inc., obtained
four loans from the respondent Allied Banking Corporation covered by four promissory notes of
P100,000 each (entire obligation: P493,556.61). Despite demands made on them by the respondent Bank,
the petitioners failed to pay. Respondent Allied Banking filed with the RTC Cebu City, a complaint
seeking payment by the petitioners, jointly and solidarily.

Petitioner Foundation denied incurring indebtedness from respondent Bank alleging that the loans were
obtained by petitioner Tan in his personal capacity and for his own use. It maintained that it never
authorized petitioner Tan to co-sign as President and that the respondent Bank fully knew that the loans
contracted were made in petitioner Tan’s personal capacity.

Petitioner Tan admitted that he contracted the loans from the respondent Bank in his personal capacity
and that he had an unwritten agreement with the respondent Bank that these loans would be renewed on a
year-to-year basis and paid from the proceeds of his shares of stock in the Lapulapu Industries Corp.
According to petitioner Tan, the respondent Banks employee required him to affix two signatures on
every promissory note (blank loan documents), assuring him that the loan documents would be filled out
in accordance with their agreement. However, after he signed and delivered the loan documents to the
respondent Bank, these were filled out in a manner not in accord with their agreement, such that the
petitioner Foundation was included as party thereto. Tan denied that the two demand letters were sent to
them and the signatures on the registry return were not his.

RTC rendered judgment requiring the defendants Tan and Lapulapu Foundation to pay jointly and
solidarily. Court of Appeals (CA) in applying the parol evidence rule, rejected petitioner Tan’s assertion
that there was an unwritten agreement between him and the respondent Bank. CA applied the doctrine of
piercing the veil of corporate entity in holding the petitioners jointly and solidarily liable. The evidence
showed that petitioner Tan had represented himself as the President, opened savings and current accounts
in its behalf, and signed the loan documents for and in behalf of the latter. CA found that the petitioner
Foundation had allowed Tan to act as though he had the authority to contract the loans in its behalf.

ISSUE: Whether the contention of Tan that an unwritten agreement with the bank to contract loans in his
personal capacity would remove liability from the Foundation.

RULING: NO. The Court particularly finds as incredulous petitioner Tans allegation that he was made to
sign blank loan documents and that the phrase IN MY OFFICIAL/PERSONAL CAPACITY was
superimposed by the respondent Banks employee despite petitioner Tans protestation. The Court is hard
pressed to believe that a businessman of petitioner Tans stature could have been so careless as to sign
blank loan documents.

In contrast, as found by the CA, the promissory notes clearly showed upon their faces that they are the
obligation of the petitioner Foundation, as contracted by petitioner Tan in his official and personal
capacity. Moreover, the application for credit accommodation, the signature cards of the two accounts in
the name of petitioner Foundation, as well as New Current Account Record, all accompanying the
promissory notes, were signed by petitioner Tan for and in the name of the petitioner Foundation. These
documentary evidence unequivocally and categorically establish that the loans were solidarily contracted
by the petitioner Foundation and petitioner Tan.

As a corollary, the parol evidence rule likewise constrains this Court to reject petitioner Tans claim
regarding the purported unwritten agreement between him and the respondent Bank on the payment of the
obligation. Section 9, Rule 130 of the of the Revised Rules of Court provides that [w]hen the terms of an
agreement have been reduced to writing, it is to be considered as containing all the terms agreed upon and
there can be, between the parties and their successors-in-interest, no evidence of such terms other than the
contents of the written agreement.

In this case, the promissory notes are the law between the petitioners and the respondent Bank. These
promissory notes contained maturity dates as follows: February 5, 1978, March 28, 1978, April 11, 1978
and May 5, 1978, respectively. That these notes were to be paid on these dates is clear and explicit.
Nowhere was it stated therein that they would be renewed on a year-to-year basis or rolled-over annually
until paid from the proceeds of petitioner Tans shares in the Lapulapu Industries Corp. Accordingly, this

Remedial Law Review II (CY 2018-2019) – Saturday 1:00pm – 5:00pm: Case Digest Compilation [Atty. Brondial] Page 354
purported unwritten agreement could not be made to vary or contradict the terms and conditions in the
promissory notes.

Evidence of a prior or contemporaneous verbal agreement is generally not admissible to vary, contradict
or defeat the operation of a valid contract. While parol evidence is admissible to explain the meaning of
written contracts, it cannot serve the purpose of incorporating into the contract additional
contemporaneous conditions which are not mentioned at all in writing, unless there has been fraud or
mistake. No such allegation had been made by the petitioners in this case.
Finally, the appellate court did not err in holding the petitioners jointly and solidarily liable as it applied
the doctrine of piercing the veil of corporate entity. The petitioner Foundation asserts that it has a
personality separate and distinct from that of its President, petitioner Tan, and that it cannot be held
solidarily liable for the loans of the latter.

DOCTRINE: Parol Evidence Rule; No other evidence to be received other than the contents of the
written agreement.—The parol evidence rule likewise constrains this Court to reject petitioner Tan’s
claim regarding the purported unwritten agreement between him and the respondent Bank on the
payment of the obligation Section 9, Rule 130 of the of the Revised Rules of Court provides that “[w]hen
the terms of an agreement have been reduced to writing, it is to be considered as containing all the terms
agreed upon and there can be, between the parties and their successors-in-interest, no evidence of such
terms other than the contents of the written agreement.”

Same; Parol evidence cannot serve the purpose of incorporating into the contract additional
contemporaneous conditions which are not mentioned at all in writing.—Evidence of a prior or
contemporaneous verbal agreement is generally not admissible to vary, contradict or defeat the
operation of a valid contract. While parol evidence is admissible to explain the meaning of written
contracts, it cannot serve the purpose of incorporating into the contract additional contemporaneous
conditions which are not mentioned at all in writing, unless there has been fraud or mistake. No such
allegation had been made by the petitioners in this case.

Remedial Law Review II (CY 2018-2019) – Saturday 1:00pm – 5:00pm: Case Digest Compilation [Atty. Brondial] Page 355
C.2.b.3 MODESTO LEOVERAS vs. CASIMERO VALDEZ

G.R. No. 169985; June 15, 2011

FACTS: Respondent and the petitioner executed an Agreement, allotting their portions of the subject
property, to wit: Petitioner Modesto Leoveras – 3,020 square meters and Respondent Casimero Valdez –
7,544.27 square meters.

In 1996, the respondent learned that the petitioner had already obtained in his name two TCTs: one -
covering an area of 3,020 square meters; and two - covering an area of 1,004 square meters. Thus,
respondent filed a complaint for Annulment of Title, Reconveyance and Damages against the petitioner,
seeking the reconveyance of the 1,004-square meter portion on the ground that the petitioner is entitled
only to the 3,020 square meters identified in the parties' Agreement.

In his defense, the petitioner claimed: (1) that the parties has agreed that the extent of their ownership
would be based on their actual possession; (2) that he actually possessed and subsequently acquired has a
total area of 4,024 square meters, which he subdivided into two portions and caused to be covered by the
two TCTs in question; and (3) that respondent participated in executing an Affidavit of confirmation of
subdivision, which corrected the mistake in the previously executed Agreement and confirmed the
petitioner's ownership over the disputed property.

The RTC dismissed the complaint. CA reversed the RTC. CA noted the discrepancy between the
respondent's signatures as appearing in the Affidavit, on one hand, and the documents on record, on the
other.

ISSUE:Whether or not an allegation of ownership that is contrary to those expressly stated in an


agreement may be used as evidence.

RULING: The answer is in the negative. The petitioner does not dispute the due execution and the
authenticity the Agreement entered into between him and the respondent. However, he claims that since
the Agreement does not reflect the true intention of the parties, the Affidavit was subsequently executed
in order to reflect the parties' true intention. Factual findings of the CA holding that such affidavit is
spurious due to discrepancy of respondent’s signature therein leads us to rely only on the agreement as the
basis for the claim of ownership of both parties.

Remedial Law Review II (CY 2018-2019) – Saturday 1:00pm – 5:00pm: Case Digest Compilation [Atty. Brondial] Page 356
C.2.b.4 SPOUSES BONIFACIO AND LUCIA PARAS,Petitioners, v. KIMWA CONSTRUCTION
AND DEVELOPMENT CORPORATION,Respondent.

G.R. No. 171601; April 8, 2015; LEONEN, J.:

FACTS: Spouses Paras and Kimwa Construction entered into a contract "Agreement for Supply of
Aggregates" (Agreement) where 40,000 cubic meters of aggregates were "allotted" by Lucia as supplier
to Kimwa. Kimwa was to pick up the allotted aggregates at Lucia's permitted area in Toledo City at
P240.00 per truckload. Pursuant to the Agreement, Kimwa hauled 10,000 cubic meters of aggregates.
Sometime after this, however, Kimwa stopped hauling aggregates. Claiming that in so doing, Kimwa
violated the Agreement, Lucia, joined by her husband, Bonifacio filed the Complaint for breach of
contract with damages that is now subject of this Petition.

RTC ruled for petitioners. The Agreement stipulated that the allotted aggregates were set aside
exclusively for Kimwa. It was contrary to human experience for Kimwa to have entered into an
Agreement with Lucia without verifying the latter’s authority as a concessionaire. Considering that the
Special Permit granted to Lucia (petitioners' Exhibit "A" before the trial court) clearly indicated that her
authority was good for only six (6) months from November 14, 1994, the trial court noted that Kimwa
must have been aware that the 40,000 cubic meters of aggregates allotted to it must necessarily be hauled
by May 15, 1995. As it failed to do so, it was liable to Spouses Paras for the total sum of P720,000.00, the
value of the 30,000 cubic-meters of aggregates that Kimwa did not haul, in addition to attorney's fees and
costs of suit.

CA reversed. Faulted the trial court for basing its findings on evidence presented which were supposedly
in violation of the Parol Evidence Rule. Said that the Agreement in no way obligated respondent to haul
all 40,000 cubic meters before May 15, 1995

ISSUE: Whether or not petitioners Spouses Paras were able to establish that respondent Kimwa was
obliged to haul a total of 40,000 cubic meters of aggregates on or before May 15, 1995.

RULING: YES. There is no violation of the Parol Evidence Rule. We reverse the Decision of the Court
of Appeals and reinstate that of the Regional Trial Court. Respondent Kimwa is liable for failing to haul
the remainder of the quantity which it was obliged to acquire from petitioner Lucia Paras.

Rule 130, Section 9 of the Revised Rules on Evidence provides for the Parol Evidence Rule, the rule on
admissibility of documentary evidence when the terms of an agreement have been reduced into writing:

Section 9. Evidence of written agreements. — When the terms of an agreement have been reduced to
writing, it is considered as containing all the terms agreed upon and there can be, between the parties and
their successors in interest, no evidence of such terms other than the contents of the written agreement.

However, a party may present evidence to modify, explain or add to the terms of written agreement if he
puts in issue in his pleading:

(a) An intrinsic ambiguity, mistake or imperfection in the written agreement;


(b) The failure of the written agreement to express the true intent and agreement of the parties thereto;
(c) The validity of the written agreement; or
(d) The existence of other terms agreed to by the parties or their successors in interest after the execution
of the written agreement.

The term "agreement" includes wills.

Per this rule, reduction to written form, regardless of the formalities observed, "forbids any addition to, or
contradiction of, the terms of a written agreement by testimony or other evidence purporting to show that
different terms were agreed upon by the parties, varying the purport of the written contract."

This rule is animated by a perceived wisdom in deferring to the contracting parties’ articulated intent. In
choosing to reduce their agreement into writing, they are deemed to have done so meticulously and
carefully, employing specific — frequently, even technical — language as are appropriate to their
context. From an evidentiary standpoint, this is also because "oral testimony . . . coming from a party who
has an interest in the outcome of the case, depending exclusively on human memory, is not as reliable as

Remedial Law Review II (CY 2018-2019) – Saturday 1:00pm – 5:00pm: Case Digest Compilation [Atty. Brondial] Page 357
written or documentary evidence. Spoken words could be notoriously unreliable unlike a written contract
which speaks of a uniform language." As illustrated in Abella v. Court of Appeals:

Without any doubt, oral testimony as to a certain fact, depending as it does exclusively on human
memory, is not as reliable as written or documentary evidence. "I would sooner trust the smallest slip of
paper for truth," said Judge Limpkin of Georgia, "than the strongest and most retentive memory ever
bestowed on mortal man." This is especially true in this case where such oral testimony is given by . . . a
party to the case who has an interest in its outcome, and by . . . a witness who claimed to have received a
commission from the petitioner.

This, however, is merely a general rule. Provided that a party puts in issue in its pleading any of the
four(4) items enumerated in the second paragraph of Rule 130, Section 9, "a party may present evidence
to modify, explain or add to the terms of the agreement[.]" Raising any of these items as an issue in a
pleading such that it falls under the exception is not limited to the party initiating an action. In Philippine
National Railways v. Court of First Instance of Albay, this court noted that "if the defendant set up the
affirmative defense that the contract mentioned in the complaint does not express the true agreement of
the parties, then parol evidence is admissible to prove the true agreement of the parties[.]" Moreover, as
with all possible objections to the admission of evidence, a party’s failure to timely object is deemed a
waiver, and parol evidence may then be entertained.

Apart from pleading these exceptions, it is equally imperative that the parol evidence sought to be
introduced points to the conclusion proposed by the party presenting it. That is, it must be relevant,
tending to "induce belief in [the] existence" of the flaw, true intent, or subsequent extraneous terms
averred by the party seeking to introduce parol evidence.

In sum, two (2) things must be established for parol evidence to be admitted: first, that the existence of
any of the four (4) exceptions has been put in issue in a party’s pleading or has not been objected to by
the adverse party; and second, that the parol evidence sought to be presented serves to form the basis of
the conclusion proposed by the presenting party.

DOCTRINE: Two (2) things must be established for parol evidence to be admitted: (1) first, that the
existence of any of the four (4) exceptions has been put in issue in a party’s pleading or has not been
objected to by the adverse party; and (2) second, that the parol evidence sought to be presented serves to
form the basis of the conclusion proposed by the presenting party.

Four exceptions:
(a) An intrinsic ambiguity, mistake or imperfection in the written agreement;
(b) The failure of the written agreement to express the true intent and agreement of the parties thereto;
(c) The validity of the written agreement; or
(d) The existence of other terms agreed to by the parties or their successors in interest after the execution
of the written agreement.

Remedial Law Review II (CY 2018-2019) – Saturday 1:00pm – 5:00pm: Case Digest Compilation [Atty. Brondial] Page 358
C.2.b.5 PHILIPPINE NATIONAL BANK v. GAYAM PASIMIO

G.R. No. 205590; September 02, 2015

Facts: Pasimio filed suit against PNB for the recovery of a sum of money and damages, she alleged
having a peso and dollar time deposit accounts with PNB in the total amount of P4,322,057.57 and
US$5,170.80, respectively; that both investment placements have matured; and when she sought to
withdraw her deposit money with accrued interests, PNB refused to oblige.

PNB admitted the fact of deposit placement but it claimed that Pasimio is without right to insist on their
withdrawal, the deposited amount having already been used in payment of her outstanding loan
obligations to the bank. PNB narrated how the set off of sort came about: Pasimio and her husband took
out three "loans against deposit hold-out" from the PNB Sucat branch, as follows: Three Million One
Hundred Thousand Peso (P3,100,000) loan on March 21, 2001; a One Million Seven Hundred Thousand
Peso (P1,700,000) loan on April 2, 2001; and a Thirty-One Thousand One Hundred US Dollar (US$31,1
00) loan on December 7, 2001.

During the trial following the joinder of issues, Pasimio denied obtaining any loan from PNB, let alone
receiving the corresponding loan proceeds. While conceding signing certain documents which turned out
to be the Peso Loans Against Peso/FX Deposit Loan Applications, the Promissory Notes and Hold-out on
Savings Deposit/ Peso/FX Time Deposit and Assignment of Deposit Substitute and the Disclosure
Statements of Loan/Credit Transaction (Loan Documents), she professed not understanding what they
really meant. She agreed to affix her signature on these loan documents in blank or in an incomplete state,
she added, only because the PNB Sucat branch manager and Customer Relations Officer led her to
believe that what she was signing were related to new high-yielding PNB products. Pasimio would also
deny re-lending the loan proceeds to Paolo Sun.

The RTC ruled in favor of Pasimio. The disposition is predicated on the postulate that Pasimio had
proven by convincing evidence that she did not obtain any loan accommodation from PNB. As a
corollary, the trial court held that there was no evidence showing the release by PNB of the loan proceeds
to Pasimio. CA affirmed the RTC decision.

ISSUE:Whether or not the CA erred in affirming the RTC Decision granting Pasimio's complaint for a
sum of money.

RULING: YES. In upholding the RTC's finding respecting Pasimio's never having received any loan
proceeds, the CA doubtless disregarded the rule holding that a promissory note is the best evidence of the
transaction embodied therein; also, to prove the existence of the loan, there is no need to submit a
separate receipt to prove that the borrower received the loan proceeds. Indeed, a promissory note
represents a solemn acknowledgment of a debt and a formal commitment to repay it on the date and under
the conditions agreed upon by the borrower and the lender. As has been held, a person who signs such an
instrument is bound to honor it as a legitimate obligation duly assumed by him through the signature he
affixes thereto as a token of his good faith. If he reneges on his promise without cause, he forfeits the
sympathy and assistance of this Court and deserves instead its sharp repudiation.

The Court has also declared that a mere denial of the receipt of the loan, which is stated in a clear and
unequivocal manner in a public instrument, is not sufficient to assail its validity. To overthrow the recitals
of such instrument, convincing and more than merely preponderant evidence is necessary. A contrary rule
would throw wide open doors to fraud. Following this doctrine, Pasimio's notarized promissory notes
bearing her signature and that of her husband must be upheld, absent, as here, strong, complete, and
conclusive proof of their nullity.

The promissory notes, bearing Pasimio's signature, speak for themselves. To repeat, Pasimio has not
questioned the genuineness and due execution of the notes. By signing the promissory notes, she is
deemed to acknowledge receipt of the corresponding loan proceeds. Withal, she cannot plausibly set up
the defense that she did not apply for any loan, and receive the value of the notes or any consideration
therefor in order to escape her liabilities under these promissory notes.

But the foregoing is not all. PNB presented evidence that strengthened its allegation on the existence of
the loan. Here, each promissory note was supported by a corresponding loan application form and
disclosure statement, all of which carried Pasimio's signatures. Isolated from each other, these documents

Remedial Law Review II (CY 2018-2019) – Saturday 1:00pm – 5:00pm: Case Digest Compilation [Atty. Brondial] Page 359
might not prove the existence of the loan, but when taken together, collectively, they show that Pasimio
took the necessary steps to contract loans from PNB and was aware of their terms and conditions.

Finally, it is well to consider this rule: that when the terms of an agreement have been reduced to writing,
it is to be considered as containing all such terms, and, therefore, there can be, between the parties and
their successors-in-interest, no evidence of the terms of the agreement other than the contents of the
writing.

Under this rule, parol evidence or oral evidence cannot be given to contradict, change or vary a written
document, except if a party presents evidence to modify, explain, or add to the terms of a written
agreement and puts in issue in his pleadings: (a) an intrinsic ambiguity, mistake, or imperfection in the
written agreement; (b) the failure of the written agreement to express the true intent and agreement of the
parties; (c) the validity of the written agreement; and (d) the existence of other terms agreed to by the
parties or their successors-in-interest after the execution of the written agreement. Such evidence,
however, must be clear and convincing and of such sufficient credibility as to overturn the written
agreement. Since no evidence of such nature is before the Court, the documents embodying the loan
agreement of the parties should be upheld.

WHEREFORE, premises considered, the petition is GRANTED. The assailed Decision of the Court of
Appeals dated January 23, 2013 in CA-G.R. CV No. 94079 is REVERSED and SET ASIDE. Respondent
Ligaya M. Pasimio's complaint in Civil Case No. CV-05-0195 before the Regional Trial Court of
Paranaque City, Branch 196 is DISMISSED for lack of merit.

Remedial Law Review II (CY 2018-2019) – Saturday 1:00pm – 5:00pm: Case Digest Compilation [Atty. Brondial] Page 360
C.2.c.1 HEIRS OF LOURDES SAEZ SABANPAN vs. ALBERTO C. COMORPOSA et al.

G.R. No. 152807; August 12, 2003; PANGANIBAN, J.:

FACTS: Petitioners filed an action for unlawful detainer against respondents and alleged that the
disputed property was owned by Marcos Saez, predecessor of petitioners; that Marcos’ son Adolfo, for
humanitarian reasons, allowed respondents to occupy a portion of Marcos Saez’ land without paying any
rental. On 7 May 1998, a formal demand was made upon the respondents to vacate the premises but the
latter refused to vacate the same and claimed that they were the legitimate claimants and the actual and
lawful possessors of the premises. MTC rendered judgment in favor of petitioners. On appeal, RTC
reversed the said decision. Affirming the RTC, the CA upheld the right of respondents as claimants and
possessors. The CA lend credence to the Certification issued by the DENR’s community environment and
natural resources (CENR) officer was proof that when the cadastral survey was conducted, the land was
still alienable and was not yet allocated to any person. Therefore, respondents after sufficiently proving
their actual, physical, open, notorious, exclusive, continuous and uninterrupted possession thereof since
1960 have better right to possess alienable and disposable land of the public domain. Hence, this Petition,
petitioners avers that CA gravely abuse its discretion in giving weight to the CENR Officer’s
Certification, which only bears the facsimile of the alleged signature of a certain Jose F. Tagorda.

ISSUE: Whether or not a certification issued by a public officer bearing a facsimile signature is
inadmissible in evidence.

RULING: The rule stated in Garvida v. Sales Jr that – “Pleadings filed via fax machines are not
considered originals and are at best exact copies. As such, they are not admissible in evidence, as there is
no way of determining whether they are genuine or authentic” is not applicable to the instant case. The
Certification, on the other hand, is being contested for bearing a facsimile of the signature of CENR
Officer Jose F. Tagorda. The facsimile referred to is not the same as that which is alluded to in Garvida.
The one mentioned here refers to a facsimile signature, which is defined as a signature produced by
mechanical means but recognized as valid in banking, financial, and business transactions. Note that the
CENR officer has not disclaimed the Certification. In fact, the DENR regional director has acknowledged
and used it as reference in his Order dated April 2, 1998. If the Certification were a sham as petitioner
claims, then the regional director would not have used it as reference in his Order. Instead, he would have
either verified it or directed the CENR officer to take the appropriate action, as the latter was under the
former’s direct control and supervision.

Remedial Law Review II (CY 2018-2019) – Saturday 1:00pm – 5:00pm: Case Digest Compilation [Atty. Brondial] Page 361
C.2.c.2 TORRES VS PAGCORELLERY MARCH G. TORRES, VS PHILIPPINE
AMUSEMENT and GAMING CORPORATION, represented by ATTY. CARLOS R. BAUTISTA,
JR.,

G.R. No. 193531; December 14, 2011; PERALTA, J.:

FACTS: Petitioner was a Slot Machine Operations Supervisor (SMOS) of respondent Philippine


Amusement and Gaming Corporation (PAGCOR). On the basis of an alleged intelligence report of
padding of the Credit Meter Readings, then Casino Filipino-Hyatt (CF Hyatt), and internal security
personnel of respondent PAGCOR, and in connivance with slot machine customers, respondent
PAGCOR's Corporate Investigation Unit conducted an investigation to verify the veracity of such
report. The CIU discovered the scheme of CMR padding which was committed by adding zero after the
first digit of the actual CMR of a slot machine or adding a digit before the first digit of the actual
CMR, e.g., a slot machine with an actual CMR of P5,000.00 will be issued a CMR receipt with the
amount of either P50,000.00 or P35,000.00.

Petitioner was then required to explain in writing within seventy-two (72) hours from receipt thereof.
Petitioner wrote Manager Bangsil a letter explanation/refutation of the charges against him. He denied
any involvement or participation in any fraudulent manipulation of the CMR or padding of the slot
machine receipts, and he asked for a formal investigation of the accusations against him. On August 4,
2007, petitioner received a letter dismissing him from the service. Petitioner then filed with the CSC a
Complaint against PAGCOR and its Chairman Efraim Genuino for illegal dismissal, non-payment of
backwages and other benefits. Respondent PAGCOR filed its Comment wherein it alleged, among others,
that petitioner failed to perfect an appeal within the period and manner provided by the Uniform Rules on
Administrative Cases in the Civil Service Law. On June 23, 2008, the CSC, treating petitioner's complaint
as an appeal from the PAGCOR's decision dismissing petitioner from the service, issued Resolution No.
081204 denying petitioner's appeal. In dismissing the petition, the CA found that petitioner failed to
adduce clear and convincing evidence that he had filed a motion for reconsideration. It found insufficient
to merit consideration petitioner's claim that he had sent through a facsimile transmission a
letter/reconsideration dated August 13, 2007 addressed to PAGCOR's Chairman, members of the Board
of Directors and the Merit Systems Protection Board; that assuming arguendo that a letter reconsideration
was indeed sent through a facsimile transmission, such facsimile transmission is inadmissible as
electronic evidence under the Electronic Commerce Act of 2000

ISSUE: Whether the letter of reconsideration allegedly sent by Torres through facsimile transmission,
assuming that such was in fact sent, may be considered as a pleading which necessarily tolls the period to
appeal to the CSC

RULING: No. Clearly, a motion for reconsideration may either be filed by mail or personal
delivery. When a motion for reconsideration was sent by mail, the same shall be deemed filed on the date
shown by the postmark on the envelope which shall be attached to the records of the case. On the other
hand, in case of personal delivery, the motion is deemed filed on the date stamped thereon by the proper
office. And the movant has 15 days from receipt of the decision within which to file a motion for
reconsideration or an appeal therefrom.

A facsimile is not a genuine and authentic pleading. It is, at best, an exact copy preserving all the marks
of an original. Without the original, there is no way of determining on its face whether
the facsimile pleading is genuine and authentic and was originally signed by the party and his counsel. It
may, in fact, be a sham pleading.

Moreover, a facsimile transmission is not considered as an electronic evidence under the Electronic
Commerce Act. In MCC Industrial Sales Corporation v. Ssangyong Corporation, the terms "electronic
data message" and "electronic document," as defined under the Electronic Commerce Act of 2000, do not
include a facsimile transmission.  Accordingly, a facsimile transmission cannot be considered
as electronic evidence.  It is not the functional equivalent of an original under the Best Evidence Rule and
is not admissible as electronic evidence.

Doctrine: A facsimile is not a genuine and authentic pleading. It is, at best, an exact copy preserving all
the marks of an original. Without the original, there is no way of determining on its face whether
the facsimile pleading is genuine and authentic and was originally signed by the party and his counsel. It
may, in fact, be a sham pleading.

Remedial Law Review II (CY 2018-2019) – Saturday 1:00pm – 5:00pm: Case Digest Compilation [Atty. Brondial] Page 362
Remedial Law Review II (CY 2018-2019) – Saturday 1:00pm – 5:00pm: Case Digest Compilation [Atty. Brondial] Page 363
C.2.c.3 RUSTAN ANG v. CA

G.R. No. 182835; April 20, 2010

FACTS: Irish Sagud and Rustan Ang became "on-and-off" sweethearts until Irish decided to break up
with Rustan after learning that he had taken a live‐in partner whom he had gotten pregnant. Before Rustan
got married, he tried to convince Irish to elope with him. Irish, however, rejected his proposal. She
changed her cellphone number but Rustan somehow managed to get hold of it and sent her text messages.
He used two cellphone numbers for sending his messages. Irish replied to his text messages but it was to
ask him to leave her alone.

On June 5, 2005, Irish received through multimedia message service (MMS) a picture of a naked woman
with her face superimposed on the figure. The sender's cellphone number was one of the numbers that
Rustan used. After she got the obscene picture, Irish got other text messages from Rustan. He boasted that
it would be easy for him to create similarly scandalous pictures of her. He also threatened to spread the
picture through the internet. Irish sought the help of the police in apprehending Rustan. Under police
supervision, she contacted Rustan and asked him to meet her at the Lorentess Resort.

When Rustan came, police officers intercepted and arrested him. They searched him and seized his Sony
Ericsson P900 cellphone and several SIM cards. While Rustan was being questioned at the police station,
he shouted at Irish: "Malandi ka kasi!"

Rustan claims that he went to meet Irish because she asked him to help her identify a prankster who was
sending her malicious text messages. Rustan got the sender's number and, pretending to be Irish,
contacted the person. Rustan claims that he got back obscene messages from the prankster, which he
forwarded to Irish from his cellphone. According to him, this explained why the obscene messages
appeared to have originated from his cellphone number. Rustan claims that it was Irish herself who sent
the obscene picture to him.

The RTC found Irish's testimony completely credible, given in an honest and spontaneous manner. The
trial court found Rustan guilty of the violation of Section 5(h) of R.A. 9262. On Rustan’s appeal to the
Court of Appeals (CA), the latter rendered a decision dated January 31, 2008, affirming the RTC
decision.

Rustan claims that the obscene picture sent to Irish through a text message constitutes an electronic
document. Thus, it should be authenticated by means of an electronic signature, as provided under
Section 1, Rule 5 of the Rules on Electronic Evidence (A.M. 01-7-01-SC). The CA denied Rustan’s
motion for reconsideration in a resolution dated April 25, 2008. Thus, Rustan filed the present for review
on certiorari.

ISSUES:
1. Whether or not the evidence used to convict Rustan was obtained from him in violation of his
constitutional rights; and
2. Whether or not the Rules on Electronic Evidence applies on criminal cases and thus, the picture sent
through a cell phone message wherein Sagud’s face was attached on the body of a nude woman may be
used as evidence for violation of Section 5(h) of R.A. 9262.

RULING:
1. Rustan argues that, since he was arrested and certain items were seized from him without any
warrant, the evidence presented against him should be deemed inadmissible. But the fact is that the
prosecution did not present in evidence either the cellphone or the SIM cards that the police officers
seized from him at the time of his arrest. The prosecution did not need such items to prove its case.
Exhibit C for the prosecution was but a photograph depicting the Sony Ericsson P900 cellphone that was
used, which cellphone Rustan admitted owning during the pre-trial conference.

Actually, though, the bulk of the evidence against him consisted in Irish’s testimony that she received the
obscene picture and malicious text messages that the sender’s cellphone numbers belonged to Rustan with
whom she had been previously in communication. Indeed, to prove that the cellphone numbers belonged
to Rustan, Irish and the police used such numbers to summon him to come to Lorentess Resort and he
did.12 Consequently, the prosecution did not have to present the confiscated cellphone and SIM cards to
prove that Rustan sent those messages.

Remedial Law Review II (CY 2018-2019) – Saturday 1:00pm – 5:00pm: Case Digest Compilation [Atty. Brondial] Page 364
Moreover, Rustan admitted having sent the malicious text messages to Irish.13 His defense was that he
himself received those messages from an unidentified person who was harassing Irish and he merely
forwarded the same to her, using his cellphone. But Rustan never presented the cellphone number of the
unidentified person who sent the messages to him to authenticate the same. The RTC did not give
credence to such version and neither will this Court. Besides, it was most unlikely for Irish to pin the
things on Rustan if he had merely tried to help her identify the sender.

2. The rules he cites do not apply to the present criminal action. The Rules on Electronic Evidence
applies only to civil actions, quasi-judicial proceedings, and administrative proceedings.

However, Rustan is raising this objection to the admissibility of the obscene picture, Exhibit A, for the
first time before this Court. The objection is too late since he should have objected to the admission of the
picture on such ground at the time it was offered in evidence. He should be deemed to have already
waived such ground for objection.

In conclusion, this Court finds that the prosecution has proved each and every element of the crime
charged beyond reasonable doubt. 

Remedial Law Review II (CY 2018-2019) – Saturday 1:00pm – 5:00pm: Case Digest Compilation [Atty. Brondial] Page 365
C.2.c.4 People of the Philippines, appellee, vs. Noel Enojas y Hingpit, Arnold Gomez y Fabregas,
Fernando Santos y Delantar, and Roger Jalandoni y Ari, appellants.

G. R. No. 204894; March, 2014; Abad, J.

~ Text messages are to be proved by the testimony of a person who was a party to the same or has
personal knowledge of them.

FACTS: PO2 Gregorio and PO2 Pangilinan were patrolling the vicinity of Toyota Alabang and SM
Southmall when they spotted a taxi that was suspiciously parked in front of the Aguila Auto Glass shop
that was driven by accused Enojas which then was asked to come with the Policemen to the station.

The mobile car stopped at a 7-11 convenience store where PO2 Pangilinan went down to relieve himself.
As PO2 Pangilinan approached the store’s door, he came upon two suspected robbers and shot it out with
them. PO2 Pangilinan shot one suspect dead and hit the other who still managed to escape. But someone
fired at PO2 Pangilinan causing his death.

On hearing the shots, PO2 Gregorio came around but the men were able to take a taxi and escape. PO2
Gregorio radioed for help and for an ambulance. On returning to his mobile car, he realized that accused
Enojas, the taxi driver they had with them had fled.

P/Insp. Torred and PO2 Rosarito immediately responded to PO2 Gregorio’s urgent call. Suspecting that
accused Enojas, the taxi driver who fled, was involved in the attempted robbery, they searched the
abandoned taxi and found a mobile phone that Enojas apparently left behind. P/Ins. Torred instructed PO3
Cambi to monitor its incoming messages.

PO3 Cambi and PO2 Rosarito monitored the messages in accused Enojas’ mobile phone and, posing as
Enojas, communicated with the other accused. The police then conducted an entrapment operation that
resulted in the arrest of accused Santos and Jalandoni. Subsequently, the police were also able to capture
accused Enojas and Gomez. The prosecution presented the transcripts of the mobile phone text messages
between Enojas and some of his co-accused.

Manifesting in open court that they did not want to adduce any evidence or testify in the case, the accused
opted to instead file a trial memorandum for their defense. They pointed out that they were entitled to an
acquittal since they were all illegally arrested and since the evidence of the text messages were
inadmissible, not having been properly identified.

RTC rendered judgment, finding all the accused guilty of murder qualified by evident premeditation and
use of armed men with the special aggravating circumstance of use of unlicensed firearms.

Upon review, CA dismissed the appeal and affirmed in toto the conviction of the accused. The CA,
however, found the absence of evident premeditation since the prosecution failed to prove that the several
accused planned the crime before committing it.

ISSUE: WON the RTC is correct in admitting as evidence the text messages that caused the arrest of
Enojas, et. al.

RULING: YES. RTC admitted them [the text messages] in conformity with the Court’s earlier
Resolution applying the Rules on Electronic Evidence to criminal actions.

Text messages are to be proved by the testimony of a person who was a party to the same or has personal
knowledge of them. Here, PO3 Cambi, posing as the accused Enojas, exchanged text messages with the
other accused in order to identify and entrap them. As the recipient of those messages sent from and to the
mobile phone in his possession, PO3 Cambi had personal knowledge of such messages and was
competent to testify on them.

[…] a crime had been committed—the killing of PO2 Pangilinan—and the investigating police officers
had personal knowledge of facts indicating that the persons they were to arrest had committed it. The text
messages to and from the mobile phone left at the scene by accused Enojas provided strong leads on the
participation and identities of the accused. Indeed, the police caught them in an entrapment using this
knowledge.

Remedial Law Review II (CY 2018-2019) – Saturday 1:00pm – 5:00pm: Case Digest Compilation [Atty. Brondial] Page 366
[…] the Court modifies the Court of Appeals decision, finding the accused guilty of homicide with the
special aggravating circumstance of use of unlicensed firearms.

Remedial Law Review II (CY 2018-2019) – Saturday 1:00pm – 5:00pm: Case Digest Compilation [Atty. Brondial] Page 367
C.2.c.5 RAMON A. SYHUNLIONG v. TERESITA D. RIVERA

GR No. 200148:2014-06-04

FACTS: Syhunliong and Rivera are respectively the private complainant and defendant in the instant
case. Syhunliong is the President of BANFF Realty and Development Corporation (BANFF) while
Rivera used to be the Accounting Manager of BANFF. She was hired in September of 2002.

About three years after, Rivera, citing personal and family matters, tendered her resignation to be
effective on February 3, 2006 and continued working for BANFF until March of the same year to
complete the turn-over of papers under her custody to Jennifer Lumapas (Lumapas) Sometime in April
of 2006, Rivera called Lumapas to request for the payment of her remaining salaries, benefits
and incentives. Lumapas informed Rivera that her benefits would be paid, but the check representing
her salaries was still unsigned, and her incentives were put on hold by Syhunliong.

On April 6, 2006, at around 11:55 a.m., Rivera sent the following text message to one of BANFF’s
official cellular phones held by Lumapas:
I am expecting that. grabe talaga sufferings ko dyan hanggang pagkuha ng last pay ko. I don’t
deserve this because I did my job when I was still there. God bless. Sana yung pagsimba niya,
alam niya real meaning.
Minutes later, Rivera once again texted another message, which reads:
Kailangan release niya lahat [nang] makukuha ko diyan including incentive up to the last
datena nandyan ako para di na kami abot sa labor.
Subsequently, on December of 2006, Rivera filed before the National Labor Relations Commission a
complaint against Syhunliong for underpaid salaries, 13th to 16th month and incentive pay, gratuities and
tax refund.

On April 16, 2007 pending the resolution of the aforecited labor case, Syhunliong instituted against
Rivera a complaint for libel, and the public prosecutor finds probable cause to indict Rivera the
crime of libel.

Rivera filed a Motion to Quash the aforequoted information. She argued that the text message, which
was the subject of the libel complaint, merely reflected the undue stress she had suffered due to the
delay in the release of her unpaid salaries, benefits and incentives.

Further, the facts charged in the information did not constitute the crime of libel as the elements of
malice and the making of defamatory imputation for public consumption were wanting. Her text
message was not prompted by ill will or spite, but was merely sent as part of her duty to defend her own
interests.

During the arraignment on October 11, 2007, Rivera entered a plea of not guilty.
The lower court concurred with the Public Prosecutor’s finding that there was probable cause to indict
Rivera for having ascribed to Syhunliong the possession of a vice or defect, or for having committed an
act, tending to cause dishonor or discredit to the latter’s name. As a defense, Rivera said her text
message falls squarely within the parameters of "Privileged Communication" or the elements of Article
353 of the Revised Penal Code are not fully established by the Prosecution’s evidence. Thereafter,
the lower court issued an Order on June 18, 2009 denying Rivera’s motion for reconsideration of the
foregoing.

On July 11, 2011, the Appellate Court rendered their decision directing the dismissal of the
information for libel filed against Rivera.

The CA likewise explained that:


The focal issue to the parties in the present case is whether the facts charged in the information as well
as the undeniable facts appearing on the record show that an offense of libel has been committed.
Our criminal law convincingly provide us with a definition of libel –
It is a public and malicious imputation of a crime, or of a vice or defect ... or any act, omission,
condition, status or circumstance tending to cause the dishonor, discredit or contempt of ... a person. x
x x.

ISSUES: Whether or not the CA correctly ruled that the facts charged in the information do not
constitute the offense of libel?

Remedial Law Review II (CY 2018-2019) – Saturday 1:00pm – 5:00pm: Case Digest Compilation [Atty. Brondial] Page 368
Whether or not the CA committed reversible error in ordering the outright dismissal of the
complaint of Syhunliong on the putative ground that the allegedly libelous text messages were
privileged communication?

RULING: There is no merit in the instant petition.


Prescription had set in. Well settled rule in statutory construction that the liberal construction of
prescriptive laws on criminal statutes emanates from the liberality of the State, any doubt on this matter
must be resolved in favor of the grantee thereof, the accused. As prescription of the crime is the loss by
the State of the right to prosecute and punish the same.

In the case at bar, it is extant in the records that Syhunliong filed his complaint against Rivera more
than one year after the allegedly libelous message was sent to Lumapas. Whether the date of the filing
of the complaint is April 16, 2007 or August 18, 2007, it would not alter the fact that its institution was
made beyond the prescriptive period provided for in Article 90 of the RPC.

In relation thereto, Article 89 of the Revised Penal Code provides that the prescription of crime has the
effect of totally extinguishing the criminal liability.

Prescription of the crime is already a compelling reason for this Court to order the dismissal of the
libel information, but the Court still stresses that the text message which Rivera sent to Lumapas falls
within the purview of a qualified privileged communication.

The rule on privileged communication means that a communication made in good faith on
any subject matter in which the communicator has an interest, or concerning which he has a duty,
is privileged if made to a person having a corresponding duty.

In order to prove that a statement falls within the purview of a qualified privileged communication
under Article 354, No. 1, the following requisites must concur:
(1) the person who made the communication had a legal, moral, or social duty to make the
communication, or at least, had an interest to protect, which interest may either be his own or of the one
to whom it is made;
(2) the communication is addressed to an officer or a board, or superior, having some interest or duty in
the matter, and who has the power to furnish the protection sought ; and
(3) the statements in the communication are made in good faith and without malice.
Presiding from the above, the Court thus finds no error in the CA's declaration that Rivera's text
message falls with in the ambit of a qualified privileged communication since she was speaking in
response to duty, to protect her own interest, and not out of an intent to injure the reputation of
Syhunliong. Besides, there was no unnecessary publicity of the message beyond that of conveying it to
the party concerned.

Remedial Law Review II (CY 2018-2019) – Saturday 1:00pm – 5:00pm: Case Digest Compilation [Atty. Brondial] Page 369
C.2.c.6 ELLA M. BARTOLOME, Complainant vs. ROSALIE B. MARANAN, COURT
STENOGRAPHER III, REGIONAL TRIAL COURT, BRANCH 20, IMUS, CAVITE,
Respondent.

FACTS: This administrative matter started through the sworn affidavit complaint of Ella M.
Bartolome(complainant) filed against Rosalie B. Maranan [respondent, Court Stenographer III, Regional
Trial Court (RTC), Branch 20, Imus, Cavite], charging her with extortion, graft and corruption, gross
misconduct and conduct unbecoming of a court employee. The complainant alleged that the respondent
asked money from her in the amount of ₱200,000.00, which was later reduced to ₱160,000.00, to
facilitate the filing of her case for annulment of marriage. She further alleged that the respondent
undertook to have the case decided in her favor without the need of court appearances during the
proceedings of the case. In support of her allegations, the complainant attached to her affidavit-complaint,
among others, the transcribed electronic communications (text messages) between her and the respondent;
and a versatile compact disc (VCD) containing the video taken during the entrapment operation
conducted against the respondent.

ISSUE: WON the pieces of evidence she submitted are sufficient to prove the respondent’s anomalous
activities, and prayed for the immediate resolution of her complaint.

RULING: Yes. Ephemeral electronic communications are now admissible evidence, subject to certain
conditions. "Ephemeral electronic communication" refers to telephone conversations, text messages,
chatroom sessions, streaming audio, streaming video, and other electronic forms of communication the
evidence of which is not recorded or retained. It may be proven by the testimony of a person who was a
party to the communications or has personal knowledge thereof. Under Section 1, Rule 11 of A.M. No.
01-7-01-SC, audio, photographic and video evidence of events, acts or transactions shall be admissible
provided it shall be shown, presented or displayed to the court and shall be identified, explained or
authenticated by the person who made the recording or by some other person competent to testify on the
accuracy thereof. In the present case, there is no doubt regarding the probative value of the text messages
as evidence in considering the present case. The complainant, who was the recipient of the text messages
and who therefore has personal knowledge of these text messages, identified the respondent as the sender
through cellphone number 09175775982. The respondent herself admitted that her conversations with the
complainant had been thru SMS messaging and that the cellphone number reflected in the complainant’s
cellphone from which the text messages originated was hers. She confirmed that it was her cellphone
number during the entrapment operation conducted by the Imus Cavite Police.

Remedial Law Review II (CY 2018-2019) – Saturday 1:00pm – 5:00pm: Case Digest Compilation [Atty. Brondial] Page 370
C.2.c.7 BBB vs. AAA

750 SCRA 188; Feb. 9, 2015; J. Reyes

FACTS: BBB and AAA dated in 1993. AAA had a boy named CCC from a previous relationship.
During their marriage. They bore two more children namely, [DDD] and [EEE] . [BBB] and [AAA]
married civil rights on October 10, 2002 and thereafter, the birth certificates of the children, including
[CCC’s], was amended to change their civil status to legitimated by virtue of the said marriage. The
relationship, both admit, was far from ideal and has had its share of happy moments and heated
arguments. [BBB] alleges that [AAA’s] irrational jealousy has caused their frequent arguments..

BBB left the family home and soon after AAA decided to leave and brought the children. [AAA], on the
other hand, alleges that their heated arguments were often due to [BBB’s] incessant womanizing. The
breaking point for [AAA] came when, [BBB’s] alleged mistress, a woman by the name of [FFF], insulted
and humiliated [AAA] in public, in the presence of [BBB] himself, who, according to [AAA], did nothing
to stop the same. BBB displayed acts of marital infidelity which exposed AAA to public ridicule causing
her emotional and psychological distress. While BBB alleged that FFF was only a professional colleague,
he continued to have public appearances with her which did not help to dispel AAA’s accusation that the
two had an extra-marital relation. Further, BBB verbally abused AAA either in person or through text
messages. AAA filed a Temporary Restraining Order, RTC granted the application and CA affirmed the
decision with modification.

ISSUE: Whether or not the unauthenticated text messages adduced by AAA is admissible.

RULING: BBB posits that the text messages offered by AAA as evidence were unauthenticated; hence,
doubt exists as to their admissibility The doubt raised by BBB anent the admissibility of the text messages
as evidence is not genuinely a legal issue.
BBB effectively admitted in the pleadings filed with this Court (SC) and CA that he indeed sent the text
messages attributed to him by AAA, in the pleadings BBB attempted to justify why he sent the messages
to AAA in doing so, in effect, admitted authorship of messages which AAA adduced as evidence, it is
likewise did not deny ownership of the cellphone number from which the text messages were sent.
The issue of admissibility of the text messages requires an interpretation of the rules of evidence, this
Court does not find the same to be necessary. While BBB had admitted authorship of the text messages,
he pleads for this Court to consider those messages as inadmissible for allegedly being unauthenticated.
BBB’s arguments are unbearably self -contradictory and he cannot be allowed to take refuge under
technical rules of procedure to assail what is already apparent

Principle
Any question as to the admissibility of text messages as evidence is rendered moot and academic if the
party raising such issue admits authorship of the subject messages.

Remedial Law Review II (CY 2018-2019) – Saturday 1:00pm – 5:00pm: Case Digest Compilation [Atty. Brondial] Page 371
C.2.c.8 ASTORGA AND REPOL LAW OFFICES, REPRESENTED BY ATTY. ARNOLD B.
LUGARES, Complainant, v. ALEXANDER D. VILLANUEVA, SHERIFF IV, REGIONAL TRIAL
COURT, BRANCH 60, MAKATI CITY, Respondent.

A.M. No. P-09-2668; February 24, 2015

FACTS: An administrative case was filed by Atty. Lugares against Sheriff Villanueva for alleged
extortion.

The Sheriff refused to execute the judgment and allegedly asked for 8000 which was lowered to 5000 for
the judgment to be executed.
The only evidence Atty. Lugares presented to prove the allegations are text messaged from Villanueva.

In the trial court, the case was dismissed for lack of evidence. Primarily because the text messages was
not considered as evidence.

ISSUE: Whether or not a text message is a substantial evidence that may be used in this administrative
case?

RULING: Yes. There is substantial evidence to support Atty. Lugares’ allegation of neglect of duty.

In previous administrative cases involving other court personnel, text messages were admitted as
evidence and given probative value by this court. In those cases, the court considered the content of the
text messages and the identification of the person sending them as substantial evidence to prove the
commission of administrative offenses.

Atty. Lugares was able to present the text messages he received in his cellular phone. He attached
photographs of the screen of his cellular phone, showing the messages as they were received. He
submitted respondent’s calling card that contained the same phone number seen in the text messages.
Through this calling card, he was able to prove that respondent was the source of the text messages.
Respondent denied meeting with Atty. Lugares, but he never denied sending the text messages to him.

The content of the text messages from respondent and the circumstances within which they were made
constitute substantial evidence that justify the finding of administrative liability.

Remedial Law Review II (CY 2018-2019) – Saturday 1:00pm – 5:00pm: Case Digest Compilation [Atty. Brondial] Page 372
C.3.c.1 LUISA NAVARRO MARCOS* vs. THE HEIRS OF THE LATE DR. ANDRES
NAVARRO, JR

G.R. No. 198240, July 03, 2013

FACTS: Petitioner discovered that respondents are claiming exclusive ownership of the subject lot.
Respondents based their claim on the Affidavit of Transfer of Real Property where Andres, Sr. (common
ascendant of both petitioner and respondent) donated the subject lot to Andres, Jr. Believing that the
affidavit is a forgery, the sisters, requested a handwriting examination of the affidavit. The PNP
handwriting expert PO2 Mary Grace Alvarez found that Andres, Sr.’s signature on the affidavit and the
submitted standard signatures of Andres, Sr. were not written by one and the same person.

Thus, the sisters sued the respondents for annulment of the deed of donation. After the pre-trial,
respondents moved to disqualify PO2 Alvarez as a witness. They argued that the RTC did not authorize
the handwriting examination of the affidavit. RTC granted respondents’ motion and disqualified PO2
Alvarez as a witness. Petitioners elevated the case before the CA by way of petition for certiorari. CA
denied the petition.

ISSUE: Whether or not expert’s testimony who is not authorized by the trial court is disqualified from
being a witness.

RULING: No. A witness must only possess all the qualifications and none of the disqualifications
provided in the Rules of Court. Section 20, Rule 130 of the Rules on Evidence provides the qualifications
of a witness, i.e., all persons who can perceive, and perceiving, can make known their perception to
others, may be witnesses.

The disqualifications are as follows: (1) Section 19, Rule 130 disqualifies those who are mentally
incapacitated and children whose tender age or immaturity renders them incapable of being witnesses; (2)
Section 20 of the same rule provides for disqualification based on conflicts of interest or on relationship;
(3) Section 21 provides for disqualification based on privileged communications; and (4) Section 15 of
Rule 132 may not be a rule on disqualification of witnesses but it states the grounds when a witness may
be impeached by the party against whom he was called. The specific enumeration of disqualified
witnesses excludes the operation of causes of disability other than those mentioned in the Rules.

As a handwriting expert of the PNP, PO2 Alvarez can surely perceive and make known her perception to
others. We have no doubt that she is qualified as a witness. She cannot be disqualified as a witness since
she possesses none of the disqualifications specified under the Rules. Respondents’ motion to disqualify
her should have been denied by the RTC for it was not based on any of these grounds for disqualification.

The RTC rather confused the qualification of the witness with the credibility and weight of her testimony.
Moreover, Section 49, Rule 130 of the Rules of Evidence is clear that the opinion of an expert witness
may be received in Evidence.

Remedial Law Review II (CY 2018-2019) – Saturday 1:00pm – 5:00pm: Case Digest Compilation [Atty. Brondial] Page 373
C.3.c.i.1 PEOPLE vs GOLIMLIM

G.R. NO. 145225; April 02, 2004

FACTS:A mental retardate is not per se disqualified from testifying. AAA is a mental retardate. When
her mother Amparo Hachero (Amparo) left for Singapore to work, she entrusted Evelyn to the care and
custody of her sister Jovita Guban (Jovita) and the latter‘s husband appellantSalvador. In August 1996,
Jovita left the house to meet someone, leaving Evelyn with Golimlim. Taking advantage of the situation,
Golimlim instructed AAA to sleep, and soon after she had laid down, he kissed her and took off her
clothes. As he poked at her an object which to Evelyn felt like a knife, he proceeded to insert his penis
into her vagina. When Jovita arrived, AAA told her about what Golimlim did to her. Jovita, however, did
not believe her. Lorna Hachero (Lorna), AAA‘s half-sister, received a letter from their mother Amparo
instructing her to fetch Evelyn from Sorsogon and allow her to stay in Quezon City. Dutifully, Lorna
immediately proceeded to Golimlim‘s home and brought Evelyn with her to Manila. A week after she
brought Evelyn to stay with her, Lorna suspected that her sister was pregnant as she noticed her growing
belly. She thereupon brought her to a doctor for check-up and ultrasound examination. The examinations
revealed that Evelyn was indeed pregnant. She thus asked her sister how she became pregnant, to
which Evelynreplied that appellant had sexual intercourse with her while holding a knife. The Regional
Trial Court (RTC) of Sorsogon convicted Golimlim of the crime of rape. On appeal, the Court
of Appeals affirmed the conviction.

ISSUE:Whether or not the testimony of a mental retardate should be given weight and credence

RULING: The trial judge‘s assessment of the credibility of witnesses‘ testimonies is accorded great


respect on appeal in the absence of grave abuse of discretion on its part, it having had the advantage of
actually examining both real and testimonial evidence including the demeanor of the witnesses. A mental
retardate or a feebleminded person is not, per se, disqualified from being a witness, her mental condition
not being a vitiation of her credibility. It is now universally accepted that intellectual weakness, no matter
what form it assumes, is not a valid objection to the competency of a witness so long as the latter can still
give a fairly intelligent and reasonable narrative of the matter testified to. In the present case,
no cogent reason can be appreciated to warrant a departure from the findings of the trial court with
respect to the assessment of Evelyn‘s testimony. It is settled that sexual intercourse with a woman who is
a mental retardate constitutes statutory rape which does not require proof that the accused used force or
intimidation in having carnal knowledge of the victim for conviction. The fact of Evelyn‘s mental
retardation was not, however, alleged in the Information and, therefore, cannot be the basis
for conviction. Such notwithstanding, that force and intimidation attended the commission of the crime,
the mode of commission alleged in the Information, was adequately proven. It bears stating herein that
the mental faculties of a retardate being different from those of a normal person, the degree of force
needed to overwhelm him or her is less. Hence, a quantum of force which may not suffice when the
victim is a normal person may be more than enough when employed against an imbecile.

Remedial Law Review II (CY 2018-2019) – Saturday 1:00pm – 5:00pm: Case Digest Compilation [Atty. Brondial] Page 374
C.3.c.ii.1 MAXIMO ALVAREZ vs. SUSAN RAMIREZ

G.R. No. 143439; October 14, 2005; Sandoval-Gutierrez, J.

Digested by: Jesus Fernan Montemayor

FACTS: Susan Ramirez (Susan) is the complaining witness in an arson case pending before RTC –
Malabon City where the accused therein is Maximo Alvarez (Maximo) who is the husband of Esperanza
G. Alvarez (Esperanza), sister of Susan.

During the trial, Esperanza was called to the witness stand as a prosecution witness against her husband
Maximo which was not then objected to by the defense. Her testimony was offered to prove, among
others, that Maximo committed the crime of arson and that her sister and the estranged wife of the
accused, Esperanza, was living with her in the house which was set on fire by Maximo.

On direct examination, Esperanza testified that Maximo is her husband. And in the course thereof,
Maximo showed “uncontrolled emotions” prompting the Court to suspend the proceedings. Thereafter,
the accused filed a motion to disqualify Esperanza from testifying against him pursuant to Rule 130 on
marital disqualification. The court granted the said motion and disqualified Esperanza from further
testifying and deleting her testimony from the records.

As a consequence, Susan filed a petition for certiorari with the Court of Appeals assailing the order of
disqualification. The appellate court rendered a decision nullifying and setting aside the assailed order.
Hence, the present petition for review on certiorari.

ISSUE: WON the estranged wife can testify against her husband in a criminal proceeding for arson?

RULING: YES. Like all other general rules, the marital disqualification rule has its own exceptions,
both in civil actions between the spouses and in criminal cases for offenses committed by one against the
other. Thus, where the marital and domestic relations are so strained that there is no more harmony to be
preserved nor peace and tranquility which may be disturbed, the reason based upon such harmony and
tranquility fails. In such a case, identity of interests disappears and the consequent danger of perjury
based on that identity is non-existent. Likewise, in such a situation, the security and confidences of
private life, which the law aims at protecting, will be nothing but ideals, which through their absence,
merely leave a void in the unhappy home.

In the present case, the offense of arson attributed to Maximo, directly impairs the conjugal relation
between him and his wife Esperanza. His act, as embodied in the Information for arson filed against him,
eradicates all the major aspects of marital life such as trust, confidence, respect and love by which virtues
the conjugal relationship survives and flourishes.

Further, records show that prior to the commission of the offense, the relationship between Maximo and
his wife was already strained. In fact, they were separated de facto almost six months before the incident.
Indeed, the evidence and facts presented reveal that the preservation of the marriage between Maximo
and Esperanza is no longer an interest the State aims to protect.

Finally, it bears emphasis that the State, being interested in laying the truth before the courts so that the
guilty may be punished and the innocent exonerated, must have the right to offer the direct testimony of
Esperanza, even against the objection of the accused, because (as stated by this Court in a case “it was the
latter himself who gave rise to its necessity.”

Hence, in view of the foregoing, Esperanza is allowed to testify against her estranged husband Maximo as
an exception to the marital disqualification rule.

Remedial Law Review II (CY 2018-2019) – Saturday 1:00pm – 5:00pm: Case Digest Compilation [Atty. Brondial] Page 375
C.3.c.ii.2 PEOPLE OF THE PHILIPPINES, Petitioner, vs. THE HONORABLE JUANITO C.
CASTANEDA, JR., HONORABLE CAESAR A. CASANOVA, HONORABLE CIELITO N.
MINDARO-GRULLA, AS ASSOCIATE JUSTICES OF THE SPECIAL SECOND DIVISION,
COURT OF TAX APPEALS; and MYRNA M. GARCIA AND CUSTODIO MENDOZA
VESTIDAS, JR., Respondents.

G.R. No. 208290; December 11, 2013

FACTS: Myrna M. Garcia and Custodio Mendoza Vestidas, Jr. as owner/proprietress and broker of
Plinth Enterprise were charged with violation of the Tariff and Customs Code of the Philippines when
they imported into the Port of Manila, 858 cartons containing 17,160 pieces of Anti-Virus Software
Kaspersky Internet Security Premium 2012, and falsely declaring them to contain 40 pallets/1,690 cartons
of CD kit cleaner and plastic CD case, said imported items (Kaspersky) having customs duties amounting
to Three Million Three Hundred Forty One Thousand Two Hundred Forty Five Pesos (Php 3,341,245) of
which only the amount of One Hundred Thousand Three Hundred Sixty Two Pesos (Php100,362) was
paid.

Garcia and Vestidas, Jr. filed their Omnibus Motion to File Demurrer to Evidence with Leave of Court to
Cancel Hearing Scheduled on January 21, 2013,which was granted by the CTA. Thereafter, they filed the
Demurrer to Evidence, dated January 13, 2012, claiming that the prosecution failed to prove their guilt
beyond reasonable doubt.

On July 24, 2013, the Run After the Smugglers (RATS) Group, Revenue Collection Monitoring Group
(RCMG), as counsel for the BOC, received a copy of the July 15, 2013 Resolution of the CTA ordering
the entry of judgment in the case.

The BOC then filed a petition for certiorari, ascribing grave abuse of discretion on the part of the CTA.

At the outset, it should be noted that the petition was filed beyond the reglementary period for the filing
thereof under Rule 65. The petition itself stated that a copy of the May 15, 2013 Resolution was received
by the BOC two (2) days after its promulgation, or on May 17, 2013. Nonetheless, the RATS was only
alerted by the developments in the case on July 24, 2013, when Atty. Danilo M. Campos Jr. (Atty.
Campos) received the July 15, 2013 Resolution of the CTA ordering the entry of judgment in the case,
considering that no appeal was taken by any of the parties. According to Atty. Campos, it was only on
that occasion when he discovered the May 15, 2013 Resolution of the CTA.

ISSUE:Whether the petition for certiorari should be given due course despite its late filing.

RULING: No, Section 4, Rule 65 of the 1997 Rules of Civil Procedure is explicit in stating that certiorari
should be instituted within a period of 60 days from notice of the judgment, order or resolution sought to
be assailed. The 60-day period is inextendible to avoid any unreasonable delay that would violate the
constitutional rights of parties to a speedy disposition of their case. While there are recognized exceptions
to such strict observance, there should be an effort on the part of the party invoking liberality to advance a
reasonable or meritorious explanation for his/her failure to comply with the rules.

In the case at bench, no convincing justification for the belated filing of the petition was advanced to
warrant the relaxation of the Rules. Notably, the records show that the petition was filed only on August
12, 2013, or almost a month late from the due date which fell on July 16, 2013. To excuse this grave
procedural lapse will not only be unfair to the other party, but it will also sanction a seeming rudimentary
attempt to circumvent standing rules of procedure. Suffice it to say, the reasons proffered by the petitioner
do not carry even a tinge of merit that would deserve leniency. The late filing of the petition was borne
out of the petitioner’s failure to monitor incoming court processes that needed to be addressed by the
office. Clearly, this is an admission of inefficiency, if not lack of zeal, on the part of an office tasked to
effectively curb smuggling activities which rob the government of millions of revenue every year.

The display of patent violations of even the elementary rules leads the Court to suspect that the case
against Garcia and Vestidas Jr. was doomed by design from the start. The failure to present the certified
true copies of documentary evidence; the failure to competently and properly identify the misdeclared
goods; the failure to identify the accused in court; and, worse, the failure to file this petition on time
challenging a judgment of acquittal, are tell-tale signs of a reluctant and subdued attitude in pursuing the
case. This stance taken by the lawyers in government service rouses the Court’s vigilance against
inefficiency in the administration of justice.

Remedial Law Review II (CY 2018-2019) – Saturday 1:00pm – 5:00pm: Case Digest Compilation [Atty. Brondial] Page 376
Even the error committed by the RATS in filing a motion for reconsideration with the CTA displays gross
ignorance as to the effects of an acquittal in a criminal case and the constitutional proscription on double
jeopardy. Had the RATS been eager and keen in prosecuting the respondents, it would have, in the first
place, presented its evidence with the CTA in strict compliance with the Rules.

In any case, even if the Court decides to suspend the rules and permit this recourse, the end result would
remain the same. While a judgment of acquittal in a criminal case may be assailed in a petition for
certiorari under Rule 65 of the Rules of Court,it must be shown that there was grave abuse of discretion
amounting to lack or excess of jurisdiction or a denial of due process.In this case, a perusal of the
challenged resolutions of the CTA does not disclose any indication of grave abuse of discretion on its
partor denial of due process.The records are replete with indicators that the petitioner actively participated
during the trial and, in fact, presented its offer of evidence and opposed the demurrer.

Grave abuse of discretion is defined as capricious or whimsical exercise of judgment as is equivalent to


lack of jurisdiction. The abuse of discretion must be patent and gross as to amount to an evasion of a
positive duty or a virtual refusal to perform a duty enjoined by law, or to act at all in contemplation of
law, as where the power is exercised in an arbitrary and despotic manner by reason of passion and
hostility. Here, the subject resolutions of the CTA have been issued in accordance with the rules on
evidence and existing jurisprudence.

DOCTRINE: Section 4, Rule 65 of the 1997 Rules of Civil Procedure is explicit in stating that certiorari
should be instituted within a period of 60 days from notice of the judgment, order or resolution sought to
be assailed. The 60-day period is inextendible to avoid any unreasonable delay that would violate the
constitutional rights of parties to a speedy disposition of their case.

Remedial Law Review II (CY 2018-2019) – Saturday 1:00pm – 5:00pm: Case Digest Compilation [Atty. Brondial] Page 377
C.3.c.iii.1 Razon vs. CA

ENRIQUE RAZON, petitioner, vs. INTERMEDIATE APPELLATE COURT and VICENTE B.


CHUIDIAN, in his capacity as Administrator of the Estate of the Deceased JUAN T. CHUIDIAN,
respondents.
G.R. No. 74315, March 16, 1992

VICENTE B. CHUIDIAN, petitioner, vs. INTERMEDIATE APPELLATE COURT, ENRIQUE


RAZ0N, and E. RAZON, INC., respondents.

FACTS:
- Enrique Razon organized the E. Razon, Inc. for the purpose of bidding for the arrastre services in South
Harbor, Manila
- stock certificate No. 003 for 1,500 shares of stock of defendant corporation was issued in the name of
Juan T. Chuidian
- On the basis of the 1,500 shares of stock, the late Juan T. Chuidian and after him, the plaintiff-appellant,
were elected as directors of E. Razon, Inc
- From the time the certificate of stock was issued on April 1966 up to April 1971, Enrique Razon had not
questioned the ownership by Juan T. Chuidian of the shares of stock in question and had not brought any
action to have the certificate of stock over the said shares cancelled.
- The certificate of stock was in the possession of defendant Razon who refused to deliver said shares to
the plaintiff, until the same was surrendered by defendant Razon and deposited in a safety box in
Philippine Bank of Commerce.
- Thus, the 1,500 shares of stook under Stock Certificate No. 003 were delivered by the late Chuidian to
Enrique because it was the latter who paid for all the subscription on the shares of stock in the defendant
corporation and the understanding was that he (defendant Razon) was the owner of the said shares of
stock and was to have possession thereof until such time as he was paid therefor by the other nominal
incorporators/stockholders
-In G.R. No. 74306, petitioner Enrique Razon assails the appellate court's decision on its alleged
misapplication of the dead man's statute rule under Section 20(a) Rule 130 of the Rules of Court.
According to him, the "dead man's statute" rule is not applicable to the instant case. Moreover, the private
respondent, as plaintiff in the case did not object to his oral testimony regarding the oral agreement
between him and the deceased Juan T. Chuidian that the ownership of the shares of stock was actually
vested in the petitioner unless the deceased opted to pay the same

ISSUE: Whether or not the petitioner's testimony is within the prohibition under Dead Man’s Statute.

RULING: No. The case was not filed against the administrator of the estate, nor was it filed upon claims
against the estate.

-Sec. 20. Disqualification by reason of interest or relationship — The following persons cannot testify as
to matters in which they are interested directly or indirectly, as herein enumerated.
(a) Parties or assignors of parties to a case, or persons in whose behalf a case is prosecuted, against an
executoror administrator or other representative of a deceased person, or against a person of unsound
mind, upon a claim or demand against the estate of such deceased person or against such person of
unsound mind, cannot testify as to any matter of fact accruing before the death of such deceased person or
before such person became of unsound mind." (Emphasis supplied)
- The purpose of the law is to "guard against the temptation to give false testimony in regard to the
transaction in question on the part of the surviving party."
- The rule, however, delimits the prohibition it contemplates in that it is applicable to a case against the
administrator or its representative of an estate upon a claim against the estate of the deceased person.

-In the instant case, the testimony excluded by the appellate court is that of the defendant (petitioner
herein) to the affect that the late Juan Chuidian, (the father of private respondent Vicente Chuidian, the
administrator of the estate of Juan Chuidian) and the defendant agreed in the lifetime of Juan Chuidian
that the 1,500 shares of stock in E. Razon, Inc. are actually owned by the defendant unless the deceased
Juan Chuidian opted to pay the same which never happened. The case was filed by the administrator of
the estate of the late Juan Chuidian to recover shares of stock in E. Razon, Inc. allegedly owned by the
late Juan T. Chuidian.
-It is clear, therefore, that the testimony of the petitioner is not within the prohibition of the rule. The case
was not filed against the administrator of the estate, nor was it filed upon claims against the estate.

Remedial Law Review II (CY 2018-2019) – Saturday 1:00pm – 5:00pm: Case Digest Compilation [Atty. Brondial] Page 378
-Furthermore, the records show that the private respondent never objected to the testimony of the
petitioner as regards the true nature of his transaction with the late elder Chuidian. The petitioner's
testimony was subject to cross-examination by the private respondent's counsel. Hence, granting that the
petitioner's testimony is within the prohibition of Section 20(a), Rule 130 of the Rules of Court, the
private respondent is deemed to have waived the rule.

DOCTRINE: TESTIMONIAL EVIDENCE – DISQUALIFICATION – DEAD MAN’S STATUTE


Section 20(a) Rule 130 of the Rules of Court (Section 23 of the Revised Rules on Evidence) States:
Sec. 20.Disqualification by reason of interest or relationship — The following persons cannot testify as to
matters in which they are interested directly or indirectly, as herein enumerated.
(a) Parties or assignors of parties to a case, or persons in whose behalf a case is prosecuted, against an
executoror administrator or other representative of a deceased person, or against a person of unsound
mind, upon a claim or demand against the estate of such deceased person or against such person of
unsound mind, cannot testify as to any matter of fact accruing before the death of such deceased person
or before such person became of unsound mind." (Emphasis supplied)

Remedial Law Review II (CY 2018-2019) – Saturday 1:00pm – 5:00pm: Case Digest Compilation [Atty. Brondial] Page 379
C.3.c.iii.2 LILIBETH SUNGA-CHAN and CECILIA SUNGA, petitioners, vs.LAMBERTO T.
CHUA, respondent.

G.R. No. 143340; August 15, 2001; GONZAGA-REYES, J.:

FACTS:Lamberto T. Chua (hereafter respondent) filed a complaint against Lilibeth Sunga Chan
(hereafter petitioner Lilibeth) and Cecilia Sunga (hereafter petitioner Cecilia), daughter and wife,
respectively of the deceased Jacinto L. Sunga (hereafter Jacinto), for "Winding Up of Partnership Affairs

Respondent alleged that, he verbally entered into a partnership with Jacinto in the distribution of Shellane
Liquefied Petroleum Gas (LPG) in Manila. For business convenience, respondent and Jacinto allegedly
agreed to register the business name of their partnership, SHELLITE GAS APPLIANCE CENTER
(hereafter Shellite), under the name of Jacinto as a sole proprietorship. Respondent allegedly delivered his
initial capital contribution of P100,000.00 to Jacinto while the latter in turn produced P100,000.00 as his
counterpart contribution, with the intention that the profits would be equally divided between them. The
partnership allegedly had Jacinto as manager, assisted by Josephine Sy (hereafter Josephine), a sister of
the wife respondent, Erlinda Sy

While Jacinto furnished respondent with the merchandise inventories, balance sheets and net worth of
Shellite, respondent however suspected that the amount indicated in these documents were understated
and undervalued by Jacinto and Josephine for their own selfish reasons and for tax avoidance.

Upon Jacinto's death in the later part of 1989, his surviving wife, petitioner Cecilia and particularly his
daughter, petitioner Lilibeth, took over the operations, control, custody, disposition and management of
Shellite without respondent's consent. Despite respondent's repeated demands upon petitioners for
accounting, inventory, appraisal, winding up and restitution of his net shares in the partnership,
petitioners failed to comply. Petitioner Lilibeth allegedly continued the operations of Shellite, converting
to her own use and advantage its properties.

Trial court rendered its Decision against the defendants DIRECTING them to render an accounting and to
wind up the affairs of the partnership and terminate its business activities

CA affirmed.

Petitioners invoke the “Dead Man's Statute”. Petitioners question the finding of the trial court and the
Court of Appeals that a partnership existed between respondent and Jacinto (In the absence of any written
document to show such partnership between respondent and Jacinto)

ISSUE: Whether or not the Dead Man’s Statute applies to this case so as to render inadmissible
Lamberto’s testimony and that of his witness Josephine.

RULING: No. The "Dead Man's Statute" provides that if one party to the alleged transaction is precluded
from testifying by death, insanity, or other mental disabilities, the surviving party is not entitled to the
undue advantage of giving his own uncontradicted and unexplained account of the transaction.

Two reasons forestall the application of the "Dead Man's Statute" to this case.

First, petitioners filed a compulsory counterclaim against respondents in their answer before the trial
court, and with the filing of their counterclaim, petitioners themselves effectively removed this case from
the ambit of the "Dead Man's Statute”. Well entrenched is the rule that when it is the executor or
administrator or representatives of the estates that sets up the counterclaim, the plaintiff, herein
respondent, may testify to occurrences before the death of the deceased to defeat the counterclaim.
Moreover, as defendant in the counterclaim, respondent is not disqualified from testifying as to matters of
facts occurring before the death of the deceased, said action not having been brought against but by the
estate or representatives of the deceased

Second, the testimony of Josephine is not covered by the "Dead Man's Statute" for the simple reason that
she is not "a party or assignor of a party to a case or persons in whose behalf a case is prosecuted."
Records show that respondent offered the testimony of Josephine to establish the existence of the
partnership between respondent and Jacinto. Petitioners' insistence that Josephine is the alter ego of
respondent does not make her an assignor because the term "assignor" of a party means "assignor of a

Remedial Law Review II (CY 2018-2019) – Saturday 1:00pm – 5:00pm: Case Digest Compilation [Atty. Brondial] Page 380
cause of action which has arisen, and not the assignor of a right assigned before any cause of action has
arisen." Plainly then, Josephine is merely a witness of respondent, the latter being the party plaintiff.

DOCTRINE:"SEC. 23. Disqualification by reason of death or insanity of adverse party. – Parties or


assignors of parties to a case, or persons in whose behalf a case is prosecuted, against an executor or
administrator or other representative of a deceased person, or against a person of unsound mind, upon a
claim or demand against the estate of such deceased person, or against such person of unsound mind,
cannot testify as to any matter of fact occurring before the death of such deceased person or before such
person became of unsound mind."

The "Dead Man's Statute" provides that if one party to the alleged transaction is precluded from
testifying by death, insanity, or other mental disabilities, the surviving party is not entitled to the undue
advantage of giving his own uncontradicted and unexplained account of the transaction.But before this
rule can be successfully invoked to bar the introduction of testimonial evidence, it is necessary that:
"1. The witness is a party or assignor of a party to case or persons in whose behalf a case in prosecuted.
2. The action is against an executor or administrator or other representative of a deceased person or a
person of unsound mind;
3. The subject-matter of the action is a claim or demand against the estate of such deceased person or
against person of unsound mind;
4. His testimony refers to any matter of fact of which occurred before the death of such deceased person
or before such person became of unsound mind."

Remedial Law Review II (CY 2018-2019) – Saturday 1:00pm – 5:00pm: Case Digest Compilation [Atty. Brondial] Page 381
C.3.c.iii.3 Bordalba vs. CA

FACTS:The instant controversy stemmed from Lot No. 1242 (Lot No. 799-C) with an area of 1,853
square meters and located at Barrio Looc, Mandaue City. The subject lot is part of a parcel of land
situated on the corner of Mabini and Plaridel Streets in Mandaue City, and originally owned by the late
spouses Carmeno Jayme and Margarita Espina de Jayme. In 1947, an extra-judicial partition, written in
the Spanish language was executed, describing said parcel of land which disposed the same as follows:
1) 1/3 in favor of - (a) their grandchild Nicanor Jayme, the deceased spouse of private respondent
Candida Flores and the father of private respondents Emmanuel, Dina, Evelia and Gesila, all
surnamed Jayme; and (b) their grandchild Asuncion Jayme-Baclay, whose heirs are private
respondents Agelio Baclay, Elnora Baclay and Carmen Jayme-Daclan;
2) 1/3 to their daughter Elena Jayme Vda. de Perez, mother of petitioner Teresita P. Bordalba; and
3) 1/3 to an unidentified party.

Built on the land adjudicated to the heirs of the spouses is Nicanor Jayme’s house, which his family
occupied since 1945.

In 1964, Elena Jayme Vda. de Perez, petitioner’s mother, filed with the RTC of Cebu, an amended
application for the registration of the lot described with the boundaries. Elena Jayme Vda. de Perez
alleged that the lot sought to be registered was originally a part of a land owned by her late parents, the
spouses Carmeno and Margarita Jayme; and that 1/3 of said land was adjudicated to her in an extra-
judicial partition. She further stated that a portion of the lot for which title is applied for is occupied by
Nicanor Jayme with her permission.

Nicanor Jayme and Asuncion Jayme-Baclay filed their opposition contending that said application
included the 1/3 portion inherited by them in the 1947 extra-judicial partition. The case was, however,
dismissed for lack of interest of the parties.

In 1979, petitioner filed with the Bureau of Lands of Cebu City an application seeking the issuance of a
Free Patent over the same lot subject of the aborted application of her mother, Elena Jayme, now known
as Lot No. 1242 (799-C). Petitioner was successfully granted Free Patent No. (VII-I) 11421 and OCT No.
0-571 (FP) over said lot. Thereafter, petitioner caused the subdivision and titling of Lot No. 1242 (799-
C), into 6 lots, as well as the disposition of two parcels thereof.

Upon learning of the issuance in favor of petitioner of the aforesaid Free Patent and OCT over Lot No.
1242, as well as the conveyances made by petitioner involving the lot subject of the controversy, private
respondents filed with the RTC of Mandaue City, a complaint against petitioner Teresita Bordalba,
spouses Genaro and Rita Cabahug, Rural Bank of Mandaue and the Director of the Bureau of Lands.

In the said complaint, private respondents prayed that the Free Patent and OCT, as well as TCT Nos.
22771-22776 be declared void and ordered cancelled. Private respondents also prayed that they be
adjudged owners of Lot No. 1242 (799-C), and that spouses Cabahug as well as the Rural Bank of
Mandaue be declared buyers and mortgagee in bad faith, respectively.

Petitioner, on the other hand, averred that Lot No. 1242 (799-C) was acquired by her through purchase
from her mother, who was in possession of the lot in the concept of an owner since 1947. In her answer,
petitioner traced her mother’s ownership of the lot partly from the 1947 deed of extra-judicial partition
presented by private respondents, and claimed that Nicanor Jayme, and Candida Flores occupied a portion
of Lot No. 1242 (799-C) by mere tolerance of her mother. On cross-examination, petitioner admitted that
the properties of the late Carmeno and Margarita Jayme were partitioned by their heirs in 1947, but
claimed that she was not aware of the existence of said Deed of Extra-judicial Partition. She, however,
identified one of the signatures in the said Deed to be the signature of her mother.

The trial court found that fraud was employed by petitioner in obtaining the Free Patent and OCT and
declared said patent and title void and ordered its cancellation. However, it declared that spouses Genaro
and Rita Cabahug as well as the Rural Bank of Mandaue are purchasers and mortgagee in good faith,
respectively; and consequently upheld as valid the sale of Lot No. 1242-A covered by TCT No. 22771
(FP) to spouses Cabahug, and the mortgage of Lot No. 1242-B covered by TCT No. 22772 in favor of the
Rural Bank of Mandaue.

Remedial Law Review II (CY 2018-2019) – Saturday 1:00pm – 5:00pm: Case Digest Compilation [Atty. Brondial] Page 382
Both parties appealed to the Court of Appeals, which affirmed with modification the decision of the trial
court. It ruled that since private respondents are entitled only to 1/3 portion of Lot No. 1242 (799-C),
petitioner should be ordered to reconvey 1/3 of Lot No. 1242 (799-C) to private respondents.

Petitioner filed the instant petition, assailing the decision of the CA. Petitioner contends that the
testimonies given by the witnesses for private respondents which touched on matters occurring
prior to the death of her mother should not have been admitted by the trial court, as the same
violated the dead man’s statute.

ISSUE: Whether the trial court erred in admitting the testimonies given by the witnesses for private
respondents in violation of the dead man’s statute.

RULING: No. As to the alleged violation of the dead man’s statute, suffice it to state that said rule finds
no application in the present case. The dead man’s statute does not operate to close the mouth of a witness
as to any matter of fact coming to his knowledge in any other way than through personal dealings with the
deceased person, or communication made by the deceased to the witness.

Since the claim of private respondents and the testimony of their witnesses in the present case is
based, inter alia, on the 1947 Deed of Extra-judicial Partition and other documents, and not on dealings
and communications with the deceased, the questioned testimonies were properly admitted by the trial
court.

DOCTRINE:The dead man’s statute does not operate to close the mouth of a witness as to any matter of
fact coming to his knowledge in any other way than through personal dealings with the deceased person,
or communication made by the deceased to the witness.

Remedial Law Review II (CY 2018-2019) – Saturday 1:00pm – 5:00pm: Case Digest Compilation [Atty. Brondial] Page 383
C.3.c.iv.1 Josielene Lara Chan vs. Johnny T. Chan

G.R. no. 179786; July 24, 2013; Ponente: J. Abad

FACTS: Josielene filed a petition for the declaration of nullity of her marriage to respondent Johnny
Chan. The petitioner claimed that Johnny failed to care for and support his family and that a psychiatrist
diagnosed him as mentally deficient due to incessant drinking and excessive use of prohibited drugs. She
also alleged that she convinced Johnny to undergo hospital confinement for detoxification and
rehabilitation.

Johnny answered that it was Josielene who failed in her wifely duties. To save their marriage, he agreed
to marriage counseling but when he and Josielene got to the hospital, two men forcibly held him by both
arms while another gave him an injection. By then, their marriage relationship could no longer be
repaired.

During the pre-trial, Josielene pre-marked the Philhealth Claim Form  that Johnny attached to his answer
as proof that he was forcibly confined at the rehabilitation unit of a hospital. The form carried a
physician’s handwritten note that Johnny suffered from methamphetamine and alcohol abuse. On August
22, 2006 Josielene filed with the RTC a request for the issuance of a subpoena duces tecum addressed to
Medical City, covering Johnny’s medical records when he was there confined. The request was
accompanied by a motion to "be allowed to submit in evidence" the records sought by subpoena duces
tecum. Johnny opposed the motion, arguing that the medical records were covered by physician-patient
privilege. The RTC sustained the opposition and denied Josielene’s motion.

On appeal, CA ruled that if courts were to allow the production of medical records, then patients would
be left with no assurance that whatever relevant disclosures they may have made to their physicians
would be kept confidential. The prohibition covers not only testimonies, but also affidavits, certificates,
and pertinent hospital records. The CA added that, although Johnny can waive the privilege, he did not do
so in this case. He attached the Philhealth form to his answer for the limited purpose of showing his
alleged forcible confinement.

ISSUE: whether or not CA erred in sustaining the decision of court a quo, covering Johnny’s hospital
records on the ground that these are covered by the privileged character of the physician-patient
communication?

RULING: NO, under Section 24(c) Rule 130 of ROC, the physician-patient privileged communication
rule essentially means that a physician who gets information while professionally attending a patient
cannot in a civil case be examined without the patient’s consent as to any facts which would blacken the
latter’s reputation. This rule is intended to encourage the patient to open up to the physician, relate to him
the history of his ailment, and give him access to his body, enabling the physician to make a correct
diagnosis of that ailment and provide the appropriate cure. Any fear that a physician could be compelled
in the future to come to court and narrate all that had transpired between him and the patient might
prompt the latter to clam up, thus putting his own health at great risk.

To allow the disclosure during discovery procedure of the hospital records—the results of tests that the
physician ordered, the diagnosis of the patient’s illness, and the advice or treatment he gave him—would
be to allow access to evidence that is inadmissible without th patient’s consent. Physician memorializes
all these information in the patient’s records. Disclosing them would be the equivalent of compelling the
physician to testify on privileged matters he gained while dealing with the patient, without the latter’s
prior consent.

Also, Josielene’s request for subpoena duces tecum is premature. She will have to wait for trial to begin
before making a request for the issuance of a subpoena duces tecum covering Johnny’s hospital records. It
is when those records are produced for examination at the trial, that Johnny may opt to object, not just to
their admission in evidence, but more so to their disclosure. The trial in the case had not yet begun.
Consequently, it cannot be said that Johnny had already presented the Philhealth claim form in evidence,
the act contemplated above which would justify Josielene into requesting an inquiry into the details of his
hospital confinement. Johnny was not yet bound to adduce evidence in the case when he filed his answer.
Any request for disclosure of his hospital records would again be premature.

Remedial Law Review II (CY 2018-2019) – Saturday 1:00pm – 5:00pm: Case Digest Compilation [Atty. Brondial] Page 384
C.3.c.iv.2 JUDGE UBALDINO A. LACUROM, Presiding Judge, Regional Trial Court,
Cabanatuan City, Branch 29 and Pairing Judge, Branch 30 vs. ATTY. ELLIS F. JACOBA and
ATTY. OLIVIA VELASCO-JACOBA,

FACTS: Contempt judgment was rendered against Atty. Velasco-Jacoba by Judge Lacurom for filing a
motion with disrespectful contents. Atty. Velasco Jacoba moved for reconsideration and claimed that the
author of the said motion was her husband, Atty. Jacoba who was then suspended and therefore could not
sign the said motion.

Judge Lacorum then issued an order directing Atty. Jacoba to explain why he should not be cited for
contempt. Atty. Jacoba invoked the marital privilege rule in evidence.

ISSUE: Whether or not the marital privilege rule will apply to disregard the statement of Atty. Velasco
Jacoba against Atty. Jacoba.

RULING:

The marital privilege rule, being a rule of evidence, may be waived by failure of the claimant to object
timely to its presentation or by any conduct that may be construed as implied consent. This waiver applies
to Jacoba who impliedly admitted authorship of the 30 July 2001 motion.

The Code of Professional Responsibility provides:


Rule 11.03.—A lawyer shall abstain from scandalous, offensive or menacing language or
behavior before the Courts.
Rule 11.04.—A lawyer shall not attribute to a Judge motives not supported by the record or have
no materiality to the case.

No doubt, the language contained in the 30 July 2001 motion greatly exceeded the vigor required of
Jacoba to defend ably his client’s cause. We recall his use of the following words and phrases: abhorrent
nullity, legal monstrosity, horrendous mistake, horrible error, boner, and an insult to the judiciary and an
anachronism in the judicial process. Even Velasco-Jacoba acknowledged that the words created "a
cacophonic picture of total and utter disrespect."

The Jacobas nonetheless try to exculpate themselves by saying that every remark in the 30 July 2001
motion was warranted.

Remedial Law Review II (CY 2018-2019) – Saturday 1:00pm – 5:00pm: Case Digest Compilation [Atty. Brondial] Page 385
C.3.c.iv.3 CLARITA J. SAMALA vs. ATTY. LUCIANO D. VALENCIA

A.C. No. 5439; January 22, 2007

FACTS:Samala filed a complaint for disbarment against Atty. Luciano D. Valencia for serving on two
separate occasions as counsel for contending parties. Records show that Valencia acted as counsel for
Valdez in three separate cases. In of the cases Valencia represented Valdez and Alba against Bayuga and
Bustamante.

However, on a subsequent estafa case between Valdez and Alba, Valencia represented Valdez against
Alba his previous client. In his defense Valencia, respondent, avers that he already severed his
representation for Alba when the latter charged respondent with estafa.

ISSUE: Whether or not the termination of lawyer-client relationship allows a counsel to act as counsel
against a previous client.

RULING: The answer is in the negative. The termination of the relation of attorney and client provides
no justification for a lawyer to represent an interest adverse to or in conflict with that of the former client.
The reason for the rule is that the client's confidence once reposed cannot be divested by the expiration of
the professional employment. Consequently, a lawyer should not, even after the severance of the relation
with his client, do anything which will injuriously affect his former client in any matter in which he
previously represented him nor should he disclose or use any of the client's confidences acquired in the
previous relation.

The stern rule against representation of conflicting interests is founded on principles of public policy and
good taste. It springs from the attorney's duty to represent his client with undivided fidelity and to
maintain inviolate the client's confidence as well as from the injunction forbidding the examination of an
attorney as to any of the privileged communications of his client.

Remedial Law Review II (CY 2018-2019) – Saturday 1:00pm – 5:00pm: Case Digest Compilation [Atty. Brondial] Page 386
C.3.c.iv.4 COMMISSIONER JOSE T. ALMONTE, VILLAMOR C. PEREZ, NERIO ROGADO,
and ELISA RIVERA, petitioners, vs.HONORABLE CONRADO M. VASQUEZ and
CONCERNED CITIZENS, respondents.

G.R. No. 95367; May 23, 1995; MENDOZA, J.:

FACTS:This is a petition for certiorari, prohibition, and mandamus to annul the subpoena duces


tecum and orders issued by respondent Ombudsman, requiring petitioners Nerio Rogado and Elisa
Rivera, as chief accountant and record custodian, respectively, of the Economic Intelligence and
Investigation Bureau (EIIB) to produce "all documents relating to Personal Services Funds for the year
1988 and all evidence, such as vouchers (salary) for the whole plantilla of EIIB for 1988" and to enjoin
him from enforcing his orders.

Petitioner Jose T. Almonte was formerly Commissioner of the EIIB, while Villamor C. Perez is Chief of
the EIIB's Budget and Fiscal Management Division. The subpoena duces tecum was issued by the
Ombudsman in connection with his investigation of an anonymous letter alleging that funds representing
savings from unfilled positions in the EIIB had been illegally disbursed. The letter, purporting to have
been written by an employee of the EIIB and a concerned citizen, was addressed to the Secretary of
Finance, with copies furnished several government offices, including the Office of the Ombudsman. Both
petitioner Almonte nad Perez denied the allegations in the letter.

The Graft Investigation Officer of the Ombudsman's office, Jose F. Saño, found the comments
unsatisfactory, being "unverified and plying only on generalizations without meeting specifically the
points raised by complainant as constitutive of the alleged anomalies." 3 He, therefore, asked for authority
to conduct a preliminary investigation. Anticipating the grant of his request, he issued a subpoena 4 to
petitioners Almonte and Perez, requiring them to submit their counter-affidavits and the affidavits of their
witnesses, as well as a subpoena duces tecum 5 to the Chief of the EIIB's Accounting Division ordering
him to bring "all documents relating to Personal Services Funds for the year 1988 and all evidence, such
as vouchers (salary) for the whole plantilla of EIIB for 1988."

In his Order dated June 15, 1990, 6 respondent Ombudsman granted the motion to quash the subpoena in
view of the fact that there were no affidavits filed against petitioners. But he denied their motion to quash
the subpoena duces tecum. He ruled that petitioners were not being forced to produce evidence against
themselves, since the subpoena duces tecum was directed to the Chief Accountant, petitioner Nerio
Rogado. Petitioners’ MR was denied, hence, this petition.

Issue: Whether petitioners can be ordered to produce documents relating to personal services and salary
vouchers of EIIB employees on the plea that such documents are classified.

RULING: Yes
At common law a governmental privilege against disclosure is recognized with respect to state secrets
bearing on military, diplomatic and similar matters. This privilege is based upon public interest of such
paramount importance as in and of itself transcending the individual interests of a private citizen, even
though, as a consequence thereof, the plaintiff cannot enforce his legal rights. 10

On the other hand, where the claim of confidentiality does not rest on the need to protect military,
diplomatic or other national security secrets but on a general public interest in the confidentiality of his
conversations, courts have declined to find in the Constitution an absolute privilege of the President
against a subpoena considered essential to the enforcement of criminal laws. 

In the case at bar, there is no claim that military or diplomatic secrets will be disclosed by the production
of records pertaining to the personnel of the EIIB. Indeed, EIIB's function is the gathering and evaluation
of intelligence reports and information regarding "illegal activities affecting the national economy, such
as, but not limited to, economic sabotage, smuggling, tax evasion, dollar salting." 18 Consequently, while
in cases which involve state secrets it may be sufficient to determine from the circumstances of the case
that there is reasonable danger that compulsion of the evidence will expose military matters without
compelling production, 19 no similar excuse can be made for a privilege resting on other considerations.

Nor has our attention been called to any law or regulation which considers personnel records of the EIIB
as classified information. To the contrary, COA Circular No. 88-293, which petitioners invoke to support
their contention that there is adequate safeguard against misuse of public funds, provides that the

Remedial Law Review II (CY 2018-2019) – Saturday 1:00pm – 5:00pm: Case Digest Compilation [Atty. Brondial] Page 387
"only item of expenditure which should be treated strictly confidential" is that which refers to the
"purchase of information and payment of rewards."

On the other hand, the Ombudsman is investigating a complaint that several items in the EIIB were filled
by fictitious persons and that the allotments for these items in 1988 were used for illegal purposes. The
plantilla and other personnel records are relevant to his investigation. He and his Deputies are designated
by the Constitution "protectors of the people" and as such they are required by it "to act promptly on
complaints in any form or manner against public officials or employees of the Government, or any
subdivision, agency or instrumentality thereof, including government-owned or controlled corporation." 

His need for the documents thus outweighs the claim of confidentiality of petitioners. What is more,
while there might have been compelling reasons for the claim of privilege in 1988 when it was asserted
by petitioners, now, seven years later, these reasons may have been attenuated, if they have not in fact
ceased. The agents whose identities could not then be revealed may have ceased from the service of the
EIIB, while the covert missions to which they might have been deployed might either have been
accomplished or abandoned. On the other hand, the Ombudsman's duty to investigate the complaint that
there were in 1988 unfilled positions in the EIIB for which continued funding was received by its officials
and put to illegal use, remains.

Doctrine:At common law a governmental privilege against disclosure is recognized with respect to state
secrets bearing on military, diplomatic and similar matters. This privilege is based upon public interest
of such paramount importance as in and of itself transcending the individual interests of a private citizen,
even though, as a consequence thereof, the plaintiff cannot enforce his legal rights. 

Remedial Law Review II (CY 2018-2019) – Saturday 1:00pm – 5:00pm: Case Digest Compilation [Atty. Brondial] Page 388
SENATE OF THE PHILIPPINES vs. EDUARDO R. ERMITA
G.R. No. 169777; April 20, 2006

FACTS: Petition for certiorari and prohibition proffer that the President abused her power by issuing
E.O. 464 “Ensuring Observance of the Principles of Separation of Powers, Adherence to the Rule on
Executive Privilege and Respect for the Rights of Public Officials Appearing in Legislative Inquiries in
Aid of Legislation Under the Constitution, and for Other Purposes”.

Petitioners pray for such declaration as null and void for being unconstitutional. In the exercise of its
legislative power, the Senate of the Philippines, through its various Senate Committees, conducts
inquiries or investigations in aid of legislation which call for, inter alia, the attendance of officials and
employees of the executive department, bureaus, and offices including those employed in Government
Owned and Controlled Corporations, the Armed Forces of the Philippines (AFP), and the Philippine
National Police (PNP). The Committee of the Senate issued invitations to various officials of the
Executive Department for them to appear as resource speakers in a public hearing on the railway project,
others on the issues of massive election fraud in the Philippine elections, wire tapping, and the role of
military in the so-called “Gloriagate Scandal”. Said officials were not able to attend due to lack of consent
from the President as provided by E.O. 464, Section 3 which requires all the public officials enumerated
in Section 2(b) to secure the consent of the President prior to appearing before either house of Congress.

ISSUE: Whether Section 3 of E.O. 464, which requires all the public officials, enumerated in Section
2(b) to secure the consent of the President prior to appearing before either house of Congress, valid and
constitutional?

HELD: No.

The enumeration in Section 2 (b) of E.O. 464 is broad and is covered by the executive privilege. The
doctrine of executive privilege is premised on the fact that certain information must, as a matter of
necessity, be kept confidential in pursuit of the public interest. The privilege being, by definition, an
exemption from the obligation to disclose information, in this case to Congress, the necessity must be of
such high degree as to outweigh the public interest in enforcing that obligation in a particular case.
Congress undoubtedly has a right to information from the executive branch whenever it is sought in aid of
legislation. If the executive branch withholds such information on the ground that it is privileged, it must
so assert it and state the reason therefor and why it must be respected. The infirm provisions of E.O. 464,
however, allow the executive branch to evade congressional requests for information without need of
clearly asserting a right to do so and/or proffering its reasons therefor. By the mere expedient of invoking
said provisions, the power of Congress to conduct inquiries in aid of legislation is frustrated.

Remedial Law Review II (CY 2018-2019) – Saturday 1:00pm – 5:00pm: Case Digest Compilation [Atty. Brondial] Page 389
ROMULO L. NERI vs. SENATE COMMITTEE
G.R. No. 180643; September 4, 2008

FACTS: The Senate issued various Senate Resolutions directing the SBRC to conduct an investigation
regarding the NBN-ZTE deal, among others. Neri, the head of NEDA, was then invited to testify before
the Senate Blue Ribbon. He disclosed that the COMELEC Chairman Abalos offered him P200M in
exchange for his approval of the NBN Project, that he informed PGMA about the bribery and that she
instructed him not to accept the bribe. However, when probed further on what they discussed about the
NBN Project, he refused to answer, invoking “executive privilege”. In particular, he refused to answer the
questions on (a) whether or not President Arroyo followed up the NBN Project, (b) whether or not she
directed him to prioritize it,  and (c) whether or not she directed him to approve. As a result, the Senate
cited him for contempt.

ISSUE: Whether or not the communications elicited by the 3 questions covered by executive privilege.

RULING: The SC recognized the executive privilege which is the Presidential communications
privilege.  It pertains to “communications, documents or other materials that reflect presidential decision-
making and deliberations and that the President believes should remain confidential.” Presidential
communications privilege  applies to decision-making of the President. It is rooted in the constitutional
principle of separation of power and the President’s unique constitutional role. The claim of executive
privilege is highly recognized in cases where the subject of inquiry relates to a power textually committed
by the Constitution to the President, such as the area of military and foreign relations. The information
relating to these powers may enjoy greater confidentiality than others. Elements of presidential
communications privilege:

1)      The protected communication must relate to a “quintessential  and non-delegable presidential


power.” - i.e. the power to enter into an executive agreement with other countries. This authority of the
President to enter into executive agreements without the concurrence of the Legislature has traditionally
been recognized in Philippine jurisprudence.  
2)         The communication must be authored or “solicited and received” by a close advisor of the
President or the President himself.  The judicial test is that an advisor must be in “operational proximity”
with the President. 3)         The presidential communications privilege remains a qualified privilege that
may be overcome by a showing of adequate need, such that the information sought “likely contains
important evidence” and by the unavailability of the information elsewhere by an appropriate
investigating authority. - there is no adequate showing of a compelling need that would justify the
limitation of the privilege and of the unavailability of the information elsewhere by an appropriate
investigating authority. 

Remedial Law Review II (CY 2018-2019) – Saturday 1:00pm – 5:00pm: Case Digest Compilation [Atty. Brondial] Page 390
C.3.e.1 PEOPLE OF THE PHILIPPINES vs. HERMANOS CONSTANTINO, JR. y BINAYUG,
a.k.a. “JOJIT,"

G.R. No. 199689; March 12, 2014

FACTS: On January 20, 2005, at around 2:00 in the afternoon, Police Superintendent Mariano
Rodriguez, the Chief of Police of Tuguegarao City, received a report from a confidential informant (CI)
that a certain Jojit was selling illegal drugs in the said city. P/Supt. Rodriguez immediately formed a buy-
bust group. The buy-bust money, consisting of one ₱500.00 bill and five ₱100.00 bills, were dusted with
fluorescent powder and their respective serial numbers were recorded in the police blotter.
Around 8:00 in the evening of the same day, the team proceeded to Reynovilla St., Caritan Centro,
Tuguegarao City, the place where, according to the CI, Jojit was selling shabu. Thereafter, Constantino
was brought to the police station where the recovered drugs and money were turned over to the
investigator, SPO2 Tamang. The recovered drugs were then marked with the initials "A-1" and "A-2."
The incident was recorded in the police blotter with an inventory of the recovered drugs and money.
Later that evening, at around ten o’clock, P/Supt. Rodriguez and SPO2 Tamang submitted to the
Philippine National Police (PNP) Crime Laboratory Services, Camp Marcelo Adduru, Tuguegarao City, a
request for laboratory examination of two plastic sachets with white crystalline substance marked as "A-
1" and "A-2" to determine the presence of dangerous drugs; as well as both hands of Constantino, one
piece ₱500.00 bill, and five pieces ₱100.00 bills, to determine the presence of the ultra violet powder. Per
Chemistry Report No. D-08-200511 and Physical Identification Report No. PI-04-2005,12 prepared by
Police Senior Inspector (P/ SInsp.) Mayra Matote Madria,Forensic Chemist, the contents of the two
plastic sachets tested positive for Methamphetamine Hydrochloride; while the other specimens tested
positive for the presence of bright-yellow ultraviolet fluorescent powder.

Constantino denied the accusation against him and asserted that he was merely framed-up.
The following day, January 21, 2005, the police officers again brought Constantino to the PNP Crime
Laboratory. Along the way, one of the police escorts forced Constantino to hold a certain amount of
money. Constantino tried to resist but he could not really do anything because he was handcuffed. After
his examination, Constantino was detained and was told that he was suspected of selling shabu.
RTC found Constantino guilty as charged. It was affirmed by the Court of Appeals.

Constantino contests his conviction, averring inconsistencies in the testimonies of the prosecution
witnesses, particularly, on the circumstances of the marking of the two plastic sachets containing shabu
allegedly confiscated from him. Different people claim to have made the marking "NBT" on the two
plastic sachets and gave various explanations as to what the initials "NBT" stand for.

ISSUE: W/N the prosecution failed to establish a crucial link in the chain of custody of the shabu.

RULING: YES. The following links must be established in the chain of custody in a buy-bust situation:
first, the seizure and marking, if practicable, of the illegal drug recovered from the accused by the
apprehending officer; second, the turn over of the illegal drug seized by the apprehending officer to the
investigating officer; third, the turn over by the investigating officer of the illegal drug to the forensic
chemist for laboratory examination; and fourth, the turn over and submission of the marked illegal drugs
seized from the forensic chemist to the court. As held in People vs. Zakaria, crucial in proving the chain
of custody is the marking of the seized dangerous drugs or other related items immediately after they are
seized from the accused, for the marking upon seizure is the starting point in the custodial link that
succeeding handlers of the evidence will use as reference point. Moreover, the value of marking of the
evidence is to separate the marked evidence from the corpus of all other similar or related evidence from
the time of seizure from the accused until disposition at the end of criminal proceedings, obviating
switching, “planting” or contamination of evidence. A failure to mark at the time of taking of initial
custody imperils the integrity of the chain of custody that the law requires.

The failure of the prosecution to establish the evidence’s chain of custody is fatal to its case as the Court
can no longer consider or even safely assume that the integrity and evidentiary value of the confiscated
dangerous drug were properly preserved. In light of the foregoing, Constantino is acquitted of the crime
charged, not because the Court accords credence to his defense of frame-up, but because the prosecution
failed to discharge its burden of proving his guilt beyond reasonable doubt.

Remedial Law Review II (CY 2018-2019) – Saturday 1:00pm – 5:00pm: Case Digest Compilation [Atty. Brondial] Page 391
C.3.e.2 PEOPLE VS GANDIA

FACTS:
Appellant was accused of the crime of rape committed to Remelyn, a 3 year old girl.
The procesuction presented Amalia, the mother of the victim. The testimony of TulonMik, Remelyns
neighbor and a barangay kagawad in their area was also presented by the prosecution. Mik testified that
he saw appellant carrying Remelyn in his arms and they were on their way toward the ipil-ipil trees.
RTC found him guilty of the offense charged.

Appellant appealed contending, first, that TulonMiks testimony is weak, on the ground that Mik is a
relative of the husband of Amalia. He also questions the credibility of Mik because of his failure to
confront appellant when he saw him carrying Remelyn. Neither did Mik inform Amalia about what he
saw when Amalia was looking for Remelyn. Appellant insists that it was Daylen whom he carried and not
Remelyn. Second, he stresses the fact that Remelyn did not make any categorical statement that he
sexually molested her. Third, he maintains that the accusation of flight against him is false. Fourth, he
avers that the offer of compromise by his parents as tendered to Amalia Loyola should not be taken
against him, while the offer of compromise he allegedly made to Amalias husband, as relayed by Amalia
in her testimony, should be excluded as evidence for being hearsay. Finally, he submits that
inconsistencies in the testimony of Alex Loyola and Cabano should not be counted against him on the
ground that any finding of guilt must rest on the strength of the prosecutions evidence.

ISSUE: Whether or not the guilt of the accused for the crime of rape has been proven beyond reasonable
doubt

RULING: YES
Following the principle of res inter aliosactaalterinocere non debet, the actions of his parents cannot
prejudice the appellant, since he was not a party to the said conversation, nor was it shown that he was
privy to the offer of compromise made by them to the mother of the victim. They cannot be considered as
evidence against appellant. A witness can only testify on facts which are based on his personal knowledge
or perception
The court ruled to convict appellant for simple rape, and not for qualified rape.
First, appellants attempt to discredit the testimony of Mik cannot succeed. It is true that Mik is a relative
by affinity of Amalia Loyola. It is hoary jurisprudence, however, that mere relationship to one of the
parties, without a showing of any other improper motive, is not sufficient basis to impair the credibility of
the witness. In the case at bar, appellant cannot impute any ill motive for Mik to testify adversely against
him.

Next, appellant tried to capitalize on the fact that Remelyn never made any statement that he sexually
molested her. This is a specious argument. Remelyn had told her mother, Crazy Lendoy forced me.
Remelyn was 3 1/2 years old at the time. At such an infantile age, she could not be expected to have a
comprehension of the concept of rape.

Appellants charge that the offers of compromise allegedly made by the parents of the appellant to Amalia,
and by the appellant himself to Amalias husband should not have been taken against him by the trial
court, even if sustained, will not exculpate him. To be sure, the offer of compromise allegedly made by
appellant to AmaliaLoyolas husband is hearsay evidence, and of no probative value. It was only Amalia
who testified as to the alleged offer, and she was not a party to the conversation which allegedly
transpired at the Hagonoy Municipal Jail. The offer of compromise allegedly made by the appellants
parents to Amalia may have been the subject of testimony of Amalia. They cannot be considered as
evidence against appellant but we reiterate that these errors are not enough to reverse the conviction of the
appellant.

Remedial Law Review II (CY 2018-2019) – Saturday 1:00pm – 5:00pm: Case Digest Compilation [Atty. Brondial] Page 392
C.3.e.3 DOLDOL VS PEOPLE

FACTS: The team of State Auditors conducted various audit of the cash and cash account of Conrado C.
Doldol, the Municipal Treasurer of Urbiztondo, Pangasinan. The audit covered the General Fund, Special
Education Fund and Trust Fund in his custody. They noted that there is a shortage in the fund, the State
Auditors demanded the immediate refund of the said amount, and for Doldol to submit a written
explanation on the said shortage. Doldol failed to respond and was, thereafter, relieved of his duties. He
was directed to transfer the account to Assistant Municipal Treasurer LoidaCancino.

On the same day, Doldol wrote the Provincial Treasurer requesting that a re-audit be conducted on his
cash and cash account, taking exception to the findings of the State Auditors.

Instead of pursuing his request for a re-audit, Doldol opted to refund the missing funds. On September 15,
1995, he remitted P200,000.00 to the Acting Municipal Treasurer. Doldol promised to pay the balance of
his shortage, as follows: P200,000.00 on October 31, 1995, and P884,139.66 on or before November 30,
1995. However, he reneged on his promise.

ISSUE: WON the Doldol is guilty of the crime of Malversation of Public Funds

RULING: YES
As guided by Section 27, Rule 130 of the Rules on Evidence, payment, particularly when taken in
conjunction with appellants commitment to gradually pay the remainder of the missing funds, is a clear
offer of compromise which must be treated as an implied admission of appellants guilt that he embezzled
or converted the missing funds to his personal use

Except for his bare testimony, the petitioner offered no competent and credible evidence to prove that the
missing funds were actually cash advances of employees in the municipality. The petitioner could have
offered in evidence the documents evidencing the names of the recipients and amounts of the cash
advances, but failed to do so. Moreover, the petitioner wrote the Provincial Auditor and offered to refund
the missing funds as follows: P200,000.00 on September 15, 1995, P200,000.00 on or before October 31,
1995, and P884,139.66 on November 30, 1995. He was able to pay only P200,000.00 on September 15,
1995, and failed to remit the balance of his shortage. Such partial restitution of the petitioners of the cash
shortage is an implied admission of misappropriation of the missing funds.

Remedial Law Review II (CY 2018-2019) – Saturday 1:00pm – 5:00pm: Case Digest Compilation [Atty. Brondial] Page 393
C.3.e.4 Richard A. Cambe vs. Office of the Ombudsman, et al./Senator Ramon "Bong" Revilla, Jr.
Vs. Office of the Ombudsman, et al./Senator Ramon "Bong" Revilla, Jr. Vs. Office of the
Ombudsman, et al./Richard A. Cambe Vs. Office of the Ombudsman, et al./John Raymund De Asis
Vs. Conchita Carpio Morales, et al./Ronald John Lim Vs. Conchita Carpio Morales, et al./Janet
Lim Napoles Vs. Conchita Carpio Morales, et al./Mario L. Relampagos, et al. Vs. Sandiganbayan
and People of the Philippines

G.R. Nos. 212014-15/G.R. Nos. 212427-28/G.R. Nos. 212694-95/G.R. Nos. 213477-78/G.R. Nos.
213532-33/G.R. Nos. 213536-37/G.R. Nos. 218744-59; December 6, 2016

FACTS: Before this Court are consolidated petitions filed by petitioners Senator Ramon "Bong" Revilla,
Jr. (Sen. Revilla), Richard A. Cambe (Cambe), Janet Lim Napoles (Napoles or Janet Napoles), John
Raymund De Asis (De Asis), and Ronald John Lim (Lim), which commonly assail the Joint Resolution
dated March 28, 2014 and the Joint Order dated June 4, 2014 of the Office of the Ombudsman
(Ombudsman) in OMB-C-C-13-0316 and OMB-C-C-13-0395 finding probable cause to indict them,
along with several others, for the crimes of Plunder, defined and penalized under Section 2 in relation to
Section 1 ( d) (1 ), (2), and ( 6) of Republic Act No. (RA) 7080, as amended (one [1] count) and/or of
violation of Section 3 (e) of RA 30195 (sixteen [16] counts).

Petitioners are all charged as co-conspirators for their respective participations in the illegal pillaging of
public funds sourced from the Priority Development Assistance Fund (PD.AF) of Sen. Revilla for the
years 2006 to 2010, in the total amount of P517,000,000.00.

As alleged, the PDAF scheme commences with Napoles meeting with a legislator -in this case, Sen.
Revilla -with the former giving an offer to "acquire" his PDAF allocation in exchange for a "commission"
or "kickback" amounting to a certain percentage of the PDAF. Upon their agreement on the conditions of
the PDAF acquisition, including the project for which the PDAF will be utilized, the corresponding
Implemeting Agencies (IA) tasked to implement the same, and the legislator's "commission" or
"kickback" ranging from 40-60% of either the project cost or the amount stated in the Special Allotment
Release Order (SARO), the legislator would then write a letter addressed to the Senate President for the
immediate release of his PDAF, who in tum, will endorse such request to the DBM for the release of the
SARO. By this time, the initial advance portion of the "commission" would be remitted by Napoles to the
legislator. Upon release of the SARO, Napoles would then direct her staff -including whistleblowers
Benhur Luy (Luy), Marina Sula (Sula), and Merlina Suñas (Suñas) -to prepare PDAF documents
containing, inter alia, the preferred JLN-controlled NGO that will be used as a "conduit" for the
implementation of the project, the project proposals of the identified NGO, and the endorsement letters to
be signed by the legislator and/or his staff, all for the approval of the legislator; and would remit the
remaining portion or balance of the "commission" of the legislator, which is usually delivered by her
staff, Lim and De Asis.

Once the documents are approved, the same would be transmitted to the IA which would handle the
preparation of the Memorandum of Agreement (MOA) to be executed by the legislator's office, the IA,
and the chosen NGO. Thereafter, the DBM would release the Notice of Cash Allowance (NCA) to the IA
concerned, the head/official of which, in tum, would expedite the transaction and release of the
corresponding check representing the PDAF disbursement, in exchange for a ten percent (10%) share in
the project cost. Among those tasked by Napoles to pick up the checks and deposit them to the bank
accounts of the NGO concerned were Luy, Suñas, and De Asis. Once the funds are in the account of the
JLN-controlled NGO, Napoles would then call the bank to facilitate the withdrawal thereof. Upon
withdrawal of the said funds by Napoles's staff, the latter would bring the proceeds to the office of JLN
Corporation for accounting. Napoles would then decide how much will be left in the office and how much
will be brought to her residence in Taguig City. De Asis, Lim, Luy, and Suñas were the ones instructed to
deliver the money to Napoles's residence. Finally, to liquidate the disbursements, Napoles and her staff
would manufacture fictitious lists of beneficiaries, liquidation reports, inspection reports, project activity
reports, and similar documents that would make it appear that the PDAF-funded projects were
implemented when, in fact, they were not since they were actually inexistent or, in other words, "ghost"
projects. Under this modus operandi, Sen. Revilla, with the help of petitioners, among others, allegedly
funneled his PDAF amounting to around P517,000,000.00 to the JLN-controlled NGOs and, in return,
received "commissions" or "kickbacks" amounting to at least P224,512,500.00.

In his defense, Revilla filed his Counter-Affidavit dated January 16, 2014, contending that: (a) his and
Cambe's signatures in the PDAF documents were forgeries; (b) the utilization of his PDAF had "always
been regular and above-board."; (c) his involvement in the release of his PDAF is limited; and (d) there is

Remedial Law Review II (CY 2018-2019) – Saturday 1:00pm – 5:00pm: Case Digest Compilation [Atty. Brondial] Page 394
"no credible proof" to show that he committed said illegal acts and that conspiracy exists between him
and all the other persons involved in the PDAF scam.

Cambe, on the other hand, filed his Counter-Affidavit dated January 20, 2014 and Supplemental Counter-
Affidavit dated March 12, 2014, maintaining that: (a) his signatures in the PDAF documents were all
forgeries; and (b) he did not receive any money from Sen. Revilla's PDAF nor connive with any of the
alleged co-conspirators to acquire ill-gotten wealth.

In a Joint Resolution68 dated March 28, 2014 (March 28, 2014 Joint Resolution), the Ombudsman found
probable cause to indict, among others, petitioners Sen. Revilla, Cambe, Napoles, De Asis, and Lim of
one (1) count of Plunder, and all the petitioners (along with several others), except Lim, of sixteen (16)
counts of violation of Section 3 (e) of RA 3019.

Thus, the Ombudsman held that probable cause exists against Sen. Revilla, Cambe, Napoles, De Asis, and
Lim for Plunder, considering that: (a) Sen. Revilla was a public officer at the time material to the charges;
( b) with the help of his co-accused, who are public officers and private individuals, Sen. Revilla amassed,
accumulated, or acquired HI-gotten wealth through their intricate modus operandi as described above;
and ( c) such ill-gotten wealth amounted to at least P224,512,500.00, way more than the threshold amount
of P50,000,000.00 required in the crime of Plunder..

Cambe seeks to annul and set aside the Ombudsman's March 14, 2014 Joint Order which denied his
motion to suspend proceedings, arguing that the COA's issuance of an Order of Execution is a condition
precedent to the filing of the criminal complaints against him.

Sen. Revilla seeks to annul the March 28, 2014 Joint Resolution and the June 4, 2014 Joint Order of the
Ombudsman finding probable cause against him for the crimes charged. Among others, Sen. Revilla
faults the Ombudsman for allegedly disregarding his defense of forgery, and further contends that in the
absence of other competent testimony, the Ombudsman cannot consider the whistle blowers' testimonies
who purportedly were his co-conspirators in the PDAF scam, pursuant to the res inter alias acta rule.

Napoles similarly seeks to nullify the Ombudsman's March 28, 2014 Joint Resolution and June 4, 2014
Joint Order finding probable cause against her for Plunder and for violation of Section 3 (e) of RA 3019.
Essentially, she argues that the complaints did not establish the specific acts of the crimes she supposedly
committed. She likewise contends that since she is not a public officer, she cannot be subjected to
prosecution by the Ombudsman before the Sandiganbayan. Napoles's

ISSUE: the Ombudsman's finding of probable cause against all petitioners are correct.

RULING: Yes, there is probable cause against the petitioners should therefore stand trial for the crimes
they were charged.

Probable Cause against Revilla.


The finding of probable cause against Sen. Revilla is amply supported by the evidence on record. At the
forefront are the PDAF documents, consisting of the written endorsements signed by Sen. Revilla himself
requesting the IAs to release his PDAF funds to the JLN-controlled NGOs, as well as other documents
that made possible the processing of his PDAF, e.g., the MOAs executed by the legislator's office, the IA,
and the chosen NGO. All these documents -even those not actually signed by Sen. Revilla -directly
implicate him for the crimes charged, as they were nonetheless, all issued under the authority of his
Office as Senator of the Republic of the Philippines. In Belgica v. Ochoa (Belgica), this Court observed
that "the defining feature of all forms of Congressional Pork Barrel would be the authority of legislators
to participate in the post-enactment phases of project implementation. At its core, legislators -may it be
through project lists, prior consultations or program menus -have been consistently accorded post-
enactment authority to identify the projects they desire to be funded through various Congressional Pork
Barrel allocations." It is through this mechanism that individual legislators, such as Sen. Revilla, were
able to practically dictate the entire expenditure of the PDAF allocated to their offices throughout the
years.

Anent Sen. Revilla's claim that his signatures in the documentary evidence presented were forged, it must
be emphasized that "the findings of the x x x prosecutor [on the issue of forgery) should be ventilated in a
full-blown trial[.] [This] is highlighted by the reality that the authenticity of a questioned signature cannot
be determined solely upon its general characteristics, or its similarities or dissimilarities with the genuine
signature. The duty to determine the authenticity of a signature rests on the judge who must conduct an

Remedial Law Review II (CY 2018-2019) – Saturday 1:00pm – 5:00pm: Case Digest Compilation [Atty. Brondial] Page 395
independent examination of the signature itself in order to arrive at a reasonable conclusion as to its
authenticity. Accordingly, Sen. Revilla's evidence of forgery, including the findings of his purported
handwriting experts, Rogelio G. Azores (Azores) and Forensic Document Examiner Atty. Desiderio A.
Pagui, (Pagui) cannot be readily credited at this stage of the proceedings.

It is significant to emphasize that the Ombudsman had thoroughly passed upon the veracity of Sen.
Revilla's signatures on the PDAF documents. As explicitly stated in the March 28, 2014 Joint Resolution:
"[a]t all evei;its, the Special Panel members, after a prima facie comparison with their naked eyes of the
questioned signatures appearing in the PDAF documents and the original signatures of [Sen.] Revilla and
Cambe in their respective counter-affidavits, opine that both sets of signatures, which bear the same style
and flourish, were written by one and the same hands. Findings of fact by the Office of the Ombudsman
are conclusive when supported by substantial evidence, as in this case.

The testimonies of the whistleblowers -which the prosecution submitted before the Ombudsman -are, in
fact, the most integral evidence against Sen. Revilla, since they provide a detailed account on the inner
workings of the PDAF scam to which Sen. Revilla was directly involved. It should be pointed out that, of
all the Senators, only the Offices ' of Sen. Revilla, Sen. Juan Ponce Enrile (Sen. Enrile ), and Sen.
Jinggoy: Estrada (Sen. Estrada) were explicitly implicated to have dealt with in the plunder of their
PDAF. Also, it is apparent that whistleblowers Suñas, Sula, and Luy had personal knowledge of the
conspiracy since they were employees of JLN Corporation -the epicenter of the entire PDAF operation
-and in their respective capacities, were individually tasked by to prepare the pertinent documents,
liquidate the financial transactions, follow up the release of the NCAs with the DBM, and/or facilitate the
withdrawal of PDAF funds deposited in the NGOs' accounts.

In any event, even if it is assumed that the rule on res inter alias acta were to apply during preliminary
investigation, the treatment of the whistleblowers' statements as hearsay is bound by the exception on
independently relevant statements. "Under the doctrine of independently relevant statements, regardless
of their truth or falsity, the fact that such statements have been made is relevant. The hearsay rule does not
apply, and the statements are admissible as evidence. Evidence as to the making of such statement is not
secondary but primary, for the statement itself may constitute a fact in issue or be circumstantially
relevant as to the existence of such a fact. Undoubtedly, the testimonies of the whistleblowers are
independently relevant to prove the involvement of Sen. Revilla and his co-accused in the present
controversy, considering their respective participations in the entire PDAF scam.

Probable Cause against Cambe.


The same conclusion obtains with respect to the petition of Cambe in G.R. Nos. 212794-95 assailing the
Ombudsman's finding of probable cause against him. is no dispute that Ca:mbe was Sen. Revilla's trusted
aide, being his Chief of Staff. By such authority, he also exercised operational control over the affairs of
Sen. Revilla's office, including the allocation of his PDAF. In fact, Cambe' s signatures explicitly appear
on several PDAF documents, such as the MOAs allowing the IAs to transfer Sen. Revilla's PDAF funds
allocated for certain projects to various JLN-controlled NGOs. Cambe was personally identified by the
whistleblowers to have received PDAF money for himself and for Sen. Revilla.

In simple terms, Cambe allegedly acted as a liaison between Sen. Revilla and Napoles. For the same
reasons above-discussed, there should be 'no valid objection against the appreciation of the PDAF
documents and whistle blowers' testimonies as evidence to establish probable cause against Cambe at this
stage of the proceedings. He also has no right to be furnished copies of the counter-affidavits .of his co-
respondents.

Probable Cause against Napoles.


Records clearly show that Napoles, in all reasonable likelihood, played an integral role in the illegal
utilization, diversion, and disbursement of Sen. Revilla's PDAF. In fact, she was tagged as the
mastermind of the PDAF scam.

Based on the evidence in support thereof such as the PDAF documents, whistleblowers' testimonies, the
accounts of the IA officials, and the COA report, as well as the field verifications of the FIO,
Ombudsman, this Court is convinced that there lies probable cause against Janet Napoles for the charge of
Plunder as it has been prima facie established that she, in conspiracy with Sen. Revilla, Cambe, and other
personalities, was significantly involved in the afore-described modus operandi to obtain Sen. Revilla's
PDAF amounting to at least P50,000,000.00 in "kickbacks." In the same manner, there is probable cause
against Napoles for violations of Section 3 (e) of RA 3019, as it is ostensible that their conspiracy
to,illegally divert PDAF Funds to "ghost" projects caused undue prejudice to the government. That a

Remedial Law Review II (CY 2018-2019) – Saturday 1:00pm – 5:00pm: Case Digest Compilation [Atty. Brondial] Page 396
private individual, such as Napoles, could not be charged for Plunder and violations of Section 3 ( e) of
RA 3019 because the offenders in those crimes are public officers is a complete misconception. It has
been long-settled that while the primary offender in the aforesaid crimes are public officers, private
individuals may also be held liable for the same if they are found to have conspired with said officers in
committing the same. This proceeds from the fundamental principle that in cases of conspiracy the act of
one is the act of all. In this case, since it appears that Napoles has acted in concert with public officers in
the pillaging of Sen. Revilla's PDAF, the Ombudsman correctly indicted her as a co-conspirator for the
aforementioned crimes.

WHEREFORE, the petitions are DISMISSED for lack of merit. The findings of probable cause against all
petitioners are hereby AFFIRMED and the Sandiganbayan, as trial court, is DIRECTED to
commence/continue with the necessary proceedings in these cases with deliberate dispatch

Remedial Law Review II (CY 2018-2019) – Saturday 1:00pm – 5:00pm: Case Digest Compilation [Atty. Brondial] Page 397
C.3.f People vs. Dacanay

Remedial Law Review II (CY 2018-2019) – Saturday 1:00pm – 5:00pm: Case Digest Compilation [Atty. Brondial] Page 398
C.3.f.1 Diaz vs. People

G.R. No. 144293, February 18, 2013, 691 SCRA 139; Ponente: PANGANIBAN, J.

Topic: EVIDENCE; BURDEN OF PROOF AND PRESUMPTIONS

FACTS:In an amended Information, petitioner Josue R. Ladiana was charged with murder. During his
arraignment, petitioner, assisted by his counsel de parte, pled not guilty. After due trial, the
Sandiganbayan found him guilty of homicide, not murder.

The prosecution presented five (5) witnesses, namely: Caridad M. San Juan, PO2 Leopoldo Cacalda, Dr.
Rogelio M. Javan, SPO2 Percival A. Gabinete, and Maria T. Cortez (Cortez).
Cortez declared that he is a retired Assistant Prosecutor of Laguna. That prior to the conduct of the
examination-in-chief on Cortez, the defense counsel made an admission as to the authorship, authenticity,
and voluntariness of the execution of the counter-affidavit of accused Ladiana, which was subscribed and
sworn to before Cortez. In said counter-affidavit, accused Ladiana allegedly admitted to making the fatal
shots on Francisco. However, accused Ladiana allegedly did so in self-defense as Francisco was then
purportedly attacking accused Ladiana and had, in fact, already inflicted a stab wound on the arm of
accused Ladiana.

After the presentation of Cortez, the prosecution filed its formal offer of evidence and rested its case.
The Sandiganbayan ruled that the prosecution had been able to establish the guilt of petitioner beyond
reasonable doubt. The court a quo held that his Counter-Affidavit, in which he had admitted to having
fired the fatal shots that caused the victims death, may be used as evidence against him. It underscored the
admission made by the defense as to the authorship, the authenticity and the voluntariness of the
execution of the Counter-Affidavit.
Hence, this Petition.

Issue: Whether the Counter-Affidavit he executed during the preliminary investigation of this case is
admissible proof showing his complicity in the crime.

RULING: Yes. The Constitution bars the admission in evidence of any statement extracted by the police
from the accused without the assistance of competent and independent counsel during a custodial
investigation. However, a counter-affidavit voluntarily presented by the accused during the preliminary
investigation, even if made without the assistance of counsel, may be used as evidence against the affiant.
Custodial interrogation is the questioning initiated by law enforcement officers after a person has been
taken into custody or otherwise deprived of his freedom of action in any significant way.

In the present case, petitioner admits that the questioned statements were made during the preliminary
investigation, not during the custodial investigation. However, he argues that the right to competent and
independent counsel also applies during preliminary investigations.

We disagree. A preliminary investigation is an inquiry or a proceeding to determine whether there is


sufficient ground to engender a well-founded belief that a crime has been committed, and that the
respondent is probably guilty thereof and should be held for trial.

There is no question that even in the absence of counsel, the admissions made by petitioner in his
Counter-Affidavit are not violative of his constitutional rights. It is clear from the undisputed facts that it
was not exacted by the police while he was under custody or interrogation. Hence, the constitutional
rights of a person under custodial investigation as embodied in Article III, Section 12 of the 1987
Constitution, are not at issue in this case.

We do not, however, agree with the Sandiganbayans characterization of petitioners Counter-Affidavit as


an extrajudicial confession. It is only an admission. Sections 26 and 33 of Rule 130 of the Revised Rules
on Evidence distinguish one from the other as follows:
SEC. 26. Admissions of a party. The act, declaration or omission of a party as to a
relevant fact may be given in evidence against him.
SEC. 33. Confession. The declaration of an accused acknowledging his guilt of the
offense charged, or of any offense necessarily included therein, may be given in evidence
against him.

Remedial Law Review II (CY 2018-2019) – Saturday 1:00pm – 5:00pm: Case Digest Compilation [Atty. Brondial] Page 399
In a confession, there is an acknowledgment of guilt; in an admission, there is merely a statement of fact
not directly involving an acknowledgment of guilt or of the criminal intent to commit the offense with
which one is charged.26 Thus, in the case at bar, a statement by the accused admitting the commission of
the act charged against him but denying that it was done with criminal intent is an admission, not a
confession.

The Counter-Affidavit in question contains an admission that petitioner actually shot the victim when the
latter was attacking him. We quote the pertinent portion:
[K]aya itong si Kapitan San Juan ay sumugod at hinawakan ako sa may leeg ng aking
suot na T-shirt upang ako ay muling saksakin; sa dahilang hindi ako makatakbo o
makaiwas sa kabila ng aking pananalag hanggang magpaputok ako ng pasumala sa
kanya; sa bilis ng pangyayari ay hindi ko alam na siya ay tinamaan;28cräläwvirtualibräry
Through the above statement, petitioner admits shooting the victim -- which eventually led to the latters
death -- but denies having done it with any criminal intent. In fact, he claims he did it in self-defense.
Nevertheless, whether categorized as a confession or as an admission, it is admissible in evidence against
him.

Further, we do not doubt the voluntariness of the Counter-Affidavit. Petitioner himself submitted it to the
public prosecutor to justify his actions in relation to the charges hurled against him. It escapes this Court
how he can cavalierly deny a document that he has voluntarily submitted and originally relied upon in his
defense.

In general, admissions may be rebutted by confessing their untruth or by showing they were made by
mistake. The party may also establish that the response that formed the admission was made in a jocular,
not a serious, manner; or that the admission was made in ignorance of the true state of facts.29 Yet,
petitioner never offered any rationalization why such admissions had been made, thus, leaving them
unrebutted. In addition, admissions made under oath, as in the case at bar, are evidence of great weight
against the declarant. They throw on him the burden of showing a mistake.

Petitioner contends that nowhere in the transcripts of this case can it be found that he has admitted to the
authorship, the authenticity or the voluntariness of the Counter-Affidavit. We quote verbatim the
proceedings in the Sandiganbayan:

PJ GARCHITORENA
Well, he will identify the person who took the oath before him. Will you deny that it was
your client who took the oath before the Fiscal at the preliminary investigation?
ATTY. ILAGAN
We will admit that, your Honor.
PJ GARCHITORENA
So in that case we will have no question about the authorship, authenticity and the
voluntariness of the execution of the counter-affidavit dated July 31, 1990? Companiero?
ATTY ILAGAN
Admitted, your Honor.

The admissions of petitioner made through his counsel cannot be any clearer. To be sure, the unbroken
stream of judicial dicta is that, in the conduct of their case, clients are bound by the actions of their
counsels, save when the latters negligence is so gross, reckless and inexcusable that the former are
deprived of their day in court. Also, clients, being bound by the actions of their counsels, cannot complain
that the result of the litigation might have been different had their lawyers proceeded differently. A
counsel may err as to the competency of witnesses, the sufficiency and the relevance of evidence, the
proper defense, the burden of proof, the introduction or the withholding of witnesses or pieces of
evidence, or the manner of arguing the case. This Court, however, has ruled several times that those are
not even proper grounds for a new trial, unless the counsels incompetence is so gross that the clients are
prevented from fairly presenting their case.

Remedial Law Review II (CY 2018-2019) – Saturday 1:00pm – 5:00pm: Case Digest Compilation [Atty. Brondial] Page 400
C.3.f.2 THE PEOPLE OF THE PHILIPPINES,appellee vs.FELICIANO ULIT y
TAMPOY,appellant.

G.R. Nos. 131799-801; February 23, 2004; CALLEJO, SR., J

FACTS: Feliciano Ulit y Tampoy was found guilty beyond reasonable doubt of 2 counts of qualified rape
and two counts of acts of lasciviousness (i.e. kissing and touching sexual organ) of his 11-year old niece
Lucelle Serrano by the RTC Makati. Ulit was said to forced himself to his niece Lucelle through
intimation and while armed with a knife.

During the confrontation with the parents of Lucelle and Ulit in the barangay office, the accused admitted
to raping Lucelle. The said admission was reflected in the Sinumpaang Salaysay prepared in the Office of
the Barangay Chairman which was signed by Ulit in the presence of the barangay chairman and the
barangay tanods. From the barangay headquarters, Ulit was brought to the Makati City Police
Headquarters where the parents and Lucelle filed a complaint against him for rape and acts of
laciviousness. A custodial investigation was conducted wherein Ulit was without counsel during which he
admitted having raped the victim. A report of the investigation was prepared by the police of the victim’s
complaint.

During trial, the evidence the Ulit’s Sinumpaang Salaysay before the barangay chairman as part of the
testimony of Barangay Tanod was objected by Ulit to its admission on the ground that the he was not
assisted by counsel and that, he was forced and coerced into signing the same.
RTC admitted the statement as part of Brgy.Tanod’s testimony.

Ulit likewise objected to the admissibility of Lucelle’s sworn statement on the ground that she was
incompetent to give the same because of her mental illness. RTC admitted the sworn statement of Lucelle
in evidence as part of her testimony.

After the offer of evidence, the counsel of Ulit manifested to the court to change his plea from "not
guilty" to "guilty" and manifested that he would no longer adduce any evidence in his defense in the
lasciviousness cases because the prosecution failed to prove his guilt beyond reasonable doubt for the
crimes charged therein. The trial court suspended the proceedings and gave the appellant forty-five
minutes to confer with his counsel. When trial resumed, the appellant reiterated his earlier manifestation.
When told by the court that he could be sentenced to death for the rape charges, the appellant stood pat on
his decision. He was re-arraigned in the 2 rape cases with the assistance of the same counsel and entered
his plea of guilty to the charges.

RTC found Ulit guilty and declared that even prescinding from the appellant’s plea of guilty, the
prosecutor adduced proof beyond reasonable doubt of the guilt of the appellant for qualified rape and that
even if Lucelle did not testify on the contents of her sworn statementthe same were admissible in
evidence as part of the res gestae.

ISSUES:
1) Whether or not the procedure for change of plea of Ulit to guilty in a criminal case proper?
2) Whether or not the sworn statement of Lucelle is hearsay because she failed to testify on the contents
and merely identified her signature?
3) Whether or not the Sinumpaang Salaysay (extrajudicial) to the barangay chairman is admissible even if
not assisted by counsel?

RULING:
1) NO. Here, Ulit was charged with a capital offense. When the appellant informed the trial court of his
decision to change his plea of "not guilty" to "guilty," it behooved the trial court to conduct a searching
inquiry into the voluntariness and full comprehension of the consequences of his plea as mandated by
Section 6, Rule 116 of the Revised Rules of Criminal Procedure.
In People vs. Camay, this Court enumerated the following duties of the trial court under the rule:
1. The court must conduct a searching inquiry into the voluntariness and full comprehension [by the
accused] of the consequences of his plea;
2. The court must require the prosecution to present evidence to prove the guilt of the accused and precise
degree of his culpability; and
3. The court must require the prosecution to present evidence in his behalf and allow him to do so if he
desires.

Remedial Law Review II (CY 2018-2019) – Saturday 1:00pm – 5:00pm: Case Digest Compilation [Atty. Brondial] Page 401
In People vs. Aranzado, we formulated the following guidelines as to how the trial court may conduct its
searching inquiry:
(1) Ascertain from the accused himself (a) how he was brought into the custody of the law; (b) whether he
had the assistance of a competent counsel during the custodial and preliminary investigations; and (c)
under what conditions he was detained and interrogated during the investigations. These the court shall do
in order to rule out the possibility that the accused has been coerced or placed under a state of duress
either by actual threats of physical harm coming from malevolent or avenging quarters.
(2) Ask the defense counsel a series of questions as to whether he had conferred with, and completely
explained to, the accused the meaning and consequences of a plea of guilty.
(3) Elicit information about the personality profile of the accused, such as his age, socio-economic status,
and educational background, which may serve as a trustworthy index of his capacity to give a free and
informed plea of guilty.
(4) Inform the accused the exact length of imprisonment or nature of the penalty under the law and the
certainty that he will serve such sentence. Not infrequently indeed an accused pleads guilty in the hope of
a lenient treatment or upon bad advice or because of promises of the authorities or parties of a lighter
penalty should he admit guilt or express remorse. It is the duty of the judge to see to it that the accused
does not labor under these mistaken impressions.
(5) Require the accused to fully narrate the incident that spawned the charges against him or make him
reenact the manner in which he perpetrated the crime, or cause him to supply missing details or
significance.

Here, the trial court failed to ascertain whether he was assisted by counsel when he executed his
Sinumpaang Salaysay while detained at the barangay hall; and, if he was not so assisted by counsel,
whether he had waived his right thereto, before and when he signed his Sinumpaang Salaysay. The trial
court also failed to ask the appellant why he was pleading guilty to a rape committed in November 1996,
when in his Sinumpaang Salaysay, he confessed to having raped the victim only in February 1997 and
March 2, 1997. The trial court did not even inquire from the appellant who prepared and typed his
Sinumpaang Salaysay and if the contents of his statement were explained to him before he signed the
same. The trial court did not explain the following to the appellant, in plain and simple terms so as to be
understood by him: (a) the elements of the crime of qualified rape; (b) the circumstances of relationship
and the minority of the victim; and (c) that his plea of guilty to qualified rape would not mitigate the
penalty for the crime in light of Article 63 of the Revised Penal Code. Neither did the trial court inquire
from the appellant’s counsel whether the meaning and the consequences of a guilty plea were explained
to the appellant in a language or dialect known to and understood by him.

2) NO. We do not agree with the ruling of the trial court that the contents of the sworn statement of
Lucelle are hearsay, simply because she did not testify thereon and merely identified her signatures
therein. By hearsay evidence is meant that kind of evidence which does not derive its value solely from
the credence to be attributed to the witness herself but rests solely in part on the veracity and competence
of some persons from whom the witness has received the information. It signifies all evidence which is
not founded upon the personal knowledge of the witness from whom it is elicited, and which,
consequently, is not subject to cross-examination. The basis for the exclusion appears to lie in the fact that
such testimony is not subject to the test which can ordinarily be applied for the ascertainment of truth of
testimony, since the declarant is not present and available for cross-examination. In criminal cases, the
admission of hearsay evidence would be a violation of the constitutional provision while the accused shall
enjoy the right to confront and cross-examine the witness testifying against him. Generally, the affidavits
of persons who are not presented to testify on the truth of the contents thereof are hearsay evidence. Such
affidavit must be formally offered in evidence and accepted by the court; otherwise, it shall not be
considered by the court for the simple reason that the court shall consider such evidence formally offered
and accepted.

In this case, Lucelle testified on and affirmed the truth of the contents of her sworn statement which she
herself had given. As gleaned from the said statement, she narrated how and when the appellant raped and
subjected her to lascivious acts. She was cross-examined by the appellant’s counsel and answered the trial
court’s clarificatory questions. The prosecution offered her sworn statement as part of her testimony and
the court admitted the same for the said purpose without objection on the part of the appellant.

3) YES. Although the appellant was not assisted by counsel at the time he gave his statement to the
barangay chairman and when he signed the same, it is still admissible in evidence against him because he
was not under arrest nor under custodial investigation when he gave his statement.

Remedial Law Review II (CY 2018-2019) – Saturday 1:00pm – 5:00pm: Case Digest Compilation [Atty. Brondial] Page 402
The exclusionary rule is premised on the presumption that the defendant is thrust into an unfamiliar
atmosphere and runs through menacing police interrogation procedures where the potentiality for
compulsion, physical and psychological, is forcefully apparent. As intended by the 1971 Constitutional
Convention, this covers "investigation conducted by police authorities which will include investigations
conducted by the municipal police, the PC and the NBI and such other police agencies in our
government." The barangay chairman is not deemed a law enforcement officer for purposes of
applying Section 12(1) and (3) of Article III of the Constitution. Under these circumstances, it cannot be
successfully claimed that the appellant’s statement before the barangay chairman is inadmissible.

DOCTRINE: Assistance of counsel during execution of the extrajudicial confession to a barangay


chairman is not necessary and such extrajudicial confession is admissible. The barangay chairman is
not deemed a law enforcement officer for purposes of applying Section 12(1) and (3) of Article III of
the Constitution.

Remedial Law Review II (CY 2018-2019) – Saturday 1:00pm – 5:00pm: Case Digest Compilation [Atty. Brondial] Page 403
C.3.f.3 PEOPLE OF THE PHILIPPINES vs. BENJAMIN SAYABOC y SEGUBA et al.

G.R. No. 147201; January 15, 2004


FACTS: Herein accused were charged with murder. He was arrested pursuant to the testimony of two (2)
eyewitnesses. During his custodial investigation the accused made an extrajudicial confession after the
following was recited to him: “I would like to inform you Mr. Sayaboc that questions will be asked to you
regarding an incident last December 2, 1994 at the Rooftop, Brgy. Quezon, Solano, Nueva Vizcaya, in
connection with the shooting of Joseph Galam, owner of the said Disco House as a result of his death.
Before questions will be asked of you I would like to inform you about your ri[g]hts under the new
Constitution of the Philippines, as follows: That you have the right to remain silent or refuse to answer the
questions which you think will incriminate you; That you have the right to seek the services of a counsel
of your own choice or if not, this office will provide you a lawyer if you wish.”

The appellants argue that the extrajudicial confession of Sayaboc may not be admitted in evidence against
him because Atty. Cornejo, the PAO lawyer who was his counsel during the custodial investigation, was
not a competent, independent, vigilant, and effective counsel. He was ineffective because he remained
silent during the entire proceedings.

ISSUES:
(1) Whether or not an extrajudicial confession made during custodial investigation wherein the rights of
the accused were merely recited to him may be admissible in evidence against such accused.
(2) Whether a counsel who remain silent at the time the extrajudicial confession was made qualifies as an
independent counsel.

RULING:
(1) The answer is in the negative. The showing of a spontaneous, free, and unconstrained giving up of a right
is missing. The right to be informed requires "the transmission of meaningful information rather than just
the ceremonial and perfunctory recitation of an abstract constitutional principle." It should allow the
suspect to consider the effects and consequences of any waiver he might make of these rights. More so
when the suspect is one like Sayaboc, who has an educational attainment of Grade IV, was a stranger in
Nueva Vizcaya, and had already been under the control of the police officers for two days previous to the
investigation, albeit for another offense.
(2) A counsel who remains silent all through-out the investigation shows that there is lacking of a faithful
attempt at each stage of the investigation to make Sayaboc aware of the consequences of his actions.
The right to a competent and independent counsel means that the counsel should satisfy himself, during
the conduct of the investigation, that the suspect understands the import and consequences of answering
the questions propounded. Counsel should be able, throughout the investigation, to explain the nature of
the questions by conferring with his client and halting the investigation should the need arise. The duty of
a lawyer includes ensuring that the suspect under custodial investigation is aware that the right of an
accused to remain silent may be invoked at any time.

Remedial Law Review II (CY 2018-2019) – Saturday 1:00pm – 5:00pm: Case Digest Compilation [Atty. Brondial] Page 404
C.3.f.4 CARLOS L. TANENGGEE vs. PEOPLE OF THE PHILIPPINES

G.R. No. 179448, June 26, 2013

FACTS: Petitioner was charged with estafa through falsification of commercial documents. The
prosecution alleged that:
(1) the petitioner as branch manager caused the preparation of promissory notes (PN) and cashier’s check
in the name of one of their valued client; (2) that by forging the signature of such client, petitioner was
able to obtain the proceeds of the loan evidenced by the PN.

After the discovery of the irregular loans, an internal audit was conducted and an administrative
investigation was held in the Head Office of Metrobank, during which appellant signed a written
statement in the form of questions and answers admitting the commission of the allegations in the
Information. Trial court convicted the accused. On appeal, the CA affirmed his conviction.

Elevating the case before the SC, petitioner avers that the written statement should not be admitted as
evidence against him as it was taken in violation of his rights under Section 12, Article III of the
Constitution, particularly of his right to remain silent, right to counsel, and right to be informed of the first
two rights. Hence, the same should not have been admitted in evidence against him.

ISSUE: Whether or not a statement of an accused-employee made during administrative investigation


conducted by his employer may not be admitted an evidence against the former on the ground that it was
made without the assistance of a counsel.

RULING: The answer is in the negative. The constitutional proscription against the admissibility of
admission or confession of guilt obtained in violation of Section 12, Article III of the Constitution is
applicable only in custodial interrogation. Custodial interrogation means any questioning initiated by law
enforcement authorities after a person is taken into custody or otherwise deprived of his freedom of action
in any significant manner.

In the present case, while it is undisputed that petitioner gave an uncounselled written statement regarding
an anomaly discovered in the branch he managed, the following are clear: (1) the questioning was not
initiated by a law enforcement authority but merely by an internal affairs manager of the bank; and, (2)
petitioner was neither arrested nor restrained of his liberty in any significant manner during the
questioning. Clearly, petitioner cannot be said to be under custodial investigation and to have been
deprived of the constitutional prerogative during the taking of his written statement. No error can
therefore be attributed to the courts below in admitting in evidence and in giving due consideration to
petitioner’s written statement as there is no constitutional impediment to its admissibility.

Remedial Law Review II (CY 2018-2019) – Saturday 1:00pm – 5:00pm: Case Digest Compilation [Atty. Brondial] Page 405
C.3.f.5 MARIA PAZ FRONTRERAS Y ILAGAN VS PEOPLE OF THE PHILIPPINES

G.R. No. 190583; DECEMBER 7, 2015; REYES, J.:

FACTS: The petitioner was the Vault Custodian of the 685 Old Balara, Tandang Sora, Quezon City
Branch of Cebuana Lhuillier Pawnshop. She was tasked to safe keep all the pawned items and jewelry
inside the branch vault. Likewise employed in the same branch were Teresita Salazar and Jeannelyn
Carpon who served as Branch Manager and District Manager respectively. On October 27, 1998, a
surprise audit was conducted at the Old Balara branch of Cebuana’s internal auditors, Escartin and
Talampas. The audit revealed that 156 pieces of jewelry, with an aggregate value of Php 1,250,800.00
were missing. A cash shortage of 848.60 was likewise discovered. When the petitioner was asked to
explain the discrepancy, she told Escartin that she would reduce her explanation into writing. The next
day, an audit report was sent to Marcelino Finolan, Area manager of Cebuana.

Upon receipt of the audit report, Finolan immediately proceeded to old Balara branch to conduct an
investigation. He called Escarin and the petitioner for a meeting during which the petitioner handed over
several pawn tickets while Escarin gave him a handwritten letter made by the petitioner, which reads:
October 28, 1998

Sa Kinauukulan:

Sir, nagconduct po ng audit kahapon Oct 27, 1998 dito sa Old Balara I at nadiskubre po na maraming
nawawalang item. Sir ang lahat pong ito ay mga sanla naming. Ang involved po ditto ay ang appraiser
Tess Salazar, District Manager JEnnelyn Uy Carpon at ako po Vault Custodian Ma. Paz Frontreras.
Tong iba pong item ay mga tubos na at nakatago lang po ang papel. Nagsimula po ito nong buwan ng
Hulyo. Dala na rin pong matinding pangangailangan sa pera. Ito lamang po ang tangi kong mailalahad
at iyan din po ang katotohanan.
Sumasainyo,
[signed]
Ma Paz Frontreras

On May 10, 1999, an information for qualified theft was filed before the RTC against the petitioner,
Salazar and Carpon.

The RTC found sufficient circumstantial evidence establishing that the petitioner perpetrated the offense
of qualified theft thus, both Salazar and Carpon were acquitted.

Petitioner then moved for reconsideration but was denied by the court. However, her penalty was reduced
by considering the surrender of the pawned tickets as mitigating circumstance analogous to voluntary
surrender and the necessity mentioned in the handwritten explanation as analogous to incomplete
justification.

Petitioner then appealed to the CA but the petition was rejected by the CA and upheld the RTC’s findings
and conclusion. However, CA disagreed with the RTC that the return by the petitioner of the pawn tickets
can be deemed as mitigating circumstance of voluntary surrender.

ISSUES:
1. Whether or not the handwritten letter of the petitioner is admissible in evidence.
2. Whether or not the handwritten letter can be deemed as mitigating circumstance of voluntary
surrender.

RULING:
8) YES. The language of the confession letter was straightforward, coherent and clear. It bore no
suspicious circumstances tending to cast doubt upon its integrity and it was replete with details which
could only be known to the petitioner.
A confession, whether judicial or extrajudicial, if voluntarily or freely made, constitutes evidence of
higher order since it is supported by the strong presumption that no sane person or one normal mind will
deliberately and knowingly confess himself to be the perpetrator of a crime, unless prompted by truth and
conscience. The admissibility and validity of a confession, thus hinges on its voluntariness, a condition
vividly present in this case. The petitioner’s extrajudicial confession coupled with the circumstantial
evidence all point to her as the perpetrator of the unlawful taking.

Remedial Law Review II (CY 2018-2019) – Saturday 1:00pm – 5:00pm: Case Digest Compilation [Atty. Brondial] Page 406
9) YES. Anent the appreciation of mitigating circumstances, the Court agrees with RTC that the
petitioner’s extrajudicial confession through the handwritten letter coupled with the act of surrendering
the redeemed pawn tickets and thereafter going to the police station can be taken as an analogous
circumstance of voluntary surrender under Article 13, paragraph 10 in relation to paragraph 7 of RPC.

Based on the same extrajudicial confession, the petitioner is also entitled to the mitigating circumstance
of no intention to commit so grave a wrong under paragraph 3 again in relation to paragraph 10 both of
Article 13. Based on her letter, the petitioner misappropriated the redemption payments under her custody
and control because she was constrained by extreme necessity for money.

Remedial Law Review II (CY 2018-2019) – Saturday 1:00pm – 5:00pm: Case Digest Compilation [Atty. Brondial] Page 407
C.3.f.6 PEOPLE OF THE PHILIPPINES vs. ROMALDO LUMAYAG Y DELA CRUZ, DIONY
OPINIANO Y VERANO, AND JERRY1 DELA CRUZYDIAZ

FACTS: In the Information dated December 3, 1997, Opiniano, Romaldo Lumayag (Lumayag), and
Jerry Dela Cruz (Dela Cruz) were charged with the crime of robbery with homicide. The three (3) accused
pleaded not guilty during their arraignment on January 12, 1998. No stipulations of fact were entered
during pre-trial. Joint trial ensued.

The Spouses Santos were dealers of soft drinks and beer. They maintained a store, adjacent to their two-
storey house. Their daughter, Estrella, helped manage the store daily from 8:00 a.m. or 9:00 a.m. to 3:00
p.m. or 4:00 p.m.13 Dela Cruz was their stay-in helper. He had been working for them for only three (3)
to five (5) days before the couple were killed.

Around 2:30 a.m. of November 30, 1997, Estrella received a call from her sister that their parents were
stabbed. When she went up to the second floor, she found the master bedroom in shambles, and noticed
that some money and her mother's pieces of jewelry were missing. She also noticed that the kitchen knife
was missing. It had a "black rubber band wrapped around the handle[.]" She later found the knife full of
blood inside a case of beer. The knife was turned over to the La Loma police.

Around 9:00 p.m. of the previous day, November 29, 1997, P02 Paule and SPO1 Eduardo Roderno
(SPO1 Roderno) of the Caloocan police were traversing C-3 Road aboard a police-marked vehicle when
they noticed a man carrying a heavy-looking bag. When they approached him, the man ran away. After a
brief chase, the man was cornered. P02 Paule noticed that his right leg was stained with blood and his
right waistline was bulging with an object, which turned out to be a double bladed 9-inch mini kris. He
did not answer when asked about the bloodstain on his leg. They brought him to the police station where
he identified himself as Jerry Dela Cruz.

Upon further interrogation, Dela Cruz verbally confessed that he and his companions, whom he later
revealed as "Ango" or Lumayag, and Opiniano, "had just killed and robbed an old couple." He was
supposed to bring the contents of the bag to his cohorts in the illegal settlers' area in Malabon. During
cross-examination, PO2 Paule affirmed that Dela Cruz was not aided by a lawyer, nor was his confession
reduced into writing. PO2 Paule further testified that when they informed Dela Cruz of his right to a
lawyer, the latter remained silent.

Dela Cruz then accompanied the police officers to the scene of the crime. They called the La Loma Police
Station, which had jurisdiction over the case. PO2 Paule and the other Caloocan police operatives,
together with Dela Cruz, then proceeded to Letre, Malabon where they were able to apprehend Opiniano.

About 1:00 p.m. on November 30, 1997, P02 Flores and other La Loma police officers, together with
Dela Cruz, were dispatched to Kaysikat, Antipolo, Rizal where they arrested Lumayag. When Lumayag
was frisked, two (2) coin wrappers bearing initials were found inside his pocket. Estrella later identified
the initials in the coin wrappers as hers.

On February 8, 2000, Branch 76, Regional Trial Court, Quezon City rendered a Decision, which found
Opiniano and Lumayag guilty as principals of the crime of robbery with homicide and imposed upon
them the penalty of reclusion perpetua. On the other hand, the trial court found Dela Cruz as an accessory
to the crime and imposed upon him an indeterminate prison sentence of two (2) years, four (4) months,
and one (1) day to four (4) years and two (2) months of prision correccional.

Only Opiniano appealed the Regional Trial Court's decision. In view of People v. Mateo, this Court
referred the case to the Court of Appea1s for intermediate review. CA Affirmed the decision of the RTC.
The records of this case were elevated to the Supreme Court on February 14, 2008, pursuant to the Court
of Appeals' October 18, 2007 Resolution, which gave due course to Opiniano's Notice of Appeal.

Appellant Opiniano contends, however, that the totality of the circumstantial evidence is "insufficient to
support [his] conviction beyond reasonable doubt." He further argues that the extra-judicial confession of
Dela Cruz, implicating him in the crime, is inadmissible in evidence, as it was obtained without the
assistance of counsel. 

ISSUE: WON the extra-judicial confession of Dela Cruz is admissible as evidence against Opiniano.

Remedial Law Review II (CY 2018-2019) – Saturday 1:00pm – 5:00pm: Case Digest Compilation [Atty. Brondial] Page 408
RULING: Dela Cruz's extrajudicial confession without counsel at the police station without a valid
waiver of the right to counsel - that is, in writing and in the presence of counsel - is inadmissible in
evidence.It is undisputed that Dela Cruz was neither assisted by a lawyer nor was his confession reduced
into writing.Further, when the police officers informed Dela Cruz of his right to a lawyer, the latter did
not say anything. Even so, such silence did not constitute a valid waiver of his right to remain silent and
to have a competent and independent counsel. Article III, Section 12 of the Constitution states that
"[t]hese rights cannot be waived except in writing and in the presence of counsel."

Dela Cruz was merely told of his Constitutional rights, but he was never asked whether he understood
what he was told or whether he wanted to exercise or avail himself of such rights.

This kind of perfunctory giving of the so-called Miranda rights is what this Court has previously frowned
upon as ineffective and inadequate compliance with the mandates of the Constitution. Any confession
obtained under these circumstances is flawed and cannot be used as evidence not only against the
declarant but also against his co-accused.

In People v. Jara, this Court held that where a confession was illegally obtained from two (2) of the
accused, and consequently were not admissible against them, with much more reason should the same be
inadmissible against a third accused who had no participation in its execution. 

Hence, Dela Cruz's extrajudicial confession .is likewise inadmissible against appellant Opiniano.
Nonetheless, even without Dela Cruz's extra-judicial confession, Opiniano's conviction still
stands.1âwphi1 The eyewitness account of Dela Cruz, corroborated by the testimony and findings of Dr.
Arizala and Forensic Biologist Buan, suffices to convict accused-appellant Opiniano of the crime
charged.

Remedial Law Review II (CY 2018-2019) – Saturday 1:00pm – 5:00pm: Case Digest Compilation [Atty. Brondial] Page 409
C.3.g.1 PEOPLE OF THE PHILIPPINES, plaintiff-appellee, vs. RAUL SANTOS Y NARCISO,
MARIO MORALES Y BACANI, PETER DOE and RICHARD DOE, Accused, RAUL
SANTOS y NARCISO, accused-appellant

G.R. No. 100225-26; May 11, 1993

FACTS: Santos was charged and convicted of murder and frustrated murder. On appeal, accused maked
the following assignment of errors: 1) his identification in the police line-up by the two witnesses is
inadmissible as he was not afforded his right to counsel; 2) he questions the trial court for admitting a
sworn statement by one Ronaldo Guerrero, a witness in another criminal case accused was also charged
with the murder which had taken place in the very same site where Bautista and Cupcupin were
ambushed as such accused contends that the affidavit of Guerrero was hearsay evidence, considering that
the prosecution did not present Guerrero as a witness during trial.

ISSUES:
a) whether of not identitfication in the police line-up is not admissible in the ground that the accused
was not provided with counsel
b) whether the trial court may not admit a sworn statement of a witness taken from another criminal
case therein the accused for both cases are one and the same

RULING:

a) There is "no real need to afford a suspect the service of counsel at police line-up." The customary
practice is, of course, that it is the witness who is investigated or interrogated in the course of a
police line-up and who gives a statement to the police, rather than the accused who is not
questioned at all at that stage. There is nothing in the record of this case which shows that in the
course of the line-up, the police investigators sought to extract any admission or confession from
appellant Santos. The investigators did not in fact interrogate appellant Santos during the line-up
and he remained silent after he had been identified by Bautista and Bohol.

b) When the prosecution first presented the sworn statement of Guerrero in order to show criminal
propensity on the part of appellant Santos, the defense objected to admission of such sworn
statement; the trial court sustained the objection and rejected the evidence for the purpose it was
initially offered. However, the trial court admitted the same as falling within one or more of the
exceptions set out in Section 34, Rule 130 of the Rules of Court. the trial court did not commit
reversible error in admitting the Guerrero affidavit for the limited purpose for proving knowledge
or plan or scheme, and more specifically, that appellant knew that the particular corner of two (2)
particular streets in Malabon was a good place to ambush a vehicle and its passengers. Appellant
also had waived the hearsay character of this evidence by failure seasonably to object to the
admission of the affidavit; it is too late in that day to raise the hearsay rule in the appellant's
memorandum after prosecution and defense had presented their respective cases and had made
their respective offers of evidence.

Remedial Law Review II (CY 2018-2019) – Saturday 1:00pm – 5:00pm: Case Digest Compilation [Atty. Brondial] Page 410
C.3.g.2 People vs. Nardo

PEOPLE OF THE PHILIPPINES vs. ALFREDO NARDO y ROSALES

G.R. No. 133888; March 1, 2001

FACTS: Herein accused was charged of raping his eldest daughter. The prosecution presented the victim
as its main witness, while, the defense presented a number of witnesses who testified to different occasion
for which the victim was caught lying.

After the trial, trial court, giving credence to the testimony of the victim, convicted the accused. On
appeal, the accused presented series of letters allegedly written by the victim to the defense counsel
asking said counsel to help her father be acquitted.

ISSUES:
(1) Whether or not a minor witness’ credibility may be assailed by proving that she lies on a number of
occasion.
(2) Whether or not letters written by the witness after trial containing details that is contrary to testimony
made in open court constitutes recantation of said testimony.

RULING:
(1) The answer is in the negative. Rule 130, Section 34, of the Rules of Court provides that: "Evidence
that one did or did not do a certain thing at one time is not admissible to prove that he did nor did not do
the same or a similar thing at another time; but it may be received to prove a specific intent or knowledge,
identity, plan, system, scheme, habit, custom or usage, and the like." While lying may constitute a habit,
we believe that the falsehoods committed by Lorielyn, assuming them for the moment to be true, are petty
and inconsequential.

They are not as serious as charging one’s own father of the sordid crime of rape, with all of its serious
repercussions. Furthermore, as a rule, findings by the trial court on the credibility of witnesses are not to
be disturbed, for the trial court is in a better position to pass upon the same. Lastly, jurisprudence dictates
that testimonies of child-victims are given full weight and credit, since when a woman, more so if she is a
minor, says that she has been raped, she says in effect all that is necessary to show that rape was
committed. Youth and immaturity are generally badges of truth and sincerity.

(2) An affidavit of recantation, being usually taken ex parte, would be considered inferior to the testimony
given in open court. It would be a dangerous rule to reject the testimony taken before a court of justice
simply because the witness who gave it later on changed his/her mind for one reason or another. Such a
rule would make a solemn trial a mockery, and place the proceedings at the mercy of unscrupulous
witnesses.

Recantations are frowned upon by the courts because they can easily be obtained from witnesses through
intimidation or for monetary consideration. A retraction does not necessarily negate an earlier declaration.
Especially, recantations made after the conviction of the accused deserve only scant consideration.

Remedial Law Review II (CY 2018-2019) – Saturday 1:00pm – 5:00pm: Case Digest Compilation [Atty. Brondial] Page 411
C.3.g.3 REPUBLIC OF THE PHILIPPINES vs. HEIRS OF FELIPE ALEJAGA SR.

G.R. No. 146030, December 3, 2002

FACTS: Respondent Felipe Alejaga, Sr. filed Free Patent Application. In relation to the said application,
Recio, Land Inspector, submitted a report of his investigation to the Bureau of Lands. Less than 3 months
after the application, free patent was issued.

Sometime in April 1979, the heirs of Ignacio Arrobang requested for an investigation for irregularities in
the issuance of the title of a foreshore land in favor of respondent. Isagani Cartagena, Supervising Special
Investigator, submitted his Report. In that report, Recio supposedly admitted that he had not actually
conducted an investigation and ocular inspection of the parcel of land. Thereafter, government instituted
an action for Annulment/Cancellation of Patent and Title and Reversion against respondent.
Trial court ruled in favor of the petitioner. In reversing the RTC, CA brushed aside as hearsay Isagani
Cartagena’s testimony that Land Inspector Efren L. Recio had not conducted an investigation on the free
patent application of Felipe Alejaga Sr.

ISSUE: Whether or not testimony based on a report which relates an admission of a third person who was
not presented as witness is inadmissible in evidence for being a hearsay.

RULING: The answer is in the negative. A witness may testify as to the state of mind of another person
— the latter’s knowledge, belief, or good or bad faith — and the former’s statements may then be
regarded as independently relevant without violating the hearsay rule. Recio’s alleged admission may be
considered as "independently relevant."

Thus, because Cartagena took the witness stand and opened himself to cross- examination, the
Investigation Report he had submitted to the director of the Bureau of Lands constitutes part of his
testimony. Those portions of the report that consisted of his personal knowledge, perceptions and
conclusions are not hearsay. On the other hand, the part referring to the statement made by Recio may be
considered as independently relevant.

The doctrine on independently relevant statements holds that conversations communicated to a witness by
a third person may be admitted as proof that, regardless of their truth or falsity, they were actually made.
Evidence as to the making of such statements is not secondary but primary, for in itself it (a) constitutes a
fact in issue or (b) is circumstantially relevant to the existence of such fact.
Since Cartagena’s testimony was based on the report of the investigation he had conducted, his testimony
was not hearsay and was, hence, properly admitted by the trial court.

Remedial Law Review II (CY 2018-2019) – Saturday 1:00pm – 5:00pm: Case Digest Compilation [Atty. Brondial] Page 412
C.3.h ANNA LERIMA PATULA, petitioner, vs. PEOPLE OF THE PHILIPPINES, respondent.

G.R. No. 164457; April 11, 2012; BERSAMIN, J.:

FACTS: Petitioner, a sales representative at Footlucker’s Chain of Stores, was charged with estafa for
failure to account and deliver the proceeds of the sales and collection to the said company.

During the trial, the prosecution, to prove the misappropriation, submitted the following documentary
evidence: (1)the receipts allegedly issued by petitioner to each of her customers; (2) the ledgers listing
the accounts of each customer with the corresponding receipt numbers for each payments; and (3) the
confirmation sheets accomplished by Guivencan.

The Prosecution then formally offered its documentary exhibits, including Exhibits B to YY and their
derivatives (like the originals and duplicates of the receipts supposedly executed and issued by petitioner),
inclusive, the confirmation sheets used by Guivencan in auditing the accounts served by petitioner, and
Guivencan's so-called Summary (Final Report) of Discrepancies. Prosecution also presented Guivencan
to testify on the entries in the documentary evidence. Petitioner’s counsel interposed a continuing
objection on the ground that the figures entered in the Exhibits and their derivatives, inclusive, were
hearsay because the persons who had made the entries were not themselves presented in court.

ISSUE: Whether or not Guivencan's testimony on the ledgers and receipts (Exhibits B to YY, and their
derivatives, inclusive) to prove petitioner's misappropriation or conversion was in admissible for being
hearsay.

RULING: NO. To elucidate why the Prosecution’s hearsay evidence was unreliable and untrustworthy,
and thus devoid of probative value, reference is made to Section 36 of Rule 130, Rules of Court, a rule
that states that a witness can testify only to those facts that she knows of her personal knowledge; that is,
which are derived from her own perception, except as otherwise provided in the Rules of Court. The
personal knowledge of a witness is a substantive prerequisite for accepting testimonial evidence that
establishes the truth of a disputed fact. A witness bereft of personal knowledge of the disputed fact cannot
be called upon for that purpose because her testimony derives its value not from the credit accorded to her
as a witness presently testifying but from the veracity and competency of the extrajudicial source of her
information.

In case a witness is permitted to testify based on what she has heard another person say about the facts in
dispute, the person from whom the witness derived the information on the facts in dispute is not in court
and under oath to be examined and cross-examined. The weight of such testimony then depends not upon
the veracity of the witness but upon the veracity of the other person giving the information to the witness
without oath. The information cannot be tested because the declarant is not standing in court as a witness
and cannot, therefore, be cross-examined.

It is apparent, too, that a person who relates a hearsay is not obliged to enter into any particular, to answer
any question, to solve any difficulties, to reconcile any contradictions, to explain any obscurities, to
remove any ambiguities; and that she entrenches herself in the simple assertion that she was told so, and
leaves the burden entirely upon the dead or absent author. Thus, the rule against hearsay testimony rests
mainly on the ground that there was no opportunity to cross-examine the declarant. The testimony may
have been given under oath and before a court of justice, but if it is offered against a party who is
afforded no opportunity to cross-examine the witness, it is hearsay just the same.

Moreover, the theory of the hearsay rule is that when a human utterance is offered as evidence of the truth
of the fact asserted, the credit of the assertor becomes the basis of inference, and, therefore, the assertion
can be received as evidence only when made on the witness stand, subject to the test of cross-
examination. However, if an extrajudicial utterance is offered, not as an assertion to prove the matter
asserted but without reference to the truth of the matter asserted, the hearsay rule does not apply…. This
kind of utterance is hearsay in character but is not legal hearsay. The distinction is, therefore, between (a)
the fact that the statement was made, to which the hearsay rule does not apply, and (b) the truth of the
facts asserted in the statement, to which the hearsay rule applies.

Section 36, Rule 130 of the Rules of Court is understandably not the only rule that explains why
testimony that is hearsay should be excluded from consideration. Excluding hearsay also aims to preserve
the right of the opposing party to cross-examine the original declarant claiming to have a direct
knowledge of the transaction or occurrence. If hearsay is allowed, the right stands to be denied because

Remedial Law Review II (CY 2018-2019) – Saturday 1:00pm – 5:00pm: Case Digest Compilation [Atty. Brondial] Page 413
the declarant is not in court. It is then to be stressed that the right to cross-examine the adverse party’s
witness, being the only means of testing the credibility of witnesses and their testimonies, is essential to
the administration of justice.

WHEREFORE, the Court SETS ASIDE AND REVERSES the decision convicting ANNA LERIMA
PATULA of estafa as charged, and ACQUITS her for failure of the Prosecution to prove her guilt beyond
reasonable doubt, without prejudice to a civil action brought against her for the recovery of any amount
still owing in favor of Footlucker’s Chain of Stores, Inc.

DOCTRINES: The rule against hearsay testimony rests mainly on the ground that there was no
opportunity to cross-examine the declarant.—It is apparent, too, that a person who relates a hearsay is
not obliged to enter into any particular, to answer any question, to solve any difficulties, to reconcile any
contradictions, to explain any obscurities, to remove any ambiguities; and that she entrenches herself in
the simple assertion that she was told so, and leaves the burden entirely upon the dead or absent author.
Thus, the rule against hearsay testimony rests mainly on the ground that there was no opportunity to
cross-examine the declarant. The testimony may have been given under oath and before a court of justice,
but if it is offered against a party who is afforded no opportunity to cross-examine the witness, it is
hearsay just the same.

If an extrajudicial utterance is offered, not as an assertion to prove the matter asserted but without
reference to the truth of the matter asserted, the hearsay rule does not apply.—The theory of the
hearsay rule is that when a human utterance is offered as evidence of the truth of the fact asserted, the
credit of the assertor becomes the basis of inference, and, therefore, the assertion can be received as
evidence only when made on the witness stand, subject to the test of cross-examination. However, if an
extrajudicial utterance is offered, not as an assertion to prove the matter asserted but without reference
to the truth of the matter asserted, the hearsay rule does not apply. For example, in a slander case, if a
prosecution witness testifies that he heard the accused say that the complainant was a thief, this
testimony is admissible not to prove that the complainant was really a thief, but merely to show that the
accused uttered those words. This kind of utterance is hearsay in character but is not legal hearsay. The
distinction is, therefore, between (a) the fact that the statement was made, to which the hearsay rule does
not apply, and (b) the truth of the facts asserted in the statement, to which the hearsay rule applies.

The right to cross-examine the adverse party’s witness, being the only means of testing the credibility
of witnesses and their testimonies, is essential to the administration of justice.—Section 36, Rule 130 of
the Rules of Court is understandably not the only rule that explains why testimony that is hearsay should
be excluded from consideration. Excluding hearsay also aims to preserve the right of the opposing party
to cross-examine the original declarant claiming to have a direct knowledge of the transaction or
occurrence. If hearsay is allowed, the right stands to be denied because the declarant is not in court. It is
then to be stressed that the right to cross-examine the adverse party’s witness, being the only means of
testing the credibility of witnesses and their testimonies, is essential to the administration of justice.

Section 19, Rule 132 of the Rules of Court distinguishes between a public document and a private
document for the purpose of their presentation in evidence.—Section 19, Rule 132 of the Rules of Court
distinguishes between a public document and a private document for the purpose of their presentation in
evidence, viz.: Section 19. Classes of documents.—For the purpose of their presentation in evidence,
documents are either public or private. Public documents are: (a) The written official acts, or records of
the official acts of the sovereign authority, official bodies and tribunals, and public officers, whether of
the Philippines, or of a foreign country; (b) Documents acknowledged before a notary public except last
wills and testaments, and (c) Public records, kept in the Philippines, of private documents required by
law to be entered therein. All other writings are private.

Requisites before Entries in the Course of Business Could be Excepted from the Hearsay Rule.—The
terse yet sweeping manner of justifying the application of Section 43 was unacceptable due to the need to
show the concurrence of the several requisites before entries in the course of business could be excepted
from the hearsay rule. The requisites are as follows: (a) The person who made the entry must be dead or
unable to testify; (b) The entries were made at or near the time of the transactions to which they refer; (c)
The entrant was in a position to know the facts stated in the entries; (d) The entries were made in his
professional capacity or in the performance of a duty, whether legal, contractual, moral, or religious; (e)
The entries were made in the ordinary or regular course of business or duty.

Remedial Law Review II (CY 2018-2019) – Saturday 1:00pm – 5:00pm: Case Digest Compilation [Atty. Brondial] Page 414
C.3.h.i.1 PEOPLE OF THE PHILIPPINES vs. ROMEO D. CALINA WAN a.k.a "MEO"

G.R. No. 226145; February 13, 2017; MENDOZA, J.:

FACTS:In an Information, Calinawan was charged with murder for killing Janice Nevado Silan (Janice).
During his arraignment, he entered a plea of "Not Guilty." After the pre-trial was terminated, trial ensued.

The Version of the Prosecution

At around midnight on September 26, 2007, Marigor Silan (Marigor), Janice's seven (7)-year old
daughter, saw Calinawan stabbing her mother in their kitchen. Thereafter, Calinawan quickly fled the
scene. Meanwhile, Jonathan Nevado (Jonathan), Janice's brother and neighbor, was awakened by shouts
coming from his sister's house. He rushed to her house and saw her children crying. After bringing her
children to his house, he went looking for Janice whom he saw outside a neighbor's house pleading for
help. Seeing her bloodied, he carried her and asked her who stabbed her, and she answered it was
Calinawan who did it. Then, Jonathan brought Janice to the hospital. When Darwin Silan, Janice's
husband, arrived at the hospital, he also asked her who stabbed her and she reiterated that it was
Calinawan. After three (3) days, Janice died in spite of the medical treatment at the hospital.

The RTC convicted Calinawan for murder. The trial court noted that Marigor positively and categorically
identified him as the one who stabbed her mother. It noted that she was able to identify him because of his
amputated fingers. In addition, the trial court pointed out that the dying declaration of Janice to Jonathan
corroborated Marigor's statement that Calinawan killed her mother. The RTC stated that his positive
identification trumped his denial and alibi, which were considered as inherently weak defenses.

Aggrieved, Calinawan appealed before the CA.

The CA sustained Calinawan's conviction but modified the award of damages.

Issue:Whether the dying declaration of Janice is admissible in evidence.

RULING: The Court finds that Calinawan is criminally liable for the killing of Janice. The defense of
denial and alibi fails in light of positive identification

Dying Declaration; Rule on Res Gestae

Marigor's positive identification was further bolstered by the statement of Janice to Jonathan that it was
Calinawan who stabbed her.

The courts a quo considered the said statement as an admissible dying declaration. For a dying declaration
to be deemed an exception to the hearsay rule, the following conditions must concur: (a) the declaration
must concern the cause and surrounding circumstances of the declarant's death; (b) that at the time the
declaration was made, the declarant was conscious of his impending death; (c) the declarant was
competent as a witness; and
(d) the declaration is offered in a criminal case for Homicide, Murder, or Parricide where the declarant is
the victim.

In this case, the Court notes that in her affidavit, Janice said that she thought she could survive the attack.
She never thought that she was dying. In fact, she was optimistic of her recovery. In view of this, there
seems to be a doubt whether she was aware of her impending death.

Granting there is such doubt, Janice's statement, nevertheless, is admissible as an exception to the hearsay
rule for being part of res gestae.

In order for a statement to be considered part of res gestae, the following elements must concur: (a) the
principal act, the res gestae, is a startling occurrence;
(b) the statement was made before the declarant had time to contrive or devise; and
(c) the statement concerns the occurrence in question and its immediately attending circumstances.

All the foregoing elements are present in the case at bench.

Remedial Law Review II (CY 2018-2019) – Saturday 1:00pm – 5:00pm: Case Digest Compilation [Atty. Brondial] Page 415
First, the stabbing incident constituted the startling occurrence. Second, Janice never had the opportunity
to fabricate a statement implicating Calinawan because she immediately identified him as her attacker
when Jonathan saw her shortly after the assault took place. Lastly, the statement of Janice concerned the
circumstances surrounding her stabbing.

Remedial Law Review II (CY 2018-2019) – Saturday 1:00pm – 5:00pm: Case Digest Compilation [Atty. Brondial] Page 416
C.3.h.i.2 People vs. Gatarin

G.R. No. 198022; April 7, 2014; J. Peralta

FACTS: One evening, Umali was riding a bicycle on his way home when he saw Januario being mauled
by two persons opposite Dom’s Studio in Poblacion, Mabini, Batangas. Upon seeing the incident, he
stayed in front of the church until such time that the accused ran away and were chased by policemen who
alighted from the police patrol vehicle.

On the same night, SPO3 Mendoza and PO1 Coronel were on board their patrol vehicle performing their
routine patrol duty when they met two men, later identified as the accused, who were running at a fast
speed. When asked why they were running, the accused did not answer prompting the policemen to chase
them. The policemen, however, were unsuccessful in catching them and when it became evident that they
could no longer find them, they continued patrolling the area. There they saw Januario lying on the street
in front of Dom’s studio. As he was severely injured, the policemen immediately boarded Januario to the
patrol vehicle and brought him to the Zigzag Hospital. While inside the vehicle, SPO3 Mendoza asked
Januario who hurt him. He answered that it was "Jay-R and his uncle" who stabbed him.

At the Zigzag Hospital, Januario was attended to by Dr. Rasa who found him in critical condition. Three
fatal wounds caused by a bladed weapon were found in Januario’s body which eventually caused his
death.

ISSUE: Whether or not the victim’s statement to SPO3 Mendoza is a dying declaration

RULING: NO. A dying declaration, although generally inadmissible as evidence due to its hearsay
character, may nonetheless be admitted when the following requisites occur, namely: (a) the declaration
concerns the cause and the surrounding circumstances of the declarant’s death; (b) it is made when death
appears to be imminent and the declarant is under a consciousness of impending death; (c) the declarant
would have been competent to testify had he or she survived; and (d) the dying declaration is offered in a
case in which the subject of inquiry involves the declarant’s death.

In the case at bar, it appears that not all the requisites of a dying declaration are present. From the records,
no questions relative to the second requisite was propounded to Januario. It does not appear that the
declarant was under the consciousness of his impending death when he made the statements.
The rule is that, in order to make a dying declaration admissible, a fixed belief in inevitable and imminent
death must be entered by the declarant. It is the belief in impending death and not the rapid succession of
death in point of fact that renders a dying declaration admissible. The test is whether the declarant has
abandoned all hopes of survival and looked on death as certain impending. Thus, the utterances made by
Januario could not be considered as a dying declaration.

ISSUE: Whether the test of admissibility to SPO3 Mendoza as a part of res gestae is applicable in the
case.
RULING: Even if Januario’s utterance could not be appreciated as a dying declaration, his statements
may still be appreciated as part of the res gestae.

Res gestae refers to the circumstances, facts, and declaration that grow out of the main fact and serve to
illustrate its character and are so spontaneous and contemporaneous with the main fact as to exclude the
idea of deliberation and fabrication.
The test of admissibility of evidence as a part of the res gestae is, therefore, whether the act, declaration,
or exclamation is so interwoven or connected with the principal fact or event that it characterizes as to be
regarded as a part of the transaction itself, and also whether it clearly negates any premeditation or
purpose to manufacture testimony.

When Januario gave the identity of the assailants to SPO3 Mendoza, he was referring to a startling
occurrence which is the stabbing by accused and his co-accused. At that time, Januario and the witness
were in the vehicle that would bring him to the hospital, and thus, had no time to contrive his
identification of the assailant. His utterance about assailant and his co-accused having stabbed him, in
answer to the question of SPO3 Mendoza, was made in spontaneity and only in reaction to the startling
occurrence.

Definitely, the statement is relevant because it identified the accused as the authors of the crime. Verily,
the killing of Januario perpetrated by accused, is adequately proven by the prosecution.

Remedial Law Review II (CY 2018-2019) – Saturday 1:00pm – 5:00pm: Case Digest Compilation [Atty. Brondial] Page 417
C.3.h.i.3 PEOPLE OF THE PHILIPPINES, Plaintiff-Appellee, v. BERNABE P. PALANAS ALIAS
"ABE"

G.R. No. 214453; June 17, 2015

FACTS:At around 6:40 in the morning of March 26, 2006, SPO2 Borre took his five (5)-month-old
grandson outside his residence at Block 14, Kenneth Street corner Eusebio Avenue, Pasig City. PO3
Leopoldo Zapanta (PO3 Zapanta), who slept at SPO2 Borre's residence, was watching television when
four (4) successive gunshots rang out. PO3 Zapanta looked through the open door of SPO2 Borre's house
and saw two (2) men armed with .38 caliber revolvers standing a meter away from SPO2 Borre. He saw
Palanas deliver the fourth shot to SPO2 Borre, but he could not identify the other shooter. Thereafter, the
two (2) assailants fled on a motorcycle.

PO3 Zapanta, together with SPO2 Borre's stepson Ramil Ranola (Ramil), brought SPO2 Borre to the
Pasig City General Hospital. On the way to the hospital, SPO2 Borre told Ramil and PO3 Zapanta that it
was "Abe," "Aspog," or "Abe Palanas" - referring to his neighbor, Palanas -who shot him. This statement
was repeated to his wife, Resurreccion Borre (Resurreccion), who followed him at the hospital. At around
11 o'clock in the morning of even date, SPO2 Borre died due to gunshot wounds on his head and trunk.

For his part, Palanas interposed the defense of denial and alibi. He claimed that on March 25, 2006 he
was in Paranaque City attending to the needs of his sick father. The next day, he went to a baptism in
Tondo, Manila and stayed there from morning until 9 o'clock in the evening, after which he returned to
his father in Paranaque City. He maintained that he was not aware of the death of SPO2 Borre until he
was informed by a neighbor that Resurreccion was accusing him of killing her husband. He also denied
any knowledge why Resurreccion would blame him for SPO2 Borre's death.

In a Decision dated October 20, 2010, the RTC convicted Palanas of the crime of Murder and sentenced
him to suffer the penalty of reclusion perpetua.

The RTC found that the prosecution had established beyond reasonable doubt that Palanas and his
companion were the ones who killed SPO2 Borre through the positive identification of the eyewitnesses
to the incident. Moreover, SPO2 Borre's statements that Palanas shot him constituted an ante mortem
statement and formed part of the res gestae, and, thus, admissible as evidence against Palanas.

In a Decision dated January 16, 2014, the CA affirmed the RTC's ruling.

ISSUE:Whether or not the dying declaration of Borre can be admiited as evidence into court warranting
the convinction of Palanas

RULING: Yes, it can be admitted.

The CA is also correct in admitting SPO2 Borre's statements on his way to the hospital as evidence, both
as a dying declaration and as part of the res gestae.

For a dying declaration to constitute an exception to the hearsay evidence rule, four (4) conditions must
concur: (a) the declaration must concern the cause and surrounding circumstances of the declarant's death;
(b) that at the time the declaration was made, the declarant is conscious of his impending death; (c) the
declarant was competent as a witness; and (d) the declaration is offered in a criminal case for Homicide,
Murder, or Parricide where the declarant is the victim.

On the other hand, a statement to be deemed to form part of the res gestae, and thus, constitute another
exception to the rule on hearsay evidence, requires the concurrence of the following requisites: (a) the
principal act, the res gestae, is a startling occurrence; (b) the statements were made before the declarant
had time to contrive or devise; and (c) the statements must concern the occurrence in question and its
immediately attending circumstances.

In the case at bar, SPO2 Borre's statements constitute a dying declaration, given that they pertained to the
cause and circumstances of his death and taking into consideration the number and severity of his
wounds, it may be reasonably presumed that he uttered the same under a fixed belief that his own death
was already imminent. This declaration is considered evidence of the highest order and is entitled to
utmost credence since no person aware of his impending death would make a careless and false
accusation. Verily, because the declaration was made in extremity, when the party is at the point of death

Remedial Law Review II (CY 2018-2019) – Saturday 1:00pm – 5:00pm: Case Digest Compilation [Atty. Brondial] Page 418
and when every motive of falsehood is silenced and the mind is induced by the most powerful
considerations to speak the truth, the law deems this as a situation so solemn and awful as creating an
obligation equal to that which is imposed by an oath administered in court.

In the same vein, SPO2 Borre's statements may likewise be deemed to form part of the res gestae. "Res
gestae refers to the circumstances, facts, and declarations that grow out of the main fact and serve to
illustrate its character and are so spontaneous and contemporaneous with the main fact as to exclude the
idea of deliberation and fabrication. The test of admissibility of evidence as a part of the res gestae is,
therefore, whether the act, declaration, or exclamation is so intimately interwoven or connected with the
principal fact or event that it characterizes as to be regarded as a part of the transaction itself, and also
whether it clearly negates any premeditation or purpose to manufacture testimony." In this case, SPO2
Borre's statements refer to a startling occurrence, i. e., him being shot by Palanas and his companion.
While on his way to the hospital, SPO2 Borre had no time to contrive the identification of his assailants.
Hence, his utterance was made in spontaneity and only in reaction to the startling occurrence. Definitely,
such statement is relevant because it identified Palanas as one of the authors of the crime. Therefore, the
killing of SPO2 Borre, perpetrated by Palanas, is adequately proven by the prosecution.

Remedial Law Review II (CY 2018-2019) – Saturday 1:00pm – 5:00pm: Case Digest Compilation [Atty. Brondial] Page 419
C.3.h.ii People of the Philippines vs Arnel Bernal

GR No. 132791 & 140465-66; September 2, 2002; Corona, J:

FACTS: In the evening of 6 February 1996, the accused (Bernal) were on board a tricycle on their way to
the Benedisco Pub. While Fernando and Rey were at the gate, they heard a gunshot. When Fernando look
at the tricycle, he saw the accused holding a gun. Fernando rushed to the tricycle. Fernando heard a
second gunshot and he knew that the accused shot Pedrito. Fernando immediately attacked the accused
and got a hold of him until the police arrived.

Police superintendent Sarte and his men picked up the accused. SPO4 Napoleon Pascual testified that the
accused is not a holder of any license or authorized to possess any kind of firearm. Accused denied
culpability and offered his own recollection of the incident. The court find the accused guilty beyond
reasonable doubt of the crime of murder.

ISSUE:Whether or not the trial court erred in appreciating evident premeditation and treachery.

RULING:No. There was no evident premeditation and treachery.

Evident premeditation cannot be deduced from mere presumption or speculation. It must be proven
clearly. Evident premeditation cannot be appreciated without proof of how and when the plan to kill was
hatched or how much time elapsed before it was carried out.

We find no evidence directly showing any pre-conceived plan or devise employed by the accused to kill
the victim. Accused did not went to the pub to kill the victim but to attend important matters. The mere
existence of grudge between the parties is not sufficient to establish premeditated killing.

Remedial Law Review II (CY 2018-2019) – Saturday 1:00pm – 5:00pm: Case Digest Compilation [Atty. Brondial] Page 420
C.3.h.iiiTizon vs. CA

G.R. No. 121027; July 31, 1997; REGALADO, J.

FACTS: Petitioners Tison and Dezoller are the niece and nephew of the deceased
TeodoraDezollerGuerro, who is the sister of petitioners‘ father, HermogenesDezoller. The deceased was
survived by her husband Martin Guerrero who upon Teodora‘s death audjudicated unto himself the land
in dispute then sold it to respondent Domingo. Subsequently, petitioners filed an action for reconveyance,
claiming that they are entitled to inherit ½ of the property in question by right of representation.
During the hearing, petitioner Tison was presented as the lone witness with the ff documentary evidence
to prove their filiation to their father and there aunt: a family picture; baptismal certificates of Teodora
and HermogenesDezoller; certificates of destroyed records of birth of TeodoraDezoller and
HermogenesDezoller; death certificates of HermogenesDezoller and TeodoraDezoller Guerrero;
certification of destroyed records of live birth of Corazon and Rene Dezoller; joint affidavits of Pablo
Verzosa and MelitonSitjar attesting to the parents, date and place of birth of Corazon and Rene Dezoller;
joint affidavit of Juliana Cariaga and Manuela Cariaga attesting to the fact of marriage between Martin
Guerrero and TeodoraDezoller; and the marriage certificate of Martin and Teodora Guerrero.
Private respondent Domingo filed a Demurrer to Evidence on the ground that petitioners failed to prove
their legitimate filiation with the deceased Teodora Guerrero in accordance with Art. 172 of the Family
Code. The trial court granted the demurrer to evidence and dismissed the complaint for reconveyance. In
upholding the dismissal, respondent Court of Appeals declared that the documentary evidence presented
by herein petitioners, such as the baptismal certificates, family picture, and joint affidavits are all
inadmissible and insufficient to prove and establish filiation. Hence, this appeal by certiorari.

ISSUE: WON the testimony of petitioner Tison, that her deceased aunt declared her as niece, is
admissible in evidence as a declaration about pedigree.

RULING: Yes. The primary proof to be considered in ascertaining the relationship between the parties
concerned is the testimony of Corazon DezollerTison to the effect that TeodoraDezoller Guerrero in her
lifetime, or sometime in 1946, categorically declared that the former is Teodora's niece. Such a statement
is considered a declaration about pedigree which is admissible, as an exception to the hearsay rule, under
Section 39, Rule 130 of the Rules of Court, subject to the following conditions: (1) that the declarant is
dead or unable to testify; (2) that the declarant be related to the person whose pedigree is the subject of
inquiry; (3) that such relationship be shown by evidence other than the declaration; and (4) that the
declaration was made ante litem motam, that is, not only before the commencement of the suit involving
the subject matter of the declaration, but before any controversy has arisen thereon.

There is no dispute with respect to the first, second and fourth elements. What remains for analysis is the
third element, that is, whether or not the other documents offered in evidence sufficiently corroborated the
declaration made by TeodoraDezoller Guerrero in her lifetime regarding the pedigree of petitioner
Corazon DezollerTison or, if at all, it is necessary to present evidence other than such declaration.

A distinction must be made as to when the relationship of the declarant may be proved by the very
declaration itself, or by other declarations of said declarant, and when it must be supported by evidence
aliunde. – The general rule, therefore, is that where the party claiming seeks recovery against a relative
common to both claimant and declarant, but not from the declarant himself or the declarant's estate, the
relationship of the declarant to the common relative may not be proved by the declaration itself. There
must be some independent proof of this fact. As an exception, the requirement that there be other proof
than the declarations of the declarant as to the relationship, does not apply where it is sought to reach the
estate of the declarant himself and not merely to establish a right through his declarations to the property
of some other member of the family.

We are sufficiently convinced, and so hold, that the present case is one instance where the general
requirement on evidence aliundemay be relaxed. Petitioners are claiming a right to part of the estate of
the declarant herself. Conformably, the declaration made by TeodoraDezoller Guerrero that petitioner
Corazon is her niece, is admissible and constitutes sufficient proof of such relationship, notwithstanding
the fact that there was no other preliminary evidence thereof, the reason being such declaration is
rendered competent by virtue of the necessity of receiving such evidence to avoid a failure of justice.
More importantly, there is in the present case an absolute failure by all and sundry to refute that
declaration made by the decedent.
Applying the general rule in the present case would nonetheless produce the same result. For while the
documentary evidence submitted by petitioners do not strictly conform to the rules on their admissibility,

Remedial Law Review II (CY 2018-2019) – Saturday 1:00pm – 5:00pm: Case Digest Compilation [Atty. Brondial] Page 421
we are however of the considered opinion that the same may be admitted by reason of private
respondent's failure to interpose any timely objection thereto at the time they were being offered in
evidence. It is elementary that an objection shall be made at the time when an alleged inadmissible
document is offered in evidence, otherwise, the objection shall be treated as waived, since the right to
object is merely a privilege which the party may waive.

Accordingly, the Certificate of Marriage (Exhibit S) wherein it is indicated that the parents of
TeodoraDezoller are IsabeloDezoller and Cecilia Calpo, as well as the Certificates of Baptism of
TeodoraDezoller (Exhibit H) and HermogenesDezoller (Exhibit J) which both reflect the names of their
parents as IsabeloDezoller and Cecilia Calpo, to show that HermogenesDezoller is the brother of
TeodoraDezoller Guerrero; and the Death Certificate of HermogenesDezoller (Exhibit K) the entries
wherein were made by petitioner Corazon DezollerTison as his daughter, together with the Joint
Affidavits of Pablo Verzosa and MelitonSitjar (Exhibits N and P), to prove that herein petitioners are the
children of HermogenesDezoller — these can be deemed to have sufficiently established the relationship
between the declarant and herein petitioners. This is in consonance with the rule that a prima facie
showing is sufficient and that only slight proof of the relationship is required. Finally, it may not be amiss
to consider as in the nature of circumstantial evidence the fact that both the declarant and the claimants,
who are the subject of the declaration, bear the surname Dezoller.

Remedial Law Review II (CY 2018-2019) – Saturday 1:00pm – 5:00pm: Case Digest Compilation [Atty. Brondial] Page 422
C.3.h.ivFRANCISCO L. JISON vs. COURT OF APPEALS and MONINA JISON

G.R. No. 124853; February 24, 1998; DAVIDE, JR., J.:

FACTS: Monina Jison filed a petition for recognition as an illegitimate child of petitioner Francisco
Jison. In her complaint, she alleged that: (1) at the end of 1945 or the start of 1946, however,
FRANCISCO impregnated Esperanza F. Amolar (who was then employed as the nanny of
FRANCISCO's daughter, Lourdes); (2) MONINA was born on 6 August 1946, in Dingle, Iloilo; (3) since
childhood, she had enjoyed the continuous, implied recognition as an illegitimate child of FRANCISCO
by his acts and that of his family; and (4) that FRANCISCO gave her support and spent for her education,
such that she obtained a Master's degree, became a certified public accountant (CPA) and eventually, a
Central Bank examiner. At trial on the merits, MONINA presented as documentary evidence letters
written by Francisco’s relatives as proof of her recognition as illegitimate daughter of the latter. The trial
court dismissed the complaint. On appeal, CA reversed the ruling of the trial court and held that Monina
was able to establish her filiation as FRANCISCO's illegitimate daughter not just preponderant but
overwhelming evidence on record. Francisco elevated the case before the SC and assailed the
admissibility of the letters of his relatives.

ISSUE: Whether or not letter of the relatives of a putative father is admissible in evidence as part of the
family reputation.

RULING: NO. Under Rule 130, Section 39, the contents of these documents may not be admitted, there
being no showing that the declarants-authors were dead or unable to testify, neither was the relationship
between the declarants and MONINA shown by evidence other than the documents in question. Neither
may it be admitted under under Rule 130, Section 40. Rule 130, Section 40, provides: Sec. 40. Family
reputation or tradition regarding pedigree. — The reputation or tradition existing in a family previous to
the controversy, in respect to the pedigree of any one of its members, may be received in evidence if the
witness testifying thereon be also a member of the family, either by consanguinity or affinity. Entries in
family bibles or other family books or charts, engravings on rings, family portraits and the like may be
received as evidence of pedigree. (emphasis supplied) It is evident that this provision may be divided into
two (2) parts: the portion containing the first underscored clause which pertains to testimonial evidence,
under which the documents in question may not be admitted as the authors thereof did not take the
witness stand; and the section containing the second underscored phrase. What must then be ascertained
is whether letter presented in this case as private documents, fall within the scope of the clause "and the
like" as qualified by the preceding phrase "entries in family bibles or other family books or charts,
engravings on rights and family portraits," We hold that the scope of the enumeration contained in the
second portion of this provision, in light of the rule of ejusdem generis, is limited to objects which are
commonly known as "family possessions," or those articles which represent, in effect, a family's joint
statement of its belief as to the pedigree of a person. These have been described as objects "openly
exhibited and well known to the family," or those "which, if preserved in a family, may be regarded as
giving a family tradition." Plainly then, letters, as private documents not constituting "family possessions"
as discussed above, may not be admitted on the basis of Rule 130, Section 40. Neither may these exhibits
be admitted on the basis of Rule 130, Section 41 regarding common reputation, it having been observed
that: the weight of authority appears to be in favor of the theory that it is the general repute, the common
reputation in the family, and not the common reputation in community, that is a material element of
evidence going to establish pedigree. Thus, matters of pedigree may be proved by reputation in the
family, and not by reputation in the neighborhood or vicinity, except where the pedigree in question is
marriage which may be proved by common reputation in the community.

Remedial Law Review II (CY 2018-2019) – Saturday 1:00pm – 5:00pm: Case Digest Compilation [Atty. Brondial] Page 423
C.3.h.vi.1 People vs. Feliciano

G.R. No. 196735; May 5, 2014

FACTS: On December 8, 1994, at around 12:30 to 1:00 in the afternoon, seven (7) members of the
Sigma Rho fraternity were eating lunch at the Beach House Canteen, near the Main Library of the
University of the Philippines, Diliman, when they were attacked by several masked men carrying baseball
bats and lead pipes. An information for murder, docketed as Criminal Case No. Q95-6113 3, was filed
against several members of the Scintilla Juris fraternity, namely, Danilo Feliciano, Jr., Julius Victor L.
Medalla,et al with the Regional Trial Court of Quezon City, Branch 219. According to Leandro Lachica,
Grand Archon of Sigma Rho Fraternity, he looked around when Venturina shouted, and he saw about ten
(10) men charging toward them.10 The men were armed with baseball bats and lead pipes, and their
heads were covered with either handkerchiefs or shirts. He was lying on his back and when he looked up,
he saw another group of four (4) to five (5) men coming toward him, led by Benedict Guerrero. On the
night of December 8, 1994, the officers of Sigma Rho advised the victims to lodge their complaints with
the National Bureau of Investigation. Their counsel, Atty. Frank Chavez, told the U.P. Police that the
victims would be giving their statements before the National Bureau of Investigation, promising to give
the U.P. Police copies of their statements. On February 28, 2002, the trial court rendered its decision with
the finding that Robert Michael Alvir, Danilo Feliciano, Jr., Christopher Soliva, Julius Victor Medalla,
and Warren Zingapan were guilty beyond reasonable doubt of murder and attempted murder and were
sentenced to, among other penalties, the penalty of reclusion perpetua. CA affirmed the decision of the
Regional Trial Court for the conviction of Felciano et,al.

ISSUE: WON the prosecution was able to prove beyond reasonable doubt that accused-appellants
attacked private complainants and caused the death of Dennis Venturina.

RULING: YES. The introduction of the prosecution of testimonial evidence that tends to prove that the
accused were masked but the masks fell off does not prevent them from including disguise as an
aggravating circumstance. As a general rule, the findings of fact by the trial court, when affirmed by the
appellate court, are given great weight and credence on review. It is the policy of this Court to sustain the
factual findings of the trial court on the reasonable assumption that it is in a better position to assess the
evidence before it, particularly the testimonies of the witnesses, who reveal much of themselves by their
deportment on the stand. The exception that makes the rule is where such findings arc clearly arbitrary or
erroneous as when they are tainted with bias or hostility or are so lacking in basis as to suggest that they
were reached without the careful study and perceptiveness that should characterize a judicial decision. As
a general rule, "[a] witness can testify only to the facts he knows of his personal knowledge; that is, which
are derived from his own perception,”. The term res gestae has been defined as "those circumstances
which are the undersigned incidents of a particular litigated act and which are admissible when illustrative
of such act. In a general way, res gestae refers to the circumstances, facts, and declarations that grow out
of the main fact and serve to illustrate its character and are so spontaneous and contemporaneous with the
main fact as to exclude the idea of deliberation and fabrication. Persons who witness an event perceive
the same from their respective points of reference. Therefore, almost always, they have different accounts
of how it happened. Certainly, we cannot expect the testimony of witnesses to a crime to be consistent in
all aspects because different persons have different impressions and recollections of the same incident.
The statements made by the bystanders, although admissible, have little persuasive value since the
bystanders could have seen the events transpiring at different vantage points and at different points in
time.

DOCTRINE: The term res gestae has been defined as "those circumstances which are the undersigned
incidents of a particular litigated act and which are admissible when illustrative of such act. In a general
way, res gestae refers to the circumstances, facts, and declarations that grow out of the main fact and
serve to illustrate its character and are so spontaneous and contemporaneous with the main fact as to
exclude the idea of deliberation and fabrication.

Remedial Law Review II (CY 2018-2019) – Saturday 1:00pm – 5:00pm: Case Digest Compilation [Atty. Brondial] Page 424
C.3.h.vi.2 PEOPLE OF THE PHILIPPINES, Plaintiff-Appellee, vs. GILBERTO VILLARICO, SR.
@ "BERTING", GILBERTO VILLARICO, JR., JERRY RAMENTOS, and RICKY
VILLARICO,

G.R. No. 158362; April 4, 2011

FACTS: Version of the Prosecution


At around 7:50 p.m. on August 8, 1999, Haide was busy preparing dinner in the kitchen. Haide’s sister-
in-law Remedios Cagatan was attending to her child who was answering the call of nature near the toilet.
From where she was, Remedios saw all the accused as they stood at the rear of the kitchen aiming their
firearms at the door – Ricky Villarico was at the left side, and Gilberto, Jr. stood behind him, while
Gilberto, Sr. was at the right side, with Ramentos behind him. When Gilberto, Jr. noticed Remedios, he
pointed his gun at her, prompting Remedios to drop to the ground and to shout to Lolita Cagatan, her
mother-in-law and Haide’s mother: Nay, Nay tawo Nay (Mother, mother, there are people outside,
mother). At that instant, Remedios heard three gunshots. Francisco Cagatan, the father of Haide, also
heard the gunshots just as he was coming out of the toilet, making him instinctively jump into a hole,
from where he was able to see and recognize Gilberto, Sr., Gilberto, Jr. and Ricky who were then
standing by the kitchen door. Lolita also heard the gunshots while she was in the sala. She recalled that
Haide then came towards her from the kitchen, asking for help and saying that he was shot by Berting. At
that, she and Remedios brought the wounded Haide to Clinica Ozarraga, where he was treated for gunshot
wounds on his left scapular region (back of left shoulder) and right elbow. He succumbed shortly
thereafter due to hypovolemic shock or massive loss of blood.

Version of the Defense


The accused denied the accusations and each proffered an alibi. Gilberto, Sr. claimed that he was sleeping
in his home with a fever when he heard a gunshot. He insisted that he learned that Haide had been shot
only in the next morning. His denial and alibi were corroborated by his wife Carmelita and his daughter
Jersel. Gilberto, Jr. testified that on the day of the incident, he went to Liloan, Bonifacio, Misamis
Occidental at around 5:00 p.m. to visit his girlfriend together with Charlie Bacus and Randy Hernan.
They stayed there until 9:00 p.m.It was only in the morning that Randy’s father informed them that Haide
had been shot Ricky declared that he stayed throughout the whole evening of August 8, 1999 in the house
of his aunt Flordeliza. Myrna Hernan, a neighbor of Flordeliza, corroborated his testimony. Ramentos
alleged that he was drinking tuba with others at the store owned by Cinderella Bacus at the time of the
shooting; and that he went home at around 9:00 p.m. after his group was done drinking. He did not recall
hearing any gunshots while drinking and came to know of the shooting only from a certain Anecito
Duyag on the following morning.

After trial, the RTC convicted the four accused of homicide aggravated by dwelling. The RTC accorded
faith to the positive identification of the accused by the Prosecution’s witnesses, and disbelieved their
denial and alibis due to their failure to show the physical improbability for them to be at the crime scene,
for the distances between the crime scene and the places where the accused allegedly were at the time of
the commission of the crime were shown to range from only 100 to 700 meters. On intermediate review,
the CA modified the RTC’s decision, holding instead that murder was established beyond reasonable
doubt because the killing was attended by treachery.

ISSUE:Whether or not an identification, to be positive, have to be made by a witness who actually saw
the assailants

RULING: No. Positive identification refers to proof of identity of the assailant.

The statement of Haide to his mother that he had just been shot by the group of Berting – uttered in the
immediate aftermath of the shooting where he was the victim – was a true part of the res gestae. The
statement was admissible against the accused as an exception to the hearsay rule under Section 42, Rule
130 of the Rules of Court, which provides:
Section 42. Part of the res gestae. - Statements made by a person while a startling
occurrence is taking place or immediately prior or subsequent thereto with respect to the
circumstances thereof, may be given in evidence as part of the res gestae. So, also,
statements accompanying an equivocal act material to the issue, and giving it a legal
significance, may be received as part of the res gestae.

The term res gestae refers to "those circumstances which are the undesigned incidents of a particular
litigated act and which are admissible when illustrative of such act." In a general way, res gestae includes

Remedial Law Review II (CY 2018-2019) – Saturday 1:00pm – 5:00pm: Case Digest Compilation [Atty. Brondial] Page 425
the circumstances, facts, and declarations that grow out of the main fact and serve to illustrate its
character and which are so spontaneous and contemporaneous with the main fact as to exclude the idea of
deliberation and fabrication. The rule on res gestae encompasses the exclamations and statements made
by either the participants, victims, or spectators to a crime immediately before, during, or immediately
after the commission of the crime when the circumstances are such that the statements were made as a
spontaneous reaction or utterance inspired by the excitement of the occasion and there was no opportunity
for the declarant to deliberate and to fabricate a false statement.

The test of admissibility of evidence as a part of the res gestae is whether the act, declaration, or
exclamation is so intimately interwoven or connected with the principal fact or event that it characterizes
as to be regarded a part of the principal fact or event itself, and also whether it clearly negatives any
premeditation or purpose to manufacture testimony. A declaration or an utterance is thus deemed as part
of the res gestae that is admissible in evidence as an exception to the hearsay rule when the following
requisites concur: (a) the principal act, the res gestae, is a startling occurrence; (b) the statements were
made before the declarant had time to contrive or devise; and (c) the statements must concern the
occurrence in question and its immediately attending circumstances.

We find that the requisites concurred herein. Firstly, the principal act – the shooting of Haide – was a
startling occurrence. Secondly, his statement to his mother about being shot by the group of Berting was
made before Haide had time to contrive or to devise considering that it was uttered immediately after the
shooting. And, thirdly, the statement directly concerned the startling occurrence itself and its attending
circumstance (that is, the identities of the assailants). Verily, the statement was reliable as part of the res
gestae for being uttered in spontaneity and only in reaction to the startling occurrence.

Remedial Law Review II (CY 2018-2019) – Saturday 1:00pm – 5:00pm: Case Digest Compilation [Atty. Brondial] Page 426
C.3.h.vi.3 People of the Philippines, plaintiff-appellee, vs. Anthony Melchor Palmones, Anthony
Baltazar Palmones, accused-appellants.

G.R. No.136303; July 18, 2000; Gonzaga-Reyes, J.

FACTS: An information charging accused-appellants of the crime of murder of SPO2 Asim Mamansal,
hitting and inflicting upon the latter gunshot wounds on the vital parts of his body which caused his death.

The prosecution first presented Sonny Boy Redovan stating that upon being informed that something
something had happened to his uncle SP02 Asim Mamansal they rushed to the Hospital and proceeded to
the emergency room. Upon seeing his uncle, the witness went near the victim and asked what had
happened to which the victim answered that he had been waylaid. Thereafter Redovan asked the victim
who the perpetrators were and the victim answered that it was "Juany and Tony Palmones", the
nicknames of the two accused-appellants. Redovan claimed that while he was talking with his uncle, there
were attendants, nurses, and other bystanders whom he did not know present inside the emergency room.

On cross-examination, he testified that he was able to talk with his uncle for about one hour and that the
most important part of their conversation was the identification of his uncle's assailants. He stated that it
did not occur to his mind to immediately report to the police what his uncle had told him as his mind was
troubled at that time.

[…]

The third witness for the prosecution was P/Inspt Alexander Camilon-Tagum who testified that upon
failing to pursue a motorcycle which was suspected to have been ridden by the assailants to escape from
the place of incident, he and his men proceeded to the residence of the suspects, identified in the radio by
a certain PO3 Aniceta, where the brother of the accused-appellants, Triny Palmones, met them. He asked
Triny Palmones where his brothers were and the latter responded that he didn't know. He then asked
Triny Palmones whether his brothers owned a motorcycle and the latter admitted that they owned a
Kawasaki motorcycle which matched the description of the motorcycle he had been chasing. He then told
his men to continue pursuing the assailants and after exhausting all efforts, he proceeded to the
Kidapawan Doctor's Hospital. He confronted the victim in the emergency room and asked him about his
assailants. The victim answered that it was Juany and Tony Palmones. At that time, he claimed that Dr.
Aguayo and two other medical ladies were inside the room.

On cross-examination, he testified that he was able to speak with Alice Villamor about the incident but
that she told him that she was not able to identify the assailant even though she was right beside the
victim because of darkness. He admitted that when he went to the hospital, he was already entertaining
the idea that the suspects were Juany and Tony Palmones because of the radio call he received earlier. He
likewise admitted that the only question which he asked the victim was "who shot you?" and that he was
not able to reduce his findings to writing.

For their part, accused-appellants presented ten (10) witnesses to support their case.

RTC rendered its questioned decision finding accused-appellants guilty of the crime of murder.

Accused-appellants filed an appeal with the SC.

The OSG, for its part, filed a Manifestation in Lieu of Brief where it recommended that the accused-
appellants be acquitted of the crime charged against them, where the OSG reasoned that the identity of
the assailants was not sufficiently established by the evidence of the prosecution and that the trial court
erred in admitting the alleged dying declaration of the victim as an exception to the hearsay rule.

ISSUE: WON the RTC erred in convicting accused-appellants of murder which decision was based
largely on the alleged dying declaration of the victim made to two witnesses of the prosecution and the
apparent weakness of their defense of alibi.

RULING: YES.

As a rule, a dying declaration is hearsay, and is inadmissible as evidence. This is pursuant to Rule 130,
section 30 of the Rules of Court.

Remedial Law Review II (CY 2018-2019) – Saturday 1:00pm – 5:00pm: Case Digest Compilation [Atty. Brondial] Page 427
There are several exceptions however to the rule of inadmissibility of hearsay evidence, the first one of
which is the admissibility of dying declarations given under the circumstances specified in Section 31,
Rule 130 of the Rules of Court.

As such, the requirements for the admissibility of an ante mortem statement are:

f. it must concern the crime and the surrounding circumstances of the declarant's death;
g. at the time it was made, the declarant was under a consciousness of impending death;
h. the declarant was competent as a witness; and
i. the declaration was offered in a criminal case for murder, murder or parricide win which the
decedent was the victim.

In cases where an alleged dying declaration is sought to be admitted, it must be proven that that the
declaration was made "under a consciousness of impending death" which means simply that the declarant
is fully aware that he is dying or going to die from his wounds or injuries soon or imminently, or shall
have a complete conviction that death is at hand, or there must be "a settled hopeless expectation."

In the instant case, it was not established by the prosecution that the statements of the declarant
concerning the cause and surrounding circumstances of his death were made under the consciousness of
impending death. No proof to this effect was ever presented by the prosecution. It was not shown whether
Sonny Boy Redovan or Inspector Alexander Tagum ever asked the victim whether he believed that he
was going to die out of his injuries or any other similar question. Sonny Boy Redovan claimed that he
was able to talk with the victim for around an hour but the only thing he revealed of their conversation
was the alleged identification of the victim of his two assailants. For his part, Inspector Tagum admitted
that the only question he asked of the victim was if the victim knew who had shot him

Neither may the alleged statements attributed to the victim be admissible as part of the res gestae. Res
gestae refers to those exclamations and statements made by either the participants, victims, or spectators
to a crime immediately before, during, or immediately after the commission of a crime, when the
circumstances are such that the statements were made as a spontaneous reaction or utterance inspired by
the excitement of the occasion and there was no opportunity for the declarant to deliberate and to
fabricate a false statement.

In order to admit statements as evidence part of the res gestae, the element of spontaneity is critical. The
following factors have generally been considered in determining whether statements offered in evidence
as part of the res gestae have been made spontaneously:

3) the time that lapsed between the occurrence of the act or transaction and the making of the
statement;
4) the place where the statement was made;
5) the condition of the declarant when he made the statement;
6) the presence or absence of intervening events between the occurrence and the statement relative
thereto; and
7) the nature and circumstances of the statement itself.

Tested against these factors to test the spontaneity of the statements attributed to the victim, we rule that
these statements fail to qualify as part of the res gestae. When Mamansal allegedly uttered the statements
attributed to him, an appreciable amount of time had already elapsed from the time that he was shot as the
victim was shot at around 10:00 p.m. but he only uttered the statements attributed to him about 30
minutes to an hour later. Moreover, he allegedly made these statements not at the scene of the crime but at
the hospital where he was brought for treatment. Likewise, the trip from the scene of the crime to the
hospital constituted an intervening event that could have afforded the victim opportunity for deliberation.
These circumstances, taken together, indubitably show that the statements allegedly uttered by Mamansal
lack the requisite spontaneity in order for these to be admitted as part of the res gestae.

[…] the weakness of the alibi of the two accused-appellants cannot be held against them in view of the
absence of a clear and positive identification of them as the perpetrators of the crime. And while their
alibi may not have been proven so satisfactorily as to leave no room for doubt, such an infirmity cannot
strengthen the weakness of the prosecution's evidence, the reason being that in a criminal prosecution, the
State must rely on the strength of its own evidence and not on the weakness of the defense.

Remedial Law Review II (CY 2018-2019) – Saturday 1:00pm – 5:00pm: Case Digest Compilation [Atty. Brondial] Page 428
DOCTRINE: Res gestae refers to those exclamations and statements made by either the participants,
victims, or spectators to a crime immediately before, during, or immediately after the commission of a
crime, when the circumstances are such that the statements were made as a spontaneous reaction or
utterance inspired by the excitement of the occasion and there was no opportunity for the declarant to
deliberate and to fabricate a false statement.

In order to admit statements as evidence part of the res gestae, the element of spontaneity is critical. The
following factors have generally been considered in determining whether statements offered in evidence
as part of the res gestae have been made spontaneously:

10) the time that lapsed between the occurrence of the act or transaction and the making of the
statement;
11) the place where the statement was made;
12) the condition of the declarant when he made the statement;
13) the presence or absence of intervening events between the occurrence and the statement relative
thereto; and
14) the nature and circumstances of the statement itself.

Remedial Law Review II (CY 2018-2019) – Saturday 1:00pm – 5:00pm: Case Digest Compilation [Atty. Brondial] Page 429
C.3.h.vii PHILIPPINE AIRLINES, INC. vs. JAIME J. RAMOS et al.

G.R. No. 92740; March 23, 1992

FACTS: Respondents filed an action for damages against petitioner alleging the following: (1) they are
passengers of PAL Flight No. 264 on September 24, 1985; (2) they check-in at least one (1) hour before
the published departure time; (3) no one was at the check-in counter until 30 minutes before departure;
(4)upon checking-in, they were informed that their tickets were cancelled and the seats awarded to chance
passengers; (5) they have to take the bus instead; and (6) they suffered damages due to the cancellation.
Petitioner disclaims any liability, claiming that the non-accommodation of Respondent on said flight was
due to their having check-in late for their flight.

During the trial, defendant presented the check-in counter clerk at their Naga Branch on the date of
respondent’s scheduled flight. The clerk testified that: (1) the respondents were late and that he noted the
time of check-in on their tickets; and (2) there were other passengers who came late before the
respondents. Inrelation to the testimony, two documentary evidence were offered, namely: (1) the ticket
bearing the notation “late 4:02” of the clerk; and (2) the passenger manifest showing the other names of
other passengers who were also late.

Respondent objected to the documentary evidence submitted and argued that those are self-serving.

ISSUE: Whether or not the entries made on a ticket by employees of a party in the course of their
business may not be given weight on the ground that the same is self-serving.

RULING: The answer is in the negative. The plane tickets of the private respondents with the notation
"late 4:02" stamped on the flight coupon by the check-in clerk immediately upon the check-in of private
respondents and the passengerManifest of Flight PR 264 which showed the non-accommodation of Capati
and Go and the private respondents are entries made in the regular course of business which the, private
respondents failed to overcome with substantial and convincing evidence other than their testimonies.
Consequently, they carry more weight and credence. A writing or document made contemporaneously
with a transaction in which are evidenced facts pertinent to an issue, when admitted as proof of those
facts, is ordinarily regarded as more reliable proof and of greater probative force than the oral testimony
of a witness as to such facts based upon memory and recollection.

Spoken words could be notoriously unreliable as against a written document that speaks a uniform
language.

Private respondents’ only objection to these documents is that they are self-serving cannot be sustained.
The hearsay rule will not apply in this case as statements, acts or conduct accompanying or so nearly
connected with the main transaction as to form a part of it, and which illustrate, elucidate, qualify or
characterize the act, are admissible as part of the res gestae.

Remedial Law Review II (CY 2018-2019) – Saturday 1:00pm – 5:00pm: Case Digest Compilation [Atty. Brondial] Page 430
C.3.h.viiii.1Rudy Lao vs. Standard

August 14, 2003; Quisumbing, J.

FACTS: Rudy Lao is the owner of a Fuso truck with Plate No. FCG-538. The truck was insured with
respondent Standard Insurance Co., Inc. under Policy No. CV-21074 for the maximum amount of
P200,000 and an additional sum of P50,000 to cover any damages that might be caused to his goods.

While the policy was in effect, an accident occurred. At around 8:00 p.m. of April 24, 1985, in Barangay
Buhang, Jaro, Iloilo City, the insured truck bumped another truck, with Plate No. FBS-917, also owned
by petitioner Lao. The latter truck was running ahead of the insured truck and was bumped from the rear.
The insured truck sustained damages estimated to be around P110,692, while the damage to the other
truck and to properties in the vicinity of the accident, were placed at P35,000 more or less.

Petitioner filed a claim with the insurance company for the proceeds from his policy. However, the claim
was denied by the insurance company on the ground that when its adjuster went to investigate the matter,
it was found that the driver of the insured truck, Leonardo Anit, did not possess a proper driver’s license
at the time of the accident. The restriction in Leonardo Anit’s driver’s license provided that he can only
drive four-wheeled vehicles weighing not more than 4,500 kgs. Since the insured truck he was driving
weighed more than 4,500 kgs., he therefore violated the “authorized driver” clause of the insurance
policy. Respondent also presented a police blotter record:
xxx
C-UN-85       DAMAGE TO PROPERTY W/ PHY INJURIES – R/ IMPRUDENCE
11:30 PM – Sgt. A. Bernas informed this office that a collision took place at Brgy. Buhang, Jaro, IC.
Investigation conducted by Pat. Villahermosa, assisted by Lt. P. Baclaron (OD), disclosed that at about
8:00 PM this date at the aforementioned place, a collision took place between a truck (Hino) with Plate Nr
FB[S] 917 owned by Rudy Lao and driven by BOY GIDDIE Y COYEL, 38 yrs, a res. of Balasan, Iloilo,
with License Nr DLR 1108142 and another truck with Plate Nr. FCG-538 owned by Rudy Lao and driver
(sic) by LEONARDO ANIT Y PANES, 33 yrs, a res. of Brgy Laya, Balasan, Iloilo with License Nr
1836482.… 
xxx

Petitioner claims that at the time of the accident, it was in fact another driver named Giddie Boy Y. Coyel
who was driving the insured truck. Giddie Boy possessed a driver’s license authorizing him to drive
vehicles such as the truck which weighed more than 4,500 kgs. As evidence, petitioner presented the
Motor Vehicle Accident Report wherein the Investigating Officer, Pat. Felipe D. Villahermosa, stated that
it was Giddie Boy driving the insured truck and not Leonardo Anit. The said report was made three days
after the accident or on April 27, 1985. However, respondent insurance company was firm in its denial of
the claim.

ISSUE: WON the police blotter is admissible as evidence.

RULING: YES, the police blotter was properly admitted as they form part of official records. However,
they are of little probative value.

Entries in police records made by a police officer in the performance of the duty especially enjoined by
law are prima facie evidence of the fact therein stated, and their probative value may be either
substantiated or nullified by other competent evidence. Although police blotters are of little probative
value, they are nevertheless admitted and considered in the absence of competent evidence to refute the
facts stated therein.

In this case, the entries in the police blotter reflected the information subject of the controversy. Stated
therein was the fact that Leonardo Anit was driving the insured truck with plate number FCG-538. This
is unlike People v. Mejia, where we said that “entries in the police blotters should not be given undue
significance or probative value,” since the Court there found that “the entries in question are sadly
wanting in material particulars”.

Furthermore, in this case the police blotter was identified and formally offered as evidence. The person
who made the entries was likewise presented in court; he identified and certified as correct the entries he
made on the blotter. The information was supplied to the entrant by the investigating officer who did not
protest about any inaccuracy when the blotter was presented to him. No explanation was likewise given
by the investigating officer for the alleged interchange of names.

Remedial Law Review II (CY 2018-2019) – Saturday 1:00pm – 5:00pm: Case Digest Compilation [Atty. Brondial] Page 431
C.3.h.viiii.2 MEYNARDO SABILI vs. COMMISSION ON ELECTIONS and FLORENCIO
LIBREA

G.R. No. 193261; April 24, 2012

FACTS: When petitioner filed his COC for mayor of Lipa City for the 2010 elections, he stated therein
that he had been a resident of the city for two (2) years and eight (8) months. Prior to the 2010 elections,
he had been twice elected (in 1995 and in 1998) as Provincial Board Member representing the 4th District
of Batangas. During the 2007 elections, petitioner ran for the position of Representative of the 4th District
of Batangas, but lost. The 4th District of Batangas includes Lipa City.

However, it is undisputed that when petitioner filed his COC during the 2007 elections, he and his family
were then staying at his ancestral home in Barangay (Brgy.) Sico, San Juan, Batangas. Private respondent
Florencio Librea filed a "Petition to Deny Due Course and to Cancel Certificate of Candidacy and to
Disqualify a Candidate for Possessing Some Grounds for Disqualification" against him before the
COMELEC. Citing Section 78 in relation to Section 74 of the Omnibus Election Code, private respondent
alleged that petitioner made material misrepresentations of fact in the latter’s COC and likewise failed to
comply with the one-year residency requirement under Section 39 of the Local Government Code.
Allegedly, petitioner falsely declared under oath in his COC that he had already been a resident of Lipa
City for two years and eight months prior to the scheduled 10 May 2010 local elections.

For ease of later discussion, petitioner’s evidence shall be grouped as follows: (1) his Income Tax Returns
and corresponding Official Receipts for the years 2007 and 2008; (2) Certification from the barangay
captain of Pinagtong-ulan; (3) Affidavit of his common-law wife, Bernadette Palomares; and (4)
Affidavits from a previous property owner, neighbors, Certificate of Appreciation from the barangay
parish and Memorandum from the local chapter of Guardians Brotherhood, Inc. The COMELEC Second
Division granted the Petition of private respondent, declared petitioner as disqualified from seeking the
mayoralty post in Lipa City, and cancelled his Certificate of Candidacy. Petitioner moved for
reconsideration of the 26 January 2010 Resolution of the COMELEC, during the pendency of which the
10 May 2010 local elections were held. The next day, he was proclaimed the duly elected mayor of Lipa
City after garnering the highest number of votes cast for the said position. He accordingly filed a
Manifestation with the COMELEC en banc to reflect this fact.

The COMELEC en banc denied the Motion for Reconsideration of petitioner.


Hence, petitioner filed with this Court a Petition under Rule 64 in relation to Rule 65 of the Rules of
Court, seeking the annulment of the 26 January 2010 and 17 August 2010 Resolutions of the COMELEC.
Petitioner attached to his Petition a Certificate of Canvass of Votes and proclamation of Winning
Candidates for Lipa City Mayor and Vice-Mayor issued by the City/Municipal Board of Canvassers, as
well as a copy of his Oath of Office. He also attached to his Petition another Certification of Residency
issued by Pinagtong-ulan Barangay Captain DominadorHonrade and sworn to before a notary public.

ISSUE: Whether the COMELEC erred when it did not consider the Certification issued by Pinagtong-
ulan Barangay Captain DominadorHonrade on the ground that the said Certification was not sworn to
before a notary public

RULING: YES.
The COMELEC did not consider in the first instance the Certification issued by Pinagtong-ulan Barangay
Captain DominadorHonrade (Honrade) that petitioner had been residing in BrgyPinagtong-ulan since
2007. When this oversight was raised as an issue in petitioner’s Motion for Reconsideration, the
COMELEC brushed it aside on the ground that the said Certification was not sworn to before a notary
public and, hence, "cannot be relied on." Subsequently, petitioner presented another, substantially
identical, Certification from the said Pinagtongulan Barangay Captain, save for the fact that it had now
been sworn to before a notary public.

We disagree with the COMELEC’s treatment of the Barangay Captain’s Certification and find the same
tainted with grave abuse of discretion. Even without being sworn to before a notary public, Honrade’s
Certification would not only be admissible in evidence, but would also be entitled to due consideration.
Rule 130, Section 44 of the Rules of Court provides:

SEC. 44. Entries in official records.—Entries in official records made in the performance of his duty by a
public officer of the Philippines, or by a person in the performance of a duty specially enjoined by law,
are prima facie evidence of the facts therein stated.

Remedial Law Review II (CY 2018-2019) – Saturday 1:00pm – 5:00pm: Case Digest Compilation [Atty. Brondial] Page 432
In Country Bankers Insurance Corporation v. Lianga Bay and Community Multipurpose Cooperative,
Inc., we explained that the following three (3) requisites must concur for entries in official records to be
admissible in evidence: (a) The entry was made by a public officer, or by another person specially
enjoined by law to do so; (b) It was made by the public officer in the performance of his duties, or by
such other person in the performance of a duty specially enjoined by law; and (c) The public officer or
other person had sufficient knowledge of the facts stated by him, which facts must have been acquired by
him personally or through official information.

As to the first requisite, the Barangay Secretary is required by the Local Government Code to "keep an
updated record of all inhabitants of the barangay." Regarding the second requisite, we have explicitly
recognized in Mitra v. Commission on Elections, that "it is the business of a punong barangay to know
who the residents are in his own barangay." Anent the third requisite, the Barangay Captain’s exercise of
powers and duties concomitant to his position requires him to be privy to these records kept by the
Barangay Secretary.

Accordingly, there is basis in faulting the COMELEC for its failure to consider Honrade’s Certification
on the sole ground that it was initially not notarized.

Remedial Law Review II (CY 2018-2019) – Saturday 1:00pm – 5:00pm: Case Digest Compilation [Atty. Brondial] Page 433
C.3.h.viiii.3SIMPLICIA CERCADO-SIGA AND LIGAYA CERCADO-BELISON VS. VICENTE
CERCADO, JR., MANUELA C. ARABIT, LOLITA C. BASCO, MARIA C. ARALAR AND
VIOLETA C. BINADAS, 

752 SCRA 514

FACTS: Petitioners Simplicia Cercado-Siga (Simplicia) and Ligaya Cercado-Belison (Ligaya) claimed
that they are the legitimate children of the late Vicente and Benita Castillo (Benita), who were married
last 9 October 1929 in Pililla, Rizal. In support of the existence thereof, petitioners presented a copy of
the Contrato Matrimonial which was issued by Iglesia Filipina Independiente church. Petitioners insist
that the Contrato Matrimonial is a public document because it is required by law to be recorded in the
local civil registrar and the National Statistics Office (NSO). Petitioners claim to have in their possession
a duplicate original of the Contrato Matrimonial which should be regarded as original. Granting that the
Contrato Matrimonial is a private document, petitioners maintain that said document should be
considered an ancient document which should be excluded from the requirement of authentication. In
their Answer, respondents alleged that they are the legitimate heirs of Vicente and Leonora, who were
married on 27 June 1977 as evidenced by a marriage certificate registered with the Local Civil Registrar
of Binangonan, Rizal. In their Comment, respondents submit that the Contrato Matrimonial is a private
document and the fact that marriages are required to be registered in the local civil registrar does not ipso
facto make it a public document. Respondents assert that the certificate of baptism is likewise a private
document which tends to prove only the administration of the sacrament of baptism and not the veracity
of the declarations therein. Respondents moreover refute the certification issued by the local civil registry
arguing that it does not prove filiation but only the fact that there is no record of Ligaya on file with said
office.

ISSUE: Whether or not the marriage contract or Contrato Matrimonial is sufficient to prove the fact of
marriage.

RULING: Under Section 20, Rule 132, Rules of Court, before a private document is admitted in
evidence, it must be authenticated either by the person who executed it, the person before whom its
execution was acknowledged, any person who was present and saw it executed, or who after its
execution, saw it and recognized the signatures, or the person to whom the parties to the instruments had
previously confessed execution thereof. As observed by the Court of Appeals, petitioners failed to present
any one of such witnesses. In fact, only Simplicia testified that her mother gave her the marriage contract.
Unfortunately however, she was not present during its execution nor could she identify Benita’s
handwriting because Simplicia admitted that she is illiterate. While petitioners concede that the marriage
contract is a private document, they now argue that it is an ancient document which need not be
authenticated. Petitioners’ argument still has no merit.

Section 21, Rule 132 defines an ancient document as one that:


1) is more than 30 years old;
2) is produced from custody in which it would naturally be found if genuine; and
3) is unblemished by any alteration or by any circumstance of suspicion.
The marriage contract was executed on 9 October 1929, hence it is clearly more than 30-years old. On its
face, there appears to be no evidence of alteration. The marriage contract however does not meet the
second requirement. Ancient documents are considered from proper custody if they come from a place
from which they might reasonably be expected to be found. Custody is proper if it is proved to have had a
legitimate origin or if the circumstances of the particular case are such as to render such an origin
probable. If a document is found where it would not properly and naturally be, its absence from the
proper place must be satisfactorily accounted for.

Remedial Law Review II (CY 2018-2019) – Saturday 1:00pm – 5:00pm: Case Digest Compilation [Atty. Brondial] Page 434
C.3.h.ixMeralco vs. Quisumbing
Manila Electric Company (Meralco) vs. Hon. Quisumbing and Meralco Employees and Workers
Association (MEWA)

February 22, 2000, J. Ynares-Santiago.

FACTS: MEWA, the duly recognized labor organization of the rank-and-file employees of MERALCO
informed the latter of its intention to re-negotiate the terms and conditions of their existing Collective
Bargaining Agreement (CBA). Thereafter, collective bargaining negotiations proceeded. However,
despite the series of meetings between the negotiating panels of MERALCO and MEWA, the parties
failed to arrive at “terms and conditions” acceptable to both of them which prompted MEWA to file a
Notice of Strike with the National Capital Region Branch of the National Conciliation and Mediation
Board (NCMB) of the DOLE on the grounds of bargaining deadlock and unfair labor practices. Upon the
existence of the strike, MERALCO filed an Urgent Petition with the DOLE Secretary praying that the
latter should assume jurisdiction over the labor dispute and to enjoin the striking employees to go back to
work. Acting on such petition, the Labor Secretary granted and resolved the labor dispute by ordering the
grant of a 4,500 wage increase, as well as a new and improved fringe benefits under the remaining 2 years
of the CBA for the rank-and-file employees. Unsatisfied, MERALCO filed a supplement to the motion
for reconsideration alleging that the Secretary did not property appreciate the effect of the awarded wages
and benefits on MERALCO’s financial viability. Hence, in a decision on January 27, 1999, the orders of
the Secretary were set aside and the wage increase is now from 1,900 to 2,200. MERALCO warns that if
the wage increase of 2,200 per month as ordered by the Secretary is allowed, it would simply pass the
cost covering such increase to the consumers through increase in the rate of electricity.

ISSUE: Whether or not the argument is tenable.

RULING: No. This is a non sequitur. The Court cannot be threatened with such a misleading argument.
An increase in the prices of electric current needs the approval of the appropriate regulatory government
agency and does not automatically result from a mere increase in the wages of petitioner’s employees.
Besides, this argument presupposes that petitioner is capable of meeting a wage increase. The All Asia
Capital report upon which the Union relies to support its position regarding the wage issue can not be an
accurate basis and conclusive determinant of the rate of wage increase. Section 45 of Rule 130 Rules of
Evidence provides: "Commercial lists and the like. - Evidence of statements of matters of interest to
persons engaged in an occupation contained in a list, register, periodical, or other published compilation
is admissible as tending to prove the truth of any relevant matter so stated if that compilation is published
for use by persons engaged in that occupation and is generally used and relied upon by them therein."

Under the afore-quoted rule, statement of matters contained in a periodical may be admitted only "if that
compilation is published for use by persons engaged in that occupation and is generally used and relied
upon by them therein." As correctly held in our Decision dated January 27, 1999, the cited report is a
mere newspaper account and not even a commercial list. At most, it is but an analysis or opinion which
carries no persuasive weight for purposes of this case as no sufficient figures to support it were presented.
Neither did anybody testify to its accuracy. It cannot be said that businessmen generally rely on news
items such as this in their occupation. Besides, no evidence was presented that the publication was
regularly prepared by a person in touch with the market and that it is generally regarded as trustworthy
and reliable. Absent extrinsic proof of their accuracy, these reports are not admissible. In the same
manner, newspapers containing stock quotations are not admissible in evidence when the source of the
reports is available. With more reason, mere analyses or projections of such reports cannot be admitted. In
particular, the source of the report in this case can be easily made available considering that the same is
necessary for compliance with certain governmental requirements.

Remedial Law Review II (CY 2018-2019) – Saturday 1:00pm – 5:00pm: Case Digest Compilation [Atty. Brondial] Page 435
C.3.h.xi.1 PEOPLE VS ORTIZ-MIYAKO

G.R. Nos. 115338-39  September 16, 1997

FACTS:Accused-appellant Lanie Ortiz-Miyake was charged with illegal recruitment in large scale in the
Regional Trial Court of Makati on a complaint initiated by Elenita Marasigan, Imelda Generillo and
Rosamar del Rosario.  In addition, she was indicted for estafa by means of false pretenses in the same
court, the offended party being Elenita Marasigan alone. 

A judgment was rendered by said court convicting appellant of both crimes as charged. In convicting
appellant of illegal recruitment in large scale, the lower court adopted a previous decision of the
Metropolitan Trial Court of Parañaque as a basis for the judgment. Said previous decision was a
conviction for estafa involving the same circumstances in the instant case, wherein complainants
Generillo and Del Rosario charged appellant with two counts of estafa.   In thus convicting appellant in
the illegal recruitment case, the court adopted the facts and conclusions established in the estafa decision
as its own findings of facts and as its rationale for the conviction in the case before it.
 
ISSUE: Whether adoption by Makati Trial Court of the facts stated in the Paranaque Trial Court does not
fall under the exception of the right to confrontation?
 
RULING: NO. Under the law, the accused in a criminal case is guaranteed the right of confrontation.
This right, however, is not absolute as it is recognized that it is sometimes impossible to recall or produce
a witness who has already testified in a previous proceeding, in which event his previous testimony is
made admissible as a distinct piece of evidence, by way of exception to the hearsay rule previous
testimony is made admissible because it makes the administration of justice orderly and expeditious
(section 47 of Rule 130). Under these rules, the adoption by the Makati trial court of the facts stated in
the decision of the Parañaque trial court does not fall under the exception to the right of confrontation as
the exception contemplated by law covers only the utilization of testimonies of absent witnesses made in
previous proceedings, and does not include utilization of previous decisions or judgments. In the instant
case, the prosecution did not offer the testimonies made by complainants Generillo and Del Rosario in the
previous estafa case.  Instead, what was offered, admitted in evidence, and utilized as a basis for the
conviction in the case for illegal recruitment in large scale was the previous decision in the estafa case. A
previous decision or judgment, while admissible in evidence, may only prove that an accused was
previously convicted of a crime. It may not be used to prove that the accused is guilty of a crime charged
in a subsequent case, in lieu of the requisite evidence proving the commission of the crime, as said
previous decision is hearsay.  To sanction its being used as a basis for conviction in a subsequent case
would constitute a violation of the right of the accused to confront the witnesses against him.

Remedial Law Review II (CY 2018-2019) – Saturday 1:00pm – 5:00pm: Case Digest Compilation [Atty. Brondial] Page 436
C.3.h.xi.2 HARRY GO, ET AL., vs. PEOPLE OF THE PHILIPPINES

G.R. No. 185527, July 18, 2012; Perlas-Bernabe, J.

Digested by:Jesus Fernan Montemayor

FACTS: Petitioners Harry Go, Tonny Ngo, Jerry Ngo and Jane Go were charged before the MeTC of
Manila for Other Deceits under Article 318 of the Revised Penal Code (RPC). Upon arraignment, the
accused pleaded not guilty to the charge. The prosecution’s complaining witness Li Luen Ping is a frail
old businessman from Laos, Cambodia and who traveled to the Philippines in order to attend the hearings
of this case. However, due to his unavailability, the trial dates were subsequently postponed.

Thus, the prosecution filed a Motion to Take Oral Deposition of the said witness alleging that he was
being treated for lung infection in Laos, Cambodia and that, upon doctor’s advice, he could not make the
long travel to the Philippines by reason of ill health. Notwithstanding the opposition, the MeTC granted
the said motion prompting the petitioners to file a petition for certiorari before the RTC.

The RTC granted the petition and declared the MeTC orders as null and void. It was further ruled that
Section 17, Rule 23 on the taking of depositions of witnesses in civil cases cannot apply suppletorily to
the case since there is a specific provision in the Rules of Court with respect to the taking of depositions
of prosecution witnesses in criminal cases, which is primarily intended to safeguard the constitutional
rights of the accused to meet the witness against him face to face. Upon denial by the RTC of the motion
for reconsideration, the prosecution elevated the case to the CA.

The appellate court ruled that no grave abuse of discretion can be imputed upon the MeTC for allowing
the deposition-taking of the complaining witness Li Luen Ping because no rule of procedure expressly
disallows the taking of depositions in criminal cases and that, in any case, petitioners would still have
every opportunity to cross-examine the complaining witness and make timely objections during the taking
of the oral deposition either through counsel or through the consular officer who would be taking the
deposition of the witness. Hence, this petition for review on certiorari.

ISSUE: WON the deposition of a prosecution witness taken before the consular office abroad is valid and
will not violate the constitutional right of the accused to confront the witness face to face?

RULING:NO. As a rule, the examination of witnesses must be done orally before a judge in open court.
However, the Rules of Court recognizes the conditional examination of witnesses and the use of their
depositions as testimonial evidence in lieu of direct court testimony. And for purposes of taking the
deposition in criminal cases, more particularly of a prosecution witness who would foreseeably be
unavailable for trial, the testimonial examination should be made before the court, or at least before the
judge, where the case is pending as required by the clear mandate of Section 15, Rule 119 of the Revised
Rules of Criminal Procedure.

Since the conditional examination of a prosecution witness must take place at no other place than the
court where the case is pending, the RTC properly nullified the MeTC's orders granting the motion to take
the deposition of Li Luen Ping before the Philippine consular official in Laos, Cambodia. Certainly, to
take the deposition of the prosecution witness elsewhere and not before the very same court where the
case is pending would not only deprive a detained accused of his right to attend the proceedings but also
deprive the trial judge of the opportunity to observe the prosecution witness' deportment and properly
assess his credibility, which is especially intolerable when the witness' testimony is crucial to the
prosecution's case against the accused.

Here, the CA ignored the procedure under the Revised Rules of Criminal Procedure for taking the
deposition of an unavailable prosecution witness when it upheld the trial court's order allowing the
deposition of prosecution witness Li Luen Ping to take place in a venue other than the court where the
case is pending.

DOCTRINE: It must be emphasized that while the prosecution must provide the accused every
opportunity to take the deposition of witnesses that are material to his defense in order to avoid charges
of violating the right of the accused to compulsory process, the State itself must resort to deposition-
taking sparingly if it is to guard against accusations of violating the right of the accused to meet the
witnesses against him face to face. Great care must be observed in the taking and use of depositions of

Remedial Law Review II (CY 2018-2019) – Saturday 1:00pm – 5:00pm: Case Digest Compilation [Atty. Brondial] Page 437
prosecution witnesses to the end that no conviction of an accused will rely on ex parte affidavits and
depositions.

Remedial Law Review II (CY 2018-2019) – Saturday 1:00pm – 5:00pm: Case Digest Compilation [Atty. Brondial] Page 438
C.3.h.xii.1 People vs. Ibanez
PEOPLE OF THE PHILIPPINES, Plaintiff-Appellee, vs. EDWIN IBANEZ y ALBANTE
and ALFREDO (FREDDIE) NULLA y IBANEZ, Accused-appellants.

G.R. No. 197813 September 25, 2013

FACTS: Appellants Edwin and Alfredo, with Jesus Monsillo y Taniares (Jesus), were all charged in an
Information for Murder under Article 248 of the Revised Penal Code. During arraignment, Edwin and
Alfredo pleaded not guilty. Jesus, on the other hand, remained at large; the case against him was archived.
Thereafter, trial ensued. The prosecution’s version was testified to by the victim’s wife and daughter, in
succession. On that fateful day, Wilfredo was invited by Alfredo to a drinking session with Jesus and
Edwin making them a party of four. Rachel, Wilfredo’s daughter, an adolescent at the time, was
underneath the house (silong in the vernacular) of a neighbor, three (3)meters away from the place where
Wilfredo and his companions were ostensibly in merrymaking. Rachel saw Edwin snatched a t-shirt from
a nearby clothesline, and hooded the tshirt over the head and face of Wilfredo. Robbed of vision as his
head was fully covered, Wilfredo was wrestled and pinned down by Edwin, while Alfredo boxed the left
side of Wilfredo’s chest. Jesus, armed with a long iron bar, swung at and hit Wilfredo in the head.
Terrified, Rachel stood immobilized as she watched the attack on father. Thereafter, she saw her mother
running out of their house and crying for help. Edwin and Alfredo stood beside Jesus; Edwin held a white
shirt.

Forthwith, Jesus and Alfredo ran away while Edwin went home. Rowena asked for help to bring Wilfredo
to the hospital. However, Wilfredo did not reach the hospital alive and was pronounced dead on arrival.
The two accused-appellants pointed to Jesus as the sole culprit, proclaimed their innocence and professed
to being at the scene of the crime only because of their curiosity for what had occurred.
On appeal, Edwin and Alfredo found no reprieve. The Court of Appeals did not deviate from the RTC’s
ruling and affirmed in toto its finding of guilt.

ISSUE: Whether the accused are guilty of murder

RULING: YES.
Both lower courts, however, found the testimony of Rachel credible: This Court finds the testimony of
Rachel clear and convincing. The testimony flows from a person who was present in the place where the
killing occurred. They are replete with details sufficient to shift the burden of evidence to appellants. We
have no reason to doubt Rachel’s credibility. Her candid account of the incident, standing alone, clearly
established the components of the crime of murder.

Appellants’ defense of denial, not sufficiently proven, cannot overcome the conclusions drawn from said
evidence. We find no cogent reason to deviate from the findings and conclusions of the trial court.
Rachel’s testimony was delivered in a firm, candid, and straightforward manner. There is no showing that
Rachel wavered from the basic facts of her testimony, even when she was subjected to a rigorous
examination.

We find no error in the lower courts’ disposal of the issue. We have scrutinized the testimony of lone
eyewitness, Rachel. Throughout her testimony, in her direct, cross and re-direct and re-cross
examinations, she candidly recounted the events surrounding the killing of her father.
As the lower courts have done, we accord full faith and credence to Rachel’s testimony. She was young
and unschooled, but her narration of the incident was categorical, without wavering. It has no markings of
a concocted story, impressed upon her by other people.

We cannot take Rachel’s testimony lightly simply because she was a mere child when she witnessed the
incident and when she gave her testimony in court. There is no showing that her mental maturity rendered
her incapable of testifying and of relating the incident truthfully.

With exceptions provided in the Rules of Court, all persons who can perceive, and perceiving, can make
known their perception to others, may be witnesses. That is even buttressed by the Rule on Examination
of a Child Witness which specifies that every child is presumed qualified to be a witness. To rebut this
presumption, the burden of proof lies on the party challenging the child's competence. Only when
substantial doubt exists regarding the ability of the child to perceive, remember, communicate, distinguish
truth from falsehood, or appreciate the duty to tell the truth in court will the court, motuproprio or on
motion of a party, conduct a competency examination of a child. Thus, petitioners’ flimsy objections on

Remedial Law Review II (CY 2018-2019) – Saturday 1:00pm – 5:00pm: Case Digest Compilation [Atty. Brondial] Page 439
Rachel’s lack of education and inability to read and tell time carry no weight and cannot overcome the
clear and convincing testimony of Rachel as to who killed her father.
We likewise note that the line of questioning of the defense during crossexamination on the competency
of Rachel to read and tell time did not distract her in recollecting how her father was attacked by accused-
appellants. From her position underneath the house of her "KuyaUnyo," she saw her father, Wilfredo,
attacked by accused-appellants. Although she was astonished as the happening unfolded, her ability to
perceive, remember, and make known her perception was not diminished.

Remedial Law Review II (CY 2018-2019) – Saturday 1:00pm – 5:00pm: Case Digest Compilation [Atty. Brondial] Page 440
C.3.h.xii.2 People vs. Esugon

Remedial Law Review II (CY 2018-2019) – Saturday 1:00pm – 5:00pm: Case Digest Compilation [Atty. Brondial] Page 441
C.3.h.i.1 PEOPLE OF THE PHILIPPINES, plaintiff-appellee, vs.PO2 ALBERT ABRIOL,
MACARIO ASTELLERO, and JANUARIO DOSDOS, accused-appellants.

G.R. No. 123137; October 17, 2001; QUISUMBING, J.:

FACTS: At around 11:50 P.M., June 5, 1993, Romeo Sta. Cruz, Jr., a radio news reporter then aboard his
jeep, had just reached the ABS-CBN compound in P. del Rosario Street, Cebu City, when he heard a
couple of gunshots. He looked around and saw a man running unsteadily towards the intersection of P. del
Rosario Street and Jones Avenue (Osmeña Boulevard). The man was shouting "Tabang, tabang!" ("Help!
Help!"). Sta. Cruz, Jr., saw a red "Jiffy" make a U-turn near the gate of the city central school that nearly
ran over the man shouting for help. The man turned back and staggered towards the direction of Bacalso
Avenue and Urgello Private Road, but after a few meters on wobbly legs, he stopped and collapsed.
Meanwhile, the "Jiffy" followed. It stopped beside the fallen figure and a tall, thin man alighted. The
man fired several shots at the prostrate figure. He boarded the "Jiffy" which sped away towards Leon
Kilat Street. Romeo Sta. Cruz, Jr., moved his jeep and focused its headlights on the victim. In the
meantime, PO3 Alexander Rustela was at a vulcanizing shop near the intersection of Bacalso Avenue and
Leon Kilat Street, when he heard gunshots coming from the north. He ran towards where the gunshots
came and saw people scampering. All of a sudden, the "Jiffy" with three persons on board sped past him
and made an abrupt left turn at Leon Kilat Street. Rustela immediately radioed for assistance. Minutes
later, patrol car No. 201 with PO2 Herbert Ramos on board arrived. Rustela boarded the car and they
followed the "Jiffy," while broadcasting an alarm to police headquarters and other mobile patrol cars. On
nearby Colon Street, SPO1 Eleazar Abrigana and PO2 Romeo Abellana were cruising aboard patrol car
No. 208, when they heard a radio message that the suspects in the shooting incident were aboard a "Jiffy."
As they turned left at Leon Kilat Street, they saw the "Jiffy" heading towards Carbon Market. They
pursued the "Jiffy" which stopped in front of the Don Bosco Building near BBRC, when police car
No. 205, with PO Eugenio Badrinas and PO2 Gerald Cue aboard, blocked the "Jiffy's" path. Cue
fired a warning shot and three persons alighted. The driver was appellant Astellero, whom Cue had
recognized and seen before at the BBRC. Abrigana and Cue approached the trio who stood a meter
away from the "Jiffy." SPO1 Abrigana frisked appellant Albert Abriol and seized from his waist a .
38 caliber revolver with serial number PO8485 with six (6) empty shells in its cylinder. Under
Abriol's seat, the police also found a .45 caliber pistol bearing serial number PGO 13506 with nine
(9) live rounds in its magazine and another .45 caliber pistol with serial number 52469 loaded with
five (5) unfired bullets. While the patrol cars were chasing the "Jiffy," another police team proceeded to
the crime scene in response to the alarm. This team from Police Station No. 3 in San Nicolas, Cebu City
rushed the victim to the Cebu City Medical Center, where he was pronounced dead on arrival.
Meanwhile, PO3 Celso Seville, Jr., a homicide investigator of Police Station No. 3 found four (4) .45
caliber shells some four (4) feet away from the victim's body, and two (2) deformed slugs where the
victim had lain, and submitted them to the Region 7 PNP Crime Laboratory for ballistics testing. The
following day, appellants underwent a paraffin test. The hands of appellants were found positive for
gunpowder residues. A chemistry test on the firearms showed that the three handguns were also
positive. Inspector Myrna Areola, Chief of the Chemistry Section of the PNP Region 7 Crime
Laboratory, stated in her testimony that the firearms had been fired, 16 and that appellants had
fired the guns within a period of seventy-two (72) hours prior to the examination.

Appellants contend that that the testimony of expert witness P/Inspector Lemuel Caser, the prosecution's
ballistics expert, clearly shows that: (1) He is ignorant about such ballistics instruments such as the
micrometer, goniometer, and pressure barrel. 35 (2) He is not conversant with "the required references
concerning ballistics," particularly books on the subject by foreign authorities. He could not
"scientifically determine the caliber of a bullet." Since P/Inspector Caser lacked adequate training
and expertise in ballistics, they claim that his opinion that the test bullets and cartridges matched
the slugs and cartridges recovered from the scene of the crime was not reliable. Appellants also
assail Caser's failure to take the necessary photographs to support his findings.

ISSUE: Whether or not the appellants should be convicted on the ground OF THE CRIMES OF
MURDER AND ILLEGAL POSSESSION OF FIREARMS on the ground of the testimony of the expert
witness?

RULING: Yes. An expert witness is "one who belongs to the profession or calling to which the subject
matter of the inquiry relates and who possesses special knowledge on questions on which he proposes to
express an opinion." There is no definite standard of determining the degree of skill or knowledge that a
witness must possess in order to testify as an expert. It is sufficient that the following factors be present:
(1) training and education; (2) particular, first-hand familiarity with the facts of the case; and (3)

Remedial Law Review II (CY 2018-2019) – Saturday 1:00pm – 5:00pm: Case Digest Compilation [Atty. Brondial] Page 442
presentation of authorities or standards upon which his opinion is based. The question of whether a
witness is properly qualified to give an expert opinion on ballistics rests with the discretion of the trial
court.

Here, the trial court that P/Inspector Caser qualifies as a ballistics expert. He is a licensed criminologist,
trained at the Ballistics Command and Laboratory Center in Fort Bonifacio, in the PNP Crime Laboratory
in Camp Crame, and in the National Bureau of Investigation. He had previously testified as an expert
witness in at least twenty-seven murder and homicide cases all over the country. An expert witness need
not present comparative microphotographs of test bullets and cartridges to support his findings.
Examination under a comparison microscope showing that the test bullet and the evidence bullet both
came from the same gun is sufficient. Moreover, the ballistician conclusively found similar characteristic
markings in the evidence, test cartridges and slugs

DOCTRINE: There is no definite standard of determining the degree of skill or knowledge that a
witness must possess in order to testify as an expert. It is sufficient that the following factors be present:
(1) training and education; (2) particular, first-hand familiarity with the facts of the case; and (3)
presentation of authorities or standards upon which his opinion is based. The question of whether a
witness is properly qualified to give an expert opinion on ballistics rests with the discretion of the trial
court.

Remedial Law Review II (CY 2018-2019) – Saturday 1:00pm – 5:00pm: Case Digest Compilation [Atty. Brondial] Page 443
C.3.h.i.2 Bautista vs. CA

FACTS:A parcel of land situated along Maceda St., Sampaloc, Manila, which contained an area of
approximately 105 square meters, was previously owned and registered in the name of the late Cesar
Morelos under TCT No. 27604. Cesar is the uncle of petitioner Laura Morelos Bautista, being the brother
of her mother, Rosario Morelos.

Cesar, who was married to Rosario Duran, did not have any children. Rosario died in 1972. Cesar died of
cardiac arrest in 1982. During his lifetime, Cesar sold and conveyed the parcel of land in favor of
petitioner Laura Morelos Bautista, as evidenced by a "Deed of Absolute Sale" (Deed) notarized by Luis
M. de Guzman. Accordingly, TCT No. 254843 was issued in the name of petitioner Laura Bautista.

Respondent Fernando Morelos, claiming to be the illegitimate child of Cesar Morelos with Angelina Lim-
Gue, instituted a complaint for the declaration of nullity of sale and title with damages before the RTC of
Manila. At the trial, he presented testimonies of expert witnesses who claimed that the signature of
Cesar Morelos on the Deed and the fingerprint appearing on his Residence Certificate were not his.

Petitioners countered that the Deed was valid. The witness to the Deed, Carmelita Marcelino, testified
that she saw Cesar Morelos and petitioner Laura Bautista sign the same.

After hearing, the court a quo rendered judgment declaring the Deed executed between the late Cesar
Morelos in favor of Laura Bautista valid, and dismissed for insufficient evidence the claims and
counterclaims for damages of the parties.

Respondent appealed to the Court of Appeals, which reversed and set aside the judgment of the trial
court.

On appeal to the Supreme Court, petitioner asserted the validity of the Deed and invoked the testimony of
Carmelita Marcelino, the instrumental witness to the signing of the document, who confirmed that it was
the decedent Cesar Morelos who affixed his signature to the document.

On the other hand, respondent contended that the decedent's signature on the Deed was forged. He
presented the testimony of Francisco Cruz, Jr., Chief Examiner of the PC-INP Crime Laboratory
Service, that the signature of decedent on the questioned instrument, when compared to other
documents bearing the authentic signature of Cesar Morelos, did not match and appeared to have
been authored by a different person. Cruz, Jr. declared that the latest document bearing the genuine
signature of the decedent is dated March 31, 1982, while the alleged forged signature was made on April
5, 1982, or a mere lapse of five days. According to him, it is not possible to have significant variation
between the two signatures, considering the proximity of time when the signatures where affixed.

Another witness, Major Braulio Monge, Chief of the Fingerprint Division of the PC-INP, testified
that the thumbmark of Cesar Morelos appearing on the residence certificate indicated in the Deed,
when compared to those affixed on previous residence certificates issued to the decedent, did not match
and appears to be the thumbmark of another person.

ISSUE:Whether the testimonies of the expert witnesses are conclusive to be a strong basis to nullify a
duly executed and notarized Deed of Absolute Sale.

RULING: No. Under Rule 132, Section 22 of the Rules of Court, the genuineness of handwriting may be
proved in the following manner: (1) by any witness who believes it to be the handwriting of such person
because he has seen the person write; or he has seen writing purporting to be his upon which the witness
has acted or been charged; (2) by a comparison, made by the witness or the court, with writings admitted
or treated as genuine by the party, against whom the evidence is offered, or proved to be genuine to the
satisfaction of the judge.

It is well-settled that a duly notarized contract enjoys the prima facie presumption of authenticity and due
execution as well as the full faith and credence attached to a public instrument. To overturn this legal
presumption, evidence must be clear, convincing and more than merely preponderant to establish that
there was forgery that gave rise to a spurious contract.

As a general rule, forgery cannot be presumed and must be proved by clear, positive and convincing
evidence. The burden of proof lies on the party alleging forgery. In Heirs of Severa P. Gregorio v. Court

Remedial Law Review II (CY 2018-2019) – Saturday 1:00pm – 5:00pm: Case Digest Compilation [Atty. Brondial] Page 444
of Appeals, we held that due to the technicality of the procedure involved in the examination of the
forged documents, the expertise of questioned document examiners is usually helpful; however,
resort to questioned document examiners is not mandatory and while probably useful, they are not
indispensable in examining or comparing handwriting.

Hence, a finding of forgery does not depend entirely on the testimony of handwriting experts.
Although such testimony may be useful, the judge still exercises independent judgment on the issue
of authenticity of the signatures under scrutiny; he cannot rely on the mere testimony of the
handwriting expert.

The authenticity of signatures is not a highly technical issue in the same sense that questions concerning,
e.g., quantum physics or topology or molecular biology, would constitute matters of a highly technical
nature. The opinion of a handwriting expert on the genuineness of a questioned signature is
certainly much less compelling upon a judge than an opinion rendered by a specialist on a highly
technical issue.

In the case at bar, the presumption of validity and regularity prevails over allegations of forgery and
fraud. As against direct evidence consisting of the testimony of a witness who was physically present at
the signing of the contract and who had personal knowledge thereof, the testimony of an expert witness
constitutes indirect or circumstantial evidence at best. Carmelita Marcelino, the witness to the Deed of
Absolute Sale, confirmed the genuineness, authenticity and due execution thereof. Having been
physically present to see the decedent Cesar Morelos and petitioner Laura Bautista affix their signatures
on the document, the weight of evidence preponderates in favor of petitioners.

DOCTRINE:A finding of forgery does not depend entirely on the testimony of handwriting experts.
Although such testimony may be useful, the judge still exercises independent judgment on the issue of
authenticity of the signatures under scrutiny; he cannot rely on the mere testimony of the handwriting
expert.

The authenticity of signatures is not a highly technical issue in the same sense that questions concerning,
e.g., quantum physics or topology or molecular biology, would constitute matters of a highly technical
nature. The opinion of a handwriting expert on the genuineness of a questioned signature is certainly
much less compelling upon a judge than an opinion rendered by a specialist on a highly technical issue.

Remedial Law Review II (CY 2018-2019) – Saturday 1:00pm – 5:00pm: Case Digest Compilation [Atty. Brondial] Page 445
C.3.h.i.3 Bobby “Abel” Avelino vs. People of the Philippines

G.R. 181444; July 17, 2013; J. Villarama, Jr.

FACTS: Boby Avelino and et al. waw charged of murder with the qualifying circumstances of treachery
and evident premeditation. Upon arraignment, petitioner and his co-accused pleaded not guilty, others
remain at-large.

That on September 2000, Renato Sosas an employee of Bobby in his wood business was directed to
summons the other co-accused to meet at Bobby’s warehouse in Tondo. When the group met, Renato
Sosas who was just a step away from the group, was astounded when he heard Bobbuy utter "Papatayin si
Chairman." Bobot Tuwad reacted by asking Boby "Sino pong chairman?", to which Bobby replied "Sino
pa, Ninong Chairman Gener."

On October 2000, While Chairman Generoso Hispano was entering the nearest route near the center
island, a man suddenly emerged and blocked Chairman Hispano’s vehicle. It was witnessed by
Manalansang who peeped at the direction of Chairman Hispano’s jeep and saw three men wearing
bonnets, two of whom were strategically blocking the jeep of Chairman Hispano. The third man, who was
wearing a green jacket and positioned himself near the gutter, fired successive shots at Chairman Hispano
and thereafter approached the jeep of Chairman Hispano. He pulled down from the jeep the almost
lifeless body of Chairman Hispano. Since Manalangsang was situated near the third assailant, he failed to
identify the other two assailants. However, Manalangsang positively identified the third assailant as
appellant Bobby "Abel" Avelino, whom he saw stooping down at the Chairman’s body and pulling the
opening of his bonnet down to his chin to ascertain if the Chairman was still alive. Thereafter, appellant
and the other assailants drove away using the owner-type jeep of Chairman Hispano. However, on their
way towards Divisoria, the jeep was incidentally blocked by a tricycle and a white car. When the police
arrived at the crime scene, Chairman Hispano was already dead. The owner-type jeep of Chairman
Hispano was recovered in front of house No. 440, Orbiztondo Street, Binondo, Manila, with several
pieces of empty shells of 9 mm caliber gun scattered on its floor.

The defense, advanced an alibi that Bobby and his wife went to the Land Transportation Office in Pasay
City to renew his license as they planned to go to Baguio that day. But as he was issued a temporary
license late in the afternoon, instead of going home, he and his wife checked in at the Pharaoh Hotel in
Sta. Cruz, Manila to spend the night. He parked his car along Dasmariñas Bridge and slept. Later, he
woke up to transfer his car but his car was gone. Thus, he and his wife went to the police station in Sta.
Cruz, Manila then to the AntiCarnapping Unit along U.N. Avenue to report the incident. At the latter
location, they learned from a certain Tata Randy, an acquaintance and former police officer, that the
victim had been gunned down. Around 1:00 a.m., he and his wife returned to the hotel. On October 23,
2000, he was arrested by agents of the NBI.

The RTC found Bobby and his co-accused guilty beyond reasonable doubt. On appeal to CA, the court
denied petitioner’s appeal.

Bobby sought to reverse his conviction, arguing that the CA erred in relying on the testimonies of the
prosecution witnesses Manalangsang and Cañada and disregarding the inconsistencies between the
statements of Manalangsang and the findings of the medico-legal and SOCO PSI Cabamongan as to the
position of the gunman. He also reiterated his defense of denial and alibi. Petitioner claims that
Manalangsang’s statements that Hispano was shot in a downward direction conflict with the findings of
the medico-legal that the trajectory of the bullets is in an upward direction.

ISSUES:
1. Whether or not SOCO PSI Cambongan opinion material?
2. Whether or not the defense of denial and alibi cannot prevail over the witness’ positive
identification of the accused-appellants?

RULING:
1. NO, the testimony of Dr. Salen is pertinent and enlightening she answered about the trajectory of
bullet “It depends on the matter of the position of the head when the head was hit. It could be when the
trajectory is upward it could be lying down with his back and the gunman and the barrel of the gun is here
and if we will put the normal position of the body it is still upward but the normal position is like that
so…”

Remedial Law Review II (CY 2018-2019) – Saturday 1:00pm – 5:00pm: Case Digest Compilation [Atty. Brondial] Page 446
Clearly, the fact that the trajectory of the bullets is in an upward direction does not negate the veracity of
Manalangsang’s statement that Hispano was shot by the gunman from an elevated plane.

The CA was also correct in not giving credence to the opinion of SOCO PSI Cabamongan as regards the
position of the gunman when the latter shot Hispano. Cabamongan asserted that the gunman was on board
the owner-type jeep when Hispano was shot, which is opposed to Manalangsang’s testimony. However,
case records reveal that Cabamongan was presented as an ordinary witness. Hence, his opinion
regarding the location of the gunman in relation to the place where the empty shells were found is
immaterial.

Expert evidence is admissible only if: (a) the matter to be testified to is one that requires expertise, and (b)
the witness has been qualified as an expert. In this case, counsel for the petitioner failed to make the
necessary qualification upon presenting Cabamongan during trial.

2. YES, For alibi to prosper, it is not enough to prove that appellant was somewhere else when the
crime was committed; he must also demonstrate that it was physically impossible for him to have been at
the scene of the crime at the time of its commission. Unless substantiated by clear and convincing proof,
such defense is negative, self-serving, and undeserving of any weight in law. Denial, like alibi, as an
exonerating justification, is inherently weak and if uncorroborated regresses to blatant impotence. Like
alibi, it also constitutes self-serving negative evidence which cannot be accorded greater evidentiary
weight than the declaration of credible witnesses who testify on affirmative matters.

Manalangsang unequivocally identified the petitioner as the gunman. Manalangsang was able to identify
the petitioner because the latter revealed his face when he pulled down the bonnet he was wearing,
thereby exposing his eyes, nose, mouth, and chin. Moreover, the certainty of Manalangsang in identifying
the petitioner as the one who shot Hispano is bolstered by the fact that he and petitioner were neighbors
for five years in Baseco. The RTC cites the following statement by Manalangsang as an added indication
of his certainty – "Si Avelino, kahit ubod ng layo, kahit naglalakad lang, kilala ko na. Dahil unang-una,
matagal ko na siyang kilala, dahil ako hindi niya ako gaanong kilala, pero sila kilala ko, kahit nakatagilid,
kilala ko siya." It cannot be denied that once a person gains familiarity of another, identification becomes
quite an easy task even from a considerable distance.

Remedial Law Review II (CY 2018-2019) – Saturday 1:00pm – 5:00pm: Case Digest Compilation [Atty. Brondial] Page 447
C.3.h.ii PEOPLE OF THE PHILIPPINES vs. EMILIANO DURANAN, a.k.a "Kalbo,"

FACTS: Emiliano Duranan was charged with the crime of rape committed against Maria Nympha who is
a feebleminded. He was convicted by the trial court on the basis of the testimony of Nympha. On appeal,
Duranan argues that under Art. 335 (2) of the Revised Penal Code, an essential element for the
prosecution for rape of a mental retardate is a psychiatric evaluation of the complainant's mental age to
determine if her mental age is under twelve.

ISSUE: Whether or not a psychiatric evaluation is necessary to establish a mental condition of a person.

RULING:No, Section 50 Rule 130 of the Revised Rules on Evidence provides:


Opinion of Ordinary witnesses. -- The opinion of a witness for which proper basis is given may
be received in evidence regarding ---
a. the identify of a person about whom he has adequate knowledge;
b. a handwriting with which he has sufficient familiarity; and
c. the mental sanity of a person with whom he is sufficiently acquainted.
Discussing this provision of the Rule on Evidence, Sen. Vicente J. Francisco writes in his treatise:
The mother of an offended party in case of rate, though not a psychiatrist, if she knows the physical and
mental condition of the party, how she was born, what she is suffering from, and what her attainments
are, is competent to testify on the matter.

It is competent for the ordinary witness to give his opinion as to the sanity or mental condition of a
person, provided the witness has had sufficient opportunity to observe the speech, manner, habits, and
conduct of the person in question. Generally, it is required that the witness details the factors and reasons
upon which he bases his opinion before he can testify as to what it is. As the Supreme Court of Vermont
said: "A non-expert witness may give his opinion as to the sanity or insanity of another, when based upon
conversations or dealings which he has had with such person, or upon his appearance, or upon any fact
bearing upon his mental condition, with the witness' own knowledge and observation, he having first
testified to such conversations, dealings, appearance or other observed facts, as the basis for his opinion.
In the case at bar, Virginia Lozada, mother or Nympha, testified on the mental condition of her daughter.

The court said that the statement of Nympha’s mother that Nympha is "quite intelligent" must be read in
the context of her previous statement that complainant "thinks like a child but from her narration or
statement we can see that her declaration are (sic) true or believable." Thus, what complainant's mother
meant was that complainant, although she thought like a child, nevertheless could tell others what
happened to her.

Remedial Law Review II (CY 2018-2019) – Saturday 1:00pm – 5:00pm: Case Digest Compilation [Atty. Brondial] Page 448
C.3.i People vs. Deopita

G.R. No. 130601; 04 December 2000

FACTS:Dominga Pikit-pikit was on her way home from work when suddenly a man appeared from
behind, looped his arm around her neck and warned her not to shout or else she would die. The man
dragged her through the banana plantation towards the cornfields. Dominga got a good look at the man,
who turned out to be Rafael Diopita, as he sat on her thighs and proceeded to divest her of her belongings.
Thereafter, Diopita announced his desire to have carnal knowledge of Dominga. After having his way
with her, Diopita threatened Dominga not to tell anyone about the incident or else he would shoot her.

Dominga was able to report the incident to the police and gave the description of the suspect and his
possible whereabouts. A colored white/yellow, size 10 slipper was found in the scene of the crime. In a
police line-up, Dominga readily pointed to Diopita which was further bolstered by the fact that when the
police had him try the slipper, it easily fitted him. In his defense, Diopita posed the alibi that he was at an
informal Bible session of the Jehovah’s Witnesses at the time of the crime.

The trial court convicted Diopita stating that alibi is a weak form of defense. Among Diopita’s arguments
is that it was impossible for him to have committed the crime charged since he is a person of good moral
character, holding as he does the position of “Ministerial Servant” in the congregation of Jehovah’s
Witnesses, and that he is a godly man, a righteous person, a responsible family man and a good Christian
who preaches the word of God.

ISSUE:Whether or not Diopita is entitles to an acquittal simply because of his previous good moral
character and exemplary conduct.

RULING: No. The conviction was affirmed by the Supreme Court, ratiocinating that the fact that Diopita
is endowed with such "sterling" qualities hardly justifies the conclusion that he is innocent of the crime
charged. Similarly, his having attained the position of "Ministerial Servant" in his faith is no guarantee
against any sexual perversion and plunderous proclivity on his part.

Indeed, religiosity is not always an emblem of good conduct, and it is not the unreligious alone who
succumbs to the impulse to rob and rape. An accused is not entitled to an acquittal simply because of his
previous good moral character and exemplary conduct. The affirmance or reversal of his conviction must
be resolved on the basic issue of whether the prosecution had discharged its duty of proving his guilt
beyond any peradventure of doubt. Since the evidence of the crime in the instant case is more than
sufficient to convict, the evidence of good moral character of accused-appellant is unavailing.

Remedial Law Review II (CY 2018-2019) – Saturday 1:00pm – 5:00pm: Case Digest Compilation [Atty. Brondial] Page 449
D. BURDEN OF PROOF AND PRESUMPTIONS

D.1 FEBTC vs. Chante


FAR EAST BANK TRUST COMPANY, Petitioner,  vs. ROBERTO MAR CHANTE, a.k.a.
ROBERT MAR G. CHAN, Respondents.
G.R. No. 170598; October 9, 2013
BERSAMIN, J.:

Facts: Chan, was a current account depositor of petitioner FEBTC at its Ongpin Branch (Current Account
No. 5012-00340-3). FEBTC issued to him Far East Card No. 05-01120-5-0 with July 1993 as the expiry
date.
Civil Case No. 92-61706 sprang from the complaint brought by petitioner FEBTC on July 1, 1992 in the
RTC, to recover from Chan the principal sum of P770,488.30 representing the unpaid balance of the
amount fraudulently withdrawn from Chan’s Current Account No. 5012-00340-3 with the use of Far East
Card No. 05- 01120-5-0. FEBTC alleged that at the time of the ATM withdrawal transactions, there was
an error in its computer system known as "system bug" whose nature had allowed Chan to successfully
withdraw funds in excess of his current credit balance of P198,511.70; and that Chan had taken advantage
of the system bug to do the withdrawal transactions.
On his part, Chan denied liability. Although admitting his physical possession of Far East Card No. 05-
01120-5-0 on May 4 and May 5, 1992, he denied making the ATM withdrawals totalling P967,000.00,
and instead insisted that he had been actually home at the time of the withdrawals. He alluded to a
possible "inside job" as the cause of the supposed withdrawals, citing a newspaper report to the effect that
an employee of FEBTC’s had admitted having debited accounts of its depositors by using his knowledge
of computers as well as information available to him. Chan claimed that it would be physically impossible
for any human being like him to stand long hours in front of the ATM facility just to withdraw funds. He
contested the debiting of his account, stating that the debiting had affected his business and had caused
him to suffer great humiliation after the dishonor of his sufficiently- funded checks by FEBTC.
The RTC rendered judgment in favor of FEBTC.
Chan appealed. The CA promulgated the assailed decision, reversing the RTC’s judgment. FEBTC
moved for reconsideration, but the CA denied its motion.

Issue: Whether FEBTC discharged its burden of proof?

Held: No.
The CA ruled that FEBTC did not because –
“After a review of the records of this case, we find the totality of evidence submitted by FEBTC
insufficient to establish the crucial facts that would justify a judgment in its favor.
To our mind, the fact that Chan’s account number and ATM card number were the ones used for the
withdrawals, by itself, is not sufficient to support the conclusion that he should be deemed to have made
the withdrawals.”
In his answer, he denied using the card to withdraw funds from his account on the dates in question, and
averred that the withdrawals had been an "inside job." His denial effectively traversed FEBTC’s claim of
his direct and personal liability for the withdrawals, that it would lose the case unless it competently and
sufficiently established that he had personally made the withdrawals himself, or that he had caused the
withdrawals. In other words, it carried the burden of proof.
Burden of proof is a term that refers to two separate and quite different concepts, namely: (a) the risk of
non-persuasion, or the burden of persuasion, or simply persuasion burden; and (b) the duty of producing
evidence, or the burden of going forward with the evidence, or simply the production burden or the
burden of evidence. In its first concept, it is the duty to establish the truth of a given proposition or issue
by such a quantum of evidence as the law demands in the case at which the issue arises. In its other
concept, it is the duty of producing evidence at the beginning or at any subsequent stage of trial in order
to make or meet a prima facie case. Generally speaking, burden of proof in its second concept passes
from party to party as the case progresses, while in its first concept it rests throughout upon the party
asserting the affirmative of the issue.
The party who alleges an affirmative fact has the burden of proving it because mere allegation of the fact
is not evidence of it. Verily, the party who asserts, not he who denies, must prove.
In civil cases, the burden of proof is on the party who would be defeated if no evidence is given on either
side. This is because our system frees the trier of facts from the responsibility of investigating and
presenting the facts and arguments, placing that responsibility entirely upon the respective parties. The
burden of proof, which may either be on the plaintiff or the defendant, is on the plaintiff if the defendant
denies the factual allegations of the complaint in the manner required by the Rules of Court; or on the
defendant if he admits expressly or

Remedial Law Review II (CY 2018-2019) – Saturday 1:00pm – 5:00pm: Case Digest Compilation [Atty. Brondial] Page 450
impliedly the essential allegations but raises an affirmative defense or defenses, that, if proved, would
exculpate him from liability.
Section 1, Rule 133 of the Rules of Court sets the quantum of evidence for civil actions, and delineates
how preponderance of evidence is determined.
As the rule indicates, preponderant evidence refers to evidence that is of greater weight, or more
convincing, than the evidence offered in opposition to it. It is proof that leads the trier of facts to find that
the existence of the contested fact is more probable than its nonexistence.
Being the plaintiff, FEBTC must rely on the strength of its own evidence instead of upon the weakness of
Chan’s evidence. Its burden of proof thus required it to preponderantly demonstrate that his ATM card
had been used to make the withdrawals, and that he had used the ATM card and PIN by himself or by
another person to make the fraudulent withdrawals. Otherwise, it could not recover from him any funds
supposedly improperly withdrawn from the ATM account. We remind that as a banking institution,
FEBTC had the duty and responsibility to ensure the safety of the funds it held in trust for its depositors.
It could not avoid the duty or evade the responsibility because it alone should bear the price for the fraud
resulting from the system bug on account of its exclusive control of its computer system.

Remedial Law Review II (CY 2018-2019) – Saturday 1:00pm – 5:00pm: Case Digest Compilation [Atty. Brondial] Page 451
D.2.a.i IBAAN RURAL BANK VS. CA

Doctrine: Estoppel in pais arises when one, by his acts, representations or admissions, or by his own
silence when he ought to speak out, intentionally or through culpable negligence, induces another to
believe certain facts to exist and such other rightfully relies and acts on such belief, so that he will be
prejudiced if the former is permitted to deny the existence of such facts.

Facts:
Spouses Cesar and Leonila Reyes were the owners of three (3) lots covered by Transfer Certificate of
Title (TCT) Nos. 33206, 33207 and 33208 of the Register of Deeds of Lipa City. On March 21, 1976, the
spouses mortgaged these lots to petitioner Ibaan Rural Bank, Inc. On June 11, 1976, with the knowledge
and consent of the petitioner, the spouses as sellers, and Mr. and Mrs. Ramon Tarnate [herein private
respondents] as buyers, entered into a Deed of Absolute Sale with Assumption of Mortgage of the lots in
question. Private respondents failed to pay the loan and the bank extra-judicially foreclosed on the
mortgaged lots. The Provincial Sheriff conducted a public auction of the lots and awarded the lots to the
bank, the sole bidder. On December 13, 1978, the Provincial Sheriff issued a Certificate of Sale which
was registered on October 16, 1979. The certificate stated that the redemption period expires two (2)
years from the registration of the sale. No notice of the extrajudicial foreclosure was given to the private
respondents. On September 23, 1981, private respondents offered to redeem the foreclosed lots and
tendered the redemption amount of P77,737.45. However, petitioner Bank refused the redemption on the
ground that it had consolidated its titles over the lots. The Provincial Sheriff also denied the redemption
on the ground that private respondents did not appear on the title to be the owners of the lots.

Private respondents filed a complaint to compel the bank to allow their redemption of the foreclosed lots.
They alleged that the extrajudicial foreclosure was null and void for lack of valid notice and demand upon
them. They further argued that they were entitled to redeem the foreclosed lots because they offered to
redeem and tendered the redemption price before October 16, 1981, the deadline of the 2-year redemption
period. The bank opposed the redemption, contending that the private respondents had no right to redeem
the lots because they were not the real parties in interest; that at the time they offered to redeem on
September 23, 1981, the right to redeem had prescribed, as more than one year had elapsed from the
registration of the Certificate of Sale on October 16, 1979; that there was no need of personal notice to
them because under Section 3 of Act 3135, only the posting of notice of sale at three public places of the
municipality where the properties are located was required.

RTC and CA ruled in favor of the respondents.

Issue: What was the period of redemption: two years as unilaterally fixed by the sheriff in the contract, or
one year as fixed by law?

RULING: 2 years. Estoppel in pais arises when one, by his acts, representations or admissions, or by his
own silence when he ought to speak out, intentionally or through culpable negligence, induces another to
believe certain facts to exist and such other rightfully relies and acts on such belief, so that he will be
prejudiced if the former is permitted to deny the existence of such facts.

When petitioner received a copy of the Certificate of Sale registered in the Office of the Register of
Deeds of Lipa City, it had actual and constructive knowledge of the certificate and its contents. For two
years, it did not object to the two-year redemption period provided in the certificate. Thus, it could be said
that petitioner consented to the two-year redemption period specially since it had time to object and did
not. When circumstances imply a duty to speak on the part of the person for whom an obligation is
proposed, his silence can be construed as consent. By its silence and inaction, petitioner misled private
respondents to believe that they had two years within which to redeem the mortgage. After the lapse of
two years, petitioner is estopped from asserting that the period for redemption was only one year and that
the period had already lapsed.

Remedial Law Review II (CY 2018-2019) – Saturday 1:00pm – 5:00pm: Case Digest Compilation [Atty. Brondial] Page 452
D.2.a.ii Alcaraz vs. Tangga-an
Facts: On October 4, 1994, respondents Pedro Tangga-an, MenasTangga-an, Virginia III Yvette Tangga-
an, Cecil Villaflor, Hermes Tangga-an, Venus Tangga-an, Jupiter Tangga-an, Yvonne Fri and Vivien
Tangga-an filed a complaint for unlawful detainer, with damages, docketed as Civil Case No. R-33928,
against petitioner spouses Reynaldo Alcaraz and Esmeralda Alcaraz. They alleged that the late Virginia
Tangga-an (spouse of Pedro and mother of the other respondents) leased a residential building to the
petitioner spouses. The land where the building was erected was still owned by the National Housing
Authority (NHA) at the time. After two years, the spouses failed to pay the rent. Despite repeated
demands to pay the rent and to vacate the premises, the petitioner spouses refused. According to the
spouses, however, the ownership of the lot on which the house stood resulted in the cancellation of the
contract of lease between respondents and petitioner spouses. The transfer of title was issued in the name
of Virgilio (son of respondent Pedro and brothers of the other respondents). Thereafter, they paid their
rent to the new owner of the lot since the respondents had no longer the right to collect the rentals.

The Municipal Trial Court ruled in favor of the respondents since the petitioner spouses failed to prove
that the subject lot belongs only to Virgilio. On appeal, the Regional Trial Court affirmed the decision of
the MTC. The Court of Appeals affirmed both decisions of the MTC and the RTC. Hence, this petition
for review.

Before the supreme court, the petitioners contended that the courts a quo erred in not finding that
VirgilioTanggaan became the new owner not only of the lot but also of the residential house. They claim
that, before she died, Virginia, the original owner of the subject house, waived and ceded her rights over
the land in favor of Virgilio. The said transfer allegedly included the subject house because, pursuant to
Article 440 of the Civil Code, "the ownership of the property gives the right of accession to everything
which is produced thereby, or which is incorporated or attached thereto, either naturally or artificially."
They also maintain that the NHA executed a deed of sale of both the house and the lot in favor of
Virgilio. According to the petitioner spouses, the tax declaration over the house in the name of respondent
Hermes Tangga-an, as trustee of the other respondents, was self-serving and had no probative value
compared to the certificate of title over the lot in the name of VirgilioTangga-an.

Issue: Whether or not the petitioner spouses, as lessees, were excused from paying the rent because of the
change in the ownership of the land on which the rented house was built?

RULING: No. Section 2, Rule 131 of the Rules of Court provides as a conclusive presumption that:
Sec. 2.Conclusive presumptions. — The following are instances of conclusive presumptions: (a)
Whenever a party has, by his own declaration, act, or omission, intentionally and deliberately led another
to believe a particular thing true, and to act upon such belief, he cannot, in any litigation arising out of
such declaration, act or omission, be permitted to falsify it

After recognizing the validity of the lease contract for two years, the petitioner spouses are barred from
alleging the automatic cancellation of the contract on the ground that the respondents lost ownership of
the house after Virgilio acquired title over the lot. The courts a quo were unanimous in holding that the
petitioner spouses failed to
substantiate their factual averment that Virgilio not only acquired the lot but also the
house. After examining the records, we found nothing to disprove the facts determined by the lower
courts. All the petitioner spouses presented was Virgilio's uncertified xerox copy of the certificate of title
over the lot. No document was ever shown evidencing cession of the subject house in Virgilio's favor.
Virgilio's title could not be used to prove ownership over the house built on said lot as it carried no
reference at all to the house. A building by itself is a real or immovable property distinct from the land on
which it is constructed and therefore can be a separate subject of contracts. On the other hand, the
respondents proved that, as compulsory heirs of Virginia, they were the rightful owners of the subject
house. They presented a tax declaration in the name of their trustees, co-respondent Hermes Tangga-an
and his wife, which tax declaration sufficiently evidences their co-ownership and acquisition of title
following the death of the decedent Virginia.

Remedial Law Review II (CY 2018-2019) – Saturday 1:00pm – 5:00pm: Case Digest Compilation [Atty. Brondial] Page 453
D.2.a.iiiUniversity of Mindanao vs. BSP
Facts:
Guillermo B. Torres and Dolores P. Torres incorporated and operated two (2) thrift banks: (1) First Iligan
Savings & Loan Association, Inc. (FISLAI); and (2) Davao Savings and Loan Association, Inc. (DSLAI).
Guillermo B. Torres chaired both thrift banks. He acted as FISLAI's President, while his wife, Dolores P.
Torres, acted as DSLAI's President and FISLAI's Treasurer. Upon Guillermo B. Torres' request,
BangkoSentralngPilipinas issued a P1.9 million standby emergency credit to FISLAI. On May 25, 1982,
University of Mindanao's Vice President for Finance, SaturninoPetalcorin, executed a deed of real estate
mortgage over University of Mindanao's property in Cagayan de Oro City in favor of
BangkoSentralngPilipinas. "The mortgage served as security for FISLAI's PI.9 Million loan" It was
allegedly executed on University of Mindanao's behalf. As proof of his authority to execute a real estate
mortgage for University of Mindanao, SaturninoPetalcorin showed a Secretary's Certificate signed by
University of Mindanao's Corporate Secretary, Aurora de Leon. The Secretary’s certificate states among
others the authorizing of the chairman to appoint SatuninoPactolerin to represent the University of
Mindanao to transact, transfer, convey, lease, mortgage, or otherwise hypothecate the subject properties.
SaturninoPetalcorin executed another deed of real estate mortgage, allegedly on behalf of University of
Mindanao, over its two properties in Iligan City. This mortgage served as additional security for FISLAI's
loans. FISLAI and DSLAI eventually merged with DSLAI as the surviving corporation in an effort to
rehabilitate the thrift banks due to the heavy withdrawals of depositors. DSLAI later became known as
Mindanao Savings and Loan Association, Inc. (MSLAI). MSLAI failed to recover from its losses.
BangkoSentralngPilipinas later on foreclosed the mortgaged properties. University of Mindanao filed two
Complaints for nullification and cancellation of mortgage. One Complaint was filed before the
Regional Trial Court of Cagayan de Oro City, and the other Complaint was filed before the
Regional Trial Court of Iligan City. University of Mindanao alleged that it did not obtain any loan
from BangkoSentralngPilipinas and that Aurora De Leon’s certification was anomalous. That it
never authorized SaturninoPetalcorin to execute real estate mortgage contracts involving its
properties to secure FISLAI's debts and it never ratified the execution of the mortgage contracts.
The Regional Trial Courts ruled in favor of University of Mindanao. The Court of Appeals however ruled
that "although BSP failed to prove that the UM Board of Trustees actually passed a Board Resolution
authorizing Petalcorin to mortgage the subject real properties, Aurora de Leon's Secretary's Certificate"
clothed Petalcorin with apparent and ostensible authority to execute the mortgage deed on its behalf.
BangkoSentralngPilipinas merely relied in good faith on the Secretary's Certificate. University of
Mindanao is estopped from denying SaturninoPetalcorin's authority.

Before the Supreme Court, the petitioner contends that it is not bound by the by the real estate mortgage
contracts executed by SaturninoPetalcorin because the acts of an officer that are not authorized by the
board of directors/trustees do not bind the corporation unless the corporation ratifies the acts or holds the
officer out as a person with authority to transact on its behalf.

Issue:
WON petitioner is bound by the real estate mortgage executed by Petalcorin?

RULING:
No. Petitioner is correct. Corporations are artificial entities granted legal personalities upon their creation
by their incorporators in accordance with law. Unlike natural persons, they have no inherent powers.
Third persons dealing with corporations cannot assume that corporations have powers. It is up to those
persons dealing with corporations to determine their competence as expressly defined by the law and their
articles of incorporation.

A corporation may exercise its powers only within those definitions. Corporate acts that are outside those
express definitions under the law or articles of incorporation or those "committed outside the object for
which a corporation is created" are ultra vires. The only exception to this rule is when acts are necessary
and incidental to carry out a corporation's purposes, and to the exercise of powers conferred by the
Corporation Code and under a corporation's articles of incorporation. This exception is specifically
included in the general powers of a corporation under Section 36 of the Corporation Code.

Petitioner does not have the power to mortgage its properties in order to secure loans of other persons. As
an educational institution, it is limited to developing human capital through formal instruction. It is not a
corporation engaged in the business of securing loans of others.
Hiring professors, instructors, and personnel; acquiring equipment and real estate; establishing housing
facilities for personnel and students; hiring a concessionaire; and other activities that can be directly
connected to the operations and conduct of the education business may constitute the necessary and

Remedial Law Review II (CY 2018-2019) – Saturday 1:00pm – 5:00pm: Case Digest Compilation [Atty. Brondial] Page 454
incidental acts of an educational institution. Securing FISLAI's loans by mortgaging petitioner's
properties does not appear to have even the remotest connection to the operations of petitioner as an
educational institution. Securing loans is not an adjunct of the educational institution's conduct of
business. It does not appear that securing third-party loans was necessary to maintain petitioner's
business of providing instruction to individuals.

Presumptions are "inference[s] as to the existence of a fact not actually known, arising from its usual
connection with another which is known, or a conjecture based on past experience as to what course
human affairs ordinarily take." Presumptions embody values and revealed behavioral expectations under
a given set of circumstances. Presumptions may be conclusive or disputable. Conclusive presumptions
are presumptions that may not be overturned by evidence, however strong the evidence is. They are made
conclusive not because there is an established uniformity in behavior whenever identified circumstances
arise. They are conclusive because they are declared as such under the law or the rules. Rule 131, Section
2 of the Rules of Court identifies two (2) conclusive presumptions:
SEC. 2.Conclusive presumptions. — The following are instances of conclusive presumptions:
(a) Whenever a party has, by his own declaration, act, or omission, intentionally and deliberately led
another to believe a particular thing true, and to act upon such belief, he cannot, in any litigation arising
out of such declaration, act or omission, be permitted to falsify it;
(b) The tenant is not permitted to deny the title of his landlord at the time of the commencement of the
relation of landlord and tenant between them. On the other hand, disputable presumptions are
presumptions that may be overcome by contrary evidence. They are disputable in recognition of the
variability of human behavior. Presumptions are not always true. They may be wrong under certain
circumstances, and courts are expected to apply them, keeping in mind the nuances of every experience
that may render the expectations wrong. Thus, the application of disputable presumptions on a given
circumstance must be based on the existence of certain facts on which they are meant to operate. "
[P]resumptions are not allegations, nor do they supply their absence[.]" Presumptions are conclusions.
They do not apply when there are no facts or allegations to support them. If the facts exist to set in motion
the operation of a disputable presumption, courts may accept the presumption. However, contrary
evidence may be presented to rebut the presumption. Courts cannot disregard contrary evidence offered to
rebut disputable presumptions. Disputable presumptions apply only in the absence of contrary evidence or
explanations. In this case, the presumption that the execution of mortgage contracts was within
petitioner's corporate powers does not apply. Securing third-party loans is not connected to petitioner's
purposes as an educational institution.

Remedial Law Review II (CY 2018-2019) – Saturday 1:00pm – 5:00pm: Case Digest Compilation [Atty. Brondial] Page 455
D.2.b.1 Rosaroso vs. Soria
Facts:
Spouses Luis Rosaroso and HonorataDuazo acquired several real properties through the years. When
Honorata died, Luis married Lourdes Pastor Rosaroso.

Sometime later, a complaint was filed by some of Luis’ children against their sibling Lucila, Lourdes and
Meridian Realty Corporation among others. It was alleged by the petitioners that Luis, with the full
knowledge and consent of his second wife, Lourdes, executed a Deed of Absolute Sale of some of the
properties in their favor. (first sale)

Petitioners also alleged that a second sale took place when the respondents through unscrupulous means,
made Luis sign a Deed of Absolute Sale conveying to Meridian three (3) parcels of residential land. They
further averred that Meridian was in bad faith when it did not make any inquiry as to who were the
occupants and owners of said lots; and that had Meridian only investigated, it would have been informed
as to the true status of the subject properties and would have desisted in pursuing their acquisition.
(second sale)

On their part, respondents Lucila and Laila contested the First Sale in favor of petitioners. They submitted
that even assuming that it was valid, petitioners were estopped from questioning the Second Sale in favor
of Meridian because they failed not only in effecting the necessary transfer of the title, but also in
annotating their interests on the titles of the questioned properties. With respect to the assailed SPAs and
the deed of absolute sale executed by Luis, they claimed that the documents were valid because he was
conscious and of sound mind and body when he executed them. In fact, it was Luis together with his wife
who received the check payment issued by Meridian where a big part of it was used to foot his hospital
and medical expenses.

Respondent Meridian, in its Answer with Compulsory Counterclaim, averred that Luis was fully aware of
the conveyances he made. In fact, Sophia Sanchez (Sanchez), Vice-President of the corporation,
personally witnessed Luis affix his thumb mark on the deed of sale in its favor. As to petitioners'
contention that Meridian acted in bad faith when it did not endeavor to make some inquiries as to the
status of the properties in question, it countered that before purchasing the properties, it checked the titles
of the said lots with the Register of Deeds of Cebu and discovered therein that the First Sale purportedly
executed in favor of the plaintiffs was not registered with the said Register of Deeds. Finally, it argued
that the suit against it was filed in bad faith.

On her part, Lourdes posited that her signature as well as that of Luis appearing on the deed of sale in
favor of petitioners, was obtained through fraud, deceit and trickery. She explained that they signed the
prepared deed out of pity because petitioners told them that it was necessary for a loan application. In
fact, there was no consideration involved in the First Sale. With respect to the Second Sale, she never
encouraged the same and neither did she participate in it. It was purely her husband's own volition that the
Second Sale materialized. She, however, affirmed that she received Meridian's payment on behalf of her
husband who was then bedridden.

RTC ruled in favor of the petitioners. On appeal, the CA reversed and set aside the RTC decision. The
CA ruled that the first deed of sale in favor of petitioners was void because they failed to prove that
they indeed tendered a consideration for the four (4) parcels of land. It relied on the testimony of
Lourdes that petitioners did not pay her husband. The price or consideration for the sale was
simulated to make it appear that payment had been tendered when in fact no payment was made at
all.
With respect to the validity of the Second Sale, the CA stated that it was valid because the documents
were notarized and, as such, they enjoyed the presumption of regularity. Although petitioners alleged that
Luis was manipulated into signing the SPAs, the CA opined that evidence was wanting in this regard. Dr.
Arlene LetigioPesquira, the attending physician of Luis, testified that while the latter was physically
infirmed, he was of sound mind when he executed the first SPA.

Issue: WON the first sale is void?

RULING: No. The fact that the first deed of sale was executed, conveying the subject properties in favor
of petitioners, was never contested by the respondents. What they vehemently insist, though, is that the
said sale was simulated because the purported sale was made without a valid consideration.

Remedial Law Review II (CY 2018-2019) – Saturday 1:00pm – 5:00pm: Case Digest Compilation [Atty. Brondial] Page 456
Under Section 3, Rule 131 of the Rules of Court, the following are disputable presumptions: (1) private
transactions have been fair and regular; (2) the ordinary course of business has been followed; and (3)
there was sufficient consideration for a contract.

These presumptions operate against an adversary who has not introduced proof to rebut them. They create
the necessity of presenting evidence to rebut the prima facie case they created, and which, if no proof to
the contrary is presented and offered, will prevail. The burden of proof remains where it is but, by the
presumption, the one who has that burden is relieved for the time being from introducing evidence in
support of the averment, because the presumption stands in the place of evidence unless rebutted.

In this case, the respondents failed to trounce the said presumption. Aside from their bare allegation that
the sale was made without a consideration, they failed to supply clear and convincing evidence to back up
this claim. It is elementary in procedural law that bare allegations, unsubstantiated by evidence, are not
equivalent to proof under the Rules of Court.

The CA decision ran counter to this established rule regarding disputable presumption. It relied heavily
on the account of Lourdes who testified that the children of Luis approached him and convinced him to
sign the deed of sale, explaining that it was necessary for a loan application, but they did not pay the
purchase price for the subject properties. This testimony, however, is self-serving and would not amount
to a clear and convincing evidence required by law to dispute the said presumption. As such, the
presumption that there was sufficient consideration will not be disturbed.

Granting that there was no delivery of the consideration, the seller would have no right to sell again what
he no longer owned. His remedy would be to rescind the sale for failure on the part of the buyer to
perform his part of their obligation pursuant to Article 1191 of the New Civil Code.

Remedial Law Review II (CY 2018-2019) – Saturday 1:00pm – 5:00pm: Case Digest Compilation [Atty. Brondial] Page 457
D.2.b.2 HEIRS OF CIPRIANO TRAZONA vs. HEIRS OF DIONISIO CAÑADA

G.R. No. 175874; December 11, 2013

FACTS: Cipriano Trazona owned Lot No. 5053–H covered by Tax Declaration No. 07764. Sometime in
1997, when the heirs of Cipriano, herein petitioners, tried to secure a copy of Tax Declaration No. 07764,
they were informed that Tax Declaration No. 07764 had been cancelled and, in lieu thereof, Tax
Declaration No. 23959 was issued on 24 June 1996 in the name of Dionisio, the owner of the property
adjacent to Cipriano’s lot. Apparently, respondents had caused the issuance of Tax Declaration No. 23959
by submitting a Deed of Absolute Sale supposedly executed by Cipriano in favor of Dionisio. The deed of
sale covers a portion of Cipriano’s property which was encroached upon by Dioniso during the former’s
lifetime, but the new tax declaration issued covers the whole property of Cipriano.

Consequently, petitioners filed a Complaint against respondents for quieting of title, annulment of deed of
sale, cancellation of Tax Declaration No. 23959. Petitioners alleged therein that the Deed of Absolute
Sale dated 27 June 1956 was a forgery. During the trial, petitioners presented an expert witness testifying
to the forgery of Cipriano’s signature on the assailed deed.

RTC ruled in favor of petitioners. Based on RTC Judge’s observation Cipriano’s signature on the assailed
deedbares a difference as compared to other documents bearing his signature. On appeal, CA ruled that
petitioners had failed to prove by requisite evidence their allegation that the assailed deed was a forgery.
The deed, being a notarized document, enjoyed the presumption of authenticity and due execution. Also,
the fact that it was an ancient document that “remained unaltered after so many years, bodes well for its
authenticity.

ISSUE: Whether or not the presumption of regularity of a notarized ancient document may be assailed by
the testimony of an expert witness and independent observation of the trial court as to the forgery of
signature therein.

RULING:The answer is in the affirmative. It is true that notarized documents are accorded evidentiary
weight as regards their due execution. Nevertheless, while notarized documents enjoy the presumption of
regularity, this presumption is disputable. They can be contradicted by evidence that is clear, convincing,
and more than merely preponderant.

In this case, clear and convincing evidence that is enough to overturn the presumption of regularity of the
assailed deed was presented. First, the document examiner determined that the signature of Cipriano in
the assailed deed had been forged. No issue has been raised about his expertise. Second, the RTC did not
just rely on expert testimony in ruling that the signature was forged. It likewise supported its finding that
the signature was forged through independent observation.

Lastly, when the record management analyst from the Bureau of Archives presented the assailed deed, the
paper was noted to be white, while its supposed contemporaries in the bunch from where it was taken had
turned yellow with age. Further, when the analyst was asked the question of when the assailed deed was
received by the Bureau of Archives, she answered that it was forwarded to them only on 28 September
1987 by RTC Region 7, Notarial Division.

Remedial Law Review II (CY 2018-2019) – Saturday 1:00pm – 5:00pm: Case Digest Compilation [Atty. Brondial] Page 458
D.2.b.3 Uy vs. Lacsamana
[ GR No. 206220, Aug 19, 2015 ]
LUIS UY v. SPS. JOSE LACSAMANA AND ROSAURA* MENDOZA +
DECISION
CARPIO, J.:

- Facts:
o Petitioner Luis Uy (Uy) filed with the Regional Trial Court (RTC) a Complaint for Declaration of
Nullity of Documents with Damages against respondents Petra Rosca (Rosca), and spouses Jose
Lacsamana and Rosaura Mendoza (Spouses Lacsamana). Uy alleged that he was the lawful husband
of Rosca and they lived together as husband and wife from the time they were married in 1944 until
1973 when they separated and lived apart.
o Uy alleged that he and his wife acquired a two adjacent residential land in Batangas from the Spouses
Manuel and Spouses Contreras. Uy further alleged that Rosca, in gross and evident bad faith,
executed and signed a false and simulated Deed of Sale in favor of Spouses Lacsamana.
o Rosca denied the allegations of Uy and claimed that she lawfully acquired the subject real properties
using her paraphernal funds. Rosca added that she was never married to Uy and prayed for the
dismissal of the complaint for lack of merit. Spouses Lacsamana claimed that they were buyers in
good faith and for value and that they relied on the Torrens title which stated that Rosca was the
owner of the subject property.
o Uy questioned the registrability of the Deed of Sale before the Office of the Register of Deeds of
Batangas City. On consulta with the Land Registration Commission (LRC), LRC ruled in favor of
registration stating that since the property in question was registered in Rosca's name, such
circumstance indicated that the property belonged to Rosca, as her paraphernal property.
o Uy contends that the Deed of Sale executed by Rosca is not valid for being simulated or fictitious for
lack of consideration and consent. Uy states that no proof was presented by Spouses Lacsamana to
show that they actually paid P80,000 to Rosca for the purchase of the property and that the
consideration paid was unreasonably low and unconscionable such that it constitutes an equitable
mortgage.
- Issue: W/N the Deed of Sale executed by Rosca alone, without Uy's consent, in favor of Spouses
Lacsamana, is valid.
- Ruling:
o There is a presumption established in our Rules "that a man and woman deporting themselves as
husband and wife have entered into a lawful contract of marriage." Semper praesumitur pro
matrimonio — Always presume marriage. However, this presumption may be contradicted by a party
and overcome by other evidence.
o Marriage may be proven by any competent and relevant evidence. The testimony by one of the parties
to the marriage, or by one of the witnesses to the marriage, as well as the person who officiated at the
solemnization of the marriage, has been held to be admissible to prove the fact of marriage. The best
documentary evidence of a marriage is the marriage contract itself.
o Persons living together in apparent matrimony are presumed, absent any counter presumption or
evidence special to the case, to be in fact married. Consequently, with the presumption of marriage
sufficiently overcome, the onus probandi of defendant Rosca shifted to plaintiff Uy. It then became
the burden of plaintiff Uy to prove that he and defendant Rosca, were legally married. It became
necessary for plaintiff Uy therefore to submit additional proof to show that they were legally married.
He, however, dismally failed to do so.
o Uy submitted showed that he and Rosca were not legally married to each other. In his Petition for
Naturalization as a Filipino citizen, Uy himself stated "I am married (not legally)." Sworn Statement
of Governor of the Province of Batangas executed in support of the plaintiff Uy's Petition for
Naturalization categorically states, in Nos. 2 and 4 thereof, that plaintiff Uy was married (not legally).
The Immigrant Certificate of Residence and Alien Certificate of Registration shows that plaintiff Uy
regarded himself as "single" when filling up his civil status therein.
o Since Uy failed to discharge the burden that he was legally married to Rosca, their property relations
would be governed by Article 147 of the Family Code which applies
o Art. 147. When a man and a woman who are capacitated to marry each other, live exclusively with
each other as husband and wife without the benefit of marriage or under a void marriage, their wages
and salaries shall be owned by them in equal shares and the property acquired by both of them
through their work or industry shall be governed by the rules on co-ownership.
o In the absence of proof to the contrary, properties acquired while they lived together shall be
presumed to have been obtained by their joint efforts, work or industry, and shall be owned by them
in equal shares. A party who did not participate in the acquisition by the other party of any property

Remedial Law Review II (CY 2018-2019) – Saturday 1:00pm – 5:00pm: Case Digest Compilation [Atty. Brondial] Page 459
shall be deemed to have contributed jointly in the acquisition thereof if the former's efforts consisted
in the care and maintenance of the family and of the household.
o Neither party can encumber or dispose by acts inter vivos of his or her share in the property acquired
during cohabitation and owned in common, without the consent of the other, until after the
termination of their cohabitation. When only one of the parties to a void marriage is in good faith, the
share of the party in bad faith in the co-ownership shall be forfeited in favor of their common
children. In case of default of or waiver by any or all of the common children or their descendants,
each vacant share shall belong to the respective surviving descendants. In the absence of descendants,
such share shall belong to the innocent party. In all cases, the forfeiture shall take place upon
termination of the cohabitation.
o Properties acquired during cohabitation are presumed co-owned unless there is proof to the contrary.
o LRC in LRC Consulta, Rosca was recognized as the sole registered owner of the property.
o The Deed of Sale between Spouses Manuel and Rosca, Uy served as a mere witness to Rosca's
purchase of the land as evidenced by his signature under "signed in the presence of." This could only
mean that Uy admitted the paraphernal nature of Rosca's ownership over the property.
o The title to the property in the name of "Petra Rosca, married to Luis G. Uy" was notice to the world,
including her heirs and successors-in-interest, that such belonged to Rosca as her paraphernal
property. The words "married to" were merely descriptive of Rosca's status at the time the property
was registered in her name. Otherwise, if the property was conjugal, the title to the property should
have been in the names of Luis Uy and Petra Rosca.
o Based on the evidence she presented, Rosca was able to sufficiently overcome the presumption that
any property acquired while living together shall be owned by the couple in equal shares. The house
and lot were clearly Rosca'sparaphernal properties and she had every right to sell the same even
without Uy's consent.
o Uy did not present any proof to show that Rosca did not receive any consideration for the sale or that
the fair market value of the property at the time of the sale to prove that the purchase price was
unreasonably low or unconscionable.
o The Deed of Sale, executed by Rosca on her paraphernal property in favor of Spouses Lacsamana, is
valid.
o Denied.

Remedial Law Review II (CY 2018-2019) – Saturday 1:00pm – 5:00pm: Case Digest Compilation [Atty. Brondial] Page 460
D.2.b.4 Diaz vs. People
G.R. No. 208113, December 02, 2015
DOLORES DIAZ, Petitioner, v. PEOPLE OF THE PHILIPPINES AND LETICIA S. ARCILLA,
Respondents.
DECISION
PERLAS-BERNABE, J.:

- Facts:
o An Information for estafa was filed against petitioner for her alleged failure to return or remit the
proceeds from various merchandise valued at P32,000.00 received by her in trust - i.e., on
consignment basis of umbrellas and bath towels from respondent.
o Petitioner admitted having previous business dealings with respondent but not as an agent. She
clarified that she was a client who used to buy purchase order cards (POCs) and gift checks (GCs)
from respondent on installment basis and that, during each deal, she was made to sign a blank sheet of
paper prior to the issuance of POCs and GCs. She further claimed that their last transaction was
conducted in 1995, which had long been settled. However, she denied having received P32,000.00
worth of merchandise from respondent
o RTC found that the prosecution failed to establish any intent on the part of the petitioner to defraud
respondent and, thus, could not be held criminally liable. CA modified and held petitioner as civilly
liable. CA rejected petitioner's attempt to discredit the said receipt which she denied executing on the
ground that she was only made to sign blank documents, finding that even if petitioner was indeed
made to sign such blank documents, such was merely a safety precaution employed by respondent in
the event the former reneges on her obligation.
o Petitioner's claim that she was required to sign two (2) one-half sheets of paper and a trust receipt in
blank during her transactions with respondent, which she allegedly failed to retrieve after paying her
obligations
- Issue: W/N petitioner is civilly liable to respondent.
- Ruling:
o Yes. Extinction of the penal action does not carry with it the extinction of the civil liability where the
acquittal is based on reasonable doubt as only preponderance of evidence, or "greater weight of the
credible evidence," is required.
o CA correctly found that respondent was able to prove by preponderance of evidence the fact of the
transaction, as well as petitioner's failure to remit the proceeds of the sale of the merchandise worth
P32,000.00, or to return the same to respondent in case such merchandise were not sold. This was
established through the presentation of the acknowledgment receipt
o Under Section 3 (d), Rule 131 of the Rules of Court, the legal presumption is that a person takes
ordinary care of his concerns. Natural presumption is that one does not sign a document without first
informing himself of its contents and consequences. Under Section 3 (p) of the same Rule, it is
equally presumed that private transactions have been fair and regular. This behooves every
contracting party to learn and know the contents of a document before he signs and delivers it. The
effect of a presumption upon the burden of proof is to create the need of presenting evidence to
overcome the prima facie case created, thereby which, if no contrary proof is offered, will prevail
o Petitioner failed to present any evidence to controvert these presumptions. Also, respondent's
possession of the document pertaining to the obligation strongly buttresses her claim that the same
has not been extinguished.
o Denied.

Remedial Law Review II (CY 2018-2019) – Saturday 1:00pm – 5:00pm: Case Digest Compilation [Atty. Brondial] Page 461
D.2.b.5 People vs. Padrigone
[G.R. No. 137664. May 9, 2002]
PEOPLE OF THE PHILIPPINES, plaintiff-appellee, vs. ROBERTO PADRIGONE a.k.a.
ROBERTO SAN MIGUEL, accused-appellant.
DECISION
YNARES-SANTIAGO, J.:

- Facts:
o Roberto Padrigone a.k.a. Roberto San Miguel, Michael San Antonio, JocelIbaneta and
AbelardoTriumpante were charged with rape
o Appellant Roberto Padrigone and the other accused broke into the house of Rowena Contridas.
Appellant Roberto Padrigone and accused JocelIbaneta poked a knife at Rowena and her fourteen
year-old sister, Nimfa, and threatened to kill them if they reported the incident to others. They gagged
Rowena with a handkerchief and Nimfa with a handtowel. Then, appellant undressed Rowena, forced
her to lie down and sexually violated her while his co-accused watched with glee. Accused
JocelIbanita tried to rape Nimfa but failed because she was able to elude him.
o The accused took their turns.
o Rowena and Nimfa reported the incident to the police and identified appellant and his co-accused as
the perpetrators. Also, she underwent medical examination and later revealed that Rowena suffered
Acute Psychotic Depressive Condition.
o All the accused, including appellant Roberto Padrigone, interposed the defense of denial and alibi.
Appellant claimed that in that evening, he and his companions, JocelIbanita and Michael San
Antonio, visited Rowena at her house. Rowena told him that she wanted to elope with him but he
refused.
o Lower court convicted the accused.
o Appellant assails the procedural irregularities committed by the prosecution and by the trial court. He
claims that the prosecution suppressed evidence by not presenting Rowena, the victim, when the latter
should have had her sane moments. As a consequence, the trial court deprived appellant of the
opportunity to cross-examine her when she allegedly declared before the Chief of Police of Buhi that
it was only appellant who raped her which declaration became the basis for the latters conviction.
- Issue: W/N there is suppression of evidence?
- Ruling:
o Non-presentation of Rowena on the witness stand cannot be considered as suppression of evidence.
Under Rule 131, Section 3(e) of the Rules of Court, the rule that evidence willfully suppressed would
be adverse if produced does not apply if
1.the evidence is at the disposal of both parties;
2.the suppression was not willful;
3.it is merely corroborative or cumulative; and
4.the suppression is an exercise of a privilege.
o The defense had the opportunity to subpoena Rowena even if the prosecution did not present her as a
witness. Instead, the defense failed to call her to the witness stand. Second, Rowena was certified to
be suffering from Acute Psychotic Depressive Condition and thus cannot stand judicial proceedings
yet. The non-presentation, therefore, of Rowena was not willful. Third, in any case, while Rowena
was the victim, Nimfa was also present and in fact witnessed the violation committed on her sister.

Remedial Law Review II (CY 2018-2019) – Saturday 1:00pm – 5:00pm: Case Digest Compilation [Atty. Brondial] Page 462
D.2.b.6 METROPOLITAN BANK & TRUST COMPANY vs. COURT OF APPEALS and G.T.P.
DEVELOPMENTCORPORATION

G.R. No. 122899, June 8, 2000

FACTS: Mr. Chia and Respondent GTP entered into a contract of sale with assumption of mortgage
wherein the latter assumes the former’s indebtedness with Metrobank. Respondent, pursuant to the
balance declared by Metrobank prior to the execution of the sale, paid Chia’s loan balance with
Metrobank in the amount of P116K+. This notwithstanding, petitioner METROBANK refused to release
the real estate mortgage on the subject property despite repeated requests, thus prompting respondent
GTP to file an action for specific performance against petitioner METROBANK and Mr. Chia.
Metrobank refused to discharge of the real estate mortgage on the claim that the subject property still
secures "other unliquidated past due loans” as there exist a stipulation in subject Deeds of Mortgage that
mortgagors’ debts subsequently obtained would be covered by the same security. Hence, the payment
made by GPT does not extinguish the mortgage. The trial court ruled in favor of the respondent. On
appeal, CA reversed the decision. Respondent filed a motion for reconsideration.

With this unfavorable turn of events, respondent GTP, filed a motion for reconsideration with alternative
prayer to require METROBANK to furnish appellee (GTP) of the alleged unpaid balance of Mr. Chia."
At the re-scheduled date of oral arguments where METROBANK was supposed to bring before the CA
the current statement of the mortgage debt of Mr. Chia secured by the deeds of mortgage sought to be
released, METROBANK’s counsel did not appear. Consequently, CA reversed its previous decision.

ISSUE: Whether or not failure of the mortgagee to present the proof of loan secured by the mortgage
stopped him from further asserting the existence of such liability

RULING: The answer is in the affirmative. It is a well-settled rule that when the evidence tends to prove
a material fact which imposes a liability on a party, and he has it in his power to produce evidence which
from its very nature must overthrow the case made against him if it is not founded on fact, and he refuses
to produce such evidence, the presumption arises that the evidence, if produced, would operate to his
prejudice, and support the case of his adversary.

The scheduled oral arguments before the CA was supposed to be Metrobank’s golden opportunity to
prove the existence the "other unliquidated past due loans" which is the basis of its refusal to release the
mortgage property. But Metrobank failed to appear thereon.

No rule of law is better settled than that a party having it in his power to prove a fact, if it exists, which, if
proved, would benefit him, his failure to prove it must be taken as conclusive that the fact does not exist."

Remedial Law Review II (CY 2018-2019) – Saturday 1:00pm – 5:00pm: Case Digest Compilation [Atty. Brondial] Page 463
D.2.b.7 PEOPLE v. BARTE PEOPLE OF THE PHILIPPINES, Plaintiff-Appellee, vs. EDDIE
BARTE Y MENDOZA, Accused-Appellant

G.R. No. 179749; March 1, 2017; BERSAMIN, J.:

FACTS:Under review is the decision of the Court of Appeals (CA), affirming the decision RTC,
convicting the accused-appellant of violating Section 5, Article II of Republic Act No. 9165, as amended.

Eddie Barte Y Mendoza was arrested for selling shabu to a police officer-poseur buyer during a buy-bust
operation conducted. He was subsequently charged in the RTC with a violation of Section 5, Article II of
R.A. No. 9165, as amended.

Prosecution witness, P02 Rico Cabatingan, declared that he and other police officers conducted the buy-
bust operation at about 9:30 in the evening of August 10, 2002 on the basis of an information received,
that the accused-appellant was engaged in the sale of shabu. During the pre-operation conference, P02
Cabatingan was designated as the poseur buyer, and his back-up officers were P02 Baylosis and P03
Ompad. P/Insp. Grado provided the buy-bust money with marked serial number to P02 Cabatingan. The
buy-bust team then proceeded to Consuela Village at about 9:10 in the evening, on board a Suzuki
multicab driven by P03 Ompad.

P02 Cabatingan met with the accused-appellant, informing the latter that he wanted to buy shabu worth "a
peso." Upon the accused-appellant's assent to his offer, P02 Cabatingan handed the buy-bust money to
him, and in turn the latter gave him a small sachet with white colored contents. P02 Cabatingan then send
a pre-arranged signal by touching his head to the other officers, who then rushed forward and identified
themselves to the accused-appellant as policemen. They frisked and arrested him, and brought him to the
police station.

The accused-appellant, in his defense, declared that he was sitting alone near the chapel of Basak,
Mandaue City near their house in Consuela Village at around 9:30 in the evening of August 10, 2002
when police officers suddenly came and arrested him, while pointing their guns at him and forcing him to
go with them. They brought him to the police precinct on a Suzuki multicab, and upon arrival, the
arresting officers searched his person and found his ID inside his wallet. He was subsequently detained
without informing him of the reason of his arrest not until the next day, where they informed him of his
charges.

ISSUE: Whether the guilt of the accused-appellant for the crime charged proved beyond reasonable
doubt.

RULING:No After thorough review, we consider the appeal to be impressed with merit. Thus, we acquit
the accused-appellant.

Courts are cognizant of the presumption of regularity in the performance of duties of public officers. This
presumption can be overturned if evidence is presented to prove either of two things, namely: (1) that
they were not properly performing their duty, or (2) that they were inspired by any improper motive. This
case sprang from the buy-bust operation conducted by several police officers against the accused-
appellant based on the tip from a caller whose identification was only through the alias of Ogis.
Surveillance was made following such tip, but the same was unrecorded and no other proof was presented
to corroborate the policemen's conclusion that the man known as Ogis was the same man they were
looking for during the surveillance.

It is a matter of judicial notice that buy-bust operations are "susceptible to police abuse, the most
notorious of which is its use as a tool for extortion." The high possibility of abuse was precisely the
reason why the procedural safeguards embodied in Section 21 of R.A. No. 9165 have been put up as a
means to minimize, if not eradicate such abuse. The procedural safeguards not only protect the innocent
from abuse and violation of their rights but also guide the law enforcers on ensuring the integrity of the
evidence to be presented in court.

The chain of custody as an imp011ant procedural safeguard is defined under Section l (b) of Dangerous
Drugs Board Regulation No. 1, Series of 2002, as follows:
Chain of Custody means the duly recorded authorized movements and custody of seized drugs or
controlled chemicals or plant sources of dangerous drugs or laboratory equipment of each stage, from the
time or seizure/confiscation to receipt in the forensic laboratory to safekeeping to presentation in court for

Remedial Law Review II (CY 2018-2019) – Saturday 1:00pm – 5:00pm: Case Digest Compilation [Atty. Brondial] Page 464
destruction. Such record of movements and custody of seized item shall include the identity and signature
of the person who held temporary custody of the seized item, the date and time when such transfer of
custody were made in the course of safekeeping and use in court as evidence, and the final disposition.
Based on the foregoing, we regard and declare as unwarranted the RTC's position that the absence of
proof showing the compliance by the arresting lawmen with the procedure outlined under Section 21 of
RA No. 9165 was not fatal to the entrapment. Such non-compliance with the procedural safeguards under
Section 21 was fatal because it cast doubt on the integrity of the evidence presented in court and directly
affected the validity of the buy-bust operation. It put into serious question whether the sachet of shabu
had really come from the accused-appellant, and whether the sachet of shabu presented in court was the
same sachet of shabu obtained from the accused-appellant at the time of the arrest. Testimonies provided
by the police officers and the presumption of regularity in the performance of their duties did not
override the non-compliance with the procedural safeguards instituted by our laws. Indeed, anything
short of observance and compliance by the arresting lawmen with what the law required meant that the
former did not regularly perform their duties. The presumption of regularity in the performance of their
duties then became inapplicable. As such, the evidence of the State did not overturn the presumption of
innocence in favor of the accused-appellant.

Furthermore, although non-compliance with the prescribed procedural requirements would not
automatically render the seizure and custody of the contraband invalid, that is true only when there is a
justifiable ground for such non-compliance, and the integrity and evidentiary value of the seized items are
properly preserved. Any departure from the prescribed procedure must then still be reasonably justified,
and must further be shown not to have affected the integrity and evidentiary value of the confiscated
contraband. Otherwise, the non-compliance constitutes an irregularity, a red flag, so to speak, that cast
reasonable doubt on the identity of the corpus delicti.

Here, the State's agents who entrapped the accused-appellant and confiscated the dangerous drug from
him did not tender any justifiable ground for the non-compliance with the requirement of establishing
each link in the chain of custody from the time of seizure to the time of presentation. The conclusion that
the integrity and evidentiary value of the shabu confiscated were consequently not preserved became
unavoidable. The failure to prove the chain of custody should mean, therefore, that the Prosecution did
not establish beyond reasonable doubt that the sachet of shabu presented during the trial was the very
same one delivered by the accused-appellant to the poseur buyer.

Remedial Law Review II (CY 2018-2019) – Saturday 1:00pm – 5:00pm: Case Digest Compilation [Atty. Brondial] Page 465
D.2.b.8 FILOMENA G. DELOS SANTOS VS. COMMISSION ON AUDIT

G.R. No. 198457; AUGUST 13, 2013; PERLAS-BERNABE, J:

FACTS: Sometime in October 2001, then Congressman Antonio V. Cuenco of the Second District of
Cebu entered into a Memorandum of Agreement with Vicente Sotto Memorial Medical Center
(VSMMC), represented by Dr. Eusebio Alquizalas, Medical Chief, appropriating to the hospital the
amount of 1,500,000.00 Php from his Priority Development Assistance fund (PDAF) to cover the medical
assistance of the indigent patient who may avail of the benefits of the TNT Health program. Several years
after the enforcement of the MOA, allegations of forgery and falsification of the prescription and referrals
for the availment of medicines under the TNT program surfaced. On December 14, 2004, petitioner,
Filomena Delos Santos who succeeded Dr. Alquizalas, created, through hospital Order No. 1112, a fact-
finding committee to investigate the matter.. Within the same month, Beatriz Booc, State auditor who was
assigned to audit the hospital came up with her own review of the account for drugs and medicines
charged to the PDAF of Cuenco. She furnished Delos Santos the results of her review as contained in the
Audit Observation Memoranda recommending investigation to some irregularities.

In the initial investigation conducted by CoA, it was found that 133 prescriptions for vaccines, drugs and
medicine for anti-rabies allegedly dispensed by Dell Pharmacy costing more than 3 million pesos, and
already paid by VSMMC from the PDAF of Cuenco appeared to be falsified, 46 prescriptionsfor other
drugs and medicines allegedly dispensed by Dell pharmacy costing 700,000 and already paid by VSMMC
from the PDAF of Cuenco likewise appeared to be falsified and prescriptions for drugs and medicines
allegedly issued by Dell Pharmacy costing 600,000 pesos ascertained to be falsified and have not been
paid by VSMMC. Delos Santos replied that Cuenco, sometime in 2002, put up the TNT office, which was
run by its own staff. From then on, the hospital had no more participation in the said program and was
relegated to a mere bag keeper.

Consequently a Special Audit Team (SAT) conducted a special audit investigation with respect to the
findings of Booc and her team. Subsequently, on September 8, 2008, the SAT Team issued an order
disallowing the amount of 3.3 million for the payment of drugs and medicines for the anti-rabies with
falsified prescription and documents, and holding petitioners together with other VSMMC officials
solidarily liable.

Aggrieved, petitioner filed their respective appeals before the CoA which were denied, maintaining their
solidary liability.

ISSUE: Whether or not Petitioners regularly performed their official duties.

RULING: No. jurisprudence holds that, absent any showing of bad faith and malice, there is a
presumption of regularity in the performance of the official duty. However, this presumption must fail in
the presence of an explicit rule that was violated.

The court finds that the petitioners herein have equally failed to make a case justifying their non-
observance of existing auditing rules and regulations, and of their duties under MOA. Evidently,
petitioner’s neglect to properly monitor the disbursement of PDAF facilitated the validation and eventual
payment of 133 falsified prescriptions and fictitious claims for anti-rabies vaccines supplied by VSMMC
and Dell Pharmacy, despite the irregularitiesborne out by the referral slips and prescriptions related
thereto.

All told, petitioner’s acts and/or ommissions as detailed by the CoA issuances and as aforedescribed
reasonably figure into the finding that they failed to faithfully discharge their respective duties and to
exercise the required diligence which resulted to the irregular disbursement s from Cuenco’s PDAF.

Remedial Law Review II (CY 2018-2019) – Saturday 1:00pm – 5:00pm: Case Digest Compilation [Atty. Brondial] Page 466
D.2.b.9 PEOPLE OF THE PHILIPPINES vs. HADJI SOCOR CADIDIA

G.R. No. 19126; October 16, 2013

FACTS: Accused was charged with violation of RA 9165. The prosecution presented Trayvilla, a Non-
Uniformed Personnel of the PNP, who testified that on 31 July 2002 at around 6:30 in the morning, while
performing her duty as a female frisker assigned at the NAIA Terminal I, she frisked the accused Cadidia
upon her entry at the departure area and she noticed something unusual and thick in the area of Cadidia’s
buttocks. Upon inquiry, Cadidia answered that it was only her sanitary napkin which caused the unusual
thickness. Not convinced with Cadidia’s explanation, Trayvilla and her female co-employee Bagsican
brought the accused to the comfort room inside the domestic airport to check. When she and Bagsican
asked Cadidia to remove her underwear, they discovered that inside were two sachets of shabu. The two
sachets of shabu were turned over to their supervisor SPO3 Musalli I. Appang (SPO3 Appang). Trayvilla
recalled that Cadidia denied that the two sachets of shabu were hers and said that she was only asked by
an unidentified person to bring the same.

During trial, accused interposed the defense of frame-up. Both the trial court and the CA, on appeal,
conviced the accused. The accused also assails the application of presumption of regularity in the
performance of duties of the witnesses. She claimed that the self- serving testimonies of Trayvilla and
Bagsican failed to overcome her presumption of innocence guaranteed by the Constitution.

ISSUE: Whether or not the presumption of regularity in the performance of duties of a public officer may
be assailed by bare allegations of frame-up.

RULING: The answer is in the affirmative. In cases involving violations of the Dangerous Drugs Act,
credence is given to prosecution witnesses who are police officers for they are presumed to have
performed their duties in a regular manner, unless there is evidence to the contrary suggesting ill-motive
on the part of the police officers.”

In this case, the prosecution witnesses were unable to show ill-motive for the police to impute the crime
against Cadidia. Trayvilla was doing her regular duty as an airport frisker when she handled the accused
who entered the x-ray machine of the departure area. There was no pre-determined notice to particularly
search the accused especially in her private area. The unusual thickness of the buttocks of the accused
upon frisking prompted Trayvilla to notify her supervisor SPO3 Appang of the incident. The subsequent
search of the accused would only show that the two female friskers were just doing their usual task when
they found the illegal drugs inside accused’s underwear. This is bolstered by the fact that the accused on
the one hand and the two friskers on the other were unfamiliar to each other. Neither could they harbour
any ill-will against each other. The allegation of frame-up and denial of the accused cannot prevail over
the positive testimonies of three prosecution witnesses who corroborated on circumstances surrounding
the apprehension.

Remedial Law Review II (CY 2018-2019) – Saturday 1:00pm – 5:00pm: Case Digest Compilation [Atty. Brondial] Page 467
D.2.b.10 Yuk Ling Ong vs. Benjamin T. Co

G.R. No. 206653; February 25, 2015

FACTS: Petitioner Yuk Ling Ong, a British-Hong Kong national, and respondent Benjamin Co, a
Filipino citizen, were married on October 3, 1982.

Sometime in November 2008, petitioner received a subpoena from the Bureau of Immigration and
Deportation (BID) directing her to appear before the said agency because her permanent residence visa
was being subjected to cancellation proceedings. Reportedly, her marriage with respondent was nullified
by the court.When petitioner appeared before the BID, she was furnished with the copies of the following
documents: (1) petition for declaration of nullity of marriage was filed; (2) Decision of the Regional Trial
Court (RTC) declaring the marriage between petitioner and respondent as void ab initio; and (3) their
marriage contract with the subject decision annotated thereon. Petitioner was perplexed that her marriage
with respondent had been declared void ab initio.

The above documents showed that on April 26, 2001, respondent filed a petition for declaration of nullity
on the ground of psychological incapacity before the RTC.

Respondent indicated that petitioner’s address was 23 Sta. Rosa Street, Unit B-2 Manresa Garden Homes,
Quezon City. On July 29, 2002, the RTC issued summons. In his Server’s Return, process server Rodolfo
Torres, Jr. stated that, on August 1, 2002, substituted service of summons with the copy of the petition
was effected after several futile attempts to serve the same personally on petitioner. The said documents
were received by Mr. Roly Espinosa, a security officer.

Petitioner alleged that first, respondent committed extrinsic fraud because he deliberately indicated a
wrong address to prevent her from participating in the trial; second, jurisdiction over her person was not
acquired because of an invalid substituted service of summons as no sufficient explanation, showing
impossibility of personal service, was stated before resorting to substituted service of summons; third, the
alleged substituted service was made on a security guard of their townhouse and not on a member of her
household; and fourth, she was not psychologically incapacitated to perform her marital obligations.

Petitioner argues that there was an invalid substituted service of summons. The process server’s return
only contained a general statement that substituted service was resorted to “after several futile attempts to
serve the same personally,” without stating the dates and reasons of the failed attempts. 

In his Comment, filed on July 9, 2014, respondent contended that the server’s return satisfactorily stated
the reason for the resort to a substituted service of summons on August 1, 2002; and it was improbable
that petitioner failed to receive the summons because it was sent to the same address which she declared
in this present petition.

ISSUE: Whether or not the Trial Court validly acquired jurisdiction over the person of the petitioner.

RULING: Jurisdiction over the defendant is acquired either upon a valid service of summons or the
defendant's voluntary appearance in court. If the defendant does not voluntarily appear in court,
jurisdiction can be acquired by personal or substituted service of summons as laid out under Sections 6
and 7 of Rule 14 of the Rules of Court.

The landmark case of Manotoc v. CA (Manotoc) thoroughly discussed the rigorous requirements of a
substituted service of summons, to wit: 

(1) Impossibility of Prompt Personal Service 

For substituted service of summons to be available, there must be several attempts by the sheriff to
personally serve the summons within a reasonable period of one month which eventually resulted in
failure to prove impossibility of prompt service. "Several attempts" means at least three (3) tries,
preferably on at least two different dates. In addition, the sheriff must cite why such efforts were
unsuccessful. It is only then that impossibility of service can be confirmed or accepted. 

(2) Specific Details in the Return 

Remedial Law Review II (CY 2018-2019) – Saturday 1:00pm – 5:00pm: Case Digest Compilation [Atty. Brondial] Page 468
The sheriff must describe in the Return of Summons the facts and circumstances surrounding the
attempted personal service. The efforts made to find the defendant and the reasons behind the failure must
be clearly narrated in detail in the Return. The date and time of the attempts on personal service, the
inquiries made to locate the defendant, the name/s of the occupants of the alleged residence or house of
defendant and all other acts done, though futile, to serve the summons on defendant must be specified in
the Return to justify substituted service.

 (3) A Person of Suitable Age and Discretion

The sheriff must therefore determine if the person found in the alleged dwelling or residence of defendant
is of legal age, what the recipient's relationship with the defendant is, and whether said person
comprehends the significance of the receipt of the summons and his duty to immediately deliver it to the
defendant or at least notify the defendant of said receipt of summons. These matters must be clearly and
specifically described in the Return of Summons.

The server’s return utterly lacks sufficient detail of the attempts undertaken by the process server to
personally serve the summons on petitioner. The server simply made a general statement that summons
was effected after several futile attempts to serve the same personally. The server did not state the specific
number of attempts made to perform the personal service of summons; the dates and the corresponding
time the attempts were made; and the underlying reason for each unsuccessful service. He did not explain
either if there were inquiries made to locate the petitioner, who was the defendant in the case. These
important acts to serve the summons on petitioner, though futile, must be specified in the return to justify
substituted service.

The server’s return did not describe in detail the person who received the summons, on behalf of
petitioner. It simply stated that the summons was received “by Mr. Roly Espinosa of sufficient age and
discretion, the Security Officer thereat.” It did not expound on the competence of the security officer to
receive the summons.
Given that the meticulous requirements in Manotoc were not met and there was an invalid substituted
service of summons. The decision in Civil Case must be declared null and void.

The stricter rule in substituted service of summons was meant to address "the numerous claims of
irregularities in substituted service which have spawned the filing of a great number of unnecessary
special civil actions of certiorari and appeals to higher courts, resulting in prolonged litigation and
wasteful legal expenses." Although the decision in Civil Case was promulgated as early as December 11,
2002, the Court must strike it down for lack of jurisdiction over the person of petitioner. The favorable
judgment enjoyed by respondent cannot be categorized as a genuine victory because it was fought against
an adversary, who was ignorant of the existing dispute. Whatever prize bestowed upon the victor in such
a void decision must also be undone. Respondent, if he wishes to pursue, must start from scratch and
institute his action for declaration of nullity again; this time with petitioner fully aware and ready for
litigation.

Remedial Law Review II (CY 2018-2019) – Saturday 1:00pm – 5:00pm: Case Digest Compilation [Atty. Brondial] Page 469
D.2.b.11 PEOPLE OF THE PHILIPPINES vs. JESUS EDUALINO

G.R. No. 119072, April 11, 1997

FACTS: Accused Jesus Edualino was charged with consummated rape of a pregnant
woman.

Prosecution presented the following witnesses: (1) Rowena Nantiza – victim/complainant; Aileen Yayen
– eyewitness; and Dr. Rogelio Divinagracia – medico-legal. On the other hand, accused-appellant relies
on alternative defenses of alibi and consent on the part of complainant. While accusedappellant's defense
before the trial court alleges that he had left the scene of the incident together with defense witness
Calixto Flora, he alternatively raises before this Court the contention that the elements of the crime of
rape have not been established.

Accused-appellant raises the issue of the character of complainant Rowena Nantiza. It is argued that a
responsible and decent married woman, who was then three (3) months pregnant, would not be out at two
(2) o'clock in the morning getting drunk much less would a decent Filipina ask a man to accompany her to
drink beer. It is contended that complainant merely concocted the charge of rape to save her marriage
since her husband had found out that she was using drugs and drinking alcohol and even made a spectacle
of herself when she tried to seduce accused-appellant on 11 May 1994 while she was under the influence
of drug and alcohol.

ISSUE: Whether or not the accused may question his conviction by assailing the character of the victim.

RULING: It should be pointed out that the moral character of a rape victim is immaterial in the
prosecution and conviction of the accused. The Court has ruled that prostitutes can be the victims of rape.

In the present case, even if accused-appellant's allegations that the victim was drunk and under the
influence of drugs and that she (the victim) cannot be considered a decent and responsible married
woman, were true, said circumstances will not per se preclude a finding that she was raped.

The Court has repeatedly held that a medical examination of the victim is not a prerequisite in
prosecutions for rape. A person accused of rape can be convicted solely on the testimony of the victim
provided the testimony is credible, natural, convincing and otherwise consistent with human nature and
the course of things.

Remedial Law Review II (CY 2018-2019) – Saturday 1:00pm – 5:00pm: Case Digest Compilation [Atty. Brondial] Page 470
E. PRESENTATION OF EVIDENCE

E.1 PEOPLE OF THE PHILIPPINES vs. LEONARDO FABRE y VICENTE

G.R. No. 146697, July 23, 2002

FACTS: Leonardo Fabre was adjudged guilty by the RTC for raping Marilou Fabre, his own daughter.
At the trial, the prosecution presented the testimony of Marilou, that of her mother, Adela Fabre, the wife
of the accused, and that of Dr. Reinerio Jalalon, the doctor who examined Marilou, along with the
medico-legal certificate issued by Dr. Jalalon, the sworn statement of Adela, and the criminal complaint
signed by both Marilou and Adela. The defense, countered the evidence with the testimony of the accused
himself.

On appeal, the defense argues, that the testimony of appellant should acquire added strength for the
failure of the prosecution to conduct cross-examination on him and to present any rebuttal evidence.

ISSUE: Whether the trial court should give full weight as to the veracity of a testimony which was not
subjected to cross-examination by the adverse party.

RULING: The answer is in the negative. The cross-examination of a witness is a prerogative of the party
against whom the witness is called. The purpose of cross-examination is to test the truth or accuracy of
the statements of a witness made on direct examination. The party against whom the witness testifies may
deem any further examination unnecessary and instead rely on any other evidence theretofore adduced or
thereafter to be adduced or on what would be believed is the perception of the court thereon. Certainly,
the trial court is not bound to give full weight to the testimony of a witness on direct examination merely
because he is not cross-examined by the other party.allowed if the same will further the interests of
justice."

Remedial Law Review II (CY 2018-2019) – Saturday 1:00pm – 5:00pm: Case Digest Compilation [Atty. Brondial] Page 471
E.2PEOPLE OF THE PHILIPPINES vs. JESUS PEREZ y SEBUNGA

G.R. No. 142556, February 5, 2003

FACTS:This is for the automatic review of the Decision of the RTC finding Jesus S. Perez, appellant,
guilty of raping Mayia P. Ponseca and imposing death penalty upon him.

On appeal, the Appellant contends the irregularit of the identification in open court by Mayia, pointing
out that the prosecutor had already identified him as the man wearing an orange t-shirt when the
prosecutor asked Mayia to identify her alleged rapist. Appellant stresses that when Mayia identified him
in open court; she referred to him as a man named "Johnny” without giving any description or any
identifying mark. The appellant also claims that he was alone in the cell when Mayia identified him after
his arrest. Appellant bewails that the identification was not done with the usual police line-up.

ISSUE: Whether the testimony of the minor-victim should be expunged on the ground that leading
questions were asked during her testimony in court.

RULING: The answer is in the negative. As a rule, leading questions are not allowed. However, the rules
provide for exceptions when the witness is a child of tender years as it is usually difficult for such child to
state facts without prompting or suggestion. Leading questions are necessary to coax the truth out of their
reluctant lips. In the case at bar, the trial court was justified in allowing leading questions to Mayia as she
was evidently young and unlettered, making the recall of events difficult, if not uncertain.
The trend in procedural law is to give wide latitude to the courts in exercising control over the
questioning of a child witness. The reasons are spelled out in our Rule on Examination of a Child
Witness, which took effect on December 15, 2000, namely, (1) to facilitate the ascertainment of the truth,
(2) to ensure that questions are stated in a form appropriate to the developmental level of the child, (3) to
protect children from harassment or undue embarrassment, and (4) avoid waste of time. Leading
questions in all stages of examination of a child are allowed if the same will further the interests of
justice."

Remedial Law Review II (CY 2018-2019) – Saturday 1:00pm – 5:00pm: Case Digest Compilation [Atty. Brondial] Page 472
E.3 THE PEOPLE OF THE PHILIPPINES vs. JAIME CASTILLANO, SR. alias "Talino,"
RONALD CASTILLANO alias "Nono" and JAIME CASTILLANO, JR. alias "Junjun," accused,
RONALD CASTILLANO alias "Nono" and JAIME CASTILLANO, JR. alias "Junjun,"
appellants.

G.R. No. 139412, April 2, 2003; CALLEJO, SR., J.:

FACTS: Sometime in 1996, Jaime, Sr. fired his gun indiscriminately. Afraid that a stray bullet might hit
any member of his family, Diosdado accosted Jaime, Sr. and asked him to desist from firing his gun
indiscriminately. Jaime, Sr. resented the intrusion. A heated altercation ensued. Jaime, Sr. then fired his
gun towards the house of Diosdado. The incident germinated deep animosity between the two and their
respective families.

The plan of killingDiosdado by the Castillanoswas realized on that fateful night of July 8, 1996. At the
dawn of the following day, the accused attempted to escape;however, they were accosted during a check
point.

The trial court rendered a decision convicting Jaime, Jr. and Ronald of murder qualified by evident
premeditation and treachery. The trial court exonerated Jaime, Sr. of the crime on reasonable doubt. The
trial court gave no credence to Ronald’s claim that he acted in self-defense.

Appellant Jaime, Jr. avers that the prosecution failed to prove his guilt beyond reasonable doubt of the
crime charged. He asserts that the testimony of Luz Volante, the widow of Diosdado, was inconsistent
with her testimony during the preliminary examination in the municipal trial court and her sworn
statement before the police investigators.

ISSUE: Whether the credibility of the testimony of Luz, the prosecution’s principal witness, can be
impeached via her testimony during the preliminary examination before the municipal trial court or by her
sworn statement given to the police investigators even though the transcripts and sworn statement were
neither marked and offered in evidence by the appellants nor admitted in evidence by the trial court.

RULING: The Court ruled in the negative.


The Office of the Solicitor General asserts, to which the Court agrees, that the credibility of the testimony
of Luz, the prosecution’s principal witness, cannot be impeached via her testimony during the preliminary
examination before the municipal trial court nor by her sworn statement given to the police investigators
for the reason that the transcripts and sworn statement were neither marked and offered in evidence by the
appellants nor admitted in evidence by the trial court. Moreover, the appellants did not confront Luz with
her testimony during the preliminary examination and her sworn statement to the police investigators. Luz
was not, therefore, accorded a chance to explain the purported inconsistencies, as mandated by Section
13, Rule 132 of the Revised Rules of Evidence which reads:

How witness is impeached by evidence of inconsistent statement. - Before a witness can be impeached by
evidence that he has made at other times statements inconsistent with his present testimony, the
statements must be related to him, with the circumstances of the times and places and the persons present,
and he must be asked whether he made such statements, and if so, allowed to explain them. If the
statements be in writing they must be shown to the witness before any question is put to him concerning
them.

The Court agrees with the Office of the Solicitor General. Before the credibility of a witness and the
truthfulness of his testimony can be impeached by evidence consisting of his prior statements which are
inconsistent with his present testimony, the cross-examiner must lay the predicate or the foundation for
impeachment and thereby prevent an injustice to the witness being crossexamined. The witness must be
given a chance to recollect and to explain the apparent inconsistency between his two statements and state
the circumstances under which they were made.

In this case, the appellants never confronted Luz with her testimony during the preliminary examination
and her sworn statement. She was not afforded any chance to explain any discrepancies between her
present testimony and her testimony during the preliminary examination and her sworn statement. The
appellants did not even mark and offer in evidence the said transcript and sworn statement for the specific
purpose of impeaching her credibility and her present testimony. Unless so marked and offered in
evidence and accepted by the trial court, said transcript and sworn statement cannot be considered by the
court.

Remedial Law Review II (CY 2018-2019) – Saturday 1:00pm – 5:00pm: Case Digest Compilation [Atty. Brondial] Page 473
E.4.a PEOPLE VS. PLASENCIA

G.R. No. 90198. November 7, 1995; VITUG, J.:

FACTS:Antonio Plasencia, Roberto Descartin and Joelito (Julito) Descartin were accused of robbery
with homicide allegedly committed against HerminioMansueto. When arraigned, all the accused entered a
plea of “not guilty” to the charge; whereupon, trial commenced. The main defense interposed is one of
alibi. Antonio stated that he was out at sea fishing with his son. Joelito, on his part, asserted that he was in
Barrio Baod, about an hour’s walk from his residence, at the house of his fiancée. He returned to his
house, he said, only the day after. Roberto (“Ruby”), Joelito’s uncle, testified that on that fateful day, he
was in SamocoPurok 2, Iligan City, and then left for Cebu on 06 December 1984 only after receiving a
telegraph that Joelito was implicated in the crime.

RTC: did not give credence to the defense of alibi. It convicted the three accused of murder, instead of
robbery with homicide, explaining that the term “homicide” was used in the information in its generic
sense. Finding conspiracy, the trial court ruled that the killing was qualified by both treachery and abuse
of superior strength with the latter, however, being absorbed by the former.

ISSUE: Whether the use of memory aids during an examination of a witness is allowed.

RULING: The focus of this appeal is clearly one of credibility. The initial assessment on the testimony
of a witness is done by the trial court, and its findings still deserve due regard notwithstanding that the
presiding judge who pens the decision is not the one who personally may have heard the testimony. The
reliance on the transcript of stenographic notes should not, for that reason alone, render the judgment
subject to challenge. The continuity of the court and the efficacy of its decision are not affected by the
cessation from the service of the judge presiding it or by the fact that its writer merely took over from a
colleague who presided at the trial.

The use of memory aids during an examination of a witness is not altogether proscribed. Section 16, Rule
132, of the Rules of Court states:
“Sec. 16. When witness may refer to memorandum.—A witness may be allowed to refresh his
memory respecting a fact, by anything written or recorded by himself or under his direction at
the time when the fact occurred, or immediately thereafter, or at any other time when the fact
was fresh in his memory and he knew that the same was correctly written or recorded; but in
such case the writing or record must be produced and may be inspected by the adverse party,
who may, if he chooses, cross-examine the witness upon it and may read it in evidence. So, also,
a witness may testify from such a writing or record, though he retain no recollection of the
particular facts, if he is able to swear that the writing or record correctly stated the transaction
when made; but such evidence must be received with caution.” (Italics supplied.)
Allowing a witness to refer to her notes rests on the sound discretion of the trial court. In this case, the
exercise of that discretion has not been abused; the witness herself has explained that she merely wanted
to be accurate on dates and like details.

Appellants see inadvertency on Francisca’s appearing to be “jittery” on the witness stand. Nervousness
and anxiety of a witness is a natural reaction particularly in the case of those who are called to testify for
the first time. The real concern, in fact, should be when they show no such emotions. Francisca did fail in
immediately reporting the killing to the police authorities. Delay or vacillation, however, in making a
criminal accusation does not necessarily adulterate the credibility of the witness. Francisca, in her case,
has expressed fears for her life considering that the assailants, being her neighbors, could easily exact
retribution on her. Also, the hesitancy in reporting the occurrence of a crime in rural areas is not
unknown.

The alleged inconsistencies in Francisca’s testimony and in her sworn statement of 18 December 1984,
cover matters of little significance. Minor inconsistencies in the testimonies of witnesses do not detract
from their credibility; on the contrary, they serve to strengthen their credibility and are taken as badges of
truth rather than as indicia of falsehood even as they also erase suspicion of rehearsed testimony.

DOCTRINE: Evidence; Witnesses; Judgments; Judges; Transcript of Stenographic Notes; The initial
assessment on the testimony of a witness is done by the trial court, and its findings still deserve due
regard notwithstanding that the presiding judge who pens the decision is not the one who personally may
have heard the testimony; Reliance on the transcript of stenographic notes should not, for that reason

Remedial Law Review II (CY 2018-2019) – Saturday 1:00pm – 5:00pm: Case Digest Compilation [Atty. Brondial] Page 474
alone, render the judgment subject to challenge.—The focus of this appeal is clearly one of credibility.
The initial assessment on the testimony of a witness is done by the trial court, and its findings still
deserve due regard notwithstanding that the presiding judge who pens the decision is not the one who
personally may have heard the testimony. The reliance on the transcript of stenographic notes should not,
for that reason alone, render the judgment subject to challenge. The continuity of the court and the
efficacy of its decision are not affected by the cessation from the service of the judge presiding it or by the
fact that its writer merely took over from a colleague who presided at the trial.

Allowing a witness to refer to her notes rests on the sound discretion of the trial court. — The use of
memory aids during an examination of a witness is not altogether proscribed. Section 16, Rule 132, of the
Rules of Court states: “Sec. 16. When witness may refer to memorandum.—A witness may be allowed to
refresh his memory respecting a fact, by anything written or recorded by himself or under his direction at
the time when the fact occurred, or immediately thereafter, or at any other time when the fact was fresh
in his memory and he knew that the same was correctly written or recorded; but in such case the writing
or record must be produced and may be inspected by the adverse party, who may, if he chooses, cross-
examine the witness upon it and may read it in evidence. So, also, a witness may testify from such a
writing or record, though he retain no recollection of the particular facts, if he is able to swear that the
writing or record correctly stated the transaction when made; but such evidence must be received with
caution.” Allowing a witness to refer to her notes rests on the sound discretion of the trial court. In this
case, the exercise of that discretion has not been abused; the witness herself has explained that she
merely wanted to be accurate on dates and like details.

Nervousness and anxiety of a witness is a natural reaction particularly in the case of those who are
called to testify for the first time—the real concern should be when they show no such emotions.—
Appellants see inadvertency on Francisca’s appearing to be jittery” on the witness stand. Nervousness
and anxiety of a witness is a natural reaction particularly in the case of those who are called to testify for
the first time. The real concern, in fact, should be when they show no such emotions.

Delay or vacillation in making a criminal accusation does not necessarily adulterate the credibility of
witnesses.—Francisca did fail in immediately reporting the killing to the police authorities. Delay or
vacillation, however, in making a criminal accusation does not necessarily adulterate the credibility of
the witness. Francisca, in her case, has expressed fears for her life considering that the assailants, being
her neighbors, could easily exact retribution on her. Also, the hesitancy in reporting the occurrence of a
crime in rural areas is not unknown.

Minor inconsistencies in the testimonies of witnesses do not detract from their credibility—on the
contrary, they serve to strengthen their credibility and are taken as badges of truth rather than as indicia
of falsehood even as they also erase suspicion of rehearsed testimony.—The alleged inconsistencies in
Francisca’s testimony and in her sworn statement of 18 December 1984, cover matters of little
significance. Minor inconsistencies in the testimonies of witnesses do not detract from their credibility;
on the contrary, they serve to strengthen their credibility and are taken as badges of truth rather than as
indicia of falsehood even as they also erase suspicion of rehearsed testimony.

Remedial Law Review II (CY 2018-2019) – Saturday 1:00pm – 5:00pm: Case Digest Compilation [Atty. Brondial] Page 475
E.4.b Canque vs. CA

G.R. No. 96202; April 13, 1999

FACTS: Petitioner Rosella D. Canque is a contractor doing business under the name and style RDC
Construction. She entered into wo contracts with Socor Contruction as sub-contractor for her projects
with the government. On May 28, 1986, Socor sent a bill representing the balance of Canque for materials
delivered and services rendered by Socor under the two contracts.  However, Canque refused to pay,
claiming that private respondent failed to submit the delivery receipts showing the actual weight in metric
tons of the items delivered and the acceptance by the government.
 
Hence, Socor brought a suit in the RTC to recover from the Canque. During the trial, Socor presented
Dolores Aday, its bookkeeper to testify on the entries of their Book of Collectible Accounts. RTC
rendered a decision in favor of Socor. CA concurred.
Canque however, argues that the entries in Socor's Book of Collectible Accounts cannot take the place of
the delivery receipts and that such entries are mere hearsay and, thus, inadmissible. 

ISSUE: Whether or not the entries in the Book of Collectible can be admitted in court.

RULING: Yes, it can be admitted.

The admission in evidence of entries in corporate books requires the satisfaction of the following
conditions:
1. The person who made the entry must be dead, outside the country or unable to testify;
2. The entries were made at or near the time of the transactions to which they refer;
3. The entrant was in a position to know the facts stated in the entries;
4. The entries were made in his professional capacity or in the performance of a duty,
whether legal, contractual, moral or religious; and
5. The entries were made in the ordinary or regular course of business or duty. 

As petitioner points out, the business entries in question (Exh. K) do not meet the first and third
requisites. Dolores Aday, who made the entries, was presented by private respondent to testify on the
account of RDC Construction. It was in the course of her testimony that the entries were presented and
marked in evidence. There was, therefore, neither justification nor necessity for the presentation of the
entries as the person who made them was available to testify in court.

Moreover, Aday admitted that she had no personal knowledge of the facts constituting the entry. She said
she made the entries based on the bills given to her. But she has no knowledge of the truth or falsity of the
facts stated in the bills. The deliveries of the materials stated in the bills were supervised by "an engineer
for (such) functions."  The person, therefore, who has personal knowledge of the facts stated in the
entries, i.e., that such deliveries were made in the amounts and on the dates stated, was the company's
project engineer. The entries made by Aday show only that the billings had been submitted to her by the
engineer and that she faithfully recorded the amounts stared therein in the books of account. Whether or
not the bills given to Aday correctly reflected the deliveries made in the amounts and on the dates
indicated was a fact that could be established by the project engineer alone who, however, was not
presented during trial.
It is nonetheless argued by private respondent that although the entries cannot be considered an exception
to the hearsay rule, they may be admitted under Rule 132, §10 20 of the Rules of Court which provides:
Sec. 10. When witness may refer to memorandum. — A witness may be allowed to refresh
his memory respecting a fact, by anything written by himself or under his direction at the
time when the fact occurred, or immediately thereafter, or at any other time when the
fact was fresh in his memory and he knew that the same was correctly stated in the
writing; but in such case the writing must be produced and may be inspected by the
adverse party, who may, if he chooses, cross-examine the witness upon it, and may read
it in evidence. So, also, a witness may testify from such a writing, though he retain no
recollection of the particular facts, if he is able to swear that the writing correctly stated
the transaction when made; but such evidence must be received with caution.
On the other hand, petitioner contends that evidence which is inadmissible for the purpose for which it
was offered cannot be admitted for another purpose.
It should be noted, however, that Exh. K is not really being presented for another purpose. Private
respondent's counsel offered it for the purpose of showing the amount of petitioner's indebtedness.

Remedial Law Review II (CY 2018-2019) – Saturday 1:00pm – 5:00pm: Case Digest Compilation [Atty. Brondial] Page 476
This is also the purpose for which its admission is sought as a memorandum to refresh the memory of
Dolores Aday as a witness. In other words, it is the nature of the evidence that is changed, not the purpose
for which it is offered.
Be that as it may, considered as a memorandum, Exh. K does not itself constitute evidence. As explained
in Borromeo v. Court of Appeals:
Under the above provision (Rule 132, §10), the memorandum used to refresh the memory
of the witness does not constitute evidence, and may not be admitted as such, for the
simple reason that the witness has just the same to testify on the basis of refreshed
memory. In other words, where the witness has testified independently of or after his
testimony has been refreshed by a memorandum of the events in dispute, such
memorandum is not admissible as corroborative evidence. It is self-evident that a witness
may not be corroborated by any written statement prepared wholly by him. He cannot be
more credible just because he supports his open-court declaration with written
statements of the same facts even if he did prepare them during the occasion in dispute,
unless the proper predicate of his failing memory is priorly laid down. What is more,
even where this requirement has been satisfied, the express injunction of the rule itself is
that such evidence must be received with caution, if only because it is not very difficult to
conceive and fabricate evidence of this nature. This is doubly true when the witness
stands to gain materially or otherwise from the admission of such evidence . . . . 
As the entries in question (Exh. K) were not made based on personal knowledge, they could only
corroborate Dolores Aday's testimony that she made the entries as she received the bills.

Does this, therefore, mean there is no competent evidence of private respondent's claim as petitioner
argues?  The answer is in the negative. Aside from Exh. K, private respondent presented other documents:
Indeed, while petitioner had previously paid private respondent about P1,400,000.00 for deliveries made
in the past, she did not show that she made such payments only after the delivery receipts had been
presented by private respondent. On the other hand, it appears that petitioner was able to collect the full
amount of project costs from the government, so that petitioner would be unjustly enriched at the expense
of private respondent if she is not made to pay what is her just obligation the contracts.
WHEREFORE, the decision of the Court of Appeals is AFFIRMED.

Remedial Law Review II (CY 2018-2019) – Saturday 1:00pm – 5:00pm: Case Digest Compilation [Atty. Brondial] Page 477
E.5.1 YASUO IWASAWA, Petitioners, v. FELISA CUSTODIO GANGAN (A.K.A FELISA
GANGAN ARAMBULO, AND FELISA GANGAN IWASAWA) AND THE LOCAL CIVIL
REGISTRAR OF PASAY CITY, Respondent.

G.R. No. 204169; September 11, 2013; VILLARAMA, JR., J.:

FACTS: Yasuo Iwasawa (Petitioner) is a Japanese national who married Felisa Custodia Gangan (Private
Respondent). Private Respondent introduced herself as “single” and “has never married before”.
However, Private Respondent confessed to Petitioner that the former is previously married to one
Raymond Maglonzo Arambulo and that Private Respondent’s previous husband had passed away. This
prompted Petitioner to file a petition with the Regional Trial Court for declaration of his marriage
to Private Respondent as null and void on the ground that their marriage is a bigamous one.

During the trial, Petitioner offered the following pieces of documentary evidence issued by the National
Statistics Office (NSO):

(1) Certificate of Marriage between Petitioner and Private Respondent

(2) Certificate of Marriage between Private Respondent and Raymond Maglonzo Arambulo

(3) Certificate of Death of Raymond Maglonzo Arambulo

(4) Certification from the NSO to the effect that there are two entries of marriage recorded by the
office pertaining to Private Respondent to prove that Private Respondent in fact contracted two marriages,
the first one was to a Raymond Maglonzo Arambulo on June 20, 1994, and second, to Petitioner on
November 28, 2002.”

The prosecutor appearing on behalf of the Office of the Solicitor General (OSG) admitted the authenticity
and due execution of the above documentary exhibits during pre-trial.

The RTC rendered the assailed decision. It ruled that there was insufficient evidence to prove Private
Respondent’s prior existing valid marriage to another man. It held that while Petitioner offered the
certificate of marriage of Private Respondent to Arambulo, it was only Petitioner who testified about said
marriage. The RTC ruled that Petitioner’s testimony is unreliable because he has no personal knowledge
of Private Respondent’s prior marriage nor of Arambulo’s death which makes him a complete stranger to
the marriage certificate between Private Respondent and Arambulo and the latter’s death certificate. It
further ruled that Petitioner’s testimony about the NSO certification is likewise unreliable since he is a
stranger to the preparation of said document. Petitioner filed a motion for reconsideration, but the
same was denied by the RTC.

ISSUE: Whether or not the testimony of the NSO records custodian certifying the authenticity and due
execution of the public documents issued by said office was necessary before they could be accorded
evidentiary weight?

RULING: NO. There is no question that the documentary evidence submitted by petitioner are all public
documents.

As public documents, they are admissible in evidence even without further proof of their due execution
and genuineness.15 Thus, the RTC erred when it disregarded said documents on the sole ground that the
petitioner did not present the records custodian of the NSO who issued them to testify on their
authenticity and due execution since proof of authenticity and due execution was not anymore necessary.
Moreover, not only are said documents admissible, they deserve to be given evidentiary weight because
they constitute prima facie evidence of the facts stated therein. And in the instant case, the facts stated
therein remain unrebutted since neither the private respondent nor the public prosecutor presented
evidence to the contrary.

DOCTRINE: Public Documents are admissible in evidence even without further proof of their due
execution and genuiness.

Remedial Law Review II (CY 2018-2019) – Saturday 1:00pm – 5:00pm: Case Digest Compilation [Atty. Brondial] Page 478
E.5.2 Asian Terminals vs. Philam Insurance
Asian Terminals (ATI) vsPhilam Insurance (now Chartis Philippines Insurance, Inc.), 702 SCRA,
GR 181163, July 24, 2013
VILLARAMA, JR., J.:

FACTS: ASIAN TERMINALS, INC., Petitioner, vs. PHILAM INSURANCE CO., INC. (now Chartis
Philippines Insurance, Inc.), Respondent. G.R. No. 181163 July 24, 2013
Facts

Nichimen Corporation shipped to Universal Motors Corporation (Universal Motors) 219 packages
containing 120 units of brand new Nissan Pickup Truck Double Cab 4x2 model, without engine, tires and
batteries, on board the vessel S/S "Calayan Iris" from Japan to Manila. The shipment, which had a
declared value of US$81,368 or P29,400,000, was insured with Philam against all risks under Marine
Policy No. 708-8006717-4. The carrying vessel arrived at the port of Manila on April 20, 1995, and when
the shipment was unloaded by the staff of ATI, it was found that the package marked as 03-245-42K/1
was in bad order. The Turn Over Survey of Bad Order Cargoes dated April 21, 1995 identified two
packages, labeled 03-245-42K/1 and 03/237/7CK/2, as being dented and broken. Universal Motors filed a
formal claim for damages in the amount of P643,963.84 against Westwind, ATI and R.F. Revilla
Customs Brokerage, Inc. When Universal Motors’ demands remained unheeded, it sought reparation from
and was compensated in the sum of P633,957.15 by Philam. Accordingly, Universal Motors issued a
Subrogation

Receipt dated November 15, 1995 in favor of Philam. On January 18, 1996, Philam, as subrogee of
Universal Motors, filed a Complaint for damages against Westwind, ATI and R.F. Revilla Customs
Brokerage, Inc. before the RTC of Makati City, Branch 148. On September 24, 1999, the RTC rendered
judgment in favor of Philam and ordered Westwind and ATI to pay Philam, jointly and severally, the sum
of P633,957.15 with interest at the rate of 12% per annum, P158,989.28 by way of attorney’s fees and
expenses of litigation. On appeal, the CA affirmed with modification the ruling of the RTC. All the
parties moved for reconsideration, but their motions were denied in a Resolution dated January 11, 2008.
Thus, they each filed a petition for review on certiorari which were consolidated together by this Court
considering that all three petitions assail the same CA decision and resolution and involve the same
parties

ISSUE: Whether the Marine Certificate No. 708-8006717-4 and the Subrogation Receipt are private
documents?

RULING: YES. The nature of documents as either public or private determines how the documents may
be presented as evidence in court. Public documents, as enumerated under Section 19, Rule 132 of the
Rules of Court, are self-authenticating and require no further authentication in order to be presented as
evidence in court. In contrast, a private document is any other writing, deed or instrument executed by a
private person without the intervention of a notary or other person legally authorized by which some
disposition or agreement is proved or set forth. Lacking the official or sovereign character of a public
document, or the solemnities prescribed by law, a private document requires authentication in the manner
prescribed under Section 20, Rule 132 of the Rules: SEC. 20. Proof of private document. – Before any
private document offered as authentic is received in evidence, its due execution and authenticity must be
proved either: (a) By anyone who saw the document executed or written; or (b) By evidence of the
genuineness of the signature or handwriting of the maker. Any other private document need only be
identified as that which it is claimed to be. The requirement of authentication of a private document is
excused only in four instances, specifically: (a) when the document is an ancient one within the context of
Section 21, Rule 132 of the Rules; (b) when the genuineness and authenticity of the actionable document
have not been specifically denied under oath by the adverse party; (c) when the genuineness and
authenticity of the document have been admitted; or (d) when the document is not being offered as
genuine. Indubitably, Marine Certificate No. 708-8006717-4 and the Subrogation Receipt are private
documents which Philam and the consignee, respectively, issue in the pursuit of their business. Since
none of the exceptions to the requirement of authentication of a private document obtains in these cases,
said documents may not be admitted in evidence for Philam without being properly authenticated.

Remedial Law Review II (CY 2018-2019) – Saturday 1:00pm – 5:00pm: Case Digest Compilation [Atty. Brondial] Page 479
E.6.1 Aludos vs. Suerte

673 SCRA , GR No 165285 June 2012

FACTS:
 Sometime in January 1969, AludosAludos acquired from the Baguio City Government the right
to occupy two stalls in the Hangar Market in Baguio City, as evidenced by a permit issued by the
City Treasurer.
 On September 8, 1984, Aludos entered into an agreement with respondent Johnny M. Suerte for
the transfer of all improvements and rights over the two market stalls (Stall Nos. 9 and 10) for the
amount of ₱260,000.00. Suerte gave a down payment of ₱45,000.00 to Aludos, who
acknowledged receipt of the amount in a document 5 executed on the same date as the agreement
 Suerte made a subsequent payment of ₱23,000.00; hence, a total of ₱68,000.00 of the
₱260,000.00 purchase price had been made as of 1984. Before full payment could be made,
however, Aludos backed out of the agreement and returned the ₱68,000.00 to Domes and Jaime
Suerte, the mother and the father of Johnny, respectively. The return of the ₱68,000.00 down
payment was embodied in a handwritten receiptdated October 9, 1985.
  Johnny filed a complaint against Aludos before the Regional Trial Court (RTC), Branch 7,
Baguio City, for specific performance with damages, docketed as Civil Case No. 720-R. Johnny
prayed that, after due proceedings, judgment be rendered ordering Aludos to (1) accept the
payment of the balance of ₱192,000.00; and (2) execute a final deed of sale and/or transfer the
improvements and rights over the two market stalls in his favor.
 The RTC found that Aludos was a mere lessee of the market stalls, and the Baguio City
Government was the owner-lessor of the stalls. Under Article 1649 of the Civil Code, "[t]he
lessee cannot assign the lease without the consent of the lessor, unless there is a stipulation to the
contrary." As the permit issued to Aludos did not contain any provision that the lease of the
market stalls could further be assigned, and in the absence of the consent of the Baguio City
Government to the agreement, the RTC declared the agreement between Aludos and Johnny null
and void
 The CA found that there were two agreements entered into between Johnny and Lomises: one
was for the assignment of leasehold rights and the other was for the sale of the improvements on
the market stalls. The CA agreed with the RTC that the assignment of the leasehold rights was
void for lack of consent of the lessor, the Baguio City Government. The sale of the
improvements, however, was valid because these were Lomises’ private properties. For this
reason, the CA remanded the case to the RTC to determine the value of the improvements on the
two market stalls, existing at the time of the execution of the agreement.
 Lomises moved for the reconsideration of the CA ruling, contending that no valid sale of the
improvements could be made because the lease contract, dated May 1, 1985, between Lomises
and the Baguio City Government, supposedly marked as Exh. "A," provided that "[a]ll
improvements [introduced shall] ipso facto become properties of the City of Baguio."

ISSUE:
Whether the lease contract between Baguio City Government and Aludos is admissible in court

RULING:
No
Under Section 34, Rule 132 of the Rules of Court, the court shall consider no evidence which has not
been formally offered. "The offer of evidence is necessary because it is the duty of the court to rest its
findings of fact and its judgment only and strictly upon the evidence offered by the parties. Unless and
until admitted by the court in evidence for the purpose or purposes for which such document is offered,
the same is merely a scrap of paper barren of probative weight.
Unless and until admitted by the court in evidence for the purpose or purposes for which such document
is offered, the same is merely a scrap of paper barren of probative weight."
The CA has already rejected the evidentiary value of the May 1, 1985 lease contract between the Baguio
City Government and Aludos, as it was not formally offered in evidence before the RTC; in fact, the CA
admonished Aludos’ lawyer, Atty. Lockey, for making it appear that it was part of the records of the case.
Although the contract was referred to in Aludos’ answer to Johnny’s complaint  and marked as Exhibit "2"
in his pre-trial brief, a copy of it was never attached. In fact, a copy of the May 1, 1985 lease contract
"surfaced" only after Aludos filed a motion for reconsideration of the CA decision. What was formally
offered was the 1969 permit, which only stated that Aludos was permitted to occupy a stall in the Baguio
City market and nothing else.In other words, no evidence was presented and formally offered showing
that any and all improvements in the market stalls shall be owned by the Baguio City Government.

Remedial Law Review II (CY 2018-2019) – Saturday 1:00pm – 5:00pm: Case Digest Compilation [Atty. Brondial] Page 480
Upon determination by the RTC of the actual value of the improvements on the market stalls, the heirs of
Johnny Suerte should pay the ascertained value of these improvements to Aludos, who shall thereafter be
required to execute the deed of sale over the improvements in favor of the heirs of Johnny.
WHEREFORE, under these premises, the Court hereby AFFIRMS the ruling of the Court of Appeals for
the remand of the case to the Regional Trial Court of Baguio City, Branch 7, for the determination of the
value of the improvements on Stall Nos. 9 and 10 at the Refreshment Section of the Hangar Market
Compound, Baguio City as of September 8, 1984. After this determination, the Court ORDERS the heirs
of Johnny M. Suerte to pay the amount determined to the heirs of AludosAludos, who shall thereafter
execute the deed of sale covering the improvements in favor of the heirs of Johnny M. Suerte and deliver
the deed to them. Costs against the petitioner.

Remedial Law Review II (CY 2018-2019) – Saturday 1:00pm – 5:00pm: Case Digest Compilation [Atty. Brondial] Page 481
E.6.2 Westmont Investment Corporation vs. Francia, Jr. WESTMONT INVESTMENT
CORPORATION vs. AMOS P. FRANCIA, JR. et al.

G.R. No. 194128; December 7, 2011

FACTS: Respondents filed a Complaint for Collection of Sum of Money and Damages arising from their
investments against petitioner Westmont Investment Corporation (Wincorp) and respondent Pearlbank
Securities Inc. the case was set for the presentation of the defense evidence of Wincorp. On March 7,
2003, three (3) days before the scheduled hearing, Wincorp filed a written motion to postpone the
hearing. The RTC denied Wincorp’s Motionto Postpone and considered it to have waived its right to
present evidence. The Motion for Reconsideration of Wincorp was likewise denied. On September 27,
2004, the RTC rendered a decision in favor of the Francias and held Wincorp solely liable to them. The
CA affirmed with modification the ruling of the RTC. Wincorp filed an MR with the CA attaching to the
said motion evidentiary evidence which it was not able to present during trial.

ISSUE: Whether or not documents attached to a motion for reconsideration made before the appellate
court may be considered for purposes of adjudicating the merits of the case.

RULING: The answer is in the negative. Section 34, Rule 132 of the Rules on Evidence states that: "The
court shall consider no evidence which has not been formally offered. The purpose for which the evidence
is offered must be specified." A formal offer is necessary because judges are mandated to rest their
findings of facts and their judgment only and strictly upon the evidence offered by the parties at the trial.
Its function is to enable the trial judge to know the purpose or purposes for which the proponent is
presenting the evidence. On the other hand, this allows opposing parties to examine the evidence and
object to its admissibility. Moreover, it facilitates review as the appellate court will not be required to
review documents not previously scrutinized by the trial court. Evidence not formally offered during the
trial cannot be used for or against a party litigant. Neither may it be taken into account on appeal. The rule
on formal offer of evidence is not a trivial matter. Failure to make a formal offer within a considerable
period of time shall be deemed a waiver to submit it. Consequently, any evidence that has not been
offered shall be excluded and rejected. Prescinding therefrom, the very glaring conclusion is that all the
documents attached in the motion for reconsideration of the decision of the trial court and all the
documents attached in the defendant-appellant’s brief filed by defendantappellant Wincorp cannot be
given any probative weight or credit for the sole reason that the said documents were not formally offered
as evidence in the trial court because to consider them at this stage will deny the other parties the right to
rebut them.

DOCTRINE:A formal offer is necessary because judges are mandated to rest their findings of facts and
their judgment only and strictly upon the evidence offered by the parties at the trial. Its function is to
enable the trial judge to know the purpose or purposes for which the proponent is presenting the
evidence. On the other hand, this allows opposing parties to examine the evidence and object to its
admissibility

Remedial Law Review II (CY 2018-2019) – Saturday 1:00pm – 5:00pm: Case Digest Compilation [Atty. Brondial] Page 482
E.7 FORTUNE TOBACCO CORPORATION v. Com of Int. Rev. (Tender of Excluded Evidence)

GR No. 192024; Jul 01, 2015

FACTS: Fortune Tobacco Corporation is the manufacturer and producer of various cigarette brands
which its tax duties on this products was put in to question.

Since 1977, cigarettes were subject to ad valorem tax pursuant to our old Tax Code. When R.A. 8240
took effect on 1997, the ad valorem tax AVT system shifted to the specifc tax system. To implement the
provisions for a twelve percent (12%) increase of excise tax on cigarettes packed by machines by January
1, 2000, the Finance Secretary, upon recommendation of the CIR issued Revenue Regulations No. 17-99,
dated 5ecember 16, 1999.

On March 2005, Fortune Tobacco filed a claim for tax credit and refund for erroneously or illegally
collected speci0c taxes covering the period June to December 31, 2004 in the total amount of Php219,
566450. The CIR in his answer raised an affirmative defense, that the said amount being claimed as
alleged overpaid excise tax for the period covering 1 June to 31 December 2004 is not properly
documented.

On April 2009, the Former First Division of the CTA ruled that RR 17-99 is contrary to law, but there is
insufficiency of evidence on the claim for refund. Fortune Tobacco elevated its claim to the CTA
En Banc, found no cause even the Court should review Fortune Tobacco’s evidence, the result would be
the same.

ISSUE: Whether the claim for refund by Fortune Tobacco, rellying on the admissibility and the probative
value of photocopied documents that allegedly contain a recording of petitioner’s excise payments for the
period covering 1 June to 31 December 2004.

RULING: Although both the CTA Division and the CTA En Banc provisionally admitted petitioner's
Exhibit "C," the above-mentioned documents, as well as the other documentary evidence submitted by
petitioner were refused admission for being merely photocopies.

Section 3 of Administrative Matter (A.M.) No. 05-11-07 CTA, the Revised Rules of the Court of Tax
Appeals, provides that the Rules of Court shall apply suppletorily in the proceeding before the tax
tribunal. In this connection, Section 3 of Rule 130 of the Rules of Court, which lays down the Best
Evidence Rule with respect to the presentation of documentary evidence, will apply.

In addition, Fortune Tobacco failed not only to comply with the basic procedural requirement of
presenting only the original copies of its documentary evidence, but also to adhere to the requirement to
properly make its offer of proof or tender of excluded evidence for the proper consideration of the
appellate tribunal.

Section 40, Rule 132 of the Rules of Court provides:


Sec. 40. Tender of excluded evidence. - If documents or things offered in evidence are excluded by the
court, the offeror may have the same attached to or made part of the record. If the evidence excluded is
oral, the offeror may state for the record the name and other personal circumstances of the witness and the
substance of the proposed testimony.

The rule is that evidence formally offered by a party may be admitted or excluded by the court. If a party's
offered documentary or object evidence is excluded, he may move or request that it be attached to form
part of the records of the case. If the excluded evidence is oral, he may state for the record the name and
other personal circumstances of the witness and the substance of the proposed testimony. These
procedures are known as offer of proof or tender of excluded evidence and are made for purposes of
appeal. If an adverse judgment is eventually rendered against the offeror, he may in his appeal assign as
error the rejection of the excluded evidence.

It is of record that the denial of the excluded evidence was never assigned as an error in this appeal. Thus,
this Court cannot pass upon nor consider the propriety of their denial. Moreover, this Court cannot and
should not consider the documentary and oral evidence presented which are not considered to be part of
the records in the first place.

Remedial Law Review II (CY 2018-2019) – Saturday 1:00pm – 5:00pm: Case Digest Compilation [Atty. Brondial] Page 483
It has been repeatedly ruled that where documentary evidence was rejected by the lower court and the
offeror did not move that the same be attached to the record, the same cannot be considered by the
appellate court,[16] as documents forming no part of proofs before the appellate court cannot be
considered in disposing the case.[17] For the appellate court to consider as evidence, which was not
offered by one party at all during the proceedings below, would infringe the constitutional right of the
adverse party - in this case, the CIR, to due process of law.

It also bears pointing out that at no point during the proceedings before the CTA En Banc and before this
Court has petitioner offered any plausible explanation as to why it failed to properly make an offer of
proof or tender of excluded evidence. Instead, petitioner harps on the fact that respondent CIR simply
refused its claim for refund on the ground that RR 17-99 was a valid issuance. Thus, for its failure to
seasonably avail of the proper remedy provided under Section 40, Rule 132 of the Rules of Court,
petitioner is precluded from doing so at this late stage of the case. Clearly, estoppel has already stepped
in.

Remedial Law Review II (CY 2018-2019) – Saturday 1:00pm – 5:00pm: Case Digest Compilation [Atty. Brondial] Page 484
F. WEIGHT AND SUFFICIENCY OF EVIDENCE

F.1.ii.1 People of the Philippines, Plaintiff-Appellee, Vs. DelfinCaliso, Accused-Appellant

G.R. No. 183830; October 19, 2011

Ponente: Bersamin,J. (First Division)

~ In every criminal prosecution, the identity of the offender, like the crime itself, must be established by
proof beyond reasonable doubt. Indeed, the first duty of the Prosecution is not to prove the crime but to
prove the identity of the criminal, for even if the commission of the crime can be established, there can be
no conviction without proof of identity of the criminal beyond reasonable doubt.

The identification of a malefactor, to be positive and sufficient for conviction, does not always require
direct evidence from an eyewitness; otherwise, no conviction will be possible in crimes where there are
no eyewitnesses. […] trustworthy circumstantial evidence can equally confirm the identification and
overcome the constitutionally presumed innocence of the accused. […] the Court has distinguished two
types of positive identification:

a. that by direct evidence, through an eyewitness to the very commission of the act; and

b. that by circumstantial evidence, such as where the accused is last seen with the victim
immediately before or after the crime.

FACTS:On June 5, 1997 AAA died, the immediate cause of which was asphyxia, secondary to drowning
due to smothering. A lone eyewitness, 34-year old Amegable, had been clearing her farm when she heard
the anguished cries of a girl pleading for mercy. The cries came from an area with lush bamboo growth
that made it difficult for Amegable to see what was going on, Amegable then proceeded to get a better
glimpse of what was happening, hiding behind a cluster of banana trees in order not to be seen, and from
there she saw a man wearing gray short pants bearing the number "11" mark, who dragged a girl's limp
body into the river, where he submerged the girl into the knee-high muddy water and stood over her body.
In that in that whole time, Amegable could not have a look at the person’s face because he always had his
back turned towards her. Amegable nonetheless insisted that the man was Caliso, whose physical features
she was familiar with due to having seen him pass by their barangay several times prior to the incident.

Hence, accused-appellant was charged and tried for rape and homicide, but the RTC found him guilty of
murder for the killing of AAA, a mentally-retarded 16-year old girl, and sentenced Accused-Appellant to
death.

On intermediate review, the CA affirmed Caliso's conviction for murder based on the same ratiocinations
the RTC had rendered.

ISSUE: WON Amegable's identification of Caliso as the man who killed AAA was positive and reliable.

RULING: NO.
The identification of a malefactor, to be positive and sufficient for conviction, does not always require
direct evidence from an eyewitness; otherwise, no conviction will be possible in crimes where there are
no eyewitnesses. […] trustworthy circumstantial evidence can equally confirm the identification and
overcome the constitutionally presumed innocence of the accused. […] the Court has distinguished two
types of positive identification:

a. that by direct evidence, through an eyewitness to the very commission of the act; and

b. that by circumstantial evidence, such as where the accused is last seen with the victim
immediately before or after the crime.

The test to determine the moral certainty of an identification is itsimperviousness to skepticism on


account of its distinctiveness. To achieve suchdistinctiveness, the identification evidence should
encompass unique physical featuresor characteristics, like the face, the voice, the dentures, the
distinguishing marks ortattoos on the body, fingerprints, DNA, or any other physical facts that set
theindividual apart from the rest of humanity.

Remedial Law Review II (CY 2018-2019) – Saturday 1:00pm – 5:00pm: Case Digest Compilation [Atty. Brondial] Page 485
A witness' familiarity with the accused, although accepted as basis for a positiveidentification, does not
always pass the test of moral certainty due to the possibility of mistake.

No matter how honest Amegable's testimony might have been, her identification ofCaliso by a sheer look
at his back for a few minutes could not be regarded as positiveenough to generate that moral certainty
about Caliso being the perpetrator of thekilling, absent other reliable circumstances showing him to be
AAA's killer.Heridentification of him in that manner lacked the qualities of exclusivity and
uniqueness,even as it did not rule out her being mistaken.

In the absence of proof beyond reasonable doubt as to the identity of the culprit, theaccused's
constitutional right to be presumed innocent until the contrary is proved isnot overcome, and he is entitled
to an acquittal, though his innocence may be doubted. The constitutional presumption of innocence
guaranteed to every individual is of primary importance, and the conviction of the accused must rest not
onthe weakness of the defense he put up but on the strength of the evidence for theProsecution.

Remedial Law Review II (CY 2018-2019) – Saturday 1:00pm – 5:00pm: Case Digest Compilation [Atty. Brondial] Page 486
F.1.ii.2 PEOPLE OF THE PHILIPPINE vs. FELIMON PATENTES y ZAMORA

G.R. No. 190178, February 12, 2014

FACTS: Accused was charged with forcible abduction with rape. During the trial, complainant testified
that she was abducted by the accused and brought to the latter’s residence where the former was
repeatedly rape for 8 days. However, in the course the complainant’s testimony, she testified that in
between those 8 days, she was able to visit her grandmother.

Furthermore, when prosecution presented Dr. Cruz testified that he examined AAA. In his report, he
noted the following observations about AAA: (1) contusion on the breast caused by a kiss mark; (2)
hymen was intact and can readily admit a normal-sized erect male penis without sustaining any injury;
and (3) vaginal canal was negative for spermatozoa. Dr. Cruz also added that he cannot tell whether it
was AAA’s first sexual intercourse as the vagina was not injured but had healed lacerations.

On the other hand, the defense presented Wilma Enriquez, a friend of the complainant, who testified that
after the dates wherein the latter was supposedly rape, the latter visited her and discussed plans about
marrying the accused. Trial court convicted the accused. On appeal with CA, conviction of the accused
was confirmed.

ISSUE: Whether or not a person accused of rape may be convicted based solely on the testimony of
victim who positively identified him to be the perpetrator notwithstanding blatant inconsistencies therein.

RULING:
In reviewing rape cases, the Court is guided by the following principles: (1) to accuse a man of rape is
easy, but to disprove the accusation is difficult, though the accused may be innocent; (2) inasmuch as only
two persons are usually involved in the crime of rape, the testimony of the complainant should be
scrutinized with great caution; and (3) the evidence for the prosecution muststand or fall on its own merit
and should not be allowed to draw strength fromthe weakness of the evidence for the defense. So long as
the private complainant’s testimony meets the test of credibility, the accused may be
convicted on the basis thereof. The time-honored test in determining the value of the testimony of a
witness is its compatibility with human knowledge, observation and common experience of man. Thus,
whatever is repugnant to the standards of human knowledge, observation and experience becomes
incredible and must lie outside judicial cognizance.

The following inconsistencies in complainant’s testimony is found to be incompatible to human


experience: (1) the admission that she during her abduction she was brought to accused’s house where 8
family members of the latter also resides; (2) that she was not able to ask for help from any of the family
member nor any of them was able to realize that accused was keeping her against her will; (3) the
discussion of wedding plans with her friend after her ordeal; (4) admission that she was able to visit her
grandmother within the period of the alleged abduction when she was supposed to be kept inside
accused’s house against her will; and (5) lastly, that she was repeated mauled by accused on their way to
his to residence and within the duration of abduction but no physical injuries was seen during the medical
examination conducted after the incident. A conviction in a criminal case must be supported by proof
beyond reasonable doubt, which means a moral certainty that the accused is guilty; the burden of proof
rests upon the prosecution. In the case at bar, the prosecution has failed to discharge its burden of
establishing with moral certainty the truthfulness of the charge that appellant had carnal knowledge of
AAA against her will using threats, force or intimidation.

Remedial Law Review II (CY 2018-2019) – Saturday 1:00pm – 5:00pm: Case Digest Compilation [Atty. Brondial] Page 487
F.1.iii.1THE PEOPLE OF THE PHILIPPINES, plaintiff-appellee, vs. ROSENDO DELGADO
ALIAS "NONGNONG", defendant-appellant.The Office of the Solicitor General for plaintiff-
appellee.Anecio R. Guades for defendant-appellant.

G.R. No. 79672; February 15, 1990

FACTS:Rogelio Zamoras, 15 years old, testified that on August 9, 1984 about 4:30 p.m. while he was
playing with some other children near the place of Conchita Zamora's in Brgy. Pilar, he noticed that his
father, Santos Zamoras, who was walking in the street towards the reading center, was boxed by
Clemente Zamoras, Hitting the lower part of the right ear. His father retaliated by stabbing Clemente
Zamoras with a knife hitting the left breast. That after stabbing Clemente Zamoras, his father Santos
Zamoras sat on the cemented pavement placing his right hand over his left and put his head on top of it;
that believing his father was asleep and nothing more would happen, he returned to play with his
playmates; that shortly thereafter, one of his playmates told him that his father Santos was stabbed and
when he looked to the direction of his father he saw accused Nongnong (Rosendo Delgado) stabbed (sic)
his father Santos hitting the lower part of the back of the body on the right side, then the lower part of the
neck and on the chest, using a bolo, 30 inches long. Thereafter, his father died falling to the ground face
down. At that time he was more than two (2) arms lengths from his father. To help his father he picked up
a stone and hurled it at Rosendo Delgado but was (sic) not hit. That then he ran to his mother in the house
of his grandmother, about 250 meters away, telling her that father is dead killed by Rosendo Delgado.
When he ran, the accused was near about half meter to his father who was lying face down and even
stretched his father. Thereafter, he saw Rosendo Delgado and the brother walking towards the direction of
Barangay Cabatuan. That her (sic) mother, when informed, went to Brgy. Councilor Jose Sagadai and a
member of Civilian Home Defense Force (CHDF) and with them took the body of Santos Zamoras from
the place of incident and brought to the house of his grandmother. 

ISSUE: Whether or not there is a valid self defense.

RULING:No. There is no self-defense to speak of as it was not proven that there was unlawful
aggression on the part of the victim. There being no unlawful aggression, there is nothing to prevent or
repel.A person who seeks justification for his act must prove by clear and convincing evidence the
presence of the necessary justifying circumstance for having admitted wounding or killing his adversary,
and he is criminally liable unless he is able to satisfy the Court that he acted in legitimate self-defense.
For there to be legitimate self-defense, there must be three requisites as provided in paragraph 1 of Article
11 of the Revised Penal Code, namely: (1) unlawful aggression (2) reasonable necessity of the means
employed to prevent or repel it; and (3) lack of sufficient provocation on the part of the person defending
himself. (People v. Batas, supra)In the case at bar, the stabbing was from behind, done in a sudden and
unexpected manner while the deceased was sitting and his head down on his hands.The appellant
contends that the mitigating circumstances of incomplete self-defense and voluntary surrender should be
appreciated.

Remedial Law Review II (CY 2018-2019) – Saturday 1:00pm – 5:00pm: Case Digest Compilation [Atty. Brondial] Page 488
F.1.iii.2Govt. of Hongkong Special Adm. Region vs. Olalia, Jr.

PEOPLEOFTHEPHILIPPINESVS.ALFONSOFONTANILLA Y OBALDO

FACTS:AlfonsoFontanillawaschargedofmurderforallegedlystrucking Oliasin theheadwithapiece


ofwoodcalledBellangcausingthelatterfellfacedown tothe ground,butFontanilla
hithimagainintheheadwithapiece ofstone. Asa result, thevictimdied.Fontanillaclaimedself-defense
allegingthatonthenightofthe incident, hehadbeenstanding ontheroadnearhishousewhenOlais,wielding a
nightstickandappearingtobedrunk,hadboxedhiminthestomach;thatalthoughhehadthentalkedtoOlaisnicely,t
helatterhadcontinued hittinghimwithhis fists,strikinghimwithtraightblows;thatOlais,akarateexpert,had
alsokicked himwithbothhislegs;thathehadthusbeenforcedtodefend himselfbypicking upa
stonewhichhehadhitthevictim’shead. TheRTCinrejecting Fontanilla’s pleaofself-defense
notedthathedidnotsuffer anyinjurydespitehisclaim that the victimhadmauledhim;that Fontanilladid not
receiveanytreatment, andnomedicalcertificate attestedtoanyinjuryhemight
havesuffered,havingbeenimmediately releasedfromthehospital;that Fontanilla’s
failuretogiveanystatementatthetimehesurrenderedtothepolice wasinconsistentwithhisplea ofself-defense.

ISSUE:Whetheror notself-defense was proven in this case.

RULING: No. Inorderforself-defense tobeappreciated,hehadtoprovebyclearand convincing


evidencethefollowingelements: (a)unlawfulaggression onthepart of thevictim;(b)reasonable necessity
ofthemeansemployed topreventor repelit; and(c)lackofsufficientprovocationonthepartoftheperson
defendinghimself. Byinvokingself-defense, however,Fontanillaadmittedinflictingthefatalinjuries
thatcaused the death ofOlais.Itisbasicthatonce anaccusedinaprosecutionfor murderor
homicideadmittedhis inflictionof the fatalinjuriesonthedeceased,he
assumedtheburdentoprovebyclear,satisfactory andconvincing evidence the justifyingcircumstancethat
wouldavoid his criminalliability.Havingthusadmitted beingtheauthorofthedeath ofthevictim,Fontanilla
cametobeartheburdenof provingthejustifyingcircumstancetothesatisfactionofthecourt,andhewould
beheldcriminallyliableunlessheestablishedself-defense bysufficientand satisfactoryproof.He
shoulddischargetheburdenbyrelyingonthestrengthofhis ownevidence,becausetheProsecution’s
evidence,evenifweak,wouldnotbe disbelievedinview ofhisadmissionofthekilling.Nonetheless,theburdento
proveguiltbeyond reasonabledoubtremainedwiththeStateuntiltheendofthe proceedings. Fontanilla
didnotdischargehisburden.Areviewoftherecords reveals that,one, Olaisdidnotcommitunlawful aggression
againstFontanilla,and,two,Fontanilla’s actofhittingthevictim’s
headwithastone,causingthemortalinjury,wasnot proportionalto,andconstitutedanunreasonable
responsetothevictim’sfistic attackandkicks.

DOCTRINE:Clear and convincing evidence is required to prove self defense.

Inorderforself-defense tobeappreciated,hehadtoprovebyclearnd convincing


evidencetheollowingelements: (a)unlawfulaggression onthepart of thevictim;(b)reasonable necessity
ofthemeansemployed topreventor repelit; and(c)lackofsufficientprovocationonthepartoftheperson
defendinghimself.

Remedial Law Review II (CY 2018-2019) – Saturday 1:00pm – 5:00pm: Case Digest Compilation [Atty. Brondial] Page 489
F.1.iv.1 PHILIPPINE COMMERCIAL INTERNATIONAL BANK vs.
ANTONIO B. BALMACEDA and ROLANDO N. RAMOS

G.R. No. 158143, September 21, 2011

Topic: Preponderance of Evidence

FACTS: PCIB filed an action for recovery of sum of money with damages against Antonio Balmaceda.
PCIB alleged that Balmaceda, by taking advantage of his position as branch manager, fraudulently
obtained and encashed 34 Manager’s checks amounting to 11M+. PCIB also impleaded Ramos as one of
the recipients of a portion of the proceeds from Balmaceda’s alleged fraud. For failure to file an answer,
Balmaceda was declared in default. On the other hand, Ramos filed an Answer denying any knowledge of
Balmaceda’s scheme. Ramos admitted receiving money from Balmaceda as payment for the fighting
cocks that he sold to Balmaceda, but maintained that he had no knowledge of the source of Balmaceda’s
money.

RTC issued a decision in favor of PCIB. On appeal, the CA dismissed the complaint against Ramos,
holding that no sufficient evidence existed to prove that Ramos colluded with Balmaceda in the latter’s
fraudulent manipulations. According to the CA, the mere fact that Balmaceda made Ramos the payee in
some of the Manager’s checks does not suffice to prove that Ramos was complicit in Balmaceda’s
fraudulent scheme. It was also observed that other persons were also named as payees in the checks that
Balmaceda acquired and encashed, and PCIB only chose to go after Ramos. With PCIB’s failure to prove
Ramos’ actual participation in Balmaceda’s fraud, no legal and factual basis exists to hold him liable.

PCIB insists that since Ramos’ defense – anchored on mere denial of any participation in Balmaceda’s
wrongdoing – is an intrinsically weak defense, it was error for the CA to exonerate Ramos from any
liability.

ISSUE: Whether or not appellate court may exonerate defendant on the basis of his mere denial of the
imputed wrong in a civil case.

RULING: The answer is in the affirmative. While mere denial by the defendant is intrinsically weak
evidence, this does not mean that the complainant may just rely on the weakness of the defense presented
by the former. The rule is that the party asserting a claim must still provide proof of his allegation. In civil
cases, the party carrying the burden of proof must establish his case by a preponderance of evidence, or
evidence which, to the court, is more worthy of belief than the evidence offered in opposition.

On its face, all that PCIB’s evidence proves is that Balmaceda used Ramos’ name as a payee when he
filled up the application forms for the Manager’s checks. But, as the CA correctly observed, the mere fact
that Balmaceda made Ramos the payee on some of the Manager’s checks is not enough basis to conclude
that Ramos was complicit in Balmaceda’s fraud; a number of other people were made payees on the other
Manager’s checks yet PCIB never alleged them to be liable, nor did the Bank adduce any other evidence
pointing to Ramos’ participation that would justify his separate treatment from the others.

Remedial Law Review II (CY 2018-2019) – Saturday 1:00pm – 5:00pm: Case Digest Compilation [Atty. Brondial] Page 490
F.1.iv.2 DELA LLANA VS BIONG

G.R. No. 182356; December 4, 2013

FACTS: On March 30, 2000, Juan dela Llana was driving a car along North Avenue, Quezon City. His
sister, Dra. dela Llana, was seated at the front passenger seat while a certain Calimlim was at the
backseat. Juan stopped the when the signal light turned red. A few seconds after the car halted, a dump
truck owned by Rebecca Biong and driven by Joel Primero containing gravel and sand suddenly rammed
the cars rear end, violently pushing the car forward. Due to the impact, the cars rear end collapsed and its
rear windshield was shattered. Glass splinters flew, puncturing Dra. dela Llana. Apart from these minor
wounds, Dra. dela Llana did not appear to have suffered from any other visible physical injuries.

In the first week of May 2000, Dra. dela Llana began to feel mild to moderate pain on the left side of her
neck and shoulder. The pain became more intense as days passed by. Her injury became more severe. Her
health deteriorated to the extent that she could no longer move her left arm. On June 9, 2000, she
consulted with Dr. Rosalinda Milla, a rehabilitation medicine specialist, to examine her condition. Dr.
Milla told her that she suffered from a whiplash injury, an injury caused by the compression of the nerve
running to her left arm and hand. Dr. Milla required her to undergo physical therapy to alleviate her
condition.

Dra. dela Llanas condition did not improve despite three months of extensive physical therapy. She then
consulted other doctors in search for a cure. Dr. Flores, a neuro-surgeon, finally suggested that she
undergo a cervical spine surgery to release the compression of her nerve. On October 19, 2000, Dr. Flores
operated on her spine and neck, between the C5 and the C6 vertebrae. The operation released the
impingement of the nerve, but incapacitated Dra. dela Llana from the practice of her profession since
June 2000 despite the surgery.

Dra. dela Llana, on October 16, 2000, demanded from Rebecca compensation for her injuries, but
Rebecca refused to pay. Thus, on May 8, 2001, Dra. dela Llana sued Rebecca for damages before the
Regional Trial Court of Quezon City (RTC). She alleged that she lost the mobility of her arm as a result
of the vehicular accident and claimed P150,000.00 for her medical expenses (as of the filing of the
complaint) and an average monthly income of P30,000.00 since June 2000. She further prayed for actual,
moral, and exemplary damages as well as attorneys fees.

At the trial, Dra. dela Llana presented herself as an ordinary witness and Joel as a hostile witness. Dra.
Dela Llana reiterated that she lost the mobility of her arm because of the vehicular accident. To prove her
claim, she identified and authenticated a medical certificate dated November 20, 2000 issued by Dr.
Milla. The medical certificate stated that Dra. dela Llana suffered from a whiplash injury. It also
chronicled her clinical history and physical examinations. Meanwhile, Joel testified that his truck hit the
car because the trucks brakes got stuck.

In defense, Rebecca testified that Dra. dela Llana was physically fit and strong when they met several
days after the vehicular accident. She also asserted that she observed the diligence of a good father of a
family in the selection and supervision of Joel.The RTC ruled in favor of Dra. dela Llana and held that the
proximate cause of Dra. dela Llanas whiplash injury to be Joels reckless driving.
The CA reversed the RTC ruling.

ISSUES:
1. Who has the burden of proving the proximate causation between Joels negligence and Dra. dela Llanas
whiplash injury?
2. WON the medical certificate has no probative value for being hearsay.
3. WON Dra. dela Llanas medical opinion cannot be given probative value for the reason that she was not
presented as an expert witness.
4. WON the Supreme Court cannot take judicial notice that vehicular accidents cause whiplash injuries.

RULING:
1.In civil cases, a party who alleges a fact has the burden of proving it. He who alleges has the burden of
proving his allegation by preponderance of evidence or greater weight of credible evidence. The reason
for this rule is that bare allegations, unsubstantiated by evidence, are not equivalent to proof. In short,
mere allegations are not evidence.

Remedial Law Review II (CY 2018-2019) – Saturday 1:00pm – 5:00pm: Case Digest Compilation [Atty. Brondial] Page 491
In the present case, the burden of proving the proximate causation between Joels negligence and Dra. dela
Llanas whiplash injury rests on Dra. dela Llana. She must establish by preponderance of evidence that
Joels negligence, in its natural and continuous sequence, unbroken by any efficient intervening cause,
produced her whiplash injury, and without which her whiplash injury would not have occurred.

2.The medical certificate has no probative value for being hearsay. It is a basic rule that evidence,
whether oral or documentary, is hearsay if its probative value is not based on the personal knowledge of
the witness but on the knowledge of another person who is not on the witness stand. Hearsay evidence,
whether objected to or not, cannot be given credence except in very unusual circumstance that is not
found in the present case. Furthermore, admissibility of evidence should not be equated with weight of
evidence. The admissibility of evidence depends on its relevance and competence, while the weight of
evidence pertains to evidence already admitted and its tendency to convince and persuade. Thus, a
particular item of evidence may be admissible, but its evidentiary weight depends on judicial evaluation
within the guidelines provided by the Rules of Court.

3.Under the Rules of Court, there is a substantial difference between an ordinary witness and an expert
witness. The opinion of an ordinary witness may be received in evidence regarding: (a) the identity of a
person about whom he has adequate knowledge; (b) a handwriting with which he has sufficient
familiarity; and (c) the mental sanity of a person with whom he is sufficiently acquainted. Furthermore,
the witness may also testify on his impressions of the emotion, behavior, condition or appearance of a
person.43 On the other hand, the opinion of an expert witness may be received in evidence on a matter
requiring special knowledge, skill, experience or training which he shown to possess.44

However, courts do not immediately accord probative value to an admitted expert testimony, much less to
an unobjected ordinary testimony respecting special knowledge. The reason is that the probative value of
an expert testimony does not lie in a simple exposition of the experts opinion. Rather, its weight lies in the
assistance that the expert witness may afford the courts by demonstrating the facts which serve as a basis
for his opinion and the reasons on which the logic of his conclusions is founded.45

In the present case, Dra. dela Llanas medical opinion cannot be given probative value for the reason that
she was not presented as an expert witness. As an ordinary witness, she was not competent to testify on
the nature, and the cause and effects of whiplash injury. Furthermore, we emphasize that Dra. dela Llana,
during trial, nonetheless did not provide a medical explanation on the nature as well as the cause and
effects of whiplash injury in her testimony.

4. Indeed, a perusal of the pieces of evidence presented by the parties before the trial court shows that
Dra. dela Llana did not present any testimonial or documentary evidence that directly shows the causal
relation between the vehicular accident and Dra. dela Llanas injury. Her claim that Joels negligence
caused her whiplash injury was not established because of the deficiency of the presented evidence during
trial. We point out in this respect that courts cannot take judicial notice that vehicular accidents cause
whiplash injuries. This proposition is not public knowledge, or is capable of unquestionable
demonstration, or ought to be known to judges because of their judicial functions. We have no expertise
in the field of medicine. Justices and judges are only tasked to apply and interpret the law on the basis of
the parties pieces of evidence and their corresponding legal arguments.

In sum, Dra. dela Llana miserably failed to establish her case by preponderance of evidence. While we
commiserate with her, our solemn duty to independently and impartially assess the merits of the case
binds us to rule against Dra. dela Llanas favor. Her claim, unsupported by preponderance of evidence, is
merely a bare assertion and has no leg to stand on.

Remedial Law Review II (CY 2018-2019) – Saturday 1:00pm – 5:00pm: Case Digest Compilation [Atty. Brondial] Page 492
F.1.iv.3 ZACARIA A. CANDAO, ET AL., vs. PEOPLE OF THE PHILIPPINES

G.R. Nos. 186659-710, October 19, 2011; Villarama, Jr., J.

Digested by: Jesus Fernan Montemayor

FACTS: The Commission of Audit (COA) conducted an expanded special audit on the financial
transactions and operations of the Office of the Regional Governor, Autonomous Region for Muslim
Mindanao (ORG-ARMM). As stated in Special Audit Office (SAO) Report submitted by the audit team,
it was found that illegal withdrawals were made from the depository accounts of the agency through the
issuance of checks payable to the order of petitioners without the required disbursement vouchers. Thus,
the Office of the Special Prosecutor, Office of the Ombudsman-Mindanao, filed in the Sandiganbayan
criminal cases for malversation of public funds against the concerned ORG-ARMM officials/employees.

During the trial in the Graft Court, the lone prosecution witness was Heidi L. Mendoza who
testified that their expanded audit disclosed the illegal withdrawals of funds from the PNB and Treasury
accounts of ORG-ARMM involving 52 checks issued without the required disbursement vouchers. 
Specifically, their attention was caught by the fact that the Report of Checks Issued by the Deputized
Disbursing Officer (RCIDDO) showed that the subject 52 checks have no assigned voucher numbers.
However, on cross-examination, witness Mendoza was asked if the audit team had informed the office or
parties concerned that they are going to be audited (entry conference).  She replied that this was a
sensitive assignment, recalling that they were threatened after their identities were established during the
earlier audit of the same office such that she had to be brought back to Manila. Hence, during the
expanded audit, the team was unable to proceed as in ordinary situations.  While they did an entry
conference during the previous main audit, they were unable to do so at the time of the expanded audit.
Again for security reasons, the team also did not conduct an exit conference after field work; they would
be risking their lives if they discuss there and then their findings. Due to threat to her life, it was her team
supervisor and member who personally retrieved the documents in Cotabato City.  She admitted the
belated submission of original vouchers to the COA central office but these are without supporting
documents. After trial, the Sandiganbayan convicted the petitioners for the crime charged considering
that their act of co-signing the subject checks without the required disbursement vouchers of the
respective amounts covered by the checks constitute illegal withdrawals. Thus, by their collective acts,
said court concluded that petitioners conspired to effect the illegal withdrawals of public funds which,
when required by the COA to be properly accounted for, petitioners failed to do so. Hence, this petition
for review on certiorari of the Decision under Rule 45 of the Rules of Court.

ISSUE: WON the Sandiganbayan was correct in convicting the petitioners despite the non-application of
the “equipoise rule”.

RULING: YES. The Sandiganbayan committed no reversible error in holding that the testimonial and
documentary evidence presented by the petitioners failed to overcome the prima facie evidence of
misappropriation arising from petitioner’s failure to give a satisfactory explanation for the illegal
withdrawals from the ARMM funds under his custody and control.  Petitioners likewise did not
accomplish the proper liquidation of the entire amount withdrawn, during the expanded audit or any time
thereafter. There is therefore no merit in petitioners’ argument that the Sandiganbayan erred in not
applying the equipoise rule.

Under the equipoise rule, where the evidence on an issue of fact is in equipoise or there is doubt on which
side the evidence preponderates, the party having the burden of proof loses. The equipoise rule finds
application if the inculpatory facts and circumstances are capable of two or more explanations, one of
which is consistent with the innocence of the accused and the other consistent with his guilt, for then the
evidence does not fulfill the test of moral certainty, and does not suffice to produce a conviction.

Here, the prosecution was able to prove by adequate evidence that Disbursing Officer failed to account
for funds under his custody and control upon demand specifically for the illegally withdrawn from the
said funds.  In the crime of malversation, all that is necessary for conviction is sufficient proof that the
accountable officer had received public funds, that he did not have them in his possession when demand
therefor was made, and that he could not satisfactorily explain his failure to do so.  Direct evidence of
personal misappropriation by the accused is hardly necessary in malversation cases.

Remedial Law Review II (CY 2018-2019) – Saturday 1:00pm – 5:00pm: Case Digest Compilation [Atty. Brondial] Page 493
F.1.v.1 Office of the Ombudsman vs. Reyes
Office of the Ombudsman v. Reyes
Facts:
On January 11, 2001, Jaime B. Acero executed an affidavit against herein respondent Antonio Reyes and
AngelitoPeñaloza, who were the Transportation Regulation Officer II/Acting Officer-in-Charge and Clerk
III, respectively, of the Land Transportation Office (LTO) District Office in Mambajao, Camiguin. Acero
narrated thus:
That, on January 10, 2001, at about 2:00 o’clock P.M. I went to the Land Transportation Office, at
Mambajao, Camiguin to apply for a driver’s license;
That, I was made to take an examination for driver’s license applicants by a certain Tata Peñaloza whose
real name is Angelito, a clerk in said office;
That, after the examination, [Peñaloza] informed me that I failed in the examination; however if I am
willing to pay additional assessment then they will reconsider my application and I am referring to
[Peñaloza] and [Reyes];
That, I asked how much will that be and [Peñaloza] in the presence of [Reyes] answered ₱680.00, so I
agreed;
That, I then handed ₱1,000.00 to [Peñaloza] and [Peñaloza] handed it to the cashier;
That, [Peñaloza] in turn handed to me the change of ₱320.00 only and a little later I was given the LTO
Official Receipt No. 62927785 (January 10, 2001) but only for ₱180.00 which O.R. serves as my
temporary license for 60 days; and the balance of ₱500.00 was without O.R. and retained by Peñaloza;
That, I feel that the actuation of Antonio Reyes and AngelitoPeñaloza are fraudulent in that they failed to
issue receipt for the extra ₱500.00 paid to them; and [Reyes] know that I am with [the Commission on
Audit];
That, I execute this affidavit to file charges against the guilty parties.

Attached to Acero’s affidavit was the LTO Official Receipt No. 62927785, showing his payment of
₱180.00.
The above affidavit was apparently filed with the Office of the Provincial Prosecutor in Camiguin, but the
same was later referred to the Office of the Ombudsman-Mindanao. The latter office thereafter ordered
Reyes and Peñaloza to submit their counter-affidavits within ten days from notice.
WHEREFORE, there being substantial evidence, this Office finds respondent Antonio T. Reyes guilty of
grave misconduct and he is hereby meted the penalty of DISMISSAL from the service pursuant to
Section 23(c) [Grave Offenses], Rule XIV of the Rules Implementing Book V of Executive Order No.
292. Likewise, this Office finds respondent Angelito G. Peñaloza guilty of Simple Misconduct and he is
hereby meted the penalty of SUSPENSION from office without pay for a period of Six (6) months based
on Section 23(b) [Less Grave Offenses] Rule XIV of the Rules Implementing Book V of Executive Order
No. 292. In both instances, the execution of the penalties imposed shall be made immediately after the
same shall have been final and executory.
In their bid to challenge the above ruling, Reyes filed a Motion for Reconsideration cum Motion to Set
the Case for Preliminary Conference, while Peñaloza filed a Motion for Reconsideration. On February 15,
2002, the Office of the Ombudsman-Mindanao issued a Joint Order, denying the aforesaid motions of
Reyes and Peñaloza.
Reyes elevated the case to the Court of Appeals via a Petition for Review under Rule 43 of the Rules of
Court, which petition was docketed as CA-G.R. SP No. 70571.
In the assailed Decision dated July 4, 2005, the Court of Appeals granted the petition of Reyes and
reversed the judgment of the Office of the Ombudsman-Mindanao. The appellate court reasoned thus:
It must be pointed out that in the complaint-affidavit filed by Acero, it was only Peñaloza who received
the money and the balance of P 500.00 which was without O.R. was retained by Peñaloza. Nowhere in
the complaint-affidavit could one find the name of Reyes, herein petitioner, nor is it alleged there that
Reyes was around when Acero handed to Peñaloza the P 1000.00. From the evidence on record, it was,
clearly, only Peñaloza all along. Nowhere in the record is Reyes’ complicity suggested or even slightly
hinted.
x xxx
It does not appear on record that [Reyes] was the one who ordered and received the "additional
assessment". Rather, it was Peñaloza alone who approached the complainant, discussed about the
"additional assessment", and retained the balance of P 500 basing on the complaint-affidavit filed by
Acero.
Issue: Whether or not the charge of grave misconduct against Reyes was sufficiently proven by
substantial evidence?
RULING: No.
In assailing the judgment of the Court of Appeals, petitioner avers that the findings of fact of the Office of
the Ombudsman are entitled to great weight and must be accorded full respect and credit as long as they

Remedial Law Review II (CY 2018-2019) – Saturday 1:00pm – 5:00pm: Case Digest Compilation [Atty. Brondial] Page 494
are supported by substantial evidence. Petitioner argues that it is not the task of the appellate court to
weigh once more the evidence submitted before an administrative body and to substitute its own
judgment for that of the administrative agency with respect to the sufficiency of evidence.
Indeed, Section 27 of Republic Act No. 6770 mandates that the findings of fact by the Office of the
Ombudsman are conclusive when supported by substantial evidence. In administrative and quasi-judicial
proceedings, only substantial evidence is necessary to establish the case for or against a party. Substantial
evidence is more than a mere scintilla of evidence. It is that amount of relevant evidence that a reasonable
mind might accept as adequate to support a conclusion, even if other minds, equally reasonable, might
conceivably opine otherwise.
In the instant case, petitioner plainly disregarded Reyes’ protestations without giving him a similar
opportunity, as in Ruivivar, to be belatedly furnished copies of the affidavits of Peñaloza, Amper and
Valdehueza to enable him to refute the same. As it were, petitioner rendered its Decision dated September
24, 2001 on the basis of evidence that were not disclosed to Reyes. This the Court cannot sanction. A
judgment in an administrative case that imposes the extreme penalty of dismissal must not only be
based on substantial evidence but also rendered with due regard to the rights of the parties to due
process.
Case is remanded.

Remedial Law Review II (CY 2018-2019) – Saturday 1:00pm – 5:00pm: Case Digest Compilation [Atty. Brondial] Page 495
F.1.v.2 Ramos vs. BPI Family Savings Bank
Ramos v. BPI
Facts:
Ramos was employed by BPI Family in 1995 and eventually became its Vice-President for Dealer
Network Marketing/Auto Loans Division, the duties and responsibilities of which were to:
(a) receive applications for auto loans from auto dealers and salesmen; 7
(b) analyze market demands and formulate marketing strategies; and
(c) enhance dealer and manufacturer relations.
During his tenure, a client named Trezita B. Acosta (Acosta) entered into and obtained several auto and
real estate loans from BPI Family which were duly approved and promptly paid.
On December 15, 2004, Acosta purportedly secured another auto loan from BPI Family in the amount of
₱3,097,392.00 for the purchase of a Toyota Prado vehicle (subject loan) which had remained unpaid. As
it turned out, Acosta did not authorize nor personally apply for the subject loan, rendering the transaction
fraudulent.
Issue: Whether or not the factual findings of the NLRC are not supported by substantial evidence?
RULING:
No, they are not supported.
As a general rule, in certiorari proceedings under Rule 65 of the Rules of Court, the appellate court does
not assess and weigh the sufficiency of evidence upon which the Labor Arbiter and the NLRC based their
conclusion. The query in this proceeding is limited to the determination of whether or not the NLRC
acted without or in excess of its jurisdiction or with grave abuse of discretion in rendering its decision.
However, as an exception, the appellate court may examine and measure the factual findings of the
NLRC if the same are not supported by substantial evidence.
The Court has not hesitated to affirm the appellate court’s reversals of the decisions of labor tribunals if
they are not supported by substantial evidence.
The requirement that the NLRC’s findings should be supported by substantial evidence is clearly
expressed in Section 5, Rule 133 of the Rules of Court which provides that "[i]n cases filed before
administrative or quasi- judicial bodies, a fact may be deemed established if it is supported by
substantial evidence, or that amount of relevant evidence which a reasonable mind might accept as
adequate to justify a conclusion."

Remedial Law Review II (CY 2018-2019) – Saturday 1:00pm – 5:00pm: Case Digest Compilation [Atty. Brondial] Page 496
F.1.vi Lucas vs. Lucas

Facts:
On July 26, 2007, petitioner, Jesse U. Lucas, filed a Petition to Establish Illegitimate Filiation (with
Motion for the Submission of Parties to DNA Testing) before the Regional Trial Court (RTC), Branch 72,
Valenzuela City. Petitioner narrated that, sometime in 1967, his mother, Elsie Uy (Elsie), migrated to
Manila from Davao and stayed with a certain "Ate Belen (Belen)" who worked in a prominent nightspot
in Manila. Elsie would oftentimes accompany Belen to work. On one occasion, Elsie got acquainted with
respondent, Jesus S. Lucas, at Belen’s workplace, and an intimate relationship developed between the
two. Elsie eventually got pregnant and, on March 11, 1969, she gave birth to petitioner, Jesse U. Lucas.
The name of petitioner’s father was not stated in petitioner’s certificate of live birth. However, Elsie later
on told petitioner that his father is respondent. On August 1, 1969, petitioner was baptized at San Isidro
Parish, Taft Avenue, Pasay City. Respondent allegedly extended financial support to Elsie and petitioner
for a period of about two years. When the relationship of Elsie and respondent ended, Elsie refused to
accept respondent’s offer of support and decided to raise petitioner on her own. While petitioner was
growing up, Elsie made several attempts to introduce petitioner to respondent, but all attempts were in
vain.
After learning of the September 3, 2007 Order, respondent filed a motion for reconsideration. Respondent
averred that the petition was not in due form and substance because petitioner could not have personally
known the matters that were alleged therein. He argued that DNA testing cannot be had on the basis of a
mere allegation pointing to respondent as petitioner’s father. Moreover, jurisprudence is still unsettled on
the acceptability of DNA evidence.
At the risk of being repetitious, the Court would like to stress that it sees the danger of allowing an
absolute DNA testing to a compulsory recognition test even if the plaintiff/petitioner failed to establish
prima facie proof. x xx If at anytime, motuproprio and without pre-conditions, the court can indeed order
the taking of DNA test in compulsory recognition cases, then the prominent and well-to-do members of
our society will be easy prey for opportunists and extortionists. For no cause at all, or even for [sic] casual
sexual indiscretions in their younger years could be used as a means to harass them. Unscrupulous
women, unsure of the paternity of their children may just be taking the chances-just in case-by pointing to
a sexual partner in a long past one-time encounter. Indeed an absolute and unconditional taking of DNA
test for compulsory recognition case opens wide the opportunities for extortionist to prey on victims who
have no stomach for scandal.
Petitioner moved for reconsideration. On December 17, 2009, the CA denied the motion for lack of merit.
Issue: Whether or not prima facie evidence is required?
RULING:
Yes, it is required.
Although a paternity action is civil, not criminal, the constitutional prohibition against unreasonable
searches and seizures is still applicable, and a proper showing of sufficient justification under the
particular factual circumstances of the case must be made before a court may order a compulsory blood
test. Courts in various jurisdictions have differed regarding the kind of procedures which are required, but
those jurisdictions have almost universally found that a preliminary showing must be made before a court
can constitutionally order compulsory blood testing in paternity cases. We agree, and find that, as a
preliminary matter, before the court may issue an order for compulsory blood testing, the moving party
must show that there is a reasonable possibility of paternity. As explained hereafter, in cases in which
paternity is contested and a party to the action refuses to voluntarily undergo a blood test, a show cause
hearing must be held in which the court can determine whether there is sufficient evidence to
establish a prima facie case which warrants issuance of a court order for blood testing.
The same condition precedent should be applied in our jurisdiction to protect the putative father from
mere harassment suits. Thus, during the hearing on the motion for DNA testing, the petitioner must
present prima facie evidence or establish a reasonable possibility of paternity.

Remedial Law Review II (CY 2018-2019) – Saturday 1:00pm – 5:00pm: Case Digest Compilation [Atty. Brondial] Page 497
F.1.vii.1 PNB vs. Tria

FACTS:Respondent Amelio C. Tria (Tria) was a former Branch Manager of petitioner Philippine
National Bank (PNB), assigned at PNB’s Metropolitan Waterworks and Sewerage System Branch (PNB-
MWSS) located within the Metropolitan Waterworks and Sewerage System (MWSS) Compound,
Katipunan Road, Balara, Quezon City.

On September 21, 2001, MWSS opened Current Account (C/A) No. 244-850099-6 with PNB-MWSS and
made an initial deposit of PhP 6,714,621.13 on October 10, 2001. On April 16, 2003, C/A 244-850099-6
became dormant with a balance of PhP 5,397,154.07.

Tria requested a listing of the dormant accounts of PNB-MWSS and borrowed the folders of MWSS and
C/A 244-850099-6. Tria also inquired about the irregularities involving manager’s checks committed by
the bank’s former branch accountant.

In 2004, PNB-MWSS received a letter-request from MWSS instructing the deduction of PhP 5,200,000
from C/A 244-850099-6 and the issuance of the corresponding manager’s check in the same amount
payable to a certain "Atty. Rodrigo A. Reyes." The letter-request was purportedly signed and approved by
the duly authorized signatories of MWSS. Hence, C/A 244-850099-6 was re-activated in light of the
letter-request.

The letter-request, supporting documents, and Manager’s Check Application Form were then evaluated
by the bank’s Sales and Service Officer (SSO), Agnes F. Bagasani, who found the same to be in order.
Edsel B. Francisco (Francisco), who was also designated to perform the tasks of a Fund Transfer
Processor (FTP), likewise verified the letter-request and the documents from the MWSS Current Account
folder of the bank. He then effected the transaction requested by debiting C/A No. 244-850099-6 for the
purchase of a Manager’s Check payable to "Atty. Rodrigo A. Reyes". Manager’s Check No. 1165848
was, thus, prepared and issued in the name of Atty. Rodrigo A. Reyes (Atty. Reyes) for the amount
of PhP 5,200,000.00.

PNB-MWSS received cash delivery from PNB’s Cash Center in the amount of PhP 8,660,000. On the
same day, respondent Tria accompanied Atty. Reyes in presenting Manager’s Check No. 1165848 to
PNB’s Quezon City Circle Branch (PNB-Circle) for encashment and told PNB-Circle’s SSO, George T.
Flandez (Flandez), that PNB-MWSS had no available cash to pay the amount indicated in the Manager’s
Check.

To confirm the issuance of Manager’s Check No. 1165848, Flandez called PNB-MWSS and talked to its
Sales and Service Head, Geraldine C. Veniegas (Veniegas). Veniegas confirmed that PNB-MWSS issued
a manager’s check in favor of Atty. Reyes and sent a letter-confirmation through e-mail to PNB-Circle.

While waiting for the confirmation, Flandez interviewed Atty. Reyes, who told him that he was an
MWSS contractor and the amount covered by Manager’s Check No. 1165848 represented the proceeds of
his recent contract with MWSS. Atty. Reyes then showed his driver’s license and Integrated Bar of the
Philippines (IBP) identification card to Flandez and wrote the numbers of these cards on the back of the
manager’s check.

Upon receiving confirmation from PNB-MWSS regarding the manager’s check, Flandez went to the Cash
Center of PNB-Circle to pick up the cash requisition. Tria and Atty. Reyes, however, followed him with
Tria telling Flandez: "Pirmahan ko na lang ‘tong check, George. Identify ko na lang siya kasi
nagmamadali siya. Dito na lang i-receive. For security… kasi nag-iisa lang siya." Tria then placed his
signature on the check above the handwritten note "PAYEE IDENTIFIED – AMELIO C. TRIA."

In August 2004, Veniegas, the Sales and Service Head of PNB-MWSS, observed that Tria showed
sudden concern with the Minutes of the Meeting dated August 6, 2004 even if he was no longer involved
in the operations of the bank. Tria reminded her to prepare the Minutes of the Meeting. Tria then made
revisions therein. After the revised Minutes of the Meeting had been signed by all the attendees, Tria
sought to further amended the Minutes.

On November 1, 2004, Tria retired as PNB-MWSS’ Manager under PNB’s regular retirement plan.

On February 2, 2005, Zaida Pulida (Pulida), the MWSS employee in charge of C/A No. 244-850099-6,
inquired about the account’s outstanding balance. While she was trying to reconcile the records of MWSS

Remedial Law Review II (CY 2018-2019) – Saturday 1:00pm – 5:00pm: Case Digest Compilation [Atty. Brondial] Page 498
and PNB, she inquired about a debit entry dated April 22, 2004 to C/A No. 244-850099-6 in the
amount of PhP 5,200,000. Veniegas verified that PhP 5,200,000 was indeed debited and was encashed
using Manager’s Check No. 1165848 in favor of Atty. Rodrigo A. Reyes. Veniegas also attempted to
retrieve the files for the transaction but discovered that the duplicate copy of Manager’s Check No.
1165848, the manager’s check application form and the letter of authority were all missing.

Pulida notified Veniegas that MWSS did not apply for the issuance of the manager’s check payable
to Atty. Reyes. Upon verification with the IBP, it was discovered that there was no Rodrigo A. Reyes
included in its membership roster. Further, upon inspection of the PNB-MWSS microfilm copy of
Manager’s Check No. 1165848, it was shown that the check was negotiated and encashed at the PNB-
Circle on April 26, 2004 and was annotated with "ok for payment per confirmation and approval of PNB
MWSS" by Tria on the dorsal portion of the check.

PNB conducted its own investigation and, at its conclusion, sought to hold Tria liable for qualified theft.

Following a preliminary investigation, the Assistant City Prosecutor issued a Resolution stating that
Tria’s identification of the payee did not consummate the payment of the Manager’s Check. Rather, it
was held, the consummation of the payment occurred during Flandez’ approval of the encashment. The
charge for qualified theft against Tria was then dismissed due to lack of evidence and probable cause.

PNB moved for reconsideration but was denied. PNB filed a petition for review with the Department of
Justice (DOJ) and prayed for the reversal of the Resolutions issued by the Office of the City Prosecutor of
Quezon City (OCP). In 2007, then Justice Secretary Raul M. Gonzales issued a Resolution dismissing
PNB’s petition for review. PNB’s motion for reconsideration was denied.

PNB sought recourse before the Court of Appeals, which decided in favor of Tria. The CA ruled that
probable cause against Tria and Atty. Reyes was not established since the employees of PNB made the
encashment after their own independent verification of C/A No. 244-850099-6. Further, the CA deferred
to the DOJ’s determination of probable cause for the filing of an information in court as it is an executive
function and ruled that the resolutions were not reversible as PNB was unable to show that these
resolutions of the DOJ were tainted with grave abuse of discretion.

ISSUE:Whether there is probable cause to hold Tria and Atty.Reyes/John Doe for trial of the crime of
qualiied theft.

RULING: Yes. While discretionary authority to determine probable cause in a preliminary investigation
to ascertain sufficient ground for the filing of an information rests with the executive branch, such
authority is far from absolute. It may be subject to review when it has been clearly used with grave abuse
of discretion. And indeed, grave abuse of discretion attended the decision to drop the charges against Tria
as there was more than probable cause to proceed against him for qualified theft.

It must be emphasized at the outset that what is necessary for the filing of a criminal information is not
proof beyond reasonable doubt that the person accused is guilty of the acts imputed on him, but only that
there is probable cause to believe that he is guilty of the crime charged.

Probable cause, for purposes of filing a criminal information, are such facts as are sufficient to engender a
well-founded belief that a crime has been committed and that the accused is probably guilty thereof. It is
the existence of such facts and circumstances as would excite the belief in a reasonable mind, acting on
the facts within the knowledge of the prosecutor, that the person charged was guilty of the crime for
which he is to be prosecuted.  A finding of probable cause needs only to rest on evidence showing that,
more likely than not, a crime has been committed and that it was committed by the accused.

The acts of Tria and the relevant circumstances that led to the encashment of the check provide more than
sufficient basis for the finding of probable cause to file an information against him and John Doe/Atty.
Reyes for qualified theft. In fact, it is easy to infer from the factual milieu of the instant case the existence
of all the elements necessary for the prosecution of the crime of qualified theft.

DOCTRINE:Probable cause, for purposes of filing a criminal information, are such facts as are
sufficient to engender a well-founded belief that a crime has been committed and that the accused is
probably guilty thereof. It is the existence of such facts and circumstances as would excite the belief in a
reasonable mind, acting on the facts within the knowledge of the prosecutor, that the person charged was
guilty of the crime for which he is to be prosecuted.  A finding of probable cause needs only to rest on

Remedial Law Review II (CY 2018-2019) – Saturday 1:00pm – 5:00pm: Case Digest Compilation [Atty. Brondial] Page 499
evidence showing that, more likely than not, a crime has been committed and that it was committed by the
accused.

Remedial Law Review II (CY 2018-2019) – Saturday 1:00pm – 5:00pm: Case Digest Compilation [Atty. Brondial] Page 500
F.1.vii.2 Ruben Del Castillo vs. People

G.R. No. 185128; January 20, 2012; Ponente: J. Peralta

FACTS:Ruben Del Castillo was held engaging in the illegal trade of shabu as per confidential
information of SPO3 Masnayon, after conducting surveillance and test-buy operation at the house
of accused a search warrant was issued by RTC of Cebu City. By virtue of such warrant, the police officer
entered Ruben’s residence and informed the wife that they are going to conduct the search. The accused
was not in his residence, according to SPO3 Masnayon during the raid he saw the accused ran away.

SPO3 Masnayon then ordered his men to get a barangay tanod. In the presence of the barangay tanod,
Nelson Gonzalado, and the elder sister of petitioner named Dolly del Castillo, searched the house of
petitioner including the nipa hut where the petitioner allegedly ran for cover. His men who searched the
residence of the petitioner found nothing, but one of the barangay tanods was able to confiscate from the
nipa hut several articles, including 4 plastic packs containing white crystalline substance. Consequently,
the articles that were confiscated were sent to the PNP Crime Laboratory for examination. The contents of
the 4 heat- sealed transparent plastic packs were subjected to laboratory examination, the result of which
proved positive for the presence of methamphetamine hydrochloride, or shabu.

An information against Ruben was filed before the RTC charging him of violation of Sec. 16 of RA 6425.
During arraignment, petitioner, with the assistance of his counsel, pleaded not guilty.Subsequently, trial
on the merits ensued. The RTC found Ruben guilty of the crime charged. On appeal Ruben questioned the
validity of the search warrant, bu the CA affrimed the decision of the trial court.

Thus this petition, the accused-petitioner insists that there was no probable cause to issue the search
warrant, considering that SPO1 Reynaldo Matillano, the police officer who applied for it, had no personal
knowledge of the alleged illegal sale of drugs during a test-buy operation conducted prior to the
application of the same search warrant. The second argument, petitioner asserts that the nipa hut located
about 20 meters away from his house is no longer within the "permissible area" that may be searched by
the police officers due to the distance and that the search warrant did not include the same nipa hut as one
of the places to be searched. As to the third argument raised, petitioner claims that the CA erred in finding
him guilty beyond reasonable doubt of illegal possession of prohibited drugs, because he could not be
presumed to be in possession of the same just because they were found inside the nipa hut.
Issue: Whether or not the search warrant is based on probable cause? Whether or not there was illegal
search?

RULING:YES,Probable cause for a search warrant is defined as such facts and circumstances which
would lead a reasonably discreet and prudent man to believe that an offense has been committed and that
the objects sought in connection with the offense are in the place sought to be searched. A finding of
probable cause needs only to rest on evidence showing that, more likely than not, a crime has been
committed and that it was committed by the accused. Probable cause demands more than bare suspicion;
it requires less than evidence which would justify conviction.The judge, in determining probable cause, is
to consider the totality of the circumstances made known to him and not by a fixed and rigid formula,and
must employ a flexible, totality of the circumstances standard.The existence depends to a large degree
upon the finding or opinion of the judge conducting the examination. This Court, therefore, is in no
position to disturb the factual findings of the judge which led to the issuance of the search warrant. A
magistrate's determination of probable cause for the issuance of a search warrant is paid great deference
by a reviewing court, as long as there was substantial basis for that determination. Substantial basis means
that the questions of the examining judge brought out such facts and circumstances as would lead a
reasonably discreet and prudent man to believe that an offense has been committed, and the objects in
connection with the offense sought to be seized are in the place sought to be searched.A review of the
records shows that in the present case, a substantial basis exists.

YES, it must be remembered that the warrant issued must particularly describe the place to be searched
and persons or things to be seized in order for it to be valid. A designation or description that points out
the place to be searched to the exclusion of all others, and on inquiry unerringly leads the peace officers
to it, satisfies the constitutional requirement of definiteness. In the present case, the search warrant
specifically designates or describes the residence of the petitioner as the place to be searched.
Incidentally, the items were seized by a barangay tanod in a nipa hut, 20 meters away from the residence
of the petitioner. The confiscated items, having been found in a place other than the one described in the
search warrant, can be considered as fruits of an invalid warrantless search, the presentation of which as

Remedial Law Review II (CY 2018-2019) – Saturday 1:00pm – 5:00pm: Case Digest Compilation [Atty. Brondial] Page 501
an evidence is a violation of petitioner's constitutional guaranty against unreasonable searches and
seizure. 

Remedial Law Review II (CY 2018-2019) – Saturday 1:00pm – 5:00pm: Case Digest Compilation [Atty. Brondial] Page 502
F.1.viii.1 PEOPLE OF THE PHILIPPINES vs. ALBERTO ANTICAMARA y CABILLO and
FERNANDO CALAGUAS FERNANDEZ a.k.a. LANDO CALAGUAS

FACTS: Fernando Calaguas Fernandez (Lando) and Alberto Cabillo Anticamara (Al) were found guilty
beyond reasonable doubt of the crime of Murder and of the crime of Kidnapping and Serious Illegal
Detention.

AAA, the principal witness who testified in the case, did not personally see the accused kill Abad
Sulpacio. In fact, no one saw the killing of Abad.

The accused then appealed praying for the reversal of the judgment against them claiming the their guilt
were not proven beyond reasonable doubt.

ISSUE: Whether or not the lack of eye witness to a murder sufficient basis to reverse a finding of guilt
beyond reasonable doubt by the trial court.

RULING: No, the trial court found that although there was no direct eyewitness in the killing of Sulpacio
inthe prosecution adduced sufficient circumstantial evidence to establish with moral certainty the
identities and guilt of the perpetrators of the crime.

Circumstantial evidence consists of proof of collateral facts and circumstances from which the existence
of the main fact may be inferred according to reason and common experience. Circumstantial evidence is
sufficient to sustain conviction if: (a) there is more than one circumstance; (b) the facts from which the
inferences are derived are proven; (c) the combination of all circumstances is such as to produce a
conviction beyond reasonable doubt. A judgment of conviction based on circumstantial evidence can be
sustained when the circumstances proved form an unbroken chain that results in a fair and reasonable
conclusion pointing to the accused, to the exclusion of all others, as the perpetrator.

In this case, the circumstantial evidence presented by the prosecution, when analyzed and taken together,
lead to the inescapable conclusion that the appellants are responsible for the death of Sulpacio. The Court
quotes with approval the lower court's enumeration of those circumstantial evidence:
The testimony of AAA had clearly established the following facts:
1. At about 3:00 in the early morning of May 7, 2002, while she and the victim Abad Sulpacio
were sleeping inside the house of the Estrella family in Barangay Carmen, Rosales, Pangasinan
several persons entered to rob the place;
2. Inside the house, she saw and recognized the accused Lando Calaguas and Dick Tañedo, and
heard the latter uttering "somebody will die";
3. Bringing her outside the house, Lando pushed her into the Revo where she saw inside Abad
Sulpacio who was blindfolded and with his hands tied;
4. Inside the Revo, she recognized the accused Dick Tañedo, Lando Calaguas, Marvin Lim,
Roberto Tañedo, Alberto Anticamara and Fred;
5. The Revo then proceeded towards the fishpond owned by the Estrellas in Sitio Rosalia, Brgy.
San Bartolome, Rosales, Pangasinan;
6. The last time that she saw Abad Sulpacio was when he was dragged out from the vehicle by
Lando, Fred, Marvin and Al upon reaching Sitio Rosalia. At that, time Dick Tañedo stayed with
her in the vehicle;
7. Thereafter, when Fred returned to the vehicle, she heard him uttered (sic): "Make a decision
now. Abad has already four (4) bullets in his body, and the one left is for this girl."
In addition to these circumstances, the trial court further found that AAA heard Fred utter "Usapan natin
pare, kung sino ang masagasaan, sagasaan." (Our agreement is that whoever comes our way should be
eliminated). Moreover, NBI Agent Gerald V. Geralde testified that on June 23, 2002, appellant Al
admitted his participation as lookout and naming his companions Dick, Lando, Fred, Marvin and Bet as
the ones who took AAA and Sulpacio from the house of the Estrellas and brought them to the fishpond.
Al also pointed and led the authorities to a shallow grave in Sitio Rosalia, Barangay San Bartolome,
Rosales, Pangasinan, where the remains of Sulpacio were buried. The autopsy conducted on the body,
prepared by the Medico Legal Officer Dr. Bandonil, shows that several holes were found on various parts
of the body of the victim and Dr. Bandonil concluded that the cause of the victim's death was the gunshot
wounds. The report also indicates that a piece of cloth was found wrapped around the eye sockets and tied
at the back of the skull, and another cloth was also found tied at the remnants of the left wrist.
In the case at bar, although no one directly saw the actual killing of Sulpacio, the prosecution was able to
paint a clear picture that the appellants took Sulpacio away from the house of the Estrellas, tied and
blindfolded him, and brought him to another place where he was repeatedly shot and buried.

Remedial Law Review II (CY 2018-2019) – Saturday 1:00pm – 5:00pm: Case Digest Compilation [Atty. Brondial] Page 503
F.1.viii.2 PEOPLE OF THE PHILIPPINES, vs. EFREN DEOCAMPO et al.

G.R. No. 185212; February 15, 2012

FACTS: Maritess Alolod, Efren Deocampo, Edwin Deocampo, and Elmer Deocampo were charged with
double murder. RTC convicted them considering the following circumstantial evidence: (1) Efren had
always been banned from the old couple’s house because they strongly disapproved his relationship with
Maritess; (2) The old couple were enjoying good health before of May 27, 1998; (3) On May 28 they
suddenly went missing; (4) On the night of May 27 the security guard at Salaman Institute saw Efren and
Edwin standing on the school side of the fence next to the old couple’s house. The next day, the guard
discovered that the fence wire had been cut; (5) At about 2:00 a.m. of May 28 a `neighbor heard the
sound of a woman sobbing and what seemed like the butchering of a pig; (6) At break of dawn, a witness
saw Efren in the Alolod kitchen; (7) From then on Efren and his brothers frequented the old couple’s
house, with Efren wearing the old man’s watch; (8) Maritess lied about her adoptive parents going to
Cotabato City / Davao City for medical treatment when people started looking for them; (9) A witness
heard Efren instructing Maritess to plant more camote on a pile of red soil beside the house; and (10) The
bodies of the old couple were found underneath those plants.
On appeal, Accused Efren Deocampo assailed their conviction based on circumstantial evidence.

ISSUE: Whether or not the CA erred in affirming the RTC’s finding that accused Efren was responsible
for the murder of the Alolod couple based on circumstantial evidence.

RULING: The rule of evidence that applies when no witness saw the commission of the crime provides:

SEC. 4. Circumstantial evidence, when sufficient. – Circumstantial evidence is sufficient for conviction
if: (a) There is more than one circumstance; (b) The facts from which the inferences are derived are
proven; and (c) The combination of all the circumstances is such as to produce a conviction beyond
reasonable doubt.

The circumstances must constitute an unbroken chain that inexorably leads to one fair conclusion: the
accused committed the crime to the exclusion of all others.

In this case, the trial court was correct in holding that the accused was guilty due to the following facts:
(1) the presence of the accused in the victim’s house and used of the latter’s things when he had always
been banned therefrom during the lifetime of the victim; (2) the cover-up story of the adopted daughter
that her parent went away for a medical check-up; (3) the planting of camote on the ground where the
victim’s where found buried; (4) testimony of a neighbor hearing a scream the night before the victims
went missing then seeing the accused inside the house of the victims. All these when taken together leads
only to one conclusion, i.e., that the accused is guilty of the crime charged.

Remedial Law Review II (CY 2018-2019) – Saturday 1:00pm – 5:00pm: Case Digest Compilation [Atty. Brondial] Page 504
F.1.viii.3 Celedonio vs. People

EDUARDO CELEDONIO, Petitioner, 
vs.
PEOPLE OF THE PHILIPPINES, Respondent.
G.R. No. 209137; July 1, 2015
MENDOZA, J.:

Facts: The evidence for the prosecution shows that on the evening of April 21, 2007, a certain
Adriano Marquez (Marquez) witnessed the robbery perpetrated in the house of Carmencita De
Guzman (De Guzman) while she was away to attend to the wake of her deceased husband. No
one was left in the house. Marquez, whose house was opposite the house of De Guzman and
Celedonio, which were adjacent to each other, identified Celedonio as the culprit. Upon learning
of the incident, De Guzman reported it to the police and requested that Celedomo be
investigated for possibly having committed the crime, based on the account of Marquez. Later, a
follow-up operation was conducted by PO1 Rommel Roque (PO1 Roque) and SPO2 Adrian
Sugui (SPO2 Sugui), accompanied by Marquez. They proceeded to Raja Humabon St.,
Navotas, to survey the area for the possible identification and apprehension of the suspect. On
their way, Marquez pointed to a man on a motorcycle and said, "Sir, siya po si Eduardo
Celedonio." The police immediately flagged down Celedonio. PO1 Roque asked him if he was
Eduardo Celedonio, but he did not reply and just bowed his head. SPO2 Sugui informed
Celedonio of a complaint for robbery against him. Celedonio still remained silent and just bowed
his head. SPO2 Sugui asked him, "Where are the stolen items?" Celedonio then alighted from
his motorcycle and opened its compartment where PO1 Roque saw some of the stolen items,
as per report of the incident, such as the portable DVD player and a wristwatch, among others.
PO1 Roque asked Celedonio if the same were stolen, to which the latter answered, "Iyan po."
Thus, Celedonio was arrested and was informed of his constitutional rights. More items were
seized from Celedonio at the police station. RTC found Celedonio guilty beyond reasonable
doubt of the crime of Robbery with Force upon things. Insisting on his innocence, Celedonio
appealed to the Court of Appeals (CA), arguing that the RTC erred in convicting him of the
crime despite the insufficiency of the circumstantial evidence.

Issue: Whether or not the Honorable Court of Appeals gravely erred in affirming the trial court's
ruling that the petitioner's guilt was proven based on circumstantial evidence.

Ruling: No.

The SC held that, “Jurisprudence tells us that direct evidence of the crime is not the only matrix
from which a trial court may draw its conclusion and finding of guilt. The rules on evidence allow
a trial court to rely on circumstantial evidence to support its conclusion of guilt. The lack of direct
evidence does not ipso factobar the finding of guilt against the appellant. As long as the
prosecution establishes the accused-appellant's participation in the crime through credible and
sufficient circumstantial evidence that leads to the inescapable conclusion that he committed the
imputed crime, the latter should be convicted. Circumstantial evidence is sufficient for conviction
if: 1) there is more than one circumstance; 2) the facts from which the inferences are derived
are proven; and 3) the combination of all the circumstances is such as to produce a conviction
beyond reasonable doubt.

In this case, the prosecution sufficiently laid down the circumstances that, when taken together,
constituted an unbroken chain that led to a reasonable conclusion that Celedonio was the
perpetrator. The CA opined that: xxx As correctly pointed out by the trial court, these
circumstances are: accused was a next door neighbor of private complainant; he was seen by
another neighbor going over the concrete fence separating their houses and ransacking a room
in complainant's house; during the time, no one was inside complainant's house as all of them
were at the wake of private complainant's recently demised husband; two (2) days after, most of
the items discovered to have been stolen that night were found in the compartment of the
accused's motorcycle which he was riding on when accosted by the police; the items recovered
from him were identified by the complainant as her stolen property; during the trial accused
denied that the stolen items were found in his possession and claimed that they were "planted"
by the police investigators to frame him up of the robbery. In short, the accused could not
explain his possession of the recently stolen items found in his sole possession. Xxx.

Remedial Law Review II (CY 2018-2019) – Saturday 1:00pm – 5:00pm: Case Digest Compilation [Atty. Brondial] Page 505
We find the conviction of accused-appellant based on circumstantial evidence factually and
legally tenable, as the facts from which the aforementioned circumstances arose have been
proved through the positive testimony of Adriano Marquez, POi Rommel Roque and Carmencita
de Guzman. The defense does not refute the existence of the commission of robbery. In fact,
Celedonio himself acknowledged that the prosecution's circumstantial evidence, although weak,
ambiguous and inconclusive, established that 1) a robbery had been committed; 2) it was
committed recently; 3) several of the stolen items including cash were found in his possession;
and 4) he had no valid explanation for his possession of the stolen goods.”

Remedial Law Review II (CY 2018-2019) – Saturday 1:00pm – 5:00pm: Case Digest Compilation [Atty. Brondial] Page 506

You might also like